Test Bank For Organizational Behavior 19th Edition Stephen Robbins And Timothy Judge
1 richard@qwconsultancy.com
Organizational Behavior, 19e (Robbins/Judge) Chapter 1 What Is Organizational Behavior? 1) ________ get things done through other people. They make decisions, allocate resources, and direct the activities of others to attain goals. A) Assistants B) Managers C) Secretaries D) Interns E) Apprentices Answer: B Explanation: The defining characteristic of managers is that they get things done through other people. They make decisions, allocate resources, and direct the activities of others to attain goals. LO: 1.1: Define organizational behavior (OB). Difficulty: Easy Quest. Category: Concept 2) Jude Littleford, a manager at an environmental organization, is currently working on a global warming project. In addition to identifying which tasks related to creating awareness about the issue need to be done, Jude is also choosing the members of her team to engage with the public and to lobby the government. She is also assigning people as team members to ensure that tasks are undertaken on time. Which of the following categories of functions is Mitchell undertaking? A) Planning B) Organizing C) Scrutinizing D) Controlling E) Envisioning Answer: B Explanation: Managers are responsible for designing an organization's structure. This function is called organizing. It includes determining what tasks are to be done, who is to do them, how the tasks are to be grouped, who reports to whom, and where decisions are to be made. LO: 1.1: Define organizational behavior (OB). AACSB: Analytical thinking Difficulty: Hard Employability Skills: Knowledge Application and Analysis Quest. Category: Application
2 richard@qwconsultancy.com
3) Johanna Reid, a campaign manager at a child rights organization, recently started working on an illiteracy project. During the project, she needs to motivate team members to attain their project milestones and direct them through different phases of the project. Which of the following kinds of functions will these tasks be covered under? A) Planning B) Organizing C) Scrutinizing D) Evaluating E) Leading Answer: E Explanation: The leading function of managers involves motivating employees, directing their activities, selecting the most effective communication channels, and resolving conflicts among members. LO: 1.1: Define organizational behavior (OB). AACSB: Analytical thinking Difficulty: Hard Employability Skills: Knowledge Application and Analysis Quest. Category: Application 4) In his role as project manager for the "Saving Sicily's Small Farms" campaign, Arthur Swift ran into problems when the campaign could not be launched publicly according to schedule. Swift monitored the schedule to find the cause of the delay before speeding up the implementation process by allocating more members for the implementation phase. By doing this, which of the following functions is he performing? A) Controlling B) Planning C) Formulating D) Leading E) Organizing Answer: A Explanation: To ensure things are going as they should, management must monitor the organization's performance and compare it with previously set goals. If there are any significant deviations, it is management's job to get the organization back on track. Tasks which involve monitoring, comparing, and potential correcting are part of the controlling function. LO: 1.1: Define organizational behavior (OB). AACSB: Analytical thinking Difficulty: Hard Employability Skills: Knowledge Application and Analysis Quest. Category: Application
3 richard@qwconsultancy.com
5) Which of the following functions do managers undertake as part of planning functions? A) Defining an organization's goals B) Implementing strategies for achieving goals C) Executing plans to integrate activities D) Accomplishing goals of a project E) Employing strategies to coordinate tasks Answer: A Explanation: The planning function encompasses defining an organization's goals, establishing an overall strategy for achieving those goals, and developing a comprehensive set of plans to integrate and coordinate activities. LO: 1.1: Define organizational behavior (OB). Difficulty: Easy Quest. Category: Concept 6) According to Henry Mintzberg, a factory supervisor giving a group of high school students a tour of the plant may be termed as a ________. A) leader B) figurehead C) resource allocator D) negotiator E) disturbance handler Answer: B Explanation: According to Henry Mintzberg, a figurehead is a symbolic head who is required to perform a number of routine duties of a legal or social nature. LO: 1.1: Define organizational behavior (OB). Difficulty: Easy Quest. Category: Concept 7) Linda Mark is a sales manager for Diconex, a company producing commercial and household heating and cooling equipment. The company has recently launched a new heat pump. Over the next few weeks, Linda will be sharing knowledge about the product with her team members. She will also need to inspire them to reach their sales targets and clarify any doubts about the new product. Which of the following roles is Ortiz playing? A) Leader B) Liaison C) Disseminator D) Spokesperson E) Negotiator Answer: A Explanation: A leadership role comprises hiring, training, motivating, and disciplining employees. LO: 1.1: Define organizational behavior (OB). AACSB: Analytical thinking Difficulty: Moderate Quest. Category: Application
4 richard@qwconsultancy.com
8) Regina George works as a campaign manager in a not-for-profit organization in Hampshire. For the upcoming campaign against genetic engineering, she is networking with managers who are working on the issue of food safety. Through her network of contacts, she strives to gain information about the stakeholders in the food industry and other lobby groups. Which of the following roles is George most likely to be playing according to Mintzberg's classification of managerial roles? A) Figurehead B) Leader C) Liaison D) Entrepreneur E) Resource allocator Answer: C Explanation: The liaison role involves maintaining a network of contacts who provide favors and information. LO: 1.1: Define organizational behavior (OB). AACSB: Analytical thinking Difficulty: Moderate Quest. Category: Application 9) Annette Simpson works for a fashion house in Paris and is preparing for the company's upcoming line of winter clothing. She is currently researching online to know what is in vogue this season. In addition, she is also networking with contacts from the press and fashion magazine editors to understand the changing tastes of consumers. Which of the following roles is Simpson playing according to Mintzberg's classification of managerial roles? A) Figurehead B) Leader C) Symbol head D) Monitor E) Spokesperson Answer: D Explanation: According to Mintzberg's classification of managerial roles, the monitor role comprises collecting information from external organizations and institutions, typically by scanning the news media, talking with other people to learn about changes in the public's tastes and what competitors may be planning, and so forth. LO: 1.1: Define organizational behavior (OB). AACSB: Analytical thinking Difficulty: Moderate Quest. Category: Application
5 richard@qwconsultancy.com
10) According to Mintzberg's classification of managerial roles, the role of a(n) ________ is to transmit information received from outsiders or from other employees to members of the organization. A) entrepreneur B) resource allocator C) spokesperson D) leader E) disseminator Answer: E Explanation: According to Mintzberg's classification of managerial roles, the role of a disseminator is to transmit information received from outsiders or from other employees to members of the organization. LO: 1.1: Define organizational behavior (OB). Difficulty: Easy Quest. Category: Concept 11) The role of a(n) ________, according to Mintzberg's classification of managerial roles, is to transmit information to outsiders about an organization's plans, policies, actions, and results and to serve as an expert in the organization's industry. A) spokesperson B) disturbance handler C) entrepreneur D) leader E) liaison Answer: A Explanation: According to Mintzberg's classification of managerial roles, the role of a spokesperson is to transmit information to outsiders about an organization's plans, policies, actions, and results and to serve as an expert in the organization's industry. LO: 1.1: Define organizational behavior (OB). Difficulty: Easy Quest. Category: Concept 12) According to Mintzberg's classification of managerial roles, the role of a(n) ________ is to make or approve significant organizational decisions and assign human, physical, and monetary assets. A) disseminator B) liaison C) entrepreneur D) resource allocator E) disturbance handlers Answer: D Explanation: According to Mintzberg's classification of managerial roles, the role of a resource allocator is to make or approve significant organizational decisions and assign human, physical, and monetary assets. LO: 1.1: Define organizational behavior (OB). Difficulty: Easy 6 richard@qwconsultancy.com
Quest. Category: Concept 13) According to Mintzberg's classification of managerial roles, which of the following is a kind of interpersonal role? A) Monitor B) Disseminator C) Spokesperson D) Entrepreneur E) Liaison Answer: E Explanation: According to Mintzberg's classification of managerial roles, the role of a liaison is a kind of interpersonal role which involves maintaining a network of outside contacts who provide favors and information. LO: 1.1: Define organizational behavior (OB). AACSB: Interpersonal relations and teamwork Difficulty: Easy Quest. Category: Concept 14) According to Mintzberg's classification of managerial roles, which of the following is a kind of decisional role? A) Negotiator B) Disseminator C) Liaison D) Spokesperson E) Monitor Answer: A Explanation: According to Mintzberg's classification of managerial roles, the role of a negotiator is a kind of decisional role which involves representing the organization at major negotiations. LO: 1.1: Define organizational behavior (OB). Difficulty: Easy Quest. Category: Concept 15) The role of a(n) ________ is a type of informational role according to Mintzberg's classification of managerial roles. A) figurehead B) disseminator C) liaison D) entrepreneur E) negotiator Answer: B Explanation: According to Mintzberg's classification of managerial roles, the role of a disseminator is a type of informational role which involves transmitting information received from outsiders or from other employees to members of the organization. LO: 1.1: Define organizational behavior (OB). Difficulty: Easy Quest. Category: Concept 7 richard@qwconsultancy.com
8 richard@qwconsultancy.com
16) Which of the following is true regarding technical skills? A) They can be learned only through formal education. B) They encompass the ability to apply specialized knowledge. C) They are not required at all kinds of jobs. D) They are monopolized by professionals. E) They comprise the ability to understand and motivate people. Answer: B Explanation: Technical skills encompass the ability to apply specialized knowledge or expertise. All jobs require some specialized expertise, and many people develop their technical skills on the job. LO: 1.1: Define organizational behavior (OB). Difficulty: Easy Employability Skills: IT Application and Computing Skills Quest. Category: Concept 17) The ability to understand, communicate with, motivate, and support other people, both individually and in groups, may be defined as ________. A) people skills B) technical skills C) conceptual skills D) cognitive skills E) analytical skills Answer: A Explanation: The ability to understand, communicate with, motivate, and support other people, both individually and in groups, may be defined as people skills. Managers get things done through other people, and it is crucial for them to have good people skills. LO: 1.1: Define organizational behavior (OB). AACSB: Written and oral communication Difficulty: Easy Employability Skills: Communication Quest. Category: Concept
9 richard@qwconsultancy.com
18) Anna Eburt has a degree in environmental sustainability and possesses substantial knowledge about the issue of global warming. Anna was recently hired as the campaign manager at an environmental firm where she will use her knowledge to lead the public relations team of the organization. However, a few months later, the board of directors of the organization expressed dissatisfaction with Anna's performance and asked her to resign. Which of the following, if true, best explains this situation? A) Anna had no prior experience in research and development. B) Anna had weak interpersonal and networking skills to run the project. C) Anna focused on the results her team achieved rather than how they achieved those results. D) Anna is not up to date about the feasibility of using different modes of renewable energy. E) Anna had a diverse network of contacts established from her previous job. Answer: B Explanation: The ability to understand, communicate with, motivate, and support other people, both individually and in groups, defines people skills. Many people are technically proficient but poor listeners, unable to understand the needs of others, or weak at managing conflicts. Since managers get things done through other people, they must have good human skills. Though Anna had substantial knowledge about the issue of global warming, her weak interpersonal and networking skills explain why she was unable to lead the public relations team efficiently. Anna having no prior experience in research and development plays no role in explaining why she was unable to lead the public relations team. Anna focusing on the results her team achieved rather than how they achieved them does not indicate her underperformance in any way. In addition, Anna not being up to date about the feasibility of using different modes of renewable energy indicates a lack of theoretical knowledge and not the inability to lead a public relations team. Anna having a diverse network of contacts established from her previous job works against the situation because it would actually help Anna in her job. LO: 1.1: Define organizational behavior (OB). AACSB: Interpersonal relations and teamwork Difficulty: Hard Employability Skills: Communication Quest. Category: Critical Thinking
10 richard@qwconsultancy.com
19) Joann Hayes is currently working on a project to tackle climate change. During the project, she needs to find different options to replace the use of non-renewable energy and check the feasibility of different renewable energy options before choosing the most practical one. Which of the following skills does Hayes primarily need to use for these tasks? A) People skills B) Interpersonal skills C) Conceptual skills D) Communication skills E) Interactive skills Answer: C Explanation: Managers must have the mental ability to analyze and diagnose complex situations. These tasks require conceptual skills. The ability to integrate new ideas with existing processes and innovate on the job is also a crucial conceptual skill for today's managers. LO: 1.1: Define organizational behavior (OB). AACSB: Analytical thinking Difficulty: Hard Employability Skills: Critical Thinking Quest. Category: Application
11 richard@qwconsultancy.com
20) 360.org, an organization working toward curbing climate change, recently conducted an interview with Jessica for the position of a public relations officer. However, interviewers Brenda and Laura are divided over whether Jessica should be given the job. Brenda believes that Jessica does not have in-depth knowledge about the issue of global warming and its impact. On the other hand, Laura feels that Jessica would be perfect for the job because she has strong networking and interpersonal skills. Which of the following, if true, would strengthen Laura's argument? A) Gaining the support of corporate giants would comprise a large part of the job. B) Jessica will need to prepare extensive reports about the issue of climate change. C) The role will require Jessica to give presentations to environmental experts on renewable modes of energy. D) Jessica had negligible experience in research and development. E) Jessica has weak analytical skills to solve problems associated with implementing options of renewable energy. Answer: A Explanation: The ability to understand, communicate with, motivate, and support other people, both individually and in groups, defines people skills. Many people are technically proficient but poor listeners, unable to understand the needs of others, or weak at managing conflicts. As gaining the support of corporate giants is a large part of the public relations job, it would require strong networking and interpersonal skills, which according to Laura, Jessica has. This would strengthen Laura's argument. The task of preparing extensive reports does not involve any networking skills and thus would not in any way strengthen Laura's argument. Giving presentations to environmental experts would require Laura to have in-depth knowledge of global warming and does not involve networking skills. Jessica having negligible experience in research and development and weak analytical skills to solve problems associated with implementing options of renewable energy does not require networking skills and thus does not strengthen Laura's argument. LO: 1.1: Define organizational behavior (OB). AACSB: Written and oral communication Difficulty: Hard Employability Skills: Communication Quest. Category: Critical Thinking
12 richard@qwconsultancy.com
21) 360.org, an organization working toward curbing climate change, recently conducted an interview with Jessica for the position of a public relations officer. However, interviewers Brenda and Laura are divided over whether Jessica should be given the job. Brenda believes that Jessica does not have in-depth knowledge about the issue of global warming and its impact. On the other hand, Laura feels that Jessica would be perfect for the job because she has strong interpersonal skills. Which of the following, if true, would weaken Laura's argument? A) Jessica was unable to fare well in the written test which analyzed her conceptual skills. B) Jessica listened intently to the interviewers' questions before answering them. C) Jessica was unable to communicate clearly why she was right for the job. D) Jessica was very friendly and courteous with the receptionist while greeting her. E) Jessica was high on confidence about getting the job because of her strong networking skills. Answer: C Explanation: The ability to understand, communicate with, motivate, and support other people, both individually and in groups, defines people skills. Many people are technically proficient but poor listeners, unable to understand the needs of others, or weak at managing conflicts. If Jessica were unable to communicate clearly why she was right for the job, it would weaken Laura's argument that Jessica has strong interpersonal skills. Jessica being unable to fare well in the written test which analyzed her conceptual skills does not indicate Jessica's networking skills, and thus does not weaken Laura's argument. Jessica listening intently to the interviewers' questions before answering them and greeting the receptionist indicates that she has good interpersonal skills. This strengthens Laura's argument. Jessica being high on confidence about getting the job because of her strong networking skills is irrelevant to the argument. LO: 1.1: Define organizational behavior (OB). AACSB: Written and oral communication Difficulty: Hard Employability Skills: Communication Quest. Category: Critical Thinking 22) According to Fred Luthans and his associates, managers involved in traditional management activities undertook which of the following tasks? A) Motivating B) Socializing C) Decision making D) Training E) Politicking Answer: C Explanation: Fred Luthans and his associates studied more than 450 managers who were all engaged in four managerial activities of traditional management, communication, human resource management, and networking. Activities in traditional management involved decision making, planning, and controlling. LO: 1.1: Define organizational behavior (OB). Difficulty: Easy Quest. Category: Concept
13 richard@qwconsultancy.com
23) According to Fred Luthans and his associates' study of 450 managers, ________ made the most considerable contribution to the success of managers in terms of speed of promotion within their organization. A) networking B) decision making C) planning D) controlling E) staffing Answer: A Explanation: According to Fred Luthans and his associates' study of 450 managers, networking made the most considerable contribution to the success of managers in terms of speed of promotion within their organization. Human resource management activities made the least relative contribution. LO: 1.1: Define organizational behavior (OB). Difficulty: Easy Quest. Category: Concept 24) According to Fred Luthans and his associates, managers who are involved in networking activities are most likely to undertake which of the following? A) Planning B) Decision making C) Controlling D) Politicking E) Staffing Answer: D Explanation: Fred Luthans and his associates studied more than 450 managers who were all engaged in four managerial activities of traditional management, communication, human resource management, and networking. Networking activities are comprised of socializing, politicking, and interacting with outsiders. LO: 1.1: Define organizational behavior (OB). Difficulty: Easy Quest. Category: Concept 25) Which of the following best defines organizational behavior? A) It involves the study of groups of people coming together for collective bargaining. B) It involves the study of what people do in a company and how it affects the company's performance. C) It involves analyzing different people in an industry with independent profit-centered motives. D) It involves developing exclusively the knowledge of managers and senior-level employees. E) It is a field which is not influenced by factors in the external world. Answer: B Explanation: Organizational behavior refers to the study of what people do in an organization and how their behavior affects the organization's performance. LO: 1.1: Define organizational behavior (OB). Difficulty: Easy 14 richard@qwconsultancy.com
Quest. Category: Concept 26) Which of the following determinants of behavior does organizational behavior study? A) Profits B) Job satisfaction C) Perks D) Individuals E) Collective bargaining Answer: D Explanation: Organizational behavior studies the three determinants of behavior in organizations. These are comprised of individuals, groups, and structure. In addition, OB applies the knowledge gained about individuals, groups, and the effect of structure on behavior in order to make organizations work more effectively. LO: 1.1: Define organizational behavior (OB). Difficulty: Easy Quest. Category: Concept 27) Interacting with outsiders is part of which managerial activity? A) Traditional management B) Communication C) Human resources management D) Networking E) Data collection Answer: D Explanation: According to research by Luthans and his associates of 450 managers, all engaged in four managerial activities: traditional management (decision making, planning, and controlling); communication (exchanging routine information and processing paperwork); human resources management (motivating, disciplining, managing conflict, staffing, and training); and networking (socializing, politicking, and interacting with outsiders). LO: 1.1: Define organizational behavior (OB). AACSB: Diverse and multicultural work environments Difficulty: Moderate Quest. Category: Concept 28) According to a study by Monster of about a million job postings, the most frequently desired skills in job applicants are communications skills followed by other OB-related skills like problem solving and influence skills. Answer: TRUE Explanation: Communications skills topped the list of most frequently desired skills in job applicants according to Monster's study of about a million market-wide job postings. Other desired skills included problem-solving skills and influence skills. LO: 1.1: Define organizational behavior (OB). AACSB: Written and oral communication Difficulty: Easy Employability Skills: Communication Quest. Category: Concept 15 richard@qwconsultancy.com
16 richard@qwconsultancy.com
29) An organization is a consciously coordinated social unit composed of two or more people. Answer: TRUE Explanation: An organization is a consciously coordinated social unit, composed of two or more people, that functions on a relatively continuous basis to achieve a common goal or set of goals. LO: 1.1: Define organizational behavior (OB). Difficulty: Easy Quest. Category: Concept 30) The planning function involves defining an organization's goals and establishing an overall strategy for achieving those goals. Answer: TRUE Explanation: The planning function encompasses defining an organization's goals, establishing an overall strategy for achieving those goals, and developing a comprehensive set of plans to integrate and coordinate activities. LO: 1.1: Define organizational behavior (OB). Difficulty: Easy Quest. Category: Concept 31) A degree in architecture will provide a student with the people skills required for a job. Answer: FALSE Explanation: Technical skills encompass the ability to apply specialized knowledge or expertise. The ability to understand, communicate with, motivate, and support other people, both individually and in groups, defines people skills. LO: 1.1: Define organizational behavior (OB). Difficulty: Easy Quest. Category: Concept 32) Organizational behavior applies the knowledge gained about individuals, groups, and the effect of structure on behavior in order to make organizations work more effectively. Answer: TRUE Explanation: Organizational behavior is a field of study investigating the impact that individuals, groups, and structure have on behavior within organizations to apply such knowledge toward improving an organization's effectiveness. LO: 1.1: Define organizational behavior (OB). Difficulty: Easy Quest. Category: Concept
17 richard@qwconsultancy.com
33) Discuss the growing importance of interpersonal skills in the workplace. Answer: Historically, business school curricula emphasized the technical aspects of management, focusing on economics, accounting, finance, and quantitative techniques. Course work in human behavior and people skills received relatively less attention. Over the past several decades, however, business schools have realized the role that interpersonal skills play in determining a manager's effectiveness and required courses on people skills have been added to many curricula. Developing managers' interpersonal skills also helps organizations attract and keep high-performing employees. Regardless of labor market conditions, outstanding employees are always in short supply. Having managers with good interpersonal skills is likely to make the workplace more pleasant, which in turn makes it easier to hire and keep qualified people. Creating a pleasant workplace also appears to make good economic sense. LO: 1.1: Define organizational behavior (OB). AACSB: Interpersonal relations and teamwork Difficulty: Moderate Quest. Category: Concept 34) Compare and contrast the management functions of planning, organizing, and leading. Answer: The planning function encompasses defining an organization's goals, establishing an overall strategy for achieving those goals, and developing a comprehensive set of plans to integrate and coordinate activities. On the other hand, the organizing function includes determining what tasks are to be done, who is to do them, how the tasks are to be grouped, who reports to whom, and where decisions are to be made. The leading function is comprised of managers motivating employees, directing their activities, selecting the most effective communication channels, and resolving conflicts among members. LO: 1.1: Define organizational behavior (OB). AACSB: Reflective thinking Difficulty: Moderate Quest. Category: Synthesis 35) Edwina Carruthers works as a sales manager at Northwest Bank. Using Mintzberg's concept of leadership role, explain the nature of the tasks she would be performing. Answer: The leadership role comprises hiring, training, motivating, and disciplining employees. If Carruthers undertakes the leadership role, she would be hiring sales executives and other team members. She would also be training them on product knowledge and motivating them to reach their sales targets. In addition, she would be disciplining employees to follow rules and regulations and the code of conduct of the organization. LO: 1.1: Define organizational behavior (OB). AACSB: Analytical thinking Difficulty: Moderate Quest. Category: Application
18 richard@qwconsultancy.com
36) According to Mintzberg, how does the monitor role differ from the disseminator role? Answer: The monitor role involves collecting information from external organizations and institutions, typically by scanning the news media including the Internet, and talking with other people to learn of changes in the public's tastes, what competitors may be planning, and so forth. On the other hand, the disseminator role comprises transmitting information received from outsiders or from other employees to members of the organization. LO: 1.1: Define organizational behavior (OB). AACSB: Analytical thinking Difficulty: Moderate Quest. Category: Synthesis 37) How do Mintzberg's roles of a liaison, spokesperson, and entrepreneur differ from one another? Explain. Answer: According to Mintzberg's managerial roles, a person with the role of a liaison has an interpersonal role. The role involves maintaining a network of outside contacts who provide favors and information. The role of a spokesperson, on the other hand, is an informational role. It involves transmitting information to outsiders on organization plans, policies, actions, and results. This person serves as an expert on the organization's industry. In contrast, the role of an entrepreneur is a decisional role. It deals with searching organizations and its environment for opportunities and initiating projects to bring about change. LO: 1.1: Define organizational behavior (OB). AACSB: Analytical thinking Difficulty: Moderate Quest. Category: Synthesis 38) Explain the difference between conceptual skills and people skills? Answer: Conceptual skills comprise the mental ability to analyze and diagnose complex situations. The abilities to integrate new ideas with existing processes and innovate on the job are crucial conceptual skills for today's managers. On the other hand, the ability to understand, communicate with, motivate, and support other people, both individually and in groups, defines people skills. For instance, many people are technically proficient, but they are poor listeners. They are unable to understand the needs of others or are weak at managing conflicts. Managers should have good people skills because they need to get tasks done by people. LO: 1.1: Define organizational behavior (OB). AACSB: Analytical thinking Difficulty: Moderate Quest. Category: Synthesis
19 richard@qwconsultancy.com
39) Define organizational behavior and explain how it is used by organizations. Answer: Organizational behavior is the study of what people do in an organization and how their behavior affects the organization's performance. It studies three determinants of behavior in organizations: individuals, groups, and structure. In addition, OB applies the knowledge gained about individuals, groups, and the effect of structure on behavior in order to make organizations work more effectively. It is concerned specifically with employment-related situations and thus emphasizes behavior as related to concerns such as jobs, work, absenteeism, employee turnover, productivity, human performance, and management. LO: 1.1: Define organizational behavior (OB). Difficulty: Moderate Quest. Category: Concept 40) Systematic study uses ________ to look at relationships to attribute causes and effects. A) intuition B) feelings C) data D) instinct E) premonition Answer: C Explanation: Systematic study looks at relationships to attribute causes and effects, and bases the conclusions on scientific evidence, that is, on data gathered under controlled conditions and measured and interpreted in a reasonably rigorous manner. LO: 1.2: Show the value of systemic study to OB. Difficulty: Easy Quest. Category: Concept 41) Which of the following is true regarding systematic study? A) It attributes causes and effects based on intuition. B) It involves supporting decisions based on gut feelings. C) It analyzes relationships based on previous experiences. D) It involves analyzing relationships based on scientific data. E) It involves taking action based on instinct. Answer: D Explanation: Systematic study looks at relationships to attribute causes and effects, and bases the conclusions on scientific evidence, that is, on data gathered under controlled conditions and measured and interpreted in a reasonably rigorous manner. LO: 1.2: Show the value of systemic study to OB. Difficulty: Moderate Quest. Category: Concept
20 richard@qwconsultancy.com
42) In order to predict human behavior, it is best to supplement intuitive opinions with information derived from ________. A) common sense B) direct observation C) systematic study D) speculation E) organizational theory Answer: C Explanation: To make good OB decisions it is important to use evidence to supplement intuition and experience. Evidence should come through systematic study, which involves looking at relationships, attempting to attribute causes and effects, and basing conclusions on scientific evidence, that is, on data gathered under controlled conditions and measured and interpreted in a reasonably rigorous manner. LO: 1.2: Show the value of systemic study to OB. Difficulty: Easy Quest. Category: Concept 43) Analyzing relationships, determining causes and effects, and basing conclusions on scientific evidence, all constitute aspects of ________ study. A) organizational B) intuitive C) theoretical D) systematic E) case-based Answer: D Explanation: Systematic study means looking at relationships, attempting to attribute causes and effects, and basing the conclusions on scientific evidence, that is, on data gathered under controlled conditions and measured and interpreted in a reasonably rigorous manner. LO: 1.2: Show the value of systemic study to OB. Difficulty: Easy Quest. Category: Concept 44) ________ refers to basing managerial decisions on the best available scientific proof. A) Emergency management B) Personal knowledge management C) Property management D) Evidence-based management E) Knowledge management Answer: D Explanation: The basing of managerial decisions on the best available scientific evidence can be termed as evidence-based management. LO: 1.2: Show the value of systemic study to OB. Difficulty: Easy Quest. Category: Concept
21 richard@qwconsultancy.com
45) Which of the following refers to an instinctive feeling not necessarily supported by research? A) Intuition B) Reasoning C) Rationality D) Logic E) Inference Answer: A Explanation: An instinctive feeling not necessarily supported by research is known as intuition. LO: 1.2: Show the value of systemic study to OB. Difficulty: Easy Quest. Category: Concept 46) One reason for data analytics is ________. A) eliminating all risk B) predicting any event C) encouraging repeat purchases D) facilitating decision making Answer: B Explanation: The reasons for data analytics include predicting any event; detecting how much risk is incurred at any time; and preventing catastrophes large and small. LO: 1.2: Show the value of systemic study to OB. AACSB: Reflective thinking Difficulty: Moderate Quest. Category: Concept 47) ________ revolutionized the bookselling industry. A) Mintzberg's Theory of Management B) Evidence-based management C) Luthan's findings on management behavior D) Intuition E) The availability of Big Data Answer: E Explanation: Big Data refers to the extensive use of statistical compilation and analysis. Before online selling, brick-and-mortar bookstores could collect data about book sales only to make their projections about consumer interests and trends. With the advent of Amazon, suddenly a vast array of information about consumer preferences became available for tracking: what customers bought, what they looked at, how they navigated the site, and what they were influenced by. The challenge for Amazon was to identify which statistics were persistent and predictive and to use this information to develop algorithms to forecast which books customers would like to read next. LO: 1.2: Show the value of systemic study to OB. AACSB: Reflective thinking Difficulty: Moderate Quest. Category: Critical Thinking
22 richard@qwconsultancy.com
48) Which is not a member of the Gig Economy? A) A full-time manager B) A temporary worker C) A freelancer D) An independent contractor E) A contract worker Answer: A Explanation: Temporary workers, freelancers, independent contractors, and contract workers are all examples of individuals participating in the Gig Economy. Outside a full-time employee's traditional role, non-standard worker arrangements are much more common in today's workplace. LO: 1.2: Show the value of systemic study to OB. Difficulty: Moderate Quest. Category: Concept 49) The practice known as management by walking around is limited by available hours, focus, and application. Answer: TRUE Explanation: While MBWA sounds helpful, its limitations suggest that modern practices focused on building trust and relationships are more effective for management. Limitations include available hours, focus, and application. LO: 1.2: Show the value of systemic study to OB. Difficulty: Easy Quest. Category: Concept 50) Evidence-based management complements systematic study by basing managerial decisions on the best available scientific evidence. Answer: TRUE Explanation: Evidence-based management complements systematic study by basing managerial decisions on the best available scientific evidence. Systematic study and EBM add to intuition, or those gut feelings that help understand people better. LO: 1.2: Show the value of systemic study to OB. Difficulty: Easy Quest. Category: Concept
23 richard@qwconsultancy.com
51) Why is it important to complement intuition with systematic study in our attempts to understand behavior within organizations? Answer: It is important to complement intuition with systematic study to understand behavior within organizations in order to make accurate predictions of behavior. Underlying this systematic approach is the belief that behavior is not random. Rather, one can identify fundamental consistencies underlying the behavior of all individuals and modify them to reflect individual differences. The systematic study of behavior is a means to making reasonably accurate predictions. Systematic study involves looking at relationships, attempting to attribute causes and effects, and basing our conclusions on scientific evidence, that is, on data gathered under controlled conditions and measured and interpreted in a reasonably rigorous manner. LO: 1.2: Show the value of systemic study to OB. Difficulty: Moderate Quest. Category: Concept 52) What is evidence-based management (EBM)? Answer: Evidence-based management (EBM) complements systematic study by basing managerial decisions on the best available scientific evidence. For example, we want doctors to make decisions about patient care based on the latest available evidence, and EBM argues that managers should do the same, becoming more scientific in how they think about management problems. A manager might pose a managerial question, search for the best available evidence, and apply the relevant information to the question or case at hand. LO: 1.2: Show the value of systemic study to OB. Difficulty: Moderate Quest. Category: Concept 53) Explain the meaning of Big Data and discuss how its increased availability has led to new privacy concerns. Answer: As technological capabilities for handling Big Data have increased, so have issues of privacy. This is particularly true when data collection includes surveillance instruments. For instance, an experiment in Brooklyn, New York, has been designed to improve the quality of life for residents, but the researchers will collect intensive data from infrared cameras, sensors, and smartphone Wi-Fi signals. LO: 1.2: Show the value of systemic study to OB. AACSB: Analytical thinking Difficulty: Moderate Quest. Category: Concept
24 richard@qwconsultancy.com
54) What seeks to measure, explain, and sometimes change the behavior of humans and other animals? A) Meteorology B) Anthropology C) Sociology D) Lexicology E) Psychology Answer: E Explanation: Psychology seeks to measure, explain, and sometimes change the behavior of humans and other animals. Those who have contributed and continue to add to the knowledge of OB are learning theorists, personality theorists, counseling psychologists, and most important, industrial and organizational psychologists. LO: 1.3: Identify the major behavioral science disciplines that contribute to OB. Difficulty: Easy Quest. Category: Concept 55) ________ blends concepts from both psychology and sociology to focus on people's influence on one another. A) Social psychology B) Cosmology C) Parapsychology D) Anthropology E) Psychiatry Answer: A Explanation: Social psychology, generally considered a branch of psychology, blends concepts from both psychology and sociology to focus on people's influence on one another. One major study area is change, which involves how to implement it and how to reduce barriers to its acceptance. LO: 1.3: Identify the major behavioral science disciplines that contribute to OB. Difficulty: Easy Quest. Category: Concept
25 richard@qwconsultancy.com
56) Which of the following is a difference between sociology and psychology? A) Sociology studies humans and animals whereas psychology focuses exclusively on humans. B) Sociology studies people in relation to their social culture whereas psychology focuses on the individual. C) Sociology incorporates research from social sciences, philosophy, and natural sciences; psychology does not. D) Sociology uses various methods of empirical investigation whereas psychology uses limited critical analyses. E) Sociology uses only qualitative techniques whereas psychology uses both qualitative and quantitative techniques. Answer: B Explanation: While psychology focuses on the individual, sociology studies people in relation to their social environment or culture. LO: 1.3: Identify the major behavioral science disciplines that contribute to OB. Difficulty: Moderate Quest. Category: Concept 57) The study of societies to learn about human beings and their activities is known as ________. A) anthropology B) psychology C) sociology D) psychiatry E) social psychology Answer: A Explanation: Anthropology is the study of societies to learn about human beings and their activities. LO: 1.3: Identify the major behavioral science disciplines that contribute to OB. Difficulty: Easy Quest. Category: Concept 58) ________ is most likely to involve studying organizational culture, formal organization theory and structure. A) Sociology B) Psychology C) Anthropology D) Psychiatry E) Social psychology Answer: A Explanation: Sociologists study organizational culture, formal organization theory and structure, organizational technology, communications, power, and conflict. LO: 1.3: Identify the major behavioral science disciplines that contribute to OB. Difficulty: Easy Quest. Category: Concept
26 richard@qwconsultancy.com
59) Austin Reed is a graduate student helping to organize a study on individual job satisfaction. The study focuses on the top five causes of satisfaction or dissatisfaction on the job. His department is surveying 200 individuals in 100 different types of organizations. Austin is most likely a graduate student in the department of ________. A) psychology B) anthropology C) political science D) entomology E) archaeology Answer: A Explanation: Because Austin's study focuses on the causes of individual job satisfaction, he is most likely conducting the study through the department of psychology. Psychology is defined as the science which seeks to measure, explain, and sometimes change the behavior of humans and it focuses on the individual. LO: 1.3: Identify the major behavioral science disciplines that contribute to OB. AACSB: Analytical thinking Difficulty: Moderate Quest. Category: Application 60) You are bringing together faculty from different behavioral disciplines to author a new textbook in organizational behavior. Represented are professors from psychology, sociology, social psychology, anthropology, political science, and industrial engineering. Which faculty member is most likely to furnish information about personality, learning, and motivation? A) Sociology B) Psychology C) Anthropology D) Political science E) Industrial engineering Answer: B Explanation: Psychology seeks to measure, explain, and sometimes change the behavior of humans and other animals. Psychology's focus on the individual has led to contributions in the areas of learning, personality, emotions, motivational forces, and more. LO: 1.3: Identify the major behavioral science disciplines that contribute to OB. AACSB: Analytical thinking Difficulty: Moderate Quest. Category: Application
27 richard@qwconsultancy.com
61) Myriam is analyzing the gender roles of men and women in management in the United States and comparing them to the gender roles in management in Japan. She is surveying fifty male and fifty female managers in each country to compare their daily behavior. Myriam's study exemplifies how ________ contributes to OB. A) anthropology B) psychology C) archaeology D) political science E) corporate strategy Answer: A Explanation: Myriam is an anthropologist. Much of our current understanding of organizational culture, organizational environments, and differences among national cultures is a result of the work of anthropologists or those using their methods. LO: 1.3: Identify the major behavioral science disciplines that contribute to OB. AACSB: Analytical thinking Difficulty: Moderate Quest. Category: Application 62) One major study area of social psychology is change, how to implement it, and how to reduce barriers to its acceptance. Answer: TRUE Explanation: Social psychology, generally considered a branch of psychology, blends concepts from both psychology and sociology to focus on people's influence on one another. One major study area of social psychology is change, how to implement it, and how to reduce barriers to its acceptance. LO: 1.3: Identify the major behavioral science disciplines that contribute to OB. Difficulty: Easy Quest. Category: Concept 63) While sociology focuses on the individual, psychology studies people in relation to their social environment or culture. Answer: FALSE Explanation: While psychology focuses on the individual, sociology studies people in relation to their social environment or culture. LO: 1.3: Identify the major behavioral science disciplines that contribute to OB. Difficulty: Easy Quest. Category: Concept 64) Psychology seeks to measure, explain, and sometimes change the behavior of humans and other animals. Answer: TRUE Explanation: Psychology seeks to measure, explain, and sometimes change the behavior of humans and other animals. LO: 1.3: Identify the major behavioral science disciplines that contribute to OB. Difficulty: Easy Quest. Category: Concept 28 richard@qwconsultancy.com
65) Entomology is the study of societies to learn about human beings and their activities. Answer: FALSE Explanation: Anthropology is the study of societies to learn about human beings and their activities. LO: 1.3: Identify the major behavioral science disciplines that contribute to OB. Difficulty: Easy Quest. Category: Concept 66) Explain how psychology and sociology contribute to our understanding of organizational behavior? Answer: Psychology seeks to measure, explain, and change the behavior of humans. Early industrial/organizational psychologists studied the problems of fatigue, boredom, and other working conditions that could impede efficient work performance. More recently, their contributions have expanded to include learning, perception, personality, emotions, training, leadership effectiveness, needs and motivational forces, job satisfaction, decision-making processes, performance appraisals, attitude measurement, employee-selection techniques, work design, and job stress. Sociology studies people in relation to their social environment or culture. The greatest contributions by sociologists have been in the study of group behavior in organizations, organizational culture, formal organization theory and structure, organizational technology, communications, power, and conflict. LO: 1.3: Identify the major behavioral science disciplines that contribute to OB. Difficulty: Moderate Quest. Category: Concept 67) Explain the similarities and differences between the fields of psychology, social psychology, and sociology. Answer: All three fields deal with human behavior. While psychology focuses on the individual, sociology studies people in relation to their social environment or culture. Social psychology blends concepts from both psychology and sociology, though it is generally considered a branch of psychology. It focuses on people's influence on one another. Thus, it could be said that social psychology falls between the extremes of the individual focus of psychology and the large group focus of sociology. LO: 1.3: Identify the major behavioral science disciplines that contribute to OB. AACSB: Analytical thinking Difficulty: Hard Quest. Category: Synthesis
29 richard@qwconsultancy.com
68) Which of the following is true regarding contingency variables? A) They refer to situational factors that moderate the relationship between two or more variables. B) They make simple, accurate, and sweeping generalizations about concepts in organizational behavior. C) They indicate that everyone is motivated by money and financial perks. D) They refer to scientific factors which are based on universal truths. E) They remain constant irrespective of any change in the environment. Answer: A Explanation: Contingency variables refer to situational factors that moderate the relationship between two or more variables. LO: 1.4: Demonstrate why few absolutes apply to OB. Difficulty: Moderate Quest. Category: Concept 69) Raymond Mayer is conducting a study on discrimination against campaign activists based on gender. In his study, he noted that male executives who visited Indonesian villages to promote the use of renewable energy received a highly positive response. In contrast, women who visited the same villages received an antagonistic response. However, Mayer stated that his findings only applied to villages. In this study, the presence of a village is considered a ________ variable. A) global B) general C) dependent D) non-reactive E) contingency Answer: E Explanation: Contingency variables are variables that moderate the relationship between two or more variables. For example, an OB study can say x leads to y, but only under conditions specified in z. Humans are unpredictable in nature, and thus all organizational behavior studies must have contingency variables. LO: 1.4: Demonstrate why few absolutes apply to OB. AACSB: Analytical thinking Difficulty: Hard Quest. Category: Application
30 richard@qwconsultancy.com
70) Which type of variables moderate the relationship between two or more variables? A) Global B) General C) Dependent D) Non-reactive E) Contingency Answer: E Explanation: Contingency variables are variables that moderate the relationship between two or more variables. For example, an OB study can say x leads to y, but only under conditions specified in z. Humans are unpredictable in nature, and thus all organizational behavior studies must have contingency variables. LO: 1.4: Demonstrate why few absolutes apply to OB. Difficulty: Easy Quest. Category: Concept 71) Human beings are simple in nature and, thus, simple, universal principles explain all kinds of organizational behavior. Answer: FALSE Explanation: Human beings are complex, and few, if any, simple and universal principles explain organizational behavior. Since human beings are not alike, the ability to make simple, accurate, generalizations is limited. LO: 1.4: Demonstrate why few absolutes apply to OB. Difficulty: Easy Quest. Category: Concept 72) Organizational behavior cannot offer reasonably accurate explanations of human behavior or make valid predictions. Answer: FALSE Explanation: Human beings are complex, and few, if any, simple and universal principles explain organizational behavior. Since human beings are not alike, the ability to make simple, accurate, generalizations is limited. However, it does not mean that one cannot offer reasonably accurate explanations of human behavior or make valid predictions. LO: 1.4: Demonstrate why few absolutes apply to OB. Difficulty: Easy Quest. Category: Concept
31 richard@qwconsultancy.com
73) Why do only a few absolutes apply to organizational behavior? Answer: Human beings are complex, and few, if any, simple and universal principles explain organizational behavior. Since we are not alike, our ability to make simple, accurate, generalizations is limited. For instance, two people often act very differently in the same situation, and the same person's behavior changes in different situations. Not everyone is motivated by money, and people may behave differently at a religious service than they do at a party. That does not mean, of course, that we cannot offer reasonably accurate explanations of human behavior or make valid predictions. It does mean that OB concepts must reflect situational, or contingency, conditions. LO: 1.4: Demonstrate why few absolutes apply to OB. Difficulty: Moderate Quest. Category: Concept 74) ________ is a likely outcome of globalization. A) Providing poor customer service to the majority of global customers B) Increased expatriate assignments C) Managing a culturally homogeneous workforce D) Reduced integration E) Working with people from the same culture Answer: B Explanation: As a result of globalization, you are much more likely to find yourself working outside your native country as an expatriate LO: 1.5: Identify managers' challenges and opportunities in applying OB concepts. AACSB: Diverse and multicultural work environments Difficulty: Moderate Quest. Category: Critical Thinking
32 richard@qwconsultancy.com
75) Howley Home, a popular American company selling affordable home furnishings sourced from around the world, recently appointed Richard Toms to head up the company's business in Morocco. The company's business in Morocco has been growing due to the popularity of the handmade carpets Howley Home imports. After reviewing his predecessor's files and meeting with suppliers and company representatives in Morocco, Richard notices that productivity slows down on Fridays, especially in the afternoons. Richard also notices that no products are shipped on Fridays at all. Richard sends an email to the suppliers asking that they start shipping goods on Fridays and also increase the number of hours worked. Richard is confident that these actions will improve overall efficiency and contribute to higher revenues. Which of the following is a flaw in Richard's plan? A) Most of Howley Home's Moroccan suppliers ship goods Mondays through Thursdays. B) A significant share of Morocco's revenue comes from agriculture. C) Morocco is a Muslim country in which most people practice Islam. D) The government of Morocco provides substantial subsidies to start-up companies. E) Howley Home has alliances with local furniture companies in Vietnam. Answer: C Explanation: To work effectively with people from different cultures, you need to understand how their culture and background have shaped them and adapt your management style to accommodate these differences. Managers need to know the workforce's cultural norms in each country in which they do business. In Morocco, a country where most people practice Islam, Friday is a day during which many people spend time praying. While businesses may remain open, they may offer long lunches or close early to allow employees to practice their religion. In asking suppliers and representatives to work harder on Fridays, Richard has shown his lack of knowledge of the local culture. The fact that most of the company's suppliers ship goods Mondays through Thursdays indicates that Friday is reserved for something else. The government providing subsidies to start-ups and the significant share of revenue coming from agriculture are irrelevant to the discussion. While Howley Home could move its business to Vietnam, doing so would mean that the company could no longer offer customers carpets made in Morocco. LO: 1.5: Identify managers' challenges and opportunities in applying OB concepts. AACSB: Diverse and multicultural work environments Difficulty: Hard Quest. Category: Critical Thinking
33 richard@qwconsultancy.com
76) ________ refers to the heterogeneous characteristics of organizations, work groups, and teams that recognize that their workers vary in terms of gender, age, race, ethnicity, sexual orientation, and other characteristics. A) Workforce associability B) Workforce diversity C) Cultural similarity D) Organizational congruity E) Operational homogeneity Answer: B Explanation: Workforce diversity refers to the heterogeneous characteristics of organizations, work groups, and teams that recognize that their workers vary in terms of gender, age, race, ethnicity, sexual orientation, and other characteristics. LO: 1.5: Identify managers' challenges and opportunities in applying OB concepts. AACSB: Diverse and multicultural work environments Difficulty: Moderate Quest. Category: Concept 77) ________ is the act of creating and maintaining workplaces that support and leverage the diversity of their members. A) Workforce inclusion B) Workforce diversity C) Globalization D) Ethical behavior E) Corporate social responsibility Answer: A Explanation: Workforce inclusion is the act of creating and maintaining workplaces that support and leverage the diversity of their members. LO: 1.5: Identify managers' challenges and opportunities in applying OB concepts. AACSB: Diverse and multicultural work environments Difficulty: Moderate Quest. Category: Concept
34 richard@qwconsultancy.com
78) Workforce diversity ________. A) refers to an organization's self-regulated actions to benefit society or the environment beyond what is required by law B) refers to situations in which individuals are required to define right and wrong conduct C) recognizes that the workforce is heterogeneous in its gender identity, age, race, ethnicity, sexual orientation, and other characteristics D) refers to the process in which worldwide integration and interdependence are promoted across national borders E) focuses on creating and maintaining workplaces that support and leverage their members' diversity Answer: C Explanation: Workforce diversity recognizes that the workforce is heterogenous in its gender identity, age, race, ethnicity, sexual orientation, and other characteristics. LO: 1.5: Identify managers' challenges and opportunities in applying OB concepts. AACSB: Diverse and multicultural work environments Difficulty: Moderate Quest. Category: Concept 79) Which of the following is not an example of an expatriate? A) A banker from New York City working in Hong Kong B) A British diplomat stationed in Ukraine C) A businesswoman from Tokyo working in Spain D) An Italian tour guide working in Rome E) A model from New Zealand working in Paris Answer: D Explanation: An expatriate is defined as a person who works outside of their native country. An Italian tour guide working in Rome does not meet this definition. LO: 1.5: Identify managers' challenges and opportunities in applying OB concepts. AACSB: Diverse and multicultural work environments Difficulty: Hard Quest. Category: Concept 80) According to recent studies, ________. A) workers accessing Facebook after they wake up report increased satisfaction with their lives B) most employees find it easy to get away from the virtual workplace C) employees working at home feel a strong connection to the workplace and a sense of being part of a team D) the majority of companies are allowing workers to put in fewer hours E) subjects who woke up in a positive mood and then accessed Facebook frequently found that their mood worsened during the day Answer: E Explanation: Recent studies show that subjects who woke up in a positive mood and then accessed Facebook frequently found that their moods worsened during the day. LO: 1.5: Identify managers' challenges and opportunities in applying OB concepts. Difficulty: Moderate Quest. Category: Concept 35 richard@qwconsultancy.com
81) Which best describes a situation facing a large segment of today's workforce? A) An "always-on" work environment facilitated by laptops and smartphones B) A clearly defined 8-hour workday C) A day that always starts at 8:00 am and finishes at 5:00 pm D) A virtual workplace in which team bonding is strongly emphasized E) A harmonized work-life balance Answer: A Explanation: Some organizations have policies that reinforce an "always-on" culture. The typical employee in the 1960s and 1970s showed up at a specified workplace Monday through Friday and worked for clearly defined eight or nine-hour chunks of time. That is no longer true for a large segment of today's workforce because the workplace definition has expanded to include anywhere a laptop or smartphone can go. LO: 1.5: Identify managers' challenges and opportunities in applying OB concepts. Difficulty: Hard Quest. Category: Concept 82) ________ is any action that violates widely accepted moral norms. A) An ethical dilemma B) An ethical choice C) Ethical behavior D) Corporate social responsibility E) Unethical behavior Answer: E Explanation: Unethical behavior is any action that violates widely accepted moral norms. LO: 1.5: Identify managers' challenges and opportunities in applying OB concepts. AACSB: Ethical understanding and reasoning Difficulty: Easy Quest. Category: Concept
36 richard@qwconsultancy.com
83) Christopher Richardson works as a graphic designer in Sydney. He has been contacted by a large multinational to develop a new series of designs to be used for an upcoming print advertising campaign. Christopher is really excited about the opportunity because it could lead to future business and generate connections with other high paying clients. Despite already being fully committed to other clients, Christopher decides to lie to the large multinational company and say that he has the capacity to meet the deadlines the company has requested. Which of the following best describes Christopher's actions? A) Christopher has a strong sense of corporate social responsibility. B) Christopher has avoided an ethical dilemma. C) Christopher has made an ethical choice. D) Christopher has a strong moral compass. E) Christopher has engaged in unethical behavior. Answer: E Explanation: Unethical behavior is any action that violates widely accepted moral norms. Examples include lying, cheating, stealing, and harming others. LO: 1.5: Identify managers' challenges and opportunities in applying OB concepts. AACSB: Ethical understanding and reasoning Difficulty: Hard Employability Skills: Business Ethics and Social Responsibility Quest. Category: Application 84) Which of the following is true regarding work-life balance for a majority of men and women in a recent survey? A) Work-life balance was more important than money, recognition, and autonomy. B) Money was cited as more important than work-life balance. C) Work-life balance was more important than money, but less important than recognition. D) Work-life balance was more important than recognition and autonomy, but money was cited as the most important. E) Recognition was cited as the most important, followed by work-life balance, and then money. Answer: A Explanation: As a result of increased responsibilities in and out of the workplace, employees want jobs that give them flexibility in their work schedules so they can better manage work–life conflicts. Fifty-six percent of men and women in a recent study reported that work–life balance was their definition of career success, more than money, recognition, and autonomy. LO: 1.5: Identify managers' challenges and opportunities in applying OB concepts. Difficulty: Hard Quest. Category: Concept
37 richard@qwconsultancy.com
85) Which of the following is true regarding positive organizational scholarship? A) It involves monitoring negative practices in organizations and rectifying them. B) It deals with how employees should analyze weaknesses and convert them into strengths. C) It studies how organizations develop human strengths, foster vitality, build resilience, and unlock potential. D) It dwells on limitations that an organization faces and how it can turn them into opportunities. E) It involves scrutinizing loopholes in organizational practices and filling gaps in processes. Answer: C Explanation: Positive organizational scholarship studies how organizations develop human strengths, foster vitality, build resilience, and unlock potential. Some key topics in positive OB research are engagement, hope, optimism, and resilience in the face of strain. LO: 1.5: Identify managers' challenges and opportunities in applying OB concepts. Difficulty: Moderate Quest. Category: Concept 86) Which is a key topic in positive organizational behavior research? A) Engagement B) Apathy C) Despair D) Constraint E) Pessimism Answer: A Explanation: Positive organizational scholarship studies how organizations develop human strengths, foster vitality and resilience, and unlock potential. Some key topics in positive OB research are engagement, hope, optimism, and resilience in the face of strain. LO: 1.5: Identify managers' challenges and opportunities in applying OB concepts. Difficulty: Easy Quest. Category: Concept 87) Which was found to be the most important element of success for workers today? A) Money B) Recognition C) Autonomy D) Work-life balance E) A promotion Answer: D Explanation: In a recent study, 56 percent of people reported that work-life balance, more than money, recognition, and autonomy, was their definition of success. LO: 1.5: Identify managers' challenges and opportunities in applying OB concepts. Difficulty: Hard Quest. Category: Concept
38 richard@qwconsultancy.com
88) ________ is an example of an ethical dilemma. A) Should I work extra hours to complete my assignment? B) Should I e-mail my manager about queries on the project? C) Should I discuss with the management the perks being offered? D) Should I ask my manager for leave during Christmas? E) Should I "play politics" to advance my career? Answer: E Explanation: Employees are increasingly facing ethical dilemmas and ethical choices in which they are required to identify right and wrong conduct. LO: 1.5: Identify managers' challenges and opportunities in applying OB concepts. AACSB: Ethical understanding and reasoning Difficulty: Moderate Quest. Category: Concept 89) Which of the following is not a benefit of corporate social responsibility? A) It can lead workers to identify more strongly with their organizations. B) It can help workers experience a sense of meaningfulness for giving back. C) It can help workers feel as if they belong to something greater than themselves. D) It is good for business. E) It can have a negative impact on employee mental health. Answer: E Explanation: CSR does not typically have a negative impact on employee mental health. Rather, workers respond well to CSR for several reasons. CSR can lead workers to identify more strongly with their organizations, experience a sense of meaningfulness for giving back, and feel as though they belong to something greater than themselves. CSR is also good for business. LO: 1.5: Identify managers' challenges and opportunities in applying OB concepts. AACSB: Reflective thinking Difficulty: Hard Employability Skills: Business Ethics and Social Responsibility Quest. Category: Concept 90) Which managerial issues come to the forefront during times of crisis? A) Understanding how to retain employees B) Understanding employee stress C) Understanding employee satisfaction D) Understanding how to reward employees E) Understanding employee preference Answer: B Explanation: Managing employees well when times are tough is just as hard as when times are good, if not harder. In good times, understanding how to reward, satisfy, and retain employees is at a premium. In bad times, issues like stress, decision making, and coping come to the forefront. LO: 1.5: Identify managers' challenges and opportunities in applying OB concepts. Difficulty: Moderate Quest. Category: Concept
39 richard@qwconsultancy.com
91) Workforce diversity refers to how organizations are becoming more homogeneous in terms of gender, race, and ethnicity. Answer: FALSE Explanation: Workforce diversity acknowledges a workforce of women and men; many racial and ethnic groups; individuals with a variety of physical or psychological abilities; and people who differ in age and sexual orientation. LO: 1.5: Identify managers' challenges and opportunities in applying OB concepts. AACSB: Diverse and multicultural work environments Difficulty: Easy Quest. Category: Concept 92) Positive organizational scholarship is an area of OB research that explores how organizations develop human strengths, foster vitality, build resilience, and unlock potential. Answer: TRUE Explanation: Positive organizational scholarship is an area of OB research that explores how organizations develop human strengths, foster vitality, build resilience, and unlock potential. LO: 1.5: Identify managers' challenges and opportunities in applying OB concepts. Difficulty: Moderate Quest. Category: Concept 93) The rapid globalization of the world economy has largely eliminated the need for managers to know the cultural norms of the workforce in each country where they do business. Answer: FALSE Explanation: To be effective, managers need to know the cultural norms of the workforce in each country where they do business. LO: 1.5: Identify managers' challenges and opportunities in applying OB concepts. AACSB: Diverse and multicultural work environments Difficulty: Moderate Quest. Category: Concept 94) What is the Gig Economy and how has it changed the worker experience? Answer: The Gig Economy refers to a new era of work in which many people work independently and autonomously instead of in the highly structured employer-employee relationship of the past. About one-fifth of U.S. workers are part of the Gig Economy. The transformation in the world of work has led to several changes in the worker experience, such as financial instability, job insecurity, career path uncertainty, the transience of work, and physical and relational separation. LO: 1.5: Identify managers' challenges and opportunities in applying OB concepts. AACSB: Analytical thinking Difficulty: Hard Quest. Category: Concept
40 richard@qwconsultancy.com
95) Which of the following is true regarding the basic OB model presented in the text? A) The model proposes five types of variables. B) The model proceeds from outcomes to processes. C) The model is used only in manufacturing organizations. D) The model proceeds from processes to inputs. E) The model shows that outcomes can influence inputs in the future. Answer: E Explanation: Models propose three types of variables, i.e., inputs, processes, and outcomes, at three levels of analysis, which are, individual, group, and organizational. The model (shown in Exhibit 1-5) illustrates that inputs lead to processes, which lead to outcomes; we will discuss these interrelationships at each level of analysis. Notice that the model also shows that outcomes can influence inputs in the future, highlighting the broad-reaching effect that OB initiatives can have on an organization's future. LO: 1.6: Compare the three levels of analysis in this text's OB model. Difficulty: Moderate Quest. Category: Concept 96) With reference to a basic OB model, ________ are determined in advance of the employment relationship and refer to variables such as personality, group structure, and organizational culture that lead to processes. A) actions B) inputs C) outcomes D) processes E) goals Answer: B Explanation: Inputs are variables such as personality, group structure, and organizational culture that lead to processes. These variables set the stage for what will occur in an organization later. Many are determined in advance of the employment relationship. LO: 1.6: Compare the three levels of analysis in this text's OB model. Difficulty: Easy Quest. Category: Concept 97) ________ is an example of an input at an individual level. A) Motivation B) Moods C) Values D) Perception E) Emotions Answer: C Explanation: Inputs are variables such as personality, group structure, and organizational culture that lead to processes. Inputs at an individual level comprise diversity, personality, and values. Individual level inputs include diversity, personality, and values. LO: 1.6: Compare the three levels of analysis in this text's OB model. Difficulty: Moderate Quest. Category: Concept 41 richard@qwconsultancy.com
98) Which is an example of an input at an organizational level? A) Motivation B) Conflict C) Power D) Communication E) Structure Answer: E Explanation: Inputs are variables such as personality, group structure, and organizational culture that lead to processes. Inputs at an organizational level consist of structure and culture. LO: 1.6: Compare the three levels of analysis in this text's OB model. Difficulty: Moderate Quest. Category: Concept 99) ________ is an example of a process at an individual level. A) Group cohesion B) Values C) Decision making D) Withdrawal behavior E) Power and politics Answer: C Explanation: Processes are actions that individuals, groups, and organizations engage in as a result of inputs and that lead to certain outcomes. At the individual level, processes include emotions and moods, motivation, perception, and decision making. LO: 1.6: Compare the three levels of analysis in this text's OB model. Difficulty: Easy Quest. Category: Concept 100) Which is an example of a process at a group level? A) Diversity B) Communication C) Values D) Culture E) Emotions Answer: B Explanation: Processes are actions that individuals, groups, and organizations engage in as a result of inputs and that lead to certain outcomes. At the group level, processes include communication, leadership, power and politics, and conflict and negotiation. LO: 1.6: Compare the three levels of analysis in this text's OB model. Difficulty: Easy Quest. Category: Concept
42 richard@qwconsultancy.com
101) ________ is an example of a process at the organizational level. A) Attitudes and stress B) Withdrawal behavior C) Emotions and moods D) Change of practices E) Team responsibilities Answer: D Explanation: Processes are actions that individuals, groups, and organizations engage in as a result of inputs and that lead to certain outcomes. At the organizational level, human resource management and change of practices are examples of processes. LO: 1.6: Compare the three levels of analysis in this text's OB model. Difficulty: Easy Quest. Category: Concept 102) ________ is an example of an outcome at the organizational level. A) Productivity B) Diversity C) Personality D) Mood E) Culture Answer: A Explanation: Outcomes are the key variables that one wants to explain or predict and that are affected by some other variables. At the organizational level, overall productivity and survival are examples of outcomes. LO: 1.6: Compare the three levels of analysis in this text's OB model. Difficulty: Easy Quest. Category: Concept 103) ________ is an example of an outcome at an individual level. A) Structure B) Culture C) Politics D) Power E) Attitude Answer: E Explanation: Outcomes are the key variables that one wants to explain or predict and that are affected by some other variables. At the individual level, examples of outcomes are attitudes, stress, citizenship behavior, and withdrawal behavior. LO: 1.6: Compare the three levels of analysis in this text's OB model. Difficulty: Easy Quest. Category: Concept
43 richard@qwconsultancy.com
104) The discretionary behavior that is not part of an employee's formal job requirements and that contributes to the psychological and social environment of the workplace is called organizational ________ behavior. A) withdrawal B) associative C) networked D) citizenship E) cohesive Answer: D Explanation: The discretionary behavior that is not part of an employee's formal job requirements and contributes to the workplace's psychological and social environment is called organizational citizenship behavior. LO: 1.6: Compare the three levels of analysis in this text's OB model. Difficulty: Moderate Quest. Category: Concept 105) Team ________ refers to the quantity and quality of a team's work output. A) affect B) associability C) cohesion D) performance E) maturity Answer: D Explanation: Team performance refers to the quantity and quality of a team's work output. LO: 1.6: Compare the three levels of analysis in this text's OB model. AACSB: Interpersonal relations and teamwork Difficulty: Moderate Quest. Category: Concept 106) Evidence that an organization can exist and grow over the long term is called organizational survival. Answer: TRUE Explanation: Organizational survival is evidence that an organization can exist and grow over the long term. LO: 1.6: Compare the three levels of analysis in this text's OB model. Difficulty: Easy Quest. Category: Concept 107) Productivity refers to the degree to which an organization can achieve its ends at a low cost. Answer: FALSE Explanation: Productivity refers to the combination of the effectiveness and efficiency of an organization while the degree to which an organization can achieve its ends at a low cost is known as efficiency. LO: 1.6: Compare the three levels of analysis in this text's OB model. Difficulty: Easy Quest. Category: Concept 44 richard@qwconsultancy.com
108) The discretionary behavior that is not part of an employee's formal job requirements and contributes to the workplace's psychological and social environment is called cohesive behavior. Answer: FALSE Explanation: The discretionary behavior that is not part of an employee's formal job requirements and contributes to the workplace's psychological and social environment is called organizational citizenship behavior. LO: 1.6: Compare the three levels of analysis in this text's OB model. Difficulty: Easy Quest. Category: Concept 109) Group functioning refers to the quantity and quality of a group's work output. Answer: TRUE Explanation: Group functioning refers to the quantity and quality of a group's work output. In the same way that the performance of a sports team is more than the sum of individual players' performances, group functioning in work organizations is more than the sum of individual task performances. LO: 1.6: Compare the three levels of analysis in this text's OB model. Difficulty: Easy Quest. Category: Concept 110) What is withdrawal behavior? Explain. Answer: Withdrawal behavior is the set of actions that employees take to separate themselves from the organization. There are many forms of withdrawal, ranging from showing up late or failing to attend meetings to absenteeism and turnover. Employee withdrawal can have a very negative effect on an organization. The cost of employee turnover alone has been estimated to run into the thousands of dollars, even for entry-level positions. Absenteeism also costs organizations significant amounts of money and time every year. Withdrawal behavior may result in the workflow being disrupted and important decisions being delayed. In today's changing world of work, reasonable levels of employee-initiated turnover improve organizational flexibility and employee independence, and they can lessen the need for management-initiated layoffs. LO: 1.6: Compare the three levels of analysis in this text's OB model. AACSB: Analytical thinking Difficulty: Hard Quest. Category: Concept
45 richard@qwconsultancy.com
111) What is organizational citizenship behavior? Answer: The discretionary behavior that is not part of an employee's formal job requirements and that contributes to the psychological and social environment of the workplace is called organizational citizenship behavior. Successful organizations need employees who will do more than their usual job duties—who will provide performance beyond expectations. In today's dynamic workplace, where tasks are increasingly performed by teams and flexibility is critical, employees who engage in "good citizenship" behaviors help others on their team, volunteer for extra work, avoid unnecessary conflicts, respect the spirit as well as the letter of rules and regulations, and gracefully tolerate occasional work-related impositions and nuisances. LO: 1.6: Compare the three levels of analysis in this text's OB model. Difficulty: Moderate Quest. Category: Concept 112) What is team performance? Answer: Team performance refers to the quantity and quality of a team's work output. Similar to how a sport team's performance is more than the sum of the individual players' performance, team performance in work organizations is more than the sum of the individual tasks. LO: 1.6: Compare the three levels of analysis in this text's OB model. AACSB: Interpersonal relations and teamwork Difficulty: Moderate Quest. Category: Concept 113) Using the concept of organizational survival, how would you define what makes an organization successful? Why is organizational survival important to defining organizational success? Answer: Organizational survival refers to evidence that an organization is able to exist and grow over the long term. The survival of an organization depends not just on how productive the organization is, but also on how well it fits with its environment. A company that is productively making goods and services of little value to the market is unlikely to survive for long, so survival factors in things like perceiving the market successfully, making good decisions about how and when to pursue opportunities, and engaging in successful change management to adapt to new business. LO: 1.6: Compare the three levels of analysis in this text's OB model. AACSB: Analytical thinking Difficulty: Hard Quest. Category: Concept
46 richard@qwconsultancy.com
114) Which employability skill involves the ability to intentionally and strategically manage one's behavior, effort, and emotions in the pursuit of goals? A) Leadership B) Career management C) Self-management D) Critical thinking and creativity E) Social responsibility Answer: C Explanation: Self-management is defined as the ability to intentionally and strategically manage one's behavior, effort, and emotions in the pursuit of goals. It involves building skill in selfcontrol, self-monitoring, and self-regulation. LO: 1.7: Describe the key employability skills gained from studying OB that are applicable to other majors or future careers. AACSB: Interpersonal relations and teamwork Difficulty: Easy Quest. Category: Concept 115) Purposeful and goal-directed thinking used to define and solve problems and to make decisions or form judgments related to a particular situation or set of circumstances is known as ________. A) communication B) knowledge application and analysis C) critical thinking D) collaboration E) organization Answer: C Explanation: Critical thinking involves purposeful and goal-directed thinking used to define and solve problems and to make decisions or form judgments related to a particular situation or set of circumstances. LO: 1.7: Describe the key employability skills gained from studying OB that are applicable to other majors or future careers. Difficulty: Easy Quest. Category: Concept
47 richard@qwconsultancy.com
116) ________ refers to the ability or skill to influence a group toward the achievement of a vision or set of goals. A) Leadership B) Career C) Social responsibility D) Communication E) Self-Management Answer: A Explanation: Leadership focuses on the ability or skill to influence a group toward the achievement of a vision or set of goals. LO: 1.7: Describe the key employability skills gained from studying OB that are applicable to other majors or future careers. AACSB: Interpersonal relations and teamwork Difficulty: Moderate Quest. Category: Concept
48 richard@qwconsultancy.com
Organizational Behavior, 19e (Robbins/Judge) Chapter 2 Diversity, Equity, and Inclusion in Organizations 1) Kimberly Ortiz strongly believes in working for a company that promotes diversity. She believes that such organizations are respectful of differences and allow employees more exposure. She recently attended an interview where she was told that the company follows policies that focus on organizational diversity. However, when she finally joined the company, she had a strong feeling that the company's claim was not true. Which of the following, if true, weakens Kimberly's belief that the company does not encourage diversity? A) Eighty-five percent of the top management positions in the company are held by men. B) She is the only African-American member in the entire workforce. C) The workforce is not dominated by any specific ethnic or racial group. D) The previous company she worked for made a conscious effort to employ an equal number of men and women. E) Her team members believe that they are treated equally in spite of differences in performance. Answer: C Explanation: The fact that the workforce is not dominated by any one ethnic or racial group indicates that the workplace has a mix of all ethnicities and races and does not discriminate. The fact that 85 percent of the top management positions of the company are held by men works against the argument because it shows that the company is biased toward men. In addition, if Kimberly is the only African-American member in the entire workforce, it shows that the company is biased toward other racial groups. Kimberly's previously working for a company that believes in gender equality is irrelevant to this argument. Her team members being treated equally in spite of differences in performance represents unfair discrimination. It works against the argument. LO: 2.1: Describe the two major forms of workplace diversity. AACSB: Reflective thinking Difficulty: Hard Employability Skills: Knowledge Application and Analysis Quest. Category: Critical Thinking 2) Which type of diversity refers to diversity in observable attributes such as race, ethnicity, sex, and age? A) Psychographic B) Additive C) Surface-level D) Disjunctive E) Conjunctive Answer: C Explanation: Surface-level diversity refers to differences in easily perceived characteristics, such as gender, race, ethnicity, age, and disability, that do not necessarily reflect the ways people think or feel but that may activate certain stereotypes. LO: 2.1: Describe the two major forms of workplace diversity. AACSB: Diverse and multicultural work environments Difficulty: Easy Quest. Category: Concept 49 richard@qwconsultancy.com
3) Which of the following is true with respect to surface-level diversity? A) It refers to psychographic characteristics of the members of a group. B) People with surface-level diversity will also share deep-level diversity. C) It indicates differences of values, emotions and personality traits between people. D) It refers to differences in easily perceived characteristics, such as gender and race. E) It represents a person's or group's thought processes and feelings. Answer: D Explanation: Surface-level diversity refers to differences in easily perceived characteristics, such as gender, race, ethnicity, age, and disability, that do not necessarily reflect the ways people think or feel but that may activate certain stereotypes. LO: 2.1: Describe the two major forms of workplace diversity. AACSB: Diverse and multicultural work environments Difficulty: Easy Quest. Category: Concept 4) Differences in ________ indicate surface-level diversity. A) personality B) values C) style of work D) ethnicity E) attitudes Answer: D Explanation: Surface-level diversity refers to differences in easily perceived characteristics, such as gender, race, ethnicity, age, and disability, that do not necessarily reflect the ways people think or feel but that may activate certain stereotypes. LO: 2.1: Describe the two major forms of workplace diversity. AACSB: Diverse and multicultural work environments Difficulty: Easy Quest. Category: Concept 5) Which of the following most likely indicates surface-level similarity? A) Tim and Jake are colleagues who take risks and are quick decision makers. B) The employees at GenSys prefer teamwork over individual assignments. C) Nina and Chuck are colleagues who share similar views on corporate social responsibility. D) Jane and Sara grew up in the same town and went to school together. E) Hannah and Nate are both introspective and tend to be slow to reach decisions. Answer: D Explanation: Surface-level diversity refers to differences in easily perceived characteristics, such as gender, race, ethnicity, age, and disability, that do not necessarily reflect the ways people think or feel but that may activate certain stereotypes. Surface-level diversity can lead employees to make stereotypes and assumptions about others from certain demographic backgrounds. LO: 2.1: Describe the two major forms of workplace diversity. AACSB: Diverse and multicultural work environments Difficulty: Moderate Quest. Category: Concept 50 richard@qwconsultancy.com
51 richard@qwconsultancy.com
6) Which type of diversity refers to diversity with respect to attributes that are less easy to observe initially but that can be inferred after more direct experience? A) Surface-level B) Additive C) Demographic D) Deep-level E) Conjunctive Answer: D Explanation: As people get to know one another, they become less concerned about demographic differences if they see themselves as sharing more important characteristics, such as personality and values, that represent deep-level diversity. LO: 2.1: Describe the two major forms of workplace diversity. AACSB: Diverse and multicultural work environments Difficulty: Moderate Quest. Category: Concept 7) Which of the following best represents deep-level similarity? A) Colleagues who both come from the same neighborhood in Alabama B) Employees who are college graduates with a degree in business management C) Employees who speak Spanish and share similar religious beliefs D) Employees who seek challenges in assignments and like to work collaboratively E) Employees in their mid-thirties with 10 years' work experience in the publishing industry Answer: D Explanation: Demographics mostly reflect surface-level diversity, not thoughts and feelings, and can lead employees to perceive one another through stereotypes and assumptions. However, evidence has shown that as people get to know one another, they become less concerned about demographic differences if they see themselves as sharing more important characteristics, such as personality and values, that represent deep-level diversity. LO: 2.1: Describe the two major forms of workplace diversity. AACSB: Diverse and multicultural work environments Difficulty: Easy Quest. Category: Concept
52 richard@qwconsultancy.com
8) Steven Johnson, the hiring manager for Stow-Kent Industries, recently recruited several workers to staff the company's new construction factory in Alabama. Because the jobs that were to be performed involved manual labor, Steven made an effort to hire physically strong individuals who had the physical stamina to handle the job and the energy to work for long hours. Steven most likely concentrated in which of the following surface-level characteristics when selecting the new workers? A) Personality B) Religion C) Values D) Beliefs E) Age Answer: E Explanation: Steven most likely concentrated on age when selecting the new workers. Surfacelevel diversity refers to differences in easily perceived characteristics, such as gender, race, ethnicity, or age, that do not necessarily reflect the ways people think or feel but that may activate certain stereotypes. LO: 2.1: Describe the two major forms of workplace diversity. AACSB: Analytical thinking Difficulty: Moderate Employability Skills: Knowledge Application and Analysis Quest. Category: Application 9) Deep-level diversity could be characterized by a difference in ________. A) gender B) age C) ethnicity D) disability E) values Answer: E Explanation: Deep-level diversity comprises differences in values, personality, and work preferences that become progressively more important for determining similarity as people get to know one another better. LO: 2.1: Describe the two major forms of workplace diversity. AACSB: Diverse and multicultural work environments Difficulty: Easy Quest. Category: Concept
53 richard@qwconsultancy.com
10) Which type of difference between people reflects deep-level diversity? A) Race B) Age C) Ethnicity D) Gender E) Personality Answer: E Explanation: Deep-level diversity comprises differences in values, personality, and work preferences that become progressively more important for determining similarity as people get to know one another better. LO: 2.1: Describe the two major forms of workplace diversity. AACSB: Diverse and multicultural work environments Difficulty: Easy Quest. Category: Concept 11) The management at Climate Action Development needs to recruit campaign managers for its Renewable Energy Project. They are looking for candidates who are assertive, extroverted, and who can tackle challenges head-on. Which of the following deep-level characteristics should they focus on to best help them recruit the right candidate for the job? A) Race B) Age C) Ethnicity D) Gender E) Personality Answer: E Explanation: Deep-level diversity comprises differences in values, personality, and work preferences that become progressively more important for determining similarity as people get to know one another better. LO: 2.1: Describe the two major forms of workplace diversity. AACSB: Analytical thinking Difficulty: Moderate Employability Skills: Knowledge Application and Analysis Quest. Category: Application 12) ________ is/are a biographical characteristic of an employee. A) Ethnicity B) Work preferences C) Personality D) Values E) Beliefs Answer: A Explanation: Biographical characteristics refer to personal characteristics such as age, gender, race, and ethnicity that are objective and easily obtained from personnel records. These characteristics are representative of surface-level diversity. LO: 2.1: Describe the two major forms of workplace diversity. Difficulty: Easy 54 richard@qwconsultancy.com
Quest. Category: Concept 13) Which of the following is true with respect to biographical characteristics? A) They are non-objective in nature and cannot be observed. B) They represent characteristics such as personality and work ethics. C) They cannot be obtained from personnel records. D) They are representative of surface-level diversity. E) They comprise features such as motivation and commitment. Answer: D Explanation: Biographical characteristics refer to personal characteristics such as age, gender, race, and length of tenure that are objective and easily obtained from personnel records. These characteristics are representative of surface-level diversity. LO: 2.1: Describe the two major forms of workplace diversity. AACSB: Diverse and multicultural work environments Difficulty: Easy Quest. Category: Concept 14) Which of the following explains why the relationship between age and job performance is likely to be of increasing importance during the next decade? A) The workforce is shrinking, and older workers are compelled to work beyond retirement. B) A decline in immigration has opened up more entry-level positions for older workers. C) Increasing job opportunities are opening up in the domestic sector, resulting in a dearth of employees. D) U.S. legislation, in effect, outlaws mandatory retirement. E) A majority of people believe that productivity rises as a person grows older. Answer: D Explanation: U.S. legislation, for all intents and purposes, outlaws mandatory retirement. Most U.S. workers today no longer have to retire at age 70. LO: 2.1: Describe the two major forms of workplace diversity. Difficulty: Easy Quest. Category: Concept
55 richard@qwconsultancy.com
15) Raylon Inc. is in the process of recruiting a new operations manager. The human resources team has narrowed the candidate list down to two candidates but cannot come to an agreement about whom to hire. Though both candidates have the relevant qualifications for the post, one will soon be 60 years old. The other candidate is in his early thirties. Rachel, a member of the HR team, recommends hiring the older candidate, citing his years of experience and leadership ability. Tim, on the other hand, strongly recommends that the company hire the younger candidate, as he is likely to be more flexible when it comes to working. Which of the following, if true, most strengthens Rachel's argument? A) The job requires frequent travel around the company's six factories located across the country. B) Most of the employees the operations manager will be supervising are under 30 years of age. C) Research has shown that older workers are more prone to absenteeism for health-related issues. D) Almost half of the company's top managers are in the 45-65 age bracket. E) The company recently instituted a new production system that cuts waste by 30 percent. Answer: B Explanation: The fact that most of the employees the operations manager will supervise are under 30 indicates that the leadership ability of the older candidate will be a valuable asset. The frequent travel requirements favor the younger candidate and work against the argument. The statistics of higher absenteeism rates for older employees favor the younger candidate. The ages of the company's top managers are irrelevant to the argument. The new technology might make the older candidate's experience worthless and would favor the younger candidate. LO: 2.1: Describe the two major forms of workplace diversity. AACSB: Diverse and multicultural work environments Difficulty: Moderate Employability Skills: Knowledge Application and Analysis Quest. Category: Critical Thinking 16) Which of the following is the most likely reason why employers should employ older workers? A) They adjust to new technology promptly. B) Older workers have extensive work experience. C) Older workers are flexible and learn quickly. D) They have shorter tenures and hence lower pension benefits than younger workers. E) The rates of unavoidable absences are lower than those of younger workers. Answer: B Explanation: Managers see a number of positive qualities older workers bring to their jobs, such as experience, judgment, a strong work ethic, and commitment to quality. But older workers are also perceived as lacking flexibility and resisting new technology. LO: 2.1: Describe the two major forms of workplace diversity. AACSB: Diverse and multicultural work environments Difficulty: Moderate Quest. Category: Concept
56 richard@qwconsultancy.com
17) Thurman Inc., a manufacturing company in Vermont, needs to hire employees for its new office in the city. The positions require the employees to travel across the country regularly. The management has specified that they are looking for employees below the age of 40 who are young and dynamic. Which of the following beliefs is the management most likely to hold? A) Younger workers make stronger judgments. B) Younger workers are more flexible to change. C) Younger workers have more experience. D) Younger workers have stronger work ethics. E) Younger workers are more committed to quality. Answer: B Explanation: When organizations are actively seeking individuals who are adaptable and open to change, the negatives associated with age clearly hinder the initial hiring of older workers and increase the likelihood they will be let go during cutbacks. LO: 2.1: Describe the two major forms of workplace diversity. AACSB: Diverse and multicultural work environments Difficulty: Moderate Quest. Category: Concept 18) Angela Wells recently applied for a job at Spiga, a lounge in Paris. Having worked in several restaurants and lounges in and across France, she was confident that she would get the job. However, soon after the interview, she was informed that another candidate was offered the job, even though the other candidate had limited experience in comparison to her. Angela feels that she was discriminated against. Which of the following, if true, would best justify Angela's assumption? A) The woman who was selected over her could speak French fluently. B) Angela is a single mother with two young children. C) Angela does not have a degree in hospitality management. D) The lounge caters to elite business clientele in Paris. E) Angela learned of the position through a friend working at the lounge. Answer: B Explanation: The fact that Angela has two young children may bias the employer against her because working mothers are often known to be inflexible when it comes to work schedules. The fact that the other candidate could speak French fluently weakens Angela's assumption because it will help the candidate communicate better if she knows the local language. The fact that Angela does not have a degree in hospitality management weakens her assumption. The lounge catering to an elite clientele is irrelevant to this argument. In addition, Angela getting to know of the position through a friend who works at the lounge plays no role in this argument. LO: 2.1: Describe the two major forms of workplace diversity. AACSB: Reflective thinking Difficulty: Hard Quest. Category: Critical Thinking
57 richard@qwconsultancy.com
19) Raylon Inc. needs to hire a new floor supervisor. As the company has recently made an effort to increase diversity within the organization, the HR team wants to hire a qualified female candidate for the role instead of adequately qualified male candidates. However, top management insists that optimal performance is the top priority and that the candidate hired should be the person best suited for the job. Which of the following, if true, would most strengthen HR's case for hiring the female candidate? A) There are nearly as many female employees on the floor as male employees. B) The company has recently introduced policies that provide greater benefits for working and single mothers. C) Most of the top-level managers in the company are female. D) The company policy stipulates that each employee has to work at least one week per month on a night shift. E) Both candidates have the qualifications required to perform up to expectations. Answer: E Explanation: If both candidates have the potential to perform up to expectations, hiring the female candidate will allow the company to increase diversity while maintaining its performance. The fact that there are almost as many female employees on the floor as male employees would indicate that the workforce was diverse as to gender and would weaken HR's argument. The company's increasing benefits for working mothers would weaken HR's case if the candidate has children but would not affect it if she does not. The fact that most top-level managers are female weakens HR's argument as this would indicate that the company does incorporate gender diversity. The requirement that employees work a night shift one week a month is irrelevant to the argument. LO: 2.1: Describe the two major forms of workplace diversity. AACSB: Diverse and multicultural work environments Difficulty: Hard Employability Skills: Knowledge Application and Analysis Quest. Category: Critical Thinking 20) The heritage people use to identify themselves is referred to as ________. A) race B) values C) beliefs D) religion E) GMA Answer: A Explanation: Race refers to the heritage people use to identify themselves. LO: 2.1: Describe the two major forms of workplace diversity. AACSB: Diverse and multicultural work environments Difficulty: Easy Quest. Category: Concept
58 richard@qwconsultancy.com
21) Green Planet, an environmental organization, recently opened an office in Indonesia. The organization is currently looking for employees to staff a deforestation project in the country. They need to recruit individuals who understand the intricacies of Jakarta's culture and the mindset of the locals and are fluent in the local language. Which of the following are they most likely to consider while they recruit for these positions? A) Race B) Age C) Beliefs D) Gender E) Length of service Answer: A Explanation: Biographical characteristics such as age, gender, race, disability, and length of service are some of the most obvious ways employees differ. Race refers to the biological heritage people use to identify themselves. LO: 2.1: Describe the two major forms of workplace diversity. AACSB: Diverse and multicultural work environments Difficulty: Moderate Employability Skills: Knowledge Application and Analysis Quest. Category: Application 22) In the United States, Black people generally fare worse than White people in employment decisions. Answer: TRUE Explanation: In the United States, black people generally fare worse than White people in employment decisions. They receive lower ratings in employment interviews, lower job performance ratings, less pay, and fewer promotions. LO: 2.1: Describe the two major forms of workplace diversity. AACSB: Diverse and multicultural work environments Difficulty: Easy Quest. Category: Concept 23) Biographical characteristics mostly reflect deep-level rather than surface-level diversity. Answer: FALSE Explanation: Biographical characteristics mostly reflect surface-level diversity, not thoughts and feelings, and can lead employees to perceive one another through stereotypes and assumptions. LO: 2.1: Describe the two major forms of workplace diversity. AACSB: Diverse and multicultural work environments Difficulty: Easy Quest. Category: Concept
59 richard@qwconsultancy.com
24) Two colleagues who bond over similarities of growing up in a French-speaking locality can be said to have deep-level similarities rather than surface-level similarities. Answer: FALSE Explanation: Differences in easily perceived characteristics, such as gender, race, ethnicity, age, and disability, that do not necessarily reflect the ways people think or feel but that may activate certain stereotypes are known as surface-level diversity. LO: 2.1: Describe the two major forms of workplace diversity. AACSB: Diverse and multicultural work environments Difficulty: Easy Quest. Category: Concept 25) An individual's deeply held sense of or identification with their own gender that does not necessarily match their sex at birth, is not visible to others, and cannot be neatly categorized is their sexual orientation. Answer: FALSE Explanation: Sexual orientation refers to peoples' patterns of enduring physical, emotional, and/or romantic attraction towards others. Gender identity refers to peoples' deeply held sense of or identification with their own gender that does not necessarily match their sex at birth, is not visible to others, and cannot be categorized. LO: 2.1: Describe the two major forms of workplace diversity. AACSB: Diverse and multicultural work environments Difficulty: Easy Quest. Category: Concept 26) Recent U.S. legislation has instituted mandatory retirement at 70 in order to ensure that jobs are available for younger workers. Answer: FALSE Explanation: A reason that the relationship between age and job performance is likely to be of growing significance in the next decade is that U.S. legislation outlaws mandatory retirement. LO: 2.1: Describe the two major forms of workplace diversity. AACSB: Diverse and multicultural work environments Difficulty: Moderate Quest. Category: Concept 27) Women are more likely to be chosen for leadership roles. Answer: FALSE Explanation: Men are more likely to be chosen for leadership roles even though men and women are equally effective leaders. LO: 2.1: Describe the two major forms of workplace diversity. AACSB: Diverse and multicultural work environments Difficulty: Moderate Quest. Category: Concept
60 richard@qwconsultancy.com
28) The United States has laws against gender identity and sexual orientation discrimination. Answer: TRUE Explanation: The United States, along with the United Kingdom and Australia, have laws against gender identity and sexual orientation discrimination. LO: 2.1: Describe the two major forms of workplace diversity. AACSB: Diverse and multicultural work environments Difficulty: Moderate Quest. Category: Concept 29) Peoples' patterns of enduring physical, emotional, and/or romantic attraction towards others is known as gender identity. Answer: FALSE Explanation: Peoples' patterns of enduring physical, emotional, and/or romantic attraction towards others is known as sexual orientation inclusiveness and an acceptance of diversity. LO: 2.1: Describe the two major forms of workplace diversity. AACSB: Diverse and multicultural work environments Difficulty: Easy Quest. Category: Concept 30) The glass cliff refers to a situation in which an individual is put into a leadership position during an organizational crisis, a position in which the individual is likely to fail. Answer: TRUE Explanation: Research in Spain reveals that women are generally selected for leadership roles that require handling organizational crises — positions in which they are usually set up to fail, a phenomenon commonly referred to as the glass cliff. LO: 2.1: Describe the two major forms of workplace diversity. AACSB: Diverse and multicultural work environments Difficulty: Moderate Quest. Category: Concept 31) Research suggests that disclosing gender identity and sexual orientation is good for reducing work—family conflict, improving partner satisfaction, physical and mental well-being, and job satisfaction. Answer: TRUE Explanation: Research suggests that disclosing gender identity and sexual orientation is good for reducing work—family conflict, improving partner satisfaction, physical and mental well-being, and job satisfaction. LO: 2.1: Describe the two major forms of workplace diversity. AACSB: Diverse and multicultural work environments Difficulty: Moderate Quest. Category: Concept
61 richard@qwconsultancy.com
32) Less than 20 percent of Fortune 500 countries have policies that cover sexual orientation. Answer: FALSE Explanation: More than 90 percent of Fortune 500 companies have policies that cover sexual orientation. LO: 2.1: Describe the two major forms of workplace diversity. AACSB: Diverse and multicultural work environments Difficulty: Moderate Quest. Category: Concept 33) Describe some of the effects of systemic racism in the workplace for American racial and ethnic minorities. Answer: American ethnic and racial minorities report higher levels of discrimination in the workplace. Black people generally fare worse than White people in employment decisions, receiving lower ratings in employment interviews, lower job performance ratings, less pay, and fewer promotions. LO: 2.1: Describe the two major forms of workplace diversity. AACSB: Diverse and multicultural work environments Difficulty: Moderate Quest. Category: Concept 34) Explain the play of surface-level similarity and deep-level diversity in a workplace environment with the help of an example. Answer: Yoko and Kishi who work in a finance firm both hail from Miyagi. They speak Japanese fluently and easily bonded over their shared culture and love for karaoke. Such similarities refer to surface-level similarities. However, a few months later, when they had to work on an account together, they faced several disagreements due to differences in their work styles and personalities. Yoko was more impulsive and liked to make quick decisions. On the other hand, Kishi liked to deliberate over decisions and implementation processes. In addition, Yoko was optimistic about situations whereas Kishi was overly critical and skeptical about most processes. These fundamental differences in beliefs and thought processes resulted in clashes between them. Such differences represent deep-level diversity. LO: 2.1: Describe the two major forms of workplace diversity. AACSB: Diverse and multicultural work environments Difficulty: Hard Quest. Category: Synthesis 35) Compare and contrast surface-level diversity and deep-level diversity. Answer: Surface-level diversity refers to differences in easily perceived characteristics, such as gender, race, ethnicity, age, and disability, that do not necessarily reflect the ways people think or feel but that may activate certain stereotypes. On the other hand, differences in values, personality, and work preferences that become progressively more important for determining similarity as people get to know one another better are categorized under deep-level diversity. LO: 2.1: Describe the two major forms of workplace diversity. AACSB: Diverse and multicultural work environments Difficulty: Moderate Quest. Category: Concept 62 richard@qwconsultancy.com
36) Define gender identity and sexual orientation and explain their influence or lack thereof, on job performance and leadership. Answer: Gender identity refers to peoples' deeply held sense of identification with their own gender that does not necessarily match their sex at birth, is not visible to others, and cannot be neatly categorized. Sexual orientation refers to peoples' patterns of enduring physical, emotional, and/or romantic attraction toward others. Neither gender identity nor sexual orientation appear to affect job performance or leadership. LO: 2.1: Describe the two major forms of workplace diversity. AACSB: Diverse and multicultural work environments Difficulty: Moderate Quest. Category: Concept 37) Evaluate the benefits and disadvantages of an older workforce. Answer: There are a number of positive qualities older workers bring to their jobs, such as experience, judgment, a strong work ethic, and commitment to quality, but older workers are also perceived as lacking flexibility and resisting new technology. Despite the stereotypes, the majority of studies have shown little relationship between age and job performance. Instead, the evidence is more nuanced. In general, as people age, their job satisfaction tends to increase, most likely because their pay and benefits increase. LO: 2.1: Describe the two major forms of workplace diversity. AACSB: Diverse and multicultural work environments Difficulty: Moderate Quest. Category: Concept 38) Discuss women in leadership roles in relation to the concept of the glass cliff. Answer: While women may be offered a similar number of developmental experiences as men, women are less likely to be assigned challenging position by men, assignments that could help them achieve higher organizational positions. Moreover, men are more likely to be chosen for leadership roles even though men and women are equally effective leaders. In fact, as of April 2020, women held just 6 percent of CEO positions in S&P 500 companies. Furthermore, a study in Spain suggested that women are generally selected for leadership roles that require handling organizational crises — positions in which they are set up to fail, a phenomenon referred to as the glass cliff. LO: 2.1: Describe the two major forms of workplace diversity. AACSB: Diverse and multicultural work environments Difficulty: Moderate Quest. Category: Concept
63 richard@qwconsultancy.com
39) Discuss where most Fortune 500 companies stand on policies and procedures that cover sexual orientation and gender identity and more than 80 percent have policies covering gender identity. Answer: Surveys indicate that more than 90 percent of Fortune 500 companies have policies that cover sexual orientation. This represents a significant change from 2001 when only eight companies in the Fortune 500 had policies in gender identity. That number is now more than 850. While much has changed, the full acceptance and accommodation of lesbian, gay, bisexual, transgender, queer/questioning, and other gender identifications remains a work in progress. LO: 2.1: Describe the two major forms of workplace diversity. AACSB: Diverse and multicultural work environments Difficulty: Moderate Quest. Category: Concept 40) Peg Mayor, an employee at Los Alimitos Credit Union, recently filed a lawsuit against the company, claiming that she was discriminated against. Los Alimitos Credit Union has a policy of being unbiased toward employees and treating them fairly. Which of the following, if true, best justifies Peg's action? A) The company did not give her preferential treatment even though she was from the host country. B) She was asked whether she was over eighteen during the first round in the interview process. C) Her colleagues expressed their dissatisfaction over their pay and severance packages. D) She had faced a similar situation of discrimination in her previous company. E) She was not invited to regular Friday staff lunches with the guys. Answer: E Explanation: Los Alimitos Credit Union discriminated against Peg by not inviting her to regular staff lunches with the guys. If the company gave her preferential treatment because she was from the host country, the company would then be discriminating against the other employees. Asking whether she is over eighteen years of age is a question that interviewers are allowed to ask during the interview process to ensure the candidate is legally allowed to gain employment. Whether Peg faced a similar situation in the previous company is irrelevant to the argument. Peg's colleagues stating their unhappiness about their pay and severance package is not related to Peg filing a lawsuit against the company for discriminating against her. LO: 2.2: Demonstrate how workplace prejudice and discrimination undermines organizational effectiveness. AACSB: Reflective thinking Difficulty: Hard Employability Skills: Critical Thinking Quest. Category: Critical Thinking
64 richard@qwconsultancy.com
41) In a workplace, ________ involves overt threats or bullying directed at members of specific groups of employees. A) intimidation B) risk aversion C) inequity aversion D) optimism bias E) ambiguity aversion Answer: A Explanation: In a workplace, intimidation involves overt threats or bullying directed at members of specific groups of employees. LO: 2.2: Demonstrate how workplace prejudice and discrimination undermines organizational effectiveness. AACSB: Ethical understanding and reasoning Difficulty: Easy Quest. Category: Concept 42) April's colleague Nathan has consistently pestered her to go out on a date with him. Though she has refused his offer several times, he keeps persisting. She found an envelope on her desk from Nathan with inappropriate pictures and cartoons, which left her infuriated. She now intends to go discuss the matter with the human resource department of her company. Which of the following forms of discrimination is April most likely to cite? A) Sexual harassment B) Cyberstalking C) Mobbing D) Exclusion E) Electronic harassment Answer: A Explanation: Sexual harassment refers to unwanted sexual advances and other verbal or physical conduct of a sexual nature that create a hostile or offensive work environment. LO: 2.2: Demonstrate how workplace prejudice and discrimination undermines organizational effectiveness. AACSB: Analytical thinking Difficulty: Moderate Employability Skills: Knowledge Application and Analysis Quest. Category: Application
65 richard@qwconsultancy.com
43) Which of the following refers to a kind of discrimination that refers to overt threats or bullying directed at members of specific groups of employees? A) Intimidation B) Impact bias C) Inequity aversion D) Framing effect E) Benefaction Answer: A Explanation: Intimidation is a kind of discrimination which refers to overt threats or bullying directed at members of specific groups of employees. LO: 2.2: Demonstrate how workplace prejudice and discrimination undermines organizational effectiveness. AACSB: Diverse and multicultural work environments Difficulty: Moderate Quest. Category: Concept 44) Which of the following is the best example of the use of intimidation? A) Anya's boss, Kira, sets difficult targets and hints that Anya may lose her job if she doesn't meet them. B) Jill's manager, Steve, rarely considers her business ideas, but he consistently praises the contributions of Jill's colleague, Emily. C) Natasha's colleagues often make jokes using cultural or ethnic stereotypes. D) Although Kathleen had spent a longer time in the organization, her colleague Gary was promoted to a management position. E) Jane's colleagues recently forgot to invite her to a team lunch, a fact that caused her great distress. Answer: A Explanation: Intimidation is a kind of discrimination which refers to overt threats or bullying directed at members of specific groups of employees. LO: 2.2: Demonstrate how workplace prejudice and discrimination undermines organizational effectiveness. AACSB: Diverse and multicultural work environments Difficulty: Moderate Quest. Category: Concept
66 richard@qwconsultancy.com
45) Hazel Samuels has been working at her company for the past two years and consistently gets yelled at by her manager even when she is not at fault. He often makes derogatory references to her ethnicity. In addition, during team meetings, she is his target for practical jokes and nasty pranks. She is extremely upset and decides not to be subjected to such treatment any longer. Which of the following kinds of discrimination is she most likely to report to the human resource department in such a situation? A) Intimidation B) Sexual harassment C) Hazing D) Mobbing E) Incivility Answer: A Explanation: Intimidation is a kind of discrimination which refers to overt threats or bullying directed at members of specific groups of employees. LO: 2.2: Demonstrate how workplace prejudice and discrimination undermines organizational effectiveness. AACSB: Diverse and multicultural work environments Difficulty: Moderate Employability Skills: Knowledge Application and Analysis Quest. Category: Application 46) Which of the following is the best example of mockery as a tool of discriminatory treatment in organizations? A) Rifka's supervisor often stereotypes Rifka because of her ethnic heritage and makes jokes about her ethnicity. B) Because she is considerably older than the rest of her team, Hannah's colleagues often do not invite her to team lunches or informal team get-togethers. C) Joan's boss sets impossible targets and reprimands her in front of her colleagues if she fails to complete them on time. D) Sheena is the only African-American employee in her department and often feels left out of office jokes and gossip. E) Most of Leanne's co-workers are male and share inappropriate jokes at the office. Answer: A Explanation: Mockery often refers to jokes or jokes about negative stereotypes. LO: 2.2: Demonstrate how workplace prejudice and discrimination undermines organizational effectiveness. AACSB: Diverse and multicultural work environments Difficulty: Moderate Quest. Category: Concept
67 richard@qwconsultancy.com
47) An employee who may be made fun of because he is an Arab-American is being subjected to ________, a kind of discrimination in work environments. A) mockery and insults B) vandalism C) cyberstalking D) exclusion E) incivility Answer: A Explanation: Mockery and insults often refer to jokes or negative stereotypes. For instance, Arab-Americans that may be asked whether they were carrying bombs or were members of terrorist organizations is an example of mockery and insults. LO: 2.2: Demonstrate how workplace prejudice and discrimination undermines organizational effectiveness. AACSB: Diverse and multicultural work environments Difficulty: Moderate Quest. Category: Concept 48) Stacy Hanes is an African-American woman who has recently taken her first job and does not seem to like the work environment at all. Her colleagues put down her ideas at team meetings and refer to them as "stupid." In addition, they make fun of her race and often ask her if she is educated enough to work for the company. Which of the following kinds of discrimination is Hanes most likely subject to? A) Mockery B) Sexual harassment C) Stalking D) Deception E) Incivility Answer: A Explanation: Jokes or negative stereotypes, which are sometimes the result of jokes taken too far, refer to mockery and insults. LO: 2.2: Demonstrate how workplace prejudice and discrimination undermines organizational effectiveness. AACSB: Diverse and multicultural work environments Difficulty: Moderate Employability Skills: Knowledge Application and Analysis Quest. Category: Application
68 richard@qwconsultancy.com
49) ________, which may occur intentionally or unintentionally, refers to keeping certain people in a workplace away from job opportunities, social events, discussions, or informal mentoring. A) Exclusion B) Mockery C) Stalking D) Ragging E) Bullying Answer: A Explanation: Exclusion of certain people from job opportunities, social events, discussions, or informal mentoring may occur unintentionally. For instance, many women in finance claim they are assigned to marginal job roles or are given light workloads that do not lead to promotion. LO: 2.2: Demonstrate how workplace prejudice and discrimination undermines organizational effectiveness. AACSB: Diverse and multicultural work environments Difficulty: Easy Quest. Category: Concept 50) Jill Ivey has been working as a sales executive at Orbit Bank for the last two years. However, she has noticed that she is often not given an opportunity to attend training programs that her team members attend every quarter. She also is disappointed about the fact that she never gets to handle corporate clients, which provides better chances of a promotion. Which of the following kinds of discrimination is she most likely to have been subjected to? A) Exclusion B) Mockery C) Gaslighting D) Destabilization E) Mobbing Answer: A Explanation: Exclusion of certain people from job opportunities, social events, discussions, or informal mentoring may occur unintentionally. For instance, many women in finance claim they are assigned to marginal job roles or are given light workloads that do not lead to promotion. LO: 2.2: Demonstrate how workplace prejudice and discrimination undermines organizational effectiveness. AACSB: Analytical thinking Difficulty: Moderate Employability Skills: Knowledge Application and Analysis Quest. Category: Application
69 richard@qwconsultancy.com
51) Which type of discrimination occurs when women are assigned marginal job roles that do not lead to promotion? A) Hazing B) Gaslighting C) Coercion D) Exclusion E) Battery Answer: D Explanation: The exclusion of certain people from job opportunities, social events, discussions, or informal mentoring may occur unintentionally. For instance, many women in finance claim they are assigned to marginal job roles or are given light workloads that do not lead to promotion. LO: 2.2: Demonstrate how workplace prejudice and discrimination undermines organizational effectiveness. AACSB: Analytical thinking Difficulty: Moderate Quest. Category: Concept 52) Which of the following is true with respect to workplace discrimination? A) Discrimination occurs more at lower levels in the organization than higher levels. B) It may lead to reduced productivity and citizenship behavior. C) Only intentional discrimination is addressed by diversity management efforts. D) Forms of discrimination like exclusion are easy to root out. E) The discriminators are invariably aware of their action toward the victim. Answer: B Explanation: Discrimination can occur in many ways, and its effects can be just as varied depending on the organizational context and the personal biases of its members. Discrimination can lead to serious negative consequences for employers, including reduced productivity and citizenship behavior, negative conflicts, and increased turnover. LO: 2.2: Demonstrate how workplace prejudice and discrimination undermines organizational effectiveness. AACSB: Diverse and multicultural work environments Difficulty: Easy Quest. Category: Concept 53) Disparate impact occurs when employment practices have an intentional discriminatory effect on a legally protected group of people. Answer: FALSE Explanation: Disparate impact occurs when employment practices have an unintentional discriminatory effect on a legally protected group of people. LO: 2.2: Demonstrate how workplace prejudice and discrimination undermines organizational effectiveness. AACSB: Diverse and multicultural work environments Difficulty: Easy Quest. Category: Concept
70 richard@qwconsultancy.com
71 richard@qwconsultancy.com
54) While prejudice often involves negative feels, benevolent prejudice can involve positive feelings. Answer: TRUE Explanation: Although we may often think of prejudice as involving uniformly negative feelings, benevolent prejudice can involve positive feelings. LO: 2.2: Demonstrate how workplace prejudice and discrimination undermines organizational effectiveness. AACSB: Diverse and multicultural work environments Difficulty: Easy Quest. Category: Concept 55) Women being passed over for promotion even when they are performing better than men is an example of a discriminatory practice. Answer: TRUE Explanation: Actions taken by representatives of an organization that deny equal opportunity to perform or unequal rewards for performance are known as discriminatory policies or practices. LO: 2.2: Demonstrate how workplace prejudice and discrimination undermines organizational effectiveness. AACSB: Analytical thinking Difficulty: Easy Quest. Category: Concept 56) Denying career-advancing overseas assignments to certain deserving employees over others is a form of exclusion, a kind of workplace discrimination. Answer: TRUE Explanation: Exclusion of certain people from job opportunities, social events, discussions, or informal mentoring, which can occur unintentionally, refers to exclusion, a form of workplace discrimination. LO: 2.2: Demonstrate how workplace prejudice and discrimination undermines organizational effectiveness. AACSB: Diverse and multicultural work environments Difficulty: Easy Quest. Category: Concept 57) Prejudice can be either explicit or implicit. Answer: TRUE Explanation: Prejudice can be either explicit or implicit. LO: 2.2: Demonstrate how workplace prejudice and discrimination undermines organizational effectiveness. Difficulty: Moderate Quest. Category: Concept
72 richard@qwconsultancy.com
58) The Age Discrimination Employment Act (ADEA) permits employment discrimination against workers who are forty or older. Answer: FALSE Explanation: The Age Discrimination Employment Act (ADEA) prohibits employment discrimination against workers who are forty or older. LO: 2.2: Demonstrate how workplace prejudice and discrimination undermines organizational effectiveness. AACSB: Diverse and multicultural work environments Difficulty: Moderate Quest. Category: Concept 59) Subtle discrimination and microaggressions are less problematic than more overt forms of discrimination. Answer: FALSE Explanation: Subtle discrimination and microaggressions can be just as detrimental as more overt forms of discrimination. LO: 2.2: Demonstrate how workplace prejudice and discrimination undermines organizational effectiveness. AACSB: Diverse and multicultural work environments Difficulty: Moderate Quest. Category: Concept 60) Explicit bias is prejudice that may be hidden outside one's conscious awareness. Answer: FALSE Explanation: Implicit bias is prejudice that may be hidden outside one's conscious awareness. LO: 2.2: Demonstrate how workplace prejudice and discrimination undermines organizational effectiveness. AACSB: Diverse and multicultural work environments Difficulty: Easy Quest. Category: Concept
73 richard@qwconsultancy.com
61) Explain how surface-level similarity can lead to unfair discrimination. Substantiate your answer with the help of a scenario. Answer: Regina works as a sales manager in Rhode Island's Corporation Bank. She strongly believes that as a manager, she plays an important role to ensure that all employees are treated equally in the company. She notices that one of her sales teams comprises Teresa, Gloria, and Antonio, who all hail from Madrid. All three graduated from the same university and began their careers with Alliance Bank. In the last few months, Regina has had a chance to review Teresa's work and notices that she has made an exceptional number of sales in the past year, way above her set targets. When it comes to giving bonuses, she assumes that since Teresa, Gloria, and Antonio have such similar educational and professional backgrounds, they are bound to have similar productivity standards. She offers all three of them the same bonus without analyzing individual performances, assuming they performed equally well. Regina's behavior in this scenario can be characterized as unfair discrimination. A person who undertakes unfair discrimination rather than looking at individual characteristics assumes everyone in a group is the same. She is stereotyping all three employees because they have surface-level similarities of similar educational and professional backgrounds. LO: 2.2: Demonstrate how workplace prejudice and discrimination undermines organizational effectiveness. AACSB: Diverse and multicultural work environments Difficulty: Hard Quest. Category: Synthesis 62) Explain the idea of microaggressions, microinsults, and microinvalidations and how they impact employees with minority or marginalized backgrounds. Answer: Microaggressions are often referred to as automatic, subtle, stunning exchanges between people that negatively impact those with minority or marginalized backgrounds. In the workplace, research has shown subtle discrimination and microaggressions in very concerning occupations. Typically, these can take the form of microinsults such as rudeness or insensitivity, or microinvalidations taking the form of language that nullifies or negates the thoughts, feelings, or experience of others. Microaggressions can be just as detrimental as more overt forms of discrimination. LO: 2.2: Demonstrate how workplace prejudice and discrimination undermines organizational effectiveness. AACSB: Diverse and multicultural work environments Difficulty: Moderate Quest. Category: Concept
74 richard@qwconsultancy.com
63) Compare and contrast three different kinds of workplace discrimination. Answer: Sexual harassment refers to verbal or physical conduct of a sexual nature that creates a hostile or offensive work environment. Intimidation refers to overt threats or bullying directed at members of specific groups of employees. Exclusion is another kind of workplace discrimination where certain people are excluded from job opportunities, social events, discussions, or informal mentoring, and it can occur unintentionally. LO: 2.2: Demonstrate how workplace prejudice and discrimination undermines organizational effectiveness. AACSB: Diverse and multicultural work environments Difficulty: Moderate Quest. Category: Concept 64) Using an example, explain the differences and similarities between exclusion and intimidation in the workplace. Answer: Ally Beal has been working in a law firm for the last two years and has been bullied by her manager ever since she joined. She is given targets that are impossible to complete within the given time frame. When she is unable to complete her work, her manager admonishes her in front of everyone. Intimidation may be defined as overt threats or bullying directed at members of specific groups of employees. In contrast, exclusion involves certain people being excluded from job opportunities, social events, discussions, or informal mentoring. If Beal is excluded from discussions that involve making decisions about client's accounts, which are being attended by all other team members, she is being subjected to exclusion. LO: 2.2: Demonstrate how workplace prejudice and discrimination undermines organizational effectiveness. AACSB: Diverse and multicultural work environments Difficulty: Moderate Employability Skills: Knowledge Application and Analysis Quest. Category: Synthesis 65) How is incivility a form of discrimination in the workplace? Use an example to illustrate your explanation. Answer: Incivility refers to disrespectful treatment, including behaving in an aggressive manner, interrupting a person, or ignoring his/her opinions. For instance, if Mark's colleagues never listen to his ideas or suggestions, he is experiencing incivility. In addition, incivility may also involve his colleagues behaving in a hostile or threatening manner. This form of discrimination is especially hard to root out because it is impossible to observe and may occur simply because the actor is not aware of the effects of his/her actions. LO: 2.2: Demonstrate how workplace prejudice and discrimination undermines organizational effectiveness. AACSB: Diverse and multicultural work environments Difficulty: Moderate Employability Skills: Knowledge Application and Analysis Quest. Category: Application
75 richard@qwconsultancy.com
66) ________ refers to judging someone based on one's perception of the group to which that person belongs. A) Stereotyping B) Heuristics C) Social categorization D) Social justification E) Social dominance Answer: A Explanation: Stereotyping refers to judging someone based on one's perception of the group to which that person belongs. LO: 2.3: Explain how four major theoretical perspectives contribute to our understanding of workplace diversity. Difficulty: Easy Quest. Category: Concept 67) The idea that group members may often accept, rationalize, legitimize, or justify their experiences with inequality, prejudice, and discrimination compared with other groups refers to ________. A) system justification B) stereotyping C) social dominance D) social categorization E) intersectionality Answer: A Explanation: The idea that group members may often accept, rationalize, legitimize, or justify their experiences with inequality, prejudice, and discrimination compared with other groups refers to system justification. LO: 2.3: Explain how four major theoretical perspectives contribute to our understanding of workplace diversity. Difficulty: Moderate Quest. Category: Concept
76 richard@qwconsultancy.com
68) Jenna Blaxton is in charge of international marketing for a large multinational food company. Jenna is waiting to meet with a colleague's niece, Britanee Bell, who is looking for career advice from powerful women in multinational companies. When Britanee arrives, Jenna, seeing Britanee's mini skirt and heavy makeup, immediately concludes the meeting is a waste of time. Jenna, accustomed to seeing young women dressed in suits and light makeup at work, thinks that Britanee's appearance does not indicate someone who is serious about work, but instead, someone who is more interested in parties. Jenna is engaging in ________. A) inductive reasoning B) stigmatizing C) stereotyping D) system justification E) polymodality Answer: C Explanation: Stereotyping involves judging someone based on one's perception of the group to which that person belongs. LO: 2.3: Explain how four major theoretical perspectives contribute to our understanding of workplace diversity. AACSB: Analytical thinking Difficulty: Moderate Employability Skills: Knowledge Application and Analysis Quest. Category: Application 69) Which theory suggests that prejudice and discrimination are based on a complex hierarchy, with one group dominating another, and the dominating group enjoying privilege not afforded to the subordinate group? A) Intersectionality B) Social dominance C) The cultural mosaic D) Stereotyping E) System justification Answer: B Explanation: Social dominance theory suggests that prejudice and discrimination are based on a complex hierarchy, with one group dominating another, and the dominating group enjoying privilege not afforded to the subordinate group. LO: 2.3: Explain how four major theoretical perspectives contribute to our understanding of workplace diversity. Difficulty: Easy Quest. Category: Concept
77 richard@qwconsultancy.com
70) ________ refers to the idea that identities interact to form different meanings and experiences. A) Inductive reasoning B) Spatial visualization C) Cultural mosaic D) Visual perception E) Intersectionality Answer: E Explanation: Intersectionality refers to the idea that identities interact to form different meanings and experiences. LO: 2.3: Explain how four major theoretical perspectives contribute to our understanding of workplace diversity. Difficulty: Easy Quest. Category: Concept 71) Attributes that cannot be readily seen, are concealable, and convey an identity that is devalued in certain social contexts refers to ________. A) stigma B) heuristics C) inductive reasoning D) visual perception E) lateral masking Answer: A Explanation: Stigma represents attributes that cannot be readily seen, are concealable, and convey an identity that is devalued in certain social contexts. For instance, many workers have felt compelled to conceal their gender identities and sexual orientations in the workplace due to fear of repercussion and mistreatment. LO: 2.3: Explain how four major theoretical perspectives contribute to our understanding of workplace diversity. Difficulty: Easy Quest. Category: Concept 72) Which of the following is not true of stereotype threat? A) It can occur during preemployment tests and assessments. B) It can occur during evaluations and everyday workplace exchanges. C) It can lead to poor job attitudes and poor performance in the employees experiencing the threat. D) It is not diminished through increasing awareness of how stereotypes are perpetuated. E) It can be reduced by adopting transparent practices that signal the value of employees. Answer: D Explanation: Increasing awareness of how stereotypes may be perpetuated can be an effective way of diminishing stereotype threat. LO: 2.3: Explain how four major theoretical perspectives contribute to our understanding of workplace diversity. Difficulty: Moderate Quest. Category: Concept 78 richard@qwconsultancy.com
73) A process through which people make sense of others by constructing social categories, or groups sharing similar characteristics is referred to as social categorization. Answer: TRUE Explanation: Social categorization refers to a process through which people make sense of others by constructing social categories, or groups sharing similar characteristics. LO: 2.3: Explain how four major theoretical perspectives contribute to our understanding of workplace diversity. AACSB: Diverse and multicultural work environments Difficulty: Easy Quest. Category: Concept 74) Define social categorization and explain its drawbacks in the workplace. Answer: Social categorization refers to a process through which people make sense of others by constructing social categories or group sharing similar characteristics. To note differences in itself is not necessarily bad. On the other hand, unfair social categorization assumes that everyone in a group is the same rather than looking at the characteristics of individuals within the group. Ultimately, social categorization can lead people to separate and divide, forging a distinction between "us" (the ingroup) and "them" (the outgroup), and lead to bias as ingroup favoritism and outgroup derogation. LO: 2.3: Explain how four major theoretical perspectives contribute to our understanding of workplace diversity. AACSB: Diverse and multicultural work environments Difficulty: Moderate Quest. Category: Concept 75) How can social categorization processes lead to lower team performance and less diversity and inclusivity in organizations? Answer: Social categorization processes can affect OB in several ways. Social categorization can lead to the fragmentation of work groups and teams. As people associate with those more similar to themselves, form subgroups with those people, and judge others as belonging to outgroups, groups and teams can fracture, leading to lower team performance. Unfortunately, social categorization may also lead people to make inferences about what others are like or are interested in that are not true, or they may even lead people to misclassify others as belonging to that group. These incorrect inferences are at the root of much of the prejudicial thinking that acts as a barrier to diverse and inclusive workplaces. LO: 2.3: Explain how four major theoretical perspectives contribute to our understanding of workplace diversity. AACSB: Diverse and multicultural work environments Difficulty: Hard Quest. Category: Synthesis
79 richard@qwconsultancy.com
76) A team of four management trainees has been assigned to assess a client's financial stability. All four team members have just completed their MBAs and have about two years of prior work experience. Two of the team's members graduated near the bottom of their Ivy League schools, while the other two members were among the top ten from their respective Big Ten institutions. What best explains a division of the team into the Ivy League managers and the Big Ten managers? A) Stereotyping B) Fault lines C) Social dominance D) Social justification E) Intersectionality Answer: B Explanation: Fault lines refer to the perceived divisions that split groups into two or more subgroups based on individual differences such as gender, race, age, work experience, and education. LO: 2.4: Describe the role diversity plays in the interactions between people. AACSB: Analytical thinking Difficulty: Moderate Quest. Category: Concept 77) Which of the following is not true of groups and teams? A) Surface level diversity appears to decrease group conflict. B) Functional diversity may improve team performance. C) Functional diversity may improve innovation. D) Diverse groups may be less likely to fall into conformity. E) Diverse groups may make fewer factual errors. Answer: A Explanation: Surface level diversity appears to increase group conflict, especially in the early stages of the group's tenure, which often lowers group morale and raises group turnover. LO: 2.4: Describe the role diversity plays in the interactions between people. AACSB: Diverse and multicultural work environments Difficulty: Moderate Quest. Category: Concept
80 richard@qwconsultancy.com
78) Michael Wang, an American of Chinese descent, has been sent by his company to lead a new initiative in Beijing. Because Michael is fluent in Mandarin, his boss felt that he would be the perfect person for the position. Michael is excited about the chance to lead the team and wants to make a good impression on his boss. He decides to implement a reward structure for people on his team who perform especially well relative to other team members. Based on this information, we can conclude that Michael ranks ________ on ________ in Hofstede's cultural framework. A) low; masculinity B) high; power distance C) high; individualism D) low; uncertainty avoidance E) high; femininity Answer: C Explanation: Michael as an American likely follows the American preference for acting as individuals rather than as members of groups. Michael's action to create a competitive environment among team members supports the idea that Michael ranks high on individualism. LO: 2.5: Discuss the implications of cross-cultural matters for organizational behavior (OB). AACSB: Diverse and multicultural work environments Difficulty: Hard Employability Skills: Knowledge Application and Analysis Quest. Category: Application 79) Anika Bohn is part of the police force in Sweden. In her position, Anika is frequently called into dangerous situations where she comes face to face with criminals. Anika is confident in her ability as a member of the force and has a reputation for being fearless in dangerous situations. Her male colleagues see her as an important member of the force and treat her as an equal in all situations. Based on this information, which of the following is correct? A) Sweden ranks high on collectivism in Hofstede's cultural framework. B) Sweden ranks high on femininity in Hofstede's cultural framework. C) Sweden ranks low on individualism in Hofstede's cultural framework. D) Sweden ranks high on power distance in Hofstede's cultural framework. E) Sweden ranks high on masculinity in Hofstede's cultural framework. Answer: B Explanation: Sweden ranks high on femininity in Hofstede's cultural framework. Countries ranking high on femininity indicate little differentiation between male and female roles, women are treated as equals of men in all aspects of the society. LO: 2.5: Discuss the implications of cross-cultural matters for organizational behavior (OB). AACSB: Diverse and multicultural work environments Difficulty: Hard Employability Skills: Knowledge Application and Analysis Quest. Category: Application
81 richard@qwconsultancy.com
80) American businesswoman Kristen Adams is meeting with her Greek counterpart Alexander Monopoulos regarding her company's new hotel location near Athens. Alexander wants Kristen to provide more details regarding the planned opening of the hotel, but with construction behind schedule, Kristen doesn't have much to share. Alexander is frustrated by the lack of clarity on the schedule and demands to know how he is expected to work in such an uncertain and ambiguous situation. Alexander's frustration reflects ________ in Hofstede's cultural framework. A) Greece's high ranking on uncertainty avoidance B) Greece's high ranking on masculinity C) the United States low ranking on collectivism D) the United States' short-term orientation E) Greece's high ranking on femininity Answer: A Explanation: Alexander's frustration reflects Greece's high ranking on uncertainty avoidance, a national culture attribute that emphasizes the extent to which a society feels threatened by uncertain and ambiguous situations and tried to avoid them. LO: 2.5: Discuss the implications of cross-cultural matters for organizational behavior (OB). AACSB: Diverse and multicultural work environments Difficulty: Hard Employability Skills: Knowledge Application and Analysis Quest. Category: Application 81) Iyesha Alexander, a recent graduate of the same Ivy League university that her mother and grandmother went to, is ready for her first real job. Iyesha knows that as a woman, she'll be a minority in the male dominated industry in which she has chosen to work, but she is confident in her ability to advocate for what she wants. Iyesha knows that as a strong American woman, she may encounter some resistance from her counterparts in certain countries, but she is up for the challenge. Iyesha's attitude reflects ________ in the GLOBE study of culture. A) the low ranking of the United States on power distance B) the high ranking of the United States on institutional collectivism C) the low ranking of the United States on humane orientation D) the high ranking of the United States on assertiveness E) the low ranking of the United States on performance orientation Answer: D Explanation: Iyesha's attitude reflects the high ranking of the United States on assertiveness, the extent to which a society emphasizes confidence and advocating for what one wants. LO: 2.5: Discuss the implications of cross-cultural matters for organizational behavior (OB). AACSB: Diverse and multicultural work environments Difficulty: Hard Employability Skills: Knowledge Application and Analysis Quest. Category: Application
82 richard@qwconsultancy.com
82) Which of the following dimensions of culture is part of the GLOBE framework, but not that of Hofstede? A) Humane orientation B) Power distance C) Uncertainty avoidance D) Collectivism E) Long-term orientation Answer: A Explanation: Humane orientation is part of the GLOBE framework, but not that of Hofstede which is comprised of power distance, uncertainty avoidance, long-term orientation, collectivism, and masculinity. The GLOBE framework is comprised of power distance, uncertainty avoidance, future orientation, institutional collectivism, in-group collectivism, gender egalitarianism, assertiveness, humane orientation, and performance orientation. LO: 2.5: Discuss the implications of cross-cultural matters for organizational behavior (OB). AACSB: Diverse and multicultural work environments Difficulty: Easy Quest. Category: Concept 83) ________ in the GLOBE framework refers to the extent to which a society values producing results, excellence, and productivity. A) Performance orientation B) Humane orientation C) Assertiveness D) Future orientation E) Power distance Answer: A Explanation: Performance orientation in the GLOBE framework refers to the extent to which a society values producing results, excellence, and productivity. LO: 2.5: Discuss the implications of cross-cultural matters for organizational behavior (OB). AACSB: Diverse and multicultural work environments Difficulty: Easy Quest. Category: Concept
83 richard@qwconsultancy.com
84) Which of the following is not true of the GLOBE framework? A) Some dimensions resemble the dimensions in the Hofstede model. B) The GLOBE framework differentiates between what is currently done and what the society aspires toward. C) The future orientation dimension refers to the extent to which a society believes its actions can influence the future. D) The in-group collectivism dimension refers to the extent to which a society supports collective action and resource distribution. E) The GLOBE framework includes some dimensions that are not present in the Hofstede model. Answer: D Explanation: The in-group collectivism dimension refers to the extent to which a society values loyalty, pride, patriotism, and cohesion. Institutional collectivism refers to the extent to which a society supports collective action and resource distribution. LO: 2.5: Discuss the implications of cross-cultural matters for organizational behavior (OB). AACSB: Diverse and multicultural work environments Difficulty: Moderate Quest. Category: Concept 85) With reference to the Hofstede's cultural framework, ________ emphasizes a tight social framework in which people expect others in groups of which they are a part to look after them and protect them. A) uncertainty avoidance B) long-term orientation C) masculinity D) collectivism E) power distance Answer: D Explanation: Collectivism emphasizes a tight social framework in which people expect others in groups of which they are a part to look after them and protect them. LO: 2.5: Discuss the implications of cross-cultural matters for organizational behavior (OB). AACSB: Diverse and multicultural work environments Difficulty: Easy Quest. Category: Concept
84 richard@qwconsultancy.com
86) According to Hofstede's framework, ________ indicates the degree to which people in a country prefer structured over unstructured situations. A) collectivism B) power distance C) long-term orientation D) uncertainty avoidance E) individualism Answer: D Explanation: According to Hofstede's framework, the degree to which people in a country prefer structured over unstructured situations defines their uncertainty avoidance. Cultures low on uncertainty avoidance are more accepting of ambiguity, are less rule oriented, take more risks, and are more readily accept change. LO: 2.5: Discuss the implications of cross-cultural matters for organizational behavior (OB). AACSB: Diverse and multicultural work environments Difficulty: Easy Quest. Category: Concept 87) With reference to the Hofstede's framework, a class or caste system that discourages upward mobility is more likely to exist in a nation that scores ________. A) high on individualism B) low on masculinity C) high on power distance D) low on uncertainty avoidance E) high on long-term orientation Answer: C Explanation: Power distance describes the degree to which people in a country accept that power in institutions and organizations is distributed unequally. A high rating on power distance means that large inequalities of power and wealth exist and are tolerated in the culture, as in a class or caste system that discourages upward mobility. LO: 2.5: Discuss the implications of cross-cultural matters for organizational behavior (OB). AACSB: Diverse and multicultural work environments Difficulty: Moderate Quest. Category: Concept
85 richard@qwconsultancy.com
88) ________ refers to the degree to which there are clear, passive norms within societies, a clear understanding of sanctions for violating those norms, and no tolerance for deviating from those norms. A) Uncertainty avoidance B) Cultural tightness-looseness C) Power distance D) Humane orientation E) Performance orientation Answer: B Explanation: Cultural tightness-looseness refers to the degree to which there are clear, passive norms within societies, a clear understanding of sanctions for violating those norms, and no tolerance for deviating from those norms. LO: 2.5: Discuss the implications of cross-cultural matters for organizational behavior (OB). AACSB: Diverse and multicultural work environments Difficulty: Easy Quest. Category: Concept 89) Which is not associated with cultural intelligence? A) Career aspirations B) Cross-cultural leadership C) Negotiation performance D) Job performance E) Physical health Answer: E Explanation: Cultural intelligence refers to a worker's ability to effectively function in culturally diverse settings and situations. It has been found to be associated with expatriate career aspirations and adjustment, job performance, psychological health and well-being, cross-cultural leadership, negotiation performance, and OC. LO: 2.5: Discuss the implications of cross-cultural matters for organizational behavior (OB). AACSB: Diverse and multicultural work environments Difficulty: Moderate Quest. Category: Concept 90) According to Hofstede's framework, individualism describes the degree to which people in a country accept that power in institutions and organizations is distributed unequally. Answer: FALSE Explanation: According to Hofstede's framework, power distance describes the degree to which people in a country accept that power in institutions and organizations is distributed unequally. LO: 2.5: Discuss the implications of cross-cultural matters for organizational behavior (OB). AACSB: Diverse and multicultural work environments Difficulty: Easy Quest. Category: Concept
86 richard@qwconsultancy.com
91) According to Hofstede's framework, individualism describes the degree to which people in a country accept that power in institutions and organizations is distributed unequally. Answer: FALSE Explanation: According to Hofstede's framework, power distance describes the degree to which people in a country accept that power in institutions and organizations is distributed unequally. LO: 2.5: Discuss the implications of cross-cultural matters for organizational behavior (OB). AACSB: Diverse and multicultural work environments Difficulty: Easy Quest. Category: Concept 92) Cultural intelligence refers to a worker's ability to effectively function in culturally diverse settings and situations. Answer: TRUE Explanation: Cultural intelligence refers to a worker's ability to effectively function in culturally diverse settings and situations. LO: 2.5: Discuss the implications of cross-cultural matters for organizational behavior (OB). AACSB: Diverse and multicultural work environments Difficulty: Easy Quest. Category: Concept 93) Cultural values are learned. Answer: TRUE Explanation: Cultural values are learned. They are passed down through generations and vary by culture. LO: 2.5: Discuss the implications of cross-cultural matters for organizational behavior (OB). AACSB: Diverse and multicultural work environments Difficulty: Moderate Quest. Category: Concept
87 richard@qwconsultancy.com
94) Describe the five value dimensions of national culture as identified by the Hofstede's framework for assessing cultures. Answer: The five value dimensions of national culture as identified by the Hofstede's framework for assessing cultures are: a) Power distance: It is the degree to which people accept that power in institutions and organizations is distributed unequally. b) Individualism versus collectivism: Individualism is the degree to which people prefer to act as individuals rather than as members of groups and believe in individual rights above all else. Collectivism emphasizes a tight social framework in which people expect others in groups of which they are a part to look after them and protect them. c) Masculinity versus femininity: Hofstede's construct of masculinity is the degree to which the culture favors traditional masculine roles such as achievement, power, and control as opposed to viewing men and women as equals. A high femininity rating means the culture sees little differentiation between male and female roles and treats women as the equals of men in all respects. d) Uncertainty avoidance: The degree to which people in a country prefer structured over unstructured situations defines their uncertainty avoidance. e) Long-term versus short-term orientation: People in a culture with long-term orientation look to the future and value thrift, persistence, and tradition. In a short-term orientation, people value the here and now; they accept change more readily and don't see commitments as impediments to change. LO: 2.5: Discuss the implications of cross-cultural matters for organizational behavior (OB). AACSB: Diverse and multicultural work environments Difficulty: Moderate Quest. Category: Concept 95) An organization that is striving to provide access to the same opportunities for all workers, recognizing that some people are afforded privileges and advantages while others are confronted with barriers and obstacles is focused on ________. A) diversity B) equity C) legitimacy D) inclusion E) social responsibility Answer: B Explanation: An organization that is striving to provide access to the same opportunities for all workers, recognizing that some people are afforded privileges and advantages while others are confronted with barriers and obstacles is focused on equity. LO: 2.6: Describe how organizations manage diversity effectively. AACSB: Diverse and multicultural work environments Difficulty: Easy Quest. Category: Concept
88 richard@qwconsultancy.com
96) According to the common ingroup identity model ________. A) bias can be reduced, and inclusion can be fostered by transforming workers' focus on what divides them to what unites them B) stamina focus on what unites rather than what divides has little effect on implicit bias C) using a focus on what unites rather than what divides stifles group creativity D) the more often people from diverse backgrounds interact with one another, the more prejudice will increase over time E) the more often people from diverse backgrounds interact with one another, the more discrimination will increase Answer: A Explanation: According to the common ingroup identity model, bias can be reduced and inclusion can be fostered by transforming workers' focus on what divides them to what unites them. LO: 2.6: Describe how organizations manage diversity effectively. AACSB: Diverse and multicultural work environments Difficulty: Moderate Quest. Category: Concept 97) Creating an environment in which all people feel valued, welcome, and included refers to ________. A) equity B) inclusion C) common in-group identity D) diversity management E) affirmation Answer: B Explanation: Creating an environment in which all people feel valued, welcome, and included refers to inclusion. LO: 2.6: Describe how organizations manage diversity effectively. AACSB: Diverse and multicultural work environments Difficulty: Easy Quest. Category: Concept 98) The use of evidence-based strategies to manage and leverage the inherent diversity of the workforce is known as diversity management. Answer: TRUE Explanation: The use of evidence-based strategies to manage and leverage the inherent diversity of the workforce is known as diversity management. LO: 2.6: Describe how organizations manage diversity effectively. AACSB: Diverse and multicultural work environments Difficulty: Easy Quest. Category: Concept
89 richard@qwconsultancy.com
99) Intergroup contact is a highly effective strategy for managing diversity in countries that emphasize power distance and status hierarchies between people and groups. Answer: FALSE Explanation: Intergroup contact is a less effective strategy for managing diversity in countries that emphasize power distance and status hierarchies between people and groups. LO: 2.6: Describe how organizations manage diversity effectively. AACSB: Diverse and multicultural work environments Difficulty: Moderate Quest. Category: Concept 100) The hiring process is one of the least effective places to apply diversity management. Answer: FALSE Explanation: The hiring process is one of the most effective places to apply diversity management. Hiring managers need to value fairness and objectivity in selecting employees and focus on the productive potential of new recruits. LO: 2.6: Describe how organizations manage diversity effectively. AACSB: Diverse and multicultural work environments Difficulty: Moderate Quest. Category: Concept 101) In the United States, affirmative action involves hitting quotas for minority groups. Answer: FALSE Explanation: In the United States, affirmative action does not involve hitting quotas for minority groups, but rather a good faith effort to recruit, select, and train qualified minorities to enhance representation and fair treatment. LO: 2.6: Describe how organizations manage diversity effectively. AACSB: Diverse and multicultural work environments Difficulty: Moderate Quest. Category: Concept 102) One method of enhancing workforce diversity is to target recruitment messages to specific demographic groups that are underrepresented in the workforce. Answer: TRUE Explanation: One method of enhancing workforce diversity is to target recruitment messages to specific demographic groups that are underrepresented in the workforce. LO: 2.6: Describe how organizations manage diversity effectively. AACSB: Diverse and multicultural work environments Difficulty: Moderate Quest. Category: Concept
90 richard@qwconsultancy.com
103) Diversity advertisements that fail to show women and minorities in positions of organizational leadership send a negative message about the organization. Answer: TRUE Explanation: Diversity advertisements that fail to show women and minorities in positions of organizational leadership send a negative message about the organization. LO: 2.6: Describe how organizations manage diversity effectively. AACSB: Diverse and multicultural work environments Difficulty: Moderate Quest. Category: Concept 104) A diversity culture refers to the shared perceptions of diversity-enhancing policies, practices, and procedures among members of an organization. Answer: FALSE Explanation: A diversity climate refers to the shared perceptions of diversity-enhancing policies, practices, and procedures among members of an organization. A diversity culture refers to the shared diversity values, prioritization of diversity, and belief that it should be fostered by members of an organization. LO: 2.6: Describe how organizations manage diversity effectively. AACSB: Diverse and multicultural work environments Difficulty: Easy Quest. Category: Concept 105) What are the three strategies used in diversity management? Answer: Diversity management involves focusing on diversity, equity, and inclusion. A focus on equity involves celebrating, rather than denigrating, the differences between people and enhancing the representation of diverse, marginalized people in the workforce. A focus on equity involves striving to provide access to the same opportunities for all workers, recognizing that some people are afforded privileges and advantages while others are confronted with barriers and obstacles. A focus on inclusion involves creating an environment in which all people feel valued, welcomed, and included. LO: 2.6: Describe how organizations manage diversity effectively. AACSB: Diverse and multicultural work environments Difficulty: Moderate Quest. Category: Synthesis 106) What is common ingroup identity? Answer: The idea behind common ingroup identity involves transforming workers' focus on what divides them to what unifies them, changing perceptions of "us" and "them" to a more inclusive "we." In essence, the theory uses the same social categorization mechanisms that divide people, to once again, unite them. LO: 2.6: Describe how organizations manage diversity effectively. AACSB: Diverse and multicultural work environments Difficulty: Moderate Quest. Category: Concept
91 richard@qwconsultancy.com
107) Describe some ways by which management can attract, select, and recruit diverse employees in an organization. Answer: A method of enhancing workforce diversity is to target recruiting messages to specific demographic groups underrepresented in the workforce. This means placing advertisements in publications geared toward specific demographic groups; recruiting at colleges, universities, and other institutions with significant numbers of underrepresented minorities; and forming partnerships with associations. Diversity advertisements that fail to show women and minorities in positions of organizational leadership send a negative message about the diversity climate at an organization. The selection process is one of the most important places to apply diversity efforts. Managers who hire need to value fairness and objectivity in selecting employees and focus on the productive potential of new recruits. When managers use a well-defined protocol for assessing applicant talent, and the organization clearly prioritizes nondiscrimination policies, qualifications become far more important in determining who gets hired than demographic characteristics. LO: 2.6: Describe how organizations manage diversity effectively. AACSB: Diverse and multicultural work environments Difficulty: Moderate Quest. Category: Concept 108) Discuss the role of leadership in promoting organizational diversity. Answer: Managers need to take an active role as leaders in pursuing diversity, equity, and inclusion in their organizations. Apart from the roles leaders play, leaders should seek to develop personal knowledge, skills, and competencies related to diversity management. Managers should assume four primary roles: (1) advocate for diversity as a resource, (2) promote positive intergroup interactions, (3) stimulate discussions and conversations among those of different backgrounds to manage the organization's knowledge, and (4) encourage continuous reflection on the organization's diversity processes, practices, and goals. LO: 2.6: Describe how organizations manage diversity effectively. AACSB: Diverse and multicultural work environments Difficulty: Moderate Quest. Category: Concept 109) Discuss the basic components necessary for a workforce program to encourage diversity within the organization. Answer: Effective, comprehensive workforce programs encouraging diversity have three distinct components. First, they teach managers about the legal framework for equal employment opportunity and encourage fair treatment of all people regardless of their demographic characteristics. Second, they teach managers how a diverse workforce will be better able to serve a diverse market of customers and clients. Third, they foster personal development practices that bring out the skills and abilities of all workers, acknowledging how differences in perspective can be a valuable way to improve performance for everyone. LO: 2.6: Describe how organizations manage diversity effectively. AACSB: Diverse and multicultural work environments Difficulty: Moderate Quest. Category: Concept
92 richard@qwconsultancy.com
93 richard@qwconsultancy.com
110) Explain why affirmative action and attempts to encourage diversity within organizations may be received unfavorably by employees. What can organizations do to minimize unfavorable reactions to policies? Answer: Student responses may vary but should incorporate the idea that most negative reactions to employment discrimination are based on the idea that discriminatory treatment is unfair. Regardless of race or gender, people are generally in favor of diversity-oriented programs, including affirmative action, if they believe the policies ensure everyone a fair opportunity to show their skills and abilities. LO: 2.6: Describe how organizations manage diversity effectively. AACSB: Diverse and multicultural work environments Difficulty: Moderate Quest. Category: Concept Organizational Behavior, 19e (Robbins/Judge) Chapter 3 Job Attitudes 1) Which of the following refers to evaluative statements or judgments concerning objects, people, or events? A) Attitude B) Behavior C) Appearance D) Demeanor E) Performance Answer: A Explanation: Attitudes are evaluative statements, either favorable or unfavorable, about objects, people, or events. They reflect how one feels about something. LO: 3.1: Contrast the three components of an attitude. Difficulty: Easy Quest. Category: Concept 2) The statement, "A person who eats meat and then fights for animal rights demonstrates double standards" is an evaluative statement. Such an opinion constitutes the ________ component of an attitude. A) cognitive B) affective C) reflective D) behavioral E) reactive Answer: A Explanation: The cognitive component of an attitude is a description of, or belief in the way things are, which is exemplified in the statement, "A person who eats meat and then fights for animal rights demonstrates double standards." LO: 3.1: Contrast the three components of an attitude. Difficulty: Moderate Quest. Category: Concept 3) Which of the following statements represents the cognitive component of attitude? 94 richard@qwconsultancy.com
A) I have decided to inform my supervisor that I will be quitting my job. B) I intend to work during the weekend to meet the month's deadline. C) I feel upset about having to work during Christmas. D) It is disappointing to know that I did not get a good evaluation. E) This job is not giving me an opportunity to explore my skills. Answer: E Explanation: The cognitive component of an attitude is a description of, or belief in the way things are, which is exemplified in the statement, "This job is not giving me an opportunity to explore my skills." LO: 3.1: Contrast the three components of an attitude. AACSB: Analytical thinking Difficulty: Moderate Quest. Category: Concept 4) Abigail Jones is a sales executive at Orbit Bank in Brussels. She is the best performer on her team and often gets the highest number of corporate accounts for the company. However, she feels that she does not get sufficient credit for her hard work. During lunch, she says to her colleague, "I have been getting the largest accounts for the bank for the past eight months. Yet, my manager never acknowledges the kind of effort I put in to get these accounts." Which component of attitude is being demonstrated by Jones? A) Positive component B) Cognitive component C) Affective component D) Behavioral component E) Reflective component Answer: B Explanation: The cognitive component of an attitude is a description of, or belief in the way things are, which is exemplified in the statement, "My manager never acknowledges the kind of effort I put in to get these accounts." LO: 3.1: Contrast the three components of an attitude. AACSB: Analytical thinking Difficulty: Moderate Employability Skills: Knowledge Application and Analysis Quest. Category: Application 5) Which of the following is an example of the affective component of an attitude? A) Believing that one achieved all objectives of a project B) Relying on the information of a company's annual report C) Perceiving whistleblowing as the right thing to do D) Feeling hurt at being unfairly accused of a wrongdoing E) Deciding to fire an employee because of underperformance Answer: D Explanation: The affective component refers to the emotional, or feeling, segment of an attitude. LO: 3.1: Contrast the three components of an attitude. Difficulty: Moderate Quest. Category: Concept 95 richard@qwconsultancy.com
96 richard@qwconsultancy.com
6) Christina Johnson is feeling upset that she was not chosen as chairperson for the upcoming election meeting. Christina's feelings represent the ________ component of an attitude. A) reactive B) behavioral C) cognitive D) affective E) reflective Answer: D Explanation: The affective component represents the emotional, or feeling, segment of an attitude. LO: 3.1: Contrast the three components of an attitude. AACSB: Analytical thinking Difficulty: Moderate Employability Skills: Knowledge Application and Analysis Quest. Category: Application 7) Janice Hartley works as a writer at a fashion magazine in New York. She was recently asked by her editor to write an article on "10 must-haves for the autumn season." Her editor has now sent back the article saying it is not interesting enough to hold the attention of the reader. Janice is upset and disappointed about the feedback. Which component of an attitude is represented in this scenario? A) Positive component B) Cognitive component C) Affective component D) Behavioral component E) Evaluative component Answer: C Explanation: The emotional, or feeling, segment of an attitude is known as the affective component. It is reflected in this scenario because Janice is upset and disappointed about the feedback received. LO: 3.1: Contrast the three components of an attitude. AACSB: Analytical thinking Difficulty: Moderate Employability Skills: Knowledge Application and Analysis Quest. Category: Application
97 richard@qwconsultancy.com
8) Kimberley Mayfield recently evaluated her subordinate's progress report. She now plans to inform her about the objectives she did not achieve and how she can perform better. By doing this, Mayfield will be demonstrating the ________ component of an attitude. A) cognitive B) affective C) reflective D) behavioral E) reactive Answer: D Explanation: The behavioral component of an attitude describes an intention to behave in a certain way toward someone or something. By planning to inform her subordinate of her performance, Mayfield is demonstrating the behavioral component of attitude. LO: 3.1: Contrast the three components of an attitude. AACSB: Analytical thinking Difficulty: Moderate Employability Skills: Knowledge Application and Analysis Quest. Category: Application 9) Which of the following statements is an example of the behavioral component of an attitude? A) I have decided to apply for the position of a campaigner in the climate department. B) I am thrilled to know that the human resource department is looking for a climate campaigner. C) The position of a climate campaigner is challenging and interesting. D) The position of a climate campaigner will allow me to explore my skills as a campaigner. E) I think the position of a climate campaigner involves extensive travel. Answer: A Explanation: The behavioral component of an attitude describes an intention to behave in a certain way toward someone or something. The statement, "I have decided to apply for the position of a campaigner in the climate department" represents an intention to apply for the position and demonstrates the behavioral component of attitude. LO: 3.1: Contrast the three components of an attitude. AACSB: Analytical thinking Difficulty: Moderate Quest. Category: Concept
98 richard@qwconsultancy.com
10) Sarah Mayer works as a security officer and is in charge of keeping track of who is in the office at any given time. She notices that some employees do not sign out of the office when they go out for meals, which makes it difficult to keep track of attendance of employees. Even though Mayer has repeatedly asked the employees to sign out, they have not followed her advice. She now decides to report the issue to her supervisor. Which of the following components of an attitude is being demonstrated by Mayer? A) Affective B) Behavioral C) Positive D) Cognitive E) Knowledge Answer: B Explanation: The behavioral component of an attitude is an intention to behave in a certain way toward someone or something. Mayer's intention to report the issue to her supervisor represents the behavioral component of her attitude. LO: 3.1: Contrast the three components of an attitude. AACSB: Analytical thinking Difficulty: Moderate Employability Skills: Knowledge Application and Analysis Quest. Category: Application 11) Attitudes are favorable or unfavorable evaluative statements about objects, people, or events. Answer: TRUE Explanation: Attitudes in OB are defined as evaluative statements or judgments concerning objects, people, or events. LO: 3.1: Contrast the three components of an attitude. Difficulty: Easy Quest. Category: Concept 12) The three components of an attitude are cognition, affect, and behavior. Answer: TRUE Explanation: Typically, researchers have assumed that attitudes have three components: cognition, affect, and behavior. LO: 3.1: Contrast the three components of an attitude. Difficulty: Easy Quest. Category: Concept 13) The affective component of attitude is the emotional, or feeling, segment of an attitude. Answer: TRUE Explanation: The affective component of attitude is the emotional, or feeling, segment of an attitude. It sets the stage for the more critical part of an attitude. LO: 3.1: Contrast the three components of an attitude. Difficulty: Easy Quest. Category: Concept
99 richard@qwconsultancy.com
14) The cognitive component of an attitude describes an intention to behave in a certain way toward someone or something. Answer: FALSE Explanation: The behavioral component of an attitude describes an intention to behave in a certain way toward someone or something. The cognitive component refers to the opinion or belief segment of an attitude. LO: 3.1: Contrast the three components of an attitude. Difficulty: Easy Quest. Category: Concept 15) Describe the three components of an attitude. Answer: The three components of an attitude are cognition, affect, and behavior. 1. The cognitive component is a description of or belief in the way things are. 2. Affect is the emotional or feeling segment of an attitude. 3. The behavioral component of an attitude refers to an intention to behave in a certain way toward someone or something. LO: 3.1: Contrast the three components of an attitude. Difficulty: Moderate Quest. Category: Concept 16) Using a workplace example, explain why it is difficult to analyze the three components of an attitude separately. Answer: The three components of an attitude are cognition, affect, and behavior. Since they are all related, it is difficult to discuss or analyze them separately. For example, a person may believe that he was unjustly passed over for promotion. This is a cognitive evaluation, but it occurs at the same time that the negative feeling, or affective attitude, takes place. The final behavior of looking for a new job occurs in conjunction with both the cognitive and affective attitudes. LO: 3.1: Contrast the three components of an attitude. Difficulty: Moderate Quest. Category: Concept 17) Which of the following is not one of the powerful characteristics that change the nature of the attitudes-behavior relationship? A) The viability of the attitude B) The importance of the attitude C) The presence of social pressures D) Whether a person has direct experience with the attitude E) The accessibility of the attitude Answer: A Explanation: Several powerful characteristics change the nature of the attitude-behavior relationship: the importance of the attitude, its correspondence to behavior; its accessibility, the presence of social pressures, and whether a person has direct experience with the attitude. LO: 3.2: Summarize the relationship between attitudes and behavior. Difficulty: Easy Quest. Category: Concept 100 richard@qwconsultancy.com
18) Which of the following does cognitive dissonance indicate between two or more attitudes or between behavior and attitudes? A) Congruity B) Tenacity C) Solidarity D) Consistency E) Incompatibility Answer: E Explanation: Cognitive dissonance is defined as any incompatibility an individual might perceive between two or more attitudes or between behavior and attitudes. LO: 3.2: Summarize the relationship between attitudes and behavior. Difficulty: Easy Quest. Category: Concept 19) ________ refers to any incompatibility between two or more attitudes or between behavior and attitudes. A) Organizational dissonance B) Cognitive dissonance C) Attitudinal clarification D) Positivity offset E) Affective reactance Answer: B Explanation: Cognitive dissonance is defined as any incompatibility an individual might perceive between two or more attitudes or between behavior and attitudes. Organizational dissonance, attitudinal clarification, positivity offset, and affective reactance are not types of attitude or behavior in OB. LO: 3.2: Summarize the relationship between attitudes and behavior. Difficulty: Easy Quest. Category: Concept
101 richard@qwconsultancy.com
20) Walter Freeman has dreamed of going to space since he was a small child. Now, as a highly successful leader of a company that offers clients solutions to reducing their carbon footprints, Walter finally has the opportunity to realize his dream. Walter has been offered a spot on a space tourist flight. The flights will last approximately 12 minutes giving its passengers the rare opportunity to see the earth below. Walter is excited about the chance to go to space but can't stop thinking about the amount of fuel that will be burned during the 12-minute joy ride. Walter is likely experiencing ________. A) cognitive dissonance B) emotional contagion C) ethical evasion D) self-concordance E) positivity offset Answer: A Explanation: There is a contradiction in what Walter Freeman is feeling and the situation in which he finds herself. He is experiencing cognitive dissonance, which may be defined as incompatibility an individual might perceive between two or more attitudes or between behavior and attitudes. LO: 3.2: Summarize the relationship between attitudes and behavior. AACSB: Analytical thinking Difficulty: Hard Employability Skills: Knowledge Application and Analysis Quest. Category: Application
102 richard@qwconsultancy.com
Sonia works as a lab research assistant at Frost Labs, which conducts trials for cosmetic products such as sprays, dyes, ointments, soaps, and the like. She joined the firm right out of college and has been working there for almost two years now. 21) Which of the following, if true, would most strengthen the argument that Sonia is experiencing cognitive dissonance? A) She believes that cosmetics help boost women's self-confidence. B) She believes that testing products on animals is an unethical practice. C) She majored in pharmacology as part of her master's degree program in college. D) She recently received a positive performance review from her supervisor. E) She is confounded by the fact that the cosmetic industry rakes in almost $7 billion annually. Answer: B Explanation: Cognitive dissonance implies an incompatibility between two or more attitudes or between behavior and attitudes. If Sonia believes that testing products on animals is unethical and yet conducts trials for cosmetic products, which would most likely be conducted on animals, it suggests that she is experiencing a conflict between her behavior and attitude. If Sonia believes that cosmetics help boost women's self-confidence, it would weaken the argument that she is experiencing cognitive dissonance, as would the fact that she majored in pharmacology, which is a discipline related to the field of cosmetic testing. A positive performance review is irrelevant or, at the most, only slightly weakens the argument that she is experiencing cognitive dissonance. The fact that Sonia is confounded by the annual revenue generated by the cosmetic industry is irrelevant to the argument. LO: 3.2: Summarize the relationship between attitudes and behavior. AACSB: Reflective thinking Difficulty: Hard Quest. Category: Critical Thinking
103 richard@qwconsultancy.com
22) Which of the following, if true, would weaken the argument that Sonia is experiencing cognitive dissonance? A) She believes that beauty is merely a superficial aspect of one's personality. B) She does not wear makeup as she is aware of the ingredients that go into making these products. C) She recently attended a presentation on the harmful effects of certain chemicals used in cosmetics. D) She believes that research and testing are an integral part of providing consumers with safe products. E) She feels that this field of work does not allow her to utilize her full potential. Answer: D Explanation: Cognitive dissonance implies an incompatibility between two or more attitudes or between behavior and attitudes. If Sonia believes that research and testing are integral to providing consumers with safe products, it suggests that her attitude and behavior (conducting trials) are in sync with one another, thereby weakening the argument that she is experiencing cognitive dissonance. If she believes that beauty is a superficial aspect of one's personality and does not wear makeup because she is aware of the ingredients that go into making such products, then it would imply an incongruence between her behavior and attitude, thereby strengthening the argument. Similarly, if Sonia is aware of the harmful effects of the chemicals used in cosmetics, it would strengthen the argument that she is experiencing cognitive dissonance. If she feels that this field of work does not allow her to utilize her full potential, it would also strengthen the argument for cognitive dissonance. LO: 3.2: Summarize the relationship between attitudes and behavior. AACSB: Reflective thinking Difficulty: Hard Quest. Category: Critical Thinking
104 richard@qwconsultancy.com
23) Anna Jonas owns a manufacturing firm in Indonesia and strongly believes that it is important that workers' rights be respected. However, because of the recent economic meltdown, she makes the management pay workers a wage which is below ethical standards. In addition, the working conditions are below standards because of low investment in safety equipment. She knows her actions are unethical but continues to do so to avoid major losses. Jonas is most likely to be experiencing ________. A) cognitive dissonance B) emotional contagion C) ethical evasion D) self-concordance E) positivity offset Answer: A Explanation: There is a contradiction in what Anna Jonas is feeling and the situation in which she finds herself. She is experiencing cognitive dissonance, which may be defined as incompatibility an individual might perceive between two or more attitudes or between behavior and attitudes. LO: 3.2: Summarize the relationship between attitudes and behavior. AACSB: Analytical thinking Difficulty: Hard Employability Skills: Knowledge Application and Analysis Quest. Category: Application 24) Which of the following statements is most likely to be true regarding cognitive dissonance? A) High dissonance is accompanied by high rewards. B) People are less inspired to reduce dissonance when it is within their control. C) People are less likely to reduce dissonance when the behavior is crucial. D) People are more motivated to reduce dissonance when attitudes are important. E) The desire to reduce dissonance is not affected by moods and emotions. Answer: D Explanation: People seek a stable consistency among their attitudes and between their attitudes and their behavior. Any form of inconsistency is uncomfortable, and individuals therefore attempt to reduce or minimize it. When there is dissonance, people alter either their attitudes or the behavior to minimize the dissonance, or they develop a rationalization for the discrepancy. LO: 3.2: Summarize the relationship between attitudes and behavior. Difficulty: Moderate Quest. Category: Concept
105 richard@qwconsultancy.com
25) Joseph Pierce is the managing director of Drake Coal Power Plant in North Yorkshire. He knows that coal is a major contributor to climate change and has made his research team study impacts of coal on the environment. After knowing the facts, he faces a high degree of dissonance between his values and behavior. Which of the following is the most likely to do to reduce the dissonance between his belief and behavior? A) Get government sanction to build and develop more power plants across the country B) Increase revenue by building power plants in developing and underdeveloped countries C) Reassure the public that there is no correlation between environment and coal production D) Hire more employees to fill positions in new power plant stations E) Provide locals benefits to substantiate for the effects of coal power stations Answer: C Explanation: Festinger proposed that cases of attitude following behavior illustrate the effects of cognitive dissonance, any incompatibility an individual might perceive between two or more attitudes or between behavior and attitudes. Research has generally concluded that people do seek consistency among their attitudes and between their attitudes and their behavior. They either alter the attitudes or the behavior, or they develop a rationalization for the discrepancy. LO: 3.2: Summarize the relationship between attitudes and behavior. AACSB: Analytical thinking Difficulty: Hard Employability Skills: Knowledge Application and Analysis Quest. Category: Application 26) Research generally supports the idea that ________ predict(s) ________. A) behavior; attitudes B) emotions; attitude C) attitudes; future behavior D) thought processes; moods E) conduct; feelings Answer: C Explanation: Research, in general, supports the idea that attitudes predict future behavior. LO: 3.2: Summarize the relationship between attitudes and behavior. Difficulty: Moderate Quest. Category: Concept
106 richard@qwconsultancy.com
27) Sandy Gilmore, a sales manager at a software company in Seattle has noticed over the past few months that one member of her team, Kelly Nolan, is showing real potential as a future project leader. Sandy thinks Kelly might be just the person to lead the next sales pitch to an important client, however she has heard through the office grapevine that Kelly may be looking for a new job. Which of the following questions would be most helpful to Sandy in understanding Kelly's intentions toward the next sales pitch? However, Anderson heard from the grapevine that Kenneth may be quitting the job. She now needs to know whether she has the intention to lead the campaign in the next few months, which are crucial months for the campaign. Which of the following questions would best help Anderson understand Kenneth's intention toward the project? A) How do you like your job on the whole? B) Do you like the work culture in the organization? C) Do you see yourself working with us in the next six months? D) Are you motivated enough to achieve all project milestones? E) Are you happy with your overall job performance? Answer: C Explanation: Specific attitudes tend to predict specific behaviors, whereas general attitudes tend to best predict general behaviors. For instance, asking someone about her intention to stay with an organization for the next six months is likely to better predict turnover for that person than asking her how satisfied she is with her job overall. On the other hand, overall job satisfaction would better predict a general behavior, such as whether the individual was engaged in her work or is motivated to contribute to her organization. LO: 3.2: Summarize the relationship between attitudes and behavior. AACSB: Analytical thinking Difficulty: Easy Employability Skills: Knowledge Application and Analysis Quest. Category: Application 28) With reference to cognitive dissonance, in which of the following situations is the attitudebehavior relationship most likely to be strong? A) The affective component of the attitude is weak. B) The behavior is not affected by the cognitive component of the attitude. C) The attitude does not reflect the person's fundamental values. D) The behavior is affected by external factors and is not entirely under the control of the person. E) The attitude refers to something with which the person has direct experience. Answer: E Explanation: The attitude-behavior relationship is likely to be much stronger if an attitude refers to something with which a person has direct personal experience. LO: 3.2: Summarize the relationship between attitudes and behavior. Difficulty: Moderate Quest. Category: Concept
107 richard@qwconsultancy.com
29) Which of the following is true with regard to moderating variables in attitude relationships? A) Attitudes that our memories can easily access are more likely to predict our behavior. B) General attitudes tend to predict particular or specific behaviors. C) Attitudes are less likely to be remembered if frequently expressed or talked about. D) The relationship between an attitude and a behavior is weaker if an attitude involves a direct relation to personal experience. E) Conflicts between attitudes and behavior occur when there are no social pressures. Answer: A Explanation: Attitudes that our memories can easily access are more likely to predict our behavior. One would be more likely to remember attitudes one frequently expresses. So, the more one talks about an attitude on a subject, the more likely one is to remember it, and the more likely it is to shape a behavior. LO: 3.2: Summarize the relationship between attitudes and behavior. Difficulty: Moderate Quest. Category: Concept 30) People seek consistency between their attitudes and their behaviors. Answer: TRUE Explanation: Any form of inconsistency is uncomfortable, and individuals will, therefore, attempt to reduce it. They will seek a stable state, which is a minimum of dissonance. LO: 3.2: Summarize the relationship between attitudes and behavior. Difficulty: Easy Quest. Category: Concept 31) Cognitive dissonance explains the linkage between attitudes and behavior. Answer: TRUE Explanation: Cases of attitude following behavior illustrate the effects of cognitive dissonance, any incompatibility an individual might perceive between two or more attitudes or between behavior and attitudes. LO: 3.2: Summarize the relationship between attitudes and behavior. Difficulty: Easy Quest. Category: Concept 32) It is relatively easy to avoid dissonance. Answer: FALSE Explanation: No individual can completely avoid dissonance. LO: 3.2: Summarize the relationship between attitudes and behavior. Difficulty: Easy Quest. Category: Concept
108 richard@qwconsultancy.com
33) Individuals will be more motivated to reduce dissonance when they believe the dissonance is due to something they cannot control. Answer: FALSE Explanation: Individuals will be more motivated to reduce dissonance when the attitudes or behavior are important or when they believe the dissonance is due to something they can control. LO: 3.2: Summarize the relationship between attitudes and behavior. Difficulty: Easy Quest. Category: Concept 34) If there is an inconsistency between an individual's attitude on a specific issue and his/her behavior, there are only two courses of action available—alter the attitude or alter the behavior. Answer: FALSE Explanation: If there is an inconsistency between an individual's attitude on a specific issue and his/her behavior, the individual can either alter the attitude or the behavior, or he or she may develop a rationalization for the discrepancy. LO: 3.2: Summarize the relationship between attitudes and behavior. Difficulty: Easy Quest. Category: Concept 35) Attitudes that our memories can easily access are more likely to predict our behavior. Answer: TRUE Explanation: Attitudes that our memories can easily access are more likely to predict our behavior. LO: 3.2: Summarize the relationship between attitudes and behavior. Difficulty: Easy Quest. Category: Concept 36) Discrepancies between attitudes and behavior tend to occur when social pressures to behave in certain ways hold exceptional power, as in most organizations. Answer: TRUE Explanation: Discrepancies between attitudes and behavior tend to occur when social pressures to behave in certain ways hold exceptional power, as in most organizations. LO: 3.2: Summarize the relationship between attitudes and behavior. Difficulty: Easy Quest. Category: Concept
109 richard@qwconsultancy.com
37) Using a workplace example and the three main components of attitudes as a framework, explain how people seek consistency among their attitudes and their behavior by reducing cognitive dissonance. Answer: Student examples may vary. The following is a sample response. Harry, working for a tobacco company, can ignore the scientific information that tobacco is harmful because he receives high rewards in the form of a high salary. This allows him to reduce the cognitive dissonance between his feelings of discomfort with his company's product and his job satisfaction. The cognitive part of Harry's attitude is the evaluation that tobacco is harmful. The affective part of the attitude is feeling certain discomfort knowing that he works for a company that is harming people. The behavioral component of Harry's attitude is ignoring the information to continue to receive the high salary. LO: 3.2: Summarize the relationship between attitudes and behavior. Difficulty: Hard Quest. Category: Synthesis 38) Explain the theory of cognitive dissonance and how individuals seek consistency among their attitudes, and between their attitudes and their behavior. Answer: Cognitive dissonance refers to any incompatibility an individual might perceive between two or more attitudes or between behavior and attitudes. Festinger argued that any form of inconsistency is uncomfortable and that individuals will attempt to reduce the dissonance and, hence, the discomfort. They will seek a stable state in which there is a minimum of dissonance. Research has generally concluded that people seek consistency among their attitudes and between their attitudes and their behavior. They do this by altering either the attitudes or the behavior or by developing a rationalization for the discrepancy. They can deny that any clear causation between the attitude and the behavior has been established. They can brainwash themselves by continually articulating the benefits of the attitude or the behavior. They can acknowledge the negative consequences of the attitude or behavior but rationalize it. They can accept the research evidence and begin actively working to better the conditions. Alternately, they can quit the attitude or the behavior because the dissonance is too great. LO: 3.2: Summarize the relationship between attitudes and behavior. Difficulty: Moderate Quest. Category: Concept
110 richard@qwconsultancy.com
39) Karen Archer is the hiring manager for a coal mining company in Kentucky. Karen finds her job especially fulfilling because she knows how important employment in the mines is for the local community where few other job prospects exist. Recently, Karen learned that top management has been hiding some disturbing information on the levels of toxins in the air in the deeper parts of the mine. Although only a couple of people are exposed to the risk, which seems slight, Karen is very upset because she views the people she hires as family. Using the knowledge that you have about Karen, explain the cognitive dissonance that she is experiencing and two ways that Karen could alter her behavior or attitude to reduce the dissonance. Answer: Students' answers may vary. The following is a sample response. As a hiring manager, Karen is horrified with the thought that she could be willfully exposing people to toxic air. The dissonance she feels is the contradiction between a sense of loyalty and love for her job and her anger at the company's actions. To reduce the dissonance, Karen could rationalize that all mines have some toxic air at lower levels and that it is worth the small risk to a few people because her company provides jobs for the whole community. Karen could accept certain excuses and evidence that the company provides, such as the toxins have only minor potential health effects Karen can ignore the dissonance because she values her job as a hiring manager in a community with few jobs. LO: 3.2: Summarize the relationship between attitudes and behavior. AACSB: Analytical thinking Difficulty: Moderate Employability Skills: Knowledge Application and Analysis Quest. Category: Application 40) Which type of employees strongly identify with and care about the kind of work they do? A) Employees with high job involvement. B) Employees with high cognitive dissonance. C) Employees with low psychological empowerment. D) Employees with low organizational commitment. E) Employees with high emotional contagion. Answer: A Explanation: Employees with high job involvement strongly identify with and care about the kind of work they do. LO: 3.3: Compare the major job attitudes. Difficulty: Moderate Quest. Category: Concept
111 richard@qwconsultancy.com
41) A positive feeling about one's job resulting from an evaluation of its characteristics is known as ________. A) job satisfaction B) job design C) positivity offset D) constructive dismissal E) picketing Answer: A Explanation: When people speak of employee attitudes, they usually mean job satisfaction, which describes a positive feeling about a job resulting from an evaluation of its characteristics. A person with a high level of job satisfaction holds positive feelings about his or her job, while a person with a low level holds negative feelings. LO: 3.3: Compare the major job attitudes. Difficulty: Easy Quest. Category: Concept 42) Which of the following statements represents a person's job attitude? A) I enjoy my work because it offers me challenges and helps me hone my networking skills. B) I believe my work has a direct impact on the processes of the organization. C) I believe my work performance indicates my caliber and potential. D) I agree with the organization about its commitment to the environment. E) I believe in the organization's objectives about supporting the underprivileged. Answer: A Explanation: When people speak of employee attitudes, they usually mean job satisfaction, which describes a positive feeling about a job resulting from an evaluation of its characteristics. A person with a high level of job satisfaction holds positive feelings about his or her job, while a person with a low level holds negative feelings. LO: 3.3: Compare the major job attitudes. AACSB: Analytical thinking Difficulty: Moderate Quest. Category: Concept
112 richard@qwconsultancy.com
43) Jane's high job involvement is best represented by which of the following actions? A) Jane always complains about her work to her colleagues. B) Jane actively takes part in team activities and proactively takes up additional job responsibilities. C) Jane shares the organization's vision of supporting renewable energy. D) Jane dislikes the unethical practices followed by her employer. E) Jane wants to continue working for the organization because many of her college friends are working there. Answer: B Explanation: Job involvement measures the degree to which people identify psychologically with their job and consider their perceived performance level important to self-worth. Employees with a high level of job involvement strongly identify with and really care about the kind of work they do. LO: 3.3: Compare the major job attitudes. AACSB: Interpersonal relations and teamwork Difficulty: Moderate Employability Skills: Knowledge Application and Analysis Quest. Category: Application 44) Employees' beliefs in the degree to which they influence their work environment, their competence, the meaningfulness of their jobs, and their perceived autonomy is termed as ________. A) psychological empowerment B) organizational dissent C) organizational commitment D) employee engagement E) secondary action Answer: A Explanation: Employees' beliefs in the degree to which they influence their work environment, competence, meaningfulness of their job, and their perceived autonomy is termed as psychological empowerment. LO: 3.3: Compare the major job attitudes. Difficulty: Easy Quest. Category: Concept
113 richard@qwconsultancy.com
45) ________ refers to the degree to which a person identifies with his or her job, actively participates in it, and considers his or her performance as being important to self-worth. A) Emotional contagion B) Job involvement C) Job stability D) Emotional dissonance E) Direct action Answer: B Explanation: Job involvement measures the degree to which people identify psychologically with their job and consider their perceived performance level important to self-worth. Employees with a high level of job involvement strongly identify with and really care about the kind of work they do. LO: 3.3: Compare the major job attitudes. Difficulty: Easy Quest. Category: Concept 46) Lorenzo Martinez has led three of his company's biggest deals over the last year. Lorenzo is proud of his work because the company had been chasing two of the clients for more than 18 months. Lorenzo expected to be recognized for his efforts at his company's year-end banquet and was disappointed that his name wasn't even among those on the Honorable Mentions list. Now, Lorenzo is feeling frustrated and resentful. He knows that with his bonus, he makes good money and has decent benefits, but the lack of recognition for his hard work is becoming a real problem for him. How can Lorenzo's job attitude best be described? A) Low job involvement B) Low job satisfaction C) High psychological empowerment D) High positivity offset E) Low emotional contagion Answer: B Explanation: Lorenzo is beginning to have a negative feeling toward his job resulting in low job satisfaction. He believes that he has invested tremendously in the organization, which shows a high job involvement. He also demonstrates a low psychological empowerment because he succumbs to the situation and feels there is nothing he can do to make the situation better. LO: 3.3: Compare the major job attitudes. AACSB: Analytical thinking Difficulty: Moderate Employability Skills: Knowledge Application and Analysis Quest. Category: Application
114 richard@qwconsultancy.com
47) Ben has been working as a process executive at an accounting firm for the past two years. A hard worker, his work is of good quality, and he often puts in extra hours at the office to ensure his schedules are on track. Noticing his efforts, the management offers him a 20 percent hike. Two months later, Ben submits his resignation and soon joins a startup organization, at a senior position. Which of the following best explains this situation? A) Ben felt the hike was unwarranted. B) Ben, though highly skilled, lacked ambition. C) Ben was motivated by the prospect of extrinsic rewards. D) Ben found his work to be routine and monotonous. E) Ben was gunning for a position at the competing firm all along. Answer: D Explanation: Even if Ben were a highly competent employee, he would be more likely to quit in spite of the salary hike if he found his work to be routine and monotonous. This implies a lack of job satisfaction and involvement. If Ben felt the hike was unwarranted, it would still not drive him to resign from the firm. Similarly, if Ben were motivated by the extrinsic rewards, it would not explain his leaving the firm as he was given a salary hike. It cannot be argued that Ben lacked ambition as he found employment with another firm, at a senior position. It also cannot be argued that Ben was aiming for a position at the competing firm all along, because he worked as a process executive for two years and he joined a startup company. LO: 3.3: Compare the major job attitudes. AACSB: Reflective thinking Difficulty: Hard Quest. Category: Critical Thinking 48) Which of the following questions best helps understand an employee's organizational commitment? A) Do you feel you have a good working relationship with team members? B) Do you feel the organization will take care of you and your family in times of need? C) Do you enjoy your work even in the face of challenges? D) Does the organization provide valuable feedback to help you increase productivity? E) Do you believe in the organization's objective on sustainable work practices? Answer: E Explanation: The degree to which an employee identifies with a particular organization and its goals and wishes to maintain membership in the organization is referred to as organizational commitment. LO: 3.3: Compare the major job attitudes. Difficulty: Easy Quest. Category: Concept
115 richard@qwconsultancy.com
49) Which of the following best describes organizational commitment? A) The degree to which employees identify with the organization they work for and its goals B) The state of discord caused by opposition of values between people working together C) The degree to which an employee's sense of cognitive dissonance is related to his/her job D) The employee's degree of disagreement or differential opinions about organizational practices E) The degree to which employees believe their work impacts their organization significantly Answer: A Explanation: The degree to which an employee identifies with a particular organization and its goals and wishes to maintain membership in the organization is known as organizational commitment. LO: 3.3: Compare the major job attitudes. Difficulty: Easy Quest. Category: Concept 50) Farm2Table, a Colorado-based food cooperative, wants to know how its employees feel about working at the company. Which of the following suggests strong organizational commitment from the employee? A) I am a great believer in the importance of a healthy diet, and this is why I am glad to be working for a company that is trying to make simple, healthy food available to all. B) I enjoy working for Farm2Table because of the growth prospects that it offers and the wide range of responsibilities each one of us has. C) Even though I only work in the production department, it's good to know that my feedback gets considered during our interactive in-house sessions. D) The challenging nature of the work and the thoughtful and generous way the firm treats its staff has made Farm2Table the best place I have ever worked in. E) My company offers me good perks and benefits. Answer: A Explanation: The degree to which an employee identifies with a particular organization and its goals and wishes to maintain membership in the organization is referred to as organizational commitment. LO: 3.3: Compare the major job attitudes. AACSB: Analytical thinking Difficulty: Hard Employability Skills: Knowledge Application and Analysis Quest. Category: Application
116 richard@qwconsultancy.com
51) The degree to which employees believe the organization values their contribution and cares about their well-being is known as ________. A) job involvement B) psychological empowerment C) employee engagement D) perceived organizational support E) organizational commitment Answer: D Explanation: Perceived organizational support (POS) is the degree to which employees believe the organization values their contribution and cares about their well-being. Research shows that people perceive their organization as supportive when rewards are deemed fair, when employees have a voice in decisions, and when they see their supervisors as supportive. LO: 3.3: Compare the major job attitudes. Difficulty: Easy Quest. Category: Concept 52) If an organization wants to understand the degree of perceived organizational support among employees, which question should it ask? A) Do you believe that employee engagement has a large impact on productivity? B) Do you believe there is sufficient employee engagement in your department? C) Do your beliefs align with the organization's vision and long-term goals? D) Do you feel the long-term goals on sustainable work practices are workable? E) Do you feel the organization has sufficient recognition rewards to value good work? Answer: E Explanation: Perceived organizational support (POS) is the degree to which employees believe the organization values their contribution and cares about their well-being. Research shows that people perceive their organization as supportive when rewards are deemed fair, when employees have a voice in decisions, and when they see their supervisors as supportive. LO: 3.3: Compare the major job attitudes. Difficulty: Moderate Quest. Category: Concept
117 richard@qwconsultancy.com
53) As a war zone journalist, Harry von der Hoeven often works for weeks at a time without a day off. Harry finds his job to be stimulating and doesn't mind the long work hours because he knows that his employer will allow him to take a month or two off to recoup following an assignment. Which of the following best represents Harry's feeling? A) Low employee engagement B) High perceived organizational support C) Low organizational commitment D) Low job involvement E) High interest in whistleblowing Answer: B Explanation: Perceived organizational support (POS) is the degree to which employees believe the organization values their contribution and cares about their well-being. LO: 3.3: Compare the major job attitudes. AACSB: Analytical thinking Difficulty: Moderate Employability Skills: Knowledge Application and Analysis Quest. Category: Application 54) Employees are most likely to perceive their organization as supportive when ________. A) they experience a cognitive dissonance between their job attitude and behavior B) the affective component of the attitude is extremely strong C) there is a weak relationship between their attitude and behavior D) they are treated fairly by other organization members E) they experience an emotional contagion while performing their job duties Answer: D Explanation: People perceive their organization as supportive when they are treated fairly by other organization members, have a high-quality relationship with the organization, and perceive the organization's practices to be supportive. LO: 3.3: Compare the major job attitudes. Difficulty: Moderate Quest. Category: Concept
118 richard@qwconsultancy.com
55) When Sarah Raines found herself homeless following a devastating house fire, her boss told her to take the time she needed to find temporary housing and help her children adjust to their new situation. Sarah ended up taking nearly two months off work and while she managed to complete a project she was working on, she missed the deadline by a month. Even with the huge amount of stress in her life, Sarah never worried that her job was in jeopardy because of her absence or late completion of her work. Which job attitude best characterizes this example? A) Psychological empowerment B) Perceived organizational support C) Organizational commitment D) Employee engagement E) Job involvement Answer: B Explanation: Sarah believed that her organization would be supportive through her time of stress. This major job attitude is called perceived organizational support. The belief and sense that a person has an effect in the work environment is psychological empowerment. Organizational commitment, the degree to which a person identifies with the organization, and job involvement, the degree to which a person identifies with the job and participates in it, are both considered major job attitudes, as is employee engagement, a person's involvement with the job, but these were not what calmed Sarah's fears. LO: 3.3: Compare the major job attitudes. AACSB: Analytical thinking Difficulty: Hard Employability Skills: Knowledge Application and Analysis Quest. Category: Application 56) Bryan Eusebius has a positive attitude toward his organization. He feels the management treats all employees fairly in matters concerning rewards, is understanding toward their needs and requirements, and allows them to have a voice in decisions. Bryan's attitude toward his organization is indicative of ________. A) emotional contagion B) positivity offset C) perceived organizational support D) cognitive dissonance E) self-concordance Answer: C Explanation: Perceived organizational support (POS) is the degree to which employees believe the organization values their contribution and cares about their well-being. Research shows that people perceive their organization as supportive when rewards are deemed fair, when employees have a voice in decisions, and when they see their supervisors as supportive. LO: 3.3: Compare the major job attitudes. AACSB: Analytical thinking Difficulty: Hard Employability Skills: Knowledge Application and Analysis Quest. Category: Application
119 richard@qwconsultancy.com
57) ________ refers to the degree of enthusiasm an employee feels for the job. A) Employee engagement B) Cognitive dissonance C) Emotional contagion D) Positivity offset E) Self-concordance Answer: A Explanation: Employee engagement refers to the degree of enthusiasm an employee feels for the job. Highly engaged employees have a passion for their work and feel a deep connection to their company whereas disengaged employees put time, but not energy or attention, into their work. LO: 3.3: Compare the major job attitudes. Difficulty: Easy Quest. Category: Concept 58) Which of the following is most likely to be a characteristic of disengaged employees? A) They feel a deep connection for their organization. B) They have a sense of passion for their work. C) They have the lowest levels of turnover. D) They put in time but give no attention to their work. E) They proactively take up responsibilities. Answer: D Explanation: Employee engagement refers to the degree of enthusiasm an employee feels for the job. Highly engaged employees have a passion for their work and feel a deep connection to their company. On the other hand, disengaged employees put time, but not energy or attention, into their work. LO: 3.3: Compare the major job attitudes. Difficulty: Moderate Quest. Category: Concept 59) Marcus Green has been asked to put together a survey for his company's employees asking about their engagement levels in the organization. Which question should Marcus be sure to include in his survey? A) Do you believe in the organization's five-year plan? B) Do you believe the organization is concerned about your well-being? C) Do you enjoy your everyday tasks and achieving project milestones? D) Do you feel the organization values your work? E) Do you agree with the organization's stand on sustainable work practices? Answer: C Explanation: Employee engagement refers to an individual's involvement with, satisfaction with, and enthusiasm for the work he or she does. LO: 3.3: Compare the major job attitudes. Difficulty: Moderate Quest. Category: Concept
120 richard@qwconsultancy.com
60) Which of the following statements is most likely to be true about the major job attitudes? A) They are highly distinct from one another. B) They are generally resistant to change. C) They tend to overlap one another. D) They are not correlated to one another. E) They increase cognitive dissonance. Answer: C Explanation: Evidence suggests that job attitudes are highly related. There is some distinctiveness among them, but they overlap greatly, for various reasons including the employee's personality. LO: 3.3: Compare the major job attitudes. Difficulty: Moderate Quest. Category: Concept 61) Disengaged employees cost organizations money in terms of lost productivity. Answer: TRUE Explanation: Disengaged employees cost organizations money. One study suggests that organizations can lose up to $550 billion annually in lost productivity. LO: 3.3: Compare the major job attitudes. Difficulty: Moderate Quest. Category: Concept 62) Job involvement measures the degree to which people identify psychologically with the organization's mission and vision. Answer: FALSE Explanation: Job involvement is the degree to which a person identifies with a job, actively participates in it, and considers performance important to self-worth. LO: 3.3: Compare the major job attitudes. Difficulty: Easy Quest. Category: Concept 63) Organizational identification refers to the extent to which employees define themselves by the same characteristics that define their organization. Answer: TRUE Explanation: Organizational identification refers to the extent to which employees define themselves by the same characteristics that define their organization. LO: 3.3: Compare the major job attitudes. Difficulty: Easy Quest. Category: Concept
121 richard@qwconsultancy.com
64) Employees' belief in the degree to which they affect their work environment, competence, meaningfulness of their job, and autonomy in their work is known as job involvement. Answer: FALSE Explanation: Employees' belief in the degree to which they affect their work environment, competence, meaningfulness of their job, and autonomy in their work is known as psychological empowerment. LO: 3.3: Compare the major job attitudes. Difficulty: Easy Quest. Category: Concept 65) Employee engagement refers to the degree of enthusiasm an employee feels for the job. Answer: TRUE Explanation: Employee engagement refers to the degree of enthusiasm an employee feels for the job. LO: 3.3: Compare the major job attitudes. Difficulty: Easy Quest. Category: Concept 66) As compared to permanent and self-employed individuals, temporary agency workers tend to be more committed to their organizations, their occupation, and their form of employment. Answer: FALSE Explanation: As compared to permanent and self-employed individuals, temporary agency workers tend to be less committed to their organizations, their occupation, and their form of employment. LO: 3.3: Compare the major job attitudes. Difficulty: Moderate Quest. Category: Concept 67) Explain the concept of power distance and the difference between high power distance and low power distance. Answer: Power distance is the degree to which people in a country accept that power in institutions and organizations is distributed unequally, is lower. In low power distance countries like the United States, people are more likely to view work as an exchange than as a moral obligation, so employees look for reasons to be supported by their organizations. In high power distance countries like China, employee POS perceptions are not as deeply based on demonstrations of fairness, support, and encouragement. LO: 3.3: Compare the major job attitudes. Difficulty: Moderate Quest. Category: Synthesis
122 richard@qwconsultancy.com
68) Compare and contrast job satisfaction, job involvement, and organizational commitment. Answer: Job satisfaction refers to an individual's general attitude toward his/her job. A person with a high level of job satisfaction holds positive attitudes toward the job, while a person who is dissatisfied with his/her job holds negative attitudes about the job. Job involvement, on the other hand, measures the degree to which a person identifies psychologically with his/her job and considers his/her perceived performance level important to self-worth. Employees with a high level of job involvement strongly identify with and really care about the kind of work they do. Organizational commitment is defined as a state in which an employee identifies with a particular organization and its goals and wishes to maintain membership in the organization. Thus, high organizational commitment means identifying with one's employing organization. LO: 3.3: Compare the major job attitudes. Difficulty: Moderate Quest. Category: Synthesis 69) An analysis of employee survey results at Brown, Ingersoll, and Smith Investments revealed that a majority of employees were not satisfied with their work. In an effort to understand why employees are unhappy, Tim Scott, the company's vice-president of Human Resources holds a meeting with various department managers. One manager suggests that offering new training opportunities and advanced training could be beneficial. What assumption is the manager making? A) Employees feel that their efforts are not being adequately recognized by the management. B) Employees are highly skilled and value economic rewards over and above opportunities for growth. C) Employees are stifled by the degree of centralized control and bureaucracy within the organization. D) Employees feel that their employer does not value them enough to make investments in them. E) Employees feel excluded from organizational issues that have an influence on them. Answer: D Explanation: Given that the manager suggests providing employees with more opportunities for training, she is most likely assuming that employees feel dissatisfied because their employer is not investing in their growth and development. It would be incorrect to assume that employees are highly skilled and value economic rewards over and above opportunities for growth as this directly conflicts with the manager's suggestion of providing employees with training opportunities. It would also be wrong to assume that employees are feeling excluded from organizational issues, are stifled by the degree of centralized control and bureaucracy and are experiencing a lack of recognition as these are not directly related to training. LO: 3.4: Identify the two approaches for measuring job satisfaction. AACSB: Reflective thinking Difficulty: Hard Quest. Category: Critical Thinking
123 richard@qwconsultancy.com
70) Which of the following statements is true about measuring job satisfaction? A) The single global rating system is not very time consuming. B) The summation of job facets approach takes into account cognitive dissonance experienced by employees. C) The summation of job facets approach prevents managers from zeroing in on problems. D) The summation of job facets approach involves responding to one particular question. E) The single global rating approach is rarely used for measuring job satisfaction. Answer: A Explanation: The single global rating system as well as the summation of job facets are two popular approaches of measuring job satisfaction. The single global rating method is not very time consuming, thus freeing time for other tasks, and the summation of job facets helps managers zero in on problems and deal with them faster and more accurately. LO: 3.4: Identify the two approaches for measuring job satisfaction. Difficulty: Moderate Quest. Category: Concept 71) Which of the following has been shown to typically provide the lowest level of satisfaction? A) Work itself B) Pay C) Promotion D) Supervision E) Coworkers Answer: B Explanation: The facets of job satisfaction levels can vary widely. Research shows that people have typically been more satisfied with their jobs overall, the work itself, and their supervisors and coworkers than they have been with their pay and promotion opportunities. In fact, pay was found to provide the lowest level of job satisfaction. LO: 3.4: Identify the two approaches for measuring job satisfaction. AACSB: Reflective thinking Difficulty: Moderate Quest. Category: Concept 72) The single global rating approach to measuring job satisfaction is more sophisticated than the summation of job facets approach. Answer: FALSE Explanation: The single global rating approach to measuring job satisfaction is less sophisticated than the summation of job facets approach. LO: 3.4: Identify the two approaches for measuring job satisfaction. Difficulty: Easy Quest. Category: Concept
124 richard@qwconsultancy.com
73) Job satisfaction describes a positive feeling about a job, resulting from an evaluation of its characteristics. Answer: TRUE Explanation: When people speak of employee attitudes, they usually mean job satisfaction, which describes a positive feeling about a job resulting from an evaluation of its characteristics. LO: 3.4: Identify the two approaches for measuring job satisfaction. Difficulty: Easy Quest. Category: Concept 74) Asking employees how they feel about key elements in a job and then adding the results to create an overall job satisfaction score is the summation of job facets method to job satisfaction. Answer: TRUE Explanation: The summation of job facets identifies key elements in a job such as the nature of the work, supervision, present pay, promotion opportunities, and relations with co-workers. Respondents rate these on a standardized scale, and researchers add the ratings to create an overall job satisfaction score. LO: 3.4: Identify the two approaches for measuring job satisfaction. Difficulty: Easy Quest. Category: Concept 75) Rachel, a sales executive at a bank, earns $35,000 a year. Her company often allows her to work her schedule around her daughter's school athletic events. She believes in the goals of her organization and works hard toward achieving them. Rachel's husband, Peter, has recently joined an engineering firm as a designer. He earns $75,000 a year. His job involves working long hours from Monday to Friday, and his work is often stressful. But he enjoys his work and is passionate about the projects he works on. Due to Peter's time constraints, his wife, Rachel, attends all their children's school events. How would you compare Rachel and Peter's levels of job satisfaction? Describe how you would measure their satisfaction level if the level of comfortable living in their country is $40,000. In addition, name at least three major job attitudes that play a part in each of their satisfaction levels. Answer: Students' answers may vary. The following is a sample response. Rachel is likely to be less satisfied than Peter in the pure measurement of pay because she makes less than the $40,000 level required for a comfortable living in the United States. However, she is more likely to have a higher degree of perceived organizational support because her company allows her a flexible schedule around her family, whereas Peter's company does not. Rachel shows a high level of organizational commitment because she identifies with the organization's objectives. Peter is most likely to have a high level of job involvement because he feels passionate about and cares for the job he does. His level of job satisfaction can be increased by providing more flexibility whereas Rachel's level of job satisfaction can be increased by increasing her pay. LO: 3.4: Identify the two approaches for measuring job satisfaction. AACSB: Analytical thinking Difficulty: Hard Quest. Category: Synthesis
125 richard@qwconsultancy.com
76) Which of the following is true about the correlation between salary and job satisfaction? A) Pay is not related to job satisfaction for employees in underdeveloped countries. B) Salary and perks do not have a role in overall happiness of employees from poor countries. C) Pay does not play a critical role in job satisfaction when an individual reaches a level of comfortable living. D) Financial perks and benefits always create job satisfaction irrespective of standards of living. E) High salary level always creates organizational commitment. Answer: C Explanation: For people who are poor or who live in poor countries, pay does correlate with job satisfaction and overall happiness. But once an individual reaches a level of comfortable living (in the United States, that occurs at about $40,000 a year, depending on the region and family size), the relationship between pay and job satisfaction virtually disappears. People who earn $80,000 are, on average, no happier with their jobs than those who earn closer to $40,000. LO: 3.5: Summarize the main causes of job satisfaction. Difficulty: Moderate Quest. Category: Concept 77) Charles, Kim, Elle, and Sunil are college friends and work in New York City. Comfortable living in the New York metro area occurs at about $50,000 a year. Charles makes $24,000 a year, Kim makes $30,000 a year, Elle makes $50,000 a year, and Sunil makes $75,000 a year. Which of the following is most likely to be true with reference to correlation between pay and job satisfaction? A) Charles is more satisfied with his job in comparison with Kim. B) Kim is more satisfied with the job than Sunil. C) Charles and Sunil share identical levels of job satisfaction related to pay. D) Elle and Sunil are most likely to have a similar level of job satisfaction. E) Charles and Kim have the same levels of job satisfaction in relation to their pay. Answer: D Explanation: Research shows that once a person earns a comfortable living, the correlation between pay and job satisfaction disappears. Thus, in this scenario, Elle and Sunil are likely to be equally satisfied with their jobs. For those earning under the amount of comfortable living, pay is related to job satisfaction. LO: 3.5: Summarize the main causes of job satisfaction. AACSB: Analytical thinking Difficulty: Hard Employability Skills: Knowledge Application and Analysis Quest. Category: Application
126 richard@qwconsultancy.com
78) Dennis Galvan works as a campaigner at Green Earth, an environmental organization. Every month, his organization arranges a team outing where they play disc golf, soccer, and participate in relays and other team-building activities. The human resource department ensures that there are regular interactions between employees through team dinners and cultural events. According to the information given in this case, which of the following is most likely to be the reason for Galvan's high level of satisfaction toward his job? A) Salary packaging B) Promotion and growth C) Constructive dismissal D) Social interactions E) Profile of work Answer: D Explanation: Generally, interdependence, feedback, social support, and interaction with coworkers outside the workplace are strongly related to job satisfaction. LO: 3.5: Summarize the main causes of job satisfaction. AACSB: Interpersonal relations and teamwork Difficulty: Moderate Employability Skills: Knowledge Application and Analysis Quest. Category: Application 79) Laura Choi recently started her first job out of college. In her new position, Laura is a junior member of a consulting team assigned to work with companies to develop their social media presence. As a new member of the team, Laura has been tasked with some of the more menial aspects of the job but is looking forward to taking on a bigger role in the future. During her first few weeks on the job, Laura has demonstrated positive core self-evaluations. Which of the following did she most likely do? A) Mentioned that she was confident about her basic competence B) Expressed dissatisfaction with the nature of work C) Set less ambiguous goals and objectives D) Gave up easily on tasks when faced with difficulties E) Did not consider her work as challenging and fulfilling Answer: A Explanation: People who have positive core self-evaluations are those who believe in their inner worth and basic competence and are more satisfied with their jobs than those with negative core self-evaluations. Not only do they see their work as more fulfilling and challenging, but they are also more likely to gravitate toward challenging jobs in the first place. LO: 3.5: Summarize the main causes of job satisfaction. Difficulty: Hard Employability Skills: Knowledge Application and Analysis Quest. Category: Application
127 richard@qwconsultancy.com
80) Employees with positive core self-evaluations believe in their inner worth and basic competence and are more satisfied with their jobs than those with negative core self-evaluations. The concept of positive core self-evaluations indicates that ________. A) promotions and growth opportunities influence job satisfaction B) employees should be regularly monitored and provided feedback C) job conditions have a direct influence on job satisfaction D) personality plays a role in job satisfaction E) pay always has a direct correlation with job satisfaction Answer: D Explanation: Job satisfaction is not just about job conditions. Personality also plays a role in it. Research has shown that people who have positive core self-evaluations, who believe in their inner worth and basic competence, are more satisfied with their jobs than those with negative core self-evaluations. LO: 3.5: Summarize the main causes of job satisfaction. Difficulty: Moderate Quest. Category: Concept 81) Job conditions—especially the intrinsic nature of the work itself, social interactions, and supervision—are important predictors of job satisfaction. Answer: TRUE Explanation: Job conditions—especially the intrinsic nature of the work itself, social interactions, and supervision—are important predictors of job satisfaction. Although each is important, and although their relative value will vary across employees, the intrinsic nature of the work is most important. LO: 3.5: Summarize the main causes of job satisfaction. Difficulty: Easy Quest. Category: Concept 82) People who have positive core self-evaluations (CSEs)—who believe in their inner worth and basic competence—are more satisfied with their jobs than people with negative CSEs. Answer: TRUE Explanation: In the context of career commitment, CSE influences job satisfaction as people with high levels of both CSE and career commitment may realize particularly high job satisfaction. LO: 3.5: Summarize the main causes of job satisfaction. Difficulty: Easy Quest. Category: Concept 83) The relationship between pay and job satisfaction virtually disappears when one earns a pay package sufficient for or more than the comfortable living amount. Answer: TRUE Explanation: When an individual reaches a level of comfortable living, the relationship between pay and job satisfaction virtually disappears. LO: 3.5: Summarize the main causes of job satisfaction. Difficulty: Easy Quest. Category: Concept 128 richard@qwconsultancy.com
84) Identify the major causes of job satisfaction and explain how they contribute to increased job satisfaction. Answer: The major job satisfaction facets are the work itself, pay, advancement opportunities, supervision, and co-workers. Enjoying the work itself is almost always the facet most strongly correlated with high levels of overall job satisfaction. Most people prefer work that is challenging and stimulating over work that is predictable and routine. For people who are poor or who live in poor countries, pay does correlate with job satisfaction and with overall happiness. Once an individual reaches a level of comfortable living, however, the relationship virtually disappears. Personality also plays a role in job satisfaction. Research has shown that people who have positive core self-evaluations, who believe in their inner worth and basic competence, are more satisfied with their jobs than those with negative core self-evaluations. Not only do they see their work as more fulfilling and challenging, but they are also more likely to gravitate toward challenging jobs in the first place. Those with negative core self-evaluations set less ambitious goals and are more likely to give up when confronting difficulties. LO: 3.5: Summarize the main causes of job satisfaction. Difficulty: Moderate Quest. Category: Concept 85) Which of the following is moderately correlated with organizational citizenship behavior? A) Emotional contagion B) Job satisfaction C) Absenteeism D) Turnover rate E) Cognitive dissonance Answer: B Explanation: Job satisfaction is moderately correlated with organizational citizenship behavior. LO: 3.6: Identify four outcomes of job satisfaction. Difficulty: Easy Quest. Category: Concept 86) Job satisfaction leads to employee's organizational citizenship behavior (OCB). Answer: TRUE Explanation: Job satisfaction leads to employee's organizational citizenship behavior (OCB). One reason is trust. Research in 18 countries suggests that managers reciprocate employees' OCB with trusting behaviors. Individuals who feel their coworkers support them are also more likely to engage in helpful behaviors than those who have antagonistic coworker relationships. LO: 3.6: Identify four outcomes of job satisfaction. Difficulty: Easy Quest. Category: Concept 87) What is the relationship between job satisfaction and life satisfaction? Answer: Life satisfaction and job satisfaction are related. Research shows that life satisfaction decreases when people lose their jobs. For most people, work is an integral part of life, and many people derive meaning from the roles they fulfill. LO: 3.6: Identify four outcomes of job satisfaction. 129 richard@qwconsultancy.com
Difficulty: Moderate Quest. Category: Concept 88) Substance abuse, stealing at work, endless scrolling on social media while on the clock, tardiness, gossip, and absenteeism are all examples of ________. A) employee neglect B) counterproductive work behavior C) reduced productivity D) employee engagement E) criminal activity Answer: B Explanation: Actions that actively damage the organization, including stealing, behaving aggressively toward coworkers, or being late or absent are counterproductive work behavior. LO: 3.7: Identify four employee responses to job dissatisfaction. Difficulty: Easy Quest. Category: Concept 89) ________ can be understood using the exit-voice-loyalty-neglect framework. A) Increasing growth opportunities B) Emotional contagion C) Dissatisfaction D) Regular feedback E) High employee engagement Answer: C Explanation: The theoretical model of the exit-voice-loyalty-neglect framework is helpful in understanding the consequences of dissatisfaction. LO: 3.7: Identify four employee responses to job dissatisfaction. Difficulty: Easy Quest. Category: Concept 90) Which response includes actively and constructively attempting to improve conditions, including suggesting improvements, discussing problems with superiors, and undertaking some forms of union activity? A) Exit B) Voice C) Loyalty D) Neglect E) Acceptance Answer: B Explanation: The voice response includes actively and constructively attempting to improve conditions, including suggesting improvements, discussing problems with superiors, and undertaking some forms of union activity. LO: 3.7: Identify four employee responses to job dissatisfaction. Difficulty: Easy Quest. Category: Concept
130 richard@qwconsultancy.com
131 richard@qwconsultancy.com
91) For the past several months, Paulina Grabowski has been unhappy in her job. Her friends have suggested that it's time to move to a new company, but Paulina responds that the company is a good one and that she is sure that things will improve in time. Paulina's response to this situation can best be referred to as ________. A) exit B) voice C) loyalty D) neglect E) ratification Answer: C Explanation: The loyalty response means passively but optimistically waiting for conditions to improve, including speaking up for the organization in the face of external criticism and trusting the organization and its management to "do the right thing." LO: 3.7: Identify four employee responses to job dissatisfaction. AACSB: Analytical thinking Difficulty: Moderate Employability Skills: Knowledge Application and Analysis Quest. Category: Application 92) Keita Kimathi is disappointed with the lack of recognition she has received for her work on three large projects for her company. Keita feels that the extra hours she has put in on the projects have been overlooked and further that there is now a general expectation that she will work longer hours and even put in some time on weekends. Keita has scheduled a meeting with her boss to discuss the situation and possible solutions. Keita's response to the problem is referred to as ________. A) voice B) loyalty C) neglect D) exit E) acceptance Answer: A Explanation: According to the exit-voice-loyalty-neglect framework, the voice response includes actively and constructively attempting to improve conditions, including suggesting improvements, discussing problems with superiors, and undertaking some forms of union activity. LO: 3.7: Identify four employee responses to job dissatisfaction. AACSB: Analytical thinking Difficulty: Moderate Employability Skills: Knowledge Application and Analysis Quest. Category: Application
132 richard@qwconsultancy.com
93) Daniel Sherris, an accountant for a large consulting company, has reached his limit. Daniel is tired of being asked to work 60 hours plus every week and the sarcastic response he gets when he asks for time off. Daniel has already started looking for a new job and plans to quit his current one as soon as possible. Daniel's actions reflect the ________ response. A) acceptance B) neglect C) exit D) voice E) loyalty Answer: C Explanation: The exit response, according to the exit-voice-loyalty-neglect framework, directs behavior toward leaving the organization, including looking for a new position as well as resigning. LO: 3.7: Identify four employee responses to job dissatisfaction. AACSB: Analytical thinking Difficulty: Moderate Employability Skills: Knowledge Application and Analysis Quest. Category: Application 94) Dante Smith has just returned from another long weekend. Dante has been burning through his vacation days even though its just two months into the new year. Dante finds it hard to go to a job where he is unhappy and sometimes just calls in sick or arrives to work late. Dante is expressing his dissatisfaction though the ________ response A) acceptance B) neglect C) voice D) loyalty E) exit Answer: B Explanation: According to the exit-voice-loyalty-neglect framework, the neglect response passively allows conditions to worsen and includes chronic absenteeism or lateness, reduced effort, and increased error rate. LO: 3.7: Identify four employee responses to job dissatisfaction. AACSB: Analytical thinking Difficulty: Moderate Employability Skills: Knowledge Application and Analysis Quest. Category: Application
133 richard@qwconsultancy.com
95) Joe Dailey is unhappy with his job because he has not received the promotion due to him several years ago. However, in spite of this, he speaks up to support his company's actions even when the local newspaper is criticizing them. Which of the following types of response represents his behavior? A) Exit B) Voice C) Loyalty D) Neglect E) Acceptance Answer: C Explanation: According to the exit-voice-loyalty-neglect framework, the loyalty response means passively but optimistically waiting for conditions to improve. LO: 3.7: Identify four employee responses to job dissatisfaction. AACSB: Analytical thinking Difficulty: Moderate Employability Skills: Knowledge Application and Analysis Quest. Category: Application 96) Sarah Mayer works as a marketing executive and has been unhappy with her job description for several months now. Over the months, she has regularly discussed with her manager how her skills lie in administrative tasks instead of in marketing. In addition, she discusses how she can make a smooth transition into the administrative role. Which type of response is Mayer using in this situation? A) Exit B) Voice C) Loyalty D) Neglect E) Performance Answer: B Explanation: According to the exit-voice-loyalty-neglect framework, the voice response includes actively and constructively attempting to improve conditions, including suggesting improvements, discussing problems with superiors, and undertaking some forms of union activity. LO: 3.7: Identify four employee responses to job dissatisfaction. AACSB: Analytical thinking Difficulty: Moderate Employability Skills: Knowledge Application and Analysis Quest. Category: Application
134 richard@qwconsultancy.com
97) Which type of response best explains attending union meetings as a means of coping with job dissatisfaction? A) Exit B) Voice C) Loyalty D) Neglect E) Ratification Answer: B Explanation: According to the exit-voice-loyalty-neglect framework, the voice response includes actively and constructively attempting to improve conditions, including suggesting improvements, discussing problems with superiors, and undertaking some forms of union activity. LO: 3.7: Identify four employee responses to job dissatisfaction. Difficulty: Moderate Quest. Category: Concept 98) Sylvia's company has been hit hard during the pandemic. Her boss has told her that she will only get about half the hours she is accustomed to and that she might need to take a cut in pay to help get her company through the difficult time. Even though Sylvia is upset with the situation, she continues to arrive at work each morning with a smile on her face, determined to do her best. Sylvia's response can best be described as ________. A) acceptance B) voice C) neglect D) exit E) loyalty Answer: E Explanation: Sylvia demonstrates loyalty to the company and passively waits for things to improve, which is a constructive response. Sylvia is not voicing her negative feelings about less work, continues to show up for work, and does not neglect her duties. She is passive, rather than active, in her behavior. LO: 3.7: Identify four employee responses to job dissatisfaction. AACSB: Analytical thinking Difficulty: Hard Employability Skills: Knowledge Application and Analysis Quest. Category: Application
135 richard@qwconsultancy.com
99) ________ is a type of response to dissatisfaction that is constructive and passive. A) Loyalty B) Neglect C) Voice D) Reflect E) Exit Answer: A Explanation: Loyalty is considered a passive constructive response to dissatisfaction. LO: 3.7: Identify four employee responses to job dissatisfaction. Difficulty: Easy Quest. Category: Concept 100) Which behaviors in the exit-voice-loyalty-neglect framework are associated with productivity, absenteeism, and turnover? A) Voice and neglect B) Neglect and loyalty C) Voice and exit D) Loyalty and voice E) Exit and neglect Answer: E Explanation: In the exit-voice-loyalty-neglect framework, the exit response directs behavior toward leaving the organization, including looking for a new position as well as resigning. The neglect response passively allows conditions to worsen and includes chronic absenteeism or lateness, reduced effort, and increased error rate. Exit and neglect behaviors encompass the performance variables—productivity, absenteeism, and turnover. LO: 3.7: Identify four employee responses to job dissatisfaction. Difficulty: Moderate Quest. Category: Concept 101) The exit-voice-loyalty-neglect framework expands employee response to include voice and loyalty-constructive behaviors that allow individuals to ________. A) revolt against the management B) reject unionization and focus on team-building activities C) identify psychologically with their jobs D) tolerate unpleasant situations or revive satisfactory working conditions E) decrease emotional contagion Answer: D Explanation: The exit-voice-loyalty-neglect framework expands employee response to include voice and loyalty-constructive behaviors that allow individuals to tolerate unpleasant situations or revive satisfactory working conditions. LO: 3.7: Identify four employee responses to job dissatisfaction. Difficulty: Moderate Quest. Category: Concept
136 richard@qwconsultancy.com
102) Job dissatisfaction is more likely to translate into ________ when employees feel or perceive they have many available alternatives and when employees have high human capital. A) high productivity B) employee engagement C) increased customer satisfaction D) turnover E) organizational citizenship behavior Answer: D Explanation: Job dissatisfaction is more likely to translate into turnover when employment opportunities are plentiful because employees perceive it is easy to move. When employees have high human capital—i.e., high education, high ability—job dissatisfaction is more likely to translate into turnover because they have, or perceive, many available alternatives. LO: 3.7: Identify four employee responses to job dissatisfaction. Difficulty: Easy Quest. Category: Concept 103) Under which of the following conditions is job dissatisfaction most likely to result in turnover? A) Employees have negative core self-evaluation. B) Employees have high education and ability. C) Employment opportunities are scarce. D) Employees experience emotional contagion. E) Employees have supportive relationships with their co-workers. Answer: B Explanation: Job dissatisfaction is more likely to translate into turnover when employment opportunities are plentiful because employees perceive it is easy to move. In addition, when employees have high human capital—i.e., high education, high ability—job dissatisfaction is more likely to translate into turnover because they have, or perceive, many available alternatives. LO: 3.7: Identify four employee responses to job dissatisfaction. Difficulty: Easy Quest. Category: Concept
137 richard@qwconsultancy.com
104) Job dissatisfaction and antagonistic relationships with co-workers predict a variety of behaviors organizations find undesirable, including unionization attempts, substance abuse, undue socializing, and tardiness. These behaviors are indicators of a broader syndrome called ________. A) employee withdrawal B) organizational commitment C) job involvement D) cognitive dissonance E) positivity offset Answer: A Explanation: Job dissatisfaction and antagonistic relationships with co-workers predict a variety of behaviors organizations find undesirable, including unionization attempts, substance abuse, stealing at work, undue socializing, and tardiness. Researchers argue these behaviors are indicators of a broader syndrome called deviant behavior in the workplace (or counterproductive behavior or employee withdrawal). LO: 3.7: Identify four employee responses to job dissatisfaction. Difficulty: Moderate Quest. Category: Concept 105) Simon's garage is full of office supplies he has taken from his workplace. Simon feels justified in his actions because he believes his boss is unfairly asking him to work harder than what his salary and position warrant. Simon views his collection of office supplies as compensation for the extra work he puts in. Simon's behavior is an example of ________. A) employee OCB B) customer satisfaction C) high productivity D) turnover rate E) counterproductive work behavior Answer: E Explanation: Job dissatisfaction and antagonistic relationships with co-workers predict a variety of behaviors organizations find undesirable, including unionization attempts, substance abuse, stealing at work, undue socializing, and tardiness. Researchers argue these behaviors are indicators of a broader syndrome called counterproductive work behavior in the workplace (or counterproductive behavior or employee withdrawal). LO: 3.7: Identify four employee responses to job dissatisfaction. AACSB: Analytical thinking Difficulty: Moderate Employability Skills: Knowledge Application and Analysis Quest. Category: Application
138 richard@qwconsultancy.com
106) Brian is often late to work and then typically spends the first hour of his workday scrolling through his social media accounts before starting his work. After a couple of hours, Brian takes a long lunch and sometimes returns to work intoxicated. Brian's actions are an example of ________. A) citizenship B) counterproductive work behavior C) organizational commitment D) positive affect E) satisficing Answer: B Explanation: Job dissatisfaction and antagonistic relationships with co-workers predict a variety of behaviors organizations find undesirable, including substance abuse, stealing at work, undue socializing, and tardiness. Researchers argue these behaviors are indicators of a broader syndrome called counterproductive work behavior (or deviant behavior in the workplace or employee withdrawal). LO: 3.7: Identify four employee responses to job dissatisfaction. AACSB: Analytical thinking Difficulty: Moderate Employability Skills: Knowledge Application and Analysis Quest. Category: Application 107) Synergy Inc. is a medium-sized logistics company. The management is facing tough times as the workers are dissatisfied and are engaging in a number of deviant workplace behaviors such as stealing and substance abuse during working hours. The management is considering various options to curb these counterproductive behaviors. What would be a better way to deal with such forms of workplace deviance? A) The management should let employees realize their wrongdoing on their own. B) The management should attack the source of the problem, i.e., the dissatisfaction. C) The management must introduce surveillance to ensure smooth flow of work. D) The management must suspend employees who engage in deviant workplace behaviors. E) The management must restrict the activities that adversely affect productivity. Answer: B Explanation: Workers who do not like their jobs "get even" in various ways. Since those ways can be quite creative, controlling only one behavior, such as introduction of an absence control policy, leaves the root cause untouched. To effectively control the undesirable consequences of job dissatisfaction, employers should attack the source of the problem, i.e., the dissatisfaction rather than try to control the different responses. LO: 3.7: Identify four employee responses to job dissatisfaction. AACSB: Analytical thinking Difficulty: Moderate Employability Skills: Knowledge Application and Analysis Quest. Category: Application
139 richard@qwconsultancy.com
108) Why should managers be interested in their employees' attitudes? A) They result from behavior. B) They lead to self-concordance. C) They give warnings of potential problems. D) They result in emotional contagion. E) They result in cognitive dissonance. Answer: C Explanation: Managers should be interested in their employees' attitudes because attitudes give warnings of potential problems and influence behavior, resulting in higher or lower profits and productivity. Some researchers believe that attitudes follow behavior, not the other way around. Although negative attitudes can cause problems, there are many positive reasons managers should look at job attitudes. Job satisfaction and attitudes in the United States are generally high. Cognitive dissonance is the result of a disparity between an attitude and a behavior. LO: 3.7: Identify four employee responses to job dissatisfaction. Difficulty: Moderate Quest. Category: Concept 109) If a manager wants to raise employee satisfaction, the manager should ________. A) reevaluate employee pay B) increase benefits C) reduce work hours D) focus on the intrinsic parts of the job E) increase employee productivity Answer: D Explanation: The most important thing managers can do to raise employee satisfaction is focus on the intrinsic parts of the job, such as making the work challenging and interesting. Although paying employees poorly will likely not attract high-quality employees to the organization or keep high performers, managers should realize that high pay alone is unlikely to create a satisfying work environment. LO: 3.7: Identify four employee responses to job dissatisfaction. Difficulty: Easy Quest. Category: Concept 110) A satisfied workforce does not guarantee successful organizational performance. In order to improve organizational effectiveness, managers ________. A) must use 360-degrees appraisal procedures B) must provide classroom training C) must make jobs easier and targets achievable D) must offer periodic salary increments E) must try to improve employee attitudes Answer: E Explanation: Creating a satisfied work force is hardly a guarantee of successful organizational performance, but evidence strongly suggests that whatever managers can do to improve employee attitudes will likely result in heightened organizational effectiveness. LO: 3.7: Identify four employee responses to job dissatisfaction. Difficulty: Easy 140 richard@qwconsultancy.com
Quest. Category: Concept 111) Actions that actively damage the organization including stealing, behaving aggressively toward coworkers, or being late or absent are referred to as counterproductive work behavior. Answer: TRUE Explanation: Counterproductive work behavior refers to actions that actively damage the organization including stealing, behaving aggressively toward coworkers, or being late or absent. LO: 3.7: Identify four employee responses to job dissatisfaction. Difficulty: Easy Quest. Category: Concept 112) Passively but optimistically waiting for conditions to improve and trusting management "to do the right thing" are part of the loyalty construct. Answer: TRUE Explanation: The loyalty response means passively but optimistically waiting for conditions to improve, including speaking up for the organization in the face of external criticism and trusting the organization and its management to do the right thing. LO: 3.7: Identify four employee responses to job dissatisfaction. Difficulty: Easy Quest. Category: Concept 113) "Neglect" is an active and constructive response to dissatisfaction. Answer: FALSE Explanation: The neglect response to dissatisfaction passively allows conditions to worsen. LO: 3.7: Identify four employee responses to job dissatisfaction. Difficulty: Easy Quest. Category: Concept 114) To effectively control the undesirable consequences of job dissatisfaction, employers should try to control the different responses to dissatisfaction. Answer: FALSE Explanation: To effectively control the undesirable consequences of job dissatisfaction, employers should attack the source of the problem—the dissatisfaction—rather than try to control the different responses. LO: 3.7: Identify four employee responses to job dissatisfaction. Difficulty: Easy Quest. Category: Concept 115) Creating a satisfied workforce guarantees successful organizational performance. Answer: FALSE Explanation: Creating a satisfied workforce is hardly a guarantee of successful organizational performance, but evidence strongly suggests that whatever managers can do to improve employee attitudes will likely result in heightened organizational effectiveness. LO: 3.7: Identify four employee responses to job dissatisfaction. Difficulty: Easy Quest. Category: Concept 141 richard@qwconsultancy.com
142 richard@qwconsultancy.com
116) Kim has had a rough week. First, she was passed over for a promotion at work that she thought was almost certain, then her debit card was declined when she went to buy coffee. Kim's current salary is $36,000 a year. Between the low salary and lack of promotion, Kim is feeling very dissatisfied with her work situation. Using examples, provide two active responses that Kim could take to express her dissatisfaction to her boss. Then, develop a strategy for Kim's supervisor that will entice Kim to stay with the company. Answer: Students' answers may vary. The following is a sample response. Kim's active responses could be to voice her dissatisfaction to her supervisor, lobbying for her promotion or seeking reasons why she was not promoted. Her other active option would be to exit the company, beginning immediately to look for another job. If the immediate supervisor is aware of Kim's dissatisfaction and values her as an employee, she needs to proactively discuss why Sally was passed over for the promotion. She could design a training program or a longer-term plan of how Kim can reach her income and responsibility goals with small measurable steps that would receive rewards, such as lunch gift cards or time off, in place of the income that Kim feels she is denied. LO: 3.7: Identify four employee responses to job dissatisfaction. AACSB: Analytical thinking Difficulty: Hard Employability Skills: Knowledge Application and Analysis Quest. Category: Application 117) Marcus just learned that his request for time off over the upcoming holiday has not been approved. Marcus knows that he can't afford to lose his job. How can Marcus express his dissatisfaction with the situation to his supervisor? Describe three negative, passive responses that Marcus could take. Then, suppose that Marcus' supervisor catches him manifesting counterproductive work behavior. What is the likely outcome of Marcus/ behavior? Answer: Students' answers may vary. The following is a sample response. Since Marcus cannot quit his job, his responses will be to passively neglect his work. He could manifest this in a number of ways, from increased absenteeism to workplace deviance, such as surfing the Internet during work hours or stealing work supplies. If Marcus' manager was to catch him surfing the Internet, he/she would most likely reprimand Marcus, increasing his level of dissatisfaction and level of job neglect. A better solution would be to question Marcus regarding the reasons behind his job dissatisfaction and open up lines of communication which would allow him a chance to actively and positively voice her his grievance. LO: 3.7: Identify four employee responses to job dissatisfaction. AACSB: Analytical thinking Difficulty: Moderate Employability Skills: Knowledge Application and Analysis Quest. Category: Application
143 richard@qwconsultancy.com
118) What are the four ways that employees can express job dissatisfaction and how do they differ? Answer: Employees can express dissatisfaction by four different ways. These are exit, voice, loyalty, and neglect. These four responses differ along two dimensions: constructive/destructive and active/passive. Behavior directed toward leaving the organization is defined as exit behavior. It is active and destructive. On the other hand, voice involves actively and constructively attempting to improve conditions, including suggesting improvements, discussing problems with superiors, and engaging in some forms of union activity. In contrast, loyalty involves passively but optimistically waiting for conditions to improve. It is passive and constructive. Neglect is passively allowing conditions to worsen, including chronic absenteeism or lateness, reduced effort, and increased error rate. Hence, it is a passive and destructive response. LO: 3.7: Identify four employee responses to job dissatisfaction. Difficulty: Moderate Quest. Category: Synthesis 119) Is employee satisfaction related to positive customer outcomes? Answer: The evidence indicates that satisfied employees increase customer satisfaction and loyalty. In service organizations, customer retention and defection are highly dependent on how front-line employees deal with customers. Satisfied employees are more likely to be friendly, upbeat, and responsive, which customers appreciate. Since satisfied employees are less prone to turnover, customers are also more likely to encounter familiar faces and receive experienced service. These qualities build customer satisfaction and loyalty. LO: 3.7: Identify four employee responses to job dissatisfaction. Difficulty: Moderate Quest. Category: Concept 120) Explain the relationship between job satisfaction and turnover. Answer: The relationship between job satisfaction and turnover is stronger than between satisfaction and absenteeism. The satisfaction-turnover relationship also is affected by alternative job prospects. For instance, if an employee is presented with an unsolicited job offer, job dissatisfaction is less predictive of turnover because the employee is more likely leaving in response to "pull" (the lure of the other job) than "push" (the unattractiveness of the current job). Similarly, job dissatisfaction is more likely to translate into turnover when employment opportunities are plentiful because employees perceive it is easy to move. Finally, when employees have high "human capital" (high education, high ability), job dissatisfaction is more likely to translate into turnover because they have, or perceive, many available alternatives. LO: 3.7: Identify four employee responses to job dissatisfaction. Difficulty: Moderate Quest. Category: Concept Organizational Behavior, 19e (Robbins/Judge) Chapter 4 Emotions and Moods 1) ________ is a generic term covering a broad range of feelings, including emotions and moods. A) Cognition B) Affect C) Knowledge 144 richard@qwconsultancy.com
D) Insight E) Attention Answer: B Explanation: Affect is a generic term covering a broad range of feelings, including emotions and moods. Emotions are intense discrete and short-lived often caused by a specific event. Moods are less intense feelings than emotions. LO: 4.1: Differentiate between emotions and moods. Difficulty: Easy Quest. Category: Concept 2) Which of the following is a characteristic of emotions? A) Emotions are short-lived. B) Emotions last for a longer time period than moods. C) Emotions are never action-oriented in nature. D) Emotions lack a contextual stimulus. E) Emotions involve less intense feelings than moods. Answer: A Explanation: Emotions are intense, discrete and short-lived, often caused by a specific event. Emotions are reactions to a person. For instance, seeing a friend at work may make one feel glad or an event dealing with a rude client may make one feel angry. LO: 4.1: Differentiate between emotions and moods. Difficulty: Moderate Quest. Category: Concept 3) ________ describe an intense, discrete and short-lived feeling experience caused by a specific event. A) Perception B) Cognition C) Mood D) Reasoning E) Emotions Answer: E Explanation: Emotions are intense, discrete and short-lived, often caused by a specific event. Emotions are reactions to a person. For instance, seeing a friend at work may make one feel glad or an event dealing with a rude client may make one feel angry. LO: 4.1: Differentiate between emotions and moods. Difficulty: Easy Quest. Category: Concept
145 richard@qwconsultancy.com
4) Michelle Anouk works at the drive through window of her local coffee house. Michelle usually enjoys interacting with customers in the morning, but last week, Michelle stormed out of the coffee house and angerly walked around the back. Michelle's coworker told her boss that the previous customer had yelled at Michelle for being too slow with his order. Michelle's anger can best be described as ________. A) a perception B) an emotion C) a recognition D) an insight E) a mood Answer: B Explanation: Michelle is experiencing an intense feeling of anger, which is an emotion directed toward the unpleasant customer. Emotions are intense, discrete and short-lived, often caused by a specific event. LO: 4.1: Differentiate between emotions and moods. AACSB: Analytical thinking Difficulty: Moderate Employability Skills: Knowledge Application and Analysis Quest. Category: Application 5) Which of the following refers to feelings that tend to be less intense than emotions and that lack a contextual stimulus? A) Affect B) Cognition C) Mood D) Thought E) Reaction Answer: C Explanation: Moods are feelings that tend to be less intense than emotions and that lack a contextual stimulus. LO: 4.1: Differentiate between emotions and moods. Difficulty: Easy Quest. Category: Concept
146 richard@qwconsultancy.com
6) Robert Springer woke up in the morning and felt a sense of joy and peace as he got ready for work. At work, though challenges came his way, he tackled them without getting stressed about them. This feeling lasted for several days that week. What is one of the reasons that Springer's feeling can be categorized as a mood and not as an emotion? A) It is more intense than emotions. B) It is brought about by a specific event. C) It is prolonged in nature. D) It is a positive feeling. E) It is indicated by a facial expression. Answer: C Explanation: Springer's general ambiguous feeling of happiness is ongoing or prolonged, which is a characteristic of a mood. Contextual stimulus, strong feelings, and being directed at an event or person are characteristics of an emotion. In addition, both mood and emotion can be positive. LO: 4.1: Differentiate between emotions and moods. AACSB: Analytical thinking Difficulty: Hard Employability Skills: Knowledge Application and Analysis Quest. Category: Application 7) When Ariana Winter is told that her dog has been killed by a car, she feels deeply saddened. Her feelings are then overcome by a surge of anger at the car driver. However, minutes later, she is able to laugh and talk about the good memories spent with her pet. What is one of the reasons that Winter's reaction can be categorized as an emotion? A) Its cause is general and ambiguous. B) It lasted for a brief time period. C) It is not action oriented. D) It is a negative feeling. E) It is cognitive in nature. Answer: B Explanation: Emotions are intense, discrete and short-lived; often caused by a specific event. Most experts believe emotions are more fleeting than moods. Emotions tend to come and go fairly quickly, maybe even in a matter of seconds. LO: 4.1: Differentiate between emotions and moods. AACSB: Analytical thinking Difficulty: Hard Employability Skills: Knowledge Application and Analysis Quest. Category: Application
147 richard@qwconsultancy.com
8) Which of the following statements is most likely to be true regarding moods? A) Moods are always brought about by a specific event. B) Moods last for a very short period of time, such as a few seconds. C) As compared to emotions, moods are more likely to be clearly revealed by facial expressions. D) Moods are more fleeting than emotions. E) Moods are less intense feelings than emotions. Answer: E Explanation: Moods are less intense feelings than emotions, and their cause is often general and unclear. They are generally not indicated by distinct facial expressions. Unlike moods, emotions like anger and disgust tend to be more clearly revealed by facial expressions. Emotions are more likely to be caused by a specific event, and emotions are more fleeting than moods. LO: 4.1: Differentiate between emotions and moods. Difficulty: Easy Quest. Category: Concept 9) Katherine Connor started her day with a cup of coffee while reading the newspaper. However, something about the day makes her feel low. When she reaches the office, she feels upset and distracted from work although she cannot pinpoint a reason for feeling this way. She continues to feel upset and distracted through the rest of the day. Which of the following is she most likely to be experiencing? A) A response B) A reaction C) A counteraction D) An attitude E) A mood Answer: E Explanation: Moods are cognitive in nature and their cause is often general and unclear. They last longer than emotions—for several hours or days. LO: 4.1: Differentiate between emotions and moods. AACSB: Analytical thinking Difficulty: Easy Employability Skills: Knowledge Application and Analysis Quest. Category: Application
148 richard@qwconsultancy.com
10) Peter Stiles works as a customer care agent for a cellular phone company. One afternoon, Peter receives a call from an irate customer who is having trouble connecting his mobile device. Peter provided instructions on how to connect the phone and even offered to ask his supervisor for additional help, but the customer continued to yell at him and call him names. After the customer hung up, Peter took a deep breath to try to lower his stress level and then, still angry, excused himself to get some air. Why is Peter's anger an emotion rather than a mood? A) The cause of his behavior is unclear. B) The feeling of being angry is prolonged. C) The anger has a contextual stimulus. D) The response is void of action. E) The response is cognitive in nature. Answer: C Explanation: Peter's feelings are brought about by being yelled at, which is a contextual stimulus for his behavior. His response is short, rather than prolonged. These are characteristics of an emotion. LO: 4.1: Differentiate between emotions and moods. AACSB: Analytical thinking Difficulty: Hard Employability Skills: Knowledge Application and Analysis Quest. Category: Application 11) Alana Gore works as a campaign manager at a nonprofit organization in Manhattan. She was recently asked by her supervisor to give a presentation on the progress of the ongoing climate campaign. However, throughout the presentation, her colleague Kelly Simmons interrupts her by asking irrelevant questions. In addition, she tries to insult her by constantly criticizing key findings and objectives achieved. Though Gore initially tried to be patient, she eventually loses her cool and retorts back, asking her to keep her questions and clarifications for the end of the presentation. Which of the following best describes why Gore's reaction can be categorized as an emotion instead of a mood? A) Its cause is general and ambiguous. B) It is not action oriented. C) It incorporates both positive and negative dimensions of affect. D) It is less cognitive in nature than moods. E) It is caused by a specific event. Answer: E Explanation: Gore's anger results in action when she retorts back at Simmons, which is a characteristic of emotion. Emotions are caused by a specific event, which in this scenario is Simmons criticizing and interrupting her presentation. LO: 4.1: Differentiate between emotions and moods. AACSB: Analytical thinking Difficulty: Hard Employability Skills: Knowledge Application and Analysis Quest. Category: Application
149 richard@qwconsultancy.com
12) Which of the following have an unclear, general cause, last for several hours or days, and are generally not indicated by distinct expressions? A) Emotions B) Feelings C) Perceptions D) Moods E) Reactions Answer: D Explanation: Moods last hours or even days and their causes are often general and unclear. They are generally not indicated by distinct facial expressions. LO: 4.1: Differentiate between emotions and moods. Difficulty: Easy Quest. Category: Concept 13) Which of the following are caused by a specific event, are very brief, and are accompanied by distinct facial expressions? A) Emotions B) Perceptions C) Attitudes D) Moods E) Reactions Answer: A Explanation: Emotions are intense feelings directed at someone or something. Most experts believe emotions are more fleeting than moods. Emotions tend to come and go fairly quickly, maybe even in a matter of seconds. Unlike moods, emotions like anger and disgust tend to be more clearly revealed by facial expressions. LO: 4.1: Differentiate between emotions and moods. Difficulty: Easy Quest. Category: Concept 14) ________ is one of the six universal emotions agreed on by researchers. A) Anger B) Shame C) Apathy D) Indifference E) Distress Answer: A Explanation: Many researchers agree on six universal emotions. These comprise anger, fear, sadness, happiness, disgust, and surprise. LO: 4.1: Differentiate between emotions and moods. Difficulty: Moderate Quest. Category: Concept
150 richard@qwconsultancy.com
15) Which of the following directly corresponds with low negative affect? A) Bored B) Nervous C) Content D) Relaxed E) Serene Answer: D Explanation: Relaxed directly corresponds with low negative affect. LO: 4.1: Differentiate between emotions and moods. Difficulty: Easy Quest. Category: Concept 16) Which of the following directly corresponds with high positive affect? A) Content B) Excited C) Fatigued D) Bored E) Relaxed Answer: B Explanation: Excited directly corresponds with high positive affect, while boredom is a pure marker of low positive affect. LO: 4.1: Differentiate between emotions and moods. Difficulty: Easy Quest. Category: Concept 17) Which of the following directly corresponds with low positive affect? A) Stressed B) Relaxed C) Bored D) Nervous E) Alert Answer: C Explanation: Bored directly corresponds with low positive affect, while excited is a pure marker of high positive affect. LO: 4.1: Differentiate between emotions and moods. Difficulty: Easy Quest. Category: Concept
151 richard@qwconsultancy.com
18) A pure marker of high negative affect is ________. A) depression B) relaxation C) fatigue D) boredom E) nervousness Answer: E Explanation: Nervousness directly corresponds with high negative affect. LO: 4.1: Differentiate between emotions and moods. Difficulty: Easy Quest. Category: Concept 19) Negative affect is a dimension consisting of ________ at the high end. A) nervousness B) relaxation C) contentment D) serenity E) depression Answer: A Explanation: Negative affect is a dimension consisting of nervousness, stress, and anxiety at the high end. LO: 4.1: Differentiate between emotions and moods. Difficulty: Easy Quest. Category: Concept 20) Emile Suarez is working as an assistant to the governor's press secretary. Emile is usually responsible for behind-the-scenes activities, but because his boss is on maternity leave, Emile is now getting ready to hold his first press conference. Emile is experiencing a range of emotions including tension, nervousness, and stress. Which of the following dimensions is he experiencing? A) Low positive affect B) Neutral affect C) Low negative affect D) High positive affect E) High negative affect Answer: E Explanation: Negative affect is a dimension consisting of nervousness, stress, and tension at the high end. LO: 4.1: Differentiate between emotions and moods. AACSB: Analytical thinking Difficulty: Easy Employability Skills: Knowledge Application and Analysis Quest. Category: Application
152 richard@qwconsultancy.com
21) Brad Parsons, a manager at the local branch of a steakhouse restaurant chain, has been feeling dissatisfied with his position for months. Brad believes he is overqualified for the job and that he would prefer a more fulfilling position. During his shifts at the steakhouse, Brad often feels tired, bored, and depressed. Which of the following dimensions is Brad likely experiencing? A) Low positive affect B) Neutral affect C) Low negative affect D) High positive affect E) High negative affect Answer: A Explanation: Signs of positive affect are boredom, sluggishness, and tiredness at the low end. LO: 4.1: Differentiate between emotions and moods. AACSB: Analytical thinking Difficulty: Moderate Employability Skills: Knowledge Application and Analysis Quest. Category: Application 22) In which dimension do emotions such as relaxation, serenity, and calmness fall? A) Low positive affect B) Neutral affect C) Low negative affect D) High positive affect E) High negative affect Answer: C Explanation: Low negative affect is a dimension consisting of relaxation, calmness, and serenity. LO: 4.1: Differentiate between emotions and moods. Difficulty: Easy Quest. Category: Concept 23) Which of the following refers to the tendency of most individuals to experience a mildly positive mood at zero input when nothing in particular is going on? A) Low positive affect B) Positivity offset C) Blunted affect D) Negativity bias E) Residual positivity Answer: B Explanation: Positivity offset means that at zero input, most individuals experience a mildly positive mood. For most people, positive moods are somewhat more common than negative moods. LO: 4.1: Differentiate between emotions and moods. Difficulty: Easy Quest. Category: Concept
153 richard@qwconsultancy.com
154 richard@qwconsultancy.com
24) ________ is most likely to be experienced during a positivity offset. A) Nervousness B) Depression C) Stress D) Anxiety E) Relaxation Answer: E Explanation: Positivity offset is defined as the tendency of most individuals to experience a mildly positive mood at zero input. Relaxation is an example of a mildly positive mood. LO: 4.1: Differentiate between emotions and moods. Difficulty: Easy Quest. Category: Concept 25) Andrea Dumas recently shared the results of a survey she completed asking workers at the auto parts plant where she works about their emotions and happiness quotient. Given that she often heard complaints about poor working conditions in the plant and the company's limited insurance benefits, Andrea had been expected to find that employees generally had a negative mood. Instead, Andrea found that the majority of workers responded that they were generally positive while they were at work. What might explain Andrea's findings? A) Butterfly effect B) Framing effect C) Anchoring bias D) Distinction bias E) Positivity offset Answer: E Explanation: Positivity offset is defined as the tendency of most individuals to experience a mildly positive mood at zero input. LO: 4.1: Differentiate between emotions and moods. AACSB: Analytical thinking Difficulty: Hard Employability Skills: Knowledge Application and Analysis Quest. Category: Application 26) Moods are more intense than emotions and invariably arise because of a specific event acting as a stimulus. Answer: FALSE Explanation: Emotions are defined as intense, discrete and short-lived feeling experiences that are often caused by a specific event. Moods are feelings that tend to be less intense than emotions and often lack a contextual stimulus. LO: 4.1: Differentiate between emotions and moods. Difficulty: Easy Quest. Category: Concept
155 richard@qwconsultancy.com
27) Research shows that ethical decisions are primarily based on feelings rather than on thoughts. Answer: TRUE Explanation: Numerous studies show that ethical decisions are primarily based on feelings rather than on thoughts, even though we tend to see moral issues as logical and reasonable, not as emotional. LO: 4.1: Differentiate between emotions and moods. AACSB: Ethical understanding and reasoning Difficulty: Moderate Quest. Category: Concept 28) Moods are usually accompanied by distinct facial expressions. Answer: FALSE Explanation: Moods are generally not indicated by distinct facial expressions. LO: 4.1: Differentiate between emotions and moods. Difficulty: Easy Quest. Category: Concept 29) Emotions tend to last for several days, for a much longer time period than moods. Answer: FALSE Explanation: Emotions are more fleeting than moods. Moods last longer than emotions—for hours or even days. LO: 4.1: Differentiate between emotions and moods. Difficulty: Easy Quest. Category: Concept 30) Studying facial expressions is a good way to identify basic emotions. Answer: FALSE Explanation: Psychologists try to identify basic emotions by studying facial expressions but have found the process difficult. One problem is that some emotions are too complex to be easily represented on our faces. LO: 4.1: Differentiate between emotions and moods. Difficulty: Easy Quest. Category: Concept 31) Nervousness directly corresponds with high negative affect. Answer: TRUE Explanation: Nervousness directly corresponds with high negative affect. LO: 4.1: Differentiate between emotions and moods. Difficulty: Easy Quest. Category: Concept
156 richard@qwconsultancy.com
32) Negative affect is a dimension consisting of nervousness, stress, and anxiety at the high end and boredom, depression, and fatigue at the low end. Answer: TRUE Explanation: Negative affect is a dimension consisting of nervousness, stress, and anxiety at the high end and boredom, depression, and fatigue at the low end. LO: 4.1: Differentiate between emotions and moods. Difficulty: Easy Quest. Category: Concept 33) Positivity offset may be defined as the tendency of most individuals to experience a negative mood when nothing in particular is going on. Answer: FALSE Explanation: Positivity offset may be defined as the tendency of most individuals to experience a mildly positive mood when nothing in particular is going on. So, for most people, positive moods are somewhat more common than negative moods. LO: 4.1: Differentiate between emotions and moods. Difficulty: Easy Quest. Category: Concept 34) Interestingly, research indicates that our responses to moral emotions differ from our responses to other emotions. Answer: TRUE Explanation: When we feel moral anger, for instance, we may be more likely to confront the situation that causes it. However, we cannot assume our emotional reactions to events on a moral level will be the same as someone else's. Moral emotions are learned, usually in childhood, and thus they are not universal like innate emotions. LO: 4.1: Differentiate between emotions and moods. Difficulty: Easy Quest. Category: Concept 35) Explain why understanding emotions across cultures is important. Answer: Differentiating emotions across cultures is quite important in this era of globalization: you may find yourself communicating with people from radically different cultures. Understanding the nuances in how they express emotion can be very helpful. Studying how emotions are displayed, educating yourself on culture-specific emotions, and paying attention to cues when interacting with people from other cultures can help you become a better crosscultural communicator. LO: 4.1: Differentiate between emotions and moods. Difficulty: Moderate Quest. Category: Concept
157 richard@qwconsultancy.com
36) Explain how affect, emotions, and moods are different from each other. Answer: Affect is a generic term that covers a broad range of feelings that people experience. It is a concept that encompasses both emotions and moods. Emotions are intense, discrete and short-lived feeling experiences that are often caused by a specific event. On the other hand, moods are feelings that tend to be less intense than emotions and that lack a contextual stimulus. LO: 4.1: Differentiate between emotions and moods. Difficulty: Moderate Quest. Category: Concept 37) Compare and contrast emotions and moods. Answer: Emotions are more likely to be caused by a specific event, and emotions are more fleeting than moods. Unlike moods, emotions like anger and disgust tend to be more clearly revealed by facial expressions. In addition, some researchers state that emotions may be more action-oriented and they may lead one to take some immediate action. On the other hand, moods may be more cognitive, meaning they may cause one to think or brood for a while. LO: 4.1: Differentiate between emotions and moods. AACSB: Analytical thinking Difficulty: Moderate Quest. Category: Synthesis 38) Individual differences in strength with which individuals experience their emotions is known as ________. A) the Hawthorne effects B) affect intensity C) intelligence quotient D) positivity offset E) ambiguity effect Answer: B Explanation: Affect intensity is defined as the individual differences in the strength with which individuals experience their emotions. Affectively intense people experience both positive and negative emotions more deeply. For instance, when they are sad, they are very sad, and when they are happy, they are very happy. LO: 4.2: Identify the sources of emotions and moods. Difficulty: Easy Quest. Category: Concept
158 richard@qwconsultancy.com
39) Ben Ervin often experiences emotions in a much stronger manner than most of his other colleagues. Events that do not provoke any significant emotional response from another person send him into fits of happiness, anger, or depression. In view of such a situation, Ervin is displaying a high level of ________. A) intelligence quotient B) sentience quotient C) intellectual giftedness D) butterfly effect E) affect intensity Answer: E Explanation: Affect intensity is defined as the individual differences in the strength with which individuals experience their emotions. Affectively intense people experience both positive and negative emotions more deeply: when they are sad, they are very sad, and when they are happy, they are very happy. LO: 4.2: Identify the sources of emotions and moods. AACSB: Analytical thinking Difficulty: Moderate Employability Skills: Knowledge Application and Analysis Quest. Category: Application 40) The tendency of people to associate two events when there is no connection is referred to as ________. A) affect intensity B) focusing effect C) distinction bias D) omission bias E) illusory correlation Answer: E Explanation: Illusory correlation is the tendency of people to associate two events that have no connection. LO: 4.2: Identify the sources of emotions and moods. Difficulty: Easy Quest. Category: Concept
159 richard@qwconsultancy.com
41) Gerard Yoder believes that every time he picks up his daughter, Penny, from school, it brings him good luck. The last time he picked her up from school, he won a lottery ticket. Today, after he received her from school, he reached his office and found out that he has been promoted. Which of the following concepts best describes Yoder's belief? A) Affect intensity B) Bandwagon effect C) Illusory superiority D) Positive bias E) Illusory correlation Answer: E Explanation: Illusory correlation is the tendency of people to associate two events when there is no connection. LO: 4.2: Identify the sources of emotions and moods. AACSB: Analytical thinking Difficulty: Moderate Employability Skills: Knowledge Application and Analysis Quest. Category: Application 42) Positive affect is highest for the U.S., China, Germany and Japan on ________. A) Sundays B) Mondays C) Saturdays D) Tuesdays E) Wednesdays Answer: A Explanation: In these four cultures, people tend to experience the highest level of positive affect on Sundays. Refer to exhibit 4-4. LO: 4.2: Identify the sources of emotions and moods. AACSB: Diverse and multicultural work environments Difficulty: Easy Quest. Category: Concept 43) Which of the following is a common response to emotionally charged and stressful situations? A) Deeper engagement in the situation B) High positive affect C) Low negative affect D) Disengagement from the situation E) Chronic boredom Answer: D Explanation: Mounting levels of stress can worsen our moods as we experience more negative emotions. When situations are overly emotionally charged and stressful, we have a natural response to disengage, to literally look away. LO: 4.2: Identify the sources of emotions and moods. Difficulty: Easy Quest. Category: Concept 160 richard@qwconsultancy.com
44) If you want to schedule a meeting when most coworkers will be in a positive mood, you should schedule the meet for ________. A) 10:30 in the morning B) 8 am C) just after lunch D) around 4 in the afternoon E) after 1:30 pm but before 3 pm Answer: A Explanation: Research shows that moods vary by the time of day. Most of us follow the same pattern. On a typical weekday, levels of positive affect tend to rise in the morning after sunrise, peaking in the late morning (ten o'clock to noon) and then slowly decline until early evening. LO: 4.2: Identify the sources of emotions and moods. Difficulty: Moderate Employability Skills: Knowledge Application and Analysis Quest. Category: Application 45) Which of the following statements is true regarding sources of moods and emotions? A) Reduced sleep makes a person more alert during waking hours. B) Physical and informal activities increase positive mood. C) Weather has a controlling effect on moods. D) Intensity of negative emotions increases with age. E) Mounting levels of stress at work create positive emotions. Answer: B Explanation: Research suggests activities that are physical, such as skiing or hiking with friends, informal, such as going to a party, or epicurean, like eating with others, are more strongly associated with increases in positive mood than events that are formal or sedentary. LO: 4.2: Identify the sources of emotions and moods. Difficulty: Easy Quest. Category: Concept 46) Carlos is upset because his boss gave him work to finish over the weekend. It is Friday. Which of the following is most likely to help Carlos lift his spirits? A) Watching TV with his friends B) Playing a video game at home C) Attending a meeting D) Washing his car E) Going for a run Answer: D Explanation: Carlos should go for a run. Research consistently shows that exercise enhances people's positive moods and reduces perceptions of fatigue. Exercise can also protect against persistent negative moods and improve the ability to recover from negative experiences. Compared to going for a run, the other choices are more sedentary. LO: 4.2: Identify the sources of emotions and moods. Difficulty: Easy Quest. Category: Concept 161 richard@qwconsultancy.com
162 richard@qwconsultancy.com
SuperMaxx Manufacturing wants to cut costs and improve global production efficiency. Managers have decided to implement a new factory schedule for its Japanese factory. Because the new schedule will require all employees to work some nights and weekends, managers think it will not be popular. SuperMaxx will also be increasing automation at its factory in China. 47) If managers want to introduce the new schedule for its Japanese facility when employees are in their best mood, managers should make the announcement on a ________. A) Sunday B) Thursday C) Wednesday D) Monday E) Friday Answer: A Explanation: Managers should introduce the new factory schedule on Sunday, when positive affect is highest in Japan. LO: 4.2: Identify the sources of emotions and moods. AACSB: Diverse and multicultural work environments Difficulty: Hard Quest. Category: Critical Thinking 48) If managers want to ensure that employees continue to complete their shifts, managers should avoid ________ when announcing its new strategy in China. A) Wednesday B) Friday C) Tuesday D) Saturday E) Monday Answer: E Explanation: Managers should avoid making the announcement in China on Monday. In China, negative affect is highest on Monday. Learning that they might lose their job to automation would likely further increase negative affect among employees. LO: 4.2: Identify the sources of emotions and moods. AACSB: Diverse and multicultural work environments Difficulty: Hard Quest. Category: Critical Thinking
163 richard@qwconsultancy.com
49) Under which of the following conditions, if true, would the manager's suggestion least likely be accepted? A) The company conducts one-day offsite events on an annual basis to allow employees some time to unwind. B) The skills required by the company are highly specialized, and hence the company is shortstaffed. C) The employees, though experiencing high levels of stress, also report high levels of job engagement. D) The company's employees have reported high stress levels for more than three years in a row. E) The company's leading competitor was recently voted as one of the best places to work due to its focus on employee well-being. Answer: B Explanation: If the company is short staffed, then the company will most likely not be able to allow each of its employees to go on a seven-day mandatory vacation. Hence, this would weaken the likelihood that the manager's suggestion will be accepted. If the company's employees have reported high stress levels for more than three years in a row, then this would increase the likelihood that the manager's suggestion will be accepted. The fact that the company's leading competitor was recently voted as one of the best places to work due to its focus on employee well-being only slightly increases the likelihood that the manager's suggestion will be accepted. If the company conducts one-day offsite events on an annual basis, it only enhances the likelihood that the manager's suggestion will be accepted, as one-day events are not sufficient to combat work stress. The fact that the employees report high levels of stress as well as job engagement neither increases nor decreases the likelihood that the manager's suggestion will be accepted. LO: 4.2: Identify the sources of emotions and moods. AACSB: Reflective thinking Difficulty: Hard Quest. Category: Critical Thinking 50) Which of the following statements regarding gender identity in relation to emotional reactions is true? A) Evidence shows that on average, men are more emotionally expressive than women. B) Multiple studies show that men experience emotions more intensely than women. C) Women express anger more frequently than men is only true in situational conditions. D) Stereotypical perceptions of women as "emotional" and men as "angry" in the workplace persist despite little evidence they are true. E) Men display trivial positive emotions more frequently than women. Answer: D Explanation: Stereotypical perceptions of women as "emotional" and men as "angry" in the workplace persist despite little evidence they are true. LO: 4.2: Identify the sources of emotions and moods. Difficulty: Moderate Quest. Category: Concept
164 richard@qwconsultancy.com
51) The term illusory correlation refers to the individual differences in the strength with which individuals experience their emotions. Answer: FALSE Explanation: The term affect intensity refers to the individual differences in the strength with which individuals experience their emotions. Illusory correlation is the tendency of people to associate two events when in reality there is no connection. LO: 4.2: Identify the sources of emotions and moods. Difficulty: Easy Quest. Category: Concept 52) Research shows that for most people, weather has little effect on mood. Answer: TRUE Explanation: Research shows that for most people, weather has little effect on mood. LO: 4.2: Identify the sources of emotions and moods. Difficulty: Easy Quest. Category: Concept 53) Poor or reduced sleep makes it difficult to control emotions and can impair job satisfaction. Answer: TRUE Explanation: Poor or reduced sleep makes it difficult to control emotions and can impair job satisfaction. LO: 4.2: Identify the sources of emotions and moods. Difficulty: Easy Quest. Category: Concept 54) How does stress affect emotions and moods? Answer: Stressful daily events at work, such as a nasty e-mail, an impending deadline, the loss of a big sale, or a reprimand from the boss, negatively affect moods. The effects of stress also build over time. Mounting levels of stress can worsen our moods, and we experience more negative emotions. LO: 4.2: Identify the sources of emotions and moods. Difficulty: Easy Quest. Category: Concept 55) How do social interaction impact emotions and mood? Answer: The interactions we have with other people can affect our emotions and moods. For example, negative interactions at work not only affect your emotions at work, but they are also so powerful that these emotions can "spillover" and affect your relationships with family members. Social interactions also do not solely result in purely positive or negative emotions. LO: 4.2: Identify the sources of emotions and moods. Difficulty: Moderate Quest. Category: Concept
165 richard@qwconsultancy.com
56) Discuss emotions within the context of gender identity. Answer: While many people believe the stereotype that women are more emotional than men, the evidence shows that it is not always correct. People also tend to make attributions about women's personalities in ways that might be based on stereotypes of typical emotional reactions. One study showed that when viewing pictures of faces, participants interpreted the women's emotional expressions as being dispositional, whereas the men's expressions were interpreted as situational. LO: 4.2: Identify the sources of emotions and moods. Difficulty: Moderate Quest. Category: Concept 57) ________ refers to an employee's organizationally desired emotions during interpersonal transactions at work. A) Emotional labor B) Self-concordance C) Emotional liability D) Negative affect E) Cognitive deviance Answer: A Explanation: Emotional labor is defined as an employee's organizationally desired emotions during interpersonal transactions at work LO: 4.3: Show the impact emotional labor has on employees. Difficulty: Easy Quest. Category: Concept 58) As a flight attendant on a transatlantic route, Mark Duffy works long and sometimes chaotic hours. Even so, Mark knows that no matter how tired he may feel, part of his job is to greet and smile at passengers as they enter the aircraft and to project an aura of calm competence. Mark's job requires ________. A) social loafing B) emotional labor C) cognitive dissonance D) self-concordance E) positivity offset Answer: B Explanation: Emotional labor is defined as a situation in which an employee expresses organizationally desired emotions during interpersonal transactions at work, meaning that his/her job demands a certain set of emotional responses regardless of true feelings. LO: 4.3: Show the impact emotional labor has on employees. AACSB: Analytical thinking Difficulty: Hard Employability Skills: Knowledge Application and Analysis Quest. Category: Application
166 richard@qwconsultancy.com
59) Gil works as a bartender in an upscale dinner club in Los Angeles. She has a number of regular customers including one man who stops in almost every evening after work. He orders the same drink every day and after being served, always says "thank you, darling, you're looking hot tonight" to Gil. Despite hating being called darling and told she looks hot, Gil always smiles at the customer and goes on with her work. What is Gil experiencing? A) Illusory correlation B) Emotional dissonance C) Positive affect D) Emotional selection E) Positivity offset Answer: B Explanation: Emotional dissonance is defined as the inconsistency between the emotions people feel and the emotions they project. LO: 4.3: Show the impact emotional labor has on employees. AACSB: Analytical thinking Difficulty: Moderate Employability Skills: Knowledge Application and Analysis Quest. Category: Application 60) Carrie recently received the Employee of the Year award at the call center where she works as a customer service representative. She was chosen for the award because she is known to be highly courteous toward even the most difficult of callers and is efficient at handling their queries. A month after receiving the award, Carrie submits her resignation. According to her exit interview, she felt burnt out. Which of the following best explains this situation? A) Carrie was overwhelmed by the award. B) Carrie lacked the skills and motivation to perform her job in an effective manner. C) Carrie felt the strain of having to constantly fake positive emotions towards her customers. D) Carrie was demotivated by the lack of recognition and respect from her peers. E) Carrie was unable to empathize with her callers and the problems that they faced. Answer: C Explanation: This situation relates to emotional dissonance. Carrie was most likely feeling the strain of having to project one emotion while feeling another, especially when dealing with difficult callers, which ultimately caused her to feel burnt out. It cannot be argued that Carrie was overwhelmed by the award as it would not cause her to feel burnt out. The fact that Carrie won the Employee of the Year award proves that she was skilled and motivated, and that she received recognition for her work. It cannot be argued that she lacked empathy as she was courteous with her callers and efficiently handled their queries. LO: 4.3: Show the impact emotional labor has on employees. AACSB: Reflective thinking Difficulty: Hard Quest. Category: Critical Thinking
167 richard@qwconsultancy.com
61) Which of the following refers to inconsistencies between the emotions people feel and the emotions they project? A) Collective efficacy B) Emotional dissonance C) Social loafing D) Positivity offset E) Self-concordance Answer: B Explanation: Emotional dissonance is defined as the inconsistency between the emotions people feel and the emotions they project. LO: 4.3: Show the impact emotional labor has on employees. Difficulty: Easy Quest. Category: Concept 62) ________ emotions are our actual emotions. A) Felt B) Conditional C) Surface D) Disseminative E) Cognitive Answer: A Explanation: Felt emotions are an individual's actual emotions. In contrast, displayed emotions are those that the organization requires workers to show and considers appropriate in each job. LO: 4.3: Show the impact emotional labor has on employees. Difficulty: Easy Quest. Category: Concept 63) ________ refers to the practice of hiding feelings and foregoing emotional expressions in response to display rules. A) Surface acting B) Anchoring bias C) Active camouflage D) Bandwagon effect E) Framing effect Answer: A Explanation: Surface acting is defined as hiding one's feelings and foregoing emotional expressions in response to display rules, or rather outwardly showing emotions that are different from those that are felt. LO: 4.3: Show the impact emotional labor has on employees. Difficulty: Easy Quest. Category: Concept
168 richard@qwconsultancy.com
64) Surface acting ________. A) involves foregoing emotional expressions in response to display rules B) involves changing one's likes and dislikes to be in sync with display rules C) involves displaying one's true feelings explicitly toward a person D) deals with expressing felt emotions toward a person or event E) involves trying to modify one's true feelings based on display rules Answer: A Explanation: Surface acting is hiding feelings and foregoing emotional expressions in response to display rules. Surface acting deals with displayed emotions, and deep acting deals with felt emotions. Research shows surface acting is more stressful to employees because it entails feigning their true emotions. LO: 4.3: Show the impact emotional labor has on employees. Difficulty: Moderate Quest. Category: Concept 65) Which of the following is an example of surface acting? A) An employee expressing her true admiration for her colleague B) An employee masking her emotions of distrust toward the management C) An employee striving to like her job even though it feels uninteresting D) An employee retorting back at an annoying customer E) A colleague trying to get along with a difficult teammate Answer: B Explanation: Surface acting is hiding feelings and foregoing emotional expressions in response to display rules. An employee masking her emotions of distrust toward the management is an example of surface acting. LO: 4.3: Show the impact emotional labor has on employees. AACSB: Interpersonal relations and teamwork Difficulty: Easy Quest. Category: Concept 66) ________ refers to the practice of modifying one's true feelings based on display rules. A) Surface acting B) Deep acting C) Social loafing D) Behavioral contagion E) Collective efficacy Answer: B Explanation: Deep acting refers to the practice of modifying one's true feelings based on display rules. LO: 4.3: Show the impact emotional labor has on employees. Difficulty: Easy Quest. Category: Concept
169 richard@qwconsultancy.com
67) Which of the following is an example of deep acting? A) Sasha ignores a colleague's question because she knows he has a personal grudge against her. B) Andy conceals her emotions and politely talks to a customer even though he is annoying her. C) Liza smiles at her manager even though last week she was passed over for a promotion. D) Dave tries to get along with a co-worker because they will soon be working together on a project. E) Javier is tired of working long hours and calls in sick on Monday to skip work. Answer: D Explanation: Deep acting refers to trying to modify one's true feelings based on display rules. In this situation, Dave is trying to know his colleague better because they will soon be working together on a project. LO: 4.3: Show the impact emotional labor has on employees. AACSB: Analytical thinking Difficulty: Hard Quest. Category: Application 68) Which type of emotions does surface acting involve? A) Negative B) Weak C) Cognitive D) Strong E) Displayed Answer: E Explanation: Displayed emotions are those that the organization requires workers to show and considers appropriate in each job, therefore a person engaging in surface acting is only concerned with displayed emotions. Deep acting deals with felt emotions. Surface acting is not defined regarding negative, weak, or strong emotions. LO: 4.3: Show the impact emotional labor has on employees. Difficulty: Easy Quest. Category: Concept 69) According to research, American workplaces expect employees to display positive emotions and suppress negative emotions. Answer: TRUE Explanation: Research suggests that in U.S. workplaces, it is expected that employees should typically display positive emotions like happiness and excitement and suppress negative emotions like fear, anger, disgust, and contempt. LO: 4.3: Show the impact emotional labor has on employees. Difficulty: Moderate Quest. Category: Concept
170 richard@qwconsultancy.com
70) Surface acting is hiding feelings and emotional expressions in response to display rules. Answer: TRUE Explanation: Surface acting is hiding feelings and emotional expressions in response to display rules. LO: 4.3: Show the impact emotional labor has on employees. Difficulty: Easy Quest. Category: Concept 71) Compare and contrast felt emotions and displayed emotions. Answer: Felt emotions are an individual's actual emotions. In contrast, displayed emotions are those that the organization requires workers to show and considers appropriate in each job. They're not innate; they're learned. LO: 4.3: Show the impact emotional labor has on employees. Difficulty: Moderate Quest. Category: Concept 72) Explain the relationship of emotional labor to emotional dissonance. Answer: Emotional labor may be defined as a situation in which an employee expresses organizationally desired emotions during interpersonal transactions at work. Emotional labor creates dilemmas and emotional dissonance for employees. Emotional dissonance may be explained as inconsistencies between the emotions people feel and the emotions they project. Emotional dissonance can take a heavy toll on employees, and bottled-up feelings of frustration, anger, and resentment can eventually lead to emotional exhaustion and burnout. LO: 4.3: Show the impact emotional labor has on employees. Difficulty: Moderate Quest. Category: Concept 73) ________ refers to a model which suggests that workplace events cause emotional reactions on the part of employees, which then influence workplace attitudes and behaviors. A) Ironic process theory B) Implicit personality theory C) Disengagement theory D) Affective events theory E) Construal level theory Answer: D Explanation: Affective events theory (AET) refers to a model that suggests that workplace events cause emotional reactions on the part of employees, which then influence workplace attitudes and behaviors. LO: 4.4: Describe affective events theory. Difficulty: Easy Quest. Category: Concept
171 richard@qwconsultancy.com
74) Which of the following is true according to the affective events theory? A) Employees react emotionally to events at work, which affects their job performance and satisfaction. B) Employees who score high on emotional stability are more likely to react strongly to negative events. C) Employees who experience emotional dissonance at work can only be motivated with the help of money. D) An employee's emotions are independent of the employee's age and gender. E) Employees scoring high on the personality trait of extraversion tend to be more successful at the workplace. Answer: A Explanation: Affective events theory (AET) demonstrates that employees react emotionally to things that happen to them at work, and this reaction influences their job performance and satisfaction. LO: 4.4: Describe affective events theory. Difficulty: Moderate Quest. Category: Concept 75) Using affective events theory, identify which of the following is an example of a positive event for remote workers? A) A quick and productive Zoom meeting with colleagues B) Being passed over for a promotion C) Colleagues refusing to carry their share of work D) Receiving conflicting directions from different managers E) Receiving a gift from a colleague on a birthday Answer: A Explanation: The affective events theory recognizes that emotions are a response to an event in the work environment. This environment creates work events that can be hassles, uplifting events, or both. For remote workers, a positive affective event might be a quick and productive Zoom meeting with colleagues. LO: 4.4: Describe affective events theory. Difficulty: Moderate Quest. Category: Concept 76) According to the affective events theory, work events trigger positive or negative emotional reactions among employees and these reactions influence their job performance and satisfaction. Answer: TRUE Explanation: The affective events theory (AET) demonstrates that employees react emotionally to things that happen to them at work, and this reaction influences their job performance and satisfaction. Work events trigger positive or negative emotional reactions, to which employees' personalities and moods predispose them to respond with greater or lesser intensity. LO: 4.4: Describe affective events theory. Difficulty: Easy Quest. Category: Concept
172 richard@qwconsultancy.com
77) What is affective events theory and how is it associated with job performance and satisfaction? Answer: A model called affective events theory (AET) demonstrates that employees react emotionally to things that happen to them at work, and this reaction influences their job performance and satisfaction. The theory begins by recognizing that emotions are a response to an event in the work environment. This environment creates work events that can be hassles, uplifting events, or both. These work events trigger positive or negative emotional reactions, to which employees' personalities and moods predispose them to respond with greater or lesser intensity. LO: 4.4: Describe affective events theory. Difficulty: Moderate Quest. Category: Concept 78) Rachel Hartley is a former top management graduate student who now works as a project manager at a retail firm. However, despite her strong educational background, she often finds it difficult to manage her team efficiently. She is not able to understand her team's emotions when they feel frustrated with a work challenge or when they need appreciation for a job well done. Which of the following does Hartley suffer from? A) Downing effect B) Low emotional dissonance C) Depressive realism D) Low cognitive dissonance E) Low emotional intelligence Answer: E Explanation: Emotional intelligence (EI) refers to the ability to detect and to manage emotional cues and information. LO: 4.5: Describe emotional intelligence. AACSB: Analytical thinking Difficulty: Hard Employability Skills: Knowledge Application and Analysis Quest. Category: Application 79) ________ refers to a person's ability to detect and to manage emotional cues and information. A) Emotional intelligence B) Self-concordance C) Emotional spectrum D) Illusory correlation E) Cognitive dissonance Answer: A Explanation: Emotional intelligence (EI) is a person's ability to (1) perceive emotions in the self and others, (2) understand the meaning of these emotions, and (3) regulate one's emotions. LO: 4.5: Describe emotional intelligence. Difficulty: Easy Quest. Category: Concept
173 richard@qwconsultancy.com
174 richard@qwconsultancy.com
80) Roger Smith is pondering how to respond to complaints about a member of his staff, Sonkat. Sonkat joined the company about six months ago and while he has proven to be a hard worker, other staff members have complained that Sonkat makes rude, off the cuff remarks during meetings without considering whether they might offend someone else, overstays his welcome when he stops by colleagues' desks, and leaves a mess almost every day in the office kitchen. Despite the efforts of several team members to encourage Sonkat to adapt to office norms, Sonkat seemingly remains oblivious to the upset he is causing. Sonkat's inability to be self-aware suggests that he has low ________. A) cognitive intelligence B) self-esteem C) negative affect D) emotional intelligence E) job satisfaction Answer: D Explanation: Emotional intelligence (EI) is a commonly accepted term that is based on three factors: a person's ability to (1) be self-aware (to recognize one's own emotions when experiencing them), (2) detect emotions in others, and (3) manage emotional cues and information. In general, people who know their emotions, or EI, are more effective. Sonkat shows that he is incapable of reading his own emotions, or those of others, revealing a low EI. LO: 4.5: Describe emotional intelligence. AACSB: Analytical thinking Difficulty: Moderate Employability Skills: Knowledge Application and Analysis Quest. Category: Application 81) Wanda Elder works as a human resource executive. Being helpful is inherent in her nature, and she regularly interacts with employees, helping them understand their needs and easing difficulties. She reads emotional cues of people with ease and knows how to express her emotions without violating organizational norms. Which of the following is Elder characterized by? A) High cognitive dissonance B) Low self-esteem C) High emotional dissonance D) High emotional intelligence E) Low job satisfaction Answer: D Explanation: Emotional intelligence (EI) is a person's ability to (1) perceive emotions in the self and others, (2) understand the meaning of these emotions, and (3) regulate one's emotions. LO: 4.5: Describe emotional intelligence. AACSB: Analytical thinking Difficulty: Moderate Employability Skills: Knowledge Application and Analysis Quest. Category: Application
175 richard@qwconsultancy.com
82) Whether a candidate has high emotional intelligence would be a significant factor when considering a candidate for the job of a(n) ________. A) event planner who needs to coordinate with several people B) engineer working on developing a software program C) web researcher who needs to consolidate online travel information D) copy-editor who checks spelling and grammatical errors in manuscripts E) technical writer who needs to create a 1000-word article every day Answer: A Explanation: Emotional intelligence (EI) is a person's ability to (1) perceive emotions in the self and others, (2) understand the meaning of these emotions, and (3) regulate one's emotions. It is specifically useful in a job that needs one to interact with several other people. LO: 4.5: Describe emotional intelligence. AACSB: Analytical thinking Difficulty: Moderate Employability Skills: Knowledge Application and Analysis Quest. Category: Application 83) Which of the following statements is true regarding emotional intelligence (EI)? A) It refers to a score derived from standardized tests to measure intelligence. B) It refers to an aptitude for intellectual activities that cannot be acquired with personal effort. C) It comprises a person's ability to detect others' and one's own feelings. D) It is based on rationality instead of intuitive appeal to make judgments. E) It refers to a person's ability to suppress his/her true emotions and express organizationally accurate emotions. Answer: C Explanation: The term emotional intelligence refers to a person's ability to detect and to manage emotional cues and information. LO: 4.5: Describe emotional intelligence. Difficulty: Moderate Quest. Category: Concept 84) Which of the following is an argument used against emotional intelligence? A) It is not genetically influenced and thus is void of an underlying biological factor. B) It does not have the capacity to predict criteria that matter. C) It can be learned by experience. D) It cannot be measured easily and measures of EI are diverse. E) It is not closely related to intelligence and personality. Answer: D Explanation: Many critics have raised questions about measuring EI. Because EI is a form of intelligence, they argue, there must be right and wrong answers for it on tests. The measures of EI are diverse, and researchers have not subjected them to as much rigorous study as they have measures of personality and general intelligence. LO: 4.5: Describe emotional intelligence. Difficulty: Easy Quest. Category: Concept
176 richard@qwconsultancy.com
177 richard@qwconsultancy.com
85) Sydney Bell is in charge of recruitment for a new assisted living facility in New Hampshire. Sydney knows that because the jobs the staff will be doing involve a high degree of social interaction, it is especially important to select the right people. Sydney is considering using a measure of EI in the selection process. What is EI and how could it help Sydney recruit valued employees? Explain why Sydney should not rely completely on EI in the recruitment process. Answer: Emotional intelligence (EI) is a person's ability to (1) be self-aware (to recognize his/her own emotions when he/she experiences them), (2) detect emotions in others, and (3) manage emotional cues and information. People who know their own emotions and are good at reading emotional cues—for instance, knowing why they're angry and how to express themselves without violating norms—are most likely to be effective. One implication from the evidence on EI to date is that employers should consider it a factor in hiring employees, especially in jobs that demand a high degree of social interaction. Sydney is hiring individuals who will be regularly engaging with residents of the facility, other staff, and family members. In other words, the job requires a great deal of social interaction. Sydney should be aware that EI is hard to measure and is very vaguely defined so should not be relied on entirely in the selection process. LO: 4.5: Describe emotional intelligence. AACSB: Analytical thinking Difficulty: Hard Quest. Category: Synthesis 86) Define emotional intelligence and explain the arguments for and against emotional intelligence. Answer: Emotional intelligence is a person's ability to: (a) be self-aware (to recognize his/her own emotions when he/she experiences them), (b) detect emotions in others, and (c) manage emotional cues and information. People who know their own emotions and are good at reading emotional cues—for instance, knowing why they're angry and how to express without violating norms—are more likely to be effective. The arguments for EI include: a) Intuitive Appeal: There's a lot of intuitive appeal to the EI concept. Most everyone would agree that it is good to possess street smarts and social intelligence. b) EI Predicts Criteria That Matter: Evidence is mounting that suggests a high level of EI means a person will perform well on the job. c) EI Is Biologically Based: One study has shown that people with damage to the part of the brain that governs emotional processing (lesions in an area of the prefrontal cortex) score significantly lower on EI tests. The arguments against EI include: a) EI Is Too Vague a Concept: To many researchers, it's not clear what EI is. b) EI Can't Be Measured: Many critics have raised questions about measuring EI. Because EI is a form of intelligence, for instance, then there must be right and wrong answers about it on tests, they argue. c) The Validity of EI Is Suspect: Some critics argue that because EI is so closely related to intelligence and personality, once you control for these factors, EI has nothing unique to offer. LO: 4.5: Describe emotional intelligence. Difficulty: Hard Quest. Category: Concept 178 richard@qwconsultancy.com
87) Which of the following is not true regarding emotion regulation? A) It can be defined as a person's ability to understand the meaning of emotions. B) Persons engaging in emotion regulation might try to cheer themselves when they're feeling down. C) It involves identifying and modifying the emotions one feels. D) Emotion regulation is a strong predictor of task performance for some jobs and organization citizenship behavior. E) Individuals with low levels of self-esteem are less likely to try to improve their sad moods. Answer: A Explanation: The central idea behind emotion regulation is to identify and modify the emotions one feels. Recent research suggests that emotion regulation is a strong predictor of task performance for some jobs and organizational citizenship behaviors. One strategy people use to change their emotions is surface acting. Finally, individuals with low self-esteem are less likely to try to improve their sad moods, perhaps because they are less likely than others to feel they deserve to be in a good mood. LO: 4.6: Identify strategies for emotion regulation. Difficulty: Easy Quest. Category: Concept 88) FlyME Airways was surprised by some recent survey results revealing that customers were dissatisfied with the level of service they received. All staff at FlyME Airways is trained to greet passengers and inquire about their day. Flight attendants are trained to introduce themselves and engage in casual conversation with passengers sitting in their section. Which of the following, if true, best explains this situation? A) Customers come to FlyME Airways for budget flights. B) Customers associate superior service with efficiency and not friendliness. C) FlyME Airways offers both short domestic and long-haul international flights. D) The training period for the staff is three weeks long. E) The staff at FlyME Airways is friendly but not familiar. Answer: B Explanation: If passengers associate superior service with efficiency and not friendliness, then this would most likely explain why FlyME Airways received low customer satisfaction scores. The airline's training process that encouraged causal conversation with passengers would not be welcomed by passengers who want to be quickly served and then left alone to read, sleep, or watch movies. If customers come to FlyME Airways for budget flights, then it does not explain their dissatisfaction as their interactions with staff are not related to ticket prices. Even if the training period for the staff is three weeks long it does not explain customers' dissatisfaction. In customer service, one is expected to be friendly but not familiar with the customer, hence, this too would not explain customers' dissatisfaction. The fact that FlyME offers both domestic and international flights should result in customer satisfaction. LO: 4.6: Identify strategies for emotion regulation. AACSB: Reflective thinking Difficulty: Hard Quest. Category: Critical Thinking
179 richard@qwconsultancy.com
180 richard@qwconsultancy.com
89) Open expression of emotions instead of keeping emotions "bottled up" can help individuals to regulate their emotions. Answer: TRUE Explanation: Research shows that the open expression of emotions can help individuals to regulate their emotions as opposed to keeping them "bottled up." Social sharing can reduce anger reactions when people can talk about the facts of a bad situation, their feelings about the situation, or any positive aspects of the situation. LO: 4.6: Identify strategies for emotion regulation. Difficulty: Moderate Quest. Category: Concept 90) Cognitive reappraisal is one method to regulate emotions effectively. Answer: TRUE Explanation: Cognitive reappraisal, or reframing our outlook on an emotional situation, is one way to regulate emotions effectively. LO: 4.6: Identify strategies for emotion regulation. Difficulty: Easy Quest. Category: Concept 91) The process of reframing our outlook on an emotional situation is known as social sharing. Answer: FALSE Explanation: Reframing our outlook on an emotional situation is referred to as cognitive reappraisal. Social sharing refers to venting. LO: 4.6: Identify strategies for emotion regulation. Difficulty: Easy Quest. Category: Concept 92) Emotional suppression used in crisis situations appears to help an individual recover from the event emotionally, while emotional suppression used as an everyday emotion regulation technique can take a toll on mental ability, emotional ability, health, and relationships. Answer: TRUE Explanation: Emotional suppression used in crisis situations appears to help an individual recover from the event emotionally, while emotional suppression used as an everyday emotion regulation technique can take a toll on mental ability, emotional ability, health, and relationships. Thus, unless we're truly in a crisis situation, acknowledging rather than suppressing our emotional responses to situations, and re-evaluating events after they occur, yields the best outcomes. LO: 4.6: Identify strategies for emotion regulation. Difficulty: Hard Quest. Category: Concept
181 richard@qwconsultancy.com
93) Jeremy Ross is a senior financial advisor for a mutual fund company. One of the requirements of Jeremy's job is that he be knowledgeable, friendly, and reassuring to clients. Consider the emotional labor that Jeremy must exert every day and the significance of the emotional intelligence Jeremy needs to be successful in is position. Describe how he applies his emotions and moods in his job. Answer: Jeremy must engage in surface acting daily. He must smile and act positive regardless of his felt emotions to be successful. Certainly, there will be times when Jeremy has emotional dissonance or feels a disparity between his true emotions and how he must appear to his clients. Jeremy uses his emotions and moods positively to guide his clients into making quick decisions in the purchase of the products offered by his firm. People in good moods tend to make quicker decisions. Finally, Jeremy applies his moods and positive emotions to provide excellent customer service to his clients. He spreads his good mood through emotional contagion and can get his customers to make larger orders and remain loyal to him in the competitive pharmaceutical industry. LO: 4.6: Identify strategies for emotion regulation. AACSB: Analytical thinking Difficulty: Hard Quest. Category: Synthesis 94) Discuss the emotional labor experienced by a customer service executive. Explain how a worker's emotional state can affect customer service and customers. Answer: Most jobs require emotional labor, an employee's expression of organizationally desired emotions during interpersonal transactions at work. A worker's emotional state influences customer service, which influences levels of repeat business and levels of customer satisfaction. Providing quality customer service makes demands on employees because it often puts them in a state of emotional dissonance. Over time, this state can lead to job burnout, decline in job performance, and lower job satisfaction. In addition, employees' emotions may also transfer to the customer. Studies indicate a matching effect between employee and customer emotions, an effect that is called emotional contagion, the "catching" of emotions from others. The primary explanation of emotional contagion is that when someone experiences positive emotions and laughs and smiles at you, you begin to copy that person's behavior. So, when employees express positive emotions, customers tend to respond positively. Emotional contagion is important because when customers catch the positive moods or emotions of employees, they shop longer. When an employee feels unfairly treated by a customer, for example, it's harder for him to display the positive emotions his organization expects of him. LO: 4.6: Identify strategies for emotion regulation. AACSB: Analytical thinking Difficulty: Moderate Quest. Category: Synthesis
182 richard@qwconsultancy.com
95) Which of the following is not true regarding negotiations, emotions, and moods? A) Displaying anger can be effective but feeling prideful about your performance appears to impair future negotiations. B) Individuals who do poorly in negotiations develop negative perceptions of their counterparts. C) A manager that has been involved in unsuccessful negotiations may be less willing to cooperate in future negotiations. D) Angry negotiators who have less power than their opponents have significantly worse outcomes. E) Emotions do not play a role in negotiations. Answer: E Explanation: It is not true that emotions do not play a role in negotiations. In fact, displaying anger can be effective but feeling prideful about your performance appears to impair future negotiations. Furthermore, Angry negotiators who have less power than their opponents have significantly worse outcomes. Those individuals who do poorly in negotiations experience negative emotions, develop negative perceptions of their counterparts, and are less willing to share information or be cooperative in future negotiations. LO: 4.7: Apply concepts about emotions and moods to specific OB issues. AACSB: Diverse and multicultural work environments Difficulty: Moderate Quest. Category: Concept 96) Bad moods can contribute to safety issues at work. Which of the following is not associated with a bad mood and therefore a safety issue? A) Anxiousness B) Assurance C) Pessimism D) Homesickness E) Fearfulness Answer: B Explanation: Bad moods can contribute to injury at work in several ways. Individuals in negative moods tend to be more anxious, which can make them less able to cope effectively with hazards. A person who is always fearful will be more pessimistic about the effectiveness of safety precautions because they feel they will just get hurt anyway, or might panic or freeze up when confronted with a threatening situation. Two studies of migrant workers and expatriate military personnel found that negative, longing feelings of homesickness predicted both job performance and safety behaviors. LO: 4.7: Apply concepts about emotions and moods to specific OB issues. Difficulty: Moderate Quest. Category: Concept
183 richard@qwconsultancy.com
97) Since joining her company a little over a year ago, Jessica La Paz has been promoted once and is already in line for another promotion. Jessica routinely gets excellent feedback from her manager who notes that Jessica is excited and enthusiastic about her work and meets every challenge with a sense of optimism. Jessica's dimension can best be identified as being ________. A) low positive affect B) neutral affect C) low negative affect D) high positive affect E) high negative affect Answer: D Explanation: Positive affect is a dimension consisting of positive emotions such as excitement, self-assurance, and cheerfulness at the high end. LO: 4.7: Apply concepts about emotions and moods to specific OB issues. AACSB: Analytical thinking Difficulty: Moderate Employability Skills: Knowledge Application and Analysis Quest. Category: Application 98) Which of the following is not true of counterproductive work behaviors? A) Research reveals that as compared to sadness; anger correlates with more aggressive counterproductive workplace behavior. B) It can be traced to positive emotions and can take many forms. C) It includes activities like gossiping or surfing the Internet instead of working. D) Unethical behaviors are also known as deviant behaviors. E) An envious employee could undermine other employees and take credit for what they have achieved. Answer: B Explanation: Unethical behaviors, also known as deviant behaviors, can do a great deal of harm in organizations and often take the form of counterproductive work behaviors. Unethical workplace behavior can be traced to negative (not positive) emotions and can take many forms. People who feel negative emotions are more likely than others to engage in short-term deviant behavior at work such as gossiping or excessively surfing the Internet instead of working. LO: 4.7: Apply concepts about emotions and moods to specific OB issues. AACSB: Ethical understanding and reasoning Difficulty: Moderate Quest. Category: Concept
184 richard@qwconsultancy.com
99) Which of the following statements is true regarding the effects of moods and emotions on decision making? A) Moods and emotions have no direct influence on decision making. B) People in good moods are less likely than others to use heuristics to make decisions. C) Positive emotions tend to diminish problem-solving skills and analytical skills. D) People with positive emotions are slower at processing information than depressed people. E) Individuals in a negative mood may take higher risks than they do in a positive mood. Answer: E Explanation: OB researchers are increasingly finding that moods and emotions have important effects on decision making. People in good moods or experiencing positive emotions are more likely than others to use heuristics, or rules of thumb, to help make good decisions quickly. Positive emotions enhance problem-solving skills. Individuals in a negative mood may take higher risks than they do in a positive mood. LO: 4.7: Apply concepts about emotions and moods to specific OB issues. Difficulty: Moderate Quest. Category: Concept 100) Grace and Alice are team members at one of Denver's biggest publicity companies. Both women participated in the same training program, are at the same level in the organization, and are assigned a similar amount of work. Despite their busy days, Grace is always in a good mood, while Alice generally appears to be in a negative mood. Compared to Alice, Grace is more likely to ________. A) take an extended time at processing information B) have poor problem-solving skills C) make poorer decisions D) find more original solutions to problems E) weigh all possible options rather than the most likely ones Answer: D Explanation: People in good moods or experiencing positive emotions are more likely than others to use heuristics, or rules of thumb, to help make good decisions quickly. Positive emotions enhance problem-solving skills. OB researchers continue to debate the role of negative emotions and moods in decision making. One study shows that participants made choices reflecting more originality in a negative mood. LO: 4.7: Apply concepts about emotions and moods to specific OB issues. AACSB: Analytical thinking Difficulty: Moderate Employability Skills: Knowledge Application and Analysis Quest. Category: Application
185 richard@qwconsultancy.com
101) Dawn Wang heads an ad agency in Texas and regularly needs to work with copywriters, artists, and designers to come up with effective branding solutions for products. For one of the company's esteemed clients, Wang and her team need to brainstorm ideas for a slogan for the client's new line of clothing. In such a situation, why is it particularly important for Wang to keep her team happy? A) People take a long time at processing information when they are in a good mood. B) People tend to weigh all options to make less likely decisions when in a good mood. C) People are less likely to use rules of thumb when they are in a good mood. D) People tend to make decisions quickly when they are in a good mood. E) People are more creative and produce more ideas when they are in a good mood. Answer: E Explanation: People in good moods tend to be more creative than people in bad moods. They produce more ideas and more options, and others think their ideas are original. LO: 4.7: Apply concepts about emotions and moods to specific OB issues. AACSB: Analytical thinking Difficulty: Moderate Employability Skills: Knowledge Application and Analysis Quest. Category: Application 102) Which of the following statements is true regarding negotiation? A) Displaying a negative emotion at a negotiation can never be effective. B) Angry negotiators who have less power than opponents have better outcomes. C) Individuals who do poorly in a negotiation are more willing to be cooperative in future negotiations. D) Feeling bad about your performance during a negotiation tends to impair future negotiations. E) Individuals who do poorly in a negotiation develop positive perceptions of their counterpart. Answer: D Explanation: Displaying a negative emotion (such as anger) can be effective but feeling bad about your performance appears to impair future negotiations. Individuals who do poorly in a negative emotion, develop negative perceptions of their counterpart, and are less willing to share information or be cooperative in future negotiations. Anger should be used selectively in negotiation: angry negotiators who have less information or less power than their opponents have significantly worse outcomes. LO: 4.7: Apply concepts about emotions and moods to specific OB issues. Difficulty: Moderate Quest. Category: Concept
186 richard@qwconsultancy.com
103) Tiffany Crowe works as a showroom executive at a clothing boutique in London. Today, she is in a positive mood and instantly greets and smiles at a customer who walks into the store. The customer feels welcomed by her response. While browsing the store, Crowe helps the customer with products and passes on her positive mood to the customer. The customer leaves the store feeling content with a smile on her face. Which of the following best describes this situation? A) Emotional contagion B) Positivity offset C) Cognitive dissonance D) Framing effect E) Bandwagon effect Answer: A Explanation: Emotional contagion refers to the process by which people's emotions are caused by the emotions of others. For instance, employees' emotions can transfer to the customer. The primary explanation is that when someone experiences positive emotions and laughs and smiles at you, you tend to respond positively. Emotional contagion is important because customers who catch the positive moods or emotions of employees shop longer. LO: 4.7: Apply concepts about emotions and moods to specific OB issues. AACSB: Analytical thinking Difficulty: Hard Employability Skills: Knowledge Application and Analysis Quest. Category: Application 104) Green Earth, an environmental organization in Oklahoma, recently recruited Phyllis Galvan as a project coordinator. Galvan's manager has noticed that ever since she has joined, she has been extremely positive about her work and takes up challenges without complaining. In addition, she maintains a positive ambience by helping colleagues and smiling at everyone she interacts with. Her behavior has had a positive effect on her colleagues, who have started behaving in a similar manner. Which of the following concepts is demonstrated in this example? A) Emotional dissonance B) Ben Franklin effect C) Unit bias D) Anchoring effect E) Emotional contagion Answer: E Explanation: Emotional contagion refers to the process by which people's emotions are caused by the emotions of others. LO: 4.7: Apply concepts about emotions and moods to specific OB issues. AACSB: Analytical thinking Difficulty: Moderate Employability Skills: Knowledge Application and Analysis Quest. Category: Application
187 richard@qwconsultancy.com
105) When people behave in ways that violate established norms and threaten the organization, its members, or both, we say that they are engaged in ________. A) counterproductive work behaviors B) social alienation behaviors C) role engulfment D) behavioral contagion E) behavioral modernity Answer: A Explanation: People often behave in ways that violate established norms and threaten the organization, its members, or both. Such actions are called counterproductive work behaviors. For instance, an envious employee could backstab another employee, negatively distort others' successes, and positively distort his/her own accomplishments. LO: 4.7: Apply concepts about emotions and moods to specific OB issues. Difficulty: Easy Quest. Category: Concept 106) Jonathan Cowan is upset because he was passed over for a promotion he had really worked hard for. In response to his anger toward the manager and the company, he has started gossiping about the manager with his colleagues and assigning blame on him unnecessarily. Jonathan's behavior is an example of ________. A) deviant workplace behavior B) distinction bias C) cognitive dissonance D) positivity offset E) emotional dissonance Answer: A Explanation: People often behave in ways that violate established norms and threaten the organization, its members, or both. Such actions are called workplace deviant behaviors. For instance, an envious employee could backstab another employee, negatively distort others' successes, and positively distort his/her own accomplishments. LO: 4.7: Apply concepts about emotions and moods to specific OB issues. AACSB: Analytical thinking Difficulty: Hard Employability Skills: Knowledge Application and Analysis Quest. Category: Application
188 richard@qwconsultancy.com
107) Which of the following is an example of counterproductive behavior? A) Sabotaging one's organization's computers B) Taking blame for not achieving a deadline C) Putting forward one's views in a brainstorming session D) Working during the weekend to make up for backlog E) Requesting a two-day leave during the Christmas season Answer: A Explanation: People often behave in ways that violate established norms and threaten the organization, its members, or both. Such actions are called counterproductive work behaviors. For instance, an envious employee could backstab another employee, sabotage the organization's computers, negatively distort others' successes, and positively distort his/her own accomplishments. LO: 4.7: Apply concepts about emotions and moods to specific OB issues. AACSB: Analytical thinking Difficulty: Hard Quest. Category: Concept 108) Most managers agree that the evidence on EI indicates that it should not be a factor in hiring employees, especially for jobs that demand a high degree of social interaction. Answer: FALSE Explanation: One implication from the evidence on EI is that employees should consider it a factor in hiring employees, especially for jobs that demand a high degree of social interaction. LO: 4.7: Apply concepts about emotions and moods to specific OB issues. Difficulty: Easy Quest. Category: Concept 109) People in good moods tend to be more creative than people in bad moods. Answer: TRUE Explanation: Supervisors should actively try to keep employees happy because doing so creates more good moods, which in turn lead people to be more creative. LO: 4.7: Apply concepts about emotions and moods to specific OB issues. Difficulty: Easy Quest. Category: Concept 110) Leaders are perceived as more effective when they share positive emotions and followers are more creative in a positive emotional environment. Answer: TRUE Explanation: Leaders are perceived as more effective when they share positive emotions, and followers are more creative when they are in a positive environment. LO: 4.7: Apply concepts about emotions and moods to specific OB issues. Difficulty: Moderate Quest. Category: Concept
189 richard@qwconsultancy.com
111) Explain how managers can influence moods. Answer: Managers can use humor and give their employees small tokens of appreciation for work well done. Also, when leaders themselves are in good moods, group members are more positive, and as a result they cooperate more. Finally, selecting positive team members can have a contagion effect because positive moods transmit from team member to team member. One study of professional cricket teams found players' happy moods affected the moods of their team members and positively influenced their performance. Thus, it proves useful for managers to select team members predisposed to experience positive moods. LO: 4.7: Apply concepts about emotions and moods to specific OB issues. AACSB: Interpersonal relations and teamwork Difficulty: Moderate Quest. Category: Concept Organizational Behavior, 19e (Robbins/Judge) Chapter 5 Personality and Individual Differences 1) According to John Holland's personality-job fit theory, which job is most suitable for people belonging to the realistic type? A) Economist B) Teacher C) Mechanic D) Lawyer E) Biologist Answer: C Explanation: Realistic personalities prefer physical activities that require skill, strength, and coordination. They thrive in jobs such as mechanics, drill press operators, assembly-line workers, and farmers. LO: 5.1: Describe the differences between person-job fit and person-organization fit. Difficulty: Moderate Quest. Category: Concept 2) Help Aid, a non-profit organization in Indonesia, works at providing education in underdeveloped countries. They are now looking to hire a teacher who can educate children in Namibia and prepare them for jobs which will enable them to sustain themselves. According to Holland's personality-job fit theory, people belonging to which of the following personality types will best suit Help Aid's requirement? A) Conventional B) Realistic C) Enterprising D) Artistic E) Social Answer: E Explanation: According to Holland's personality-job fit theory, social personalities are friendly, sociable, cooperative, and understanding. Careers for social personalities include social worker, teacher, and counselor among others. LO: 5.1: Describe the differences between person-job fit and person-organization fit. AACSB: Analytical thinking 190 richard@qwconsultancy.com
Difficulty: Hard Employability Skills: Knowledge Application and Analysis Quest. Category: Application
191 richard@qwconsultancy.com
3) According to John Holland's personality-job fit theory, people belonging to the ________ type prefer verbal activities in which there are opportunities to influence others and attain powers. A) realistic B) conventional C) artistic D) enterprising E) investigative Answer: D Explanation: With reference to John Holland's personality-job fit theory, people belonging to the enterprising type prefer verbal activities in which there are opportunities to influence others and attain powers. LO: 5.1: Describe the differences between person-job fit and person-organization fit. Difficulty: Moderate Quest. Category: Concept 4) Based on the personality-job-fit theory, people belonging to the investigating type prefer ________. A) activities that involve helping and developing others B) activities that involve thinking, organizing, and understanding C) physical activities that require skill, strength, and coordination D) ambiguous and imaginative activities that allow creative expression E) activities in which there are opportunities to influence others Answer: B Explanation: With reference to John Holland's personality-job fit theory, people belonging to the investigating type prefer activities that involve thinking, organizing, and understanding. LO: 5.1: Describe the differences between person-job fit and person-organization fit. Difficulty: Easy Quest. Category: Concept 5) Which of the following is one of the personality types in John Holland's personality-job fit theory? A) Analytic B) Imaginative C) Practical D) Investigative E) Intuitive Answer: D Explanation: John Holland's personality-job fit theory presents six personality types. These types are realistic, investigative, social, conventional, enterprising, and artistic. LO: 5.1: Describe the differences between person-job fit and person-organization fit. Difficulty: Easy Quest. Category: Concept
192 richard@qwconsultancy.com
6) The person-organization fit theory suggests that people are attracted to and selected by organizations that match their values and leave when there is no compatibility. Answer: TRUE Explanation: The person-organization fit theory suggests that people are attracted to and selected by organizations that match their values and leave when there is no compatibility. LO: 5.1: Describe the differences between person-job fit and person-organization fit. Difficulty: Moderate Quest. Category: Concept 7) Following person-organization fit guidelines when hiring should result in higher employee satisfaction and reduced turnover. Answer: TRUE Explanation: Person-organization fit essentially means that people are attracted to and are selected by organizations that match their values. Following these guidelines when hiring should yield employees who fit better with the organization, which should, in turn, result in higher employee satisfaction and reduced turnover. LO: 5.1: Describe the differences between person-job fit and person-organization fit. Difficulty: Moderate Quest. Category: Concept 8) According to John Holland's personality-job fit theory, individuals belonging to the realistic type are persistent, stable, and genuine. Answer: TRUE Explanation: According to John Holland's personality-job fit theory, individuals belonging to the realistic type are shy, genuine, stable, conforming, practical, and persistent. LO: 5.1: Describe the differences between person-job fit and person-organization fit. Difficulty: Easy Quest. Category: Concept 9) According to John Holland's personality-job fit theory, individuals belonging to the conventional type prefer ambiguous activities that allow creative expression. Answer: FALSE Explanation: According to John Holland's personality-job fit theory, individuals belonging to the conventional type prefer rule-regulated, orderly, and unambiguous activities. LO: 5.1: Describe the differences between person-job fit and person-organization fit. Difficulty: Easy Quest. Category: Concept 10) Person-group fit is important in team settings. Answer: TRUE Explanation: Person-group fit is important in team settings, where the dynamics of team interactions significantly affect work outcomes. LO: 5.1: Describe the differences between person-job fit and person-organization fit. AACSB: Interpersonal relations and teamwork Difficulty: Easy Quest. Category: Concept 193 richard@qwconsultancy.com
11) What is John Holland's personality-job fit theory? Answer: The effort to match job requirements with personality characteristics is best articulated in John Holland's personality-job fit theory. Holland presents six personality types and proposes that satisfaction and the propensity to leave a position depend on how well individuals match their personalities to a job. Holland developed the Vocational Preference Inventory questionnaire, which contains 160 occupational titles. Respondents indicate which they like or dislike, and their answers form personality profiles. The personality-job fit theory describes six personality types. These types are: a) Realistic: Prefers physical activities that require skill, strength, and coordination b) Investigative: Prefers activities that involve thinking, organizing, and understanding c) Social: Prefers activities that involve helping and developing others d) Conventional: Prefers rule-regulated, orderly, and unambiguous activities e) Enterprising: Prefers verbal activities in which there are opportunities to influence others and attain power f) Artistic: Prefers ambiguous and unsystematic activities that allow creative expression The key points of this model are that (1) there do appear to be intrinsic differences in personality among individuals, (2) there are different types of jobs, and (3) people in jobs congruent with their personality should be more satisfied and less likely to voluntarily resign than people in incongruent jobs. LO: 5.1: Describe the differences between person-job fit and person-organization fit. Difficulty: Moderate Quest. Category: Concept 12) What is person-environment fit? Answer: All dimensions of fit are sometimes broadly referred to as person-environment fit. Each dimension can predict work attitudes, which are partially based on culture. A meta-analysis of person-environment fit in East Asia, Europe, and North America suggested that the dimensions of person-organization and person-job fit are the strongest predictors of positive work attitudes and performance in North America. These dimensions are important to a lesser degree in Europe, and they are least important in East Asia. LO: 5.1: Describe the differences between person-job fit and person-organization fit. Difficulty: Moderate Quest. Category: Synthesis
194 richard@qwconsultancy.com
13) Which of the following is true about how we speak of peoples' personalities? A) We tend to use the same small set of adjectives to describe how people act. B) A recent survey revealed a succinct set of 112 adjectives to describe people they know. C) We tend to use many adjectives to describe how people act and seem to think. D) A recent survey revealed just 211 adjectives to describe people they know. E) We tend to describe personality traits using approximately 55 adjectives. Answer: C Explanation: When we speak of someone's personality, we use many adjectives to describe how they act and seem to think; in fact, participants in a recent study used 624 distinct adjectives to describe people they knew. LO: 5.2: Describe personality, the way it is measured, and the factors that shape it. Difficulty: Easy Quest. Category: Concept 14) Which of the following is true about personality assessments used in organizations? A) Personality assessments are used by approximately 10 percent of all large U.S. companies. B) Personality assessments have been decreasingly used in diverse organizational settings. C) Personality assessments have been increasingly used in diverse organizational settings. D) Personality assessments have become increasingly expensive and thus, have slightly lost favor. E) Personality assessments are used by approximately 25 percent of all large U.S. companies. Answer: C Explanation: Personality assessments have been increasingly used in diverse organizational settings. In fact, eight of the top 10 U.S. private companies and 57 percent of all large U.S. companies use them. LO: 5.2: Describe personality, the way it is measured, and the factors that shape it. AACSB: Diverse and multicultural work environments Difficulty: Easy Quest. Category: Concept 15) What is the total number of ways in which an individual reacts to and interacts with the world around them? A) Talent B) Skill C) Knowledge D) Heredity E) Personality Answer: E Explanation: Personality is the total number of ways in which an individual reacts to and interacts with the world around them LO: 5.2: Describe personality, the way it is measured, and the factors that shape it. Difficulty: Easy Quest. Category: Concept
195 richard@qwconsultancy.com
16) Esther Lugo has gone for an interview at an advertising firm in Manhattan and has been asked to complete a self-report survey to help interviewers understand if she is the right candidate for the job. From the interview, they have found that she is extroverted, empathic, scrupulous, and cooperative in nature, which are key characteristics needed for the job. These characteristics about Lugo indicate her ________. A) talent B) skill C) knowledge D) genealogy E) personality Answer: E Explanation: Personality is the sum of ways in which an individual reacts to and interacts with others. LO: 5.2: Describe personality, the way it is measured, and the factors that shape it. AACSB: Analytical thinking Difficulty: Moderate Employability Skills: Knowledge Application and Analysis Quest. Category: Application 17) The most common means of measuring personality is through ________. A) in-depth interviews B) self-report surveys C) career portfolio D) reference interviews E) stress interviews Answer: B Explanation: The most common means of measuring personality is through self-report surveys. In such surveys, individuals evaluate themselves on a series of factors, such as "I worry a lot about the future." LO: 5.2: Describe personality, the way it is measured, and the factors that shape it. Difficulty: Easy Quest. Category: Concept 18) Which of the following is a drawback of self-report surveys? A) They can hold only a limited number of questions. B) They involve a large number of people in the evaluation process. C) They may have biased results if the surveyor's judgment is biased. D) They comprise a large number of complex statistical data. E) They may result in the respondent rating themselves better to create a good impression. Answer: E Explanation: The most common means of measuring personality is through self-report surveys. In general, when people know their personality scores are going to be used for hiring decisions, they rate themselves as about half a standard deviation more conscientious and emotionally stable than if they are taking the test to learn about themselves. LO: 5.2: Describe personality, the way it is measured, and the factors that shape it. Difficulty: Moderate 196 richard@qwconsultancy.com
Quest. Category: Concept 19) Which type of survey involves a co-worker or another observer to conduct an independent assessment of personality? A) Observer-ratings surveys B) Paid surveys C) Knowledge surveys D) Deviation surveys E) Cadastral surveys Answer: A Explanation: Observer-ratings surveys provide an independent assessment of personality. In such surveys, a co-worker or another observer does the rating. LO: 5.2: Describe personality, the way it is measured, and the factors that shape it. Difficulty: Easy Quest. Category: Concept 20) Yolanda Jones has been asked to evaluate the behavior of one her team members, Stephanie Sloan. Stephanie is being considered for a promotion, and human resources wants to ensure she will fit in with the new team. Among the questions being asked are whether Stephanie is free from distractions and whether she pays enough attention to her job. The evaluation Yolanda is being asked to conduct can best be described as a(n) ________. A) observer-ratings survey B) informational interview C) knowledge survey D) participant observation E) focus group survey Answer: A Explanation: Observer-ratings surveys provide an independent assessment of personality. In such surveys, a co-worker or another observer does the rating. LO: 5.2: Describe personality, the way it is measured, and the factors that shape it. AACSB: Interpersonal relations and teamwork Difficulty: Moderate Employability Skills: Knowledge Application and Analysis Quest. Category: Application 21) ________ refers to enduring characteristics that describe an individual's behavior. A) Social identity B) Personality trait C) Behavioral contagion D) Self-concordance E) Emotional labor Answer: B Explanation: Personality traits refer to enduring characteristics that describe an individual's behavior. LO: 5.2: Describe personality, the way it is measured, and the factors that shape it. Difficulty: Easy 197 richard@qwconsultancy.com
Quest. Category: Concept 22) Which of the following statements is true about personality? A) Personality is free from the influence of the environment. B) Personality remains constant over time. C) Personality can be measured solely through personal interviews. D) Personality can be described in terms of the measurable traits a person exhibits. E) Parents highly influence the development of their offspring's personality. Answer: D Explanation: Personality is the sum of ways in which an individual reacts to and interacts with others. Personality can be described in terms of the measurable traits a person exhibits. LO: 5.2: Describe personality, the way it is measured, and the factors that shape it. Difficulty: Moderate Quest. Category: Concept 23) An advantage of self-report surveys is the level of accuracy. Answer: FALSE Explanation: A drawback of self-report surveys is the level of accuracy. For instance, a good candidate may be in a bad mood when taking the survey, and that will make the score less accurate. LO: 5.2: Describe personality, the way it is measured, and the factors that shape it. Difficulty: Easy Quest. Category: Concept 24) As compared to observer-ratings surveys, self-reporting surveys are a better predictor of success on the job. Answer: FALSE Explanation: Research suggests that observer-ratings surveys are a better predictor of success on the job. LO: 5.2: Describe personality, the way it is measured, and the factors that shape it. Difficulty: Easy Quest. Category: Concept 25) Artificial intelligence has improved the ability to score personality tests and reduce faking. Answer: TRUE Explanation: Modern advancements in technology and artificial intelligence have improved the ability to score personality tests, reduce faking, and adaptively present items for more accurate personality assessment. LO: 5.2: Describe personality, the way it is measured, and the factors that shape it. Difficulty: Moderate Quest. Category: Concept
198 richard@qwconsultancy.com
26) Compare and contrast the two main means of measuring personality. Answer: The most common means of measuring personality is through self-report surveys, with which individuals evaluate themselves on a series of factors. On the other hand, observer-rating surveys provide an independent assessment of personality. An analysis of a large number of observer-reported personality surveys shows that a combination of self-reports surveys and observer-reports predict performance better than any other type of information. LO: 5.2: Describe personality, the way it is measured, and the factors that shape it. Difficulty: Moderate Quest. Category: Concept 27) How has artificial intelligence (AI) changed the way personality tests are scored? Answer: Modern advancements in technology and artificial intelligence have improved the ability to score personality tests, reduce faking, and adaptively present items for more accurate personality assessment. For example, machine learning has been used to select the most informative questions for traditional personality tests, detecting people who fake personality tests, measuring personality through the actual language and words people use on social media, and even for scoring job applicants' personal essays. LO: 5.2: Describe personality, the way it is measured, and the factors that shape it. Difficulty: Moderate Quest. Category: Concept 28) Which of the following is true of the psychopathy dimension of the Dark Triad? A) It denotes clinical mental illness. B) It refers to a tendency for a lack of concern for others and a lack of guilt or remorse when actions cause harm. C) Individuals scoring high on this dimension like to be the center of attention. D) Individuals scoring high on this dimension have a grandiose sense of self-importance. E) It refers to the degree to which an individual is pragmatic, maintains social distance, and believes that ends can justify means. Answer: B Explanation: The psychopathy dimension of the Dark Triad refers to a tendency for a lack of concern for others and a lack of guilt or remorse when actions cause harm It does not denote clinical mental illness. Individuals scoring high on narcissism like to be the center of attention and have a grandiose sense of self-importance, Machiavellianism refers to the degree to which an individual is pragmatic, maintains social distance, and believes than ends can justify means. LO: 5.3: Describe the strengths and weaknesses of the Myers-Briggs Type Indicator (MBTI) personality framework, the Big Five Model, and the Dark Triad. Difficulty: Moderate Quest. Category: Concept
199 richard@qwconsultancy.com
29) Which personality assessment consists of 100 questions where respondents are classified as extraverted or introverted, sensing or intuitive, thinking or feeling, and judging or perceiving? A) Myers-Briggs Type Indicator B) DiSC C) Dark Triad D) Wonderlic Ability Test E) Trait Activation Theory Answer: A Explanation: The Myers-Briggs Type Indicator (MBTI) is the most widely used personality assessment instrument in the world. It is a one-hundred question personality test that asks people how they usually feel or act in particular situations. Respondents are classified as extraverted or introverted, sensing or intuitive, thinking or feeling, and judging or perceiving. LO: 5.3: Describe the strengths and weaknesses of the Myers-Briggs Type Indicator (MBTI) personality framework, the Big Five Model, and the Dark Triad. Difficulty: Easy Quest. Category: Concept 30) According to the Myers-Briggs Type Indicator (MBTI) classification, people belonging to the ________ type are practical and prefer routine and order and focus on details. A) sensing B) extraverted C) feeling D) perceiving E) intuitive Answer: A Explanation: According to the Myers-Briggs Type Indicator classification, people belonging to the sensing type are practical and prefer routine and order and focus on details. LO: 5.3: Describe the strengths and weaknesses of the Myers-Briggs Type Indicator (MBTI) personality framework, the Big Five Model, and the Dark Triad. Difficulty: Easy Quest. Category: Concept
200 richard@qwconsultancy.com
31) People belonging to the ________ classification in the Myers-Briggs Type Indicator (MBTI) are flexible and spontaneous. A) thinking B) judging C) introverted D) sensing E) perceiving Answer: E Explanation: According to the Myers-Briggs Type Indicator (MBTI) classification, people belonging to the perceiving type are flexible and spontaneous. Feeling types rely on their personal values and emotions while handling problems. Extraverted individuals are outgoing, sociable, and assertive. Introverts are quiet and shy. Thinking types use reason and logic to handle problems. LO: 5.3: Describe the strengths and weaknesses of the Myers-Briggs Type Indicator (MBTI) personality framework, the Big Five Model, and the Dark Triad. Difficulty: Easy Quest. Category: Concept 32) People belonging to the judging type in the Myers-Briggs Type Indicator (MBTI) classification ________. A) are quiet, shy and introverted B) rely on unconscious processes C) prefer their world to be ordered D) are flexible and spontaneous E) rely on personal values and emotions Answer: C Explanation: According to the Myers-Briggs Type Indicator (MBTI) classification, people belonging to the judging type want control and prefer their world to be ordered and structured. LO: 5.3: Describe the strengths and weaknesses of the Myers-Briggs Type Indicator (MBTI) personality framework, the Big Five Model, and the Dark Triad. Difficulty: Easy Quest. Category: Concept 33) Which of the following are characteristics of the intuitive type of people according to the Myers-Briggs Type Indicator (MBTI) classification? A) Prefer routine and order and focus on details B) Outgoing, sociable, and assertive in nature C) Rely on unconscious processes and look at the overall picture D) Use reason, rationality, and logic to handle problems and situations E) Rely on their personal values and emotions to make decisions Answer: C Explanation: According to the Myers-Briggs Type Indicator (MBTI) classification, people belonging to the intuitive type rely on unconscious processes and look at the "big picture." LO: 5.3: Describe the strengths and weaknesses of the Myers-Briggs Type Indicator (MBTI) personality framework, the Big Five Model, and the Dark Triad. Difficulty: Easy 201 richard@qwconsultancy.com
Quest. Category: Concept 34) Ellen Athers works as a communication executive at a travel house. She is known to be friendly with her colleagues and interacts with them regularly to build strong work relationships. She knows that her rapport with her co-workers is a crucial part of her work and invests time in these relationships. In addition, she is assertive in making decisions, and colleagues take her decisions seriously. Which of the following types is Ellen most likely to be characterized as according to the Myers-Briggs Type Indicator (MBTI) classification? A) Perceiving B) Brooder C) Extraverted D) Introverted E) Solitary Answer: C Explanation: According to the Myers-Briggs Type Indicator (MBTI) classification, extraverted people are outgoing, sociable, and assertive, whereas introverted people are quiet and shy. LO: 5.3: Describe the strengths and weaknesses of the Myers-Briggs Type Indicator (MBTI) personality framework, the Big Five Model, and the Dark Triad. AACSB: Analytical thinking Difficulty: Hard Employability Skills: Knowledge Application and Analysis Quest. Category: Application 35) Elaine Chamberlain works as a research executive at an environmental organization. Though her colleagues are helpful and friendly, because of her shy nature, she often eats her lunch at her desk and has limited interactions with them. She is glad that her nature of work does not require her to interact with her co-workers to a high extent. Which of the following types is Chamberlain most likely to be characterized as according to the Myers-Briggs Type Indicator (MBTI) classification? A) Social B) Introverted C) Exhibitionist D) Gregarious E) Extraverted Answer: B Explanation: According to the Myers-Briggs Type Indicator (MBTI) classification, extraverted people are outgoing, sociable, and assertive, whereas introverted people are quiet and shy. LO: 5.3: Describe the strengths and weaknesses of the Myers-Briggs Type Indicator (MBTI) personality framework, the Big Five Model, and the Dark Triad. AACSB: Analytical thinking Difficulty: Hard Employability Skills: Knowledge Application and Analysis Quest. Category: Application
202 richard@qwconsultancy.com
36) Valerie Sinclair, a climate campaigner at an environmental organization, invariably uses rationale to make decisions for project implementation. She believes that the right decisions can be made only through scrutiny and analysis. Each time she needs to make a decision, she weighs all options before taking action. Which of the following is a characteristic of Sinclair's personality type according to the Myers-Briggs Type Indicator (MBTI) classification? A) Intuitive B) Introverted C) Thinking D) Feeling E) Perceiving Answer: C Explanation: According to the Myers-Briggs Type Indicator (MBTI) classification, thinking types of people use reason and logic to handle problems. LO: 5.3: Describe the strengths and weaknesses of the Myers-Briggs Type Indicator (MBTI) personality framework, the Big Five Model, and the Dark Triad. AACSB: Analytical thinking Difficulty: Moderate Employability Skills: Knowledge Application and Analysis Quest. Category: Application 37) Amber Downing, a human resource executive, is in charge of the recruitment process in her organization. During the hiring process, she often needs to coordinate with prospective employees. The organizational process does not require her to inform employees who do not get through the selection process. However, she believes that it is rude not to give them an update. She thus makes sure that she sends them e-mails informing them about the selection process. Which of the following types is Downing characterized as according to the Myers-Briggs Type Indicator (MBTI) classification? A) Sensing B) Judging C) Thinking D) Feeling E) Introverted Answer: D Explanation: According to the Myers-Briggs Type Indicator (MBTI) classification, feeling types of people rely on their personal values and emotions. LO: 5.3: Describe the strengths and weaknesses of the Myers-Briggs Type Indicator (MBTI) personality framework, the Big Five Model, and the Dark Triad. AACSB: Analytical thinking Difficulty: Hard Employability Skills: Knowledge Application and Analysis Quest. Category: Application
203 richard@qwconsultancy.com
38) People who are classified as being introverted-intuitive-thinking-judging people on the Myers-Briggs Type Indicator (MBTI) are ________. A) visionaries B) organizers C) conceptualizers D) executors E) clarifiers Answer: A Explanation: According to the Myers-Briggs Type Indicator (MBTI) classification, people with an INTJ (introverted, intuitive, thinking, judging) personality type are visionaries with original minds and great drive. They are skeptical, critical, independent, determined, and often stubborn. LO: 5.3: Describe the strengths and weaknesses of the Myers-Briggs Type Indicator (MBTI) personality framework, the Big Five Model, and the Dark Triad. Difficulty: Moderate Quest. Category: Concept 39) According to the Myers-Briggs Type Indicator (MBTI), people with an ESTJ personality type are characterized as ________. A) irrational B) perplexed C) decisive D) disoriented E) emotional Answer: C Explanation: According to the Myers-Briggs Type Indicator (MBTI) classification, people with an ESTJ personality type are organizers. They are realistic, logical, analytical, and decisive and have a natural head for business or mechanics. LO: 5.3: Describe the strengths and weaknesses of the Myers-Briggs Type Indicator (MBTI) personality framework, the Big Five Model, and the Dark Triad. Difficulty: Moderate Quest. Category: Concept
204 richard@qwconsultancy.com
40) People with an ENTP personality type are most likely to be classified as ________ according to the Myers-Brigg Type Indicator (MBTI). A) illogical B) innovative C) intuitive D) skeptical E) stubborn Answer: B Explanation: According to the Myers-Briggs Type Indicator (MBTI) classification, people with an ENTP personality type are conceptualizers, innovative, individualistic, versatile, and attracted to entrepreneurial ideas. Such kinds of people tend to be resourceful in solving challenging problems but may neglect routine assignments. LO: 5.3: Describe the strengths and weaknesses of the Myers-Briggs Type Indicator (MBTI) personality framework, the Big Five Model, and the Dark Triad. AACSB: Analytical thinking Difficulty: Moderate Quest. Category: Concept 41) The "E or I" represents ________ in the Myers-Briggs Type Indicator. A) enthusiastic/ingenious B) emotional/impulsive C) extraverted/introverted D) emotive/illustrative E) empathetic/innovative Answer: C Explanation: In the Myers-Briggs Type Indicator, individuals are classified as extraverted or introverted (E or I), sensing or intuitive (S or N), thinking or feeling (T or F), and judging or perceiving (J or P). LO: 5.3: Describe the strengths and weaknesses of the Myers-Briggs Type Indicator (MBTI) personality framework, the Big Five Model, and the Dark Triad. Difficulty: Moderate Quest. Category: Concept 42) The classification of S versus N represents ________ in the Myers-Briggs Type Indicator. A) sensing/intuitive B) social/perceiving C) stable/negligent D) sympathetic/thinking E) shrewd/feeling Answer: A Explanation: In the Myers-Briggs Type Indicator, individuals are classified as extraverted or introverted (E or I), sensing or intuitive (S or N), thinking or feeling (T or F), and judging or perceiving (J or P). LO: 5.3: Describe the strengths and weaknesses of the Myers-Briggs Type Indicator (MBTI) personality framework, the Big Five Model, and the Dark Triad. Difficulty: Moderate 205 richard@qwconsultancy.com
Quest. Category: Concept 43) What is the major problem with the Myers-Briggs Type Indicator as a measure of personality? A) It has an excess of classifications, which tends to confuse a person. B) It forces a person to be categorized as either one type or another. C) It does not have a clear demarcation between different personality types. D) It tends to overemphasize intuitive traits over analytical traits. E) It indicates results related exclusively to job performance. Answer: B Explanation: A problem with the Myers-Briggs Type Indicator is that it forces a person into one type or another. For instance, if one is not introverted or extraverted, there is no in-between, though in reality people can be both extraverted and introverted to some degree. LO: 5.3: Describe the strengths and weaknesses of the Myers-Briggs Type Indicator (MBTI) personality framework, the Big Five Model, and the Dark Triad. Difficulty: Easy Quest. Category: Concept 44) Which of the following is a personality assessment model that describes five basic dimensions encompassing most of the significant variation in human personality, namely extraversion, agreeableness, conscientiousness, emotional stability, and openness to experience? A) Myers-Briggs Type Indicator B) Big Five model C) Wonderlic Ability Test D) Trait Activation Theory E) DiSC Answer: B Explanation: The Big Five model is a personality assessment model that describes five basic dimensions encompassing most of the significant variations in human personality, namely extraversion, agreeableness, conscientiousness, emotional stability, and openness to experience. LO: 5.3: Describe the strengths and weaknesses of the Myers-Briggs Type Indicator (MBTI) personality framework, the Big Five Model, and the Dark Triad. Difficulty: Easy Quest. Category: Concept
206 richard@qwconsultancy.com
45) The ________ dimension of the Big Five model refers to an individual's propensity to defer to others. A) conscientiousness B) agreeableness C) extraversion D) openness to experience E) emotional stability Answer: B Explanation: The agreeableness dimension refers to an individual's propensity to defer to others. Highly agreeable people are cooperative, warm, and trusting. People who score low on agreeableness are cold, disagreeable, and antagonistic. LO: 5.3: Describe the strengths and weaknesses of the Myers-Briggs Type Indicator (MBTI) personality framework, the Big Five Model, and the Dark Triad. Difficulty: Moderate Quest. Category: Concept 46) The ________ dimension of the Big Five model is a measure of reliability. A) extraversion B) agreeableness C) conscientiousness D) openness to experience E) emotional stability Answer: C Explanation: The conscientiousness dimension is a measure of reliability. A highly conscientious person is responsible, organized, dependable, and persistent. Those who score low on this dimension are easily distracted, disorganized, and unreliable. LO: 5.3: Describe the strengths and weaknesses of the Myers-Briggs Type Indicator (MBTI) personality framework, the Big Five Model, and the Dark Triad. Difficulty: Moderate Quest. Category: Concept 47) Which is true of a highly conscientious person according to the Big Five Model? The person is likely to be ________. A) distracted, disorganized, and unreliable B) calm, self-confident, and secure C) cold, disagreeable, and antagonistic D) responsible, organized, and dependable E) reserved, timid, and quiet Answer: D Explanation: The conscientiousness dimension is a measure of reliability. A highly conscientious person is responsible, organized, dependable, and persistent. LO: 5.3: Describe the strengths and weaknesses of the Myers-Briggs Type Indicator (MBTI) personality framework, the Big Five Model, and the Dark Triad. Difficulty: Moderate Quest. Category: Concept
207 richard@qwconsultancy.com
208 richard@qwconsultancy.com
48) According to the Big Five model, a person who scores ________ is easily distracted, disorganized, and unreliable. A) low on emotional stability B) low on openness to experience C) high on agreeableness D) high on extraversion E) low on conscientiousness Answer: E Explanation: According to the Big Five model, a person who scores low on conscientiousness is easily distracted, disorganized, and unreliable. LO: 5.3: Describe the strengths and weaknesses of the Myers-Briggs Type Indicator (MBTI) personality framework, the Big Five Model, and the Dark Triad. Difficulty: Moderate Quest. Category: Concept 49) The ________ dimension of the Big Five model indicates a person's ability to withstand stress. A) emotional stability B) extraversion C) openness to experience D) agreeableness E) conscientiousness Answer: A Explanation: The emotional stability dimension, often labeled by its converse, neuroticism, taps a person's ability to withstand stress. People with positive emotional stability tend to be calm, self-confident, and secure. Those with high negative scores tend to be nervous, anxious, depressed, and insecure. LO: 5.3: Describe the strengths and weaknesses of the Myers-Briggs Type Indicator (MBTI) personality framework, the Big Five Model, and the Dark Triad. Difficulty: Moderate Quest. Category: Concept 50) Which dimension of the Big Five model addresses a range of interests and fascination with novelty? A) Emotional stability B) Extraversion C) Openness to experience D) Agreeableness E) Conscientiousness Answer: C Explanation: The openness to experience dimension addresses a range of interests and fascination with novelty. Extremely open people are creative, curious, and artistically sensitive. LO: 5.3: Describe the strengths and weaknesses of the Myers-Briggs Type Indicator (MBTI) personality framework, the Big Five Model, and the Dark Triad. Difficulty: Moderate Quest. Category: Concept 209 richard@qwconsultancy.com
51) Marina Lyon works as a reservation executive at a travel and tourism company. Though her job requires her only to efficiently book flight tickets for customers, she has also opted to undergo training to learn the process of hotel reservations. In addition, every evening she reads travelogues to be aware of upcoming travel destinations and trends. She loves the industry she works in and is eager to learn as much as she can. Considering the information given in this case, which dimension of the Big Five model best describes Lyon's personality? A) Extraversion B) Agreeableness C) Conscientiousness D) Openness to experience E) Emotional stability Answer: D Explanation: The openness to experience dimension addresses a range of interests and fascination with novelty. Extremely open people are creative, curious, and artistically sensitive. Those at the other end of the openness category are conventional and find comfort in the familiar. LO: 5.3: Describe the strengths and weaknesses of the Myers-Briggs Type Indicator (MBTI) personality framework, the Big Five Model, and the Dark Triad. AACSB: Analytical thinking Difficulty: Hard Employability Skills: Knowledge Application and Analysis Quest. Category: Application 52) Which of the following has been found to be more important than other traits in the success of CEOs of private equity companies? A) Extraversion B) Agreeableness C) Conscientiousness D) Emotional stability E) Openness to experience Answer: C Explanation: Conscientiousness is the Big Five trait most consistently related to job performance. Conscientiousness in the form of persistence, attention to detail, and setting of high standards has been found to be more important than other traits for CEOs in private equity firms. LO: 5.3: Describe the strengths and weaknesses of the Myers-Briggs Type Indicator (MBTI) personality framework, the Big Five Model, and the Dark Triad. Difficulty: Moderate Quest. Category: Concept
210 richard@qwconsultancy.com
53) Which of the following is most likely to be a drawback of highly conscientious people according to the Big Five model? A) They engage in numerous risky behaviors, such as smoking and drinking. B) They tend to indulge in irresponsible behavior and be disorganized. C) They can be too deliberate and perfectionistic. D) They focus on learning instead of performing a job efficiently. E) They excessively emphasize being creative. Answer: C Explanation: Like any trait, conscientiousness has pitfalls. Extremely conscientious individuals can be too deliberate and perfectionistic, resulting in diminished happiness and performance, which includes task performance, safety performance, and OCB. They may also become too focused on their own work to help others in the organization. Finally, they are often less creative, especially artistically. LO: 5.3: Describe the strengths and weaknesses of the Myers-Briggs Type Indicator (MBTI) personality framework, the Big Five Model, and the Dark Triad. Difficulty: Moderate Quest. Category: Concept 54) Among the Big Five model traits, ________ is most strongly related to life satisfaction, job satisfaction, and reduced intentions to quit and burn out. A) extraversion B) agreeableness C) conscientiousness D) emotional stability E) openness to experience Answer: D Explanation: Of the Big Five traits, emotional stability is most strongly related to life satisfaction, job satisfaction, and reduced intentions to quit and burnout. People with high emotional stability can adapt to unexpected or changing demands in the workplace. LO: 5.3: Describe the strengths and weaknesses of the Myers-Briggs Type Indicator (MBTI) personality framework, the Big Five Model, and the Dark Triad. Difficulty: Moderate Quest. Category: Concept
211 richard@qwconsultancy.com
55) According to the Big Five Model, which of the following is a difference between highly agreeable and less agreeable people? A) Highly agreeable people tend to do better in interpersonally oriented jobs than less agreeable people. B) Highly agreeable people are less compliant and less rule abiding than less agreeable people. C) Highly agreeable people tend to be less satisfied in their jobs than less agreeable people. D) Highly agreeable people engage in less organization citizenship behavior than less agreeable people. E) Highly agreeable people have more tendency to engage in organizational deviance than less agreeable people. Answer: A Explanation: Agreeable individuals tend to do better in interpersonally oriented jobs such as customer service. They experience less family work conflict and are less susceptible to turnover. LO: 5.3: Describe the strengths and weaknesses of the Myers-Briggs Type Indicator (MBTI) personality framework, the Big Five Model, and the Dark Triad. Difficulty: Easy Quest. Category: Concept 56) Which of the Dark Triad personality traits refers to the tendency to be arrogant, have a grandiose sense of self-importance, require excessive admiration, and have a sense of entitlement? A) Asceticism B) Stoicism C) Cynicism D) Narcissism E) Machiavellianism Answer: D Explanation: The Dark Triad is a constellation of negative personality traits consisting of Machiavellianism, narcissism, and psychopathy. Narcissism refers to the tendency to be arrogant, have a grandiose sense of self-importance, require excessive admiration, and have a sense of entitlement. Evidence suggests that narcissists are more charismatic and thus more likely to emerge as leaders, and they may even display better psychological health. LO: 5.3: Describe the strengths and weaknesses of the Myers-Briggs Type Indicator (MBTI) personality framework, the Big Five Model, and the Dark Triad. Difficulty: Moderate Quest. Category: Concept
212 richard@qwconsultancy.com
57) During an annual review, Michel Godfrey made the following assertion: "When I look at myself and my performance, I see that what I have achieved is outstanding and something no one in the organization has the capacity to undertake. Surprisingly, it has not won me the admiration of my colleagues like it should have. I also believe that I do not just deserve a raise, but need one, since without me, let's face it, the place would simply fall apart." Which of the following personality traits best describes Michel's personality? A) Minimalism B) Monasticism C) Stoicism D) Narcissism E) Machiavellianism Answer: D Explanation: Michel is a narcissist. Narcissism is the tendency to be arrogant, have a grandiose sense of self-importance, require excessive admiration, and have a sense of entitlement. LO: 5.3: Describe the strengths and weaknesses of the Myers-Briggs Type Indicator (MBTI) personality framework, the Big Five Model, and the Dark Triad. AACSB: Analytical thinking Difficulty: Hard Employability Skills: Knowledge Application and Analysis Quest. Category: Application 58) Which of the following is true about narcissists? A) They go out of their way to be helpful to colleagues. B) They have an inferiority complex about themselves. C) They are more likely to emerge as leaders. D) They do not require admiration or importance of people. E) They are meek, timid, and shy in nature. Answer: C Explanation: In psychology, narcissism describes a person who has a grandiose sense of selfimportance, requires excessive admiration, has a sense of entitlement, and is arrogant. Because narcissists often want to gain the admiration of others and receive affirmation of their superiority, they tend to "talk down" to those who threaten them, treating others as if they were inferior. Narcissists also tend to be selfish and exploitive and believe others exist for their benefit. Evidence suggests that narcissists are more charismatic and thus more likely to emerge as leaders. LO: 5.3: Describe the strengths and weaknesses of the Myers-Briggs Type Indicator (MBTI) personality framework, the Big Five Model, and the Dark Triad. Difficulty: Moderate Quest. Category: Concept
213 richard@qwconsultancy.com
59) Which of the following differentiates introverts from extraverts? A) Introverts tend to be happier in their jobs in comparison to extraverts. B) Introverts express feelings more freely compared to extraverts. C) Introverts experience more positive emotions than extraverts. D) Introverts are generally more assertive than extraverts. E) Introverts tend to be more thoughtful and reserved than extraverts. Answer: E Explanation: As compared to extraverts, introverts tend to be more thoughtful, reserved, timid, and quiet. Extraverts perform better in jobs with significant interpersonal interaction. They are socially dominant, "take charge" people who are usually more assertive than introverts. Extraversion is a relatively strong predictor of leadership emergence and behaviors in groups. Extraverts also tend to have generally high job satisfaction and reduced burnout. LO: 5.3: Describe the strengths and weaknesses of the Myers-Briggs Type Indicator (MBTI) personality framework, the Big Five Model, and the Dark Triad. Difficulty: Moderate Quest. Category: Concept 60) People scoring high on the ________ dimension of the Big Five model are more likely to be socially dominant, "take charge" people than those who score low. A) agreeableness B) conscientiousness C) openness D) extraversion E) emotional stability Answer: D Explanation: Extraverts are more socially dominant, "take charge" people who are usually more assertive than introverts. LO: 5.3: Describe the strengths and weaknesses of the Myers-Briggs Type Indicator (MBTI) personality framework, the Big Five Model, and the Dark Triad. Difficulty: Easy Quest. Category: Concept 61) ________ indicates the degree to which a person is unemotional and pragmatic and believes that ends can justify means. A) Extraversion B) Machiavellianism C) Agreeableness D) Self-concordance E) Narcissism Answer: B Explanation: Machiavellianism refers to the degree to which an individual is pragmatic, maintains emotional distance, and believes that ends can justify means. LO: 5.3: Describe the strengths and weaknesses of the Myers-Briggs Type Indicator (MBTI) personality framework, the Big Five Model, and the Dark Triad. Difficulty: Easy Quest. Category: Concept 214 richard@qwconsultancy.com
62) The most widely used and best-known personality frameworks are the Myers-Briggs Type Indicator (MBTI) and the Big Five Personality model. Both describe a person's total personality through exploration of the facets of personality. Answer: TRUE Explanation: The most widely used and best-known personality frameworks are the MyersBriggs Type Indicator (MBTI) and the Big Five Personality model. Both describe a person's total personality through exploration of the facets of personality. LO: 5.3: Describe the strengths and weaknesses of the Myers-Briggs Type Indicator (MBTI) personality framework, the Big Five Model, and the Dark Triad. Difficulty: Easy Quest. Category: Concept 63) According to the Myers-Briggs Type Indicator (MBTI) classification, people belonging to the judging type prefer less structure and random new experiences. Answer: FALSE Explanation: According to the Myers-Briggs Type Indicator (MBTI) classification, people belonging to the judging type want control and prefer their world to be ordered and structured. LO: 5.3: Describe the strengths and weaknesses of the Myers-Briggs Type Indicator (MBTI) personality framework, the Big Five Model, and the Dark Triad. Difficulty: Easy Quest. Category: Concept 64) According to the Big Five model, a person who scores high on the openness to experience dimension is conventional and finds comfort in the familiar. Answer: FALSE Explanation: According to the Big Five model, a person who scores low on the openness to experience dimension tends to be conventional and finds comfort in the familiar. On the other hand, extremely open people are creative, curious, and artistically sensitive. LO: 5.3: Describe the strengths and weaknesses of the Myers-Briggs Type Indicator (MBTI) personality framework, the Big Five Model, and the Dark Triad. Difficulty: Easy Quest. Category: Concept 65) According to the Big Five Model, the emotional stability dimension refers to an individual's propensity to defer to others. Answer: FALSE Explanation: According to the Big Five model, the agreeableness dimension refers to an individual's propensity to defer to others. The emotional stability dimension taps a person's ability to withstand stress. LO: 5.3: Describe the strengths and weaknesses of the Myers-Briggs Type Indicator (MBTI) personality framework, the Big Five Model, and the Dark Triad. Difficulty: Moderate Quest. Category: Concept
215 richard@qwconsultancy.com
66) One problem with the MBTI is that the model forces a person into one type or another; that is, you're either introverted or extraverted. There is no in-between. Answer: TRUE Explanation: One problem with the MBTI is that the model forces a person into one type or another; that is, you're either introverted or extraverted. There is no in-between. LO: 5.3: Describe the strengths and weaknesses of the Myers-Briggs Type Indicator (MBTI) personality framework, the Big Five Model, and the Dark Triad. Difficulty: Easy Quest. Category: Concept 67) According to the Big Five model, extraverts tend to be more emotionally expressive than extraverts and have better interpersonal skills. Answer: TRUE Explanation: According to the Big Five model, extraverts tend to be more emotionally expressive than introverts and have better interpersonal skills. LO: 5.3: Describe the strengths and weaknesses of the Myers-Briggs Type Indicator (MBTI) personality framework, the Big Five Model, and the Dark Triad. Difficulty: Easy Quest. Category: Concept 68) Explain the various classifications used in the Myers-Briggs Type Indicator (MBTI) framework. Answer: The Myers-Briggs Type Indicator (MBTI) is a 100-question personality test that asks people how they usually feel or act in particular situations. On the basis of their answers, individuals are classified as extraverted or introverted (E or I), sensing or intuitive (S or N), thinking or feeling (T or F), and judging or perceiving (J or P). These classifications are defined as follows: a) Extraverted (E) versus Introverted (I): Extraverted individuals are outgoing, sociable, and assertive. Introverts are quiet and shy. b) Sensing (S) versus Intuitive (N): Sensing types are practical and prefer routine and order. They focus on details. Intuitives rely on unconscious processes and look at the "big picture." c) Thinking (T) versus Feeling (F): Thinking types use reason and logic to handle problems. Feeling types rely on their personal values and emotions. d) Judging (J) versus Perceiving (P): Judging types want control and prefer their world to be ordered and structured. Perceiving types are flexible and spontaneous. LO: 5.3: Describe the strengths and weaknesses of the Myers-Briggs Type Indicator (MBTI) personality framework, the Big Five Model, and the Dark Triad. Difficulty: Moderate Quest. Category: Concept
216 richard@qwconsultancy.com
69) Describe the personality traits of people who fit into each of the following Myers-Briggs Type Indicator categories: INTJ, ESTJ, and ENTP. Answer: Intuitive, thinking, introverted, and judging people (INTJ) are visionaries with original minds and great drive. They are skeptical, critical, independent, determined, and often stubborn. ESTJs are organizers—realistic, logical, analytical, and decisive. They have a natural head for business or mechanics. ENTP type of people are conceptualizers, innovative, individualistic, and attracted to entrepreneurial ideas. They tend to be resourceful in solving problems but may neglect routine assignments. LO: 5.3: Describe the strengths and weaknesses of the Myers-Briggs Type Indicator (MBTI) personality framework, the Big Five Model, and the Dark Triad. Difficulty: Moderate Quest. Category: Concept 70) What are the five key traits in the Big Five model? Describe each one. Answer: The five key traits in the Big Five personality model are as follows: a) Extraversion: The extraversion dimension captures an individual's comfort level with relationships. Extraverts tend to be gregarious, assertive, and sociable. Introverts tend to be reserved, timid, and quiet. b) Agreeableness: The agreeableness dimension refers to an individual's propensity to defer to others. Highly agreeable people are cooperative, warm, and trusting. People who score low on agreeableness are cold, disagreeable, and antagonistic. c) Conscientiousness: The conscientiousness dimension is a measure of reliability. A highly conscientious person is responsible, organized, dependable, and persistent. Those who score low on this dimension are easily distracted, disorganized, and unreliable. d) Emotional stability: The emotional stability dimension—often labeled by its converse, neuroticism—taps a person's ability to withstand stress. People with positive emotional stability tend to be calm, self-confident, and secure. Those with high negative scores tend to be nervous, anxious, depressed, and insecure. e) Openness to experience: The openness to experience dimension addresses a range of interests and fascination with novelty. Extremely open people are creative, curious, and artistically sensitive. Those at the other end of the openness category are conventional and find comfort in the familiar. LO: 5.3: Describe the strengths and weaknesses of the Myers-Briggs Type Indicator (MBTI) personality framework, the Big Five Model, and the Dark Triad. Difficulty: Moderate Quest. Category: Concept
217 richard@qwconsultancy.com
71) Explain how the conscientiousness dimension of the Big Five Model can be used to predict behavior at work. Answer: The Big Five conscientiousness dimension is a measure of reliability. A highly conscientious person is responsible, organized, dependable, and persistent. Those who score low on this dimension are easily distracted, disorganized, and unreliable. Employees who score higher in conscientiousness develop higher levels of job knowledge, probably because highly conscientious people learn more. Higher levels of job knowledge contribute to higher levels of job performance. It is important for managers and for front-line employees. Conscientiousness, in the form of persistence, attention to detail, and setting of high standards, is considered more important than any other personality trait for job performance. It is the Big Five model trait that is most consistently related to job performance. LO: 5.3: Describe the strengths and weaknesses of the Myers-Briggs Type Indicator (MBTI) personality framework, the Big Five Model, and the Dark Triad. Difficulty: Moderate Quest. Category: Concept 72) Ray Stoddard is a project leader for a nonprofit company working to protect the coastal environment in Wales. When he starts a new project, Ray likes to have a detailed plan of what the project will entail and take an orderly approach to achieving project goals. As team leader, Ray prefers that his team follow his lead and undertake their responsibilities according to his plan. Ray his committed to his work and team members applaud his efforts and never give up attitude. Which categories of the Big Five Model and the Myers-Brigg Type Indicator would Ray fit into? Explain. Answer: Ray's characteristic of preferring to follow structure and order categorizes him into the judging category of personality according to the Myers-Briggs Type Indicator. According to the Big Five model, Ray is a highly conscientious person. This is because he comes across as dependable, responsible, organized and persistent. Ray's colleagues find that he never gives up on challenges and is reliable in nature. LO: 5.3: Describe the strengths and weaknesses of the Myers-Briggs Type Indicator (MBTI) personality framework, the Big Five Model, and the Dark Triad. AACSB: Interpersonal relations and teamwork Difficulty: Hard Quest. Category: Synthesis
218 richard@qwconsultancy.com
73) Which of the following refers to bottom-line conclusions individuals have about their capabilities, competence, and worth as a person? A) Core self-evaluation B) Authoritarianism C) Self-monitoring D) Machiavellianism E) Agreeableness Answer: A Explanation: Core self-evaluations are bottom-line conclusions individuals have about their capabilities, competence, and worth as a person. People who have positive core self-evaluations like themselves and see themselves as effective, capable, and in control of their environment. Those with negative core self-evaluations tend to dislike themselves, question their capabilities, and view themselves as powerless over their environment. LO: 5.4: Discuss how the concepts of core self-evaluation (CSE), self-monitoring, and proactive personality contribute to the understanding of personality. Difficulty: Easy Quest. Category: Concept 74) Which of the following statements is true about people with positive core self-evaluations? A) They are committed to their goals and set ambitious goals. B) They view themselves as powerless over their environment. C) They tend to question their capabilities and capacities. D) They lack persistence to achieve goals. E) They tend to be disliked by superiors and co-workers. Answer: A Explanation: People with positive core self-evaluation set more ambitious goals, are more committed to their goals, and persist longer in attempting to reach these goals. They are more popular with co-workers than those with negative core self-evaluation. LO: 5.4: Discuss how the concepts of core self-evaluation (CSE), self-monitoring, and proactive personality contribute to the understanding of personality. Difficulty: Moderate Quest. Category: Concept
219 richard@qwconsultancy.com
75) Individuals scoring ________ have a strong ability to adjust their behavior to external, situational factors and can behave differently in different situations. A) low on openness B) high on narcissism C) low on agreeableness D) high on self-monitoring E) low on conscientiousness Answer: D Explanation: Self-monitoring refers to an individual's ability to adjust his/her behavior to external, situational factors. Individuals scoring high in self-monitoring show considerable adaptability in adjusting their behavior to external situational factors. They are highly sensitive to external cues and can behave differently in different situations. LO: 5.4: Discuss how the concepts of core self-evaluation (CSE), self-monitoring, and proactive personality contribute to the understanding of personality. Difficulty: Moderate Quest. Category: Concept 76) Which of the following is true of individuals scoring low in self-monitoring? A) They tend to display their true dispositions and attitudes in every situation. B) They are pragmatic, maintain emotional distance, and believe ends can justify means. C) They have a grandiose sense of self-importance, require excessive admiration, and are arrogant. D) They are easily distracted, disorganized, and unreliable. E) They tend to dislike themselves and view themselves as powerless over their environment. Answer: A Explanation: Individuals low in self-monitoring tend to display their true dispositions and attitudes in every situation; hence, there is high behavioral consistency between who they are and what they do. LO: 5.4: Discuss how the concepts of core self-evaluation (CSE), self-monitoring, and proactive personality contribute to the understanding of personality. Difficulty: Moderate Quest. Category: Concept
220 richard@qwconsultancy.com
77) Low self-monitors tend to ________ as compared to high self-monitors. A) be more adaptable to external situations B) pay closer attention to the behavior of others C) have low consistency between personalities and actions D) be less likely to emerge as leaders E) show less commitment to their organizations Answer: D Explanation: Evidence indicates high self-monitors pay closer attention to the behavior of others and are more capable of conforming than are low self-monitors. They also receive better performance ratings, are more likely to emerge as leaders, and show less commitment to their organizations. High self-monitoring managers tend to be more mobile in their careers. LO: 5.4: Discuss how the concepts of core self-evaluation (CSE), self-monitoring, and proactive personality contribute to the understanding of personality. Difficulty: Moderate Quest. Category: Concept Jonah is currently trying to decide whether or not to allow Kate to be part of a delegation that will represent their company at an international business expo. Apart from allowing the company to market its business to potential clients, the expo will give the members of the delegation a chance to meet and network with various industry professionals and gain valuable industry insights. 78) Which of the following, if true, would most weaken Kate's chances of being selected? A) Kate is socially inept and tends to display her true disposition and attitude in every situation. B) Kate behaves differently in different situations and occasionally presents striking contradictions between her public persona and her private self. C) Kate has a strong sense of independence and often gets work done without any support from others. D) Kate is often in agreement with others' ideas and opinions. E) Kate is gregarious and comfortable in changing contexts and ambiguous situations. Answer: A Explanation: If Kate is socially inept and displays her true disposition and attitude in every situation, then this would imply that she is a low self-monitor. Considering she cannot adjust her behavior to situational demands, she would not be a good choice for the team. If Kate is gregarious and comfortable in ambiguous situations, and if she gets work done without any support from others, then this would increase her chances of being selected. If Kate conforms to others' ideas and opinions, then this would suggest that she is high on agreeableness, which would increase her chances of getting selected. If she behaves differently in different situations, and occasionally presents striking contradictions between her public persona and her private self, then this would imply that she is a high self-monitor, and this would make her a suitable choice for the delegation. LO: 5.4: Discuss how the concepts of core self-evaluation (CSE), self-monitoring, and proactive personality contribute to the understanding of personality. AACSB: Interpersonal relations and teamwork Difficulty: Hard Quest. Category: Critical Thinking 221 richard@qwconsultancy.com
79) Which of the following, if true, would most strengthen Kate's chances of being selected? A) Kate generally considers herself to be better than her colleagues. B) Kate is a go-getter—she is prepared to go to any length to get ahead. C) Kate has a conventional mindset and focuses on being efficient at her work. D) Kate often conforms to others' ideas and opinions rather than come up with her own. E) Kate is enterprising and adept at interacting with diverse groups of individuals. Answer: E Explanation: If Kate is enterprising and adept at interacting with diverse groups of individuals, then this would strengthen her chances of being selected as, at the expo, she would have to market her company as well as meet with other industry professionals. If she considers herself to be better than her colleagues, then this would imply that she is narcissistic and hence, this would weaken her chances of being selected. If she is prepared to go to any length to get ahead, then this would imply that she is a high-Mach and might even act unethically; hence, this would not strengthen her position. If she has a conventional mindset and focuses on being efficient at her work, then this would imply that she is not very creative, which would weaken her chances of being selected. Lastly, if she often conforms to other's ideas and opinions rather than come up with her own, then this would imply that she is very agreeable but lacking in original thought, and hence this would weaken her chances of being selected. LO: 5.4: Discuss how the concepts of core self-evaluation (CSE), self-monitoring, and proactive personality contribute to the understanding of personality. AACSB: Diverse and multicultural work environments Difficulty: Hard Quest. Category: Critical Thinking 80) As the marketing director for company producing racing bikes and electric bikes, Phoebe Czup feels like she has hit the jackpot by combining her love of cycling with her career goals. Since joining the company, Phoebe has worked hard to reposition the brand as a lifestyle brand for high income individuals who enjoy the outdoors. Her efforts seem to be paying off. Sales are up almost 30 percent year on year and Phoebe is excited to see the impact of several new campaigns that are being run in coordination with high end hotels that target the same customer base. Which of the following statements best describe Phoebe? A) Phoebe is a low self-monitor. B) Phoebe is a narcissist. C) Phoebe scores high in Machiavellianism. D) Phoebe has a proactive personality. E) Phoebe is a low risk-taking person. Answer: D Explanation: People with proactive personalities identify opportunities, show initiative, take action, and persevere until meaningful change occurs. They create positive change in the environment regardless of constraints or obstacles. LO: 5.4: Discuss how the concepts of core self-evaluation (CSE), self-monitoring, and proactive personality contribute to the understanding of personality. AACSB: Analytical thinking Difficulty: Hard Employability Skills: Knowledge Application and Analysis 222 richard@qwconsultancy.com
Quest. Category: Application 81) Those with a ________ personality identify opportunities, show initiative, take action, and persevere until meaningful change occurs, compared to others who passively react to situations. A) high self-monitoring B) proactive C) high Mach D) type A E) narcissist Answer: B Explanation: Those with a proactive personality identify opportunities, show initiative, take action, and persevere until meaningful change occurs, compared to others who passively react to situations. LO: 5.4: Discuss how the concepts of core self-evaluation (CSE), self-monitoring, and proactive personality contribute to the understanding of personality. Difficulty: Easy Quest. Category: Concept 82) Which of the following statements is most likely to be true regarding people with proactive personalities? A) They react passively to situations. B) They tend to display their true dispositions and attitudes in every situation. C) They tend to be selfish and exploitive and believe others exist for their benefit. D) They are more creative. E) They are narcissistic in nature and require constant admiration. Answer: D Explanation: People with proactive personalities identify opportunities, show initiative, take action, and persevere until meaningful change occurs. They have higher levels of job performance and creativity, and do not need much oversight. LO: 5.4: Discuss how the concepts of core self-evaluation (CSE), self-monitoring, and proactive personality contribute to the understanding of personality. Difficulty: Moderate Quest. Category: Concept 83) People with positive core self-evaluations feel that they are powerless over their environment. Answer: FALSE Explanation: People with positive core self-evaluations like themselves and see themselves as capable, effective, and in control of their environment. In contrast, people with negative core self-evaluations dislike themselves and view themselves as powerless over the environment. LO: 5.4: Discuss how the concepts of core self-evaluation (CSE), self-monitoring, and proactive personality contribute to the understanding of personality. Difficulty: Easy Quest. Category: Concept
223 richard@qwconsultancy.com
84) What are the typical characteristics associated with a proactive personality? Answer: People with a proactive personality identify opportunities, show initiative, take action, and persevere until meaningful change occurs, compared to others who passively react to situations. Proactives create positive change in their environment, regardless of, or even in spite of, constraints or obstacles. Not surprisingly, they have many desirable behaviors that organizations covet. They are more likely than others to be seen as leaders and to act as change agents. Proactive individuals are more likely to be satisfied with work and help others more with their tasks, largely because they build more relationships with others. Proactives are also more likely to challenge the status quo or voice their displeasure when situations aren't to their liking. If an organization requires people with entrepreneurial initiative, proactives make good candidates; however, they're also more likely to leave an organization to start their own business. As individuals, proactives are more likely than others to achieve career success. They select, create, and influence work situations in their favor. They seek out job and organizational information, develop contacts in high places, engage in career planning, and demonstrate persistence in the face of career obstacles. LO: 5.4: Discuss how the concepts of core self-evaluation (CSE), self-monitoring, and proactive personality contribute to the understanding of personality. Difficulty: Moderate Quest. Category: Concept 85) What are the implications of core self-evaluation for diversity and employee effectiveness in different cultural contexts? Answer: Research suggests that employees with positive core self-evaluations tend to more effectively adapt to change in their careers and to international environments when working in a new culture. Although one's language proficiency and nationality set employees in multinational teams apart as "different" from the rest of the group, their core self-evaluations compensated for these differentiating factors, and they were able to become better leaders in their teams. LO: 5.4: Discuss how the concepts of core self-evaluation (CSE), self-monitoring, and proactive personality contribute to the understanding of personality. AACSB: Diverse and multicultural work environments Difficulty: Hard Quest. Category: Synthesis
224 richard@qwconsultancy.com
86) How do people with positive core self-evaluation perform on the job as compared to those with negative core self-evaluation? Answer: People who have positive core self-evaluations like themselves and see themselves as effective, capable, and in control of their environment. Those with negative core self-evaluations tend to dislike themselves, question their capabilities, and view themselves as powerless over their environment. Core self-evaluations relate to job satisfaction because people positive in this trait see more challenge in their job and actually attain more complex jobs. People with positive core self-evaluations perform better than others because they set more ambitious goals, are more committed to their goals, and persist longer in attempting to reach these goals. Such people also provide better customer service, are more popular co-workers, and have careers that both begin on better footing and ascend more rapidly over time. LO: 5.4: Discuss how the concepts of core self-evaluation (CSE), self-monitoring, and proactive personality contribute to the understanding of personality. Difficulty: Moderate Quest. Category: Concept 87) What is self-monitoring? Answer: Self-monitoring refers to an individual's ability to adjust his or her behavior to external, situational factors. Individuals high in self-monitoring show considerable adaptability in adjusting their behavior to external situational factors. They are highly sensitive to external cues and can behave differently in different situations. High self-monitors are capable of presenting striking contradictions between their public persona and their private self. Low self-monitors tend to display their true dispositions and attitudes in every situation; hence, there is high behavioral consistency between who they are and what they do. Evidence indicates high selfmonitors pay closer attention to the behavior of others and are more capable of conforming than are low self-monitors. They also receive better performance ratings, are more likely to emerge as leaders, and show less commitment to their organizations. In addition, high self-monitoring managers tend to be more mobile in their careers, receive more promotions (both internal and cross-organizational), and are more likely to occupy central positions in an organization. LO: 5.4: Discuss how the concepts of core self-evaluation (CSE), self-monitoring, and proactive personality contribute to the understanding of personality. Difficulty: Moderate Quest. Category: Concept
225 richard@qwconsultancy.com
Mara is looking to send one of her team members on an international assignment to China to train a group of employees located at her company's office in Beijing. As part of the assignment, the chosen employee will have to undergo a three-week diversity training program to help him/her learn about his/her new environment. Bryan, one of Mara's colleagues, is interested in taking up the assignment. 88) Which of the following, if true, would reduce the likelihood that Bryan will be selected? A) Bryan has had little exposure to Chinese culture. B) Bryan is proficient in Chinese. C) Bryan traveled to Beijing last year on a different assignment. D) Bryan does not cope very well in ambiguous situations. E) Bryan sometimes displays striking contradictions between his public persona and his private self. Answer: D Explanation: Of the Big Five traits, openness to experience would be most important to effectiveness in international assignments. Open people are more likely to be culturally flexible—to "go with the flow" when things are different in another country. If Bryan is uncomfortable in ambiguous situations, then he would most likely not be able to manage well in a foreign environment. If he is proficient in Chinese or has traveled to Beijing in the past, then it would only increase his chances of being selected. The fact that he has had little exposure to Chinese culture is not very relevant, as he will have to attend a diversity training program if he is chosen for the assignment. The fact that he sometimes displays striking contradictions between his public persona and his private self does not influence his chances of being selected. LO: 5.5: Describe how the situation affects whether personality predicts behavior. AACSB: Diverse and multicultural work environments Difficulty: Hard Employability Skills: Knowledge Application and Analysis Quest. Category: Critical Thinking
226 richard@qwconsultancy.com
89) Which of the following, if true, would increase the likelihood that Bryan will be selected? A) Bryan's application was rejected the last time he applied for an international assignment. B) Bryan has trained several employees in the last six months. C) Bryan has had little exposure to Chinese culture. D) Bryan displays striking contradictions between his public persona and his private self. E) Bryan is quite sociable and can "think on his feet." Answer: E Explanation: Of the Big Five traits, openness to experience would be most important to effectiveness in international assignments. Open people are more likely to be culturally flexible—to "go with the flow" when things are different in another country. If Bryan is sociable and can "think on his feet," then this would strengthen his chances of being selected. Just because his application was rejected the last time he applied for an international assignment does not mean he will be given a chance this time around. Similarly, it cannot be argued that he is the right person for this job just because he has trained people in the past. The fact that he sometimes displays striking contradictions between his public persona and his private self does not influence his chances of being selected. LO: 5.5: Describe how the situation affects whether personality predicts behavior. AACSB: Reflective thinking Difficulty: Hard Quest. Category: Critical Thinking 90) The degree to which norms, cues, or standards dictate appropriate behavior is known as consistency. Answer: FALSE Explanation: Situation-strength theory proposes that the way personality translates into behavior depends on the strength of the situation. Situation-strength refers to the degree to which norms, cues, or standards dictate appropriate behavior. LO: 5.5: Describe how the situation affects whether personality predicts behavior. Difficulty: Easy Quest. Category: Concept 91) Research suggests that personality traits better predict behavior in strong situations than in strong ones. Answer: FALSE Explanation: Strong situations pressure us to exhibit the right behavior, clearly show us what that behavior is, and discourage wrong behavior. In weak situations, "anything goes," and thus we are freer to express our personality in behavior. Thus, research suggests that personality traits better predict behavior in weak situations than in strong ones. LO: 5.5: Describe how the situation affects whether personality predicts behavior. Difficulty: Easy Quest. Category: Concept
227 richard@qwconsultancy.com
92) Karen Hashimoto is a senior nurse in a busy hospital emergency room in Chicago. Karen's job is stressful, but other nurses and doctors in the emergency room admire Karen's ability to remain calm and handle problems with ease. Those working with Karen point out that she is confident of her capabilities, sees herself as an effective manager, and recognizes that in her position she is also a role model for younger members of the staff. Describe Karen's personality in terms of the Big Five Model and core self-evaluation. Answer: Karen's capability to withstand stress and to remain calm, self-confident, and secure shows that she has positive emotional stability according to the Big Five model. She also has a positive core self-evaluation because she sees herself as a capable and effective manager. People with positive self-evaluations often perform better than others because they set ambitious goals, are more committed to their goals, and persist longer to reach these goals. LO: 5.5: Describe how the situation affects whether personality predicts behavior. AACSB: Analytical thinking Difficulty: Hard Quest. Category: Synthesis 93) Percy Collins works as a life advisor at an insurance firm and often looks for constant admiration from his manager every time he sells a policy. He also feels that he is the best at his work and no other advisor in his company has the ability or the skills to sell the number of policies that he can. During meetings and team-building exercises, colleagues often find him rude, distant, and antagonistic. What categories according to the Big Five model and other personality traits are Collins most likely to be characterized by? Answer: Collins is narcissistic. The fact that he has overconfidence in himself and looks for excessive admiration makes him a narcissist. Narcissists have a grandiose sense of selfimportance, require excessive admiration, have a sense of entitlement, and are arrogant in nature. Collins also scores low on agreeableness according to the Big Five Factor Model. His colleagues see him as rude, distant, and antagonistic. People low on agreeableness are known to be cold and disagreeable. LO: 5.5: Describe how the situation affects whether personality predicts behavior. AACSB: Analytical thinking Difficulty: Moderate Quest. Category: Synthesis 94) Will Redcliff works at a large advertising agency in New York City. Many of Will's colleagues find him to be difficult to work with, noting that he is always looking for admiration from his boss and boasting about his creative abilities. Some staff have even refused to attend meetings with Will complaining that he is rude, distant, and antagonistic. In the words of one colleague "Will is convinced that he is better than anyone in the company at doing his job"! How can Will's personality trait best be described? Answer: Will is narcissistic. The fact that he has overconfidence in himself and looks for excessive admiration makes him a narcissist. A narcissist has a grandiose sense of selfimportance, requires excessive admiration, has a sense of entitlement, and is arrogant in nature. LO: 5.5: Describe how the situation affects whether personality predicts behavior. AACSB: Analytical thinking Difficulty: Moderate Employability Skills: Knowledge Application and Analysis 228 richard@qwconsultancy.com
Quest. Category: Application 95) What are the personality characteristics of Machiavellianism? Answer: The personality characteristic of Machiavellianism is named after Niccolo Machiavelli, who wrote in the sixteenth century on how to gain and use power. An individual high in Machiavellianism is pragmatic, maintains emotional distance, and believes ends can justify means. A considerable amount of research has related high- and low-Mach personalities to behavioral outcomes. High Machs manipulate more, win more, are persuaded less, and persuade others more than do low Machs. Yet high-Mach outcomes are moderated by situational factors. High Machs flourish (1) when they interact face-to-face with others rather than indirectly; (2) when the situation has a minimal number of rules and regulations, allowing latitude for improvisation; and (3) when emotional involvement with details irrelevant to winning distracts low Machs. Thus, whether high Machs make good employees depends on the type of job. In jobs that require bargaining skills or that offer substantial rewards for winning, high Machs will be productive. LO: 5.5: Describe how the situation affects whether personality predicts behavior. Difficulty: Moderate Quest. Category: Concept 96) ________ refers to an overall factor of intelligence, as suggested by the positive correlations among specific intellectual ability dimensions. A) Inductive reasoning B) IQ C) Spatial visualization D) Deductive reasoning E) General mental ability Answer: E Explanation: General mental ability refers to an overall factor of intelligence, as suggested by the positive correlations among specific intellectual ability dimensions. LO: 5.6: Demonstrate the relevance of intellectual and physical abilities to OB. Difficulty: Moderate Quest. Category: Concept 97) A job that requires the ability to make rapid, repeated flexing movements requires ________. A) the ability to coordinate simultaneous actions of different parts of the body B) dynamic strength C) explosive strength D) dynamic flexibility E) body coordination Answer: D Explanation: A job that requires the ability to make rapid, repeated flexing movements requires dynamic flexibility. LO: 5.6: Demonstrate the relevance of intellectual and physical abilities to OB. Difficulty: Moderate Quest. Category: Concept
229 richard@qwconsultancy.com
230 richard@qwconsultancy.com
98) The ________ dimension of intellectual ability refers to the ability to identify visual similarities and differences quickly and accurately. A) verbal comprehension B) deductive reasoning C) spatial visualization D) perceptual speed E) memory Answer: D Explanation: The perceptual speed dimension of intellectual ability refers to the ability to identify visual similarities quickly and accurately. LO: 5.6: Demonstrate the relevance of intellectual and physical abilities to OB. Difficulty: Easy Quest. Category: Concept 99) Samantha Jeffries is hiring an accountant for her company. Samantha should look for someone with strong ________ abilities. A) memory B) deductive reasoning C) number aptitude D) inductive reasoning E) verbal comprehension Answer: C Explanation: Samantha should hire someone with strong number aptitude abilities. Number aptitude refers to the ability to do speedy and accurate arithmetic. LO: 5.6: Demonstrate the relevance of intellectual and physical abilities to OB. Difficulty: Moderate Employability Skills: Knowledge Application and Analysis Quest. Category: Concept 100) The Wonderlic Ability Test is a twelve minute physical abilities test used by companies hiring manual labor. Answer: FALSE Explanation: The intelligence test most widely used in hiring decisions is the Wonderlic Ability Test. The test takes twelve minutes to complete and has 50 questions. LO: 5.6: Demonstrate the relevance of intellectual and physical abilities to OB. Difficulty: Moderate Quest. Category: Concept 101) An individual's capacity to perform the various tasks in a job is known as general mental ability. Answer: FALSE Explanation: An individual's capacity to perform the various tasks in a job is known as ability LO: 5.6: Demonstrate the relevance of intellectual and physical abilities to OB. Difficulty: Easy Quest. Category: Concept
231 richard@qwconsultancy.com
232 richard@qwconsultancy.com
102) What are the dimensions of intellectual ability and how do they relate to general mental ability? Answer: The seven most frequently cited dimensions making up intellectual abilities are number aptitude, verbal comprehension, perceptual speed, inductive reasoning, deductive reasoning, spatial visualization, and memory. These dimensions are positively correlated, so if you score high on verbal comprehension for example, then you are more likely to score high on spatial visualization. The correlations are high enough that researchers also recognize that intelligence takes on an overall, global form as general mental ability. LO: 5.6: Demonstrate the relevance of intellectual and physical abilities to OB. Difficulty: Moderate Quest. Category: Concept 103) ________ describes basic convictions that "a specific mode of conduct or end-state of existence is personally or socially preferable to an opposite mode of conduct". A) Values B) Attitudes C) Emotions D) Feelings E) Traditions Answer: A Explanation: Values represent basic convictions that a specific mode of conduct or end-state of existence is personally or socially preferable to an opposite or converse mode of conduct or endstate of existence. Values contain a judgmental element in that they carry an individual's ideas as to what is right, good, or desirable. LO: 5.7: Contrast terminal and instrumental values. Difficulty: Easy Quest. Category: Concept 104) When we rank an individual's values in order of their ________, we obtain the person's value system. A) intensity B) content C) context D) fluidity E) flexibility Answer: A Explanation: A value system is defined as a hierarchy based on a ranking of an individual's values in terms of their intensity. LO: 5.7: Contrast terminal and instrumental values. Difficulty: Easy Quest. Category: Concept
233 richard@qwconsultancy.com
105) Which of the following is true regarding values? A) They are void of a judgmental element. B) They are invariably fluid and flexible in nature. C) They have content and intensity attributes. D) They never change irrespective of external factors. E) They are always established in a person's later years. Answer: C Explanation: Values contain a judgmental element in that they carry an individual's ideas as to what is right, good, or desirable. Values have both content and intensity attributes. Values tend to be relatively stable and enduring. A significant portion of the values we hold is established in our early years. Values may change, but more often they are reinforced. LO: 5.7: Contrast terminal and instrumental values. Difficulty: Moderate Quest. Category: Concept 106) According to Milton Rokeach, values can be organized into two categories: ________ values and ________ values. A) instrumental; terminal B) fluid; stable C) flexible; essential D) unique; general E) flexible; enduring Answer: A Explanation: Milton Rokeach organized values into two categories: instrumental and terminal values. LO: 5.7: Contrast terminal and instrumental values. Difficulty: Easy Quest. Category: Concept 107) ________ values refer to desirable end-states. A) Essential B) Instrumental C) Critical D) Flexible E) Terminal Answer: E Explanation: According to Milton Rokeach, terminal values refer to desirable end-states. LO: 5.7: Contrast terminal and instrumental values. Difficulty: Easy Quest. Category: Concept
234 richard@qwconsultancy.com
108) Which values refer to preferable modes of behavior according to Milton Rokeach? A) Terminal B) Critical C) Instrumental D) Essential E) Flexible Answer: C Explanation: According to Milton Rokeach, instrumental values refer to preferable modes of behavior. LO: 5.7: Contrast terminal and instrumental values. Difficulty: Easy Quest. Category: Concept 109) According to Milton Rokeach ________ is a terminal value. A) personal discipline B) self-reliance C) goal-orientation D) prosperity E) self-improvement Answer: D Explanation: According to Milton Rokeach, terminal values refer to desirable end-states. These are the goals a person would like to achieve during his/her lifetime. Prosperity is an end-state goal. The other answers are means which a person could use to get to that goal. LO: 5.7: Contrast terminal and instrumental values. Difficulty: Easy Quest. Category: Concept 110) ________ an instrumental value according to Milton Rokeach. A) Economic success B) Social recognition C) Personal discipline D) World peace E) Meaning in life Answer: C Explanation: Instrumental values refer to preferable modes of behavior or means of achieving the terminal values. Personal discipline is an instrumental value, and all other options are terminal values. LO: 5.7: Contrast terminal and instrumental values. Difficulty: Easy Quest. Category: Concept
235 richard@qwconsultancy.com
111) As compared to other generations, baby boomers ________. A) lead lives shaped mainly by globalization B) are more questioning and entrepreneurial C) give high importance to hard work and loyalty D) give the highest importance to flexibility and life options E) reject success and ambition Answer: C Explanation: Dominant values for baby boomers include optimism, competition, teamwork, hard work, duty, and loyalty. LO: 5.7: Contrast terminal and instrumental values. Difficulty: Moderate Quest. Category: Concept 112) Alex was sent to Beijing to help local managers solve the problem of growing worker dissatisfaction at their manufacturing facility located in the city. As part of his visit, he decided to have a town hall meeting with the workers to understand the problems that they were facing and the reasons for their discontent. The turnout at the meeting was substantial; however, when asked for their opinions and suggestions, the crowd fell silent. As a result, Alex was unable to determine the reason for employee dissatisfaction. Which of the following, if true, best explains this situation? A) A small portion of the workers at the facility belong to the Millennial generation. B) Most of the workers at the meeting place a high value on teamwork, duty, and loyalty. C) The employees are genuinely concerned about improving their lot. D) Alex was sent to Beijing as he was one of the few employees who were conversant in the local language. E) On previous occasions, the company has yielded to employee demands. Answer: B Explanation: If most of the people at the meeting place a high value on teamwork, duty, and loyalty, then employees would be reluctant to do anything that would be considered an act of disloyalty, such as voicing their opinions. Hence, this would explain the employees' silence. If the employees were genuinely concerned about improving their lot, and the company had yielded to employee demands in the past, then the workers would have voiced their opinions. Additionally, if Alex were conversant in the local language, then this would imply that there were no language barriers between him and the workers and that they could understand him, yet they chose not to respond. It is irrelevant that a small portion of the workers at the facility were Millennials. LO: 5.7: Contrast terminal and instrumental values. AACSB: Diverse and multicultural work environments Difficulty: Hard Quest. Category: Critical Thinking
236 richard@qwconsultancy.com
113) Values contain a judgmental element in which they carry an individual's ideas as to what is right, good, or desirable. Answer: TRUE Explanation: Values represent basic convictions that a specific mode of conduct or end-state of existence is personally or socially preferable to an opposite or converse mode of conduct or endstate of existence. They contain a judgmental element in that they carry an individual's ideas as to what is right, good, or desirable. Values contain a judgmental element in which they carry an individual's ideas as to what is right, good, or desirable. LO: 5.7: Contrast terminal and instrumental values. Difficulty: Easy Quest. Category: Concept 114) An individual's value system is obtained by ranking the person's values in terms of their intensity. Answer: TRUE Explanation: Values represent basic convictions that a specific mode of conduct or end-state of existence is personally or socially preferable to an opposite or converse mode of conduct or endstate of existence. An individual's value system is obtained by ranking the person's values in terms of their intensity. LO: 5.7: Contrast terminal and instrumental values. Difficulty: Easy Quest. Category: Concept 115) Millennials value achievement and quality relationships. Answer: TRUE Explanation: Millennials, who entered the workforce from 2000 to 2015 value achievement, quality relationships, competition, open mindedness, responsibility, and uniqueness. LO: 5.7: Contrast terminal and instrumental values. Difficulty: Moderate Quest. Category: Concept 116) Using examples, compare and contrast terminal values and instrumental values according to the Rokeach value theory. Answer: Milton Rokeach created a system for organizing values. It consists of two sets of values. One set, called terminal values, refers to desirable end-states. These are the goals a person would like to achieve during his/her lifetime. Examples of terminal values are freedom and social recognition. The other set, called instrumental values, refers to preferable modes of behavior, or means of achieving the terminal values. Examples of instrumental values are kindness and goal-orientation. LO: 5.7: Contrast terminal and instrumental values. Difficulty: Moderate Quest. Category: Concept Organizational Behavior, 19e (Robbins/Judge) Chapter 6 Perception and Individual Decision Making 1) The process by which individuals organize and interpret their sensory impressions in order to 237 richard@qwconsultancy.com
give meaning to their environment is referred to as ________. A) sensation B) impression C) apprehension D) attribution E) perception Answer: E Explanation: Perception is defined as a process by which individuals organize and interpret their sensory impressions in order to give meaning to their environment. People's behavior is based on their perception of what reality is, and not on reality itself. What a person perceives can be substantially different from objective reality. LO: 6.1: Explain the factors that influence perception. Difficulty: Easy Quest. Category: Concept 2) Which of the following statements is true regarding perception? A) Perception of reality is independent of one's personality. B) Our perception of a target is not affected by the context of the situation in which the perception is made. C) Our perception of reality can be different from the objective reality. D) Our perception of reality is independent of our past experiences. E) We form a perception of a target by looking at it in isolation. Answer: C Explanation: Our perception of reality can be different from the objective reality. Our perception of reality is affected by our personality, past experiences, and the context of the situation in which the perception is made. We do not look at targets in isolation. LO: 6.1: Explain the factors that influence perception. Difficulty: Moderate Quest. Category: Concept
238 richard@qwconsultancy.com
3) When two people witness something at the same time and in the same situation yet interpret it differently, factors that operate to shape their perceptions reside in the ________. A) perceiver B) target C) timing D) context E) situation Answer: A Explanation: Since the time and situation are the same, the factors that operate to shape perception must be in the perceivers themselves. Personal characteristics that can affect perception include perceiver attitudes, personality, motives, interests, past experiences, and expectations. LO: 6.1: Explain the factors that influence perception. Difficulty: Moderate Quest. Category: Concept 4) Stan Humphries believes that anyone who voluntarily works the nightshift at the hospital must be single. He thinks that anyone who has children would prefer to work the day shift so that they can keep the same schedule as their children. What personal factor is most likely to be affecting Stan's perceptions of those who work the nightshift at the hospital? A) His financial background B) His expectations C) His interest D) His motive E) His personality Answer: B Explanation: Stan expects all nightshift workers to have children. His expectations are what color his perception. The example doesn't explain why he expects this, although it could be from a previous experience. LO: 6.1: Explain the factors that influence perception. AACSB: Analytical thinking Difficulty: Moderate Employability Skills: Knowledge Application and Analysis Quest. Category: Application
239 richard@qwconsultancy.com
5) Extremely attractive or unattractive individuals are most likely to be noticed in a group. Which of the following statements best describes the reason behind this? A) Our perception of reality depends on our past experiences. B) Our perception of reality depends on our personality. C) We don't look at targets in isolation. D) The time at which we see an object can influence our perception of the object. E) Our motives and expectations affect our perception of a target. Answer: C Explanation: Characteristics of the target we observe can affect what we perceive. Loud people are more likely to be noticed in a group than quiet ones. So, too, are extremely attractive or unattractive individuals. Because we don't look at targets in isolation, the relationship of a target to its background influences our perception of the target. LO: 6.1: Explain the factors that influence perception. Difficulty: Moderate Quest. Category: Concept 6) During team meetings, Amber Downing always notices that Rhona Law tends to ask many questions and suggest ideas at each discussion. However, Law stands out in the meetings only because she is the only one making suggestions. If both of them were part of team meetings where almost all members made suggestions and asked questions, Law would not have drawn as much attention from Downing. Which of the following factors has most likely influenced Downing's perception of Law? A) Expectation B) Interest C) Past experience D) Context E) Motive Answer: D Explanation: A number of factors operate to shape and sometimes distort perception. These factors can reside in the perceiver, in the object or target being perceived, or in the context of the situation in which the perception is made. LO: 6.1: Explain the factors that influence perception. AACSB: Analytical thinking Difficulty: Moderate Employability Skills: Knowledge Application and Analysis Quest. Category: Application
240 richard@qwconsultancy.com
7) Sandy Alexander thinks that people who use plastic water bottles rather than reusable metal bottles are wasteful and don't respect the environment. In her opinion, everyone needs to work together to protect the environment and those who use plastic water bottles are not doing their share. Which of the following factors has most likely influenced Sandy's perceptions of people who choose plastic water bottles rather than reusable metal bottles? A) Location B) Expectation C) Characteristic of the target D) Context E) Time Answer: B Explanation: A number of factors operate to shape and sometimes distort perception. These factors can reside in the perceiver, in the object or target being perceived, or in the context of the situation in which the perception is made. LO: 6.1: Explain the factors that influence perception. AACSB: Analytical thinking Difficulty: Moderate Employability Skills: Knowledge Application and Analysis Quest. Category: Application 8) ________ is a factor present in a target which may affect a person's perception. A) Attitude B) Motive C) Interest D) Novelty E) Experience Answer: D Explanation: Factors present in a target which may affect perception are novelty, motion, sound, size, background, proximity, and similarity. LO: 6.1: Explain the factors that influence perception. Difficulty: Easy Quest. Category: Concept 9) ________ is a factor present in a situation which may affect a person's perception. A) Similarity B) Size C) Expectation D) Time E) Experience Answer: D Explanation: Factors present in a situation which may affect perception are work setting, social setting, and time. LO: 6.1: Explain the factors that influence perception. Difficulty: Easy Quest. Category: Concept
241 richard@qwconsultancy.com
242 richard@qwconsultancy.com
10) Factors present in a perceiver which may affect perception are ________. A) interests B) similarities C) sounds D) proximities E) backgrounds Answer: A Explanation: Factors present in a perceiver which may affect perception are attitudes, motives, interests, experience, and expectations. LO: 6.1: Explain the factors that influence perception. Difficulty: Easy Quest. Category: Concept 11) Reggie Williams works on the set of a leading sitcom. Reggie's booming voice and outgoing manner always ensure that Reggie stands out in a crowd. Which of the following statements best explains the reason behind people noticing Reggie? A) Perception of reality depends on the perceiver's past experiences. B) Perception of reality depends on the perceiver's personality. C) Characteristics of the target affect people's perception. D) The time at which we observe behavior affects perception. E) Motives and interests of the perceiver affects perception of behavior. Answer: C Explanation: Characteristics of the target we observe can affect perception. Loud people are more likely to be noticed in a group than quiet ones. Even extremely attractive or unattractive individuals are more likely to be noticed. Since people do not look at targets in isolation, the relationship of a target to its background influences one's perception of the target. LO: 6.1: Explain the factors that influence perception. AACSB: Analytical thinking Difficulty: Easy Employability Skills: Knowledge Application and Analysis Quest. Category: Application 12) People are usually aware of the factors that influence their view of reality. Answer: FALSE Explanation: People are usually not aware of the factors that influence their view of reality. In fact, people are not even that perceptive about their own abilities. LO: 6.1: Explain the factors that influence perception. Difficulty: Easy Quest. Category: Concept 13) People's behavior is based on their perception of what reality is, not on reality itself. Answer: TRUE Explanation: People's behavior is based on their perception of what reality is, not on reality itself. The world as it is perceived is the world that is behaviorally important. LO: 6.1: Explain the factors that influence perception. Difficulty: Easy 243 richard@qwconsultancy.com
Quest. Category: Concept 14) The perception of a target is unaffected by the perceiver's personality or past experiences. Answer: FALSE Explanation: A number of factors operate to shape and sometimes distort perception. These factors can reside in the perceiver, in the object or target being perceived, or in the context of the situation in which the perception is made. LO: 6.1: Explain the factors that influence perception. Difficulty: Easy Quest. Category: Concept 15) Define perception and explain the factors that influence perception. Answer: Perception is a process by which individuals organize and interpret their sensory impressions in order to give meaning to their environment. What an individual perceives can be substantially different from objective reality. When people look at a target and attempt to interpret what they see, their interpretation is heavily influenced by personal characteristics such as their attitudes, personality, motives, interests, past experiences, and expectations. Characteristics of the target also affect perception. Since people do not look at targets in isolation, the relationship of a target to its background also influences perception, as does one's tendency to group close things and similar things together. Context matters as well. The time at which people see an object or event can influence attention, as can location, light, heat, or any number of situational factors. LO: 6.1: Explain the factors that influence perception. Difficulty: Moderate Quest. Category: Concept 16) Using an example, explain the contrast effect. Answer: Evaluation of a person's characteristics that is affected by comparisons with other people recently encountered who rank higher or lower on the same characteristics is known as the contrast effect. We don't evaluate a person in isolation. Our reaction is influenced by other persons we have recently encountered. For example, in a series of job interviews, interviewers can make distortions in any given candidate's evaluation as a result of his or her place in the interview schedule. A candidate is likely to receive a more favorable evaluation if preceded by mediocre applicants and a less favorable evaluation if preceded by strong applicants. LO: 6.1: Explain the factors that influence perception. Difficulty: Moderate Quest. Category: Concept
244 richard@qwconsultancy.com
17) ________ explains the ways in which we judge people differently, depending on the meaning we assign to a given behavior. A) Attribution theory B) Equity theory C) Object relations theory D) Attachment theory E) Cultural schema theory Answer: A Explanation: Attribution theory suggests that when we observe an individual's behavior, we attempt to determine whether it was internally or externally caused. Determination, however, depends largely on three factors, namely, distinctiveness, consensus, and consistency. LO: 6.2: Describe attribution theory. Difficulty: Easy Quest. Category: Concept 18) Attribution theory suggests that when we observe an individual's behavior, we attempt to determine whether it was internally or externally caused. That determination, however, depends largely on three factors. Which of the following is one of these three factors? A) Traceability B) Consistency C) Verifiability D) Relatedness E) Affect intensity Answer: B Explanation: Attribution theory suggests that when we observe an individual's behavior, we attempt to determine whether it was internally or externally caused. Determination, however, depends largely on three factors, namely, distinctiveness, consensus, and consistency. LO: 6.2: Describe attribution theory. Difficulty: Easy Quest. Category: Concept 19) Which of the following is an example of internally caused behavior? A) An employee was late for a team meeting because of a heavy downpour. B) An employee was laid off because the company was attempting to cut costs by laying off employees. C) An employee was fired from work because he violated a company policy. D) An employee could not attend an interview because of a delayed flight. E) An employee could not come to work because he met with an accident. Answer: C Explanation: Internally caused behaviors are those we believe to be under the personal control of the individual. LO: 6.2: Describe attribution theory. AACSB: Analytical thinking Difficulty: Moderate Quest. Category: Concept
245 richard@qwconsultancy.com
246 richard@qwconsultancy.com
20) Which of the following is an example of externally caused behavior? A) An employee postpones a meeting because he overslept. B) An employee is late to work because of a punctured tire. C) An employee was fired because he violated a company policy. D) An employee was promoted when he achieved more than the assigned objectives. E) An employee closed a sale with an important corporate client because of his excellent negotiation skills. Answer: B Explanation: Externally caused behavior is what we imagine the situation forced the individual to do. For instance, if an employee is late for work, and you attribute his arriving late to an automobile accident or a flat tire, then you are making an external attribution. LO: 6.2: Describe attribution theory. AACSB: Analytical thinking Difficulty: Moderate Quest. Category: Concept 21) According to the attribution theory, ________ is one of the three main factors which attempt to determine an individual's behavior. A) distinctiveness B) perverseness C) flexibleness D) resilience E) timorousness Answer: A Explanation: Attribution theory suggests that when we observe an individual's behavior, we attempt to determine whether it was internally or externally caused. Determination, however, depends largely on three factors, namely, distinctiveness, consensus, and consistency. LO: 6.2: Describe attribution theory. Difficulty: Easy Quest. Category: Concept 22) When individuals observe another person's behavior, they attempt to determine whether it is internally or externally caused. Which of the following attempts to explain this phenomenon? A) Pygmalion effect B) Emotional dissonance C) Attribution theory D) Two-factor theory E) Framing effect Answer: C Explanation: Attribution theory is an attempt to determine whether an individual's behavior is internally or externally caused. We judge people differently, depending on the meaning we attribute to a given behavior. Attribution theory suggests that when we observe an individual's behavior, we attempt to determine whether it was internally or externally caused. LO: 6.2: Describe attribution theory. Difficulty: Easy Quest. Category: Concept 247 richard@qwconsultancy.com
23) With reference to the attribution theory, which of the following terms indicates the extent to which an individual displays different behavior in different situations? A) Flexibility B) Integrity C) Consensus D) Consistency E) Distinctiveness Answer: E Explanation: Distinctiveness is one of three determining factors that contribute to attribution theory perceptions. Distinctiveness refers to whether an individual displays different behavior in different situations. LO: 6.2: Describe attribution theory. Difficulty: Easy Quest. Category: Concept 24) If a person responds to a particular situation in the same way over a long time period, then the attribution theory states that the behavior demonstrates ________. A) distinctiveness B) consensus C) consistency D) discontinuity E) traceability Answer: C Explanation: Consistency in a person's actions means that the person responds in the same way to the same situation over a long period of time. For instance, an employee who has not been late for several months is perceived differently from an employee who is late two or three times a week. The regularly late employee demonstrates high consistency in tardiness. LO: 6.2: Describe attribution theory. Difficulty: Easy Quest. Category: Concept 25) According to the attribution theory, if a behavior scores ________, we tend to attribute it to external causes. A) low on distinctiveness B) low on adaptability C) low on consistency D) high on stability E) low on consensus Answer: C Explanation: Consistency indicates that the person responds the same way over time. The less consistent the behavior, the more we are inclined to attribute it to external causes. LO: 6.2: Describe attribution theory. Difficulty: Moderate Quest. Category: Concept
248 richard@qwconsultancy.com
249 richard@qwconsultancy.com
26) Janice Yoder works in an environmental campaigning organization and often needs to interact with a large team for project implementation activities. However, she always finds it difficult to work as a part of a team. She always seems to have major disagreements with team members which lead to antagonistic relations between them. Though she has moved from one team to another, her relations with colleagues always seem to be hostile and cold. How would the attribution theory describe this behavior? A) Low on consensus B) High on reliability C) High on adaptability D) High on consistency E) Low on distinctiveness Answer: D Explanation: Consistency in a person's actions means that the person responds the same way over time to the same situation. Yoder has had hostile relationships with colleagues over a long period of time. Thus, her behavior shows consistency. LO: 6.2: Describe attribution theory. AACSB: Interpersonal relations and teamwork Difficulty: Moderate Employability Skills: Knowledge Application and Analysis Quest. Category: Application 27) According to the attribution theory, if everyone who faces a similar situation responds in the same way, we can say the behavior shows ________. A) distinctiveness B) tractability C) consensus D) consistency E) manageability Answer: C Explanation: If everyone who faces a similar situation responds in the same way, we can say the behavior shows consensus. LO: 6.2: Describe attribution theory. Difficulty: Easy Quest. Category: Concept
250 richard@qwconsultancy.com
28) Mark always starts his workday by saying hello to each of his colleagues. How would attribution theory describe this behavior? A) It shows consensus. B) It shows similarity. C) It shows consistency. D) It shows reliability. E) It shows distinctiveness. Answer: C Explanation: Consistency in a person's actions means that the person responds the same way over time to the same situation. An employee who never greets his colleagues when he arrives at work is perceived differently than one who greets people each day. Mark demonstrates high consistency in friendliness. LO: 6.2: Describe attribution theory. AACSB: Analytical thinking Difficulty: Moderate Employability Skills: Knowledge Application and Analysis Quest. Category: Application 29) According to the attribution theory, ________ is most likely to be attributed to an external cause. A) a behavior that scores high on consensus B) a behavior that scores low on distinctiveness C) a behavior that scores high on consistency D) a behavior that scores low on traceability E) a behavior that scores high on rigidity Answer: A Explanation: According to the attribution theory, if a behavior scores high on consensus, we tend to attribute it to external causes. LO: 6.2: Describe attribution theory. Difficulty: Moderate Quest. Category: Concept
251 richard@qwconsultancy.com
30) William is perpetually late to meetings and usually barges in shortly after they start. Last week, William surprised his colleagues by arriving to the conference room a few minutes early. According to attribution theory. William's behavior last week scores ________. A) high on reliability B) low on distinctiveness C) high on traceability D) low on consistency E) high on stability Answer: D Explanation: If a person responds the same way over time, then his or her behavior displays consistency. Since William is generally late to meetings, his behavior scores low on consistency. LO: 6.2: Describe attribution theory. AACSB: Analytical thinking Difficulty: Moderate Employability Skills: Knowledge Application and Analysis Quest. Category: Application 31) Lily Schmidt is a member of a ten-person team selling home solar systems. Lily made a number of sales during her first six months, but since then she has failed to close any deals. Lily told her boss that it was unreasonable to expect her to match her early numbers. However, based on the sales of the other team members, who have all been able to convince a number of homeowners to buy the solar system, Lily's boss feel that the expectations are on target. Using attribution theory, which of the following best characterizes Lily's behavior? A) Low distinctiveness B) High rigidity C) High traceability D) Low consensus E) Low consistency Answer: D Explanation: If everyone who faces a similar situation responds in the same way, we can say that the behavior shows consensus. LO: 6.2: Describe attribution theory. AACSB: Analytical thinking Difficulty: Moderate Employability Skills: Knowledge Application and Analysis Quest. Category: Application
252 richard@qwconsultancy.com
32) According to the attribution theory, if a behavior scores ________, we tend to attribute it to internal causes. A) low on consistency B) high on rigidity C) low on distinctiveness D) high on consensus E) low on conformity Answer: C Explanation: Distinctiveness refers to whether an individual displays different behavior in different situations. According to the attribution theory, if a behavior scores low on distinctiveness, we tend to attribute it to internal causes. LO: 6.2: Describe attribution theory. Difficulty: Easy Quest. Category: Concept 33) ________ refers to the tendency to underestimate the influence of external factors and overestimate the influence of internal factors when making judgments about the behavior of others. A) Fundamental attribution error B) Bandwagon effect C) Contrast effect D) Emotional dissonance E) Self-fulfilling prophecy Answer: A Explanation: The tendency to underestimate the influence of external factors and overestimate the influence of internal factors when making judgments about the behavior of others is referred to as the fundamental attribution error. LO: 6.2: Describe attribution theory. Difficulty: Easy Quest. Category: Concept
253 richard@qwconsultancy.com
34) Naomi Fisher, a sales manager at Pure, a water purifier company, had a new member, Leah Marshall, join her team. Though during Leah's interview, Naomi felt she would be a productive sales executive, her performance has often been below the mark. Consistently in the past three months, Leah has been unable to reach her targets and is falling substantially behind on her annual targets. Naomi assumes that Leah is not determined and motivated enough to do what it takes. Which of the following, if true, weakens Naomi's assumption? A) Leah has often arrived late for team meetings conducted in the morning. B) Leah has been assigned a sales territory where consumers are from low-income groups. C) Leah has good interpersonal skills and gets along well with her customers. D) Research showed that the company's largest competitor had a lower turnover than they did. E) Naomi recently received feedback from other team members that Leah is often uncooperative. Answer: B Explanation: The tendency to underestimate the influence of external factors and overestimate the influence of internal factors when making judgments about the behavior of others refers to the fundamental attribution error. If Leah is assigned a sales territory where consumers are from low-income groups, this shows that she does not necessarily lack the ability to perform and that external constraints are the reason for her underperformance. It thus weakens Naomi's assumption. If Leah often arrives late for meetings, it strengthens Naomi's assumption because it indicates that Leah is not motivated. Good interpersonal skills do not necessarily mean that a person is motivated, and hence, this does not weaken Naomi's assumption. Research showing that the company's largest competitor had a lower turnover than them is irrelevant to the assumption. LO: 6.2: Describe attribution theory. AACSB: Reflective thinking Difficulty: Hard Quest. Category: Critical Thinking
254 richard@qwconsultancy.com
35) Johanna Murray, a climate campaigner at The National Footprint Foundation, is known in her organization to be a campaigner of caliber and high performance. She recently worked on a campaign against global warming during which she worked extremely hard to achieve project milestones. However, the campaign failed as it could not achieve the desired objective. Due to this, her manager, Brenda Owens, gave her a poor performance appraisal. In the appraisal, Brenda said that Johanna was not motivated and failed to reach out to 25,000 people through Internet media to spread awareness about climate change. Which of the following, if true, weakens Brenda's statement? A) Johanna lacks experience in publicizing campaigns using Internet media. B) Brenda was unable to make time for Johanna to brief her on the tasks involved in carrying out the campaign's media strategy. C) Johanna recently moved from the agriculture campaign to the climate campaign. D) Johanna's previous job involved an extensive amount of researching on environmental issues. E) Brenda is known in the organization to be a fair and unbiased manager. Answer: B Explanation: Individuals and organizations tend to attribute their own successes to internal factors such as ability or effort and place the blame for failure on external factors such as bad luck or unproductive co-workers. This is known as a self-serving bias. If Brenda were unable to make time for Johanna to brief her on the tasks involved in carrying out the campaign's media strategy, then this weakens the argument because it shows that it was not entirely Johanna's fault, and that Brenda is partly to blame. If Johanna lacks experience in publicizing campaigns using Internet media, it strengthens Brenda's statement. The fact that Johanna recently moved from the agriculture campaign to the climate campaign and that Johanna's previous job involved an extensive amount of researching on environmental issues is irrelevant to Brenda's argument. Brenda's argument is strengthened if she is known to be a fair and unbiased manager because it indicates that her evaluation of Johanna is most likely true. LO: 6.2: Describe attribution theory. AACSB: Reflective thinking Difficulty: Hard Quest. Category: Critical Thinking
255 richard@qwconsultancy.com
36) Johanna Springer, who works as a sales executive at Pascal's Bank, is upset at the way her manager, Emma Womack, always calls her in for one-on-one meetings to discuss her underperformance. Though Springer makes a higher number of sales calls and works longer hours than last year, her sales figures are still low. She knows that the main reason behind her underperformance is the recent economic meltdown in the country. However, her manager feels that Springer's underperformance is the result of her laid-back attitude and has nothing to do with external factors. In this situation, Womack's behavior is characterized by a(n) ________. A) anchoring bias B) contrast effect C) fundamental attribution error D) self-fulfilling prophecy E) Pygmalion effect Answer: C Explanation: The tendency to underestimate the influence of external factors and overestimate the influence of internal factors when making judgments about the behavior of others is referred to as the fundamental attribution error. LO: 6.2: Describe attribution theory. AACSB: Analytical thinking Difficulty: Moderate Employability Skills: Knowledge Application and Analysis Quest. Category: Application 37) Which type of bias indicates the tendency of an individual to attribute his or her own successes to internal factors while putting the blame for failures on external factors? A) Status quo B) Self-serving C) Distinction D) Congruence E) Anchoring Answer: B Explanation: Individuals and organizations tend to attribute their own successes to internal factors such as ability or effort and place the blame for failure on external factors such as bad luck or unproductive co-workers. This is known as a self-serving bias. LO: 6.2: Describe attribution theory. Difficulty: Easy Quest. Category: Concept
256 richard@qwconsultancy.com
38) Meera Patel works as a researcher at a pharmaceutical company where she is part of a new product development team. Meera is not popular with her team members because she often tries to take credit for the team's successes but is quick to blame other team members when the team fails to meet its goals. At times, Meera will also suggest that the team was unable to meet expectations because of problems getting tests completed at outside labs. Meera's behavior is an example of ________ bias. A) impact B) anchoring C) confirmation D) distinction E) self-serving Answer: E Explanation: Individuals and organizations tend to attribute their own successes to internal factors such as ability or effort and place the blame for failure on external factors such as bad luck or unproductive co-workers. This is known as self-serving bias. LO: 6.2: Describe attribution theory. AACSB: Interpersonal relations and teamwork Difficulty: Moderate Employability Skills: Knowledge Application and Analysis Quest. Category: Application 39) Laura Simpson, a campaign manager at a child rights organization in Jakarta, planned a marathon for celebrities to raise money for underprivileged children. Though all arrangements for the event had been made, a few days before the event she realized that on the same day there was a political rally happening in the city which would block access to the route on which the marathon was supposed to be undertaken. In such a situation, what is Simpson, who suffers from self-serving bias, most likely to say? A) I did not do sufficient research on public events in the city. B) My colleagues did not inform me about the rally. C) The director had warned me of this. I should have known better. D) I should have weighed feasibility options for the event. E) I should have established better contacts to know about this update. Answer: B Explanation: Self-serving bias places the blame for failure on external factors such as bad luck or unproductive co-workers rather than taking personal responsibility for the failure. LO: 6.2: Describe attribution theory. AACSB: Analytical thinking Difficulty: Moderate Employability Skills: Knowledge Application and Analysis Quest. Category: Application
257 richard@qwconsultancy.com
40) Which type of bias do people engage in when it is impossible for them to assimilate everything they see, and they can take in only certain stimuli? A) Selective perception B) Cognitive dissonance C) Self-serving bias D) Emotional labor E) Self-fulfilling prophecy Answer: A Explanation: Any characteristic that makes a person, an object, or an event stand out will increase the probability we will perceive it. Since we can't observe everything going on about us, we engage in selective perception. LO: 6.2: Describe attribution theory. Difficulty: Easy Quest. Category: Concept 41) You're thinking about buying a new car and have narrowed your choices down to two models. You initially thought you were choosing less popular vehicles, but now you see the same two models everywhere. You are more likely to notice cars similar to the ones you plan to buy because of ________. A) stereotyping B) self-serving bias C) halo effect D) selective perception E) contrast effect Answer: D Explanation: Because we can't observe everything going on about us, we engage in selective perception. Selective perception allows us to "speed-read" others. Due to selective perception, we are more likely to notice cars like our own. LO: 6.2: Describe attribution theory. Difficulty: Moderate Quest. Category: Concept
258 richard@qwconsultancy.com
42) Harriet Kirby, a fundraising manager at a women's rights organization, experienced a bad incident last year with the public relations manager of a banking company who had committed to sponsor a charity event. The bank backed out at the last minute. This year, when a renowned international bank executive showed interest in sponsoring the organization's upcoming annual event, Kirby rejected their participation. She felt that banks have a casual approach toward charity events, and it is risky to involve them in the event. Which of the following best characterizes Kirby's decision? A) Selective perception B) Cognitive dissonance C) Self-serving bias D) Bandwagon effect E) Self-fulfilling prophecy Answer: A Explanation: Because we cannot assimilate all that we observe, we take in bits and pieces. But we don't choose randomly; rather, we select according to our interests, background, experience, and attitudes. This is known as selective perception. LO: 6.2: Describe attribution theory. AACSB: Analytical thinking Difficulty: Moderate Employability Skills: Knowledge Application and Analysis Quest. Category: Application 43) The tendency of people to draw a general impression about an individual on the basis of a single characteristic refers to ________. A) confirmation bias B) self-serving bias C) randomness error D) halo effect E) hindsight bias Answer: D Explanation: When we draw a general impression about an individual on the basis of a single characteristic, such as intelligence, sociability, or appearance, a halo effect is operating. A single trait is allowed to influence the overall impression of the person being judged. LO: 6.2: Describe attribution theory. Difficulty: Easy Quest. Category: Concept
259 richard@qwconsultancy.com
44) Amanda Winter worked as a public engagement coordinator at Safe Food Alliance until three months ago when her manager, Laura Morris, promoted her to the position of a sustainable food campaigner. However, soon after this, Laura noticed that Amanda was facing major difficulties in achieving campaign milestones and the project was falling behind schedule due to her lack of performance. Which of the following, if true, would most strengthen the argument that Laura was influenced by the halo effect in her decision to promote Amanda? A) Laura is known to micromanage most of her projects. B) Laura uses cultural stereotyping in order to speed up the process of decision making. C) Laura has made good hiring decisions in the past and is known to be an unbiased judge of character. D) Laura sat in on only one of Amanda's presentations prior to giving her the promotion. E) Laura worked closely with Amanda over a period of eight months. Answer: D Explanation: When we draw a general impression about an individual on the basis of a single characteristic, such as intelligence, sociability, or appearance, a halo effect is operating. A single trait is allowed to influence the overall impression of the person being judged. The fact that Laura sat in on only one of Amanda's presentations prior to giving her the promotion indicates that she was influenced by the skills that Amanda displayed during that single encounter. Her decision was not based on a comprehensive evaluation of Amanda's abilities and performance. The fact that Laura is known to micromanage most of her tasks weakens the argument that Laura experienced the halo effect. The fact that Laura uses cultural stereotyping in order to speed up the process of decision making is irrelevant to the argument because it talks about a halo effect and not cultural stereotyping. If Laura has made good hiring decisions in the past and is known to be an unbiased judge of character, it weakens the argument that she was influenced by the halo effect. If Laura worked closely with Amanda, she is most likely to know her strengths and weaknesses and would not be influenced by the halo effect. LO: 6.2: Describe attribution theory. AACSB: Critical thinking Difficulty: Hard Quest. Category: Critical Thinking 45) The halo effect involves ________. A) attributing our own successes to internal factors and failures to external factors B) judging someone on the basis of our perception of the group to which he or she belongs C) interpreting a person's behavior in comparison to others recently encountered D) drawing a general impression about an individual on the basis of a single characteristic E) underestimating the influence of external factors when making judgments about people Answer: D Explanation: We tend to draw a general impression about an individual on the basis of a single characteristic, such as intelligence, sociability, or appearance. This is known as halo effect. LO: 6.2: Describe attribution theory. Difficulty: Easy Quest. Category: Concept
260 richard@qwconsultancy.com
46) Betsey Mills works as a driver for a high-end limousine company that is frequently hired to for events in Hollywood and the greater Los Angeles area. In her job, Betsey has driven a number of wealthy individuals including politicians, actors, and other celebrities. Based on her interactions with the clients, Betsey concludes that all wealthy individuals are friendly, down to earth, and hardworking. Which of the following best characterizes Betsey's perception? A) Confirmation bias B) Self-serving bias C) Randomness error D) Halo effect E) Hindsight bias Answer: D Explanation: When we draw a general impression about an individual on the basis of a single characteristic, such as intelligence, sociability, or appearance, a halo effect is operating. A single trait is allowed to influence the overall impression of the person being judged. LO: 6.2: Describe attribution theory. AACSB: Analytical thinking Difficulty: Moderate Employability Skills: Knowledge Application and Analysis Quest. Category: Application 47) The evaluation of a person's characteristics that is affected by comparisons with other people recently encountered who rank higher or lower on the same characteristics is known as ________. A) halo effect B) contrast effect C) confirmation bias D) stereotyping E) anchoring bias Answer: B Explanation: Contrast effect is the evaluation of a person's characteristics that is affected by comparisons with other people recently encountered who rank higher or lower on the same characteristics. LO: 6.2: Describe attribution theory. Difficulty: Easy Quest. Category: Concept
261 richard@qwconsultancy.com
48) Which of the following statements is true regarding a contrast effect? A) It attributes success to internal factors and blames failure on external factors. B) It involves judging a person on the basis of perception of the group to which he or she belongs. C) It involves evaluation of a person's characteristics based on comparison with another person. D) It indicates a tendency to draw a general conclusion about a person on the basis of one feature. E) It indicates a tendency to fixate on initial information and fail to accept subsequent data. Answer: C Explanation: Contrast effect is the evaluation of a person's characteristics that is affected by comparisons with other people recently encountered who rank higher or lower on the same characteristics. LO: 6.2: Describe attribution theory. Difficulty: Easy Quest. Category: Concept 49) Jessica recently joined a new company and was first introduced to Michelle, her cubicle neighbor. Michelle came across as amiable and cheerful. During lunch she met another colleague, Carrie, who did not come across as friendly as Michelle. In this situation, Jessica's interpretation of Carrie's personality is most likely to be affected by ________. A) confirmation bias B) contrast effect C) fundamental attribution error D) self-serving bias E) bandwagon effect Answer: B Explanation: Contrast effect is the evaluation of a person's characteristics that is affected by comparisons with other people recently encountered who rank higher or lower on the same characteristics. LO: 6.2: Describe attribution theory. AACSB: Analytical thinking Difficulty: Moderate Employability Skills: Knowledge Application and Analysis Quest. Category: Application
262 richard@qwconsultancy.com
50) Judging someone on the basis of one's perception of the group to which the person belongs is called ________. A) confirmation bias B) stereotyping C) framing effect D) self-serving bias E) bandwagon effect Answer: B Explanation: Stereotyping is defined as judging someone on the basis of one's perception of the group to which that person belongs. Relying on this type of generalization helps a person make decisions quickly. LO: 6.2: Describe attribution theory. Difficulty: Easy Quest. Category: Concept 51) ________ refers to the tendency to draw a negative general impression about an individual based on a single characteristic. A) Hindsight bias B) Randomness error C) Horns effect D) Illusory superiority E) Telescoping effect Answer: C Explanation: Horns effect refers to the tendency to draw a negative general impression about an individual based on a single characteristic. LO: 6.2: Describe attribution theory. Difficulty: Easy Quest. Category: Concept
263 richard@qwconsultancy.com
52) Rachel Zelenski has been asked to help train a new hire for a barista position in the coffee shop where she works. Rachel has trained other people in the past and likes the camaraderie that usually ensues. As the new hire walks in the door, Rachel notices that she stops to grind out her cigarette before entering. Rachel, a strong anti-smoking advocate, immediately concludes that she will not like the new hire. Rachel's behavior reflects ________. A) halo effect B) contrast effect C) hindsight bias D) horns effect E) confirmation bias Answer: D Explanation: Horns effect refers to the tendency to draw a negative general impression based on a single characteristic. Although she hasn't met the new hire yet, Rachel has concluded she will not like her based on the fact that she smokes. LO: 6.2: Describe attribution theory. AACSB: Analytical thinking Difficulty: Moderate Employability Skills: Knowledge Application and Analysis Quest. Category: Application 53) A manager believes that he should not hire older workers because they can't learn new skills. This belief is an example of ________. A) anchoring bias B) fundamental attribution error C) confirmation bias D) self-serving bias E) stereotyping Answer: E Explanation: When we judge someone on the basis of our perception of the group to which he or she belongs, we are using the shortcut called stereotyping. LO: 6.2: Describe attribution theory. AACSB: Analytical thinking Difficulty: Moderate Quest. Category: Concept
264 richard@qwconsultancy.com
54) A situation in which a person inaccurately perceives another person, and the resulting expectations cause the other person to behave in ways consistent with the original perception refers to ________. A) confirmation bias B) a self-fulfilling prophecy C) attribution theory D) a contrast effect E) a bandwagon effect Answer: B Explanation: A situation in which a person inaccurately perceives another person, and the resulting expectations cause the other person to behave in ways consistent with the original perception is known as a self-fulfilling prophecy. LO: 6.2: Describe attribution theory. Difficulty: Easy Quest. Category: Concept 55) Sarah Covington, a sales manager at Synergy Corporation Bank, often keeps low expectations of her team. She feels that they are underqualified for their job and do not have substantial experience to sell a large number of accounts. Covington's team does not feel motivated enough and invariably underperforms and misses targets on a regular basis. Which of the following concepts best explains Covington's team's poor performance? A) Hindsight bias B) Self-fulfilling prophecy C) Confirmation bias D) Contrast effect E) Bandwagon effect Answer: B Explanation: A situation in which a person inaccurately perceives another person, and the resulting expectations cause the other person to behave in ways consistent with the original perception is known as a self-fulfilling prophecy. LO: 6.2: Describe attribution theory. AACSB: Interpersonal relations and teamwork Difficulty: Moderate Employability Skills: Knowledge Application and Analysis Quest. Category: Application 56) Attribution theory tries to explain the ways in which we judge people differently, depending on the meaning we attribute to a given behavior. Answer: TRUE Explanation: Attribution theory tries to explain the ways in which we judge people differently, depending on the meaning we attribute to a given behavior. LO: 6.2: Describe attribution theory. Difficulty: Easy Quest. Category: Concept
265 richard@qwconsultancy.com
57) According to attribution theory, if a behavior scores high on consensus and distinctiveness, we tend to consider it as an externally caused behavior. Answer: TRUE Explanation: According to the attribution theory, if a behavior scores high on consensus and distinctiveness, we tend to consider it as an externally caused behavior. LO: 6.2: Describe attribution theory. Difficulty: Easy Quest. Category: Concept 58) A candidate is likely to receive a more favorable evaluation if preceded by mediocre applicants and a less favorable evaluation if preceded by strong applicants. This is an example of the halo effect. Answer: FALSE Explanation: We don't evaluate a person in isolation. Our reaction is influenced by other persons we have recently encountered, and this phenomenon is explained by the contrast effect. LO: 6.2: Describe attribution theory. Difficulty: Easy Quest. Category: Concept 59) The Pygmalion effect describes how an individual's behavior is determined by others' expectations. Answer: TRUE Explanation: The Pygmalion effect (and the self-fulfilling prophecy) describes how an individual's behavior is determined by others' expectations. LO: 6.2: Describe attribution theory. Difficulty: Easy Quest. Category: Concept 60) According to attribution theory, the more consistent a behavior, the more we are inclined to attribute it to external causes. Answer: FALSE Explanation: According to attribution theory, the more consistent a behavior, the more we are inclined to attribute it to internal causes. LO: 6.2: Describe attribution theory. Difficulty: Easy Quest. Category: Concept 61) The tendency to draw a general impression about an individual on the basis of a single characteristic is known as the contrast effect. Answer: FALSE Explanation: The tendency to draw a general impression about an individual on the basis of a single characteristic is known as the halo effect. LO: 6.2: Describe attribution theory. Difficulty: Easy Quest. Category: Concept
266 richard@qwconsultancy.com
267 richard@qwconsultancy.com
62) The tendency for individuals to attribute their own successes to internal factors and put the blame for failures on external factors is known as confirmation bias. Answer: FALSE Explanation: The tendency for individuals to attribute their own successes to internal factors and put the blame for failures on external factors is known as self-serving bias. LO: 6.2: Describe attribution theory. Difficulty: Easy Quest. Category: Concept 63) The tendency to underestimate the influence of external factors and overestimate the influence of internal factors when making judgments about the behavior of others is known as the randomness error. Answer: FALSE Explanation: The tendency to underestimate the influence of external factors and overestimate the influence of internal factors when making judgments about the behavior of others is known as the fundamental attribution error. LO: 6.2: Describe attribution theory. Difficulty: Easy Quest. Category: Concept 64) Research supports a social media decision-making bias in hiring decisions. Answer: TRUE Explanation: In a series of studies using Facebook profiles, researchers found that when the applicant's political affiliation displayed on their social media matched the hiring manager's, the manager was more likely to perceive the applicant as similar, likeable, a potentially strong performer, and more hireable than one with a different political affiliation. LO: 6.2: Describe attribution theory. Difficulty: Moderate Quest. Category: Concept 65) Why do people engage in selective perception? Answer: The tendency to selectively interpret what one sees on the basis of one's interests, background, experience, and attitudes is known as selective perception. Individuals engage in selective perception because it is impossible for them to assimilate everything they see and can take in only certain stimuli. However, they do not choose randomly. Rather, they select according to their interests, background, experience, and attitudes. Selective perception allows them to speed-read others, but not without the risk of drawing an inaccurate picture. Seeing what they want to see, they can draw unwarranted conclusions from an ambiguous situation. LO: 6.2: Describe attribution theory. Difficulty: Moderate Quest. Category: Concept
268 richard@qwconsultancy.com
66) What is attribution theory? Answer: Attribution theory suggests that when we observe an individual's behavior, we attempt to determine whether it was internally or externally caused. That determination, however, depends largely on three factors: (1) distinctiveness, (2) consensus, and (3) consistency. First, distinctiveness refers to whether an individual displays different behavior in different situations. A behavior high in distinctiveness is more likely to be given an external attribution. Second, if everyone who faces a similar situation responds in the same way, we can say the behavior shows consensus. A behavior high in consensus is more likely to be considered an externally caused behavior. Third, the more consistent the behavior, the more we are inclined to attribute it to internal causes. LO: 6.2: Describe attribution theory. Difficulty: Moderate Quest. Category: Concept 67) What is the difference between fundamental attribution error and self-serving bias? Answer: When we make judgments about the behavior of other people, we have a tendency to underestimate the influence of external factors and overestimate the influence of internal or personal factors. This is called the fundamental attribution error. There is also a tendency for individuals to attribute their own successes to internal factors, such as ability or effort, while putting the blame for failure on external factors, such as bad luck or unproductive co-workers. This is called self-serving bias. LO: 6.2: Describe attribution theory. Difficulty: Moderate Quest. Category: Concept 68) Victoria Hastings works as a sales manager at a bank and her behavior is characterized by the fundamental attribution error and halo effect. Explain with the help of an example what Hastings' behavior is most likely to be toward her team in such a situation. Answer: A person who is characterized by the fundamental attribution error has a tendency to underestimate the influence of external factors and overestimate the influence of internal factors when making judgments about the behavior of others. This explains why Hastings, who is characterized by the fundamental attribution error, is most likely to blame her employees for underperformance even when they are achieving reasonable objectives during an economic meltdown. A person who is characterized by the halo effect has a tendency to form a general impression about an individual on the basis of a single characteristic. In this situation, Hastings may assume that a team member is a prospective project manager just because she has strong networking skills. On the other hand, she may form a negative impression about a team member who does not actively participate in meetings as someone who is not dedicated and interested in her job. LO: 6.2: Describe attribution theory. AACSB: Analytical thinking Difficulty: Hard Quest. Category: Synthesis
269 richard@qwconsultancy.com
69) Which of the following terms refers to choices made from among two or more alternatives? A) Inquiry B) Decision C) Perception D) Intuition E) Rationalization Answer: B Explanation: A decision refers to choices made from among two or more alternatives. Individual decision making is thus an important part of organizational behavior. But the way individuals make decisions and the quality of their choices are largely influenced by their perceptions. LO: 6.3: Explain the link between perception and decision making. Difficulty: Easy Quest. Category: Concept 70) A discrepancy between the current state of affairs and some desired state is referred to as a(n) ________. A) problem B) decision C) instinct D) intuition E) perception Answer: A Explanation: A problem refers to a discrepancy between the current state of affairs and some desired state. Decision making occurs as a reaction to a problem. LO: 6.3: Explain the link between perception and decision making. Difficulty: Easy Quest. Category: Concept 71) What is the relationship between decision making and perception? Answer: Individuals in organizations make decisions, choices from among two or more alternatives. Individual decision making is an important part of organizational behavior. But the way individuals make decisions, and the quality of their choices are largely influenced by their perceptions. Decision making occurs as a reaction to a problem. That is, a discrepancy exists between the current state of affairs and some desired state, requiring people to consider alternative courses of action. In addition, one person's problem is another person's satisfactory state of affairs. Awareness that a problem exists and that a decision might or might not be needed is a perceptual issue. Every decision requires a person to interpret and evaluate information. A person's perceptions determine which information is relevant and which is not. Throughout the entire decision-making process, perceptual distortions often surface that can bias analysis and conclusions. LO: 6.3: Explain the link between perception and decision making. Difficulty: Moderate Quest. Category: Concept
270 richard@qwconsultancy.com
72) A manager doing performance appraisals gives more weight to recent employee behaviors than to behaviors of six or nine months earlier. This shows that the manager's perception is affected by ________ bias. A) self-serving B) availability C) impact D) distinction E) hindsight Answer: B Explanation: Availability bias refers to the tendency for people to base their judgments on information that is readily available to them. Availability bias explains why managers doing performance appraisals give more weight to recent employee behaviors than to behaviors of six or nine months earlier. LO: 6.4: Contrast the rational model of decision making with bounded rationality and intuition. AACSB: Analytical thinking Difficulty: Moderate Quest. Category: Concept 73) Which of the following refers to staying with a decision even when there is clear evidence it's wrong? A) Escalation of commitment B) Fundamental attribution error C) Randomness error D) Risk aversion E) Availability bias Answer: A Explanation: Escalation of commitment refers to staying with a decision even when there is clear evidence it is wrong. LO: 6.4: Contrast the rational model of decision making with bounded rationality and intuition. Difficulty: Easy Quest. Category: Concept 74) Which type of bias refers to the tendency to believe falsely, after an outcome of an event is actually known, that one would have accurately predicted that outcome? A) Self-serving B) Confirmation C) Impact D) Hindsight E) Anchoring Answer: D Explanation: Hindsight bias is the tendency to believe falsely, after the outcome is known, that one would have accurately predicted it. Hindsight bias reduces our ability to learn from the past. It lets us think we are better predictors than we are and can make us falsely confident. LO: 6.4: Contrast the rational model of decision making with bounded rationality and intuition. Difficulty: Easy Quest. Category: Concept 271 richard@qwconsultancy.com
75) The ________ describes how individuals should behave in order to maximize some outcome. A) rational decision-making model B) flexible decision-making model C) distributive decision-making model D) associative decision-making model E) integrative decision-making model Answer: A Explanation: Rational decision-making model refers to a decision-making model that describes how individuals should behave in order to maximize some outcome. It relies on a number of assumptions, including that the decision maker has complete information, is able to identify all the relevant options in an unbiased manner, and chooses the option with the highest utility. LO: 6.4: Contrast the rational model of decision making with bounded rationality and intuition. Difficulty: Easy Quest. Category: Concept 76) Which of the following statements is true regarding the rational decision-making model? A) It takes into consideration the limited information-processing capability of individuals. B) It involves constructing simplified models without capturing all their complexity. C) It deals with satisficing decisions by seeking solutions that are satisfactory and sufficient. D) It assumes that an individual is able to identify all relevant options in an unbiased manner. E) It is an unconscious decision-making process created from distilled experience. Answer: D Explanation: The rational decision-making model relies on a number of assumptions, including that the decision maker has complete information, is able to identify all the relevant options in an unbiased manner, and chooses the option with the highest utility. LO: 6.4: Contrast the rational model of decision making with bounded rationality and intuition. Difficulty: Moderate Quest. Category: Concept 77) The first step in the rational decision-making model is ________. A) developing alternatives B) defining the problem C) identifying the decision criteria D) weighing the decision criteria E) evaluating the alternatives Answer: B Explanation: The rational decision-making model follows a six-step process. The steps are 1) Defining the problem; 2) Identifying the decision criteria; 3) Allocating weights to the criteria; 4) Developing the alternatives; 5) Evaluating the alternatives; and 6) Selecting the best alternative. LO: 6.4: Contrast the rational model of decision making with bounded rationality and intuition. Difficulty: Easy Quest. Category: Concept
272 richard@qwconsultancy.com
78) Anne Warner, a climate campaigner at an environmental organization, is in charge of implementing a campaign activity where she needs to increase the use of renewable energy in the villages of Vietnam. For her project, she uses the rational decision-making model to implement activities. She has just completed identifying an appropriate criterion for decision making and has allocated weights to the criteria. Which of the following is Warner most likely to undertake next according to the model? A) Develop options of wind, solar, and hydro energy B) Analyze the problems of the project C) Determine goals of the project D) Select hydro energy as the best option E) Weigh advantages between solar and wind energy Answer: A Explanation: The rational decision-making model follows a six-step process. The steps are 1) Define the problem; 2) Identify the decision criteria; 3) Allocate weights to the criteria; 4) Develop the alternatives; 5) Evaluate the alternatives; and 6) Select the best alternative. Anne should now develop various alternative solutions. LO: 6.4: Contrast the rational model of decision making with bounded rationality and intuition. AACSB: Analytical thinking Difficulty: Moderate Employability Skills: Knowledge Application and Analysis Quest. Category: Application 79) An individual engaged in satisficing ________. A) weighs each criterion before making a decision B) seeks solutions that are satisfactory and sufficient C) scrutinizes and evaluating each alternative in detail D) selects the best option with the highest utility E) finds optimal solutions to problems Answer: B Explanation: The limited information-processing capability of human beings makes it impossible to assimilate and understand all the information necessary to optimize. In addition, many problems likely do not have an optimal solution because they are too complicated to be broken down into the parameters of the rational decision-making model. Hence, people satisfice, that is, they seek solutions that are satisfactory and sufficient. LO: 6.4: Contrast the rational model of decision making with bounded rationality and intuition. Difficulty: Moderate Quest. Category: Concept
273 richard@qwconsultancy.com
80) Emily Boyce, a project manager at an insurance firm, regularly satisfices while making decisions. She often comes across complicated problems which would take a long time to resolve. Due to the pressing deadlines, she often meets project goals by satisficing a large number of her decisions. Which of the following is Boyce most likely to do? A) Seek complete information while making decisions B) Search for solutions that are reasonable C) Identify all possible options to solutions D) Analyze each alternative in an unbiased manner E) Choose the optimal solution to each problem Answer: B Explanation: The limited information-processing capability of human beings makes it impossible to assimilate and understand all the information necessary to optimize. In addition, many problems likely do not have an optimal solution because they are too complicated to be broken down into the parameters of the rational decision-making model. Hence, people satisfice, that is, they seek solutions that are satisfactory and sufficient. LO: 6.4: Contrast the rational model of decision making with bounded rationality and intuition. AACSB: Analytical thinking Difficulty: Moderate Employability Skills: Knowledge Application and Analysis Quest. Category: Application 81) ________ refers to the process of making decisions by constructing simplified models that extract the essential features from problems without capturing all their complexity. A) Optimal decision making B) Intuitive decision making C) Bounded rationality D) Active selection E) Incremental decision making Answer: C Explanation: The human mind cannot formulate and solve complex problems with full rationality. Thus, they operate within the confines of bounded rationality. The process of making decisions by constructing simplified models that extract the essential features from problems without capturing all their complexity is known as bounded rationality. LO: 6.4: Contrast the rational model of decision making with bounded rationality and intuition. Difficulty: Easy Quest. Category: Concept
274 richard@qwconsultancy.com
82) Which type of bias refers to the tendency for people to base their judgments on information that is easily accessible? A) Anchoring B) Availability C) Overconfidence D) Confirmation E) Hindsight Answer: B Explanation: Availability bias refers to the tendency for people to base their judgments on information that is readily available to them. LO: 6.4: Contrast the rational model of decision making with bounded rationality and intuition. Difficulty: Easy Quest. Category: Concept 83) ________ is most likely to play a significant role during a negotiation. A) Impact bias B) Normalcy bias C) Distinction bias D) Anchoring bias E) Status quo bias Answer: D Explanation: Any time a negotiation takes place, so does anchoring. For example, when a prospective employer asks how much you made in your prior job, your answer typically anchors the employer's offer. LO: 6.4: Contrast the rational model of decision making with bounded rationality and intuition. Difficulty: Easy Quest. Category: Concept 84) An unconscious process created from distilled experience is known as ________. A) process consultation B) action research C) intuitive decision making D) active selection E) emotional intelligence Answer: C Explanation: Intuitive decision making is an unconscious process created from distilled experience. Intuitive decision making occurs outside conscious thought, relies on holistic associations, is fast, and engages in emotions. LO: 6.4: Contrast the rational model of decision making with bounded rationality and intuition. Difficulty: Easy Quest. Category: Concept
275 richard@qwconsultancy.com
85) Intuitive decision making ________. A) is a slow process of decision making B) is devoid of emotions C) is the most rational way of making a decision D) occurs within conscious thought E) involves making decisions based on distilled experience Answer: E Explanation: Intuition refers to an unconscious process created out of distilled experience. It occurs outside conscious thought and relies on holistic associations, or links between disparate pieces of information. It is fast and is affectively charged, meaning it usually engages the emotions. LO: 6.4: Contrast the rational model of decision making with bounded rationality and intuition. Difficulty: Moderate Quest. Category: Concept 86) Phyllis Stintson needs to decide whether to start a campaign against deforestation in Indonesia. Though her research team has provided substantial information on the high feasibility of the project, Stintson does not go ahead with the project. If Stintson made her decision by drawing unconscious references from several different experiences in the past, her decision is most likely influenced by which of the following? A) Optimization B) Intuition C) Fundamental attribution error D) Framing effect E) Anchoring bias Answer: B Explanation: Intuition is a highly complex and highly developed form of reasoning that is based on years of experience and learning. The key to using intuition in decision making is neither to abandon nor to rely solely on intuition, but to supplement it with evidence and good judgment. LO: 6.4: Contrast the rational model of decision making with bounded rationality and intuition. AACSB: Analytical thinking Difficulty: Moderate Employability Skills: Knowledge Application and Analysis Quest. Category: Application
276 richard@qwconsultancy.com
87) Jalon Edwards works as a campaign manager at Rainforest Alliance Trust, a forest protection organization in Indonesia. She is currently working on the Palm Oil Campaign, which aims to establish stringent laws against companies which aggravate deforestation by extracting palm oil for commercial use. Her role is to establish allies with other forest protection organizations and companies which use eco-friendly products that set good examples for other companies to follow. Jalon allied with Griffin and Powell, a large multinational company, which, unknown to Jalon, also has strong ties with local logging groups in Jakarta. Which of the following, if true, would strengthen the argument that Jalon had an availability bias while establishing an ally with the company? A) Griffin and Powell ensures that all their CSR initiatives on forests are regularly and substantially publicized. B) Jalon has adequate experience in leading such campaigns. C) Jalon is well acquainted with various research techniques. D) Jalon has access to environmental records maintained by the Information Ministry. E) Rainforest Alliance Trust has strong networks with local environmental research organizations. Answer: A Explanation: The tendency for people to base their judgments on information that is readily available to them refers to availability bias. If Griffin and Powell ensures that all their corporate social responsibility (CSR) initiatives on forests are regularly and substantially publicized, then this supports the argument that Jalon has availability bias and made her decision to ally with the company after learning about their CSR initiatives. If Jalon has adequate experience in leading such campaigns and is well acquainted with various research techniques, it weakens the argument because it would mean that she would tend to be unbiased while making decisions. If Jalon has access to environmental records maintained by the Information Ministry, it would imply that she has access to a wide range of information and would not make a decision based on readily available information. If the Rainforest Alliance Trust has strong networks with local environmental research organizations, then this would also mean that Jalon has access to a wide range of information and would not make a decision based on readily available information. LO: 6.4: Contrast the rational model of decision making with bounded rationality and intuition. AACSB: Reflective thinking Difficulty: Hard Quest. Category: Critical Thinking
277 richard@qwconsultancy.com
88) Johanna Murray, a climate campaigner at The National Footprint Foundation, is known in her organization to be a campaigner of caliber and high performance. She has strong networks with the Ministry of Environment and allies with several environmental organizations in the country. Over the years, she has gained substantial knowledge on the issue of climate change. However, recently when she prepared a consolidated report on a conference she attended on climate change, it reflected major loopholes and limited information from the conference. Which of the following, if true, substantiates that Johanna had anchoring bias? A) Johanna was moved by the arguments put forth by the first speaker. B) Johanna participated actively in the interactive session conducted at the end. C) The speakers at the conference consisted of renowned environmental scientists and activists. D) Johanna has attended several conferences where the panel consisted of eminent scientists. E) Johanna was shocked by the startling facts shown during the concluding session. Answer: A Explanation: A tendency to fixate on initial information, from which one then fails to adequately adjust for subsequent information, refers to anchoring bias. If Johanna were moved by the arguments put forward by the first speaker, then this supports the argument that she had anchoring bias because this may have led her to prepare a report which had limited information and focused on initial parts of the conference. If Johanna participated actively in the interactive session conducted at the end, then this works against the argument. It is irrelevant to the argument that the speakers at the conference consisted of renowned environmental scientists and activists. If Johanna has attended several conferences where the panel consisted of eminent scientists, it works against the argument because it indicates that she is used to attending similar conferences and not overwhelmed by the presence of eminent scientists. Hence, this weakens the argument. Johanna being shocked by the startling facts shown during the concluding session weakens the argument as it shows that she did not fixate only on the information presented at the beginning of the conference. LO: 6.4: Contrast the rational model of decision making with bounded rationality and intuition. AACSB: Reflective thinking Difficulty: Hard Quest. Category: Critical Thinking 89) A person's tendency to believe he or she can predict the outcome of random events is known as the self-serving bias. Answer: FALSE Explanation: Our tendency to believe we can predict the outcome of random events is the randomness error. LO: 6.4: Contrast the rational model of decision making with bounded rationality and intuition. Difficulty: Easy Quest. Category: Concept
278 richard@qwconsultancy.com
90) A year ago, Alice Factor sent out a 'save the date' announcement for the grand opening of a new Las Vegas resort. Even though Alice has been informed that the hotel is months behind schedule and won't be ready by the grand opening date, Alice tells her assistant to send out invitations to the event. Alice's action can be described as escalation of commitment. Answer: TRUE Explanation: Escalation of commitment refers to staying with a decision even when there is clear evidence it is wrong. LO: 6.4: Contrast the rational model of decision making with bounded rationality and intuition. Difficulty: Moderate Quest. Category: Concept 91) The rational decision-making model takes into consideration the fact that all information pertaining to a problem might not be available to the decision maker. Answer: FALSE Explanation: The rational decision-making model relies on a number of assumptions, including that the decision maker has complete information, is able to identify all the relevant options in an unbiased manner, and chooses the option with the highest utility. LO: 6.4: Contrast the rational model of decision making with bounded rationality and intuition. Difficulty: Easy Quest. Category: Concept 92) Explain the role of bounded rationality in decision making. Answer: The capacity of the human mind for formulating and solving complex problems is far too small to meet the requirements for full rationality. Thus, individuals operate within the confines of bounded rationality. They construct simplified models that extract the essential features from problems without capturing all their complexity. Individuals can then behave rationally within the limits of the simple model. Once the limited set of alternatives is identified, the decision maker will begin reviewing it. But the review will not be comprehensive. Instead, the decision maker will begin with alternatives that differ only in a relatively small degree from the choice currently in effect. Following along familiar and well-worn paths, the decision maker proceeds to review alternatives only until he or she identifies an alternative that is good enough. The first alternative that meets the "good enough" criterion ends the search. Thus, the final solution represents a satisficing choice rather than an optimum one. LO: 6.4: Contrast the rational model of decision making with bounded rationality and intuition. Difficulty: Moderate Quest. Category: Concept 93) Describe anchoring bias, why it occurs, and the types of professions where it is widely used. Answer: Anchoring bias is a tendency to fixate on initial information and fail to adequately adjust for subsequent information. It occurs because our mind appears to give a disproportionate amount of emphasis to the first information it receives. Anchors are widely used by people in professions in which persuasion skills are important—advertising, management, politics, real estate, and law. LO: 6.4: Contrast the rational model of decision making with bounded rationality and intuition. Difficulty: Moderate Quest. Category: Concept 279 richard@qwconsultancy.com
94) Explain confirmation bias and how it influences the sources of information we choose. Answer: The rational decision-making process assumes we objectively gather information. But we don't. We selectively gather it. Confirmation bias represents a specific case of selective perception: we seek out information that reaffirms our past choices, and we discount information that contradicts them. We also tend to accept at face value information that confirms our preconceived views, while we are critical and skeptical of information that challenges them. Therefore, the information we gather is typically biased toward supporting views we already hold. We even tend to seek sources most likely to tell us what we want to hear, and we give too much weight to supporting information and too little to contradictory. LO: 6.4: Contrast the rational model of decision making with bounded rationality and intuition. Difficulty: Moderate Quest. Category: Concept 95) Discuss escalation of commitment and how it leads to mistakes. Answer: Escalation of commitment refers to staying with a decision even when there is clear evidence it is wrong. Individuals escalate commitment to a failing course of action when they view themselves as responsible for the failure. People who carefully gather and consider information consistent with the rational decision-making model are more likely to engage in escalation of commitment than those who spend less time thinking about their choices. They are more likely to have invested so much time and energy into making their decisions that they have convinced themselves they are taking the right course of action and do not update their knowledge in the face of new information. LO: 6.4: Contrast the rational model of decision making with bounded rationality and intuition. Difficulty: Moderate Quest. Category: Concept 96) What is hindsight bias? Answer: Hindsight bias is the tendency to believe falsely, after the outcome is known, that we'd have accurately predicted it. When we have accurate feedback on the outcome, we seem pretty good at concluding it was obvious. Hindsight bias reduces our ability to learn from the past. It lets us think we're better predictors than we are and can make us falsely confident. LO: 6.4: Contrast the rational model of decision making with bounded rationality and intuition. Difficulty: Easy Quest. Category: Concept
280 richard@qwconsultancy.com
97) Explain intuitive decision making and why it's important not to rely too heavily on it. Answer: Intuitive decision making is an unconscious process created from distilled experience. It occurs outside conscious thought and relies on holistic associations, or links between disparate pieces of information. It is fast and is affectively charged, which means that it usually engages the emotions. While intuition is not rational, it is not necessarily wrong. Nor does it always contradict rational analysis. Instead, the two can complement each other. In certain instances, relying on intuition can improve decision making. But it is important not to rely on it too heavily. This is because it is unquantifiable and thus it is hard to know when our hunches are right or wrong. The key is neither to abandon nor rely solely on intuition, but to supplement it with evidence and good judgment. LO: 6.4: Contrast the rational model of decision making with bounded rationality and intuition. Difficulty: Moderate Quest. Category: Concept 98) ________ bias refers to the tendency to seek out information that reaffirms past choices and to discount information that contradicts past judgment. A) Distinction B) Omission C) Impact D) Confirmation E) Anchoring Answer: D Explanation: Confirmation bias represents a specific case of selective perception. People seek out information that reaffirms past choices and discount information that contradicts them. They also tend to accept at face value information that confirms their preconceived views, while they are critical and skeptical of information that challenges these views. LO: 6.5: Explain how individual differences and organizational constraints affect decision making. Difficulty: Easy Quest. Category: Concept 99) Which type of bias involves fixating on initial information and failing to adequately adjust for subsequent information? A) Hindsight bias B) Overconfidence bias C) Anchoring bias D) Availability bias E) Self-serving bias Answer: C Explanation: Anchoring bias is a tendency to fixate on initial information and fail to adequately adjust for subsequent information. It occurs because our mind appears to give a disproportionate amount of emphasis to the first information it receives. LO: 6.5: Explain how individual differences and organizational constraints affect decision making. Difficulty: Easy Quest. Category: Concept 281 richard@qwconsultancy.com
100) An organization's reward system influences decision makers by suggesting which choices have better personal payoffs. Answer: TRUE Explanation: An organization's reward system influences decision makers by suggesting which choices have better personal payoffs. If the organization rewards risk aversion, managers are more likely to make conservative decisions. LO: 6.5: Explain how individual differences and organizational constraints affect decision making. Difficulty: Moderate Quest. Category: Concept 101) The preference for rational decision making processes that is common in North America is generally accepted across the world. Answer: FALSE Explanation: The overt rationality of decision-making processes that is valued in North America is not necessarily preferred across the world. LO: 6.5: Explain how individual differences and organizational constraints affect decision making. Difficulty: Easy Quest. Category: Concept 102) Achievement-striving individuals appear more susceptible to hindsight bias. Answer: TRUE Explanation: Achievement-striving individuals appear more susceptible to hindsight bias, perhaps because they have a need to justify their actions. LO: 6.5: Explain how individual differences and organizational constraints affect decision making. Difficulty: Moderate Quest. Category: Concept 103) An individual's cultural background can influence the selection of problems, the depth of analysis, the importance placed on logic and rationality, and whether organizational decisions should be made autocratically by an individual manner or collectively in groups. Answer: TRUE Explanation: The cultural background of a decision maker can influence the selection of problems, the depth of analysis, the importance placed on logic and rationality, and whether organizational decisions should be made autocratically by an individual manner or collectively in groups. LO: 6.5: Explain how individual differences and organizational constraints affect decision making. AACSB: Diverse and multicultural work environments Difficulty: Moderate Quest. Category: Concept
282 richard@qwconsultancy.com
104) Explain the effect of personality traits — intuition, self-esteem, narcissism- on decision making. Answer: Several personality traits are related to taking on specific decision-making strategies, or to experiencing errors or biases during decision-making. Employees differ in the extent to which they trust their intuitions, and this can lead them to make riskier decisions or even to be harsher when condemning unethical behavior. People with high self-esteem are strongly motivated to maintain it. So, they use the self-serving biases to preserve it. They may be more prone to blame others for their failures while taking credit for success. Narcissists naturally tend to be more prone to overconfidence and to self-serving biases. LO: 6.5: Explain how individual differences and organizational constraints affect decision making. Difficulty: Hard Quest. Category: Concept 105) According to the concept of ________, decisions are made solely on the basis of their outcomes, ideally to provide the greatest good for the greatest number. A) utilitarianism B) selective perception C) self-fulfilling prophecy D) halo effect E) contrast effect Answer: A Explanation: According to the ethical yardstick of utilitarianism, decisions are made to provide the greatest good for the greatest number. In this system, decisions are made solely on the basis of their outcomes. LO: 6.6: Contrast the three ethical decision criteria. Difficulty: Easy Quest. Category: Concept 106) Which area of study analyzes how people behave when confronted with ethical dilemmas? A) The halo effect B) Behavioral ethics C) Hindsight bias D) Positivity offset E) Selective perception Answer: B Explanation: An area of study that analyzes how people behave when confronted with ethical dilemmas is behavioral ethics. LO: 6.6: Contrast the three ethical decision criteria. Difficulty: Easy Employability Skills: Business Ethics and Social Responsibility Quest. Category: Concept
283 richard@qwconsultancy.com
107) ________ refer to individuals who report unethical practices by their employer to outsiders. A) Change agents B) Boundary spanners C) Early adopters D) Whistleblowers E) Free riders Answer: D Explanation: Whistleblowers are individuals who reveal an organization's unethical practices to the press or government agencies, using their right to free speech. LO: 6.6: Contrast the three ethical decision criteria. Difficulty: Easy Quest. Category: Concept 108) A focus on utilitarianism creates an environment that hinders productivity and efficiency. Answer: FALSE Explanation: The first ethical yardstick is utilitarianism, which proposes making decisions solely on the basis of their outcomes, ideally to provide the greatest good for the greatest number. This view dominates business decision making. It is consistent with goals such as efficiency, productivity, and high profits. LO: 6.6: Contrast the three ethical decision criteria. Difficulty: Easy Quest. Category: Concept 109) Explain how stereotyping can cause problems for some managers when making ethical decisions. Provide an example. Answer: One of the criteria of ethical decision making is to focus on individual rights. Thus, the use of stereotyping would affect the ethical decision-making process. The focus on rights calls on individuals to make decisions consistent with fundamental liberties and privileges as set forth in documents like the Bill of Rights. An emphasis on rights in decision making means respecting and protecting the basic rights of individuals. If a manager engages in stereotyping, for example, believing that all women are less productive than men, he may be inclined to base organizational decisions on this stereotype. When an important project or promotion comes up, the manager would always be inclined to reward men over women. LO: 6.6: Contrast the three ethical decision criteria. AACSB: Analytical thinking Difficulty: Moderate Quest. Category: Synthesis
284 richard@qwconsultancy.com
110) Describe the three ethical decision criteria. Answer: The first ethical yardstick is utilitarianism, which proposes making decisions solely on the basis of their outcomes, ideally to provide the greatest good for the greatest number. This view dominates business decision making. It is consistent with goals such as efficiency, productivity, and high profits. Another ethical criterion is to make decisions consistent with fundamental liberties and privileges, as set forth in documents such as the Bill of Rights. An emphasis on rights in decision making means respecting and protecting the basic rights of individuals, such as the right to privacy, free speech, and due process. A third criterion is to impose and enforce rules fairly and impartially to ensure justice or an equitable distribution of benefits and costs. Union members typically favor this view. It justifies paying people the same wage for a given job regardless of performance differences and using seniority as the primary determination in layoff decisions. LO: 6.6: Contrast the three ethical decision criteria. Difficulty: Moderate Quest. Category: Concept 111) Which of the following is not one of the four steps of creative behavior as shown in the three-stage model of creativity? A) Idea generation B) Idea creation C) Idea evaluation D) Problem formulation E) Information gathering Answer: B Explanation: The four steps of creative behavior as shown in the three-stage model of creativity are problem formulation, information gathering, idea generation, and idea evaluation. LO: 6.7: Describe the three-stage model of creativity. Difficulty: Easy Quest. Category: Concept 112) The ability to produce novel and useful ideas is referred to as ________. A) creativity B) expertise C) intelligence D) ability E) potential Answer: A Explanation: The ability to produce novel and useful ideas is referred to as creativity. LO: 6.7: Describe the three-stage model of creativity. Difficulty: Easy Quest. Category: Concept
285 richard@qwconsultancy.com
113) ________ is the stage of creative behavior in which we identify a problem or opportunity that requires a solution that is yet unknown. A) Idea generation B) Idea potential C) Problem evaluation D) Problem formulation E) Information gathering Answer: D Explanation: Problem formulation is the stage of creative behavior in which we identify a problem or opportunity that requires a solution that is yet unknown. LO: 6.7: Describe the three-stage model of creativity. Difficulty: Moderate Quest. Category: Concept 114) Idea evaluation is the process in which we develop possible solutions to a problem from relevant information and knowledge. Answer: FALSE Explanation: Idea evaluation is the process of creative behavior in which we evaluate potential solutions to identify the best one. Idea generation is the process in which we develop possible solutions to a problem from relevant information and knowledge. LO: 6.7: Describe the three-stage model of creativity. Difficulty: Easy Quest. Category: Concept 115) Describe the causes of creative behavior in the three-stage model of creativity. Answer: Creative behavior is a result of creative potential and a creative environment where that potential can be realized. Creative potential involves several factors, intelligence, personality, expertise, and ethics. Intelligence and Creativity: Smart people are more creative because they are better are solving complex problems. Personality and Creativity: Openness to experience, proactive personality, self-confidence, risk taking, tolerance and ambiguity, and perseverance are all traits of creative people. Expertise and Creativity: Expertise is the foundation for all creative work and thus is the single most important predictor of creative potential. Ethics and Creativity: Creativity is not correlated with ethicality. LO: 6.7: Describe the three-stage model of creativity. AACSB: Analytical thinking Difficulty: Moderate Quest. Category: Synthesis
286 richard@qwconsultancy.com
116) What is the creative outcome stage of the three-stage model of creativity? Answer: Creative outcome is the final stage of the three-stage model. Creative behavior does not always produce innovation, or a creative outcome. Innovations can be defined as ideas or solutions judged to be novel and useful by relevant stakeholders. Novelty itself does not generate an innovative outcome if it is not useful. Creative ideas do not implement themselves; translating them into innovative outcomes is a social process that requires utilizing other concepts including power and politics, leadership, and motivation. LO: 6.7: Describe the three-stage model of creativity. AACSB: Analytical thinking Difficulty: Moderate Quest. Category: Synthesis Organizational Behavior, 19e (Robbins/Judge) Chapter 7 Motivation Concepts 1) The processes that account for an individual's intensity, direction, and persistence of effort toward attaining a goal are referred to as ________. A) leadership B) management C) learning D) emotional labor E) motivation Answer: E Explanation: Motivation is defined as the processes that account for an individual's intensity, direction, and persistence of effort toward attaining a goal. LO: 7.1: Describe the three key elements of motivation. Difficulty: Easy Quest. Category: Concept 2) Which dimension of motivation measures how long a person can maintain effort? A) Direction B) Persistence C) Intensity D) Knowledge E) Experience Answer: B Explanation: Motivation has a persistence dimension. This measures how long a person can maintain effort. Motivated individuals stay with a task long enough to achieve their goal. LO: 7.1: Describe the three key elements of motivation. Difficulty: Easy Quest. Category: Concept 3) Which element of motivation describes how hard a person tries? A) Intelligence B) Experience C) Direction D) Intensity 287 richard@qwconsultancy.com
E) Persistence Answer: D Explanation: Intensity describes how hard a person tries. This is the element most of us focus on when we talk about motivation. LO: 7.1: Describe the three key elements of motivation. Difficulty: Easy Quest. Category: Concept
288 richard@qwconsultancy.com
4) The level of motivation varies both between individuals and within individuals at different times. Answer: TRUE Explanation: The level of motivation varies both between individuals and within individuals at different times. LO: 7.1: Describe the three key elements of motivation. Difficulty: Moderate Quest. Category: Concept 5) Define and explain motivation and its key elements. Answer: Motivation is defined as the processes that account for an individual's intensity, direction, and persistence of effort toward attaining a goal. The three key elements in our definition are intensity, direction, and persistence. Intensity describes how hard a person tries. This is the element most of us focus on when we talk about motivation. However, high intensity is unlikely to lead to favorable job performance outcomes unless the effort is channeled in a direction that benefits the organization. Therefore, we consider the quality of effort as well as its intensity. Effort directed toward, and consistent with, the organization's goals is the kind of effort we should be seeking. Finally, motivation has a persistence dimension. This measures how long a person can maintain effort. Motivated individuals stay with a task long enough to achieve their goal. LO: 7.1: Describe the three key elements of motivation. Difficulty: Moderate Quest. Category: Concept 6) The ________ level in Maslow's hierarchy of needs focuses on satisfying one's hunger, thirst, and other bodily needs. A) safety-security B) physiological C) social D) esteem E) psychological Answer: B Explanation: Maslow hypothesized that within every human being there exists a hierarchy of five needs. The lowest, most basic needs are physiological. They include hunger, thirst, shelter, sex, and other bodily needs. LO: 7.2: Compare the classic theories of motivation. Difficulty: Easy Quest. Category: Concept
289 richard@qwconsultancy.com
7) Which of the following needs would most likely motivate Joanna? A) Social B) Esteem C) Physiological D) Self-actualization E) Safety Answer: C Explanation: Joanna first needs to satisfy her basic physiological needs, which include hunger, thirst, shelter, sex, and other bodily needs. LO: 7.2: Compare the classic theories of motivation. AACSB: Analytical thinking Difficulty: Moderate Employability Skills: Knowledge Application and Analysis Quest. Category: Application 8) Which of the following needs would most likely motivate Josephine? A) Social-belongingness B) Esteem C) Physiological D) Self-actualization E) Safety Answer: A Explanation: According to Maslow, because Josephine is single and has little social interaction, she would strive to satisfy her social needs, which include affection, belongingness, acceptance, and friendship. LO: 7.2: Compare the classic theories of motivation. AACSB: Analytical thinking Difficulty: Moderate Employability Skills: Knowledge Application and Analysis Quest. Category: Application 9) Which of the following needs would most motivate Jonathan? A) Social-belongingness B) Esteem C) Physiological D) Self-actualization E) Safety Answer: D Explanation: According to Maslow, because Jonathan has satisfied all the previous needs in Maslow's hierarchy, he would be seeking self-actualization. LO: 7.2: Compare the classic theories of motivation. AACSB: Analytical thinking Difficulty: Moderate Employability Skills: Knowledge Application and Analysis Quest. Category: Application
290 richard@qwconsultancy.com
291 richard@qwconsultancy.com
10) Which of the following is a lower-level need in Maslow's hierarchy of needs? A) Social-belongingness B) Safety C) Esteem D) Self-actualization E) Recognition Answer: B Explanation: Maslow separated the five needs into higher and lower orders. Physiological and safety needs were lower-order needs, and social, esteem, and self-actualization were higher-order needs. LO: 7.2: Compare the classic theories of motivation. Difficulty: Easy Quest. Category: Concept 11) ________ is the highest-level need in Maslow's theory. A) Self-actualization B) Safety C) Social-belongingness D) Esteem E) Physiological Answer: A Explanation: Self-actualization, the drive to become what we can become, is considered the highest-level need. LO: 7.2: Compare the classic theories of motivation. Difficulty: Moderate Quest. Category: Concept 12) ________ in Maslow's hierarchy refers to the drive to become what we can become. A) Social-belongingness B) Self-actualization C) Physiological D) Esteem E) Safety Answer: B Explanation: Self-actualization refers to the drive to become what we can become. It includes growth, achieving our potential, and self-fulfillment. LO: 7.2: Compare the classic theories of motivation. Difficulty: Easy Quest. Category: Concept
292 richard@qwconsultancy.com
13) Which of the following best explains why Maslow's theory is criticized? A) The concept of self-actualization was unfounded. B) There is little evidence that needs are structured or operate in the way it describes. C) The esteem need is a more powerful motivator than self-actualization. D) Its terminology tends to alienate those to whom it is applied. E) It does not adequately describe how an organization can satisfy higher-order needs. Answer: B Explanation: Maslow's theory has received long-standing wide recognition, particularly among practicing managers. It is intuitively logical and easy to understand, and some research has validated it. Unfortunately, however, most research has not, and it hasn't been frequently researched since the 1960s. But old theories, especially intuitively logical ones, die hard. It is thus important to be aware of the prevailing public acceptance of the hierarchy when discussing motivation. LO: 7.2: Compare the classic theories of motivation. Difficulty: Moderate Quest. Category: Concept 14) Which theory is also referred to as the motivation-hygiene theory? A) Hierarchy of needs B) Goal-setting C) Self-determination D) Cognitive evaluation E) Two-factor Answer: E Explanation: Psychologist Frederick Herzberg proposed the two-factor theory—also called motivation-hygiene theory. LO: 7.2: Compare the classic theories of motivation. Difficulty: Easy Quest. Category: Concept 15) Which of the following theories proposes the idea of a dual continuum? A) Maslow's hierarchy of needs theory B) Self-determination theory C) Two-factor theory D) Cognitive evaluation theory E) McClelland's theory of needs Answer: C Explanation: Frederick Herzberg postulated the two-factor theory and proposed a dual continuum: The opposite of "satisfaction" is "no satisfaction," and the opposite of "dissatisfaction" is "no dissatisfaction." LO: 7.2: Compare the classic theories of motivation. Difficulty: Moderate Quest. Category: Concept
293 richard@qwconsultancy.com
16) According to the two-factor theory, ________. A) there exists a hierarchy of needs within every human being, and as each need is satisfied, the next one becomes dominant B) most employees inherently dislike work and must therefore be directed or even coerced into performing it C) employees view work as being as natural as rest or play, and therefore learn to accept, and even seek, responsibility D) the aspects that lead to job satisfaction are separate and distinct from those that lead to job dissatisfaction E) achievement, power, and affiliation are three important needs that help explain motivation Answer: D Explanation: The two-factor theory proposes that the factors that lead to job satisfaction are separate and distinct from those that lead to job dissatisfaction. LO: 7.2: Compare the classic theories of motivation. Difficulty: Moderate Quest. Category: Concept 17) ________ is a motivational factor according to Herzberg's two-factor theory. A) Quality of supervision B) Recognition C) Pay D) Relationships with others E) Company policies Answer: B Explanation: Herzberg characterized conditions such as quality of supervision, pay, company policies, physical working conditions, relationships with others, and job security as hygiene factors. When they're adequate, people will not be dissatisfied; neither will they be satisfied. If we want to motivate people on their jobs, Herzberg suggested emphasizing factors associated with the work itself or with outcomes directly derived from it, such as promotional opportunities, personal growth opportunities, recognition, responsibility, and achievement. LO: 7.2: Compare the classic theories of motivation. Difficulty: Moderate Quest. Category: Concept 18) ________ is considered a hygiene factor in Hertzberg's two-factor theory. A) Promotional opportunities B) Quality of supervision C) Achievement D) Recognition E) Responsibility Answer: B Explanation: According to the two-factor theory, conditions such as quality of supervision, pay, company policies, physical working conditions, relationships with others, and job security are considered hygiene factors or extrinsic factors. LO: 7.2: Compare the classic theories of motivation. Difficulty: Moderate 294 richard@qwconsultancy.com
Quest. Category: Concept 19) According to Herzberg, when ________ are adequate, people won't be dissatisfied, but they will also not be satisfied. A) achievement needs B) affiliation needs C) motivational factors D) power needs E) hygiene factors Answer: E Explanation: Herzberg suggested that the opposite of "satisfaction" is "no satisfaction," and the opposite of "dissatisfaction" is "no dissatisfaction." Removing dissatisfying characteristics from a job does not necessarily make the job satisfying. When hygiene factors are adequate, people will not be dissatisfied; neither will they be satisfied. LO: 7.2: Compare the classic theories of motivation. Difficulty: Moderate Quest. Category: Concept 20) Which of the following needs are reflected in McClelland's theory? A) Stability, growth, and security B) Achievement, power, and affiliation C) Self-actualization, stability, and safety-security D) Hygiene, control, and security E) Control, status, and self-actualization Answer: B Explanation: McClelland's theory of needs states that the need for achievement (nAch), the need for power (nPow), and the need for affiliation (nAff) help explain motivation. LO: 7.2: Compare the classic theories of motivation. Difficulty: Easy Quest. Category: Concept
295 richard@qwconsultancy.com
You manage a department of five employees. You have identified that José has a high need for achievement, Mary has a high need for power, and Tim has a high need for affiliation. Sarah scored high on the need for power and low on the need for affiliation. Doug scored low on both need for power and need for affiliation. 21) Which of these five employees is most likely to be suitable for a new assignment that involves a high degree of personal responsibility and feedback? A) José B) Mary C) Tim D) Sarah E) Doug Answer: A Explanation: Need for achievement (nAch) is the drive to excel, to achieve in relation to a set of standards. As José has a high need for achievement, he would enjoy a challenging assignment with a high degree of personal responsibility and feedback. LO: 7.2: Compare the classic theories of motivation. AACSB: Analytical thinking Difficulty: Moderate Employability Skills: Knowledge Application and Analysis Quest. Category: Application 22) ________ would be the best person to handle your responsibilities when you are on vacation? A) Joe B) Mary C) Tim D) Sarah E) Doug Answer: D Explanation: The best managers are high in their need for power and low in their need for affiliation. Sarah has a high need for power and a low need for affiliation. LO: 7.2: Compare the classic theories of motivation. AACSB: Analytical thinking Difficulty: Moderate Employability Skills: Knowledge Application and Analysis Quest. Category: Application
296 richard@qwconsultancy.com
23) Erika wants to become the head of the HR department. Although the role comes with a generous salary hike and will put her in charge of several subordinates, she is mainly pursuing this position because she believes she can do the job better than anyone else and wants people to know this. According to McClelland's theory of needs, which of the following needs is Erika primarily driven by in this case? A) The need for stability B) The need for achievement C) The need for security D) The need for affiliation E) The need for power Answer: B Explanation: Need for achievement (nAch) is the drive to excel, to achieve in relation to a set of standards. Erika is demonstrating the need to achieve. LO: 7.2: Compare the classic theories of motivation. AACSB: Analytical thinking Difficulty: Moderate Employability Skills: Knowledge Application and Analysis Quest. Category: Application 24) Which of the following statements is true according to McClelland's theory of needs? A) People with a high achievement need prefer tasks that have a high level of risk. B) People with a high achievement need are interested in motivating others to do well. C) People with a high need for power and affiliation often make good managers in large firms. D) People with a high achievement need experience great satisfaction from success that comes by luck. E) People with a high need for achievement tend to perform well in high stakes job situations. Answer: E Explanation: Employees high on nAch tend to perform very well in high-stakes conditions on the job, like work walkthroughs or sales encounters. LO: 7.2: Compare the classic theories of motivation. Difficulty: Moderate Quest. Category: Concept 25) ________ best corresponds to McClelland's need for affiliation. A) Safety B) Interpersonal relationships C) Esteem D) Self-actualization E) Physiological belongingness Answer: B Explanation: In McClelland's theory, the need for affiliation is the desire for friendly and close interpersonal relationships. LO: 7.2: Compare the classic theories of motivation. AACSB: Analytical thinking Difficulty: Moderate Quest. Category: Concept 297 richard@qwconsultancy.com
26) Maslow's hierarchy of needs has received strong research support. Answer: FALSE Explanation: While Maslow's hierarchy has received long-standing recognition, most research does not support its validity. LO: 7.2: Compare the classic theories of motivation. Difficulty: Moderate Quest. Category: Concept 27) According to Hertzberg's two-factory theory, removing dissatisfying characteristics from a job will make the job more satisfying. Answer: FALSE Explanation: According to Hertzberg's two-factory theory, removing dissatisfying characteristics from a job does not necessarily make the job satisfying; managers would be placating, rather than motivating, employees. LO: 7.2: Compare the classic theories of motivation. Difficulty: Moderate Quest. Category: Concept 28) According to Herzberg, the opposite of "satisfaction" is "dissatisfaction." Answer: FALSE Explanation: According to Herzberg, the opposite of "satisfaction" is "no satisfaction," and the opposite of "dissatisfaction" is "no dissatisfaction." LO: 7.2: Compare the classic theories of motivation. Difficulty: Moderate Quest. Category: Concept 29) McClelland's theory of needs proposes that the factors that lead to job satisfaction are separate and distinct from those that lead to job dissatisfaction. Answer: FALSE Explanation: The two-factor theory proposes that the factors that lead to job satisfaction are separate and distinct from those that lead to job dissatisfaction. McClelland's theory of needs proposes that achievement, power, and affiliation are three important needs that help explain motivation. LO: 7.2: Compare the classic theories of motivation. Difficulty: Easy Quest. Category: Concept 30) McClelland's theory of needs has received little research support. Answer: FALSE Explanation: McClelland's theory of needs has been supported by research. LO: 7.2: Compare the classic theories of motivation. Difficulty: Moderate Quest. Category: Concept
298 richard@qwconsultancy.com
31) Explain Maslow's hierarchy of needs theory. Answer: Maslow's hierarchy of needs hypothesized that within every human being there exists a hierarchy of five needs. a) The physiological needs include hunger, thirst, shelter, sex, and other bodily needs. b) Safety includes security and protection from physical and emotional harm. c) Social-belongingness includes affection, belongingness, acceptance, and friendship. d) Esteem includes internal factors such as self-respect, autonomy, and achievement and external factors such as status, recognition, and attention. e) Self-actualization is the drive to become what one is capable of becoming and it includes growth, achieving one's potential, and self-fulfillment. As each of these needs becomes substantially satisfied, the next need becomes dominant. So, according to Maslow, if you want to motivate someone, you need to understand what level of the hierarchy that person is currently on and focus on satisfying those needs at or above that level. LO: 7.2: Compare the classic theories of motivation. Difficulty: Moderate Quest. Category: Concept 32) Using Herzberg's two-factor theory, explain how a manager motivates employees. Answer: According to Herzberg, the factors that lead to job satisfaction are separate and distinct from those that lead to job dissatisfaction. Therefore, managers who seek to eliminate factors that can create job dissatisfaction may bring about peace, but not necessarily motivation. They will be placating, rather than motivating, their workers. As a result, Herzberg characterized conditions such as quality of supervision, pay, company policies, physical working conditions, relationships with others, and job security as hygiene factors. When they're adequate, people will not be dissatisfied; neither will they be satisfied. If we want to motivate people on their jobs, Herzberg suggested emphasizing factors associated with the work itself or with outcomes directly derived from it, such as promotional opportunities, personal growth opportunities, recognition, responsibility, and achievement. These are the characteristics people find intrinsically rewarding. LO: 7.2: Compare the classic theories of motivation. Difficulty: Moderate Quest. Category: Concept 33) Compare and contrast Maslow's hierarchy of needs theory and McClelland's theory of needs. Answer: It can be noted that the need for achievement in McClelland's theory is very similar to the self-actualization needs in Maslow's. Both deal with the drive to become what we are capable of becoming, which includes growth, achieving our potential, and self-fulfillment. The need for power in McClelland's theory is similar in many aspects to esteem needs in Maslow's theory. The need for affiliation is similar to the social-belongingness needs in Maslow's theory. LO: 7.2: Compare the classic theories of motivation. AACSB: Analytical thinking Difficulty: Moderate Quest. Category: Synthesis
299 richard@qwconsultancy.com
34) Which of the following theories proposes that employees' well-being and performance are influenced by the nature of their motivation for certain job activities. A) Self-serving theory B) Motivation-hygiene theory C) Two-factor theory D) Self-determination theory E) Goal setting theory Answer: D Explanation: Self-determination theory proposes that employees' well-being and performance are influenced by the nature of their motivation for certain job activities. LO: 7.3: Contrast the content-based theories of motivation, including self-determination theory, regulatory-focus theory, and job engagement theory. Difficulty: Moderate Quest. Category: Concept 35) Which of the following statements is true regarding the cognitive evaluation theory? A) People need extrinsic rewards in order to be motivated. B) Extrinsic rewards tend to reduce intrinsic interest in a task. C) Intrinsic rewards are almost as effective as extrinsic rewards. D) Externally imposed standards of work largely improve intrinsic motivation. E) Extrinsic rewards, including verbal praise, significantly decrease intrinsic motivation. Answer: B Explanation: The cognitive evaluation theory hypothesizes that extrinsic rewards will reduce intrinsic interest in a task. LO: 7.3: Contrast the content-based theories of motivation, including self-determination theory, regulatory-focus theory, and job engagement theory. Difficulty: Moderate Quest. Category: Concept 36) Self-determination theory proposes that in addition to being driven by a need for autonomy, people seek ways to achieve ________. A) competence and positive connections B) high rewards C) recognition and status D) career growth E) power and control Answer: A Explanation: Self-determination theory proposes that in addition to being driven by a need for autonomy, people seek ways to achieve competence and positive connections to others. LO: 7.3: Contrast the content-based theories of motivation, including self-determination theory, regulatory-focus theory, and job engagement theory. Difficulty: Moderate Quest. Category: Concept
300 richard@qwconsultancy.com
37) Rachel's parents used to pay her an allowance every week to feed the cats and to do a few other chores around the house. However, once her mother lost her job, her parents stopped giving her an allowance. Although Rachel quit making her bed every morning, she still continued to feed the cats. Which of the following best explains why Rachel continues to feed the cats? A) Without the extrinsic reward, the task itself is eliminated. B) With the extrinsic reward, the execution of the task relies on internal motivation. C) Without the intrinsic reward, the execution of the task relies on external motivation. D) Without the extrinsic reward, the execution of the task relies on internal motivation. E) With the intrinsic reward, the execution of the task relies on external motivation. Answer: D Explanation: By eliminating the extrinsic reward (the allowance), the explanation for Rachel's behavior (continuing to feed the cats) shifts from an external motivation to an internal motivation. Rachel must enjoy or feel some other internal desire to continue feeding the cats. LO: 7.3: Contrast the content-based theories of motivation, including self-determination theory, regulatory-focus theory, and job engagement theory. AACSB: Analytical thinking Difficulty: Hard Employability Skills: Knowledge Application and Analysis Quest. Category: Application 38) Which of the following considers how strongly people's reasons for pursuing goals are consistent with their interests and core values? A) Self-serving bias B) Self-fulfilling prophecy C) Self-concordance D) Self-actualization E) Self-efficacy Answer: C Explanation: A recent outgrowth of self-determination theory is self-concordance, which considers how strongly people's reasons for pursuing goals are consistent with their interests and core values. If individuals pursue goals because of an intrinsic interest, they are more likely to attain their goals and are happy even if they do not. LO: 7.3: Contrast the content-based theories of motivation, including self-determination theory, regulatory-focus theory, and job engagement theory. Difficulty: Easy Quest. Category: Concept
301 richard@qwconsultancy.com
39) A prevention (but not a promotion) focus is related to higher levels of ________. A) citizenship behavior B) innovation C) safety performance D) task performance E) technology Answer: C Explanation: A promotion (but not a prevention) focus is related to higher levels of task performance, citizenship behavior, and innovation; a prevention (but not a promotion) focus is related to safety performance. LO: 7.3: Contrast the content-based theories of motivation, including self-determination theory, regulatory-focus theory, and job engagement theory. Difficulty: Moderate Quest. Category: Concept 40) ________ refers to the investment of an employee's physical, cognitive, and emotional energies into job performance. A) Vicarious modeling B) Self-determination C) Job engagement D) Management by objectives E) Job analysis Answer: C Explanation: Job engagement refers to the investment of an employee's physical, cognitive, and emotional energies into job performance. LO: 7.3: Contrast the content-based theories of motivation, including self-determination theory, regulatory-focus theory, and job engagement theory. Difficulty: Easy Quest. Category: Concept 41) People who pursue goals that align with their interests and values are more satisfied with their jobs, feel they fit into their organizations better, and may perform better. Answer: TRUE Explanation: People who pursue goals that align with their interests and values are more satisfied with their jobs, feel they fit into their organizations better, and may perform better. LO: 7.3: Contrast the content-based theories of motivation, including self-determination theory, regulatory-focus theory, and job engagement theory. Difficulty: Moderate Quest. Category: Concept
302 richard@qwconsultancy.com
42) A self-regulation strategy that involves striving for goals through advancement and accomplishment is the prevention focus. Answer: FALSE Explanation: A self-regulation strategy that involves striving for goals through advancement and accomplishment is the promotion focus. A self-regulation strategy that involves striving for goals by fulfilling duties and obligations is the prevention focus. LO: 7.3: Contrast the content-based theories of motivation, including self-determination theory, regulatory-focus theory, and job engagement theory. Difficulty: Moderate Quest. Category: Concept 43) The effects of intrinsic motivation may be weaker when incentives are directly tied to performance. Answer: TRUE Explanation: The effects of intrinsic motivation may be weaker when incentives are directly tied to performance (such as a monetary bonus for each call made in a call center). LO: 7.3: Contrast the content-based theories of motivation, including self-determination theory, regulatory-focus theory, and job engagement theory. Difficulty: Moderate Quest. Category: Concept 44) What is self-determination theory? Answer: Self-determination theory is a meta-theory of motivation at work that is concerned with autonomy, intrinsic motivation, extrinsic motivation, and the satisfaction of psychological work needs. The theory proposes that employees' well-being and performance are influenced by the nature of their motivation for certain job activities. LO: 7.3: Contrast the content-based theories of motivation, including self-determination theory, regulatory-focus theory, and job engagement theory. Difficulty: Easy Quest. Category: Concept 45) A(n) ________ is any consequence immediately following a response that increases the probability that the behavior will be repeated. A) conclusion B) reinforcer C) goal D) objective E) referent Answer: B Explanation: Behavior is controlled by reinforcers—any consequences that, when immediately following responses, increase the probability that the behavior will be repeated. LO: 7.4: Understand the differences between the context-based theories of motivation: reinforcement theory and social learning theory. Difficulty: Moderate Quest. Category: Concept
303 richard@qwconsultancy.com
304 richard@qwconsultancy.com
46) The proponents of reinforcement theory view behavior as ________. A) the result of a cognitive process B) environmentally caused C) a reflection of the inner state of the individual D) a function of one's power need E) a product of heredity Answer: B Explanation: Reinforcement theory takes a behavioristic approach, arguing that reinforcement conditions behavior. Reinforcement theorists see behavior as environmentally caused. LO: 7.4: Understand the differences between the context-based theories of motivation: reinforcement theory and social learning theory. Difficulty: Moderate Quest. Category: Concept 47) Which theory suggests that people learn to behave to get something they want or to avoid something they don't want? A) Theory Y B) Theory X C) Social cognitive theory D) Operant conditioning theory E) McClelland's theory of needs Answer: D Explanation: Operant conditioning theory argues that people learn to behave to get something they want or to avoid something they don't want. Unlike reflexive or unlearned behavior, operant behavior is influenced by the reinforcement or lack of reinforcement brought about by its consequences. LO: 7.4: Understand the differences between the context-based theories of motivation: reinforcement theory and social learning theory. Difficulty: Easy Quest. Category: Concept 48) To get the best results while using reinforcement theory, rewards should be ________. A) small and given only once B) large and given at irregular intervals C) given prior to the desired behavior response D) given immediately following the desired behavior E) presented publicly with a large number of witnesses Answer: D Explanation: People will most likely engage in desired behaviors if they are positively reinforced for doing so. Rewards are most effective if they immediately follow the desired response; and that behavior that is not rewarded, or is punished, is less likely to be repeated. LO: 7.4: Understand the differences between the context-based theories of motivation: reinforcement theory and social learning theory. Difficulty: Moderate Quest. Category: Concept
305 richard@qwconsultancy.com
306 richard@qwconsultancy.com
49) Helen, a high school teacher, wants her students to actively participate more in class. She has decided to use reinforcement theory to get the required results. Which of the following methods is she most likely to use? A) She is going to call on students who never participate. B) She is going to yell at students, telling them that they are not making good grades. C) She is going to ask the students that always participate to allow the others to have a chance. D) She is going to model what active participation should look like at the beginning of class. E) She is going to give students an extra mark each time that they contribute. Answer: E Explanation: Helen is most likely going to use operant conditioning in which she gives participating students additional marks. With this practice she is motivating the students by conditioning them to expect a reward each time they demonstrate a specific behavior (speaking up in class). LO: 7.4: Understand the differences between the context-based theories of motivation: reinforcement theory and social learning theory. AACSB: Analytical thinking Difficulty: Moderate Employability Skills: Knowledge Application and Analysis Quest. Category: Application 50) The concept of operant conditioning is a part of the broader concept of ________, which argues that behavior follows stimuli in a relatively unthinking manner. A) equity theory B) expectancy theory C) cognitive behavioral therapy D) behaviorism E) humanism Answer: D Explanation: The concept of operant conditioning was part of B. F. Skinner's broader concept of behaviorism, which argues that behavior follows stimuli in a relatively unthinking manner. Skinner's form of radical behaviorism rejects feelings, thoughts, and other states of mind as causes of behavior. In short, people learn to associate stimulus and response, but their conscious awareness of this association is irrelevant. LO: 7.4: Understand the differences between the context-based theories of motivation: reinforcement theory and social learning theory. Difficulty: Moderate Quest. Category: Concept
307 richard@qwconsultancy.com
51) ________ reflects the view that we can learn through both observation and direct experience. A) Attentional process B) Retention process C) Social learning theory D) Reinforcement process E) Motivation process Answer: C Explanation: Social learning theory LO: 7.4: Understand the differences between the context-based theories of motivation: reinforcement theory and social learning theory. Difficulty: Easy Quest. Category: Concept 52) How do difficult goals result in higher performance? Answer: Goal setting theory is well supported. Evidence strongly suggests that specific goals increase performance and that difficult goals, when accepted, result in higher performance than do easy goals. Goals that are specific explicitly direct attention toward what needs to be accomplished. Once a difficult task has been accepted, we can expect that individual to exert a high level of effort to try to achieve it. LO: 7.4: Understand the differences between the context-based theories of motivation: reinforcement theory and social learning theory. Difficulty: Moderate Quest. Category: Concept 53) Which of the following statements is true regarding goal-setting theory? A) Goal commitment is more likely when individuals have an external locus of control. B) Externally generated feedback is more powerful than self-generated feedback. C) Generalized goals produce a higher level of output than specific goals. D) People do better when they get feedback on how well they are progressing toward their goals. E) Assigned goals generate greater goal commitment in low rather than high power-distance cultures. Answer: D Explanation: People do better when they get feedback on how well they are progressing toward their goals because it helps identify discrepancies between what they have done and what they want to do. Self-generated feedback—with which employees are able to monitor their own progress—is more powerful than externally generated feedback. LO: 7.5: Compare the process-based theories of motivation: expectancy theory, goal-setting theory, and self-efficacy theory. Difficulty: Moderate Quest. Category: Concept
308 richard@qwconsultancy.com
54) According to goal-setting theory, goals are more likely to have a stronger impact on performance when ________. A) goals have long time frames for completion B) tasks are complex rather than simple C) tasks are novel rather than well learned D) goals are easy rather than difficult E) tasks are independent rather than interdependent Answer: E Explanation: Goals seem to affect performance more strongly when tasks are simple rather than complex. Also, goal setting is more effective when the tasks are independent rather than interdependent. LO: 7.5: Compare the process-based theories of motivation: expectancy theory, goal-setting theory, and self-efficacy theory. Difficulty: Moderate Quest. Category: Concept 55) MBO emphasizes goals that are ________. A) tangible, verifiable, and measurable B) achievable, controllable, and profitable C) inspirational, verifiable, and creative D) tangible, rewarding, and assigned E) profitable, attainable, and self-set Answer: A Explanation: Management by objectives (MBO) emphasizes participatively set goals that are tangible, verifiable, and measurable. LO: 7.5: Compare the process-based theories of motivation: expectancy theory, goal-setting theory, and self-efficacy theory. Difficulty: Easy Quest. Category: Concept 56) MBO provides individual employees with ________. A) personal performance objectives B) greater work supervision C) predefined targets D) generalized feedback E) clear-cut growth paths Answer: A Explanation: Four ingredients are common to MBO programs: goal specificity, participation in decision making (including the setting of goals or objectives), an explicit time period, and performance feedback. LO: 7.5: Compare the process-based theories of motivation: expectancy theory, goal-setting theory, and self-efficacy theory. Difficulty: Moderate Quest. Category: Concept
309 richard@qwconsultancy.com
57) In expectancy theory, the degree to which the individual believes exerting a given amount of effort will lead to performance is the ________ relationship. A) performance-reward B) performance-power C) effort-performance D) instrumentality-effort E) rewards-personal goals Answer: C Explanation: In expectancy theory, the degree to which the individual believes exerting a given amount of effort will lead to performance is the effort-performance relationship. LO: 7.5: Compare the process-based theories of motivation: expectancy theory, goal-setting theory, and self-efficacy theory. Difficulty: Easy Quest. Category: Concept 58) The ________ theory suggests that the strength of a tendency to act in a certain way depends on the strength of an expectation that the act will be followed by a given outcome and on the attractiveness of that outcome to the individual. A) two-factor B) self-determination C) expectancy D) self-efficacy E) reinforcement Answer: C Explanation: Expectancy theory suggests that the strength of a tendency to act in a certain way depends on the strength of an expectation that the act will be followed by a given outcome and on the attractiveness of that outcome to the individual. LO: 7.5: Compare the process-based theories of motivation: expectancy theory, goal-setting theory, and self-efficacy theory. Difficulty: Easy Quest. Category: Concept 59) Which of the following refers to an individual's belief that he or she is capable of performing a task? A) Emotional contagion B) Affect intensity C) Self-efficacy D) Self-determination E) Reinforcement Answer: C Explanation: Self-efficacy is an individual's belief that he or she is capable of performing a task. The higher your self-efficacy, the more confidence you have in your ability to succeed. LO: 7.5: Compare the process-based theories of motivation: expectancy theory, goal-setting theory, and self-efficacy theory. Difficulty: Easy Quest. Category: Concept 310 richard@qwconsultancy.com
60) Which of the following ways of increasing self-efficacy involves watching someone else do the task? A) Arousal B) Vicarious modeling C) Verbal persuasion D) Enactive mastery E) Cognitive learning Answer: B Explanation: There are four ways of increasing self-efficacy: (1) give employees relevant experiences with the task (enactive mastery), (2) enable them to watch someone else do the task (vicarious modeling), (3) re-assure the employees, letting them know that they have "what it takes" to do the job (verbal persuasion), and (4) tell them to "get psyched up" (arousal). LO: 7.5: Compare the process-based theories of motivation: expectancy theory, goal-setting theory, and self-efficacy theory. Difficulty: Moderate Quest. Category: Concept 61) Which of the following sources of increasing self-efficacy involves gaining relevant experience with a particular task or job? A) Verbal persuasion B) Enactive mastery C) Vicarious modeling D) Arousal E) Cognitive learning Answer: B Explanation: Enactive mastery involves gaining relevant experience with the task or job. LO: 7.5: Compare the process-based theories of motivation: expectancy theory, goal-setting theory, and self-efficacy theory. Difficulty: Moderate Quest. Category: Concept 62) During the COVID-19 pandemic, health professionals relied on ________ to learn from one another in order to adopt best practices for combatting the pandemic. A) arousal B) enactive mastery C) visualization D) vicarious modeling E) verbal persuasion Answer: D Explanation: During the COVID-19 pandemic, health professionals relied on vicarious modeling to learn from one another in order to adopt best practices for combatting the pandemic. Vicarious modeling involves watching someone do the task. LO: 7.5: Compare the process-based theories of motivation: expectancy theory, goal-setting theory, and self-efficacy theory. Difficulty: Moderate 311 richard@qwconsultancy.com
Quest. Category: Concept 63) With reference to the four sources of self-efficacy, verbal persuasion involves becoming more confident ________. A) because you have gained relevant experience with the particular task or job B) because you see someone else doing the particular task or job C) because someone convinces you that you have the skills necessary to be successful D) because you are rewarded for performing a similar task well E) because you get energized or "psyched up" to perform the particular task or job Answer: C Explanation: Verbal persuasion involves becoming more confident because someone convinces you that you have the skills necessary to be successful. Motivational speakers use this tactic a lot. LO: 7.5: Compare the process-based theories of motivation: expectancy theory, goal-setting theory, and self-efficacy theory. Difficulty: Moderate Quest. Category: Concept 64) Which of the following ways of increasing self-efficacy is often used in training programs? A) Arousal B) Enactive mastery C) Focused training D) Vicarious modeling E) Verbal persuasion Answer: B Explanation: Enactive mastery involves giving employees relevant experiences with the task. Training programs often make use of enactive mastery by having people practice and build their skills. LO: 7.5: Compare the process-based theories of motivation: expectancy theory, goal-setting theory, and self-efficacy theory. Difficulty: Moderate Quest. Category: Concept 65) The Pygmalion effect is based on the concept of ________. A) cognitive evaluation theory B) self-concordance C) self-fulfilling prophecy D) self-determination E) evaluation theory Answer: C Explanation: The Pygmalion effect is a form of self-fulfilling prophecy in which believing something can make it true. LO: 7.5: Compare the process-based theories of motivation: expectancy theory, goal-setting theory, and self-efficacy theory. Difficulty: Moderate Quest. Category: Concept
312 richard@qwconsultancy.com
313 richard@qwconsultancy.com
66) The best way for a manager to use verbal persuasion is through the ________, a form of selffulfilling prophecy in which believing something can make it true. A) confirmation bias B) Pygmalion effect C) anchoring bias D) framing effect E) Electra complex Answer: B Explanation: The best way for a manager to use verbal persuasion is through the Pygmalion effect. The Pygmalion effect is a form of self-fulfilling prophecy in which believing something can make it true. LO: 7.5: Compare the process-based theories of motivation: expectancy theory, goal-setting theory, and self-efficacy theory. Difficulty: Moderate Quest. Category: Concept 67) Last year, managers at Excel Corp excitedly announced a new company initiative designed to encourage sale force employees to step up their game. Under the new initiative, the employee with the most sales at the end of special promo weeks would be assigned exclusive access to the company lounge that is usually reserved for special meetings with clients. Now, managers are trying to figure out why employees were not motivated by the initiative and instead could care less about "winning" the lounge access. Which expectancy theory relationship explains why the initiative failed? A) Expectancy B) Justification C) Valence D) Instrumentality E) Reinforcement Answer: C Explanation: Valence best explains why this relationship failed. Valence refers to the rewardspersonal goals relationship, or the degree to which organizational rewards satisfy an individual's personal goals and the attractiveness of the potential rewards for the individual. LO: 7.5: Compare the process-based theories of motivation: expectancy theory, goal-setting theory, and self-efficacy theory. AACSB: Analytical thinking Difficulty: Hard Employability Skills: Knowledge Application and Analysis Quest. Category: Application
314 richard@qwconsultancy.com
68) With reference to the expectancy theory, which of the following examples indicates a weak rewards-personal goals relationship? A) An employee lacks the skills required to reach the desired performance level. B) An organization's appraisal system assesses nonperformance factors such as creativity and initiative. C) An organization rewards its employees based on factors such as seniority and skill level. D) An employee believes that his manager does not like him and hence expects a poor appraisal. E) An employee works hard in order to be relocated to the Paris office but instead is transferred to Beijing. Answer: E Explanation: According to expectancy theory, the rewards-personal goals relationship indicates the degree to which organizational rewards satisfy an individual's personal goals or needs and the attractiveness of those potential rewards for the individual. If an employee puts in extra effort to be relocated to the Paris office but instead is transferred to Beijing, then it indicates a weak rewards-personal goals relationship. LO: 7.5: Compare the process-based theories of motivation: expectancy theory, goal-setting theory, and self-efficacy theory. AACSB: Analytical thinking Difficulty: Moderate Quest. Category: Application 69) If a manager incorrectly assumes that all employees want the same thing, then according to the expectancy theory, it is most likely to result in a weak ________ relationship. A) performance-achievement B) effort-performance C) performance-reward D) rewards-personal goals E) effort-satisfaction Answer: D Explanation: According to the expectancy theory, the rewards-personal goals relationship indicates the degree to which organizational rewards satisfy an individual's personal goals or needs and the attractiveness of those potential rewards for the individual. So if a manager incorrectly assumes that all employees want the same thing, then according to the expectancy theory, it is most likely to result in a weak rewards-personal goals relationship. LO: 7.5: Compare the process-based theories of motivation: expectancy theory, goal-setting theory, and self-efficacy theory. Difficulty: Moderate Quest. Category: Concept
315 richard@qwconsultancy.com
70) ________ discusses three relationships: effort-performance relationship, performance-reward relationship, and rewards-personal goals relationship. A) Goal-setting theory B) Self-efficacy theory C) Equity theory D) Expectancy theory E) Self-determination theory Answer: D Explanation: The expectancy theory discusses three relationships: effort-performance relationship, performance-reward relationship, and rewards-personal goals relationship. LO: 7.5: Compare the process-based theories of motivation: expectancy theory, goal-setting theory, and self-efficacy theory. Difficulty: Easy Quest. Category: Concept 71) Logan is an employee who processes health insurance forms. Initially he was criticized by his supervisor for sloppy work, but thereafter he improved considerably. Now he consistently processes his forms without errors and even does more than his fair share of work. However, Logan's supervisor has not responded to the extra effort he has put in, giving him no praise or monetary benefits. This leads Logan to believe that his supervisor is biased against him. According to the expectancy theory, in this situation, there is a problem in the ________ relationship. A) rewards-personal goals B) performance-awareness C) performance-reward D) performance-objectives E) performance-achievement Answer: C Explanation: The performance-reward relationship is the degree to which the individual believes performing at a particular level will lead to the attainment of a desired outcome. Employees sometimes, rightly or wrongly, perceive the boss doesn't like them. As a result, they expect a poor appraisal, regardless of effort. LO: 7.5: Compare the process-based theories of motivation: expectancy theory, goal-setting theory, and self-efficacy theory. AACSB: Analytical thinking Difficulty: Hard Employability Skills: Knowledge Application and Analysis Quest. Category: Application
316 richard@qwconsultancy.com
72) Oscar is looking for a new job. He used to be the company's top sales representative and was eagerly expecting to be promoted. However, one of the regional manager's friends was promoted to district manager instead. According to the expectancy theory, Oscar's dissatisfaction with his current job stems from a breakdown in the ________ relationship. A) performance-reward B) effort-performance C) reward-personal goal D) satisfaction-effort E) personal goal-self actualization Answer: A Explanation: The performance-reward relationship explains the degree to which the individual believes performing at a particular level will lead to the attainment of a desired outcome. LO: 7.5: Compare the process-based theories of motivation: expectancy theory, goal-setting theory, and self-efficacy theory. AACSB: Analytical thinking Difficulty: Hard Employability Skills: Knowledge Application and Analysis Quest. Category: Application 73) According to the expectancy theory, the performance-reward relationship is the degree to which ________. A) the individual believes performing at a particular level will lead to desired outcomes B) organizational rewards satisfy an individual's personal goals or needs C) organizational rewards are perceived as attractive by the individual D) the individual believes that exerting a given amount of effort will lead to performance E) organization rewards correspond with the individual's level of effort Answer: A Explanation: According to expectancy theory, the performance-reward relationship indicates the degree to which the individual believes performing at a particular level will lead to the attainment of a desired outcome. LO: 7.5: Compare the process-based theories of motivation: expectancy theory, goal-setting theory, and self-efficacy theory. Difficulty: Easy Quest. Category: Concept
317 richard@qwconsultancy.com
74) Mark is an excellent technical writer. He has never missed a deadline and all his projects are of superior quality. He now wants to telecommute two days a week, so that he can spend more time with his family. He feels that he has proven his reliability. However, his boss is unable to comply with his request and gives him a substantial raise instead. According to the expectancy theory, Mark's disappointment demonstrates a breakdown in the ________ relationship. A) performance-reward B) effort-performance C) rewards-personal goals D) effort-satisfaction E) performance-achievement Answer: C Explanation: According to expectancy theory, the rewards-personal goals relationship indicates the degree to which organizational rewards satisfy an individual's personal goals or needs and the attractiveness of those potential rewards for the individual. LO: 7.5: Compare the process-based theories of motivation: expectancy theory, goal-setting theory, and self-efficacy theory. AACSB: Analytical thinking Difficulty: Hard Employability Skills: Knowledge Application and Analysis Quest. Category: Application 75) Expectancy theory is a process-based theory of motivation. Answer: TRUE Explanation: One of the most widely accepted explanations of motivation is Victor Vroom's expectancy theory Expectancy theory argues that the strength of our tendency to act a certain way depends on the strength of our expectation of a given outcome and its attractiveness. Employees will be motivated to exert a high level of effort when they believe that it will lead to a good performance appraisal, that a good appraisal will lead to organizational rewards such as salary increases and/or intrinsic rewards, and that these rewards will satisfy their personal goals. LO: 7.5: Compare the process-based theories of motivation: expectancy theory, goal-setting theory, and self-efficacy theory. Difficulty: Moderate Quest. Category: Concept 76) In contrast to self-set goals, supervisor-set goals may lead to heightened anxiety and perceptions of uncertainty and threat. Answer: TRUE Explanation: In contrast to self-set goals which can lead to greater employee enthusiasm, supervisor-set goals may lead to heightened anxiety and perceptions of uncertainty and threat. LO: 7.5: Compare the process-based theories of motivation: expectancy theory, goal-setting theory, and self-efficacy theory. Difficulty: Moderate Quest. Category: Concept
318 richard@qwconsultancy.com
77) Self-efficacy can create a positive spiral in which those with high efficacy become more engaged in their tasks and then in turn increase performance, which increases efficacy further. Answer: TRUE Explanation: Self-efficacy can create a positive spiral in which those with high efficacy become more engaged in their tasks and then in turn increase performance, which increases efficacy further. LO: 7.5: Compare the process-based theories of motivation: expectancy theory, goal-setting theory, and self-efficacy theory. Difficulty: Moderate Quest. Category: Concept 78) Self-efficacy refers to an individual's belief that he or she is capable of performing a task. Answer: TRUE Explanation: Self-efficacy (also known as social cognitive theory or social learning theory) refers to an individual's belief that he or she is capable of performing a task. The higher your self-efficacy, the more confidence you have in your ability to succeed. LO: 7.5: Compare the process-based theories of motivation: expectancy theory, goal-setting theory, and self-efficacy theory. Difficulty: Easy Quest. Category: Concept 79) The Pygmalion effect is not a form of self-fulfilling prophecy in which believing something can make it true. Answer: FALSE Explanation: The Pygmalion effect is a form of self-fulfilling prophecy in which believing something can make it true. It is often used to describe "that what one person expects of another can come to serve a self-fulfilling prophecy." LO: 7.5: Compare the process-based theories of motivation: expectancy theory, goal-setting theory, and self-efficacy theory. Difficulty: Moderate Quest. Category: Concept 80) Expectancy theory predicts employees will exert a high level of effort if they perceive a strong relationship between effort and performance, performance and rewards, and rewards and satisfaction of personal goals. Answer: TRUE Explanation: Each of these relationships is, in turn, influenced by other factors. For effort to lead to good performance, the individual must have the ability to perform and perceive the performance appraisal system as fair and objective. The performance–reward relationship will be strong if the individual perceives that performance (rather than seniority, personal favorites, or other criteria) is rewarded. LO: 7.5: Compare the process-based theories of motivation: expectancy theory, goal-setting theory, and self-efficacy theory. Difficulty: Easy Quest. Category: Concept
319 richard@qwconsultancy.com
320 richard@qwconsultancy.com
81) Expectancy theory suggests that employees will be motivated to exert a high level of effort when they believe that it will lead to a good performance appraisal, that a good appraisal will lead to organizational rewards, and that the rewards will satisfy their personal goals. Answer: TRUE Explanation: Expectancy theory suggests that employees will be motivated to exert a high level of effort when they believe that it will lead to a good performance appraisal, that a good appraisal will lead to organizational rewards, and that the rewards will satisfy their personal goals. LO: 7.5: Compare the process-based theories of motivation: expectancy theory, goal-setting theory, and self-efficacy theory. Difficulty: Moderate Quest. Category: Concept 82) Instrumentality in expectancy theory refers to the degree to which the individual believes exerting a given amount of effort will lead to performance. Answer: FALSE Explanation: Instrumentality refers to the performance-reward relationship (the degree to which the individual believes performing at a particular level will lead to the attainment of a desired outcome.) Expectancy refers to the effort-performance reward (the degree to which the individual believes exerting a given amount of effort will lead to performance.) LO: 7.5: Compare the process-based theories of motivation: expectancy theory, goal-setting theory, and self-efficacy theory. Difficulty: Easy Quest. Category: Concept 83) What are MBO programs and how do managers use them? Answer: Management by objectives (MBO) is a program that encompasses specific goals, participatively set, for an explicit time period, with feedback on goal progress. The organization's overall objectives are translated into specific objectives for each level (divisional, departmental, individual). But because lower-unit managers jointly participate in setting their own goals, MBO works from the bottom up as well as from the top down. The result is a hierarchy that links objectives at one level to those at the next. And for the individual employee, MBO provides specific personal performance objectives. Four ingredients are common to MBO programs: goal specificity, participation in decision making (including the setting of goals or objectives), an explicit time period, and performance feedback. LO: 7.5: Compare the process-based theories of motivation: expectancy theory, goal-setting theory, and self-efficacy theory. Difficulty: Moderate Quest. Category: Concept
321 richard@qwconsultancy.com
84) Explain self-efficacy theory as it relates to performance. Answer: Self-efficacy (also known as social cognitive theory or social learning theory) refers to an individual's belief that he or she is capable of performing a task. The higher your self-efficacy, the more confidence you have in your ability to succeed. So, in difficult situations, people with low self-efficacy are more likely to lessen their effort or give up altogether, while those with high self-efficacy will try harder to master the challenge. Self-efficacy can create a positive spiral in which those with high efficacy become more engaged in their tasks and then, in turn, increase performance, which increases efficacy further. Self-efficacy may also create situations in which people get cocky and start to perform more poorly and become less motivated as a result. LO: 7.5: Compare the process-based theories of motivation: expectancy theory, goal-setting theory, and self-efficacy theory. Difficulty: Moderate Quest. Category: Concept 85) Describe the four ways of increasing self-efficacy. Answer: Self-efficacy can be increased in through enactive mastery (gaining relevant experience with the task or job), vicarious modeling (becoming more confident because you see someone else doing the task), verbal persuasion (becoming more confident because someone convinces you that you have the skills necessary to be successful), and arousal (an energized state, which drives a person to complete a task because the person gets "psyched up" and performs better). LO: 7.5: Compare the process-based theories of motivation: expectancy theory, goal-setting theory, and self-efficacy theory. Difficulty: Moderate Quest. Category: Concept 86) Explain the three relationships underlying expectancy theory. Answer: (1) Expectancy: the effort—performance relationship (the degree to which the individual believes exerting a given amount of effort will lead to performance); (2) Instrumentality: the performance—reward relationship (the degree to which the individual believes performing at a particular level will lead to the attainment of a desired outcome); (3) Valence: the rewards—personal goals relationship (the degree to which organizational rewards satisfy an individual's personal goals or needs and the attractiveness of those potential rewards for the individual.) LO: 7.5: Compare the process-based theories of motivation: expectancy theory, goal-setting theory, and self-efficacy theory. Difficulty: Moderate Quest. Category: Concept
322 richard@qwconsultancy.com
87) Explain the three relationships that form the basis of expectancy theory. Answer: Expectancy theory argues that the strength of a tendency to act in a certain way depends on the strength of an expectation that the act will be followed by a given outcome and on the attractiveness of that outcome to the individual. The theory focuses on three relationships: a) The effort-performance relationship is the probability perceived by the individual that exerting a given amount of effort will lead to performance. b) The performance-reward relationship is the degree to which the individual believes that performing at a particular level will lead to the attainment of a desired outcome. c) The rewards-personal goals relationship is the degree to which organizational rewards satisfy an individual's personal goals or needs and the attractiveness of those potential rewards for the individual. LO: 7.5: Compare the process-based theories of motivation: expectancy theory, goal-setting theory, and self-efficacy theory. Difficulty: Easy Quest. Category: Concept 88) Using expectancy theory, explain why a lot of workers are not motivated on their jobs and only do the minimum to get by. Answer: Expectancy theory helps explain why a lot of workers are not motivated on their jobs and do only the minimum necessary to get by. It can also explain employees' efforts toward goal accomplishment. For example, let us consider a work contest. The prize? An employee of the month parking spot. Some may really want this; others could not care less (valence). Maybe if the business shares a parking lot with Trader Joe's (and is thus frequently crowded), this spot would be more coveted. Next, what is the contest like? Let us say it is whoever solicits the most donations to the company charity that year. If an employee puts forth the effort to solicit donations, will they actually receive donations (expectancy)? Maybe people would be more likely to donate in one-dollar increments over $50 increments. Finally, if you do get several people to donate, how likely is it that you will win the parking spot (instrumentality)? Are there multiple parking spots up for grabs? Or just one that Avery wins every year (maybe Avery knows someone who donates a ton of money every year to this contest)? Several factors play into decisions to put forth effort, and expectancy theory suggests that these may be in part explained by valence, instrumentality, and expectancy. LO: 7.5: Compare the process-based theories of motivation: expectancy theory, goal-setting theory, and self-efficacy theory. Difficulty: Moderate Quest. Category: Concept
323 richard@qwconsultancy.com
89) In equity theory, individuals assess the ________. A) cost-benefit ratio B) efficiency-effectiveness trade-off C) quantity-quality trade-off D) outcome-input ratio E) quality of outcome Answer: D Explanation: Equity theory states that individuals compare their job inputs and outcomes with those of others and then respond to eliminate any inequities. Employees perceive what they get from a job situation (salary levels, raises, recognition) in relationship to what they put into it (effort, experience, education, competence) and then compare their outcome-input ratio with that of relevant others. LO: 7.6: Describe the forms of organizational justice, including distributive justice, procedural justice, informational justice, and interactional justice. Difficulty: Moderate Quest. Category: Concept 90) Weston Clark works for a company selling computer maintenance service plans. Business has been so strong that the company has decided to add a new person to the sales team. Weston, who has consistently received strong performance evaluations, has been asked to train the new hire who will be doing the same job as Weston. During the training process, Weston finds out that the new hire, a recent college graduate with no experience, is making more money than he is. Which of the following theories will Weston most likely use to evaluate this situation? A) Reinforcement B) Goal setting C) Equity D) Expectancy E) Operant conditioning Answer: C Explanation: Equity theory states that individuals make comparisons to referent others (compare their job inputs and outcomes with those of others and then respond to eliminate any inequities.) In this situation, because Weston is comparing income, it is likely that he will use equity theory. LO: 7.6: Describe the forms of organizational justice, including distributive justice, procedural justice, informational justice, and interactional justice. AACSB: Analytical thinking Difficulty: Moderate Employability Skills: Knowledge Application and Analysis Quest. Category: Application
324 richard@qwconsultancy.com
91) The equity theory's hypothesis has served as an important precursor to the study of ________. A) Maslow's hierarchy of needs B) distributive justice C) organizational justice D) social learning theory E) procedural justice Answer: C Explanation: Equity theory's propositions have not all held up; however, the hypothesis served as an important precursor to the study of organizational justice, or more simply fairness, in the workplace. LO: 7.6: Describe the forms of organizational justice, including distributive justice, procedural justice, informational justice, and interactional justice. Difficulty: Moderate Quest. Category: Concept 92) Based on equity theory, employees who perceive inequity will make any of the following choices except ________. A) be content with the referent B) change the inputs C) change the outcomes D) distort the perceptions of one's self E) leave the field Answer: A Explanation: Based on equity theory, employees who perceive inequity will change inputs, change outcomes, distort perceptions of self, distort perceptions of others, choose a different referent, or leave the field. LO: 7.6: Describe the forms of organizational justice, including distributive justice, procedural justice, informational justice, and interactional justice. Difficulty: Moderate Quest. Category: Concept 93) Which of the following is an example of changing inputs in regard to equity theory? A) Exerting more effort if underpaid B) Working more hours if underpaid C) Working at a higher rate if working on a piece-rate basis D) Exerting less effort if underpaid E) Working more hours to move up the hierarchy Answer: D Explanation: Exerting less effort if underpaid is an example of changing the inputs. LO: 7.6: Describe the forms of organizational justice, including distributive justice, procedural justice, informational justice, and interactional justice. Difficulty: Moderate Quest. Category: Concept
325 richard@qwconsultancy.com
94) Cynthia Grey recently discovered that her bonus structure is considerably better than that of her two co-workers even though they all have similar positions and performance evaluations. Based on equity theory, we should expect that Cynthia will ________. A) reduce the amount of work that she does on a daily basis B) compare her earnings to those of another group of employees C) increase her productivity and/or the overall quality of her work D) seek a position within the company commensurate with her pay E) begin to look for a position outside of the company Answer: C Explanation: When people see themselves as overrewarded, it creates guilt. According to equity theory, in order to reinstate a sense of equity, Cynthia will change her inputs (exert more if overpaid). She will increase her productivity and/or the overall quality of her work. LO: 7.6: Describe the forms of organizational justice, including distributive justice, procedural justice, informational justice, and interactional justice. AACSB: Analytical thinking Difficulty: Moderate Employability Skills: Knowledge Application and Analysis Quest. Category: Application
326 richard@qwconsultancy.com
Megan graduated from college three years ago and has been working at Sterba Inc. ever since. A conscientious employee, she has consistently received good performance evaluations. She recently found out that a younger colleague, who was just recruited to her team, is drawing a higher salary than she is for doing the same type of work. 95) Following this discovery, Megan starts coming to work late and her productivity begins to suffer. Which of the following is most similar to the scenario mentioned above based on the equity theory? A) Dawn starts coming to work early and stays late once she learns that the mid-term review is around the corner. B) Greg believes he works harder than any of the other members in his department as they often leave the office before him. C) Lisa starts working longer hours after learning that her co-workers earn less than she does for the same work. D) Myrtle produces a higher number of units to compensate for the lower quality of her output. E) Beth submits her resignation after she was passed over for promotion for the second time. Answer: C Explanation: Based on equity theory, employees who perceive inequity can change their inputs, i.e., exert less effort if they are underpaid, or exert more effort if they are overpaid. In the scenario, Megan is underpaid compared to her younger colleague and hence changes her inputs by slacking off at work. This situation is best mirrored by Lisa, who works longer hours (changes inputs) after realizing that she is overpaid compared to her colleagues. Dawn's behavior does not relate to the concept of equity. Greg's behavior represents a distorted perception of self. In Myrtle's situation, she is attempting to change outcomes and not her inputs. Beth chooses to deal with the perceived inequity by leaving the field. LO: 7.6: Describe the forms of organizational justice, including distributive justice, procedural justice, informational justice, and interactional justice. AACSB: Reflective thinking Difficulty: Moderate Quest. Category: Critical Thinking
327 richard@qwconsultancy.com
96) Based on the equity theory, which of the following, if true, would strengthen the argument that Megan is trying to gain a sense of equity by distorting her perception of herself? A) She thinks that her colleague is paid more because she is an Ivy League graduate. B) She believes that she is less capable than her colleague. C) She assumes that her colleague received a higher package due to changes in industry standards. D) She believes that her salary is not commensurate with her skills and experience. E) She believes that she is doing a lot better career-wise compared to the people with whom she graduated. Answer: B Explanation: Based on equity theory, employees who perceive inequity can distort their perceptions of themselves—"I used to think I worked at a moderate pace, but now I realize I work a lot harder than everyone else." If Megan believes that she is less capable than her colleague, then this would imply that she has distorted her perception of herself. If she thinks that her colleague is paid more because she is an Ivy League graduate, then she is distorting her perception of her colleague. This would also be true if she assumes that her colleague received a higher package due to changes in industry standards. If Megan believes that her salary is not commensurate with her skills and experience, then this neither strengthens nor weakens the argument—it only implies that she is experiencing a sense of inequity. If she believes that she is doing a lot better career-wise compared to the people with whom she graduated, then this would mean she is choosing a different referent. LO: 7.6: Describe the forms of organizational justice, including distributive justice, procedural justice, informational justice, and interactional justice. AACSB: Reflective thinking Difficulty: Moderate Quest. Category: Critical Thinking
328 richard@qwconsultancy.com
97) Based on the equity theory, which of the following, if true, would strengthen the argument that Megan is trying to gain a sense of equity by changing her inputs? A) She asks her boss for a raise, citing her years of experience with the company. B) She claims that extrinsic rewards make work seem like a chore and less enjoyable. C) She accepts additional responsibilities but fails to carry them out effectively. D) She takes long breaks from work and shirks her responsibilities. E) She constantly interrupts her colleague, refusing to let her get any work done. Answer: D Explanation: Based on equity theory, employees who perceive inequity can change their inputs, i.e., exert less effort if they are underpaid, or exert more effort if they are overpaid. If Megan takes long breaks from work and shirks her responsibilities, then this would mean that she is trying to gain a sense of equity by reducing her inputs. If she asks her boss for a raise by citing her years of experience with the company, then this would mean that she is trying to gain a sense of equity by changing her outcomes. If she constantly interrupts her colleague and refuses to let her get any work done, then this would mean that she is trying to reduce her colleague's inputs. If she claims that extrinsic rewards make work seem like a chore and less enjoyable, then this would imply that she is distorting her perception of herself. The argument is neither strengthened nor weakened by the fact that she accepts additional responsibilities but fails to carry them out effectively. LO: 7.6: Describe the forms of organizational justice, including distributive justice, procedural justice, informational justice, and interactional justice. AACSB: Reflective thinking Difficulty: Moderate Quest. Category: Critical Thinking 98) Jackie thinks that she is paid a lot less than other employees in her division and feels extremely resentful. She starts taking long breaks and generally wastes time. Her actions resulted from a perceived lack of ________ justice. A) interactional B) interpersonal C) procedural D) distributive E) associative Answer: D Explanation: Equity theory focuses on distributive justice, the employee's perceived fairness of the amount and allocation of rewards among individuals. LO: 7.6: Describe the forms of organizational justice, including distributive justice, procedural justice, informational justice, and interactional justice. AACSB: Analytical thinking Difficulty: Moderate Employability Skills: Knowledge Application and Analysis Quest. Category: Application
329 richard@qwconsultancy.com
99) Edith believes that the methods for determining salary hikes and bonuses in her company are extremely unfair. In this case, Edith perceives a lack of ________ justice. A) interpersonal B) distributive C) associative D) procedural E) interactional Answer: D Explanation: Procedural justice refers to the perceived fairness of the process used to determine the distribution of rewards. LO: 7.6: Describe the forms of organizational justice, including distributive justice, procedural justice, informational justice, and interactional justice. AACSB: Analytical thinking Difficulty: Moderate Employability Skills: Knowledge Application and Analysis Quest. Category: Application 100) Which of the following types of justice focuses on whether employees are treated with dignity and respect? A) Associative justice B) Interactional justice C) Distributive justice D) Interpersonal justice E) Integrative justice Answer: D Explanation: Interpersonal justice reflects whether employees are treated with dignity and respect. LO: 7.6: Describe the forms of organizational justice, including distributive justice, procedural justice, informational justice, and interactional justice. Difficulty: Easy Quest. Category: Concept
330 richard@qwconsultancy.com
101) Why do people who perceive themselves as victims of interactional injustice often blame their immediate supervisor rather than the organization at large? A) Interpersonal justice or injustice is intimately tied to the conveyer of the information. B) Interactional injustice usually occurs during face-to-face encounters. C) When people are not treated with respect, they tend to retaliate against those closest at hand. D) Interactional injustice is in the eyes of those who perceive they are disrespected. E) Interactional injustice is most often the result of the impersonal policies of the organization. Answer: A Explanation: Interpersonal justice describes an individual's perception of the degree to which she is treated with dignity, concern, and respect. When people are treated in an unjust manner (at least in their own eyes), they retaliate (for example, badmouthing a supervisor). Because people intimately connect interactional justice or injustice to the conveyer of the information, we would expect perceptions of injustice to be more closely related to the supervisor. LO: 7.6: Describe the forms of organizational justice, including distributive justice, procedural justice, informational justice, and interactional justice. Difficulty: Moderate Quest. Category: Concept 102) Interpersonal justice refers to an individual's perception of the degree to which he or she is treated with dignity, concern, and respect. Answer: TRUE Explanation: Interpersonal justice refers to an individual's perception of the degree to which he or she is treated with dignity, concern, and respect. LO: 7.6: Describe the forms of organizational justice, including distributive justice, procedural justice, informational justice, and interactional justice. Difficulty: Easy Quest. Category: Concept 103) According to the equity theory, what are the choices made by employees who perceive inequity? Answer: Based on equity theory, employees who perceive inequity will make one of six choices: 1. Change inputs (exert less effort if underpaid, or more if overpaid) 2. Change outcomes (individuals paid on a piece-rate basis can increase their pay by producing a higher quantity of units of lower quality) 3. Distort perceptions of self ("I used to think I worked at a moderate pace, but now I realize I work a lot harder than everyone else.") 4. Distort perceptions of others ("Mike's job isn't as desirable as I thought.") 5. Choose a different referent ("I may not make as much as my brother-in-law, but I'm doing a lot better than my dad did when he was my age.") 6. Leave the field (quit the job) LO: 7.6: Describe the forms of organizational justice, including distributive justice, procedural justice, informational justice, and interactional justice. Difficulty: Moderate Quest. Category: Concept
331 richard@qwconsultancy.com
104) What are distributive justice, procedural justice, and interpersonal justice? Answer: Distributive justice indicates the employee's perceived fairness of the amount and allocation of rewards among individuals. Procedural justice indicates the perceived fairness of the process used to determine the distribution of rewards. Interactional justice indicates an individual's perception of the degree to which he or she is treated with dignity, concern, and respect. Of these three forms of justice, distributive justice is most strongly related to organizational commitment and satisfaction with outcomes such as pay. Procedural justice relates most strongly to job satisfaction, employee trust, withdrawal from the organization, job performance, and citizenship behaviors. There is less evidence about interpersonal justice. LO: 7.6: Describe the forms of organizational justice, including distributive justice, procedural justice, informational justice, and interactional justice. Difficulty: Moderate Quest. Category: Concept 105) High achievers are internally driven if their jobs provide them with personal responsibility, feedback, and moderate risks. Answer: TRUE Explanation: High achievers are internally driven if their jobs provide them with personal responsibility, feedback, and moderate risks. They are not concerned with the effort— performance, performance—reward, or rewards—goal linkages. LO: 7.7: Describe how the contemporary theories of motivation complement one another. Difficulty: Moderate Quest. Category: Concept 106) High achievers are internally driven if their jobs provide them with personal responsibility, feedback, and moderate risks. Answer: TRUE Explanation: High achievers are internally driven if their jobs provide them with personal responsibility, feedback, and moderate risks. LO: 7.7: Describe how the contemporary theories of motivation complement one another. Difficulty: Moderate Quest. Category: Concept Organizational Behavior, 19e (Robbins/Judge) Chapter 8 Motivation: From Concepts to Applications 1) ________ refers to job design. A) The degree to which the job rewards competence B) The way the elements in a job are organized C) The degree to which the job contributes to profit margins D) The degree of decision-making power with the immediate superiors E) The way the job is evaluated in terms of its relative worth to other jobs Answer: B Explanation: Job design is defined as the way the elements in a job are organized. LO: 8.1: Describe how the job characteristics model (JCM) motivates through job design. Difficulty: Easy Quest. Category: Concept 332 richard@qwconsultancy.com
2) ________ defines skill variety in the job characteristics model. A) The degree to which a job requires completion of a whole and identifiable piece of work B) The degree to which work activities generate direct and clear information about performance C) The degree to which a job provides the worker freedom in scheduling and procedure D) The degree to which a job has an impact on the lives or work of other people E) The degree to which a job requires a variety of activities using different skills or talents Answer: E Explanation: The job characteristics model (JCM) describes jobs in terms of five core job dimensions: skill variety, the degree to which a job requires a variety of activities; task identity, the degree to which a job requires completion of a whole and identifiable piece of work; task significance, the degree to which a job has an impact on the lives or work of other people; autonomy, the degree to which a job provides the worker freedom in scheduling; and feedback, the degree to which carrying out work activities generates direct and clear information about performance. LO: 8.1: Describe how the job characteristics model (JCM) motivates through job design. Difficulty: Easy Quest. Category: Concept
333 richard@qwconsultancy.com
3) ________ defines task identity in the job characteristics model. A) The degree to which a job requires completion of a whole and identifiable piece of work B) The degree to which work activities generate direct and clear information about performance C) The degree to which a job provides the worker freedom in scheduling and procedure D) The degree to which a job has an impact on the lives or work of other people E) The degree to which a job requires a variety of different activities Answer: A Explanation: The job characteristics model (JCM) describes jobs in terms of five core job dimensions: skill variety, the degree to which a job requires a variety of activities; task identity, the degree to which a job requires completion of a whole and identifiable piece of work; task significance, the degree to which a job has an impact on the lives or work of other people; autonomy, the degree to which a job provides the worker freedom in scheduling; and feedback, the degree to which carrying out work activities generates direct and clear information about performance. LO: 8.1: Describe how the job characteristics model (JCM) motivates through job design. Difficulty: Easy Quest. Category: Concept 4) Adam Sears is an assembly line employee with Swenson Motors. Though Adam is popular among his supervisors and colleagues, Adam experiences low morale and lack of motivation. He feels frustrated that his job is restricted to fixing nuts and bolts on the car parts. He fears that he has no chances of advancing in his career as he cannot completely assemble a car. Which of the following is true with regard to Adam? A) Adam's job has high task complexity. B) Adam's job has a high span of control. C) Adam's job has low role definition. D) Adam's job has low task identity. E) Adam's job has high task significance. Answer: D Explanation: Adam's job has low task identity. This means that the degree to which the job requires the completion of a whole and identifiable piece of work is low. LO: 8.1: Describe how the job characteristics model (JCM) motivates through job design. AACSB: Analytical thinking Difficulty: Moderate Quest. Category: Application
334 richard@qwconsultancy.com
5) Task significance refers to the degree to which ________ in the job characteristics model. A) a job requires completion of a whole and identifiable piece of work B) a job generates direct and clear information about performance C) a job provides the worker freedom in scheduling work and determining its procedure D) a job bears an impact on the lives or work of other people E) a job requires a variety of different activities Answer: D Explanation: Task significance is the degree to which a job has an impact on the lives or work of other people. LO: 8.1: Describe how the job characteristics model (JCM) motivates through job design. Difficulty: Easy Quest. Category: Concept 6) Samantha Barnes is an emergency medical technician. Recently, during an emergency call, she was able to resuscitate a man who had a cardiac arrest. Subsequently, she spent some time calming the 12-year-old daughter of the patient and looked after her until the rest of the patient's family arrived at the hospital. Based on this description, it can be concluded that Samantha's job is high in ________. A) task identity B) feedback C) extrinsic rewards D) task significance E) job rotation Answer: D Explanation: Although Samantha's job has many job characteristics, the description provided best describes task significance. Task significance refers to the degree to which a job has an impact on the lives of other people. LO: 8.1: Describe how the job characteristics model (JCM) motivates through job design. AACSB: Analytical thinking Difficulty: Moderate Quest. Category: Application 7) Which of the following defines autonomy in the job characteristics model? A) The degree to which a job requires completion of a whole and identifiable piece of work B) The degree to which a job generates direct and clear information about performance C) The degree to which a job provides the worker freedom, independence, and discretion D) The degree to which a job has an impact on the lives or work of other people E) The degree to which a job requires a variety of different activities Answer: C Explanation: Autonomy is the degree to which a job provides the worker freedom, independence, and discretion in scheduling work and determining the procedures in carrying it out. LO: 8.1: Describe how the job characteristics model (JCM) motivates through job design. Difficulty: Easy Quest. Category: Concept
335 richard@qwconsultancy.com
8) ________ in the job characteristics model indicates the degree to which carrying out work activities generates direct and clear information about your own performance. A) Task significance B) Autonomy C) Feedback D) Task identity E) Skill variety Answer: C Explanation: According to the job characteristics model, feedback indicates the degree to which carrying out work activities generates direct and clear information about your own performance. LO: 8.1: Describe how the job characteristics model (JCM) motivates through job design. Difficulty: Easy Quest. Category: Concept 9) Which of the following series of dimensions of the job characteristics model (JCM) combine to create meaningful work? A) Autonomy, task identity, and feedback B) Skill variety, autonomy, and task significance C) Skill variety, autonomy, and feedback D) Feedback, task identity, and task significance E) Skill variety, task identity, and task significance Answer: E Explanation: The first three dimensions in JCM, skill variety, task identity, and task significance, combine to create meaningful work that the worker will view as important, valuable, and worthwhile. LO: 8.1: Describe how the job characteristics model (JCM) motivates through job design. Difficulty: Moderate Quest. Category: Concept 10) The core dimensions of the job characteristics model can be combined into a single predictive index called ________. A) extrinsic motivation score B) personal outcome score C) job diagnostic score D) motivating potential score E) potential development score Answer: D Explanation: The core dimensions of the job characteristics model can be combined into a single predictive index, called the motivating potential score. LO: 8.1: Describe how the job characteristics model (JCM) motivates through job design. Difficulty: Moderate Quest. Category: Concept
336 richard@qwconsultancy.com
11) Autonomy in the job characteristics model can best be described as ________. A) the degree to which carrying out the work activities required by a job results in the individual obtaining direct and clear information about the effectiveness of his performance B) the degree to which a job provides substantial freedom and discretion to the individual in scheduling the work and in determining the procedures to be used in carrying it out C) the degree to which a job has a substantial impact on the lives or work of other people D) the degree to which a job requires completion of a whole and identifiable piece of work E) the degree to which a job affects the lives or work of other people Answer: B Explanation: Autonomy is the degree to which a job provides substantial freedom and discretion to the individual in scheduling the work and in determining the procedures to be used in carrying it out. LO: 8.1: Describe how the job characteristics model (JCM) motivates through job design. Difficulty: Moderate Quest. Category: Concept 12) The job characteristics model describes any job in terms of five core job dimensions, and these five dimensions are skill variety, task significance, recognition, reward, and feedback. Answer: FALSE Explanation: The job characteristics model describes any job in terms of five core job dimensions, and these five dimensions are skill variety, task significance, autonomy, task identity, and feedback. Task identity is the degree to which a job requires completion of a whole and identifiable piece of work. LO: 8.1: Describe how the job characteristics model (JCM) motivates through job design. Difficulty: Easy Quest. Category: Concept 13) Cindy Washington is one of the best paramedics in her town. Cindy was recently awarded a medal by the town mayor for her outstanding performance. Based on this information, we can conclude that Cindy's job has low task significance Answer: FALSE Explanation: Task significance is the degree to which a job affects the lives or work of other people. Being an outstanding paramedic, Cindy is likely to have saved the lives of many which would be a sign of high task significance. LO: 8.1: Describe how the job characteristics model (JCM) motivates through job design. AACSB: Analytical thinking Difficulty: Moderate Employability Skills: Knowledge Application and Analysis Quest. Category: Application
337 richard@qwconsultancy.com
14) Feedback is the degree to which carrying out work activities generates direct and clear information about one's own performance. Answer: TRUE Explanation: Feedback is the degree to which carrying out work activities generates direct and clear information about one's own performance. LO: 8.1: Describe how the job characteristics model (JCM) motivates through job design. Difficulty: Easy Quest. Category: Concept 15) Jobs with high autonomy give workers a feeling of personal responsibility for the results. Answer: TRUE Explanation: Autonomy is the degree to which a job provides the worker freedom, independence, and discretion in scheduling work and determining the procedures in carrying it out. Jobs with high autonomy give workers a feeling of personal responsibility for the results. LO: 8.1: Describe how the job characteristics model (JCM) motivates through job design. Difficulty: Easy Quest. Category: Concept 16) Explain the different dimensions of the job characteristics model. Answer: The job characteristics model (JCM), which proposes that we can describe any job in terms of five core job dimensions. These are explained below. a) Skill variety is the degree to which a job requires a variety of different activities so the worker can use a number of different skills and talent. b) Task identity is the degree to which a job requires completion of a whole and identifiable piece of work. c) Task significance is the degree to which a job affects the lives or work of other people. d) Autonomy is the degree to which a job provides the worker freedom, independence, and discretion in scheduling work and determining the procedures in carrying it out. e) Feedback is the degree to which carrying out work activities generates direct and clear information about one's own performance. LO: 8.1: Describe how the job characteristics model (JCM) motivates through job design. Difficulty: Moderate Quest. Category: Concept
338 richard@qwconsultancy.com
17) Andrew is a software tester. He runs through the same types of programs day after day looking for bugs and reporting them. He is taking night classes on programming. Often, he knows the best solution to many of the bugs, but he is still learning to code. Describe two job characteristics that Andrew is striving to improve, and explain two ways that Andrew's manager can redesign his current job to help him reach his goals. Answer: Andrew is striving to have greater task identity, which is the degree to which a job requires completion of a whole and identifiable piece of work. He would like to not only look for the programming bugs, but be able to fix them. He is also looking for skill variety, or the degree to which a job requires a variety of different activities so the worker can use a number of different skills and talent. It is clear that he is bored with only the testing aspect of his job. Andrew's boss needs to redesign his job to include aspects of job enrichment, which refers to the vertical expansion of jobs. Job enrichment increases the degree to which the worker controls the planning, execution, and evaluation of his or her work. One way to enrich Andrew's job would be to put him in contact with his internal clients, the programmers. They could establish a set of bug recommendations that Andrew could fix, where he could begin to learn the types of code required. His boss should also open the feedback channels so that the programmers can tell Andrew when he is making proper adjustments and using his new skills adequately. LO: 8.1: Describe how the job characteristics model (JCM) motivates through job design. AACSB: Analytical thinking Difficulty: Hard Employability Skills: Knowledge Application and Analysis Quest. Category: Application 18) Describe the five core job dimensions of the job characteristics model. Answer: 1. Skill variety is the degree to which a job requires different activities using specialized skills and talents. 2. Task identity is the degree to which a job requires completion of a whole and identifiable piece of work. 3. Task significance is the degree to which a job affects the lives or work of other people. 4. Autonomy is the degree to which a job provides the worker freedom, independence, and discretion in scheduling work and determining the procedures for carrying it out. 5. Feedback is the degree to which carrying out work activities generates direct and clear information about your own performance. LO: 8.1: Describe how the job characteristics model (JCM) motivates through job design. AACSB: Analytical thinking Difficulty: Moderate Quest. Category: Concept
339 richard@qwconsultancy.com
19) Explain how participative management can enhance skill variety, task identity, and autonomy as described in the Job Characteristics Model (JCM). Answer: Skill variety, the degree to which a job requires a variety of different activities so the worker can use a number of different skills and talent, can be enhanced by participative management because it helps the employees view their tasks from a different perspective. It helps them understand why the management requires high productivity and efficiency from them. By learning to think differently, the employees learn different skills at the mental and relational levels. Task identity, the degree to which a job requires completion of a whole and identifiable piece of work, can be enhanced by participative management by allowing employees to make decisions on the spot without management approval. For example, the woman at the Wegman's bakery that convinced management to let her include a traditional family recipe in the product line will feel a much stronger sense of task identity. Autonomy, which is the freedom in determining work procedures and carrying them out, is enhanced by participative management because the employee is able to participate in the decision-making process. Having a voice in the process increases the sense of autonomy and control. LO: 8.1: Describe how the job characteristics model (JCM) motivates through job design. AACSB: Analytical thinking Difficulty: Hard Quest. Category: Synthesis 20) Compare and contrast skill variety and job rotation. Answer: These represent ways in which the organization can promote development of relevant skills among its employees. Skill variety refers to the degree to which a job requires a variety of different activities tapping different abilities and skills. This primarily involves the relationship between the employee and his or her job and is an individualistic method of enhancing intrinsic motivation. Job rotation involves the periodic shifting of an employee from one task to another with similar skill requirements at the same organizational level. The strengths of job rotation are that it reduces boredom, increases motivation, and helps employees better understand how their work contributes to the organization. An indirect benefit is that employees with a wider range of skills give management more flexibility in scheduling work, adapting to changes, and filling vacancies. LO: 8.1: Describe how the job characteristics model (JCM) motivates through job design. AACSB: Analytical thinking Difficulty: Moderate Quest. Category: Synthesis
340 richard@qwconsultancy.com
21) Which of the following statements is true regarding job rotation? A) It decreases the flexibility with which an organization can adapt to a change. B) It decreases employee motivation. C) It decreases productivity in the short run. D) It decreases supervisory workload. E) It decreases the flexibility in scheduling work. Answer: C Explanation: Job rotation increases the flexibility with which an organization can adapt to a change. It reduces boredom and increases motivation. Productivity tends to reduce when a worker moves into a new position, and supervisory workload tends to increase because supervisors may have to spend more time answering questions and monitoring the work of recently rotated employees. LO: 8.2: Compare the main ways jobs can be redesigned. Difficulty: Moderate Quest. Category: Concept 22) ________ refers to the periodic shifting of an employee from one task to another with similar skill requirements at the same organizational level. A) Job enlargement B) Job analysis C) Job rotation D) Job sharing E) Job enrichment Answer: C Explanation: The periodic shifting of an employee from one task to another with similar skill requirements at the same organizational level is called job rotation. LO: 8.2: Compare the main ways jobs can be redesigned. Difficulty: Easy Quest. Category: Concept 23) Job rotation requires the ________. A) presence of high level of autonomy among workers B) presence of high need for affiliation among workers C) presence of employees with similar skill sets and requirements D) presence of alternative work arrangements for employees to avail E) presence of employee involvement programs Answer: C Explanation: The periodic shifting of an employee from one task to another with similar skill requirements at the same organizational level is called job rotation. LO: 8.2: Compare the main ways jobs can be redesigned. Difficulty: Moderate Quest. Category: Concept
341 richard@qwconsultancy.com
24) Managers at Flavors, a restaurant chain, train their employees such that in the absence of employees, someone trained in the same skills can step in and do the job equally well. Thus, many modules in training are extensive as they provide employees with details of the skill sets required for different jobs. In practice, this lengthy training program does help the company as a well-trained and flexible workforce is at their disposal at all times. The managers at Flavors use ________. A) job rotation B) vertical enhancement C) telecommuting D) job sharing E) flextime Answer: A Explanation: This is an example of job rotation, or the periodic shifting of an employee from one task to another with similar skill requirements at the same organizational level. LO: 8.2: Compare the main ways jobs can be redesigned. AACSB: Analytical thinking Difficulty: Moderate Quest. Category: Application
342 richard@qwconsultancy.com
CalTrack Services has been facing employee-related issues like decreased productivity, low morale, and poor organizational commitment in recent times. At a meeting to discuss strategies to address these issues of immediate concern, Joe McEnroe, the production manager, proposed that the roles of employees working on key projects be enhanced to increase autonomy. He felt that this would enhance productivity, motivation, and accountability. Taking a cue from McEnroe, Paul Wilbur suggested job rotation as a solution to the problem they were facing. Wilbur felt that a change of job roles would make the employees keener to learn and perform. He emphasized that job rotation would help management create a larger skill base to choose from, depending on the needs of specific projects. 25) Which of the following, if true, would weaken the argument of adopting job rotation? A) The employees of the company have similar skill sets which are suited to transfer of learning. B) The company has been conducting professional training programs to keep all employees upto-date with the developments in the field. C) AmWeb, one of the major rival firms, witnessed a substantial rise in profits following its cross-training program for employees. D) The research evidence favoring job rotation has rarely used samples from the service industry. E) The company is now entering its busiest business season with multiple project deadlines. Answer: E Explanation: Job rotation is the periodic shifting of an employee from one task to another with similar skill requirements at the same organizational level. With the busiest business season coming up, time is at a premium for this company, and job rotation, though beneficial in the long run, will result in wastage of time for all employees involved; it will also cause a drop in the immediate productivity. If the employees of the company have similar skill sets which are suited to transfer of learning, then this would strengthen the argument in favor of adopting job rotation. The fact that the company has been conducting professional training programs to keep all employees up-to-date with the developments in the field does not weaken the argument of adopting job rotation. The fact that AmWeb, one of the major rival firms, witnessed a substantial rise in profits following its cross-training program for employees does not strengthen or weaken the argument of adopting job rotation for CalTrack. The fact that research evidence favoring job rotation has rarely used samples from the service industry does not strengthen or weaken the argument of adopting job rotation. LO: 8.2: Compare the main ways jobs can be redesigned. AACSB: Analytical thinking Difficulty: Hard Employability Skills: Critical Thinking Quest. Category: Critical Thinking
343 richard@qwconsultancy.com
26) Which of the following, if true, would strengthen the argument in favor of adopting job rotation? A) The company is currently facing numerous budget constraints. B) The company can enhance productivity of existing employees in various areas for different projects. C) Two front-level managers have resigned in the last quarter as they were not promoted. D) The company has launched a new brand which is cannibalizing one of its existing products. E) The company has recently given employees new responsibilities by combining their routine tasks into natural work units. Answer: B Explanation: If the company can enhance productivity of existing employees in various areas for different projects, then this would strengthen the argument for adopting job rotation. The strengths of job rotation are that it reduces boredom, increases motivation, and helps employees better understand how their work contributes to the organization. An indirect benefit is that employees with a wider range of skills give management more flexibility in scheduling work, adapting to changes, and filling vacancies. Since CalTrack needs to increase productivity, job rotation would help the company create a pool of workers who are skilled in different areas that are important for different projects. If the company is currently facing numerous budget constraints, it would weaken the argument of adopting job rotation. The fact that two front-level managers have resigned in the last quarter as they were not promoted does not strengthen the argument in favor of adopting job rotation. The fact that the company has launched a new brand which is cannibalizing one of its existing products does not strengthen the argument of adopting job rotation. If the company has recently given employees new responsibilities by combining their routine tasks into natural work units, then this does not strengthen the argument in favor of adopting job rotation. LO: 8.2: Compare the main ways jobs can be redesigned. AACSB: Analytical thinking Difficulty: Hard Employability Skills: Critical Thinking Quest. Category: Critical Thinking 27) One way to possibly make jobs more ________ would be to connect employees with the beneficiaries of their work or service that they provide. A) task significant B) prosocially motivating C) job relational D) task oriented E) enriching Answer: B Explanation: One way to make jobs more prosocially motivating is to better connect employees with the beneficiaries of their work by relating stories from customers who have found the company's products or services to be helpful. LO: 8.2: Compare the main ways jobs can be redesigned. Difficulty: Easy Quest. Category: Concept
344 richard@qwconsultancy.com
345 richard@qwconsultancy.com
28) Relational job design relates to ________. A) periodic shifting of an employee from one task to another with similar skill requirements at the same organizational level B) expansion of jobs by increasing the degree to which the worker controls the planning, execution, and evaluation of the work C) redistribution of power within the organization through work councils and board representations D) distribution of a significant degree of power between subordinates and supervisors E) design of jobs and work so employees are motivated to promote the well-being of an organization's beneficiaries Answer: E Explanation: How can managers design work so employees are motivated to promote the wellbeing of the organization's beneficiaries (customers, clients, patients, and employees) is described as relational job design. Relational job design shifts the spotlight from the employee to those whose lives are affected by the job that employee performs. LO: 8.2: Compare the main ways jobs can be redesigned. Difficulty: Moderate Quest. Category: Concept 29) ________ has a focus on prosocial motivation and is especially salient for organizations with corporate social responsibility. A) Relational job design B) Job rotation C) Flextime D) The job characteristics model E) Telecommuting Answer: A Explanation: Relational job design, with its focus on prosocial motivation, is an especially salient topic for organizations with corporate social responsibility (CSR) initiatives. LO: 8.2: Compare the main ways jobs can be redesigned. Difficulty: Moderate Quest. Category: Concept 30) Job rotation can be applied in any setting where cross-training is feasible, from manufacturing floors to hospital wards. Answer: TRUE Explanation: Job rotation can be applied in any setting where cross-training is feasible, from manufacturing floors to hospital wards. The use of job rotation has been shown to increase job satisfaction and organizational commitment. LO: 8.2: Compare the main ways jobs can be redesigned. Difficulty: Moderate Quest. Category: Concept
346 richard@qwconsultancy.com
31) Job enrichment improves employees' skills by periodically shifting an employee from one task to another with similar skill requirements at the same organizational level. Answer: FALSE Explanation: Job rotation is the periodic shifting of an employee from one task to another. LO: 8.2: Compare the main ways jobs can be redesigned. Difficulty: Easy Quest. Category: Concept 32) Personal contacts with beneficiaries is a required component of relational job design. Answer: FALSE Explanation: Personal contacts with beneficiaries may not always be necessary for relational job design. Once a child's chemotherapy comes to an end at one of the many cancer centers across the United States and he or she has successfully defeated cancer, it has become a tradition for the child to ring a bell, the sound of which is often broadcast throughout many areas of the hospital. The mere act of hearing this bell is inspiring to the staff. LO: 8.2: Compare the main ways jobs can be redesigned. Difficulty: Moderate Quest. Category: Concept 33) Define job enrichment and explain how job enrichment differs from enlarging a job. Answer: In job enrichment, high-level responsibilities are added to the job to increase a sense of purpose, direction, and meaning and intrinsic motivation. Enriching a job in this way is different from enlarging it, or adding more tasks and requirements. It involves adding another layer of responsibility and meaning. Job enrichment has its roots in Herzberg's theories of providing hygiene, or motivating factors, to the job to increase motivation. Sometimes, enrichment is not rigidly controlled by management; employees, especially those in occupations experiencing high industry growth, have been known to enrich their own jobs (and become satisfied as a result). LO: 8.2: Compare the main ways jobs can be redesigned. AACSB: Analytical thinking Difficulty: Moderate Quest. Category: Concept
347 richard@qwconsultancy.com
34) Nora Elm is a newly-appointed HR manager at Seven Oaks, a famous PR firm that is not doing too well at present. As she assumed the post of the HR manager, everyone impressed upon her the need to energize the employees and motivate them to give their best to the job. After she met with the employees and managers, she felt that many employees were demotivated because they had attained their maximum potential in their current job roles. Additionally, she felt that though the management believed strongly in its employee-friendly nature, this nature was not manifested in its actions. Those employees who were on the board had begun to mimic the management's action of attributing the lack of any constructive change to company policies. Describe any two changes that Nora should propose at the meeting with a rationale for each. Answer: The changes that Nora could propose are job rotation or job enrichment. Job rotation and job enrichment: Since employees feel they have nothing more to learn in their current jobs, the act of job rotation would help them learn something new which would be motivating and beneficial to the company as well. For employees who cannot be moved to another job, different strategies of job enrichment, like enhancing the job vertically or combining their tasks into natural work units, would be an option. These would directly satisfy some dimensions of the job characteristics model. Participative management: The employees at Seven Oaks are demotivated not only because of the stagnant nature of their jobs, but also because the company is not truly employee-friendly. The company has nominated some employees to its board of representatives but this is only a symbolic act of employee involvement. Participative management, wherein employee and managers engage in joint decision making, will go a long way in motivating employees by giving them more autonomy and representation in the real sense. LO: 8.2: Compare the main ways jobs can be redesigned. AACSB: Analytical thinking Difficulty: Hard Quest. Category: Synthesis 35) Which is not a benefit of flextime? A) Reduced absenteeism B) Increased pay C) Increased productivity D) Reduced tardiness E) Increased autonomy Answer: B Explanation: Flextime tends to reduce absenteeism and frequently improves productivity, probably for several reasons. Employees can schedule their work hours to align with personal demands, reducing tardiness and absences, and they can work when they are most productive. Flextime can also help employees balance work and family lives. LO: 8.3: Explain how specific alternative work arrangements can motivate employees. Difficulty: Moderate Quest. Category: Concept
348 richard@qwconsultancy.com
36) Beyond redesigning the nature of the work itself and involving employees in decisions, another approach to making the work environment more motivating is to alter work arrangements to meet employee needs. Which of the following is designed to give employees greater control of their schedule? A) Flextime B) Employee recognition programs C) Job rotation D) Job enlargement E) Job enrichment Answer: A Explanation: With flextime, employees can schedule their work hours to align with personal demands, reducing tardiness and absences, and they can work when they are most productive. LO: 8.3: Explain how specific alternative work arrangements can motivate employees. Difficulty: Easy Quest. Category: Concept 37) Employees who use the flextime option have to ________. A) work from the office for three days of the week B) work from home on a relatively permanent basis C) work from the office based on a schedule fixed with the compatible partner they function with D) work from the office during the common core period and put in additional hours per their convenience E) work from home only two days a week Answer: D Explanation: With flextime, employees can schedule their work hours to align with personal demands, reducing tardiness and absences, and they can work when they are most productive. Employees must work a specific number of hours per week but are free to vary their hours of work within certain limits. All employees have to be at the office for the common core period, and flextime employees enjoy flexibility as to when they put in the extra hours. LO: 8.3: Explain how specific alternative work arrangements can motivate employees. Difficulty: Easy Quest. Category: Concept 38) Which of the following refers to an arrangement that allows two or more individuals to split a traditional 40-hour-a-week job? A) Flextime B) Job sharing C) Gainsharing D) Telecommuting E) Job rotation Answer: B Explanation: Job sharing allows two or more individuals to split a traditional 40-hour-a-week job. One might perform the job from 8:00 A.M. to noon and the other from 1:00 P.M. to 5:00 P.M., or the two could work full but alternate days. LO: 8.3: Explain how specific alternative work arrangements can motivate employees. Difficulty: Easy 349 richard@qwconsultancy.com
Quest. Category: Concept 39) Janice and Shane are both senior software analysts. They have worked together on projects for six years and get along very well. Janice is anticipating the arrival of her first child and will not be able to work on a full-time basis in the future. Shane is contemplating opening his own business as a home media installation consultant and does not want to continue to work full time. They both need some income. Which of the following alternative job structures would be best for Janice and Shane? A) Job enlargement B) Telecommuting C) Job rotation D) Job sharing E) Job enrichment Answer: D Explanation: Since Janice and Shane are looking to work for fewer than the normal work hours required, the best option for them would be to participate in job sharing, which allows two or more individuals to split a traditional 40-hour-a-week job. The other structure options are still 40-hour-a-week commitments. LO: 8.3: Explain how specific alternative work arrangements can motivate employees. AACSB: Analytical thinking Difficulty: Moderate Quest. Category: Application 40) Job sharing ________. A) expands jobs by increasing the degree to which the worker controls the planning, execution, and evaluation of the work B) is also called cross-training C) allows two or more individuals to split a traditional 40-hour-a-week job D) involves periodic shifting of an employee from one task to another with similar skill requirements at the same organizational level E) is a method of job enrichment Answer: C Explanation: Job enrichment expands jobs by increasing the degree to which the worker controls the planning, execution, and evaluation of the work. Job sharing is not a method of job enrichment, it is an alternative work arrangement. Job rotation involves periodic shifting of an employee from one task to another with similar skill requirements at the same organizational level, and it is also called cross-training. Job sharing allows two or more individuals to split a traditional 40-hour-a-week job. LO: 8.3: Explain how specific alternative work arrangements can motivate employees. Difficulty: Easy Quest. Category: Concept
350 richard@qwconsultancy.com
41) ________ enables the organization to draw on the talents of more than one individual in a given job. A) Telecommuting B) Flextime C) Gainsharing D) Profit sharing E) Job sharing Answer: E Explanation: Job sharing allows an organization to draw on the talents of more than one individual in a given job. It is an opportunity to get two heads but pay for only one. LO: 8.3: Explain how specific alternative work arrangements can motivate employees. Difficulty: Moderate Quest. Category: Concept 42) The major drawback of job sharing from management's perspective is ________. A) an increase in supervisory workload B) increased costs for providing supporting infrastructure (such as desk and computer) to two employees instead of one C) difficulty in finding compatible partners D) difficulty in designing the reward structure E) difficulty in coordinating schedules Answer: C Explanation: The major drawback of job sharing from management's perspective is finding compatible pairs of employees who can successfully coordinate the intricacies of one job. LO: 8.3: Explain how specific alternative work arrangements can motivate employees. Difficulty: Moderate Quest. Category: Concept
351 richard@qwconsultancy.com
43) After the training and development manager at Add Worth, Nicole Hayes, resigned, the CEO has made it a priority that the position be filled soon by someone of equal expertise and experience as Nicole. To locate the right candidate, Jennifer Ray, the HR manager, has been contacting recruitment firms, checking job sites, and pushing for referrals from the employees. However, she has only found candidates with little or no experience in the field who were willing to work full time for the position. At the next meeting with the CEO, Jennifer proposes an alternative strategy; she suggests hiring two experienced candidates who are open to the idea of handling the training needs on a part-time basis as a team. The CEO listens to her proposal but is not too convinced by it. Which of the following, if true, would strengthen Jennifer's proposal to use job sharing for the vacant position? A) Trainers are exempted from the requirement of being at the office throughout the common core period of the workday. B) The employees of the company on the work council have planned to demand the flextime option. C) A good proportion of the company's prized workforce comprises employees from the baby boomer generation who are set to retire in the impending future. D) The headhunting firms that Jennifer contacted are usually helpful in generating a good pool of potential candidates. E) A rival firm recently adopted telecommuting to reduce administrative costs cutting into its bottom line. Answer: C Explanation: The fact that a good proportion of the company's prized workforce comprises employees of the baby boomer generation who are set to retire in the near future does strengthen the implementation of job sharing. Job sharing is useful as it helps utilize the talent of employees who do not wish to work full time like retirees and housewives. If trainers are exempted from the requirement of being at the office throughout the common core period of the workday, that does not favor the adoption of job sharing. The fact that the employees of the company on the work council have planned to demand the flextime option does not favor the adoption of job sharing. The fact that the headhunting firms that Jennifer contacted are usually helpful in generating a good pool of potential candidates does not mean that Add Worth must adopt job sharing. The fact that a rival firm recently adopted telecommuting to reduce administrative costs cutting into its bottom line does not strengthen the argument that Add Worth must implement job sharing. LO: 8.3: Explain how specific alternative work arrangements can motivate employees. AACSB: Analytical thinking Difficulty: Hard Employability Skills: Critical Thinking Quest. Category: Critical Thinking
352 richard@qwconsultancy.com
44) ________ refers to the option of working at home at least two days a week on a computer linked to the employer's office. A) Job sharing B) Social loafing C) Offshoring D) Telecommuting E) Alternative work-sharing Answer: D Explanation: Telecommuting refers to an alternative work arrangement that enables the employee to work at home at least two days a week on a computer linked to the employer's office. LO: 8.3: Explain how specific alternative work arrangements can motivate employees. Difficulty: Easy Quest. Category: Concept 45) Amanda McPherson is a working mother with one child and an ailing parent to look after in addition to her duties as a wife and job responsibilities as a legal consultant. Since Amanda always feels pressed for time, the news that the consultancy she works for would allow her to work from home two days a week came as a great relief to her. She now feels more motivated to work for the company, as the company has taken into account her personal and professional needs. The company has allowed Amanda to use the option of ________. A) gainsharing B) offshoring C) telecommuting D) homeshoring E) job sharing Answer: C Explanation: Telecommuting refers to an alternative work arrangement that enables the employee to work at home at least two days a week on a computer linked to the employer's office. LO: 8.3: Explain how specific alternative work arrangements can motivate employees. AACSB: Analytical thinking Difficulty: Moderate Quest. Category: Application
353 richard@qwconsultancy.com
46) A multinational company in a developing country is operating amidst severe space constraints, and the infrastructural conditions in the city are bad, causing a lot of lost man-hours. Anticipating rapid growth in the months to come, the company has created a recruitment plan based on its revenue expectations. However, it wants to manage this expansion without undertaking an expensive physical expansion. Which of the following could be a solution for the company? A) Profit sharing B) Group shifting C) Telecommuting D) Gainsharing E) Codetermining Answer: C Explanation: Telecommuting refers to an alternative work arrangement that enables the employee to work at home at least two days a week on a computer linked to the employer's office. LO: 8.3: Explain how specific alternative work arrangements can motivate employees. AACSB: Analytical thinking Difficulty: Moderate Employability Skills: Knowledge Application and Analysis Quest. Category: Application 47) A major benefit of telecommuting to society is ________. A) decreased productivity B) increased turnover C) reduced morale D) difficulty in scheduling work E) reduced carbon emissions Answer: E Explanation: Telecommuting has several benefits. It increases performance and job satisfaction; to a lesser degree, it reduces role stress and turnover intentions. Employees who work virtually more than 2.5 days a week tend to experience the benefits of reductions in work—family conflict more intensely than those who are in the office the majority of their workweek. Beyond the benefits to organizations and their employees, telecommuting has potential benefits to society. One study estimated that if people in the United States telecommuted half the time, carbon emissions would be reduced by approximately 51 metric tons per year. Environmental savings could come from lower office energy consumption, fewer traffic jams that emit greenhouse gases, and a reduced need for road repairs. LO: 8.3: Explain how specific alternative work arrangements can motivate employees. Difficulty: Moderate Employability Skills: Knowledge Application and Analysis Quest. Category: Concept
354 richard@qwconsultancy.com
48) Professionals in ________ are best suited to telecommuting. A) retail B) food service C) insurance D) construction E) manufacturing Answer: C Explanation: Finance, insurance, management, information technology, and other "knowledge workers" professionals can telecommute for greater periods of time without productivity loss than workers in manufacturing, construction, retail, and food service. LO: 8.3: Explain how specific alternative work arrangements can motivate employees. Difficulty: Moderate Quest. Category: Concept 49) In today's team-focused workplace, a major downside of telecommuting is ________. A) reduced feelings of isolation B) reduced job satisfaction C) dull, repetitive work D) increased social loafing E) demotivation Answer: D Explanation: In today's team-focused workplace, telecommuting may lead to social loafing (i.e., employees shirking responsibility in a team setting), especially when the employee has many family responsibilities but their teammates do not. LO: 8.3: Explain how specific alternative work arrangements can motivate employees. Difficulty: Moderate Quest. Category: Concept 50) ________ is part of the supporting element in the assess-create-support framework developed by researchers during the COVID-19 pandemic to help companies establish effective telecommuting practices. A) Need assessment B) Empowerment C) Role-modeling D) Autonomy instruction E) Flextime efficiency Answer: C Explanation: During the COVID-19 pandemic, researchers developed the assess-create-support framework to help aid in establishing effective telecommuting practices. This framework involves assessing employee telecommuting needs, creating practices that empower employees to maintain work—home boundaries, and supporting these practices through adjustment, encouragement, role-modelling, and monitoring. LO: 8.3: Explain how specific alternative work arrangements can motivate employees. AACSB: Diverse and multicultural work environments Difficulty: Moderate Quest. Category: Concept 355 richard@qwconsultancy.com
51) A sales associate working from home is telecommuting; another sales associate traveling and working from her car on a business trip is not. Answer: TRUE Explanation: This is true in the sense that a sales associate working from home is telecommuting, but a sales associate working from her car on a business trip is not. LO: 8.3: Explain how specific alternative work arrangements can motivate employees. AACSB: Analytical thinking Difficulty: Moderate Quest. Category: Concept 52) Most flextime arrangements require all employees to be at their jobs during the common core period. Answer: TRUE Explanation: Most flextime arrangements require all employees to be at their jobs during the common core period, but they may accumulate their other two hours before, after, or before and after that. LO: 8.3: Explain how specific alternative work arrangements can motivate employees. Difficulty: Moderate Quest. Category: Concept 53) The major advantage of flextime is that it can be used for all categories of jobs. Answer: FALSE Explanation: Flextime's major drawback is that it's not applicable to every job or every worker. It works well with clerical tasks for which an employee's interaction with people outside his or her department is limited. It is not a viable option for receptionists, sales personnel in retail stores, or people whose service jobs require them to be at their workstations at predetermined times. LO: 8.3: Explain how specific alternative work arrangements can motivate employees. Difficulty: Moderate Quest. Category: Concept 54) People with more education are more apt to work from home. Answer: TRUE Explanation: Telecommuting seems to mesh with the cultural transition to knowledge work (which often can be performed anywhere) and, people with more education are more apt to work from home. LO: 8.3: Explain how specific alternative work arrangements can motivate employees. Difficulty: Moderate Quest. Category: Concept
356 richard@qwconsultancy.com
55) What are the advantages and disadvantages of flextime? Answer: There are several advantages of flextime. They include reduced absenteeism, increased productivity, reduced overtime expenses, reduced hostility toward management, reduced traffic congestion around work sites, elimination of tardiness, and increased autonomy and responsibility for employees. These factors help increase employee job satisfaction. The major drawback with flextime is that it is not applicable to all jobs. It works well with clerical tasks for which an employee's interaction with people outside his or her department is limited. It is not a viable option for receptionists, sales personnel in retail stores, or similar jobs for which comprehensive service demands that people be at their workstations at predetermined times. LO: 8.3: Explain how specific alternative work arrangements can motivate employees. Difficulty: Moderate Quest. Category: Concept 56) Define telecommuting and explain its role as an alternative work arrangement. Answer: Telecommuting refers to working at home at least two days a week on a computer linked to the employer's office. This strategy suits three categories of jobs: routine informationhandling tasks, mobile activities, and professional and other knowledge-related tasks. The advantages of telecommuting include reduced role stress and turnover intentions. Employees who work virtually more than 2.5 days a week tend to experience the benefits of reductions in work-family conflict more intensely than those who are in the office the majority of the workweek. A positive relationship exists between telecommuting and supervisor performance ratings. Telecommuting has several downsides too. In today's team-focused workplace, telecommuting may lead to social loafing, especially when the employees have many family responsibilities but their teammates do not. Your manager working remotely can affect your performance negatively. Managers are also challenged to handle the demotivation of office workers who feel they are unfairly denied the freedom of telecommuters. From the employee's standpoint, telecommuting can increase feelings of isolation and reduce job satisfaction as well as coworker relationship quality. LO: 8.3: Explain how specific alternative work arrangements can motivate employees. Difficulty: Moderate Quest. Category: Concept 57) Discuss the "out of sight, out of mind" effect as it relates to telecommuting. Answer: Telecommuters are vulnerable to the "out of sight, out of mind" effect: Employees who are not physically present in the same location do not share in day-to-day informal workplace interactions, which may put them at a disadvantage when it comes to raises and promotions because they are perceived as not putting in the requisite face time (unless they put in the effort to do so virtually). However, this may not be the case if telework is the norm with the organization, there is still common contact with the supervisor, and it is clear that the employee is working (e.g., they have a heavy workload and are productive). LO: 8.3: Explain how specific alternative work arrangements can motivate employees. Difficulty: Moderate Quest. Category: Concept
357 richard@qwconsultancy.com
58) Which of the following is a participative process that uses employees' input to increase their commitment to the organization's success? A) Job enrichment B) Employee involvement C) Vertical integration D) Flextime E) Job sharing Answer: B Explanation: Employee involvement is a participative process that uses employees' input to increase their commitment to the organization's success. The logic is that if workers are engaged in decisions that affect them and have increased autonomy, they will become more motivated, committed, productive, and satisfied with their jobs. LO: 8.4: Describe how employee involvement measures can motivate employees. Difficulty: Easy Quest. Category: Concept 59) To be effective, an employee involvement program must ________. A) eliminate obsolete positions in order to save other positions B) be tailored to local and national norms C) mandatorily establish work councils in the company D) eliminate the influence of employees in managerial decision making E) implement autocratic ways of handling its employees Answer: B Explanation: Employee involvement programs differ among countries. A study of four countries confirmed the importance of modifying practices to reflect national culture. To be successful, an employee involvement program must be tailored to local and national norms. LO: 8.4: Describe how employee involvement measures can motivate employees. AACSB: Diverse and multicultural work environments Difficulty: Moderate Quest. Category: Concept 60) ________ is the distinct characteristic of participative management programs. A) Joint decision making B) Representative participation C) Performance norms D) Establishment of work councils E) Autocratic decision making Answer: A Explanation: The distinct characteristic common to all participative management programs is joint decision making, in which subordinates share a significant degree of decision-making power with their immediate superiors. LO: 8.4: Describe how employee involvement measures can motivate employees. Difficulty: Moderate Quest. Category: Concept
358 richard@qwconsultancy.com
61) Employee involvement and participation (EIP) management is a method of management where ________. A) representatives of workers form work councils and these councils must be consulted when management makes decisions about employees B) few representatives of workers sit on a company's board of directors C) subordinates share a significant degree of decision-making power with their immediate superiors D) low-level workers meet occasionally with the CEO to discuss problems within their department E) low-level workers are responsible for making corporate policy decisions Answer: C Explanation: Employee involvement and participation (EIP) management is joint decision making, in which subordinates share a significant degree of decision-making power with their immediate superiors. LO: 8.4: Describe how employee involvement measures can motivate employees. Difficulty: Moderate Quest. Category: Concept 62) Participative management and ________ are the two major forms of employee involvement. A) organizational restructuring B) job sharing C) representative participation D) job rotation E) job enlargement Answer: C Explanation: The two major forms of employee involvement are participative management and representative participation. LO: 8.4: Describe how employee involvement measures can motivate employees. Difficulty: Easy Quest. Category: Concept 63) ________ characterizes representative participation. A) Joint decision making by employees and management B) Autocratic leadership behaviors C) One-way management decision making D) Only having nonunion employees making the primary decisions in an organization E) Redistribution of power within an organization Answer: E Explanation: Representative participation aimed at redistributing power within an organization and putting labor on a more equal footing with the interests of management and stockholders. This is done by letting workers be represented by a small group of employees who participate. LO: 8.4: Describe how employee involvement measures can motivate employees. Difficulty: Moderate Quest. Category: Concept
359 richard@qwconsultancy.com
64) Board representatives and ________ are the two most common forms of representative participation. A) quality circles B) works councils C) employee unions D) task teams E) cross-functional teams Answer: B Explanation: The two most common forms of representative participation are work councils and board representatives. LO: 8.4: Describe how employee involvement measures can motivate employees. Difficulty: Easy Quest. Category: Concept 65) Groups of nominated or elected employees who must be consulted when management makes decisions about employees are referred to as ________. A) team representatives B) board representatives C) works councils D) representative participants E) telecommuters Answer: C Explanation: In representative participation, works councils are groups of nominated or elected employees who must be consulted when management makes decisions about employees. LO: 8.4: Describe how employee involvement measures can motivate employees. Difficulty: Moderate Quest. Category: Concept
360 richard@qwconsultancy.com
66) Woodworth & Baines is a retail chain that has taken up numerous measures to improve employee attitude, motivation, and organizational performance. New training programs, employee engagement initiatives, and an employee welfare council were the three main strategies implemented as part of the employee empowerment program. Norman Kilner was one of twenty employees nominated to the employee welfare council where they were required to interact with the management to promote the interests of the employees. Additionally, the job roles of these twenty employees were vertically enhanced to give them more autonomy in the organization. However, six months into the employee empowerment program, a survey by HR revealed no actual change in employee attitudes. Similarly, a survey conducted by the production manager revealed that work productivity had not increased by any substantial measure either. Which of the following, if true, would best explain this outcome? A) Management strengthened the regulations relating to employee absenteeism in this period. B) Management retained the degree of control they had in the organizational affairs. C) The company moved from a merit-based to a piece-rate pay system in the last quarter. D) The employees performed poorly because of the lack of training and development programs. E) Management implemented a 360-degree feedback system for performance appraisals. Answer: B Explanation: The fact that management retained the degree of control they had in the organizational affair does explain this outcome. The influence of representative participation on working employees seems to be minimal. Works councils are dominated by management and have little impact on employees or the organization. While participation might increase the motivation and satisfaction of employee representatives, there is little evidence this trickles down to the employees they represent. The fact that management strengthened the regulations relating to employee absenteeism in this period does not explain this outcome. The fact that the company moved from a merit-based to a piece-rate pay system in the last quarter does not explain this outcome. The outcome cannot be attributed to the employees lacking training. The fact that management implemented a 360-degree feedback system for performance appraisals does not explain this outcome. LO: 8.4: Describe how employee involvement measures can motivate employees. AACSB: Analytical thinking Difficulty: Hard Employability Skills: Knowledge Application and Analysis Quest. Category: Critical Thinking 67) What role do board representatives play in participative representation? A) Board representatives are employees who participate on a company's board of directors. B) Board representatives are management that participate in decision making for the company. C) Board representatives is another name for a company's board of directors. D) Board representatives are the management team that sits on the board of directors. E) Board representatives are employees that hire other employees for the company. Answer: A Explanation: Board representatives are employees who sit on a company's board of directors and represent employees' interests. LO: 8.4: Describe how employee involvement measures can motivate employees. AACSB: Reflective thinking Difficulty: Moderate 361 richard@qwconsultancy.com
Quest. Category: Critical Thinking 68) Participative management is characterized by the symbolic representation of employees in work councils and board meetings. Answer: FALSE Explanation: Participative management programs are characterized by joint decision making, in which subordinates share a significant degree of decision-making power with their immediate superiors. LO: 8.4: Describe how employee involvement measures can motivate employees. Difficulty: Easy Quest. Category: Concept 69) Employee involvement and participation (EIP) is a process that uses employees' input to increase their commitment to organizational success. Answer: TRUE Explanation: Employee involvement and participation (EIP) is a process that uses employees' input to increase their commitment to organizational success. LO: 8.4: Describe how employee involvement measures can motivate employees. Difficulty: Easy Quest. Category: Concept 70) Research shows that EIP programs are highly effective in India because they are consistent with India's high-power distance culture. Answer: FALSE Explanation: While U.S. employees readily accepted EIP programs, managers in India who tried to empower their employees were rated low by those employees. These reactions are consistent with India's high-power-distance culture, which accepts and expects differences in authority. LO: 8.4: Describe how employee involvement measures can motivate employees. AACSB: Diverse and multicultural work environments Difficulty: Moderate Quest. Category: Concept
362 richard@qwconsultancy.com
71) Explain employee involvement and discuss why is it important. What are the two major forms of employee involvement? Answer: Employee involvement is defined as a participative process that uses the entire capacity of employees and is designed to encourage increased commitment to the organization's success. The underlying logic is that by involving workers in those decisions that affect them and by increasing their autonomy and control over their work lives, employees will become more motivated, more committed to the organization, more productive, and more satisfied with their jobs. The two major forms of employee involvement are: a) Participative management. Participative management programs use joint decision making. Subordinates actually share a significant degree of decision-making power with their immediate superiors. b) Representative participation. Representative participation refers to worker representation by a small group of employees who actually participate on the board. The goal is to redistribute power within an organization, putting labor on a more equal footing with the interests of management and stockholders. LO: 8.4: Describe how employee involvement measures can motivate employees. Difficulty: Moderate Quest. Category: Concept 72) Using examples, explain why EIP programs should be tailored to local and national norms. Answer: To be successful, EIP programs should be tailored to local and national norms. A study of four countries, including India and the United States, confirmed the importance of modifying practices to reflect national culture. While U.S. employees readily accepted EIP programs, managers in India who tried to empower their employees were rated low by those employees. These reactions are consistent with India's high-power-distance culture, which accepts and expects differences in authority. The work culture in India may not be in as much transition as it is in China, where some employees are becoming less high-power-distance oriented. Chinese workers who were very accepting of traditional Chinese cultural values showed few benefits from participative decision making. However, Chinese workers who were less traditional were more satisfied and had higher performance ratings under participative management. Another study conducted in China showed that involvement increased employees' thoughts and feelings of job security, which enhanced their well-being. These differences within China may well reflect the current transitional nature of Chinese culture. For example, research in urban China indicated that some aspects of EIP programs–namely, those that favor consultation and expression but not participation in decision making–yield higher job satisfaction. LO: 8.4: Describe how employee involvement measures can motivate employees. AACSB: Diverse and multicultural work environments Difficulty: Hard Employability Skills: Critical Thinking Quest. Category: Synthesis
363 richard@qwconsultancy.com
73) What is internal equity? A) A list of the knowledge, skills, and abilities required in a particular job B) An arrangement of the elements of different jobs in the organization C) The worth of the job to the organization D) An organization's pay relative to pay elsewhere in its industry E) A framework of tasks, duties, and responsibilities that the job involves Answer: C Explanation: Internal equity refers to the worth of the job to the organization. It is usually established through a technical process called job evaluation. LO: 8.5: Demonstrate how different types of extrinsic pay programs can influence employee motivation. Difficulty: Moderate Quest. Category: Concept 74) ________ refers to the external competitiveness of an organization's pay relative to elsewhere in the industry. A) External benefit B) External rotation C) External costs D) External equity E) External liability Answer: D Explanation: The external competitiveness of an organization's pay relative to pay elsewhere in its industry is defined as external equity. LO: 8.5: Demonstrate how different types of extrinsic pay programs can influence employee motivation. Difficulty: Moderate Quest. Category: Concept 75) Which of the following bases a portion of an employee's pay on some individual and/or organizational measure of performance? A) Guaranteed pay program B) Variable-pay program C) Flexible benefits program D) Modular plans program E) Base pay program Answer: B Explanation: Variable-pay programs base a portion of an employee's pay on some individual and/or organizational measure of performance. LO: 8.5: Demonstrate how different types of extrinsic pay programs can influence employee motivation. Difficulty: Easy Quest. Category: Concept
364 richard@qwconsultancy.com
76) Which pay plan has long been popular as a means of compensating production workers by paying a fixed sum for each unit of production completed? A) Competency-based B) Gainsharing C) Piece-rate D) Profit-sharing E) Merit-based Answer: C Explanation: The piece-rate pay plan has long been popular as a means of compensating production workers by paying a fixed sum for each unit of production completed. A pure piecerate plan provides no base salary and pays the employee only for what he or she produces. LO: 8.5: Demonstrate how different types of extrinsic pay programs can influence employee motivation. Difficulty: Easy Quest. Category: Concept 77) An example of a piece-rate plan is ________. A) $20 for each hour of work B) $2 for each unit produced C) stock options at cheaper prices D) straight commission plan E) spot bonuses Answer: B Explanation: The piece-rate pay plan is a means of compensating production workers by paying a fixed sum for each unit of production completed. Paying $2 for each unit produced represents a piece-rate pay plan. LO: 8.5: Demonstrate how different types of extrinsic pay programs can influence employee motivation. AACSB: Analytical thinking Difficulty: Moderate Quest. Category: Application 78) Which type of pay plan pays for individual performance based on performance appraisal ratings? A) Piece-rate B) Merit-based C) Employee stock ownership D) Profit-sharing E) Gainsharing Answer: B Explanation: A merit-based pay plan pays for individual performance based on performance appraisal ratings. LO: 8.5: Demonstrate how different types of extrinsic pay programs can influence employee motivation. Difficulty: Easy Quest. Category: Concept 365 richard@qwconsultancy.com
79) Daichi Inc. is a Japanese software development firm known for its high-quality products. Recently, the company held its annual conference and awarded all those employees who were in the top five percent with substantial monetary rewards. Their performance was evaluated on the basis of target achievement, client feedback, and quality ratings. Employees whose performance had not changed drastically were given a minor raise and those who fared badly received no incentives. Many employees blamed the company for creating differences among employees because they felt that it would harm the company in the long run, but Daichi Inc. truly believes that to retain and motivate its best performers, a large incentive is a good step. Daichi Inc. is using a(n) ________ here. A) piece-rate plan B) employee stock ownership plan C) modular plan D) merit-based plan E) flexible benefits plan Answer: D Explanation: A merit-based pay plan pays for individual performance based on performance appraisal ratings. Daichi Inc. rewarded these employees on the basis of meritorious performance. LO: 8.5: Demonstrate how different types of extrinsic pay programs can influence employee motivation. AACSB: Analytical thinking Difficulty: Moderate Employability Skills: Knowledge Application and Analysis Quest. Category: Application 80) A pay plan that rewards employees for recent performance rather than historical performance is a ________ plan. A) bonus B) flexible benefits C) modular D) cafeteria E) base salary Answer: A Explanation: Bonuses are used in a pay plan that rewards employees for recent performance rather than historical performance. LO: 8.5: Demonstrate how different types of extrinsic pay programs can influence employee motivation. Difficulty: Easy Quest. Category: Concept
366 richard@qwconsultancy.com
81) An advantage of bonuses over merit-based pay plans is that ________. A) employees don't view bonuses as a part of their salary B) bonuses improve flexibility of the workforce C) bonuses are paid based on a formula D) bonuses improve performance of the group as a whole E) employees are rewarded for recent performance Answer: E Explanation: The incentive effects of performance bonuses are generally higher than those of merit pay because, rather than paying for performance years ago (that was rolled into base pay), bonuses reward recent performance. LO: 8.5: Demonstrate how different types of extrinsic pay programs can influence employee motivation. Difficulty: Moderate Quest. Category: Concept 82) Which of the following statements is true regarding a merit-based pay plan? A) It bases pay levels on how many skills employees have or how many jobs they can do. B) It is also called competency-based pay plan. C) High performers can get bigger raises. D) It distributes compensation based on an established formula designed around a company's profitability. E) A typical merit-based pay plan provides no base salary and pays the employee only for what he or she produces. Answer: C Explanation: A merit-based pay plan pays for individual performance based on performance appraisal ratings. A main advantage is that high performers can get bigger raises. LO: 8.5: Demonstrate how different types of extrinsic pay programs can influence employee motivation. Difficulty: Easy Quest. Category: Concept 83) Which type of plan distributes compensation based on some established formula designed around revenue of the company? A) A piece-rate plan B) A merit-based plan C) A skill-based plan D) A profit-sharing plan E) A competency-based plan Answer: D Explanation: A profit-sharing plan distributes compensation based on some established formula designed around a company's profitability. LO: 8.5: Demonstrate how different types of extrinsic pay programs can influence employee motivation. Difficulty: Easy Quest. Category: Concept
367 richard@qwconsultancy.com
368 richard@qwconsultancy.com
84) Which of the following is not supported by studies in regard to profit-sharing plans? A) Organizations with profit-sharing plans have higher levels of profitability than those without them. B) Profit-sharing plans create advanced levels of employee decision making. C) Profit-sharing plans are linked to higher levels of employee commitment. D) Profit-sharing plans appear to have positive impacts on employee attitudes. E) Profit-sharing plans appear to general a greater feeling of ownership. Answer: B Explanation: There is no research that shows that profit-sharing plans create advanced levels of employee decision making. LO: 8.5: Demonstrate how different types of extrinsic pay programs can influence employee motivation. Difficulty: Moderate Quest. Category: Concept 85) Managers at Beefy Burger Joint encourage employees to work hard. The company emphasizes high levels of customer service, quick table turnarounds, and menu upselling. To encourage employee loyalty and minimize costly turnover, the company offers employees paid time off and a chance to purchase company shares at below-market prices. Beefy Burger Joint is using a ________. A) flexible benefits plan B) merit-based plan C) modular plan D) employee stock ownership plan E) core-plus plan Answer: D Explanation: Under the employee stock ownership plan (ESOP), employees acquire stock, often at below-market prices. Research on ESOPs indicates they increase employee satisfaction. But their impact on performance is less clear. ESOPs for top management can reduce unethical behavior. LO: 8.5: Demonstrate how different types of extrinsic pay programs can influence employee motivation. AACSB: Analytical thinking Difficulty: Moderate Employability Skills: Knowledge Application and Analysis Quest. Category: Application
369 richard@qwconsultancy.com
86) Which of the following statements is true regarding an employee stock ownership plan (ESOP)? A) Under this plan, employees acquire company shares, often at below-market prices. B) There exists a strong correlation between ESOPs and employee performance. C) The financial stake in the company is adequate to use ESOPs as an effective means to motivate employees. D) ESOPs are effective because, like bonuses, they reward recent performance. E) ESOPs are also called competency-based pay plans. Answer: A Explanation: Under the employee stock ownership plan, employees acquire stock, often at below-market prices. Research on ESOPs indicates they increase employee satisfaction. But their impact on performance is less clear. ESOPs for top management can reduce unethical behavior. LO: 8.5: Demonstrate how different types of extrinsic pay programs can influence employee motivation. Difficulty: Easy Quest. Category: Concept 87) For employee stock ownership plans to be effective in improving performance, they must ________. A) be based solely on productivity like gainsharing plans B) be implemented such that employees psychologically experience ownership C) be determined on the basis of performance appraisals and recent performance D) be planned such that they are unaffected by company profitability E) be based on the market compensation philosophy Answer: B Explanation: ESOPs have the potential to increase employee job satisfaction and work motivation, but employees need to psychologically experience ownership. That is, in addition to their financial stake in the company, they need to be kept regularly informed of the status of the business and have the opportunity to influence it in order to significantly improve the organization's performance. LO: 8.5: Demonstrate how different types of extrinsic pay programs can influence employee motivation. Difficulty: Moderate Quest. Category: Concept 88) Profit-sharing plans distribute compensation based subjectively on management's discretion. Answer: FALSE Explanation: A profit-sharing plan distributes compensation based on some established formula designed around a company's profitability. LO: 8.5: Demonstrate how different types of extrinsic pay programs can influence employee motivation. Difficulty: Easy Quest. Category: Concept
370 richard@qwconsultancy.com
89) When guanxi plays a role in human resource (HR) practices, pay-for-performance plans tend to be less effective. Answer: TRUE Explanation: When guanxi (e.g., specific, personal connections between subordinates, supervisors, and coworkers) plays a role in human resource (HR) practices, pay-for-performance plans tend to be less effective. LO: 8.5: Demonstrate how different types of extrinsic pay programs can influence employee motivation. AACSB: Diverse and multicultural work environments Difficulty: Moderate Quest. Category: Concept 90) External equity refers to the worth of the job to the organization. Answer: FALSE Explanation: Internal equity refers to the worth of the job to the organization. It is usually established through a technical process called job evaluation. External equity refers to the competitiveness of an organization's pay relative to pay in its industry. LO: 8.5: Demonstrate how different types of extrinsic pay programs can influence employee motivation. Difficulty: Easy Quest. Category: Concept 91) A variable-pay program bases a portion of an employee's pay on some individual and/or organizational measure of performance. Answer: TRUE Explanation: A variable-pay program bases a portion of an employee's pay on some individual and/or organizational measure of performance. LO: 8.5: Demonstrate how different types of extrinsic pay programs can influence employee motivation. Difficulty: Easy Quest. Category: Concept 92) Increased transparency in variable pay programs always improves employee performance. Answer: FALSE Explanation: Pay transparency can have a negative effect of employee performance. Research suggests that it can facilitate comparisons between people, leading co-workers to become jealous and abstain from cooperating or helping one another. One thing seems to be clear: Justice perceptions (as discussed in the previous chapter) play a huge role in determining how employees react, regardless of the strategy. LO: 8.5: Demonstrate how different types of extrinsic pay programs can influence employee motivation. Difficulty: Moderate Quest. Category: Concept
371 richard@qwconsultancy.com
93) When the profits of Emm Corp. rose steadily over two quarters three years back, the employees of the company were given the offer of purchasing the company's shares at subsidized rates. Many of the employees who bought the shares have become extremely rich after the phenomenal growth Emm Corp. has had in the following years. This is an example of a bonus. Answer: FALSE Explanation: An employee stock ownership plan (ESOP) is a company-established benefit plan in which employees acquire stock, often at below-market prices, as part of their benefits. A bonus is a pay plan that rewards employees for recent performance rather than historical performance. LO: 8.5: Demonstrate how different types of extrinsic pay programs can influence employee motivation. AACSB: Analytical thinking Difficulty: Moderate Employability Skills: Knowledge Application and Analysis Quest. Category: Application 94) Compare and contrast the different variable-pay programs. Answer: a) Piece-rate plans. In piece-rate pay plans, workers are paid a fixed sum for each unit of production completed. When an employee gets no base salary and is paid only for what he or she produces, this is a pure piece-rate plan. b) Merit-based pay. Merit-based pay plans also pay for individual performance. However, unlike piece-rate plans, which is pay based on objective output, merit-based pay plans are based on performance appraisal ratings. c) Profit sharing. Profit-sharing plans are organization-wide programs that distribute compensation based on some established formula designed around a company's profitability. d) Bonuses. Bonuses can be paid exclusively to executives or to all employees. Many companies now routinely reward production employees with bonuses in thousands of dollars when company profits improve. e) ESOPs. Employee stock ownership plans (ESOPs) are company-established benefit plans in which employees acquire stock, often at below-market prices, as part of their benefits. LO: 8.5: Demonstrate how different types of extrinsic pay programs can influence employee motivation. Difficulty: Moderate Quest. Category: Concept
372 richard@qwconsultancy.com
95) What is a company bonus plan? Answer: Bonuses represent a pay plan that rewards employees for recent performance rather than historical performance. An annual bonus is a significant component of total compensation for many jobs, and many companies routinely reward production employees with bonuses in the thousands of dollars when profits improve. However, when times are bad, firms cut bonuses to reduce compensation costs. Thus, using bonuses as a variable pay program makes employees' pay more vulnerable to cuts, and this is even more problematic when bonuses are a large percentage of total pay. LO: 8.5: Demonstrate how different types of extrinsic pay programs can influence employee motivation. AACSB: Analytical thinking Difficulty: Moderate Quest. Category: Synthesis 96) Compare and contrast the benefits of intrinsic rewards, such as recognition, and extrinsic rewards, such as pay, as forms of motivation. Answer: Organizations are increasingly recognizing that both intrinsic and extrinsic rewards are important. Rewards are intrinsic in the form of employee recognition programs and extrinsic in the form of compensation systems. Intrinsic rewards range from a spontaneous and private thank-you to widely publicized formal programs in which specific types of behavior are encouraged and the procedures for attaining recognition are clearly identified. Pay, an extrinsic reward, is not the primary factor driving job satisfaction; however, it does motivate people, and companies often underestimate its importance in keeping top talent. No matter how much recognition a top performer gets, he or she will be tempted to leave if the pay scale is much lower than the market and another offer arrives. Variable pay structures, such as bonuses, are often the best production motivators. When pay is tied to performance, the employee's earnings also recognize contribution rather than being a form of entitlement. Over time, low performers' pay stagnates, while high performers enjoy pay increases commensurate with their contributions. An obvious advantage of intrinsic rewards like recognition programs is that they are inexpensive since praise is free. However, they are highly susceptible to political manipulation by management. When applied to jobs for which performance factors are relatively objective, such as sales, recognition programs are likely to be perceived by employees as fair. However, in most jobs, the criteria for good performance aren't self-evident, which allows managers to manipulate the system and recognize their favorites. Abuse can undermine the value of recognition programs and demoralize employees. Research suggests financial incentives may be more motivating in the short term, but in the long run intrinsic incentives will retain good employees if the extrinsic incentives are competitive. LO: 8.5: Demonstrate how different types of extrinsic pay programs can influence employee motivation. AACSB: Analytical thinking Difficulty: Moderate Quest. Category: Concept
373 richard@qwconsultancy.com
97) ________ refer to individualized rewards that allow each employee to choose the compensation package that best satisfies his or her current needs and situation. A) Variable pay programs B) Standardized benefits packages C) Employee recognition programs D) Employee involvement programs E) Flexible benefits plans Answer: E Explanation: Flexible benefits individualize rewards by allowing each employee to choose the compensation package that best satisfies his or her current needs and situation. Flexible benefits can accommodate differences in employee needs based on age, marital status, spouse's benefit status, and number and age of dependents. LO: 8.6: Show how flexible benefits can motivate employees. Difficulty: Easy Quest. Category: Concept 98) Which of the following statements is true regarding flexible benefits? A) Flexible benefits are awarded through uniform packages to cover the requirements of a married couple with two children. B) Flexible benefits are in accordance with the belief that employees have identical needs that must be rewarded identically. C) Profit-sharing plans are a major type of flexible benefit. D) Flexible benefits can be altered to meet employee needs based on age, marital status, and number and age of dependents. E) Flexible benefits are given based on an employee's skills, competence, and productivity. Answer: D Explanation: Flexible benefits individualize rewards by allowing each employee to choose the compensation package that best satisfies his or her current needs and situation. Flexible benefits can accommodate differences in employee needs based on age, marital status, spouse's benefit status, and number and age of dependents. LO: 8.6: Show how flexible benefits can motivate employees. Difficulty: Moderate Quest. Category: Concept 99) A flexible benefits package is designed for a male with a wife and two children at home. Answer: FALSE Explanation: Flexible benefits individualize rewards by allowing each employee to choose the compensation package that best satisfies his or her current needs and situation. These plans replace the "one-benefit-plan-fits-all" programs designed for a male with a wife and two children at home that dominated organizations for decades. LO: 8.6: Show how flexible benefits can motivate employees. Difficulty: Moderate Quest. Category: Concept
374 richard@qwconsultancy.com
100) Most managers agree that health care, retirement, and leave are essential and should be offered in some way. Answer: TRUE Explanation: it is important to recognize that flexibility does not mean that certain benefits are not essential. On the contrary, most managers suggest that health care, retirement, and leave are three benefits that are essential and should be offered in some way. LO: 8.6: Show how flexible benefits can motivate employees. Difficulty: Moderate Quest. Category: Concept 101) As more organizations worldwide adopt flexible benefits, the individual motivation they produce will likely decrease. Answer: TRUE Explanation: As more organizations worldwide adopt flexible benefits, the individual motivation they produce will likely decrease as the plans are seen as a standard work provision. LO: 8.6: Show how flexible benefits can motivate employees. AACSB: Diverse and multicultural work environments Difficulty: Moderate Quest. Category: Concept 102) Organizational norms may discourage employees from using benefits like vacation days. Answer: TRUE Explanation: Organizational norms may play a huge role: Even though the company may offer benefits, whether employees are encouraged or discouraged from using them is a whole other issue. In high-pressure work cultures, employees may be reluctant to use their vacation days to avoid missing work, falling behind, or appearing undedicated to their co-workers. LO: 8.6: Show how flexible benefits can motivate employees. Difficulty: Moderate Quest. Category: Concept 103) ________ is an example of an intrinsic reward. A) A bonus B) A profit-sharing plan C) An employee stock option D) Employee recognition E) Gainsharing Answer: D Explanation: Important work rewards can be both intrinsic and extrinsic. Rewards are intrinsic in the form of employee recognition programs and extrinsic in the form of compensation systems. LO: 8.7: Identify the motivational benefits of intrinsic rewards. Difficulty: Moderate Quest. Category: Concept
375 richard@qwconsultancy.com
104) Jack Holmes is a middle-aged, lower-level employee at an automobile service center. Though he is not paid very well, he loves his job. His supervisor speaks to him with respect, and he is the favored candidate for mentoring new employees because of the vast experience he holds in the job. Based on this information, which of the following is most likely to be the reason Jack likes his job? A) Recognition B) Prospects of growth C) Salary D) Flextime E) Job sharing Answer: A Explanation: Intrinsic rewards in the form of employee recognition programs are increasingly being recognized as important in job satisfaction and motivation. Jack's attitude stems from the recognition he receives. LO: 8.7: Identify the motivational benefits of intrinsic rewards. AACSB: Analytical thinking Difficulty: Moderate Employability Skills: Knowledge Application and Analysis Quest. Category: Application 105) Which of the following statements is true regarding employee recognition? A) Employee recognition is an extrinsic reward. B) Critics argue that employee recognition programs are highly susceptible to political manipulation by management. C) Employee recognition programs are expensive. D) Centralized programs across multiple offices in different countries can hinder the process of employee recognition programs. E) Employee stock option is a more powerful workplace motivator than recognition. Answer: B Explanation: Employee recognition is an intrinsic reward. Employee recognition programs are inexpensive. Some research suggests financial incentives may be more motivating in the short term, but in the long run it's nonfinancial incentives. According to research, recognition is the most powerful workplace motivator. Critics argue that employee recognition programs are highly susceptible to political manipulation by management. LO: 8.7: Identify the motivational benefits of intrinsic rewards. Difficulty: Moderate Quest. Category: Concept 106) On an organizational level, job design and the extrinsic rewards of pay and benefits are the only ways to motivate employees. Answer: FALSE Explanation: On an organizational level, job design and the extrinsic rewards of pay and benefits can motivate employees. But, we would be remiss if we overlooked intrinsic rewards organizations can provide such as employee recognition programs. LO: 8.7: Identify the motivational benefits of intrinsic rewards. Difficulty: Moderate 376 richard@qwconsultancy.com
Quest. Category: Concept 107) Research suggests that financial incentives may be more motivating in the short-run, but in the long-run, non-financial incentives work best. Answer: TRUE Explanation: Research suggests that financial incentives may be more motivating in the shortrun, but in the long-run, non-financial incentives work best. LO: 8.7: Identify the motivational benefits of intrinsic rewards. Difficulty: Moderate Quest. Category: Concept Organizational Behavior, 19e (Robbins/Judge) Chapter 9 Foundations of Group Behavior 1) Which of the following is true, in general, in regard to groups? A) A group is characterized by the independence of its members. B) A group typically lacks definite roles and structures. C) A group influences our emotional reactions. D) An informal gathering cannot be considered a group. E) The membership of a group does not affect how its members treat outsiders. Answer: C Explanation: Our tendency to take personal pride or offense for the accomplishments of a group is the territory of social identity theory. Social identity theory proposes that people have emotional reactions to the failure or success of their group because their self-esteem gets tied into the group's performance. LO: 9.1: Distinguish between the different types of groups. AACSB: Interpersonal relations and teamwork Difficulty: Moderate Quest. Category: Concept 2) Which of the following is true regarding formal groups? A) They are natural formations that arise in response to the need for social contact. B) They lack clearly defined structures and roles for their members. C) They have negligible impact on employee performance and behavior. D) A group of people who come together to protest against a new law make up a formal group. E) They are marked by stipulated behaviors in pursuit of organizational goals. Answer: E Explanation: A formal group is one that is defined by the organization's structure, with designated work assignments establishing tasks. In formal groups, the behaviors team members should engage in are stipulated by and directed toward organizational goals. LO: 9.1: Distinguish between the different types of groups. AACSB: Interpersonal relations and teamwork Difficulty: Moderate Quest. Category: Concept 3) Which of the following is characteristic of an informal group? A) Stipulation of expected behaviors by the organization 377 richard@qwconsultancy.com
B) Predetermined designation of tasks of members C) Pursuit of particular organizational goals D) Fulfillment of the need for social contact E) Creation of timelines and rationale Answer: D Explanation: An informal group is neither formally structured nor organizationally determined. Informal groups in the work environment meet in response to the need for social contact. LO: 9.1: Distinguish between the different types of groups. AACSB: Interpersonal relations and teamwork Difficulty: Moderate Quest. Category: Concept 4) Which of the following differentiates between formal and informal groups? A) The timeline of reference for formal groups is greater than that for informal groups. B) The impact of formal groups on organizational performance is less than that of informal groups. C) Formal groups pursue the goal of social contact, while informal groups have definite organizational goals to attain. D) Formal groups are typically smaller in size when compared to informal groups. E) Formal groups involve clearly defined tasks and roles, while informal groups are neither formally structured nor organizationally determined. Answer: E Explanation: A formal group is one that is defined by the organization's structure, with designated work assignments establishing tasks. An informal group is neither formally structured nor organizationally determined. LO: 9.1: Distinguish between the different types of groups. AACSB: Interpersonal relations and teamwork Difficulty: Moderate Quest. Category: Concept 5) Our tendency to take personal pride or offense for the accomplishments of a group we are a part of is the territory of the ________ theory. A) social exchange B) expectancy C) social identity D) reinforcement E) equity Answer: C Explanation: Our tendency to take personal pride or offense for the accomplishments of a group is the territory of social identity theory. LO: 9.1: Distinguish between the different types of groups. AACSB: Interpersonal relations and teamwork Difficulty: Moderate Quest. Category: Concept
378 richard@qwconsultancy.com
6) Which of the following is true with regard to the social identity theory? A) It proposes that members take personal pride in accomplishments but dissociate from the group in the event of any offenses or failures. B) It proposes that people have emotional reactions to the failure or success of their group because their self-esteem is tied into the group's performance. C) It proposes that people develop only one social identity in childhood that becomes more defined and concrete in the course of their lives. D) It proposes that social identities can never be detrimental as they have no effect on how members of an ingroup view members of an out-group. E) It proposes that members of an ingroup are viewed as being homogeneous, while members of an out-group are seen as being heterogeneous. Answer: B Explanation: Social identity theory proposes that people have emotional reactions to the failure or success of their group because their self-esteem gets tied into the group's performance. LO: 9.1: Distinguish between the different types of groups. AACSB: Interpersonal relations and teamwork Difficulty: Moderate Quest. Category: Concept 7) ________ explains why people categorize others as belonging to different groups. A) Social categorization B) Xenocentrism C) Ingroup favoritism D) The black sheep effect E) Hindsight bias Answer: A Explanation: Social categorization explains why people categorize others as belonging to different groups. LO: 9.1: Distinguish between the different types of groups. AACSB: Interpersonal relations and teamwork Difficulty: Moderate Quest. Category: Concept
379 richard@qwconsultancy.com
8) At Milton farms, where a large part of the management comes from the same sociocultural background, many employees of diverse ethical or cultural origins are hired only to ensure legal compliance with laws relating to diversity at the workplace. The company's day-to-day functioning leaves much to be desired in terms of promoting diversity. Recently, the owner's cousin was promoted to the post of a branch manager while three suitable candidates hailing from different cultures were blatantly overlooked. This is an example of ________. A) social loafing B) ingroup favoritism C) groupthink D) contrast bias E) social exchange Answer: B Explanation: We tend to "play favorites" or see our ingroup as better than other people. For example, some research suggests that favoritism and not hostility might cause most discriminatory behaviors. By playing favorites with their own ingroup (e.g., people of the same race/ethnicity, gender identity, etc.), employees and managers alike may be complicit in discrimination. People fall into the dangerous trap of believing that those who belong to different outgroups are "all the same," contributing to the polarization of people. LO: 9.1: Distinguish between the different types of groups. AACSB: Diverse and multicultural work environments Difficulty: Moderate Quest. Category: Application 9) Aaron Dias was working on the last shift for the day at All Needs, a retail store owned by an Asian man, when he opened the cash register and stole some money thinking that nobody would witness him stealing. However, one of the customers watched him steal the money and reported it to the manager the next day, and Aaron was terminated. Subsequently, the manager became extremely strict with all his Caucasian employees and was often heard abusing them by saying, "You guys are all the same." This scenario depicts ________. A) social loafing B) ingroup favoritism C) groupthink D) contrast bias E) social exchange Answer: B Explanation: We tend to "play favorites" or see our ingroup as better than other people. For example, some research suggests that favoritism and not hostility might cause most discriminatory behaviors. By playing favorites with their own ingroup (e.g., people of the same race/ethnicity, gender identity, etc.), employees and managers alike may be complicit in discrimination. People fall into the dangerous trap of believing that those who belong to different outgroups are "all the same," contributing to the polarization of people. LO: 9.1: Distinguish between the different types of groups. AACSB: Diverse and multicultural work environments Difficulty: Moderate Quest. Category: Application
380 richard@qwconsultancy.com
381 richard@qwconsultancy.com
10) In the context of the social identity theory, people have ________ to the failure or success of group members. A) angry reactions B) lack of empathy or pride C) emotional withdrawal D) lack of reaction E) emotional reactions Answer: E Explanation: Social identity theory proposes that people have emotional reactions to the failure or success of their group because their self-esteem gets tied to whatever happens to the group. LO: 9.1: Distinguish between the different types of groups. AACSB: Interpersonal relations and teamwork Difficulty: Moderate Quest. Category: Concept 11) ________ is when we connect with others because of our roles. A) Collective identification B) Schadenfreude C) Relational identification D) Informal group identification E) Formal group dynamics Answer: C Explanation: Relational identification is when we connect with other members of our group due to our roles. LO: 9.1: Distinguish between the different types of groups. AACSB: Interpersonal relations and teamwork Difficulty: Easy Quest. Category: Concept 12) ________ is when we connect with the aggregate characteristics of our groups. A) Relational identification B) Collective identification C) Schadenfreude D) Informal group identification E) Formal group dynamics Answer: B Explanation: Collective identification is when we connect with the aggregate characteristics of our group members. LO: 9.1: Distinguish between the different types of groups. AACSB: Interpersonal relations and teamwork Difficulty: Easy Quest. Category: Concept
382 richard@qwconsultancy.com
13) Often, our identification with our workgroups is stronger than with our organizations. Both are important to positive outcomes in attitudes and behaviors. Additionally, if we have low identification in relation to the group, there may be increased competition and decreased organizational citizenship behavior (OCB) by group members. If we have low identification with our organizations, we may experience decreased satisfaction and OCB. Answer: TRUE Explanation: Within our organizations and workgroups, we can develop many identities through: 1) relational identification, when we connect with others because of our roles, and 2) collective identification, when we connect with the aggregate characteristics of our groups. Often, our identification with our workgroups is stronger than with our organizations. Both are important to positive outcomes in attitudes and behaviors. Additionally, if we have low identification in relation to the group, there may be increased competition and decreased organizational citizenship behavior (OCB) by group members. If we have low identification with our organizations, we may experience decreased satisfaction and OCB. LO: 9.1: Distinguish between the different types of groups. AACSB: Analytical thinking Difficulty: Hard Quest. Category: Concept 14) When CEOs use we or us language, it signals to employees and stakeholders that they are part of a group. Answer: TRUE Explanation: The words leaders use matter–when CEOs use we or us language (instead of I language), this signals to employees and stakeholders alike that they are a part of a group. LO: 9.1: Distinguish between the different types of groups. AACSB: Interpersonal relations and teamwork Difficulty: Moderate Quest. Category: Concept 15) Research suggests that favoritism and not hostility might cause most discriminatory behaviors. Answer: TRUE Explanation: Research suggests that favoritism and not hostility might cause most discriminatory behaviors. LO: 9.1: Distinguish between the different types of groups. AACSB: Interpersonal relations and teamwork Difficulty: Moderate Quest. Category: Concept
383 richard@qwconsultancy.com
16) Compare and contrast groupthink and groupshift. Answer: Groupthink relates to norms and describes situations in which group pressures for conformity deter the group from critically appraising unusual, minority, or unpopular views. Groupthink attacks many groups and can dramatically hinder their performance. Groupshift describes the way group members tend to exaggerate their initial positions when discussing a given set of alternatives to arrive at a solution. In some situations, caution dominates and there is a conservative shift, while in other situations groups tend toward a risky shift. LO: 9.1: Distinguish between the different types of groups. AACSB: Reflective thinking Difficulty: Moderate Quest. Category: Synthesis 17) The first action in the group development process of a temporary group involves ________. A) setting group direction B) transition after half of time taken up C) inertia and slow progress D) the second phase of inertia E) accelerated activity Answer: A Explanation: Temporary groups with finite deadlines pass through a unique sequencing of actions (or inaction): (1) Their first meeting sets the group's direction, (2) the first phase of group activity is one of inertia and thus slower progress, (3) a transition takes place exactly when the group has used up half its allotted time, (4) this transition initiates major changes, (5) a second phase of inertia follows the transition, and (6) the group's last meeting is characterized by markedly accelerated activity. LO: 9.2: Describe the punctuated-equilibrium model of group development. AACSB: Interpersonal relations and teamwork Difficulty: Moderate Quest. Category: Concept
384 richard@qwconsultancy.com
18) The second action in the group development process of a temporary group involves ________. A) setting group direction B) transition after half of time taken up C) inertia and slow progress D) the second phase of inertia E) accelerated activity Answer: C Explanation: Temporary groups with finite deadlines pass through a unique sequencing of actions (or inaction): (1) Their first meeting sets the group's direction, (2) the first phase of group activity is one of inertia and thus slower progress, (3) a transition takes place exactly when the group has used up half its allotted time, (4) this transition initiates major changes, (5) a second phase of inertia follows the transition, and (6) the group's last meeting is characterized by markedly accelerated activity. LO: 9.2: Describe the punctuated-equilibrium model of group development. AACSB: Interpersonal relations and teamwork Difficulty: Moderate Quest. Category: Concept 19) The third action in the group development process of a temporary group involves ________. A) transition after half of time taken up B) inertia and slow progress C) the second phase of inertia D) transition and major changes E) accelerated activity Answer: A Explanation: Temporary groups with finite deadlines pass through a unique sequencing of actions (or inaction): (1) Their first meeting sets the group's direction, (2) the first phase of group activity is one of inertia and thus slower progress, (3) a transition takes place exactly when the group has used up half its allotted time, (4) this transition initiates major changes, (5) a second phase of inertia follows the transition, and (6) the group's last meeting is characterized by markedly accelerated activity. LO: 9.2: Describe the punctuated-equilibrium model of group development. AACSB: Interpersonal relations and teamwork Difficulty: Moderate Quest. Category: Concept
385 richard@qwconsultancy.com
20) The final action in the group development process of a temporary group involves ________. A) transition after half of time taken up B) accelerated activity C) inertia and slow progress D) the second phase of inertia E) transition and major changes Answer: B Explanation: Temporary groups with finite deadlines pass through a unique sequencing of actions (or inaction): (1) Their first meeting sets the group's direction, (2) the first phase of group activity is one of inertia and thus slower progress, (3) a transition takes place exactly when the group has used up half its allotted time, (4) this transition initiates major changes, (5) a second phase of inertia follows the transition, and (6) the group's last meeting is characterized by markedly accelerated activity. LO: 9.2: Describe the punctuated-equilibrium model of group development. AACSB: Interpersonal relations and teamwork Difficulty: Moderate Quest. Category: Concept 21) Which of the following statements is true with regard to the punctuated-equilibrium model of group development? A) It applies to all permanent work groups. B) It follows the five stages of forming, storming, norming, performing, and adjourning. C) It does not consider the changes in patterns of group activities in response to deadlines and time constraints. D) It involves the two strategies of groupthink and groupshift to break out of the phases of inertia. E) It characterizes groups as exhibiting long periods of inertia interspersed with brief revolutionary changes. Answer: E Explanation: The punctuated-equilibrium model characterizes groups as exhibiting long periods of inertia interspersed with brief revolutionary changes triggered primarily by members' awareness of time and deadlines. LO: 9.2: Describe the punctuated-equilibrium model of group development. AACSB: Interpersonal relations and teamwork Difficulty: Moderate Quest. Category: Concept
386 richard@qwconsultancy.com
22) Tabby Tolman is a project coordinator at Tristar Solutions. Whenever a new project comes her way, she holds a project meeting and follows it up with repeated meetings to address any concerns the team members may have and to monitor their progress. However, with the peak of the business season arriving, she has been holding just one project initiation meeting for every project to save on time. She has become increasingly worried about the team's progress with no inputs coming from the members, and she has had no time to check with them. With two weeks left to go for a high priority project of a month's duration, she has now begun to see some work coming through and is relieved. This development reflects that the group is at the stage of ________. A) forming B) inertia following the first meeting C) second phase of inertia before the group's last meeting D) storming E) transition following the halfway mark in the timeline Answer: E Explanation: In the punctuated-equilibrium model, a transition takes place exactly when the group has used up half its allotted time, and this transition initiates major changes. LO: 9.2: Describe the punctuated-equilibrium model of group development. AACSB: Analytical thinking Difficulty: Moderate Employability Skills: Knowledge Application and Analysis Quest. Category: Application 23) The determination of the behavioral patterns and assumptions through which the group approaches the project is a function of the ________ phase in the punctuated equilibrium model. A) performing B) inertia C) equilibrium D) transition E) first meeting Answer: E Explanation: In the punctuated-equilibrium model, the first step involves a meeting which sets the group's direction and then a framework of behavioral patterns and assumptions through which the group will approach its project. LO: 9.2: Describe the punctuated-equilibrium model of group development. AACSB: Interpersonal relations and teamwork Difficulty: Easy Quest. Category: Concept
387 richard@qwconsultancy.com
24) In which stage of an alternative model to the punctuated equilibrium model do members begin to work collaboratively? A) Forming B) Reforming C) Storming D) Performing E) Norming Answer: D Explanation: Alternative models suggest that teams progress through a formation stage, a conflict resolution or "storming" stage, a "norming" stage where members agree on roles and make decisions, and a "performing" stage where members begin to work collaboratively. The forming, storming, norming, and performing stages may occur at phase one of the punctuatedequilibrium model, while a second performing and conforming stage may occur in the second phase, following a short period of reforming group norms and expectations. LO: 9.2: Describe the punctuated-equilibrium model of group development. AACSB: Interpersonal relations and teamwork Difficulty: Moderate Quest. Category: Concept 25) A group's last meeting is usually characterized by a great deal of inactivity. Answer: FALSE Explanation: The opposite is true. A group's last meeting is typically characterized by a final burst of activity to finish its work. LO: 9.2: Describe the punctuated-equilibrium model of group development. AACSB: Interpersonal relations and teamwork Difficulty: Moderate Quest. Category: Application 26) A period of inertia in a temporary group with a deadline refers to the group standing still or becoming locked into a fixed course of action. Answer: TRUE Explanation: Inertia is when the group tends to stand still or becomes locked into a fixed course of action even if it gains new insights that challenge initial patterns and assumptions. LO: 9.2: Describe the punctuated-equilibrium model of group development. AACSB: Analytical thinking Difficulty: Easy Quest. Category: Application
388 richard@qwconsultancy.com
27) Alternative models to the punctuated equilibrium model suggest that teams progress through a formation stage during which conflict resolution takes place. Answer: FALSE Explanation: Alternative models suggest that teams progress through a formation stage, a conflict resolution or "storming" stage, a "norming" stage where members agree on roles and make decisions, and a "performing" stage where members begin to work collaboratively. The forming, storming, norming, and performing stages may occur at phase one of the punctuatedequilibrium model, while a second performing and conforming stage may occur in the second phase, following a short period of reforming group norms and expectations. LO: 9.2: Describe the punctuated-equilibrium model of group development. AACSB: Analytical thinking Difficulty: Moderate Quest. Category: Application 28) Setting up a group's direction is often the first activity of temporary groups. Answer: TRUE Explanation: Temporary groups with finite deadlines pass through a unique sequencing of actions (or inaction). LO: 9.2: Describe the punctuated-equilibrium model of group development. Difficulty: Easy Quest. Category: Concept 29) The development of temporary work groups with finite deadlines is best understood in terms of three sequences in the group development process. Answer: FALSE Explanation: Temporary groups with finite deadlines pass through a unique sequencing of six actions (or inaction). LO: 9.2: Describe the punctuated-equilibrium model of group development. AACSB: Interpersonal relations and teamwork Difficulty: Moderate Quest. Category: Concept 30) Explain the punctuated-equilibrium model. Answer: Temporary groups with deadlines do not seem to follow the usual five-stage model. An alternative model to explain the growth of temporary groups is the punctuated-equilibrium model. The punctuated-equilibrium model characterizes groups as exhibiting long periods of inertia interspersed with brief revolutionary changes triggered primarily by members' awareness of time and deadlines. Under this model, group development occurs along the following stages: (1) their first meeting sets the group's direction, (2) this first phase of group activity is one of inertia, (3) a transition takes place exactly when the group has used up half its allotted time, (4) this transition initiates major changes, (5) a second phase of inertia follows the transition, and (6) the group's last meeting is characterized by markedly accelerated activity. LO: 9.2: Describe the punctuated-equilibrium model of group development. AACSB: Interpersonal relations and teamwork Difficulty: Moderate Quest. Category: Concept 389 richard@qwconsultancy.com
31) ________ refers to role perception. A) The way others believe we should act in a given context B) Our view of how we are supposed to act in a given situation C) The existence of roles that are at variance with one another D) The socially defined position or rank given to group members by others E) The degree to which members are motivated to stay in the group Answer: B Explanation: Role perception is an individual's view of how he or she is supposed to act in a given situation. LO: 9.3: Show how role requirements change in different situations. Difficulty: Easy Quest. Category: Concept 32) Since she experienced her first transatlantic flight as a young child, Francis Billington has dreamed of becoming a flight attendant. Now, having recently been hired by an airline, Francis has very strong ideals about how she should behave in her position. As a flight attendant, Francis feels that she must be welcoming, efficient, and competent at all times. She also believes that is important to go the extra mile by volunteering to work less attractive schedules or offering to handle passenger complaints. This scenario reflects Francis' ________. A) role fuzziness B) role ambiguity C) role conflict D) role perception E) role status Answer: D Explanation: Role perception is an individual's view of how he or she is supposed to act in a given situation. Francis' behavior as a flight attendant is guided by her perception of how she should act in this profession. LO: 9.3: Show how role requirements change in different situations. AACSB: Analytical thinking Difficulty: Moderate Employability Skills: Knowledge Application and Analysis Quest. Category: Application 33) Which of the following defined as the way others believe you should act in a given context? A) Role fuzziness B) Role expectation C) Role conflict D) Role perception E) Role enhancement Answer: B Explanation: Role expectations are the way others believe you should act in a given context. LO: 9.3: Show how role requirements change in different situations. Difficulty: Easy Quest. Category: Concept 390 richard@qwconsultancy.com
391 richard@qwconsultancy.com
34) Most people assume that police officers should behave in a lawful manner, refrain from demonstrating favoritism to any particular group, and do their best to uphold the law. Which of the following terms best represents these beliefs? A) Diversity B) Similarity C) Role expectation D) Uncertainty reduction E) Dispersion of responsibility Answer: C Explanation: Role expectations are the way people believe others should act in a given context. This example shows the role expectations that most people have of a police officer. LO: 9.3: Show how role requirements change in different situations. AACSB: Analytical thinking Difficulty: Moderate Quest. Category: Application 35) Role conflict occurs when ________. A) compliance with one role aids our performance in another role B) compliance with the group is an attempt to understand who we are and how we fit into the world C) compliance with the group is marked by a tendency to notice and emphasize those aspects of the group that are distinctive D) compliance with one role requirement may make it difficult to comply with another E) compliance with the group is strengthened by members who have similar values and characteristics Answer: D Explanation: When compliance with one role requirement may make it difficult to comply with another, the result is role conflict. At the extreme, two or more role expectations can be mutually contradictory. LO: 9.3: Show how role requirements change in different situations. Difficulty: Easy Quest. Category: Concept
392 richard@qwconsultancy.com
36) Michael is devout and very active in his church. He is also a very dedicated employee. His manager offers him a promotion, but the new role will require him to work Sundays. Michael would like the promotion but realizes that it would force him to miss some church activities. In this situation, Michael is most likely to experience ________. A) role conflict B) social loafing C) groupthink D) role fuzziness E) groupshift Answer: A Explanation: When compliance with one role requirement may make it difficult to comply with another, the result is role conflict. Michael is experiencing conflict between his role as a church member and his role as an employee. LO: 9.3: Show how role requirements change in different situations. AACSB: Analytical thinking Difficulty: Moderate Employability Skills: Knowledge Application and Analysis Quest. Category: Application 37) Role expectations refer to our view of how we are supposed to act in a given situation. Answer: FALSE Explanation: Role expectations are the way others believe you should act in a given context. Role perception indicates our view of how we're supposed to act in a given situation. LO: 9.3: Show how role requirements change in different situations. Difficulty: Easy Quest. Category: Concept 38) Role expectations at work are often conveyed through the psychological contract. Answer: TRUE Explanation: In the workplace, we look at role expectations through the perspective of the psychological contract: an unwritten agreement that exists between employees and employer. This agreement sets out mutual expectations: what management expects from workers and vice versa. LO: 9.3: Show how role requirements change in different situations. Difficulty: Easy Quest. Category: Concept 39) What is the difference between role perception and role expectations? Answer: Our view of how we're supposed to act in a given situation is role perception. We engage in certain types of behavior based on how we believe we are supposed to behave. We get these perceptions from stimuli all around us—for example, friends, books, films, television. Role expectation is the way others believe you should act in a given context. The role of a U.S. federal judge is viewed as having propriety and dignity, whereas a football coach is seen as aggressive, dynamic, and inspiring to his players. LO: 9.3: Show how role requirements change in different situations. Difficulty: Moderate 393 richard@qwconsultancy.com
Quest. Category: Concept 40) What is the relationship between role conflict and reference groups. Answer: When compliance with one role requirement may make it difficult to comply with another, the result is role conflict. Since most employees are simultaneously in occupations, work groups, divisions, and demographic groups, these different identities can come into conflict when the expectations of one clash with the expectations of another. In such situations, they conform to the norms and expectations of important groups to which they belong or hope to belong. These important groups are reference groups, in which a person is aware of other members, defines himself or herself as a member or would like to be a member, and feels group members are significant to him or her. LO: 9.3: Show how role requirements change in different situations. AACSB: Analytical thinking Difficulty: Moderate Quest. Category: Synthesis 41) ________ refer(s) to the acceptable standards of behavior within a group that are shared by the group's members. A) Status B) Norms C) Dyads D) Goals E) Cliques Answer: B Explanation: Norms represent the acceptable standards of behavior shared by group members that express what they ought and ought not to do under certain circumstances. LO: 9.4: Demonstrate how norms exert influence on an individual's behavior. AACSB: Interpersonal relations and teamwork Difficulty: Easy Quest. Category: Concept 42) Which of the following is true of norms? When group members adopt and agree to the norms of a group, which of the following is true? A) Dress codes play a more important role. B) Written rules become very important. C) A clear set of written group norms is necessary. D) A maximum of external controls is necessary. E) They need to be accepted by all in order to be adopted. Answer: E Explanation: Norms are not just leader-established, opinion-driven policies: For them to be adopted (and not abandoned after three days), they need to be accepted by all. LO: 9.4: Demonstrate how norms exert influence on an individual's behavior. AACSB: Interpersonal relations and teamwork Difficulty: Moderate Quest. Category: Concept
394 richard@qwconsultancy.com
395 richard@qwconsultancy.com
43) Which of the following refers to the adjustment of one's behavior to align with the norms of the group? A) Deviance B) Conflict C) Divergence D) Conformity E) Appearance Answer: D Explanation: Conformity refers to the adjustment of one's behavior to align with the norms of the group. LO: 9.4: Demonstrate how norms exert influence on an individual's behavior. AACSB: Interpersonal relations and teamwork Difficulty: Easy Quest. Category: Concept 44) ________ can dictate the experience of emotions for individuals and their groups. A) Norms B) Conforming ideas C) Power divergences D) Missions E) Controls Answer: A Explanation: Norms can dictate the experience of emotions for individuals and their groups. LO: 9.4: Demonstrate how norms exert influence on an individual's behavior. AACSB: Interpersonal relations and teamwork Difficulty: Moderate Quest. Category: Concept 45) ________ groups refer to the important groups to which an individual belongs or hopes to belong. A) Interacting B) Reference C) Organizational D) Control E) Nominal Answer: B Explanation: Reference groups are groups in which a person is aware of other members, defines himself or herself as a member or would like to be a member, and feels group members are significant to him or her. LO: 9.4: Demonstrate how norms exert influence on an individual's behavior. AACSB: Interpersonal relations and teamwork Difficulty: Easy Quest. Category: Concept
396 richard@qwconsultancy.com
46) Jonas Wilkes has struggled for many years with tight finances at home. Throughout school and college, he worked really hard, earned the best grades, and prepared himself for a better life. By a better life, he meant a house in the uptown Wellington Road and a membership in the Diamond District Club where he envisions himself playing golf on weekends. The residents of Wellington Road and the members of Diamond District Club serve as a(n) ________ for him. A) nominal group B) organizational group C) control group D) intervention group E) reference group Answer: E Explanation: Reference groups are groups in which a person is aware of other members, defines himself or herself as a member or would like to be a member, and feels group members are significant to him or her. LO: 9.4: Demonstrate how norms exert influence on an individual's behavior. AACSB: Analytical thinking Difficulty: Moderate Employability Skills: Knowledge Application and Analysis Quest. Category: Application 47) ________ refers to voluntary actions that violate significant organizational norms and, in doing so, threaten the well-being of the organization or its members. A) Undue hardships B) Deviant workplace behavior C) Antidiscrimination and retaliation D) Progressive discipline E) Wrongful termination Answer: B Explanation: Deviant workplace behavior (also called antisocial behavior or workplace incivility) is voluntary behavior that violates significant organizational norms and, in doing so, threatens the well-being of the organization or its members. LO: 9.4: Demonstrate how norms exert influence on an individual's behavior. AACSB: Interpersonal relations and teamwork Difficulty: Moderate Quest. Category: Concept
397 richard@qwconsultancy.com
48) Which of the following is true with regard to deviant workplace behavior? A) It is involuntary in nature. B) Evidence demonstrates deviant workplace behavior is likely to flourish where it is supported by group norms. C) It has a negligible adverse impact on organization as it is accepted by some employees. D) Research on deviance at the workplace has shown that employees operating individually engage in more deviant behaviors than those who function as a part of groups. E) Leaving work early is an example of a property-related deviant workplace behavior. Answer: B Explanation: Like norms in general, individual employees' antisocial actions are shaped by the group context within which they work. Evidence demonstrates deviant workplace behavior is likely to flourish where it's supported by group norms. LO: 9.4: Demonstrate how norms exert influence on an individual's behavior. AACSB: Interpersonal relations and teamwork Difficulty: Moderate Quest. Category: Concept 49) Norms can cover any aspect of group behavior. Answer: TRUE Explanation: This is true; norms can cover any aspect of group behavior. LO: 9.4: Demonstrate how norms exert influence on an individual's behavior. AACSB: Interpersonal relations and teamwork Difficulty: Easy Quest. Category: Concept 50) Positive norms always portend positive outcomes in a company. Answer: FALSE Explanation: Positive group norms may well beget positive outcomes, but only if other factors are present, too. LO: 9.4: Demonstrate how norms exert influence on an individual's behavior. AACSB: Interpersonal relations and teamwork Difficulty: Moderate Quest. Category: Concept 51) Unethical or deviant workplace behaviors may be observed when work groups become characterized by negative attributes. Answer: TRUE Explanation: A workgroup can become characterized by positive or negative attributes. When those attributes are negative, unethical or deviant behaviors may be observed more frequently. LO: 9.4: Demonstrate how norms exert influence on an individual's behavior. AACSB: Ethical understanding and reasoning Difficulty: Moderate Employability Skills: Business Ethics and Social Responsibility Quest. Category: Concept
398 richard@qwconsultancy.com
52) How can norms be used to reduce deviant workplace behavior? Answer: Deviant workplace behavior (also called antisocial behavior or workplace incivility) is voluntary behavior that violates significant organizational norms and, in doing so, threatens the well-being of the organization or its members. Norms are the acceptable standards of behavior shared by their members that express what they ought and ought not to do under certain circumstances. Like norms in general, individual employees' antisocial actions are shaped by the group context within which they work. Evidence demonstrates that antisocial behavior exhibited by a work group is a significant predictor of an individual's antisocial behavior at work. In other words, deviant workplace behavior is likely to flourish where it's supported by group norms. Managers must observe the various aspects of group norms keenly. If and when deviant workplace norms surface, they must anticipate negative changes in employee cooperation, commitment, and motivation and take remedial steps. Further, establishing norms that have clear demarcations as to what constitutes deviant workplace behavior and its implications is likely to reduce such acts in the workplace. LO: 9.4: Demonstrate how norms exert influence on an individual's behavior. AACSB: Analytical thinking Difficulty: Moderate Employability Skills: Critical Thinking Quest. Category: Synthesis 53) Within the context of group decision making, explain effectiveness and efficiency. Answer: Whether groups are more effective than individuals depends on how you define effectiveness. Group decisions are generally more accurate than the decisions of the average individual in a group, but they are less accurate than the judgments of the most accurate person. In terms of speed, individuals are superior. If creativity is important, groups tend to be more effective. And if effectiveness means the degree of acceptance of achievable solutions, groups are superior. LO: 9.4: Demonstrate how norms exert influence on an individual's behavior. AACSB: Interpersonal relations and teamwork Difficulty: Moderate Quest. Category: Concept 54) How do differing cultural norms affect the choice of negotiating strategies? Answer: Cultural norms can affect many behaviors in the workplace, such as which negotiating strategy to select. Aggressive strategies are more often used in honor and face cultures, like Qatar and China, relative to dignity cultures like the United States.) The negotiation results are surprising, considering that employees in Asian cultures value harmony and cooperation in interpersonal interactions but will do what they can to compete and save face in professional contexts. Cultural norms can even influence how people respond to those who violate the norm. LO: 9.4: Demonstrate how norms exert influence on an individual's behavior. AACSB: Diverse and multicultural work environments Difficulty: Hard Quest. Category: Concept
399 richard@qwconsultancy.com
55) Which of the following statements is true regarding the effect of status on conformity pressure? A) High-status individuals are often given less freedom to deviate from norms than are other group members. B) People in high-status jobs can actively resist negative reactions to social pressure exerted by people in low-status jobs. C) Lower-status people are better able to resist conformity pressures than their high-status peers. D) Groups exert equal conformity pressure on all individuals regardless of their status. E) Compared to high-status members, low-status members are given a wider range of discretion as long as their activities aren't severely detrimental to group goal achievement. Answer: B Explanation: High-status individuals are often given more freedom to deviate from norms than are other group members. People in high-status jobs (such as physicians, lawyers, and executives) can have negative reactions to social pressure exerted by people in low-status jobs. For example, physicians actively resist administrative decisions made by lower -ranking insurance company employees. LO: 9.5: Show how status and size differences affect group performance. AACSB: Interpersonal relations and teamwork Difficulty: Moderate Quest. Category: Concept 56) Which of the following statements is true regarding the effect that size of the group has on the group's overall behavior? A) The size of the group does not affect the group's overall performance. B) Evidence indicates that larger groups are faster at completing tasks than smaller ones. C) Compared to smaller groups, larger groups are better at idea-generating. D) If the goal of the group is fact-finding, then smaller groups are more effective than larger groups. E) Evidence indicates that individuals perform better in larger groups than in smaller ones. Answer: C Explanation: Does the size of a group affect the group's overall behavior? Yes, but the effect depends on the outcome of interest (e.g., performance, creativity). Groups with a dozen or more members are good for gaining diverse input. If the goal is fact-finding or idea-generating, then larger groups should be more effective. Smaller groups of about seven members are better if the goal is performance and productivity and for "transformational" leadership techniques to be most effect (see the leadership chapter). LO: 9.5: Show how status and size differences affect group performance. AACSB: Interpersonal relations and teamwork Difficulty: Moderate Quest. Category: Concept
400 richard@qwconsultancy.com
57) Which of the following refers to status? A) The impact that support of group norms has on workplace civility B) An adjustment of one's behavior to align with the norms of the group C) A socially defined position or rank given to groups or group members by others D) The existence of roles that are at variance with one another E) The acceptable standards of behavior within a group that are shared by the group's members Answer: C Explanation: Status refers to the socially defined position or rank given to groups or group members by others. LO: 9.5: Show how status and size differences affect group performance. AACSB: Interpersonal relations and teamwork Difficulty: Easy Quest. Category: Concept 58) According to status characteristics theory, which of the following factors does not determine status? A) Control over the resources needed by the group B) Contribution to goals C) Ability to conform to group norms D) Personality E) Talent Answer: C Explanation: According to status characteristics theory, status tends to derive from one of three sources: the power a person wields over others; a person's ability to contribute to a group's goals; and an individual's personal characteristics (special talent, intelligence, money, or a friendly personality). LO: 9.5: Show how status and size differences affect group performance. AACSB: Interpersonal relations and teamwork Difficulty: Easy Quest. Category: Concept 59) Which of the following terms indicates the tendency of individuals to spend less effort when working collectively? A) Groupthink B) Collective efficacy C) Social loafing D) Groupshift E) Social facilitation Answer: C Explanation: Social loafing is the tendency for individuals to expend less effort when working in a group than when working individually. Group performance increases with group size, but the addition of new members has diminishing returns on productivity. LO: 9.5: Show how status and size differences affect group performance. AACSB: Interpersonal relations and teamwork Difficulty: Easy Quest. Category: Concept 401 richard@qwconsultancy.com
60) Chris Bradford thrives in his position as senior analyst for a major electric power project. Despite the high pressure and tight deadlines that shape his day, Chris finds his work fulfilling and meaningful. Chris recently assigned several of his best team members to work together on an important and urgent task and was happy to see the team meeting. Chris was in a state of disbelief however, when, at the halfway point on the project, he checked on the team's progress only to find that the team was far behind schedule. This scenario reflects ________. A) groupthink B) collective efficacy C) social loafing D) groupshift E) ingroup favoritism Answer: C Explanation: Social loafing is the tendency for individuals to expend less effort when working in a group than when working individually. Group performance increases with group size, but the addition of new members has diminishing returns on productivity. LO: 9.5: Show how status and size differences affect group performance. AACSB: Analytical thinking Difficulty: Moderate Employability Skills: Knowledge Application and Analysis Quest. Category: Application 61) A contributing factor with regard to social loafing is ________. A) difficult task assignments B) high collective efficacy C) severe time constraints D) diffusion of responsibility E) heavy workload Answer: D Explanation: Social loafing is the tendency for individuals to expend less effort when working in a group than when working individually. One explanation for social loafing is the diffusion of responsibility. Because group results cannot be attributed to any single person, the relationship between an individual's input and the group's output is clouded. Individuals may then be tempted to become free riders and coast on the group's efforts. LO: 9.5: Show how status and size differences affect group performance. AACSB: Interpersonal relations and teamwork Difficulty: Easy Quest. Category: Concept
402 richard@qwconsultancy.com
62) Which of the following is an effective means of countering social loafing? A) Increasing the rewards the group is given if it succeeds B) Increasing the amount by which the group's progress is monitored C) Ensuring that individual contributions to the group's outcome are identified D) Increasing the size of the group E) Increasing the group's workload Answer: C Explanation: Since group results cannot be attributed to any single person, the relationship between an individual's input and the group's output is clouded. Individuals may then be tempted to become free riders and coast on the group's efforts. To avoid this, individual efforts in a group's outcome should be identified. LO: 9.5: Show how status and size differences affect group performance. AACSB: Interpersonal relations and teamwork Difficulty: Moderate Quest. Category: Concept 63) Which of the following is true with regard to social loafing? A) Social loafing decreases when rewards are added for collective performance of the group. B) Social loafing reflects the drastic rise in employee performance when employees work collectively. C) Social loafing involves the cumulative rise in productivity when teams increase in size. D) Social loafing is a result of the emphasis on individual outcomes over shared outcomes. E) Virtual groups may have a problem with social loafing especially if members are dissimilar. Answer: E Explanation: Virtual groups may have a problem with social loafing especially if members are dissimilar and have a number of family responsibilities that detract their attention away from group performance. LO: 9.5: Show how status and size differences affect group performance. AACSB: Diverse and multicultural work environments Difficulty: Moderate Quest. Category: Concept
403 richard@qwconsultancy.com
64) GREEN is a non-profit organization that runs awareness campaigns and research programs that provide data to initiate the process of legislative changes on various aspects of the environment like forest cover, nuclear fuel, endangered species and others. GREEN operates through networks of grass-root level researchers who are grouped into flexible project teams whose roles and duties change with each new project. GREEN believes its employees to be like family, and an informal environment pervades its functioning without any compromises being made in the efficiency. Recently, the project manager of the rainwater harvesting pilot project in the suburbs has been informed of two delays in the project. When the third extension request came to him, he looked into the matter by speaking individually with the five team members. Which of the following complaints by the team members, if true, would indicate the presence of social loafing in the team? A) A team member complained that the residents of the suburb in which they were working were uncooperative and did not accept their most cordial requests for helping in data collection. B) Nancy, one of the new team members, reported that she felt the team was not cohesive enough. C) Two of the most experienced employees on the team reported having to shoulder a disproportionate amount of the current workload and asked for an internal transfer. D) The scarcity of funds was one of the major concerns that all the team members mentioned. E) The team members were dissatisfied with the compensation package as they felt they were putting in a lot of work and being paid relatively less. Answer: C Explanation: The fact that a team member complained that the residents of the suburb in which they were working were uncooperative hints at an external cause for lack of performance and does not indicate social loafing. The fact that one of the new team members reported that she felt the team was not cohesive enough does not indicate social loafing; social loafing is typically a product of excessive cohesiveness. The fact that two of the most experienced employees on the team reported having to shoulder a disproportionate amount of the current workload does indicate the presence of social loafing. One explanation for social loafing is the dispersion of responsibility. Because group results cannot be attributed to any single person, the relationship between an individual's input and the group's output is clouded. Individuals may then be tempted to become free riders and coast on the group's efforts. The scarcity of funds for the project is an external factor and cannot directly explain their bad performance. The fact that the team members were dissatisfied with the compensation package does not indicate the presence of loafing, especially since two employees have been working hard. LO: 9.5: Show how status and size differences affect group performance. AACSB: Analytical thinking Difficulty: Moderate Employability Skills: Critical Thinking Quest. Category: Critical Thinking
404 richard@qwconsultancy.com
65) Inequity in power-based forms of status tend to drive interpersonal conflict. Answer: TRUE Explanation: Generally, inequity in power-based forms of status tends to have more negative effects: It tends to drive interpersonal conflict, whereas other forms promote healthy competition for status advancement. LO: 9.5: Show how status and size differences affect group performance. AACSB: Interpersonal relations and teamwork Difficulty: Moderate Quest. Category: Concept 66) An individual's personal characteristics, like good looks or friendly personality, represent one of the dimensions of the status characteristics theory. Answer: TRUE Explanation: According to status characteristics theory, status tends to derive from one of three sources: a person's power, ability to contribute to group goals, and his or her personality characteristics. LO: 9.5: Show how status and size differences affect group performance. AACSB: Interpersonal relations and teamwork Difficulty: Moderate Quest. Category: Concept 67) High-status individuals may be more likely to deviate from norms when they have low identification (social identity) with the group. Answer: TRUE Explanation: Status has some interesting effects on the power of norms and pressures to conform. High-status individuals may be more likely to deviate from norms when they have low identification (social identity) with the group. LO: 9.5: Show how status and size differences affect group performance. AACSB: Interpersonal relations and teamwork Difficulty: Moderate Quest. Category: Concept 68) Explain the three sources of status presented in the status characteristics theory. Answer: According to status characteristics theory, status tends to derive from one of three sources: 1. The power a person wields over others. Because they likely control the group's resources, people who control the outcomes tend to be perceived as high status. 2. A person's ability to contribute to a group's goals. People whose contributions are critical to the group's success tend to have high status. 3. An individual's personal characteristics. Someone whose personal characteristics are positively valued by the group (good looks, intelligence, money, or a friendly personality) typically has higher status than someone with fewer valued attributes. LO: 9.5: Show how status and size differences affect group performance. AACSB: Reflective thinking Difficulty: Moderate Quest. Category: Concept 405 richard@qwconsultancy.com
69) Explain the different strategies managers can use to prevent social loafing. Answer: There are four ways to prevent social loafing: (1) set group goals, so the group has a common purpose to strive toward; (2) increase intergroup competition, which focuses on the shared group outcome; (3) engage in peer evaluation; (4) select members who have high motivation and prefer to work in groups; and (5) base group rewards in part on each member's unique contributions. LO: 9.5: Show how status and size differences affect group performance. AACSB: Reflective thinking Difficulty: Moderate Quest. Category: Concept 70) Which of the following refers to the degree to which members are attracted to each other and motivated to stay in the group? A) Assertiveness B) Social dominance C) Authoritativeness D) Cohesion E) Diversity Answer: D Explanation: Cohesion refers to the degree to which members are attracted to each other and motivated to stay in the group. LO: 9.6: Describe how cohesion is related to group effectiveness. AACSB: Interpersonal relations and teamwork Difficulty: Easy Quest. Category: Concept 71) Which of the following statements is true regarding the effect of group cohesiveness and performance norms on group productivity? A) When cohesiveness is low and performance norms are also low, productivity will be high. B) The productivity of the group is affected by the performance norms but not by the cohesiveness of the group. C) If cohesiveness is high and performance norms are low, productivity will be high. D) If cohesiveness is low and performance norms are high, productivity will be low. E) When both cohesiveness and performance norms are high, productivity will be high. Answer: E Explanation: Studies consistently show that the relationship between cohesiveness and productivity depends on the group's performance-related norms. If performance-related norms are high, a cohesive group will be more productive than will a less cohesive group. If cohesiveness is high and performance norms are low, productivity will be low. If cohesiveness is low and performance norms are high, productivity increases, but less than in the highcohesiveness/high-norms situation. When cohesiveness and performance-related norms are both low, productivity tends to fall into the low-to-moderate range. LO: 9.6: Describe how cohesion is related to group effectiveness. AACSB: Interpersonal relations and teamwork Difficulty: Moderate 406 richard@qwconsultancy.com
Quest. Category: Concept 72) If norms for output and cooperation with outsiders is ________, a cohesive group ________ productive. A) high; will be B) low; will be C) strong; won't be D) weak; won't be E) turbulent; will be Answer: A Explanation: If norms for output and cooperation with outsiders is high, a cohesive group will be productive. Conversely, when cohesion is low and performance norms are high, productivity will be only moderate. LO: 9.6: Describe how cohesion is related to group effectiveness. AACSB: Interpersonal relations and teamwork Difficulty: Moderate Quest. Category: Concept 73) Stimulating competition with other groups reduces group cohesiveness. Answer: FALSE Explanation: Stimulating competition with other groups encourages group cohesiveness. LO: 9.6: Describe how cohesion is related to group effectiveness. AACSB: Interpersonal relations and teamwork Difficulty: Moderate Quest. Category: Concept 74) Groups lead to decreased acceptance of a solution. Answer: FALSE Explanation: Groups lead to increased acceptance of a solution. Group members who participate in making a decision are more likely to support it enthusiastically and to encourage others how to accept it later. LO: 9.6: Describe how cohesion is related to group effectiveness. Difficulty: Moderate Quest. Category: Concept 75) Individual decisions are less time consuming than group decisions. Answer: TRUE Explanation: Group decisions have their drawbacks. They're time consuming because groups typically take more time to reach a solution. LO: 9.6: Describe how cohesion is related to group effectiveness. AACSB: Interpersonal relations and teamwork Difficulty: Moderate Quest. Category: Concept
407 richard@qwconsultancy.com
76) Amy Jones has to come up with a strategy to regulate the excessive use of the Internet by her employees in such a way that they accept and commit to the solution themselves. It is advisable for her to use group decision making rather than individual decision making in this situation. Answer: TRUE Explanation: When effectiveness of a decision is measured by the degree of acceptance the final solution achieves, group decision making is more effective. LO: 9.6: Describe how cohesion is related to group effectiveness. AACSB: Analytical thinking Difficulty: Moderate Employability Skills: Knowledge Application and Analysis Quest. Category: Application 77) Groupshift is a phenomenon that relates to the consensus norms. Answer: FALSE Explanation: Groupshift refers to a change between a group's decision and an individual decision that a member within the group would make; the shift can be toward either conservatism or greater risk, but it generally is toward a more extreme version of the group's original position. Groupthink refers to a phenomenon in which the norm for consensus overrides the realistic appraisal of alternative courses of action. LO: 9.6: Describe how cohesion is related to group effectiveness. AACSB: Interpersonal relations and teamwork Difficulty: Moderate Quest. Category: Concept 78) Describe cohesiveness as a property of groups. How can a group be made more cohesive? Answer: Cohesiveness is the degree to which members are attracted to each other and motivated to stay in the group. Some strategies to encourage cohesiveness are (1) making the group smaller, (2) encouraging agreement with group goals, (3) increasing the time members spend together, (4) increasing the group's status and the perceived difficulty of attaining membership, (5) stimulating competition with other groups, (6) giving rewards to the group rather than to individual members, and (7) physically isolating the group. LO: 9.6: Describe how cohesion is related to group effectiveness. AACSB: Interpersonal relations and teamwork Difficulty: Moderate Quest. Category: Concept
408 richard@qwconsultancy.com
79) What is the relationship between performance norms, cohesiveness, and group productivity? Answer: Cohesiveness increases group productivity, and this relationship between cohesiveness and productivity depends on the group's performance-related norms. If norms for quality, output, and cooperation with outsiders, for instance, are high, a cohesive group will be more productive than will a less cohesive group. However, if cohesiveness is high and performance norms are low, productivity will be low. If cohesiveness is low and performance norms are high, productivity increases, but less than in the high-cohesiveness/ high-norms situation. When cohesiveness and performance-related norms are both low, productivity tends to fall into the lowto-moderate range. LO: 9.6: Describe how cohesion is related to group effectiveness. AACSB: Analytical thinking Difficulty: Moderate Employability Skills: Critical Thinking Quest. Category: Synthesis 80) Increased ________ is an advantage of group decision making when compared to individual decision making. A) ambiguous responsibility B) conformity pressures C) diversity of views D) dispersion of accountability E) pace of decision making Answer: C Explanation: By aggregating the resources of several individuals, groups bring more input as well as heterogeneity into the decision process. They offer increased diversity of views. This opens up the opportunity to consider more approaches and alternatives. LO: 9.7: Contrast the strengths and weaknesses of group decision making. AACSB: Interpersonal relations and teamwork Difficulty: Moderate Quest. Category: Concept
409 richard@qwconsultancy.com
81) Group decisions are preferred to individual decisions when ________ is important in decision making. A) acceptance of a solution B) speed C) efficiency D) clear responsibility E) legitimacy Answer: A Explanation: Groups lead to increased acceptance of a solution. Many decisions fail because people don't accept the solution. Group members who participated in making a decision are more likely to enthusiastically support the decision and encourage others to accept it. Group decisions are generally more accurate than the decisions of the average individual in a group but less accurate than the judgments of the most accurate. So in terms of accuracy, groups do not offer any unique advantage over individuals. LO: 9.7: Contrast the strengths and weaknesses of group decision making. AACSB: Interpersonal relations and teamwork Difficulty: Moderate Quest. Category: Concept 82) A disadvantage of group decision making as compared to individual decision making is related to ________. A) diversity B) creativity C) acceptance of final solutions D) speed E) accuracy Answer: D Explanation: When evaluating the effectiveness and efficiency of group decision making, speed of decision making is an area where individuals fare better. LO: 9.7: Contrast the strengths and weaknesses of group decision making. AACSB: Interpersonal relations and teamwork Difficulty: Moderate Quest. Category: Concept
410 richard@qwconsultancy.com
83) Which of the following describes a phenomenon in which the norm for consensus overrides the realistic appraisal of alternative courses of action? A) Groupshift B) Social loafing C) Ingroup favoritism D) Cyber loafing E) Groupthink Answer: E Explanation: Groupthink is defined as a phenomenon in which the norm for consensus overrides the realistic appraisal of alternative courses of action. LO: 9.7: Contrast the strengths and weaknesses of group decision making. AACSB: Interpersonal relations and teamwork Difficulty: Moderate Quest. Category: Concept 84) Sonia Soans is a corporate trainer. One of her favorite ways to start a training program is to present a case study and encourage members to think of the case in depth and discuss it amongst each other. Using the results of the activity, she understands the baseline measures of the group's functioning. During one of her recent sessions, she observed that the group came to a consensus very quickly. However, when asked to present their views, the team members were not too confident and appeared to be under an illusion of unanimity. From this scenario, we can say that this group experienced ________. A) diversity B) groupshift C) role conflict D) groupthink E) social loafing Answer: D Explanation: Groupthink relates to norms and describes situations in which group pressures for conformity deter the group from critically appraising unusual, minority, or unpopular views. LO: 9.7: Contrast the strengths and weaknesses of group decision making. AACSB: Analytical thinking Difficulty: Moderate Quest. Category: Application
411 richard@qwconsultancy.com
85) One way to minimize groupthink is to ________. A) increase the group size B) encourage group leaders to develop a stronger sense of group identity C) prevent all team members from engaging in a critical evaluation of ideas at the beginning D) ask the group members to first focus on the positives of an alternative rather than the negatives E) seek input from employees before the group leader presents his or her opinions Answer: E Explanation: In order to minimize groupthink, managers should encourage group leaders to play an impartial role. Leaders should actively seek input from all members and avoid expressing their own opinions, especially in the early stages of deliberation. In addition, managers should appoint one group member to play the role of devil's advocate; this member's role is to overtly challenge the majority position and offer divergent perspectives. LO: 9.7: Contrast the strengths and weaknesses of group decision making. AACSB: Interpersonal relations and teamwork Difficulty: Moderate Quest. Category: Concept 86) ________ refers to a change between a group's decision and an individual decision that a member within the group would make; the shift can be toward either conservatism or greater risk, but it generally is toward a more extreme version of the group's original position. A) Social desirability bias B) Groupshift C) Halo effect D) Social loafing E) Ingroup favoritism Answer: B Explanation: Groupshift refers to a change between a group's decision and an individual decision that a member within the group would make; the shift can be toward either conservatism or greater risk, but it generally is toward a more extreme version of the group's original position. LO: 9.7: Contrast the strengths and weaknesses of group decision making. AACSB: Interpersonal relations and teamwork Difficulty: Moderate Quest. Category: Concept
412 richard@qwconsultancy.com
87) When ________, group shift can be observed. A) group members view themselves in better light when compared to members of the out-group B) group members tend to exaggerate the initial positions they hold when discussing a given set of alternatives and arriving at a solution C) group members notice and emphasize identities that reflect how different they are from other groups D) group members tend to prefer and endorse the products, ideas, and aspects of someone else's culture over their own E) group members fail to express alternative opinions and deviant views under the influence of the norm for consensus Answer: B Explanation: Groupshift is a change between a group's decision and an individual decision that a member within the group would make; the shift can be toward either conservatism or greater risk, but it generally is toward a more extreme version of the group's original position. LO: 9.7: Contrast the strengths and weaknesses of group decision making. AACSB: Interpersonal relations and teamwork Difficulty: Moderate Quest. Category: Concept 88) The HR department at Bailey Services is considering offering telecommuting as an option to some experienced employees. At a meeting to formalize the move, the heads of the different departments met with Laura Watson, the HR manager. Laura, who thought this meeting would be a short one, was proven wrong when all the members, who had previously agreed that telecommuting would work in their company, began coming up with divergent views. By the end, two divisions claimed they had major concerns about allowing employees to telecommute, and those in favor of it were equally vocal. This scenario depicts the operation of ________. A) conformity B) groupshift C) role conflict D) groupthink E) social loafing Answer: B Explanation: Groupshift is a change between a group's decision and an individual decision that a member within the group would make; the shift can be toward either conservatism or greater risk, but it generally is toward a more extreme version of the group's original position. LO: 9.7: Contrast the strengths and weaknesses of group decision making. AACSB: Analytical thinking Difficulty: Moderate Employability Skills: Knowledge Application and Analysis Quest. Category: Application
413 richard@qwconsultancy.com
89) Joe Sullivan and Mark Holland, members of the top management at EuAir, a European airlines, were preparing for a meeting to discuss strategies to combat the recent rise in fuel prices. Before the meeting began, Joe and Mark were discussing how oil prices significantly impact the health of the world economy. Joe spoke of how higher oil prices since 1999, partly the result of OPEC supply management policies, contributed to the global economic downturn in 2000-2001 and have not been stable since. Mark agreed but added that the right kind of strategy could help them overcome this challenge, and that they could even use the situation to hike fares and generate additional revenues. Which of the following statements, if true, would denote the occurrence of groupshift in Mark's opinions during the meeting? A) Mark thought that EuAir should suspend some of its less profitable flights in the short run in favor of the routes that have greater demand among consumers. B) Mark proposed that this was an opportunity for EuAir to use its brand name effectively and diversify into other products and services. C) Mark agreed with Joe's opinion that providing the best service possible, even if it meant incurring a loss in the short run, would be the best strategy. D) Mark proposed that the prices be hiked and additional customer service measures be included so costumers have the best experience flying with EuAir. E) Mark encouraged the top-management team to consider laying off surplus employees and rightsizing EuAir to enhance its efficiency and lower costs. Answer: D Explanation: The fact that Mark thought that EuAir should suspend some of its less profitable flights in the short run, in favor of the routes that have greater demand among consumers, does not indicate groupshift as it does not strengthen his prior view. The fact that Mark proposed that this was an opportunity for EuAir to use its brand name effectively and diversify into other products and services does not represent groupshift as it does not strengthen his prior view. The fact that Mark agreed with Joe's opinion that providing the best service possible, even if it meant incurring a loss in the short run, would be the best strategy is contrary to his prior view and does not indicate groupshift. The fact that Mark proposed that the prices be hiked and additional customer service measures be included represents an extreme version of his previously held opinion and thus denotes groupshift. The fact that Mark encouraged the top-management team to consider laying off surplus employees does not strengthen his prior view and cannot represent groupshift. LO: 9.7: Contrast the strengths and weaknesses of group decision making. AACSB: Analytical thinking Difficulty: Hard Employability Skills: Critical Thinking Quest. Category: Critical Thinking
414 richard@qwconsultancy.com
90) Joe Sullivan and Mark Holland, members of the top management at EuAir, a European airlines, were preparing for a meeting to discuss strategies to combat the recent rise in fuel prices. Before the meeting began, Joe and Mark were discussing how oil prices significantly impact the health of the world economy. Joe spoke of how higher oil prices since 1999, partly the result of OPEC supply management policies, contributed to the global economic downturn in 2000-2001. Mark agreed but added that the right kind of strategy could help them overcome this challenge, and that they could even use the situation to hike fares and generate additional revenues. Which of the following statements, if true, would weaken the argument that Mark experienced groupshift in the meeting? A) Mark argued that implementing a price rise was necessary to recover the costs in operating the flights. B) Mark stated that the company should increase operations but at lower fares so they can regain control over the market share. C) Mark believed that most of their clientele was not price sensitive. D) Mark stated that oil suppliers were going to pressure them even more in the future and the company had to resort to alternatives like biofuels. E) Mark recommended that the company should establish a fuel hedging contract with its key suppliers to ensure protection from fuel price rises. Answer: B Explanation: The fact that Mark argued that implementing a price rise was necessary to recover the costs in operating the flights does strengthen his prior view and represents groupshift. The fact that Mark stated that the company should increase operations but at lower fares is contrary to his previously held view and weakens the argument that he experienced groupshift. The fact that Mark believed that most of their clientele was not price sensitive is aligned to his view of increasing prices and does not weaken the possibility of groupshift. The fact that Mark felt that the company had to resort to alternatives like biofuels does not strengthen or weaken his argument in favor of price rise. The fact that Mark recommended establishing a fuel hedging contract with its key suppliers to ensure protection from fuel price rises does not impact his view on hiking fares and thus does not weaken the argument that he experienced groupshift. LO: 9.7: Contrast the strengths and weaknesses of group decision making. AACSB: Analytical thinking Difficulty: Hard Employability Skills: Critical Thinking Quest. Category: Critical Thinking
415 richard@qwconsultancy.com
91) Joe Sullivan and Mark Holland, members of the top management at EuAir, a European airlines, were preparing for a meeting to discuss strategies to combat the recent rise in fuel prices. Before the meeting began, Joe and Mark were discussing how oil prices significantly impact the health of the world economy. Joe spoke of how higher oil prices since 1999, partly the result of OPEC supply management policies, contributed to the global economic downturn in 2000-2001. Mark agreed but added that the right kind of strategy could help them overcome this challenge, and that they could even use the situation to hike fares and generate additional revenues. Which of the following statements, if true, would denote the occurrence of groupshift in Joe's opinions during the meeting? A) Joe proposed that the company should invite price quotes from new suppliers to reduce their dependence on their current fuel suppliers. B) Joe determined that the best strategy would be to slash prices and minimize all fringes that are used for customer service. C) Joe convinced the top management team to decrease the capacity of the flights. D) Joe recommended that the prices for business class fares be increased as they represent a less price-sensitive group of customers. E) Joe claimed that aviation fuel was their second largest expense and this was the time to contemplate obtaining a few futures contracts with suppliers. Answer: B Explanation: The fact that Joe proposed that the company should invite price quotes from new suppliers to reduce their dependence on their current fuel suppliers does not represent an extreme version of his prior view. The fact that Joe felt that the best strategy would be to slash prices and minimize all fringes that are used for customer service represents an extreme version of his prior view and denotes groupshift. The fact that Joe convinced the top management team to decrease the capacity of the flights does not indicate groupshift as it is not a stronger manifestation of his previous views on hiking fares. The fact that Joe recommended that the prices for business class fares be increased is contrary to his previous view and does not indicate groupshift. Joe's proposal of obtaining futures contracts does not strengthen or weaken his argument against hiking fares and thus does not represent groupshift. LO: 9.7: Contrast the strengths and weaknesses of group decision making. AACSB: Analytical thinking Difficulty: Hard Employability Skills: Critical Thinking Quest. Category: Critical Thinking
416 richard@qwconsultancy.com
92) Groupthink is most commonly seen in the group decision-making approach using ________. A) reference groups B) nominal groups C) brainstorming D) interacting groups E) electronic meeting groups Answer: D Explanation: The most common form of group decision making takes place in interacting groups. Members meet face-to-face and rely on both verbal and nonverbal interactions to communicate. Interacting groups often censor themselves and pressure individual members toward conformity of opinion. LO: 9.7: Contrast the strengths and weaknesses of group decision making. AACSB: Interpersonal relations and teamwork Difficulty: Moderate Quest. Category: Concept 93) Like many other marketing strategy specialists, Mark Fritz relies heavily on creativity and originality. Members of his team are selected on the basis of their ability to think divergently, and Mark often conducts activities to ensure that this ability is developed. For instance, before any new project, Mark invites his team to sit together and churn out possible ideas about the new product, its theme, and ways in which it can be projected best in the market. These sessions usually provide him with a bank of potential ideas from which the team selects some strong concepts and develops them into a campaign. To encourage freedom in these sessions, Mark has a no-evaluation policy. This is an example of ________. A) ingroup favoritism B) groupshift C) brainstorming D) groupthink E) social loafing Answer: C Explanation: This is an example of brainstorming. Brainstorming is an idea-generation process that specifically encourages any and all alternatives while withholding any criticism of those alternatives. LO: 9.7: Contrast the strengths and weaknesses of group decision making. AACSB: Analytical thinking Difficulty: Moderate Employability Skills: Knowledge Application and Analysis Quest. Category: Application
417 richard@qwconsultancy.com
94) Which of the following statements is true regarding brainstorming? A) Brainstorming encourages criticizing an idea as early as possible. B) Brainstorming overcomes the problem of "production blocking." C) Brainstorming can overcome the pressures for conformity. D) Research consistently shows that a group in a brainstorming session generates more ideas than an individual working alone. E) Brainstorming fails to develop group cohesiveness. Answer: C Explanation: Brainstorming can overcome the pressures for conformity that dampen creativity by encouraging any and all alternatives while withholding criticism. Research consistently shows individuals working alone generate more ideas than a group in a brainstorming session. One reason for this is "production blocking." When people are generating ideas in a group, many are talking at once, which blocks the thought process and eventually impedes the sharing of ideas. Brainstorming develops group cohesiveness. LO: 9.7: Contrast the strengths and weaknesses of group decision making. AACSB: Interpersonal relations and teamwork Difficulty: Moderate Quest. Category: Concept 95) The ________ technique is a group decision-making method in which individual members meet face-to-face to pool their judgments in a systematic but independent fashion. A) reference group B) nominal group C) brainstorming D) interacting group E) ingroup Answer: B Explanation: The nominal group technique is a group decision-making method in which individual members meet face-to-face to pool their judgments in a systematic but independent fashion. LO: 9.7: Contrast the strengths and weaknesses of group decision making. AACSB: Interpersonal relations and teamwork Difficulty: Easy Quest. Category: Concept
418 richard@qwconsultancy.com
96) For seven years, Bonnie Patterson has been a manager at Wayne and Watson, a legal consultancy firm. A good part of her workday involves holding meetings, and she likes to follow a well-defined schedule. For this reason, members of her team receive the agenda at the beginning of the meeting, followed by some time to contemplate the issue at hand individually. Subsequently, the team members present their ideas one after the other, the group discusses them together, and lastly, a ranking is done to choose the most favored idea. This represents the ________ approach of group decision making. A) interacting B) reference group C) brainstorming D) nominal group E) groupthink Answer: D Explanation: The nominal group technique restricts discussion or interpersonal communication during the decision-making process, hence the term nominal. Group members are all physically present, as in a traditional committee meeting, but they operate independently. LO: 9.7: Contrast the strengths and weaknesses of group decision making. AACSB: Analytical thinking Difficulty: Moderate Quest. Category: Application 97) Group decision making is preferable to individual decision making because of ________. A) conformity pressures associated with groups B) the domination of one or a few members of a group C) the ambiguous responsibility associated with group decisions D) the increased diversity of views in groups E) the time involved in arriving at group decisions Answer: D Explanation: As compared to individuals, groups generate more complete information and knowledge. By aggregating the resources of several individuals, groups bring more input as well as heterogeneity into the decision process. They offer increased diversity of views. This opens up the opportunity to consider more approaches and alternatives. Finally, groups lead to increased acceptance of a solution. Group members who participate in making a decision are more likely to support it enthusiastically and to encourage others to accept it later. However, group decisions are time-consuming because groups typically take more time to reach a solution. There are conformity pressures. The desire by group members to be accepted and considered an asset to the group can squash any overt disagreement. Group discussion can be dominated by one or a few members. If they're low-and medium-ability members, the group's overall effectiveness will suffer. Finally, group decisions suffer from ambiguous responsibility. In an individual decision, it's clear who is accountable for the final outcome. In a group decision, the responsibility of any single member is diluted. LO: 9.7: Contrast the strengths and weaknesses of group decision making. AACSB: Interpersonal relations and teamwork Difficulty: Moderate Quest. Category: Concept
419 richard@qwconsultancy.com
420 richard@qwconsultancy.com
98) Hubert Gray needs an instruction manual developed for his new product. This is the last step of the project and he has severe budget constraints. He needs a small team of technical writers to work together closely to write the manual on fairly short notice. He needs them to communicate ideas quickly, creatively, and affordably. Which of the following group techniques should Hubert consider? A) Nominal and electronic B) Brainstorming C) Brainstorming and electronic D) Electronic and interacting E) Interacting and brainstorming Answer: E Explanation: Hubert needs a group with high creativity, high cohesion, and low expense. He should choose either interacting or brainstorming techniques to get his task completed. Nominal techniques, although high in creativity and affordability, are slow and only moderately cohesive. Electronic techniques are slow and expensive and have zero cohesion. LO: 9.7: Contrast the strengths and weaknesses of group decision making. AACSB: Analytical thinking Difficulty: Hard Employability Skills: Knowledge Application and Analysis Quest. Category: Application 99) Research consistently shows that a group in a brainstorming session generates fewer ideas than individuals working alone. Answer: TRUE Explanation: Research consistently shows individuals working alone generate more ideas than a group in a brainstorming session. LO: 9.7: Contrast the strengths and weaknesses of group decision making. AACSB: Interpersonal relations and teamwork Difficulty: Moderate Quest. Category: Concept 100) The chief advantage of the nominal group technique is that it permits the group to meet formally but does not restrict independent thinking, as does the interacting group. Answer: TRUE Explanation: The chief advantage of the nominal group technique is that it permits a group to meet formally but does not restrict independent thinking, as does an interacting group. Research generally shows nominal groups outperform brainstorming groups. LO: 9.7: Contrast the strengths and weaknesses of group decision making. AACSB: Interpersonal relations and teamwork Difficulty: Easy Quest. Category: Concept
421 richard@qwconsultancy.com
101) What is the nominal group technique? Answer: The nominal group technique restricts discussion or interpersonal communication during the decision-making process. Group members are all physically present, as in a traditional committee meeting, but they operate independently. Specifically, a problem is presented and then the group takes the following steps: 1. Members meet as a group, but before any discussion takes place, each independently writes down ideas on the problem. 2. After this silent period, each member presents one idea to the group. No discussion takes place until all ideas have been presented and recorded. 3. The group discusses the ideas for clarity and evaluates them. 4. Each group member silently and independently rank-orders the ideas. The idea with the highest aggregate ranking determines the final decision. LO: 9.7: Contrast the strengths and weaknesses of group decision making. AACSB: Reflective thinking Difficulty: Moderate Quest. Category: Concept 102) What are the advantages and disadvantages of group decisions as compared to individual decisions? Answer: As compared to individuals, groups generate more complete information and knowledge. By aggregating the resources of several individuals, groups bring more input as well as heterogeneity into the decision process. They offer increased diversity of views. This opens up the opportunity to consider more approaches and alternatives. Finally, groups lead to increased acceptance of a solution. Group members who participate in making a decision are more likely to support it enthusiastically and to encourage others to accept it later. However, group decisions are time-consuming because groups typically take more time to reach a solution. There are conformity pressures. The desire by group members to be accepted and considered an asset to the group can squash any overt disagreement. Group discussion can be dominated by one or a few members. If they're low-and medium-ability members, the group's overall effectiveness will suffer. Finally, group decisions suffer from ambiguous responsibility. In an individual decision, it's clear who is accountable for the final outcome. In a group decision, the responsibility of any single member is diluted. LO: 9.7: Contrast the strengths and weaknesses of group decision making. Difficulty: Moderate Quest. Category: Concept 103) Explain the impact of brainstorming on the possibility of groupthink. Answer: Groupthink is related to norms. It represents a phenomenon that occurs when the norm for consensus overrides the realistic appraisal of alternative courses and the full expression of deviant, minority, or unpopular views. Brainstorming can overcome the pressures for conformity that dampen creativity by encouraging any and all alternatives while withholding criticism. The norm for consensus is the cause of groupthink, while brainstorming is focused on unhampered generation of ideas. LO: 9.7: Contrast the strengths and weaknesses of group decision making. AACSB: Analytical thinking Difficulty: Moderate 422 richard@qwconsultancy.com
Quest. Category: Synthesis 104) What is group polarization? Answer: We can view group polarization as a special case of groupthink. The group's decision reflects the dominant decision-making norm – toward greater caution or more risk – that develops during discussion. The shift toward polarization has several explanations. It's been argued that discussion makes the members more comfortable with each other and thus more willing to express extreme versions of their original positions. Another argument is that the group diffuses responsibility. Group decisions free any single member from accountability from the group's final choice, so a more extreme position can be taken. It's also likely that people take extreme positions because they want to demonstrate how different they are from the outgroup. LO: 9.7: Contrast the strengths and weaknesses of group decision making. AACSB: Analytical thinking Difficulty: Moderate Quest. Category: Concept Organizational Behavior, 19e (Robbins/Judge) Chapter 10 Understanding Work Teams 1) Which of the following is true of teams? A) They can sometimes achieve feats an individual could never accomplish. B) They represent a better way to use employee talents. C) They reduce the need for coordination and supervision. D) They aid in the performance of simple tasks that do not require diverse inputs. E) They strengthen the worth of individual team players over the team. Answer: A Explanation: Teams can sometimes achieve feats an individual could never accomplish. LO: 10.1: Contrast groups and teams. AACSB: Interpersonal relations and teamwork Difficulty: Moderate Quest. Category: Concept 2) Which of the following statements best defines a work group? A) A work group performs at a level greater than the sum of its inputs from individual members. B) A work group interacts primarily to share information, rather than to engage in work that requires joint effort. C) A work group consists of members who work together and generate positive synergy through coordinated effort. D) A work group consists of members having complementary skills that are applied in a coordinated fashion to the task at hand. E) A work group involves individual and mutual accountability regarding results and outcomes. Answer: B Explanation: A group consists of two or more individuals, interacting and interdependent, who have come together to achieve particular objectives. A work group is defined as a group that interacts primarily to share information and to make decisions to help each group member perform within his or her area of responsibility. They have no need or opportunity to engage in collective work that requires joint effort. 423 richard@qwconsultancy.com
LO: 10.1: Contrast groups and teams. AACSB: Interpersonal relations and teamwork Difficulty: Moderate Quest. Category: Concept
424 richard@qwconsultancy.com
3) A primary characteristic of work groups is ________. A) the emphasis on generating positive synergy B) the goal of sharing information among members C) the mutual and team-based accountability for results D) the need to attain collective performance E) the availability of complementary skill sets Answer: B Explanation: A work group is a group that interacts primarily to share information and make decisions to help each member perform within his or her area of responsibility. LO: 10.1: Contrast groups and teams. AACSB: Interpersonal relations and teamwork Difficulty: Moderate Quest. Category: Concept 4) The primary purpose of a work group is to ________. A) generate positive synergy B) improve collective performance C) inculcate a climate of trust D) share relevant information E) enhance team efficacy Answer: D Explanation: A work group is a group that interacts primarily to share information and make decisions to help each member perform within his or her area of responsibility. LO: 10.1: Contrast groups and teams. AACSB: Interpersonal relations and teamwork Difficulty: Moderate Quest. Category: Concept 5) Which of the following statements is true regarding a work team? A) Work teams are rarely used in organizations today. B) Work teams are generally less flexible than traditional departments. C) Work teams generate positive synergy through coordinated effort. D) Work teams are less responsive to changes in the internal and external environment of the company. E) Work teams involve members of random and varied skill sets. Answer: C Explanation: Today most organizations use teams. Teams are more flexible and responsive to changing events than traditional departments or other forms of permanent groupings. A work team generates positive synergy through coordinated effort. LO: 10.1: Contrast groups and teams. AACSB: Interpersonal relations and teamwork Difficulty: Moderate Quest. Category: Concept
425 richard@qwconsultancy.com
6) Which of the following is a characteristic of work teams? A) The goal of sharing information B) The presence of neutral to negative synergy C) The availability of complementary skills among members D) The individual accountability for outcomes and results E) The mere accumulation of individual efforts Answer: C Explanation: A work team generates positive synergy through coordinated effort. The individual efforts result in a level of performance greater than the sum of those individual inputs. LO: 10.1: Contrast groups and teams. AACSB: Interpersonal relations and teamwork Difficulty: Moderate Quest. Category: Concept 7) Which of the following accurately differentiates between work groups and work teams? A) Work groups are used by top-management employees, while work teams are used by lowerlevel workers. B) Work groups are used for functions relating to areas of the external environment, while work teams are exclusively used for departmental problems and issues. C) Work groups involve members who have complementary skills, while work teams use employees who have random and varied skills. D) Work teams generate a potential for an organization to generate greater outputs with no increase in inputs, while work groups cannot perform this function. E) Work teams represent the mere accumulation of individual efforts, while work groups generate a positive synergy within the organization. Answer: D Explanation: A work team generates positive synergy through coordinated effort. The individual efforts result in a level of performance greater than the sum of those individual inputs. Work teams aim at the goal of collective performance, apply complementary skills, and promote individual and mutual accountability for group results. LO: 10.1: Contrast groups and teams. AACSB: Interpersonal relations and teamwork Difficulty: Moderate Quest. Category: Concept
426 richard@qwconsultancy.com
8) The extensive use of work teams benefits organizations by ________. A) generating neutral synergy among all employees of the team B) eliminating the need for mutual accountability for results C) developing the various team member's skills such that they are random and varied D) generating a potential for creating greater outputs without increasing inputs E) ensuring that the mere accumulation of individual efforts is used for group work Answer: D Explanation: Organizations are looking for positive synergy that will allow the organizations to increase performance. The extensive use of teams creates the potential for an organization to generate greater outputs with no increase in inputs. LO: 10.1: Contrast groups and teams. AACSB: Interpersonal relations and teamwork Difficulty: Moderate Quest. Category: Concept 9) ________ refers to the processes or actions involved in engaging as a team. A) Reflexivity B) Teaming C) Mental modeling D) Problem solving E) Team efficacy Answer: B Explanation: Teams are more likely to be constantly changing and adapting rather than static entities–seeing teams as dynamic systems in this way has led many to focus more on teaming as a verb (e.g., on the processes or actions involved in engaging as a team) rather than on the team itself. LO: 10.1: Contrast groups and teams. AACSB: Interpersonal relations and teamwork Difficulty: Moderate Quest. Category: Concept 10) Teams are less flexible and responsive to changing events than traditional departments or other formal groups. Answer: FALSE Explanation: Teams are more flexible and responsive to changing events than traditional departments or other formal groups. LO: 10.1: Contrast groups and teams. AACSB: Interpersonal relations and teamwork Difficulty: Moderate Quest. Category: Concept
427 richard@qwconsultancy.com
11) Our involvement in teams positively shapes the way we think as individuals, introducing a collaborative mindset about even our personal decision making. Answer: TRUE Explanation: Our involvement in teams positively shapes the way we think as individuals, introducing a collaborative mindset about even our personal decision making. LO: 10.1: Contrast groups and teams. AACSB: Interpersonal relations and teamwork Difficulty: Moderate Quest. Category: Concept 12) The goal of a work group is to share information. Answer: TRUE Explanation: A work group is a group that interacts primarily to share information and make decisions to help each member perform within his or her area of responsibility. LO: 10.1: Contrast groups and teams. AACSB: Interpersonal relations and teamwork Difficulty: Easy Quest. Category: Concept 13) Nancy is a part of a group at work that stresses collective goals and performance. Nancy is amazed at how well the members of the team use their skills to benefit themselves and their team members. Whenever a problem arises, the team, as a whole, works to provide reasons and solutions. She enjoys working in this group as there is a lot of positive energy. This group that Nancy is a part of is a work group. Answer: FALSE Explanation: Nancy is a part of a work team. A work team generates positive synergy through coordinated effort. The individual efforts result in a level of performance greater than the sum of those individual inputs. A workgroup interacts primarily to share information, to make decisions, and to help each group member perform within his area of responsibility. LO: 10.1: Contrast groups and teams. AACSB: Analytical thinking Difficulty: Moderate Quest. Category: Application 14) Compare and contrast work groups and work teams. Answer: A work group is a group that interacts primarily to share information and to make decisions to help each member perform within his or her area of responsibility. There is no positive synergy that would create an overall level of performance that is greater than the sum of the inputs. The members are individually held accountable for results, and they have random and varied skills. A work team, on the other hand, generates positive synergy through coordinated effort. Their individual efforts result in a level of performance that is greater than the sum of those individual inputs. They consist of members who have complementary skills and are mutually accountable for outcomes. LO: 10.1: Contrast groups and teams. AACSB: Analytical thinking Difficulty: Moderate 428 richard@qwconsultancy.com
Quest. Category: Synthesis 15) A team consists of employees from the same department who meet for a few hours each week to discuss ways of improving the work environment but they do not have the authority to unilaterally implement any of their suggestions. This is most likely to be a(n) ________ team. A) cross-functional B) virtual C) self-managed work D) problem-solving E) independent Answer: D Explanation: In a problem-solving team, members share ideas or suggest how work processes and methods can be improved; they rarely have the authority to unilaterally implement any of their suggestions. LO: 10.2: Contrast the five types of team arrangements. AACSB: Interpersonal relations and teamwork Difficulty: Moderate Quest. Category: Concept 16) Problem-solving teams can ________. A) implement the recommended changes to resolve a problem B) assume responsibility for the outcomes of a solution they implemented C) provide recommendations after a discussion amongst department members D) resolve problems of different departments in an organization simultaneously E) combine the expertise of employees across different divisions and organizational levels Answer: C Explanation: In a problem-solving team, members share ideas or suggest how work processes and methods can be improved; they rarely have the authority to unilaterally implement any of their suggestions. Problem-solving teams only make recommendations. LO: 10.2: Contrast the five types of team arrangements. AACSB: Interpersonal relations and teamwork Difficulty: Moderate Quest. Category: Concept
429 richard@qwconsultancy.com
17) The supervisors of the production division of one of the branches of Georgia Mills have been informed of some irregularities noticed in inventory of raw materials by some trusted subordinates. To get a clearer picture, the supervisors had a quick meeting with the subordinates who reported the matter over lunch. The subordinates recommended rotating the schedule of employees who account for the incoming and used up stock so as to pinpoint the defaulting employee. The supervisors agreed that this was a good strategy but admitted that the matter must be escalated to the production manager before such a change can be implemented. They followed up this meeting with subsequent sessions to evaluate how the change worked. The supervisors and subordinates form a(n) ________ team in this scenario. A) cross-functional B) independent C) problem-solving D) virtual E) self-managed Answer: C Explanation: In a problem-solving team, members share ideas or suggest how work processes and methods can be improved; they rarely have the authority to unilaterally implement any of their suggestions. LO: 10.2: Contrast the five types of team arrangements. AACSB: Analytical thinking Difficulty: Moderate Quest. Category: Application 18) Groups of employees who perform highly related or interdependent jobs and take on many of the responsibilities of their former supervisors are called ________ teams. A) independent B) self-managed work C) cross-functional D) problem-solving E) traditional Answer: B Explanation: Self-managed work teams are groups of employees who perform highly related or interdependent jobs and take on many of the responsibilities of their former supervisors. LO: 10.2: Contrast the five types of team arrangements. AACSB: Interpersonal relations and teamwork Difficulty: Easy Quest. Category: Concept
430 richard@qwconsultancy.com
19) Brigette is the director of marketing for a large cosmetics company. Brigette has created a team of twelve employees from publicity to work on time-sensitive projects for the fall launch. In addition to giving the team the responsibility of planning and scheduling their own work and making all functional decisions, Brigette has asked each team member to evaluate the performance of other team members. This is an example of a ________ team A) problem-solving B) self-managed work C) cross-functional D) virtual E) task-resolution Answer: B Explanation: Self-managed work teams are groups of employees who perform highly related or interdependent jobs and take on many of the responsibilities of their former supervisors. These tasks are planning, assigning tasks to members, making operating decisions, taking action on problems, and working with suppliers and customers. LO: 10.2: Contrast the five types of team arrangements. AACSB: Analytical thinking Difficulty: Moderate Quest. Category: Application 20) Which of the following statements is true regarding self-managed work teams? A) Self-managed work teams typically consist of forty to fifty employees. B) Self-managed work teams typically manage conflicts well. C) Members of self-managed work teams typically report lower job satisfaction. D) Self-managed work teams are typically composed of employees from different departments who work independent of each other. E) In the case of self-managed work teams, former supervisory positions take on decreased importance. Answer: E Explanation: Self-managed work teams are groups of employees who perform highly related or interdependent jobs and take on many of the responsibilities of their former supervisors. Former supervisory positions take on decreased importance and are sometimes even eliminated. Selfmanaged teams do not typically manage conflicts well. Members of this team report higher levels of job satisfaction than other individuals. LO: 10.2: Contrast the five types of team arrangements. AACSB: Interpersonal relations and teamwork Difficulty: Moderate Quest. Category: Concept
431 richard@qwconsultancy.com
21) A ________ team is more likely to be made up of employees from about the same hierarchical level but different work areas. A) problem-solving B) self-managed work C) cross-functional D) traditional E) departmental Answer: C Explanation: Cross-functional teams are teams made up of employees from about the same hierarchical level but different work areas who come together to accomplish a task. LO: 10.2: Contrast the five types of team arrangements. AACSB: Interpersonal relations and teamwork Difficulty: Moderate Quest. Category: Concept 22) To retain its edge in the organic health food market, Natura has established a high-priority team comprised of senior executives from the company's production, marketing, and research divisions. These employees work together closely to study consumer attitudes about organic health foods and come up with a closely monitored development and marketing strategy for new products. This ensures that each division is informed of the specific needs, timelines, and expected outcomes of the strategy. It also makes Natura a company that adapts to changes in market trends swiftly. The team Natura uses here is of the ________ type. A) problem-solving B) self-managed work C) cross-functional D) traditional E) departmental Answer: C Explanation: Cross-functional teams are teams made up of employees from about the same hierarchical level but different work areas who come together to accomplish a task. LO: 10.2: Contrast the five types of team arrangements. AACSB: Analytical thinking Difficulty: Moderate Quest. Category: Application
432 richard@qwconsultancy.com
23) GoAir, a low-cost airline, has created a team made up of employees from production, planning, quality control, tooling, design engineering, and information systems to automate the company's C-17 program. This team is most likely to be an example of a ________ team. A) command B) cross-functional C) self-managed work D) problem-solving E) task Answer: B Explanation: Cross-functional teams are teams made up of employees from about the same hierarchical level but different work areas who come together to accomplish a task. They are an effective means of allowing people from diverse areas to exchange information, develop new ideas, solve problems, and coordinate complex projects. LO: 10.2: Contrast the five types of team arrangements. AACSB: Analytical thinking Difficulty: Moderate Quest. Category: Application 24) Which of the following statements is true regarding a cross-functional team? A) A cross-functional team is made up of employees from different hierarchical levels. B) A cross-functional team is made up of employees from the same department. C) Cross-functional teams have decreased in popularity in the last few decades. D) In the case of a cross-functional team, the early stages of development are typically very short. E) Cross-functional teams are used for developing new ideas and coordinating complex projects. Answer: E Explanation: Cross-functional teams are made up of employees from about the same hierarchical level but different work areas. Today, cross-functional teams are widely used in several organizations. Cross-functional teams are an effective means of developing new ideas and coordinating complex projects. Their early stages of development are often long, as members learn to work with diversity and complexity. LO: 10.2: Contrast the five types of team arrangements. AACSB: Interpersonal relations and teamwork Difficulty: Moderate Quest. Category: Concept
433 richard@qwconsultancy.com
25) A ________ team allows for collaboration between team members who are physically dispersed. A) problem-solving B) self-managed work C) task D) virtual E) command Answer: D Explanation: Virtual teams use computer technology to unite physically dispersed members and achieve a common goal. LO: 10.2: Contrast the five types of team arrangements. Difficulty: Moderate Quest. Category: Concept 26) In contrast to teams that interact face-to-face, virtual teams are characterized by ________. A) low popularity among companies B) the use of technology to interact C) low sharing of unique information D) low need for supervision E) low need for publicizing throughout the organization Answer: B Explanation: Virtual teams use computer technology to unite physically dispersed members and achieve a common goal. LO: 10.2: Contrast the five types of team arrangements. AACSB: Interpersonal relations and teamwork Difficulty: Moderate Quest. Category: Concept 27) Which of the following is desirable when a crucial project is delegated to a virtual team? A) Encouraging less interaction among members so they completely focus on the project B) Avoiding publicity of the team's progress and success in the organization C) Promoting a sense of independence over group spirit to encourage competition D) Engaging in close monitoring of the work being done by the team throughout the project E) Discouraging the sharing of unique information as it benefits some employees unfairly Answer: D Explanation: For virtual teams to be effective, management should ensure that (1) trust is established among members, (2) team progress is monitored closely, and (3) the efforts and products of the team are publicized throughout the organization. LO: 10.2: Contrast the five types of team arrangements. AACSB: Interpersonal relations and teamwork Difficulty: Moderate Quest. Category: Concept
434 richard@qwconsultancy.com
28) Which of the following statements is not true regarding multiteam systems? A) They are used to offset the problems associated with adding more members to an existing team. B) They are collections of two or more independent teams that share a superordinate goal. C) They are effectively teams of teams. D) They are more successful when they have boundary spanners. E) They perform better when lines of communication are restricted. Answer: E Explanation: As tasks become more complex, teams are often made bigger. However, increases in team size are accompanied by higher coordination demands, creating a tipping point at which the addition of another member does more harm than good. To solve this problem, organizations are employing multiteam systems, collections of two or more interdependent teams that share a superordinate goal. In other words, multiteam systems are a "team of teams." Research shows that multiteam systems performed better when they had boundary spanners whose job was to coordinate with other members of the other sub teams. This reduced the need for some team member communication. Restricting lines of communication was helpful because it reduced coordination demands. LO: 10.2: Contrast the five types of team arrangements. AACSB: Interpersonal relations and teamwork Difficulty: Moderate Quest. Category: Concept 29) Problem-solving teams discuss a problem, generate potential solutions, and implement them successfully. Answer: FALSE Explanation: Problem-solving teams rarely have the authority to unilaterally implement any of their suggestions. LO: 10.2: Contrast the five types of team arrangements. AACSB: Interpersonal relations and teamwork Difficulty: Easy Quest. Category: Concept 30) In self-managed teams, former supervisory positions take on decreased importance. Answer: TRUE Explanation: Self-managed work teams are groups of employees who perform highly related or interdependent jobs and take on many of the responsibilities of their former supervisors. Supervisory positions take on decreased importance and are sometimes even eliminated. LO: 10.2: Contrast the five types of team arrangements. AACSB: Interpersonal relations and teamwork Difficulty: Moderate Quest. Category: Concept
435 richard@qwconsultancy.com
31) Cross-functional teams consist of employees from about the same hierarchical level, but from different work areas, who come together to accomplish a task. Answer: TRUE Explanation: Cross-functional teams consist of employees from about the same hierarchical level, but from different work areas, who come together to accomplish a task. LO: 10.2: Contrast the five types of team arrangements. AACSB: Interpersonal relations and teamwork Difficulty: Easy Quest. Category: Concept 32) Virtual teams should be managed differently than face-to-face teams in an office, partially because virtual team members may not interact along traditional hierarchical patterns. Answer: TRUE Explanation: Virtual teams should be managed differently than face-to-face teams in an office, partially because virtual team members may not interact along traditional hierarchical patterns. Because of the complexity of interactions, research indicates that shared leadership of virtual teams may significantly enhance team performance, although the concept is still in development. LO: 10.2: Contrast the five types of team arrangements. AACSB: Interpersonal relations and teamwork Difficulty: Moderate Quest. Category: Concept 33) What are self-managed work teams? Answer: Self-managed work teams are groups of employees who perform highly related or interdependent jobs and take on many of the responsibilities of their former supervisors. Typically, this includes planning and scheduling of work, assigning tasks to members, collective control over the pace of work, making operating decisions, taking action on problems, and working with suppliers and customers. Fully self-managed work teams even select their own members and have the members evaluate each other's performance. As a result, supervisory positions take on decreased importance and may even be eliminated. LO: 10.2: Contrast the five types of team arrangements. AACSB: Interpersonal relations and teamwork Difficulty: Moderate Quest. Category: Concept 34) Describe cross-functional teams. Answer: Cross-functional teams are made up of employees from about the same hierarchical level, but from different work areas, who come together to accomplish a task. This is an effective means for allowing people from diverse areas within an organization to exchange information, develop new ideas and solve problems, and coordinate complex projects. LO: 10.2: Contrast the five types of team arrangements. AACSB: Interpersonal relations and teamwork Difficulty: Moderate Quest. Category: Concept
436 richard@qwconsultancy.com
35) Describe the special challenges that virtual teams face and explain how management can help to ensure the success of virtual teams. Answer: Virtual teams may suffer from less social rapport and less direct interaction among members. They aren't able to duplicate the normal give-and-take of face-to-face discussion. Especially when members haven't personally met, virtual teams tend to be more task-oriented and exchange less social-emotional information. Not surprisingly, virtual team members report less satisfaction with the group interaction process than do face-to-face teams. For virtual teams to be effective, management should ensure that (a) trust is established among team members (research has shown that one inflammatory remark in a team member e-mail can severely undermine team trust); (b) team progress is monitored closely (so the team doesn't lose sight of its goals, and no team member "disappears"); and (c) the efforts and products of the virtual team are publicized throughout the organization (so the team does not become invisible). LO: 10.2: Contrast the five types of team arrangements. AACSB: Interpersonal relations and teamwork Difficulty: Moderate Quest. Category: Concept 36) ________ is not one of the key components of effective teams. A) A climate of trust B) Company reputation C) Adequate resources D) Member flexibility E) Leadership and structure Answer: B Explanation: A well-established company reputation is not one of the key components of effective teams. LO: 10.3: Identify the characteristics of effective teams. AACSB: Interpersonal relations and teamwork Difficulty: Easy Quest. Category: Concept 37) Which of the following is a component of an effective team in the contextual factors category of the team competitiveness model? A) Team efficacy B) Climate of trust C) Common purpose D) Social loafing E) Specific goals Answer: B Explanation: Climate of trust is a contextual factor that affects the effectiveness of teams. Team efficacy, social loafing, specific goals, and common purpose are process-related factors that affect the effectiveness of the team. LO: 10.3: Identify the characteristics of effective teams. AACSB: Interpersonal relations and teamwork Difficulty: Moderate Quest. Category: Concept 437 richard@qwconsultancy.com
38) The team effectiveness model classifies the key components of effective teams into three general categories: context, composition, and process. Which of the following variables belongs to the composition category? A) Adequate resources B) Conflict level C) Team efficacy D) Performance evaluation system E) Personality of team members Answer: E Explanation: The personality of team members is a key component that affects the effectiveness of teams and it belongs to the composition category. LO: 10.3: Identify the characteristics of effective teams. AACSB: Interpersonal relations and teamwork Difficulty: Moderate Quest. Category: Concept 39) ________ is a process variable that influences team effectiveness according to the team effectiveness model. A) Leadership and structure B) Team efficacy C) Member flexibility D) Performance evaluation and reward systems E) Abilities of members Answer: B Explanation: Process variables are events within the team that influence effectiveness. Team efficacy is a process variable that influences effectiveness of a team. LO: 10.3: Identify the characteristics of effective teams. AACSB: Interpersonal relations and teamwork Difficulty: Easy Quest. Category: Concept
438 richard@qwconsultancy.com
40) Jean Wills, a trainer with Leverage Inc., is infuriated because the conference hall that she had booked for her morning session with a new batch of trainees will be occupied by the HR team for the day. Jean feels that the company only superficially commits to training and that it is not a priority as this lack of facilities for training is a recurring phenomenon. Recently, she had asked the HR department to supply printed copies of her material for the orientation and training modules to the new trainees, and HR refused, saying the soft copies available on the systems would suffice. Which of the following contextual areas is the source of problem between the training department and the HR department in this scenario? A) Performance evaluation B) Reward systems C) Adequate resources D) Leadership E) Structure Answer: C Explanation: Teams are part of a larger organization system; every work team relies on resources outside the group to sustain it. A scarcity of resources directly reduces the ability of a team to perform its job effectively and achieve its goals. This support includes timely information, proper equipment, adequate staffing, encouragement, and administrative assistance. LO: 10.3: Identify the characteristics of effective teams. AACSB: Analytical thinking Difficulty: Moderate Employability Skills: Knowledge Application and Analysis Quest. Category: Application 41) Araceli is a team member in a large corporation. She never speaks in team meetings because she has seen members talk behind each other's backs after the meetings. Members are constantly monitoring the other members' work and looking for mistakes to point out in a meeting. According to the information provided, which contextual factor is lacking in Araceli's team? A) Adequate resources B) Climate of trust C) Team structure D) Performance evaluations E) Leadership Answer: B Explanation: Araceli doesn't contribute because she does not feel like the group has a climate of trust. Interpersonal trust facilitates cooperation, reduces the need to monitor each other's behavior, and bonds members around the belief that others on the team won't take advantage of them. LO: 10.3: Identify the characteristics of effective teams. AACSB: Analytical thinking Difficulty: Moderate Employability Skills: Knowledge Application and Analysis Quest. Category: Application
439 richard@qwconsultancy.com
42) In ________ leadership is important to coordinate the efforts of various teams in order to produce a desired outcome. A) technical teams B) self-managed work teams C) problem-solving teams D) management teams E) multiteam systems Answer: E Explanation: Leadership is especially important in multiteam systems. Here, leaders need to empower teams by delegating responsibility to them and then play the role of facilitator, making sure the teams work together rather than against one another. LO: 10.3: Identify the characteristics of effective teams. AACSB: Interpersonal relations and teamwork Difficulty: Moderate Quest. Category: Concept 43) ________ is not part of the process of the team effectiveness model. A) A common purpose B) Climate of trust C) Specific goals D) Social loafing E) Conflict levels Answer: B Explanation: All are processes that contribute to the team effectiveness model except a climate of trust, which is part of the context. LO: 10.3: Identify the characteristics of effective teams. AACSB: Interpersonal relations and teamwork Difficulty: Moderate Quest. Category: Concept 44) Which personality dimension is especially important to high team effectiveness? A) Introversion B) Type A C) Reflexivity D) Conscientiousness E) Perfectionism Answer: D Explanation: Some dimensions identified in the Big Five personality model are particularly relevant to team effectiveness. Conscientiousness is especially important to teams. Conscientious people are good at backing up other team members and sensing when their support is truly needed. LO: 10.3: Identify the characteristics of effective teams. AACSB: Interpersonal relations and teamwork Difficulty: Moderate Quest. Category: Concept
440 richard@qwconsultancy.com
441 richard@qwconsultancy.com
45) People scoring high on ________ are valuable in teams because they're good at backing up fellow team members and at sensing when their support is truly needed. A) conscientiousness B) positivity C) emotional stability D) agreeableness E) openness to experience Answer: A Explanation: Conscientious people are valuable in teams because they're good at backing up other team members, and they're also good at sensing when that support is truly needed. LO: 10.3: Identify the characteristics of effective teams. AACSB: Interpersonal relations and teamwork Difficulty: Moderate Quest. Category: Concept 46) Which of the following statements is true regarding team composition? A) A team's performance is merely the summation of its individual members' abilities. B) High-ability teams find it hard to adapt to changing situations. C) Agreeableness is the only personality dimension that aids the ability to work in groups. D) In successful teams, members should be selected to ensure all the various roles are filled. E) A high-ability team will function well irrespective of the deficits in the abilities of the leader. Answer: D Explanation: Teams have different needs, and members should be selected to ensure all the various roles are filled. Successful work teams have selected people to play all these roles based on their skills and preferences. On many teams, individuals will play multiple roles. LO: 10.3: Identify the characteristics of effective teams. AACSB: Interpersonal relations and teamwork Difficulty: Moderate Quest. Category: Concept 47) A team's socializer is responsible for which of the following activities? A) Initiating creative ideas B) Praising, supporting, and empathizing C) Offering insightful analysis of options D) Coordinating and integrating group tasks E) Providing directions and following up with implementation Answer: B Explanation: The socializer role in a team is responsible for praising, supporting, and empathizing. LO: 10.3: Identify the characteristics of effective teams. AACSB: Interpersonal relations and teamwork Difficulty: Moderate Quest. Category: Concept
442 richard@qwconsultancy.com
48) A team's problem-solver is responsible for ________. A) providing structure B) fighting external battles C) offering insightful analysis of options D) coordinating and integrating group tasks E) innovating and clarifying Answer: E Explanation: The problem-solver role in a team is responsible for innovating and clarifying. LO: 10.3: Identify the characteristics of effective teams. AACSB: Interpersonal relations and teamwork Difficulty: Moderate Quest. Category: Concept 49) ________ describes the degree to which members of a work unit share a common demographic attribute, such as age, sex, race, educational level, or length of service. A) Social conformity B) Organizational value C) Demography D) Diversity E) Organizational multiculturalism Answer: C Explanation: Demography refers to the degree to which members of a work unit share a common demographic attribute, such as age, sex, race, educational level, or length of service in an organization, and the impact of this attribute on turnover. LO: 10.3: Identify the characteristics of effective teams. Difficulty: Moderate Quest. Category: Concept 50) Demography suggests that attributes such as age or the date of joining should help the organization in ________. A) promoting diversity and multiculturalism B) reducing discrimination in the workplace C) determining appropriate compensation packages D) assessing the training needs at an occupational level E) predicting employee turnover in the company Answer: E Explanation: Demography refers to the degree to which members of a work unit share a common demographic attribute, such as age, sex, race, educational level, or length of service in an organization, and the impact of this attribute on turnover. LO: 10.3: Identify the characteristics of effective teams. Difficulty: Moderate Quest. Category: Concept
443 richard@qwconsultancy.com
51) According to the concept of organizational demography, if team members have dissimilar experiences, it will lead to ________. A) increased employee satisfaction B) decreased level of conflicts C) higher employee motivation D) higher employee turnover E) higher team efficacy Answer: D Explanation: The logic goes like this: Conflict will be greater among those with dissimilar experiences because communication is more difficult. Similarly, the losers of a conflict are more apt to leave voluntarily or be forced out. LO: 10.3: Identify the characteristics of effective teams. AACSB: Interpersonal relations and teamwork Difficulty: Hard Quest. Category: Concept 52) Which of the following is true with regard to the impact of diversity on team effectiveness? A) High levels of diversity among team members benefit the group from the stage of forming. B) Race and gender diversity bear the highest positive impact on team performance. C) Leadership has a negligible role to play in diverse teams because the members manage themselves autonomously. D) Elements of diversity are undesirable as they interfere with team processes in the long run. E) Cultural diversity serves as an asset for tasks that call for a variety of viewpoints once the team has worked out its differences. Answer: E Explanation: Cultural diversity does seem to be an asset for tasks that call for a variety of viewpoints. But culturally heterogeneous teams have more difficulty learning to work with each other and solving problems. The good news is that these difficulties seem to dissipate with time. LO: 10.3: Identify the characteristics of effective teams. AACSB: Diverse and multicultural work environments Difficulty: Moderate Quest. Category: Concept
444 richard@qwconsultancy.com
53) Which of the following statements is true regarding a culturally diverse team? A) Elements of diversity do not interfere with team processes. B) Culturally homogeneous teams have more difficulty learning to work with each other and solving problems than culturally heterogeneous teams. C) Teams dominated with high cultural status individuals improve performance for every member. D) Cultural diversity within a team is not advisable for those tasks that call for a variety of viewpoints. E) Culturally diverse teams are more affected by surface-level diversity than by deep-level diversity. Answer: C Explanation: Evidence indicates these elements of diversity interfere with team processes, at least in the short term. Cultural diversity does seem to be an asset for tasks that call for a variety of viewpoints. But culturally heterogeneous teams have more difficulty learning to work with each other and solving problems. three Researchers in the UK found that cultural status differences affected team performance whereby individuals in teams with more high cultural status members than low cultural status members realized improved performance for every member. LO: 10.3: Identify the characteristics of effective teams. AACSB: Diverse and multicultural work environments Difficulty: Moderate Quest. Category: Concept 54) When tasks ________, cultural diversity is an asset for teams. A) require a variety of viewpoints B) are simple and routine C) involve emotional labor D) are highly technical E) required for completing the work are highly independent Answer: A Explanation: Cultural diversity does seem to be an asset for tasks that call for a variety of viewpoints. However, in other circumstances, diversity seems to interfere with team processes in the short term. LO: 10.3: Identify the characteristics of effective teams. AACSB: Diverse and multicultural work environments Difficulty: Moderate Quest. Category: Concept
445 richard@qwconsultancy.com
55) Which of the following statements is true regarding the size of teams? A) The most effective teams have twelve to fifteen members. B) When teams have excess members, cohesiveness declines. C) As team size increases, social loafing decreases. D) When teams have excess members, mutual accountability increases. E) Members of large teams coordinate work better when pressed for time. Answer: B Explanation: Generally speaking, the most effective teams have five to nine members. When teams have excess members, cohesiveness and mutual accountability decline, social loafing increases, and more people communicate less. LO: 10.3: Identify the characteristics of effective teams. AACSB: Interpersonal relations and teamwork Difficulty: Moderate Quest. Category: Concept 56) Which of the following statements represents a strategy that is desirable for making effective teams? A) Combining people with extremely high and extremely low abilities helps make a balanced team. B) Individuals should ideally have one role in a team as multiple roles reduce their effectiveness and efficiency. C) Diversity should be avoided as diverse teams involve a lot of conflict in day-to-day functioning. D) If a natural work unit is larger, its size must be maintained for the nature of the job requires large teams. E) The preference of individual members to work as a part of groups must be considered when forming teams. Answer: E Explanation: Not every employee is a team player. When people who prefer to work alone are required to team up, there is a direct threat to the team's morale and to individual member satisfaction. When selecting team members, managers should consider individual preferences along with abilities, personalities, and skills. LO: 10.3: Identify the characteristics of effective teams. AACSB: Interpersonal relations and teamwork Difficulty: Moderate Quest. Category: Concept
446 richard@qwconsultancy.com
57) Which of the following statements is true with regard to team processes? A) This category comprises variables that relate to how teams should be staffed. B) Effective teams invest time and effort to discuss and agree on a purpose that belongs to them both collectively and individually. C) Once a specific goal is chosen, effective teams resist making any changes in them so as to maintain the momentum of the team. D) Effective teams do not engage in task and relationship conflicts. E) Effective teams begin by selecting the right kind of people to be members of the team. Answer: B Explanation: Effective teams begin by analyzing the team's mission, developing goals to achieve that mission, and creating strategies for achieving the goals. Teams that consistently perform better have established a clear sense of what needs to be done and how. Members of successful teams put a tremendous amount of time and effort into discussing, shaping, and agreeing on a purpose that belongs to them both collectively and individually. LO: 10.3: Identify the characteristics of effective teams. AACSB: Interpersonal relations and teamwork Difficulty: Moderate Quest. Category: Concept 58) Which of the following illustrates a process loss from using teams? A) Organizational demography B) Role conflict C) Social loafing D) Multitasking E) Profit-sharing Answer: C Explanation: In the case of a team, when each member's contribution is not clearly visible, individuals tend to decrease their effort. This is known as social loafing. Social loafing illustrates a process loss from using teams. LO: 10.3: Identify the characteristics of effective teams. AACSB: Interpersonal relations and teamwork Difficulty: Moderate Quest. Category: Concept
447 richard@qwconsultancy.com
59) Gerald Murphy is a manager at Wright & Wayner, a publishing house which is a very employee-friendly company. The demarcation between managers and subordinates is flexible and the nature of these reporting relationships is flexible and informal. Gerald recently assigned five of eight of his subordinates to work on a new publication project with a fairly tight deadline because he believed that these efficient employees would function even better with each other's support. A week before the deadline, however, Gerald realized that the group had been shirking work as each of the members thought that the project was not just his or her responsibility. Additionally, under the guise of working together, the team was actually doing their personal work, surfing the Internet, or just socializing. This scenario reflects ________. A) gainsharing B) high process gains C) reflexive goal-setting D) job sharing E) social loafing Answer: E Explanation: In the case of a team, when each member's contribution is not clearly visible, individuals tend to decrease their effort. This is known as social loafing. Social loafing illustrates a process loss from using teams. LO: 10.3: Identify the characteristics of effective teams. AACSB: Analytical thinking Difficulty: Moderate Employability Skills: Knowledge Application and Analysis Quest. Category: Application 60) A team that has a good plan but is also willing to adjust its master plan and adapt when conditions call for it demonstrates high ________, an important variable in successful team processes. A) goal specificity B) goal congruency C) conformity D) diversity E) reflexivity Answer: E Explanation: Reflexivity is the team characteristic of reflecting on and adjusting the master plan when necessary. It is considered part of the process variables. LO: 10.3: Identify the characteristics of effective teams. AACSB: Interpersonal relations and teamwork Difficulty: Easy Quest. Category: Concept
448 richard@qwconsultancy.com
61) While discussing their marketing campaign for a new product, the members of the crossfunctional team responsible for Carver Inc. realized that a couple of changes relating to their prior plan would be beneficial. The offer of a franchising that had earlier been brushed off by the company head was discussed thoroughly, and it was decided that it would be implemented on a trial basis initially and on full scale if found to work well. From the information provided, it can be concluded that this cross-functional team has a high degree of ________. A) demography B) reflexivity C) conformity D) diversity E) uncertainty Answer: B Explanation: Reflexivity is the team characteristic of reflecting on and adjusting the master plan when necessary. It is considered part of the process variables. LO: 10.3: Identify the characteristics of effective teams. AACSB: Analytical thinking Difficulty: Moderate Employability Skills: Knowledge Application and Analysis Quest. Category: Application 62) Effective teams translate their common purpose into ________ goals. A) universal B) generic C) vast D) measurable E) diverse Answer: D Explanation: Successful teams translate their common purpose into specific, measurable, and realistic performance goals. Specific goals facilitate clear communication. They also help teams maintain their focus on getting results. LO: 10.3: Identify the characteristics of effective teams. AACSB: Interpersonal relations and teamwork Difficulty: Moderate Quest. Category: Concept
449 richard@qwconsultancy.com
63) ________ refers to the belief that effective teams have in their ability to succeed. A) Goal specificity B) Reflexivity C) Mental efficiency D) Common purpose E) Team efficacy Answer: E Explanation: Effective teams have confidence in themselves; they believe they can succeed. This is called team efficacy. LO: 10.3: Identify the characteristics of effective teams. AACSB: Interpersonal relations and teamwork Difficulty: Easy Quest. Category: Concept 64) Luis is leading his city's efforts to install electric vehicle charging systems. As part of the project Luis and his team will be meeting with representatives from major auto companies and scientists working in electric car battery storage development. Before he begins on this intensive project, Luis wants to improve the team efficacy of his team. Which of the following actions can Luis take to increase team efficacy before the project starts? A) Plan a team outing for the team members B) Make sure that his team is not comprised of more than nine people C) Spend considerable time evaluating team members and allocating proper roles for each D) Provide training to improve members' technical and interpersonal skills E) Create team goals that are very challenging Answer: D Explanation: Efficacy means that a team believes it can succeed in the job and goal that is placed before them. There are two ways of increasing efficacy: 1) provide training to improve skills and increase confidence, and 2) help the team achieve small, incremental successes. LO: 10.3: Identify the characteristics of effective teams. AACSB: Analytical thinking Difficulty: Moderate Employability Skills: Knowledge Application and Analysis Quest. Category: Application
450 richard@qwconsultancy.com
65) Which of the following is most likely to increase team efficacy? A) Providing vast and generic goals B) Helping the team achieve small successes C) Creating a team such that it has diverse members D) Ensuring that team goals are substantially difficult E) Reducing the number of members on a team drastically Answer: B Explanation: Efficacy means that a team believes it can succeed in the job and goal that is placed before them. There are two ways to increasing efficacy: 1) provide training to improve skills and increase confidence, and 2) help the team achieve small, incremental successes. LO: 10.3: Identify the characteristics of effective teams. AACSB: Interpersonal relations and teamwork Difficulty: Moderate Quest. Category: Concept 66) Mental representations of the key elements within a team's environment that team members share are called ________. A) path-goal models B) task models C) mental models D) individual models E) specific models Answer: C Explanation: Effective teams share accurate mental models which are organized mental representations of the key elements within a team's environment that team members share. LO: 10.3: Identify the characteristics of effective teams. AACSB: Interpersonal relations and teamwork Difficulty: Moderate Quest. Category: Concept
451 richard@qwconsultancy.com
67) With a deadline approaching, all seven members of Sharon's product development team were working round-the-clock and still the work was not completed in time. After the project was completed, Sharon spoke individually with the members to determine the cause for this delay. Many members complained, saying the work given to them was not in accordance with their roles. Some were unclear about which team member to approach when faced with a problem, and many underestimated the time and effort the project demanded. This team is characterized by ________. A) a clear role allocation for all team members B) dissimilar mental models C) high degree of role clarity D) excessively large team size E) a strong climate of trust Answer: B Explanation: Effective teams share accurate mental models which are organized mental representations of the key elements within a team's environment that team members share. If team members have the wrong mental models, which is particularly likely with teams under acute stress, their performance suffers. LO: 10.3: Identify the characteristics of effective teams. AACSB: Analytical thinking Difficulty: Easy Employability Skills: Knowledge Application and Analysis Quest. Category: Application 68) Which of the following is not a common characteristic of an effective team? A) The team has adequate resources. B) The team has effective leadership. C) The team has a climate of trust. D) The team has a performance and evaluation system that reflects team contributions. E) The team is the right size for the task—about 15-20 people. Answer: E Explanation: Effective teams have common characteristics. They have adequate resources, effective leadership, a climate of trust, and a performance evaluation and reward system that reflects team contributions. Effective teams also tend to be small—with fewer than 10 people, preferably of diverse backgrounds. LO: 10.3: Identify the characteristics of effective teams. AACSB: Interpersonal relations and teamwork Difficulty: Moderate Quest. Category: Concept
452 richard@qwconsultancy.com
69) When teams are performing nonroutine activities, ________ stimulate discussion, promote critical assessment of problems and options, and can lead to better team decisions. A) task conflicts B) disciplinary conflicts C) relationship conflicts D) resource allocation conflicts E) discriminatory conflicts Answer: A Explanation: When teams are performing nonroutine activities, disagreements about task content stimulate discussion, promote critical assessment of problems and options, and can lead to better team decisions. Such conflicts are called task conflicts. LO: 10.3: Identify the characteristics of effective teams. AACSB: Interpersonal relations and teamwork Difficulty: Moderate Quest. Category: Concept 70) Which of the following is an example of a task conflict? A) Will and Hilda have been removed from the team they worked with after they were overheard making derogatory comments about one of their colleague's racial origin. B) Henry and Solomon have been reprimanded by their project lead for spending too much time using the Internet for personal use at work. C) Linda and Dorothy had a disagreement over which of their employees should be assigned to work on a high-priority project. D) Sally and her manager have just had a heated argument because Sally feels she has been overlooked for a promotion that was her rightful due. E) The company head has resigned after longstanding conflict between him and his top management employees. Answer: C Explanation: When teams are performing nonroutine activities, disagreements about task content stimulate discussion, promote critical assessment of problems and options, and can lead to better team decisions. Such conflicts are called task conflicts. LO: 10.3: Identify the characteristics of effective teams. AACSB: Analytical thinking Difficulty: Hard Employability Skills: Knowledge Application and Analysis Quest. Category: Application
453 richard@qwconsultancy.com
71) If a manager wants to create and maintain an effective team, the manager should avoid ________. A) task conflicts B) reflexivity C) specific goals D) relationship conflicts E) congruent mental models Answer: D Explanation: Relationship conflicts are those based on interpersonal incompatibilities, tension, and animosity toward others; these are almost always dysfunctional. However, when teams are performing nonroutine activities, disagreements about task content (called task conflicts) stimulate discussion, promote critical assessment of problems and options, and can lead to better team decisions. LO: 10.3: Identify the characteristics of effective teams. AACSB: Interpersonal relations and teamwork Difficulty: Moderate Quest. Category: Concept 72) When individuals engage in ________, they coast on the group's effort because their particular contributions cannot be identified. A) task conflicts B) gainsharing C) retaliating D) process gains E) social loafing Answer: E Explanation: When individuals engage in social loafing, they coast on the group's effort because their particular contributions cannot be identified. Effective teams undermine this tendency by making members individually and jointly accountable for the team's purpose, goals, and approach. LO: 10.3: Identify the characteristics of effective teams. AACSB: Interpersonal relations and teamwork Difficulty: Moderate Quest. Category: Concept
454 richard@qwconsultancy.com
73) Shane Dermott is a manager at a software design firm that develops software programs for clients using multiple teams of specialists. One of his most successful teams has been performing very well for years and so it came as a surprise to Shane when he noticed a sharp drop in the team's performance. He decided to speak with the team members individually and find a solution. After interviewing all 7 team members, he determined that one of the team members performed routine administrative tasks that were hardly visible to any of the other team members and understood that this was a case of social loafing which demotivated the entire group. Which of the following would most likely improve the performance of the employee who was social loafing? A) Encouraging him to work more independently B) Making him individually responsible for definite and crucial tasks C) Empowering him to make decisions autonomously D) Providing him with administrative training E) Giving him additional assignments to complete Answer: B Explanation: This team member is engaging in social loafing. Effective teams undermine this tendency by making members individually and jointly accountable for the team's purpose, goals, and approach. Therefore, members should be clear on what they are individually responsible for and what they are jointly responsible for on the team. LO: 10.3: Identify the characteristics of effective teams. AACSB: Analytical thinking Difficulty: Moderate Employability Skills: Knowledge Application and Analysis Quest. Category: Application
455 richard@qwconsultancy.com
74) Jason Jones has been asked to assemble an eight-member self-managed work team of experienced employees to work on a project that combines the functional areas of research, production, marketing, and distribution. Jason is apprehensive about managing a team so large; he knows from prior experience that larger teams do not always result in greater productivity. Jason has also been informed by the management that this team is being established as the project must progress according to the predetermined timeline. The work lagging behind by one division will cause a cumulative delay in all the successive stages of the project's execution. Which of the following measures, if adopted by Jason, would best ensure that the team members are working efficiently? A) Implementing a training program to provide the team members with the necessary technical and human relations skills for the project B) Discouraging team members from restricting themselves to their own functional areas by experimenting with and learning the jobs done by other members of the team C) Instructing the team members to finalize the plan for the project right at the beginning and ensuring that it is implemented without any changes D) Assigning the specific tasks of the project to each team member by keeping in mind their preferences and abilities E) Implementing a group-based incentive he has planned for this team Answer: D Explanation: Implementing a training program for experienced employees is not directly going to help ensure efficiency. Discouraging team members from restricting themselves to their own functional areas is not going to contribute to team efficiency directly; it may reduce efficiency in actuality. Instructing the team members to finalize the plan for the project right at the beginning and ensuring that it is implemented without any changes may actually hamper team efficiency because of the lack of reflexivity. Assigning the specific tasks of the project to each team member by keeping in mind their preferences and abilities is likely to ensure that team members are working efficiently. Implementing a group-based incentive for this team is likely to provide greater opportunity for some employees of the group to coast on the group's effort because of the lack of individual roles and accountability. LO: 10.3: Identify the characteristics of effective teams. AACSB: Analytical thinking Difficulty: Hard Employability Skills: Knowledge Application and Analysis Quest. Category: Critical Thinking 75) A climate of trust among team members is a process and state variable relating to team effectiveness. Answer: TRUE Explanation: Common purpose, motivation, team efficacy, team identity, team cohesion, mental models, conflict, social loafing, and trust are all processes and states contributing to team effectiveness. LO: 10.3: Identify the characteristics of effective teams. AACSB: Interpersonal relations and teamwork Difficulty: Moderate Quest. Category: Concept
456 richard@qwconsultancy.com
457 richard@qwconsultancy.com
76) FDS is a government agency which holds its yearly appraisals in December and provides employees with merit-based incentives based on their individual performance ratings. Merit is the only criterion for incentives at FDS and this is likely to have a strong positive impact on group performance. Answer: FALSE Explanation: Individual performance evaluations and incentives may interfere with the development of high-performance teams. So, in addition to evaluating and rewarding employees for their individual contributions, performance rewards must recognize and promote group performance through incentive programs like gainsharing. LO: 10.3: Identify the characteristics of effective teams. AACSB: Analytical thinking Difficulty: Moderate Quest. Category: Application 77) Leadership of multiteam systems is also much different than for standalone teams. While leadership of all teams affects team performance, a multiteam leader must both facilitate coordination between teams and lead each team. Answer: TRUE Explanation: Leadership of multiteam systems is also much different than for standalone teams. While leadership of all teams affects team performance, a multiteam leader must both facilitate coordination between teams and lead each team. Research indicated that teams receiving more attention and engagement increased team empowerment, making them more effective as they sought to solve their own problems. LO: 10.3: Identify the characteristics of effective teams. AACSB: Interpersonal relations and teamwork Difficulty: Moderate Quest. Category: Concept 78) Conscientiousness and openness to experience are two personality dimensions of the Big Five personality model that predict better performance in teams. Answer: TRUE Explanation: Many of the dimensions identified in the Big Five personality model are also relevant to team effectiveness; a review of the literature identified three. Specifically, teams that rate higher on mean levels of conscientiousness and openness to experience tend to perform better. LO: 10.3: Identify the characteristics of effective teams. AACSB: Interpersonal relations and teamwork Difficulty: Moderate Quest. Category: Concept
458 richard@qwconsultancy.com
79) Effective teams develop transactive memory systems which represent the ways in which team members collect, generate, and distribute knowledge to develop a shared understanding of their environment. Answer: TRUE Explanation: Effective teams develop transactive memory systems which represent the ways in which team members collect, generate, and distribute knowledge to develop a shared understanding of their environment. LO: 10.3: Identify the characteristics of effective teams. AACSB: Interpersonal relations and teamwork Difficulty: Moderate Quest. Category: Concept 80) Individuals who normally function in action teams are unlikely to share mental models. Answer: FALSE Explanation: Individuals who normally function in action teams – teams with specialists engaged in intense, interdependent, and unpredictable tasks – are likely to share mental models. LO: 10.3: Identify the characteristics of effective teams. AACSB: Interpersonal relations and teamwork Difficulty: Moderate Quest. Category: Concept 81) Mental model refers to the degree to which a team reflects on and adjusts its master plan when necessary. Answer: FALSE Explanation: Effective teams show reflexivity, meaning they reflect on and adjust their master plan when necessary. Mental model refers to team members' shared knowledge about the key elements within their task environment. LO: 10.3: Identify the characteristics of effective teams. AACSB: Interpersonal relations and teamwork Difficulty: Moderate Quest. Category: Concept 82) Describe the three categories of key components for effective teams as shown in the team effectiveness model and give an example of each category. Answer: The key components of effective teams can be organized into three general categories. First are the resources and other contextual influences that make teams effective; an example would be adequate resources. The second relates to the team's composition; an example would be personality of team members. Finally, process variables are events within the team that influence effectiveness; an example of this would be team efficacy. LO: 10.3: Identify the characteristics of effective teams. AACSB: Interpersonal relations and teamwork Difficulty: Moderate Quest. Category: Concept
459 richard@qwconsultancy.com
83) Explain how a climate of trust improves team effectiveness. Answer: Interpersonal trust among team members facilitates cooperation, reduces the need to monitor each other's behavior, and bonds members around the belief that others on the team won't take advantage of them. Team members are more likely to take risks and expose vulnerabilities when they believe they can trust others on their team. Trust is the foundation of leadership. It allows a team to accept and commit to its leader's goals and decisions. LO: 10.3: Identify the characteristics of effective teams. AACSB: Interpersonal relations and teamwork Difficulty: Moderate Quest. Category: Concept 84) Explain the leader's role in (a) a self-managed team, (b) a multiteam, and (c) a diverse team. Answer: a) Self-managed work teams are groups of employees who perform highly related or interdependent jobs and take on many of the responsibilities of their former supervisors. Supervisory positions take on decreased importance and are sometimes even eliminated. While self-managed teams do absorb many of the duties typically assumed by managers, the manager's job then involves coordinating factors outside the group. b) Leadership is especially important in multiteam systems, in which different teams coordinate their efforts to produce a desired outcome. Here, leaders need to empower teams by delegating responsibility to them, and they play the role of facilitator, making sure the teams work together rather than against one another. c) Proper leadership can also improve the performance of diverse teams. When leaders provide an inspirational common goal for members with varying types of education and knowledge, teams are very creative. When leaders don't provide such goals, diverse teams fail to take advantage of their unique skills and are actually less creative than teams with homogeneous skills. Even teams with diverse values can perform effectively, however, if leaders provide a focus on work tasks rather than leading based on personal relationships. LO: 10.3: Identify the characteristics of effective teams. AACSB: Analytical thinking Difficulty: Moderate Quest. Category: Synthesis 85) How must roles be allocated to ensure effective team performance? Answer: Teams have different needs, and members should be selected to ensure all the various roles are filled. Teams with more experienced and skilled members perform better. However, the experience and skill of those in core roles who handle more of the workflow of the team and who are central to all work processes are especially vital. Nine potential team roles have been identified, and successful work teams have selected people to play all these roles based on their skills and preferences. To increase the likelihood the team members will work well together, managers need to understand the individual strengths each person can bring to a team, select members with their strengths in mind, and allocate work assignments that fit with members' preferred styles. LO: 10.3: Identify the characteristics of effective teams. AACSB: Interpersonal relations and teamwork Difficulty: Moderate Quest. Category: Concept 460 richard@qwconsultancy.com
86) Define organizational demography and explain its significance. Answer: The degree to which members of a work unit (group, team, or department) share a common demographic attribute, such as age, sex, race, educational level, or length of service in the organization, is the subject of organizational demography. Organizational demography suggests that attributes such as age or the date of joining should help us predict turnover. The logic goes like this: turnover will be greater among those with dissimilar experiences because communication is more difficult and conflict is more likely. Increased conflict makes membership less attractive, so employees are more likely to quit. LO: 10.3: Identify the characteristics of effective teams. AACSB: Interpersonal relations and teamwork Difficulty: Moderate Quest. Category: Concept 87) What are the implications of team size on team effectiveness? Answer: Generally speaking, the most effective teams have five to nine members. And experts suggest using the smallest number of people who can do the task. Unfortunately, managers often err by making teams too large. It may require only four or five members to develop diversity of views and skills, while coordination problems can increase exponentially as team members are added. When teams have excess members, cohesiveness and mutual accountability decline, social loafing increases, and more people communicate less. Members of large teams have trouble coordinating with one another, especially under time pressure. Teams should ideally consist of nine or fewer members. LO: 10.3: Identify the characteristics of effective teams. AACSB: Interpersonal relations and teamwork Difficulty: Moderate Quest. Category: Concept 88) What is meant by member preferences? How can it be altered? Answer: All employees do not enjoy working in teams and, given the option, many employees will select themselves out of team participation. When people who prefer to work alone are required to team up, there is a direct threat to the team's morale and to individual member satisfaction. This result suggests that when selecting team members, managers should consider individual preferences along with abilities, personalities, and skills. High-performing teams are likely to be composed of people who prefer working as part of a group. However, there are ways to encourage people who prefer working alone to engage in teamwork. Training specialists conduct exercises that allow employees to experience the satisfaction teamwork can provide. Workshops help employees improve their problem-solving, communication, negotiation, conflict-management, and coaching skills. Further, an organization's reward system must be reworked to encourage cooperative efforts rather than competitive ones. Lastly, demonstrating to these employees the intrinsic rewards, such as camaraderie, that employees can receive from teamwork may be helpful. LO: 10.3: Identify the characteristics of effective teams. AACSB: Analytical thinking Difficulty: Moderate Quest. Category: Synthesis 461 richard@qwconsultancy.com
462 richard@qwconsultancy.com
89) How do mental models contribute to task conflicts? Answer: Effective teams share accurate mental models—organized representations of key elements within a team's environment that team members share. If team members have the wrong mental models, which is particularly likely in teams under acute stress, their performance suffers. The similarity of team members' mental models matters, too. If team members have different ideas about how to do things, the team will fight over methods rather than focus on what needs to be done. LO: 10.3: Identify the characteristics of effective teams. AACSB: Analytical thinking Difficulty: Moderate Quest. Category: Synthesis 90) What is the relationship between task conflicts and reflexivity among team members? Answer: When teams are performing nonroutine activities, disagreements about task content can stimulate discussion, promote critical assessment of problems and options, and can lead to better team decisions. Such conflicts are defined as task conflicts. Research has shown that moderate levels of task conflict during the initial phases of team performance were positively related to team creativity, but both very low and very high levels of task conflict were negatively related to team performance. Effective teams also show reflexivity, meaning they reflect on and adjust their master plan when necessary. A team has to have a good plan, but it also has to be willing and able to adapt when conditions call for it. Research evidence does suggest that teams high in reflexivity are better able to adapt to conflicting plans and goals among team members. Both these concepts come under the process dimension of key variables of effective teams. Reflexivity may be understood as one way to solve task conflicts by adapting the predetermined plan to current changes. LO: 10.3: Identify the characteristics of effective teams. AACSB: Analytical thinking Difficulty: Moderate Quest. Category: Synthesis 91) Define social loafing and explain how it can be thwarted by managers. Answer: Individuals can engage in social loafing and coast on the group's effort because their particular contributions can't be identified. Effective teams undermine this tendency by making members individually and jointly accountable for the team's purpose, goals, and approach. Therefore, members should be clear on what they are individually responsible for and what they are jointly responsible for on the team. LO: 10.3: Identify the characteristics of effective teams. AACSB: Interpersonal relations and teamwork Difficulty: Moderate Quest. Category: Concept
463 richard@qwconsultancy.com
92) Teams that are high in ________ tend to be better able to adapt to conflicting plans and goals among team members. A) efficacy B) conformity C) identity D) reflexivity E) social loafing Answer: D Explanation: Effective teams show reflexivity, meaning they reflect on and adjust their purpose when necessary. A team must have a good plan, but it needs to be willing and able to adapt when conditions call for it. Interestingly, some evidence suggests that teams high in reflexivity are better able to adapt to conflicting plans and goals among team members. LO: 10.4: Explain how organizations can create effective teams. AACSB: Analytical thinking Difficulty: Easy Quest. Category: Application 93) To foster positive team identity, managers should ________. A) create a climate of respect and inclusion B) avoid the use of virtual teams C) ignore individuals' specific skills and abilities D) choose the individual with the best technical skills regardless of his or her ability to perform as a member of a team E) use only team-based motivational tools Answer: A Explanation: By recognizing individuals' specific skills and abilities, as well as creating a climate of respect and inclusion, leaders and members can foster positive team identity and improved team outcomes. LO: 10.4: Explain how organizations can create effective teams. AACSB: Interpersonal relations and teamwork Difficulty: Moderate Quest. Category: Concept
464 richard@qwconsultancy.com
94) Jason Jones has been asked to assemble an eight-member self-managed work team of experienced employees to work on a project that combines the functional areas of research, production, marketing, and distribution. Jason is apprehensive about managing a team so large; he knows from prior experience that larger teams do not always result in greater productivity. Jason has also been informed by the management that this team is being established as the project must progress according to the predetermined timeline. The work lagging behind by one division will cause a cumulative delay in all the successive stages of the project's execution. Jason plans to implement a group-based incentive that he feels would motivate each of the team members to work efficiently. Which of the following is an assumption made by Jason in implementing this plan? A) Increasing the number of experienced employees in the team will reduce the need for supervision. B) Each employee in the team is motivated to work equally hard to ensure that the group's performance is satisfactory. C) Since the team is comprised of employees who are highly skilled in their respective areas, the output of the group will be of high quality. D) Each employee will work well within the boundaries of their divisional functions, and conflicts will be relatively fewer. E) Each employee will be committed to working within the resources allocated. Answer: B Explanation: The assumption that increasing the number of experienced employees in the team will reduce the need for supervision does not have an impact on Jason's decision to implement a group-based incentive. The assumption that each employee in the team is motivated to work equally hard to ensure that the group's performance is satisfactory is implicit in Jason's belief that this group-based incentive plan will work. The assumption that since the team is comprised of employees who are highly skilled in their respective areas, the output of the group will be of high quality is not implicit in Jason's decision to implement a group-based incentive for the team. The assumption that each employee will work well within the boundaries of their divisional functions and conflicts will be relatively fewer is not implicit in Jason's decision to implement a groupbased incentive for the team. The assumption that each employee will be committed to working within the resources allocated is not implicit in Jason's decision to implement a group-based incentive for the team. LO: 10.4: Explain how organizations can create effective teams. AACSB: Reflective thinking Difficulty: Hard Employability Skills: Knowledge Application and Analysis Quest. Category: Critical Thinking
465 richard@qwconsultancy.com
95) Which is not an example of an effective reward for exceptional teamwork? A) A cash prize for the top three producers on the team. B) A full day team tour of the local vineyards and wineries. C) A happy hour for the team. D) A team dinner and movie night. E) A cash bonus of $1000 per team member. Answer: A Explanation: A traditional organization's reward system must be reworked to encourage cooperative efforts rather than competitive ones. Rewarding only the top three producers on a team encourages individual rather than team performance. LO: 10.4: Explain how organizations can create effective teams. AACSB: Interpersonal relations and teamwork Difficulty: Moderate Quest. Category: Concept
466 richard@qwconsultancy.com
96) Dina Delapaz, owner of the famous Mint chain of restaurants, planned to revamp the whole concept and structure of her outlets. With this purpose in mind, she selected five managers from her existing outlets based on their performance in relation to targets, experience, and motivation. Each manager was asked to take up an aspect of the restaurant's functioning like menu, décor, and customer service measures. Dina was sure that this group of high performers possessed a clear understanding of the ground realities, which made them the best candidates to contemplate and implement the required changes. In order to reward them, she announced a hefty bonus for their collective performance. Six months later, after the changes were implemented, Dina realized that the group had made many questionable decisions, which were resulting in problems. Which of the following, if true, would best explain this outcome? A) The team members had skills that were complementary to each other and added value to the project. B) The team members had high levels of affective commitment to their jobs as Dina was a good leader. C) Typically, these managers viewed each other as competition as Dina only rewarded the highest earning outlet with incentives. D) Two of the branches of Mint were located in the suburbs, while the other three outlets were located in busy areas and witnessed more customers. E) The team members were clear about their duties, and role ambiguity was minimal. Answer: C Explanation: The fact that the team members had skills that were complementary to each other and added value to the project would not explain the problems encountered by Dina. The fact that the team members had high levels of affective commitment to their jobs as Dina was a good leader would not explain the poor implementation of the changes Dina sought. The fact that these managers typically viewed each other as competition as Dina only rewarded the highest earning outlet with incentives would explain the failure of this project. This is because the low trust typical of the competitive group is not readily replaced by high trust with a quick change in reward systems. The fact that two branches of Mint were located in suburbs, while the others were located in busy areas, does not explain the failure of Dina's move to revamp her restaurant chain. The fact that the team members were clear about their duties and role ambiguity was minimal does not explain their failure. LO: 10.4: Explain how organizations can create effective teams. AACSB: Analytical thinking Difficulty: Hard Quest. Category: Critical Thinking
467 richard@qwconsultancy.com
97) A software development firm has witnessed substantial growth and seeks to expand and reorganize its structure to meet the demands from clients in a better manner. HR has been given a clear directive that one of the major criteria for selection of recruits would be the candidate's ability to function as a team player. This is because the company is revamping all its major functions from the traditional departmental model to an arrangement of flexible teams. Joe Sanchez, one of the firm's oldest and most experienced employees, has demonstrated considerable ability in developing the best programs, but he prefers to work alone. The company does not wish to lose resources like Sanchez and others who lack the ability to work in groups, but teamwork is essential to its reorganization. Which of the following strategies will help rectify this problem? A) Encouraging the formation of more diverse teams B) Informing all employees that ability to work in teams will be a major determinant for promotions and bonuses C) Increasing the size of teams so the contribution of people like Joe will not affect team performance overall D) Applying merit-based compensation systems instead of gainsharing E) Encouraging employees like Joe to work more independently Answer: B Explanation: Promotions, pay raises, and other forms of recognition should be given to individuals who work effectively as team members by training new colleagues, sharing information, helping resolve team conflicts, and mastering needed new skills. LO: 10.4: Explain how organizations can create effective teams. AACSB: Analytical thinking Difficulty: Moderate Quest. Category: Application 98) Cluster hiring refers to the selection of an already-existing team to work in a new role. Answer: TRUE Explanation: Team cohesion is higher in teams with female team leaders when teams are larger and more financially diverse. LO: 10.4: Explain how organizations can create effective teams. AACSB: Interpersonal relations and teamwork Difficulty: Moderate Quest. Category: Concept 99) For creating good team players, an organization's reward system must be reworked to encourage competitive efforts rather than cooperative ones. Answer: FALSE Explanation: For creating good team players, an organization's reward system must be reworked to encourage cooperative efforts rather than competitive ones. LO: 10.4: Explain how organizations can create effective teams. AACSB: Interpersonal relations and teamwork Difficulty: Moderate Quest. Category: Concept
468 richard@qwconsultancy.com
100) Problems such as lack of trust or lack of sharing information among team members in a competitive group can be swiftly changed by rewarding collective performance over individual performance. Answer: FALSE Explanation: Teams that switch from a competitive to a cooperative system do not immediately share information, and they still tend to make rushed, poor-quality decisions. Apparently, the low trust typical of the competitive group will not be readily replaced by high trust with a quick change in reward systems. These problems are not seen in teams that have consistently cooperative systems. LO: 10.4: Explain how organizations can create effective teams. AACSB: Interpersonal relations and teamwork Difficulty: Moderate Quest. Category: Concept 101) Investing in effective one-time team training eliminates the need for future team training. Answer: FALSE Explanation: Team training is not just a one-time activity–the need for team training evolves over time, and managers would do well to keep a pulse on teams' training needs in their organizations. LO: 10.4: Explain how organizations can create effective teams. AACSB: Interpersonal relations and teamwork Difficulty: Moderate Quest. Category: Concept 102) Given the increasing importance of remote work, virtual teams should be comprised of individuals with strong leadership, decision-making, interpersonal, and analytical skills in order to increase team effectiveness. Answer: TRUE Explanation: Given that remote work is becoming increasingly prevalent in the modern working world, managers would do well to select employees for virtual teams with the skills needed to excel in these environments (e.g., leadership, decision-making, interpersonal, and analytical skills). LO: 10.4: Explain how organizations can create effective teams. AACSB: Interpersonal relations and teamwork Difficulty: Moderate Quest. Category: Concept
469 richard@qwconsultancy.com
103) How can organizations create team players? Answer: The primary options for trying to turn individuals into team players include: a) Selection. Some people already possess the interpersonal skills to be effective team players. When hiring team members, in addition to the technical skills required to fill the job, care should be taken to ensure that candidates can fulfill their team roles as well as technical requirements. Many job candidates don't have team skills. This is especially true for those socialized around individual contributions. When faced with such candidates, managers basically have three options. The candidates can undergo training to "make them into team players." If this isn't possible or doesn't work, the other two options are to transfer the individual within the organization to a unit without teams (if this possibility exists) or don't hire the candidate. In established organizations that decide to redesign jobs around teams, it should be expected that some employees will resist being team players and may be untrainable. Unfortunately, such people typically become casualties of the team approach. b) Training. On a more optimistic note, a large proportion of people raised on the importance of individual accomplishments can be trained to become team players. Training specialists conduct exercises that allow employees to experience the satisfaction that teamwork can provide. They typically offer workshops to help employees improve their problem-solving, communication, negotiation, conflict-management, and coaching skills. Employees also learn the five-stage group development model. c) Rewards. The reward system needs to be reworked to encourage cooperative efforts rather than competitive ones. Promotions, pay raises, and other forms of recognition should be given to individuals for how effective they are as a collaborative team member. This doesn't mean individual contributions are ignored; rather, they are balanced with selfless contributions to the team. Examples of behaviors that should be rewarded include training new colleagues, sharing information with teammates, helping to resolve team conflicts, and mastering new skills that the team needs but in which it is deficient. LO: 10.4: Explain how organizations can create effective teams. AACSB: Interpersonal relations and teamwork Difficulty: Moderate Quest. Category: Concept
470 richard@qwconsultancy.com
104) Which of the following is true with regard to use of individuals and teams? A) Teams and individuals can be used interchangeably for all kinds of jobs without any significant impact seen on effectiveness. B) Teamwork requires less time and resources when compared to individual work. C) Individuals have increased communication, supervision, and conflict-management demands when compared to teams. D) Individuals are bound by a collective common purpose that is organizationally determined, and this typically reduces their performance. E) Task complexity and requirement of different perspectives must be used as criteria when assigning work to teams over individuals and vice versa. Answer: E Explanation: Teamwork takes more time and often more resources than individual work. Teams have increased communication demands, conflicts to manage, and meetings to run. A good indicator is the complexity of the work and the need for different perspectives. Simple tasks that don't require diverse input are probably better left to individuals. LO: 10.5: Decide when to use individuals instead of teams. AACSB: Interpersonal relations and teamwork Difficulty: Moderate Quest. Category: Concept 105) When should work be performed by an individual rather than by a team? A) When the work is complex and requires different perspectives B) When the work creates a common purpose or set of goals for the people in the group that is more than the aggregate of individual goals C) When the work is simple and does not require diverse input D) When several tasks that are interdependent are to be performed for completing the work E) When performing the work requires learning a new technology or understanding a new system Answer: C Explanation: If the tasks are simple and do not require diverse input, then individuals must be used to complete them. A team should be used when the work creates a common purpose or set of goals for the people in the group that is more than the aggregate of individual goals. Using teams makes sense when there is interdependence between tasks—the success of the whole depends on the success of each one, and the success of each one depends on the success of the others. LO: 10.5: Decide when to use individuals instead of teams. AACSB: Interpersonal relations and teamwork Difficulty: Moderate Quest. Category: Concept
471 richard@qwconsultancy.com
106) The work of your group would be better done in teams if the members of the team are ________. A) independent B) autonomous C) individualistic D) interdependent E) reflexive Answer: D Explanation: The work of your group would be better done in teams if the members of the team are interdependent. Using teams makes sense when there is interdependence among tasks such that the success of the whole depends on the success of each one, and the success of each one depends on the success of the others. LO: 10.5: Decide when to use individuals instead of teams. AACSB: Interpersonal relations and teamwork Difficulty: Moderate Quest. Category: Concept 107) Using teams makes sense when the assignment involves independent tasks. Answer: FALSE Explanation: Using teams makes sense when there is interdependence among tasks such that the success of the whole depends on the success of each one, and the success of each one depends on the success of the others. LO: 10.5: Decide when to use individuals instead of teams. AACSB: Interpersonal relations and teamwork Difficulty: Moderate Quest. Category: Concept Organizational Behavior, 19e (Robbins/Judge) Chapter 11 Communication 1) Which of the following statements is true with regard to communication? A) Communication cannot be used to motivate and control employees in an organization. B) Communication involves the transfer and understanding of meaning. C) Communication involves the mere imparting of meaning to another person or group. D) Perfect communication is not dependent on a channel and its richness. E) Perfect communication results in dissimilar mental models. Answer: B Explanation: Communication must include both the transfer and the understanding of meaning. In ideal communication, if it existed, a thought would be perfectly understood by the person receiving the message exactly how the sender would have intended. LO: 11.1: Describe the types of interpersonal communication. AACSB: Written and oral communication Difficulty: Moderate Quest. Category: Concept 2) ________ refers to the idea that both the sender and receiver are present, aware, and focused on the communication exchange. 472 richard@qwconsultancy.com
A) Conversation B) Speech exchange C) Discussion D) Synchronicity E) Listening Answer: D Explanation: Synchronicity refers to the idea that both the sender and receiver are present, aware, and focused on the communication exchange. LO: 11.1: Describe the types of interpersonal communication. AACSB: Written and oral communication Difficulty: Moderate Quest. Category: Concept 3) Advantages to synchronous communication are speed, feedback, and ________. A) discussion B) filtering C) emotional expression D) control E) exchange Answer: E Explanation: The advantages of synchronous communication are speed, feedback, and exchange. LO: 11.1: Describe the types of interpersonal communication. AACSB: Written and oral communication Difficulty: Easy Quest. Category: Concept 4) Which of the following is an example of asynchronous communication? A) Filtering B) Voicemail C) Emotional expression D) In-person discussions E) Zoom meetings Answer: B Explanation: Whenever you check your voicemail, you are engaging in asynchronous oral communication. Asynchronous communication involves verbal messages being sent and received outside a physically or psychologically present communication exchange. LO: 11.1: Describe the types of interpersonal communication. AACSB: Written and oral communication Difficulty: Hard Quest. Category: Application 5) Stan Xia wants to update his employees about the company's work-from-home policy. Because employees are scattered across multiple time zones, Stan decides to film a video to communicate the updates. Stan is engaging in ________ communication. A) filtering 473 richard@qwconsultancy.com
B) channel regulation C) active listening D) asynchronous E) reflective listening Answer: D Explanation: Stan is engaging in asynchronous communication. Asynchronous communication involves verbal messages being sent and received outside a physically or psychologically present communication exchange. LO: 11.1: Describe the types of interpersonal communication. AACSB: Written and oral communication Difficulty: Moderate Employability Skills: Knowledge Application and Analysis Quest. Category: Application 6) Which of the following must occur in synchronous communication? A) The sender and receiver must be in the same room. B) The sender and receiver must be in different rooms. C) The transmitter must be agreed to by both the sender and the receiver. D) The sender and receiver must both be present, aware, and focused on the communication. E) Messages must be received outside a physically or psychologically present communication. Answer: D Explanation: In a synchronous communication, the sender and receiver must both be present, aware, and focused on the communication. LO: 11.1: Describe the types of interpersonal communication. AACSB: Written and oral communication Difficulty: Easy Quest. Category: Concept 7) When we tune the other person out, think about what we are going to say, or rush to offer advice, we are engaging in listener ________. A) active listening B) filtering C) burnout D) noise E) reflective listening Answer: C Explanation: We are often prone to "listener burnout," in which we tune the other person out, think about what we are going to say next, or rush to offer advice. LO: 11.1: Describe the types of interpersonal communication. AACSB: Written and oral communication Difficulty: Moderate Quest. Category: Concept 8) Which of the following is not true of listener burnout? A) Stressors lead to listener burnout more quickly. B) It can occur when we are preoccupied with other things going on in our lives. 474 richard@qwconsultancy.com
C) It can occur during a discussion that is not stimulating. D) It is generally linked to the encoding part of communication. E) People may experience it if they expect the conversation to infringe upon their privacy. Answer: D Explanation: We are often prone to "listener burnout," in which we tune the other person out, think about what we are going to say next, or rush to offer advice. Stressors also lead us to listener burnout more quickly; when we are preoccupied with other things going on in our lives, it is difficult to be fully present in conversations. Not all conversations and discussions are stimulating and energizing as well–people are motivated to avoid conversations that they expect to cause conflict or that infringe upon their privacy. LO: 11.1: Describe the types of interpersonal communication. AACSB: Written and oral communication Difficulty: Moderate Quest. Category: Concept
475 richard@qwconsultancy.com
9) ________ involves actively engaging in sensing and processing others' communication messages (both subtle and overt) and then responding in ways that show we are actively engaged in the conversation. A) Synchronicity B) Active listening C) Encoding D) Asynchronous communication E) Reflective listening Answer: B Explanation: Active listening–in which we actively engage in sensing and processing others' communication messages (both subtle and overt) and then responding in ways that show we are actively engaged in the conversation–helps us become more present in our oral communication. LO: 11.1: Describe the types of interpersonal communication. AACSB: Written and oral communication Difficulty: Easy Quest. Category: Concept 10) Denise Bryan is meeting with a difficult client this afternoon. In the past, the client has expressed dissatisfaction with the way his complaints have been handled and accused Denise's team members of not listening. Denise resolves to actively engage in sensing and processing the client's subtle and overt communication messages to show the client that she cares about how he feels. Denise is engaging in ________. A) filtering B) reflective listening C) inactive emotional expression D) active listening E) encoding Answer: D Explanation: Denise is engaging in active listening. Active listening–in which we actively engage in sensing and processing others' communication messages (both subtle and overt) and then responding in ways that show we are actively engaged in the conversation–helps us become more present in our oral communication. LO: 11.1: Describe the types of interpersonal communication. AACSB: Written and oral communication Difficulty: Moderate Employability Skills: Knowledge Application and Analysis Quest. Category: Application
476 richard@qwconsultancy.com
11) Research shows that engaging in active listening ________. A) reduces employee understanding of customer needs B) distorts customer perspectives for employees C) reduces employee anxiety during difficult customer interactions D) limits employee confidence E) decreases employees' sense of self-efficacy Answer: C Explanation: One study on customer service employees in a Fortune 500 company found that active listening training led these employees to better understand customers' needs and perspectives, increased their sense of self-efficacy and confidence, and reduced their anxieties during difficult customer interactions. LO: 11.1: Describe the types of interpersonal communication. AACSB: Written and oral communication Difficulty: Moderate Quest. Category: Concept 12) ________ involves acknowledging, restating, or reformulating others' messages to provide non-judgmental affirmation and encourage them to further elaborate or share. A) The transmission of spontaneous messages B) Active listening C) The absence of noise in the communication cycle D) Synchronicity E) Reflective listening Answer: E Explanation: Reflective listening involves acknowledging, restating, or reformulating others' messages to provide non-judgmental affirmation and encourage them to further elaborate or share. LO: 11.1: Describe the types of interpersonal communication. AACSB: Written and oral communication Difficulty: Easy Quest. Category: Concept
477 richard@qwconsultancy.com
13) Angel Suarez is meeting with team members to hear their ideas on improving company benefits. Angel wants to be sure her team knows that she hears what they're saying, especially when it comes to the cost of health care insurance. To communicate her intentions, Angel frequently provides quick summaries of team member thoughts, saying "here's what I think you're saying." Angel is engaging in ________. A) active listening B) emotional expression C) filtering D) reflective listening E) encoding Answer: D Explanation: Angel is engaging in reflective listening. Reflective listening involves acknowledging, restating, or reformulating others' messages to provide non-judgmental affirmation and encourage them to further elaborate or share. Sometimes reflective listening can be as subtle as a nod of encouragement, a "yes, I understand," or an "I see"; sometimes it is as direct as a "Here's what I think you are saying…" LO: 11.1: Describe the types of interpersonal communication. AACSB: Written and oral communication Difficulty: Moderate Employability Skills: Knowledge Application and Analysis Quest. Category: Application 14) Which of the following is not true of reflective listening? A) It shows others that they have you attention. B) It encourages others to further elaborate or share. C) It can build trust. D) It can be subtle or direct. E) It involves judgmental affirmation. Answer: E Explanation: Reflective listening involves acknowledging, restating, or reformulating others' messages to provide non-judgmental affirmation and encourage them to further elaborate or share. Reflective listening enables you to show others that they have your attention, helps build trust, encourages then to confide in you, and increase the chance they you will not miss anything important in the conversation. Sometimes reflective listening can be as subtle as a nod of encouragement, a "yes, I understand," or an "I see"; sometimes it is as direct as a "Here's what I think you are saying…" LO: 11.1: Describe the types of interpersonal communication. AACSB: Written and oral communication Difficulty: Moderate Quest. Category: Concept
478 richard@qwconsultancy.com
15) ________ is part of the sensing component in the reflective listening. A) Assigning meaning and value to messages B) Expressing engagement non-verbally C) Actively sensing oral communication D) Ascertaining covert messages E) Responding timely Answer: C Explanation: Reflective listening involves sensing (actively sensing oral and nonverbal communication), processing (assigning meaning and value to messages; ascertaining subtle or covert messages), and responding (responding timely; expressing engagement orally and nonverbally.) LO: 11.1: Describe the types of interpersonal communication. AACSB: Written and oral communication Difficulty: Easy Quest. Category: Concept 16) The three parts of reflective listening are sensing, responding, and ________. A) feedback B) processing C) noise D) decoding E) encoding Answer: B Explanation: Reflective listening involves sensing (actively sensing oral and nonverbal communication), processing (assigning meaning and value to messages; ascertaining subtle or covert messages), and responding (responding timely; expressing engagement orally and nonverbally.) LO: 11.1: Describe the types of interpersonal communication. AACSB: Written and oral communication Difficulty: Easy Quest. Category: Concept 17) Which of the following is an example of oral communication? A) E-mail B) Blogs C) Instant messaging D) Newsletters E) Speeches Answer: E Explanation: Written communication includes letters, e-mail, instant messaging, blogs (discussed in the next section), newsletters, and any other method that conveys written words or symbols. Oral communication includes virtual or in-person conversations and discussions, formal meetings, and speeches. LO: 11.1: Describe the types of interpersonal communication. AACSB: Written and oral communication Difficulty: Easy 479 richard@qwconsultancy.com
Quest. Category: Application 18) Which form of communication in the workplace is almost as popular as in-person conversation? A) Instant messaging B) Newsletters C) Speeches D) E-mail E) Blogs Answer: D Explanation: Today, e-mail is so commonplace that it is about equally preferred to actual inperson communication by workers as the most popular form of workplace communication. LO: 11.1: Describe the types of interpersonal communication. AACSB: Written and oral communication Difficulty: Moderate Quest. Category: Concept 19) Compared to text messaging, instant messaging ________. A) is more reliable B) is tied to a person's wireless service C) involves sending a message through a third-party app or system D) requires a phone service E) doesn't require Internet connectivity Answer: C Explanation: Although instant messaging (IM) and text messaging refer to essentially the same activity, there are subtle differences between the two. Text messaging is, in some ways, a more reliable form of communication than IM as it is tied to people's wireless and phone services. If the person has a phone (and a plan to support it), they will likely be able to send a text message—with or without Internet connectivity. An instant message, on the other hand, involves sending a message through a third-party chat app or system (e.g., Messenger, Teams, Slack). LO: 11.1: Describe the types of interpersonal communication. AACSB: Written and oral communication Difficulty: Moderate Quest. Category: Concept
480 richard@qwconsultancy.com
20) Catherine Smith-Hawkins knows that her team is fed up with the seemingly no-ending array of Zoom meetings that have taken over their work days since the start of the COVID-19 pandemic. What is one way that Catherine can make meetings more effective? A) She should provide an agenda to attendees. B) She should avoid gathering input from attendees prior to the meeting. C) She should ensure that the meeting starts about 15 minutes after the scheduled start time. D) She should ensure that the meeting ends at least 10 minutes early. E) She should avoid delegating roles. Answer: A Explanation: Catherine can make meetings more effective by only meeting when it is truly needed; gathering input from attendees prior to starting; providing an agenda to attendees; delegating roles to attendees; limiting distractions; and starting and ending meetings on times. LO: 11.1: Describe the types of interpersonal communication. AACSB: Written and oral communication Difficulty: Moderate Employability Skills: Knowledge Application and Analysis Quest. Category: Application 21) Body language and movement, information conveyed through contact and our senses, and the physical use of space and time in interaction are all part of ________. A) lateral communication B) nonverbal communication C) paralanguage D) oral communication E) asynchronous communication Answer: B Explanation: Body language and movement, information conveyed through contact and our senses, and the physical use of space and time in interaction are all part of nonverbal communication. LO: 11.1: Describe the types of interpersonal communication. AACSB: Written and oral communication Difficulty: Easy Quest. Category: Concept
481 richard@qwconsultancy.com
22) Dynamix, a fast-paced communications industry, had been rated as one of the best places to work in the region for three years. The company has a large open concept office space that is dominated by a huge atrium where employees are encouraged to gather to talk or eat lunch, something that has always been highlighted in the best places to work reviews. Dina Mills, head of HR, was surprised that this year the company didn't even make it into the top ten of best places to work. Dina has heard through the grapevine that employees no longer want to spend time in the atrium lounge or recently renovated adjacent dining space. Which of the following, if true, would help to explain the attitude of Dynamix employees toward their workplace? A) Dina Mills has implemented a new team concept where teams are comprised of members who generate a lot of positive synergy while working together. B) Employees at Dynamix have been offered a new benefits package that offers employees a better choice of plans. C) The new kitchen facilities in the dining space at Dynamix are encouraging employees to prepare foods that other employees think smell disgusting. D) Dina Mills has implemented a monthly check-in program with all employees to gauge where the company can do better. E) Management at Dynamix has been discussing expanding the business. Answer: C Explanation: The fact that Dina has implemented a new team concept where teams are comprised of individuals who generate positive synergies would typically result in a positive feeling toward the work environment. The fact that employees have been offered a new benefits package that includes better choices should result in a positive attitude toward Dynamix. The fact that Dina Mills is checking in with employees monthly should generate a positive attitude toward work. The fact that management is discussing expanding the business suggests that the company is doing well. The fact that the new kitchen facilities are encouraging preparation of unpleasant smelling foods does explain why the company is no longer on the best places to work. In an open concept office space, the unpleasant cooking smells would permeate the entire office space. Smell and odor are powerful, symbolic forms of nonverbal communication in the workplace that can elicit a number of emotions. A pleasant smell can put you in a positive, focused state. The interesting thing about odor is it is very one-dimensional. People are often not very capable at telling differences between scents, but they can tell you one thing: whether it is pleasant or not. However, this one-dimensional nature of smell can lead to a number of negative outcomes when the smell is disgusting. For instance, fishy smells coming from others can lead people to become suspicious of those individuals, and disgusting smells can even activate prejudicial attitudes toward others. These black-and-white impacts of smell on our attitudes and behaviors are problematic because smell more often than not has nothing to do with our actual intentions or dispositions. LO: 11.1: Describe the types of interpersonal communication. AACSB: Analytical thinking Difficulty: Hard Employability Skills: Critical Thinking Quest. Category: Critical Thinking
482 richard@qwconsultancy.com
23) Which of the following statements is true regarding oral communication? A) It is tangible and verifiable. B) It can be stored for an indefinite period. C) The sender can quickly obtain feedback from the receiver. D) As compared to written communication, it is more likely to be well thought out, logical, and clear. E) In the case of oral communication, the possibility of distortion of the message decreases. Answer: C Explanation: The advantages of oral communication are speed and feedback. We can convey a verbal message and receive a response in minimal time. If the receiver is unsure of the message, rapid feedback allows the sender to quickly detect and correct it. The major disadvantage of oral communication surfaces whenever a message has to pass through a number of people: the more people, the greater the potential distortion. LO: 11.1: Describe the types of interpersonal communication. AACSB: Written and oral communication Difficulty: Easy Quest. Category: Concept 24) Written communication such as letter writing is characterized by ________. A) a high degree of potential distortion B) the speed of transmission and feedback C) the availability of nonverbal cues D) its lasting nature E) minimal time required for transmission and feedback Answer: D Explanation: Letter writing can be used to great effect in business, adding a personal touch to a communication or, alternatively, creating a lasting document to signal an official commitment. Both the sender and receiver have a record of the communication, and the message can be stored for an indefinite period. LO: 11.1: Describe the types of interpersonal communication. AACSB: Written and oral communication Difficulty: Moderate Quest. Category: Concept
483 richard@qwconsultancy.com
25) Which of the following forms of communication is characterized by the use of only body movements, intonations, facial expressions, and physical distance to communicate? A) Written communication B) Downward communication C) Nonverbal communication D) Lateral communication E) Upward communication Answer: C Explanation: Nonverbal communication deals with body movements, the intonations or emphasis we give to words, facial expressions, and the physical distance between the sender and receiver. Downward, upward, and lateral communication may or may not be nonverbal. LO: 11.1: Describe the types of interpersonal communication. AACSB: Written and oral communication Difficulty: Easy Quest. Category: Concept 26) Which of the following modes of communication does not fall under the concept of nonverbal communication? A) Physical distance B) Personal e-mails C) Intonations D) Facial expressions E) Body movement Answer: B Explanation: Nonverbal communication deals with body movements, the intonations or emphasis we give to words, facial expressions, and the physical distance between the sender and receiver. LO: 11.1: Describe the types of interpersonal communication. AACSB: Written and oral communication Difficulty: Moderate Quest. Category: Concept 27) Emotions, moods, and personality traits can all be measured using natural language processing. Answer: TRUE Explanation: Through natural language processing, researchers can train algorithms to incorporate the actual words people use in e-mail, instant messages, social media, and other written communication media (including hashtags) to measure emotions, moods, personality traits, stress, and other characteristics of employees. LO: 11.1: Describe the types of interpersonal communication. AACSB: Written and oral communication Difficulty: Moderate Quest. Category: Concept
484 richard@qwconsultancy.com
28) There is greater potential for distortion in oral communication when a message must be pass through several people. Answer: TRUE Explanation: A major disadvantage of oral communication surfaces whenever a message must pass through several people: the more people, the greater the potential distortion. LO: 11.1: Describe the types of interpersonal communication. AACSB: Written and oral communication Difficulty: Moderate Quest. Category: Concept 29) Engaging in active listening helps us become more present in our oral communication. Answer: TRUE Explanation: Active listening–in which we actively engage in sensing and processing others' communication messages (both subtle and overt) and then responding in ways that show we are actively engaged in the conversation–helps us become more present in our oral communication. LO: 11.1: Describe the types of interpersonal communication. AACSB: Written and oral communication Difficulty: Moderate Quest. Category: Concept 30) All oral communication is synchronous communication. Answer: FALSE Explanation: Oral communication can also be asynchronous communication with verbal messages send and received outside a physically or psychologically present communication exchange. LO: 11.1: Describe the types of interpersonal communication. AACSB: Written and oral communication Difficulty: Moderate Quest. Category: Concept 31) Using texting and IMs in the workplace can lead to sloppiness, curtness, negativity, and misunderstanding from recipients. Answer: TRUE Explanation: The brief, conversational, and informal nature of texting and IMs can lead to sloppiness, curtness, negativity, and misunderstanding from recipients. LO: 11.1: Describe the types of interpersonal communication. AACSB: Written and oral communication Difficulty: Moderate Quest. Category: Concept
485 richard@qwconsultancy.com
32) Nonverbal communication can be unconscious and automatic. Answer: TRUE Explanation: Nonverbal communication can be unconscious and automatic—when events happen, we react and express our emotions, and the bystanders perceiving our emotions then make appraisals based on the emotional expressions. For instance, it may be one subconscious mechanism behind the spread of racial bias in the workplace. LO: 11.1: Describe the types of interpersonal communication. AACSB: Written and oral communication Difficulty: Moderate Quest. Category: Concept 33) E-mail is so commonplace that it is about equally preferred to actual in-person communication by workers as the most popular form of workplace communication. Answer: TRUE Explanation: Today, e-mail is so commonplace that it is about equally preferred to actual inperson communication by workers as the most popular form of workplace communication. LO: 11.1: Describe the types of interpersonal communication. AACSB: Written and oral communication Difficulty: Moderate Quest. Category: Concept 34) Attributions people make about others' nonverbal behaviors are strong and consistent, even when they do not match with reality. Answer: TRUE Explanation: The attributions people make about others' nonverbal behaviors are strong and consistent, even when they do not match with reality. For instance, even though people may perceive certain nonverbal behaviors (e.g., standing up straight and posing) as dominant, they do not always match that individual's "actual" dominant personality traits. LO: 11.1: Describe the types of interpersonal communication. AACSB: Written and oral communication Difficulty: Moderate Quest. Category: Concept 35) Startup entrepreneurs adopt paralanguage when they try to persuade potential clients to do business with them even as the entrepreneur doubts their own ability to follow through. Answer: TRUE Explanation: Sounds and vocal intonations (sometimes called paralanguage) could also be considered a sensory form of nonverbal communication that serve as their own form of communication beyond verbal messages. Some researchers suggest that paralanguage serves to persuade and influence others, such as how startup entrepreneurs speak with confidence and conviction to convince a potential client despite the entrepreneurs' own reservations about their ability to follow through. LO: 11.1: Describe the types of interpersonal communication. AACSB: Written and oral communication Difficulty: Moderate Quest. Category: Concept 486 richard@qwconsultancy.com
36) Crossed arms convey a feeling of approachability. Answer: FALSE Explanation: We act out our state of being with nonverbal language. We can smile to project trustworthiness, uncross our arms to appear approachable, and stand to signal authority. LO: 11.1: Describe the types of interpersonal communication. AACSB: Written and oral communication Difficulty: Moderate Quest. Category: Concept 37) Identify and explain the three parts of reflective listening. Answer: Reflective listening involves (1) sensing (actively sensing oral and nonverbal communication), (2) processing (assigning meaning and value to messages; ascertaining subtle or covert messages), and (3) responding (responding timely; expressing engagement orally and non-verbally.) LO: 11.1: Describe the types of interpersonal communication. AACSB: Written and oral communication Difficulty: Moderate Quest. Category: Concept 38) What is the difference between synchronous communication and asynchronous communication? Answer: In a synchronous communication, the sender and receiver must both be present, aware, and focused on the communication. The advantages of synchronous communication are speed, feedback, and exchange. Asynchronous communication involves verbal messages being sent and received outside a physically or psychologically present communication exchange. Whenever you check your voice-mail, you are engaging in asynchronous oral communication. LO: 11.1: Describe the types of interpersonal communication. AACSB: Written and oral communication Difficulty: Moderate Quest. Category: Concept
487 richard@qwconsultancy.com
39) How can employees better manage the information overload associated with e-mail? Answer: Researchers and practitioners have a number of suggestions to manage your inbox effectively if you are struggling from e-mail overload: • Turn off notifications. • Check your e-mail at regular intervals (exact times during the day–for instance, every three hours). • Immediately move your e-mail out of your inbox after reading. • Use the search functionality and e-mail filters to find e-mails. • Use shortcuts to archive e-mails in a small number of categorized folders. • Single out important e-mails individually; process and treat less important e-mails in groups or batches. • Use "reply all" thoughtfully and only when all people need to receive your response (to avoid clogging their inboxes). • Do not treat e-mail like an in-person dialogue. • Avoid the temptation to skim or skip e-mails. • Think before you send. LO: 11.1: Describe the types of interpersonal communication. AACSB: Written and oral communication Difficulty: Moderate Quest. Category: Concept 40) Explain how contact and touch act as a form of nonverbal communication. Answer: Touch can help provide support during times of loss and need, communicate warmth and welcoming, and even lead to improved team performance through increases in cooperative intent. Handshaking during job interviews is important for molding and managing impressions. Touch in the workplace is, well, a touchy subject, but it is more common than one would think. Some touch and contact are required in specific occupations (e.g., doctors, personal trainers, hair stylists), and others are normatively appropriate (e.g., tapping someone on the shoulder to ask them a question). However, touching and physical contact at work have become the subject of great scrutiny, with the #MeToo movement highlighting inappropriate touching in the workplace and the COVID-19 pandemic bringing to surface the health risks of physical contact between coworkers. This scrutiny led to the question of whether contact that communicates warmth or intimacy (e.g., hugging) is appropriate in the workplace. Employees and managers need to learn and respect one another's boundaries, and when there is physical contact involved, keep a light brief touch. Moreover, in the wake of the pandemic, other forms of physical contact to replace hugs and handshakes have become more popular (e.g., fist or elbow bumps). LO: 11.1: Describe the types of interpersonal communication. AACSB: Written and oral communication Difficulty: Moderate Quest. Category: Synthesis
488 richard@qwconsultancy.com
41) Which communication method makes sense when your question or idea requires a lot of verbal explanation? A) In-person meeting B) Phone call C) Virtual meeting D) Text E) E-mail Answer: B Explanation: Phone calls are appropriate when your question or idea requires a lot of verbal explanation. LO: 11.2: Evaluate how to choose communication methods and handle barriers to effective communication. AACSB: Written and oral communication Difficulty: Moderate Quest. Category: Concept 42) Peter is working on a project. He wants to see if a colleague will be available for an inperson meeting later in the afternoon. Which communication method should Peter use to get in touch with his colleague to schedule the meeting? A) Virtual meeting B) In-person meeting C) Instant message D) E-mail E) Phone call Answer: C Explanation: Texts or instant messages are appropriate when you are asking whether another person is available for an in-person or phone meeting. LO: 11.2: Evaluate how to choose communication methods and handle barriers to effective communication. AACSB: Written and oral communication Difficulty: Moderate Employability Skills: Knowledge Application and Analysis Quest. Category: Application
489 richard@qwconsultancy.com
43) E-mails are the appropriate method of communications when ________. A) you have a quick, noncritical question multiple people are capable of answering B) you need to evaluate as a group obstacles or hurdles that may come up in an upcoming task C) you are outlining procedures, strategies, or steps others need to follow D) you need to discuss conflict, performance goals, or behavioral issues E) you need something done or answered in the next 30 minutes Answer: C Explanation: E-mails are the appropriate method of communications when you are outlining procedures, strategies, or steps others need to follow. LO: 11.2: Evaluate how to choose communication methods and handle barriers to effective communication. AACSB: Written and oral communication Difficulty: Moderate Quest. Category: Concept 44) Ellen Cary has been assigned to work on her company's sales pitch for a valuable client. Ellen is working together with three other people and needs to send a link to a document to her team members so that they can collaborate in real time. Which is the best communication method for Ellen to use? A) E-mail B) In-person meeting C) Virtual meeting D) A phone call E) Text Answer: E Explanation: Ellen should use a text or instant message to communicate with her team members. This communication method is appropriate when you are sharing information (like a document or a link) that multiple people need to collaborate or work on in real-time. LO: 11.2: Evaluate how to choose communication methods and handle barriers to effective communication. AACSB: Written and oral communication Difficulty: Moderate Quest. Category: Concept
490 richard@qwconsultancy.com
45) A condition in which information inflow exceeds an individual's processing capacity is known as ________. A) synchronicity B) channeling C) noise D) asynchronous communication E) information overload Answer: E Explanation: A condition in which information inflow exceeds an individual's processing capacity is known as information overload. LO: 11.2: Evaluate how to choose communication methods and handle barriers to effective communication. AACSB: Written and oral communication Difficulty: Easy Quest. Category: Concept 46) Which of the following is true of information overload? A) Most people dislike being overloaded with information. B) The number of people who like "so much information" increased between 2006 and 2016. C) Between 2006 and 2016 the number of people who felt information overload increased substantially. D) Smart phones have made it more difficult than ever for people to manage information. E) Socioeconomic status has no impact on information overload. Answer: B Explanation: Research over the past several decades suggests that most people actually like being overloaded with information on a daily basis. Between 2006 and 2016, for instance, the number of people who felt information overload decreased from 27 to 20 percent. On the other hand, the number of people who like "so much information" increased from 67 to 77 percent. The comfort with information overload is primarily attributable to the rise in smart gadgets and devices that people use to manage large amounts of information. These devices enable them to keep up with demands, determine what information to trust more quickly, and have control over their lives. Some research suggests that the differences in perceptions of information overload might be attributable to socioeconomic status. Not everyone has access to smart technology (e.g., smartphones, smarthomes, digital assistants, tablets, and broadband Internet). LO: 11.2: Evaluate how to choose communication methods and handle barriers to effective communication. AACSB: Written and oral communication Difficulty: Moderate Quest. Category: Concept
491 richard@qwconsultancy.com
47) Undue tension and anxiety about communication is known as ________. A) information overload B) communication apprehension C) asynchronous communication D) synchronicity E) unconscious bias Answer: B Explanation: Communication apprehension refers to undue tension and anxiety about communication. LO: 11.2: Evaluate how to choose communication methods and handle barriers to effective communication. AACSB: Written and oral communication Difficulty: Easy Quest. Category: Concept 48) Chris Bowman has always been at his best when he meets with people in-person. In fact, Chris is feeling anxious and finding it very difficult at work since his company replaced inperson meetings with Zoom calls. Chris is experiencing ________. A) information overload B) asynchronous communication C) synchronicity D) unconscious bias E) communication apprehension Answer: E Explanation: Chris is experiencing communication apprehension. Communication apprehension refers to undue tension and anxiety about communication. LO: 11.2: Evaluate how to choose communication methods and handle barriers to effective communication. AACSB: Written and oral communication Difficulty: Moderate Quest. Category: Concept
492 richard@qwconsultancy.com
49) In times of crisis, ________. A) independent problem solving should be stressed B) managers should wait until they have thought through a plan of action before sharing any facts C) managers should rely on traditional methods of communication like phone calls or in-person meetings D) effective communication involves breaking down crisis response into several stages E) Twitter should never be used to sound an alarm Answer: D Explanation: Effective communication involves breaking down crisis response into several stages. LO: 11.2: Evaluate how to choose communication methods and handle barriers to effective communication. AACSB: Written and oral communication Difficulty: Moderate Quest. Category: Concept 50) Undue tension and anxiety about oral communication, written communication, or both is called ________. A) information overload B) communication misrepresentation C) selective perception D) communication apprehension E) filtering Answer: D Explanation: Undue tension and anxiety about oral communication, written communication, or both is called communication apprehension. People who suffer from this condition may find it extremely difficult to talk with others face-to-face or may become extremely anxious when they have to use the phone, relying on memos or e-mails when a phone call would be faster and more appropriate. LO: 11.2: Evaluate how to choose communication methods and handle barriers to effective communication. AACSB: Written and oral communication Difficulty: Easy Quest. Category: Concept
493 richard@qwconsultancy.com
51) Shirley Shands is a hard worker, but Debra, her manager, is irritated with her. While Debra concedes that Shirley's work gives her no cause for complaint, Debra is tired of the interruptions in her work because of Shirley asking for repeated clarifications over what exactly she has to say in e-mails or phone calls. She often requests Debra to proofread her e-mails before sending them, and Debra feels she sends too many e-mails instead of just speaking to the person concerned. From the scenario, we can conclude that ________. A) Shirley processes her e-mails in an automatic and superficial manner B) Shirley and Debra come from a high-context culture C) Debra is engaging in social loafing D) Shirley suffers from communication apprehension E) Shirley uses filtering when communicating with Debra Answer: D Explanation: Undue tension and anxiety about oral communication, written communication, or both is called communication apprehension. People who suffer from this condition may find it extremely difficult to talk with others face-to-face or may become extremely anxious when they have to use the phone, relying on memos or e-mails when a phone call would be faster and more appropriate. LO: 11.2: Evaluate how to choose communication methods and handle barriers to effective communication. AACSB: Written and oral communication Difficulty: Moderate Employability Skills: Knowledge Application and Analysis Quest. Category: Application 52) In times of crisis, managers should try to work independently rather than together. Answer: FALSE Explanation: People should try to work together in times of crisis instead of independently. For example, research shows that people, when trying to solve a problem separately (e.g., developing a vaccine for a virus), may become overcome by confusion and an inability to learn from one another's failures–by coordinating and collaborating, they may be able to overcome the confusion and learn from one another. LO: 11.2: Evaluate how to choose communication methods and handle barriers to effective communication. AACSB: Written and oral communication Difficulty: Moderate Quest. Category: Concept 53) Communication apprehension applies to oral and written communication. Answer: TRUE Explanation: Communication apprehension refers to undue tension and anxiety about oral communication, written communication, or both. LO: 11.2: Evaluate how to choose communication methods and handle barriers to effective communication. AACSB: Written and oral communication Difficulty: Easy Quest. Category: Concept 494 richard@qwconsultancy.com
54) What is information overload and how is it a barrier to effective communication? Answer: People integrate information from multiple sources to effectively make sense of information. However, individuals have a finite capacity for processing data. When the information we must work with exceeds our processing capacity, the result is information overload. What happens when individuals have more information than they can sort and use? They tend to select, ignore, pass over, or forget it. Or they may put off further processing until the overload situation ends. In any case, lost information and less effective communication result, making it more important to deal with overload proactively. For instance, information overload may cause negotiators to give up on the deal more quickly or otherwise jump to conclusions about optimal offers.The pressure to interact with multiple people simultaneously (i.e., sometimes referred to as multicommunication) can cause people to be inadvertently rude with one another, a situation that can quickly spiral out of control. LO: 11.2: Evaluate how to choose communication methods and handle barriers to effective communication. AACSB: Written and oral communication Difficulty: Moderate Quest. Category: Concept 55) What are the implications of communication apprehension for organizations? Answer: Communication apprehension has a number of implications to effective communication in organizational contexts. For one, it can affect leadership emergence in virtual teams. For example, one study found that communication apprehension reduced the amount of electronic communication that a person engaged in and, as a consequence, the extent to which that person emerged as a team leader in the eyes of other team members. Communication apprehension can also be more or less salient at different times during a person's career. Applicants to jobs that require communication with others (e.g., role-play exercises, giving presentations, group interviews) may experience communication anxiety that may affect the personality attributions interviewers make and their overall communication skills scores. As another example, new employees may be all too familiar with the nervousness and anxiety surrounding communicating with a new supervisor. After all, you want to make a great impression! Communication apprehension refers to undue tension and anxiety about communication. LO: 11.2: Evaluate how to choose communication methods and handle barriers to effective communication. AACSB: Written and oral communication Difficulty: Moderate Quest. Category: Concept
495 richard@qwconsultancy.com
56) ________ scores highest in channel richness. A) A prerecorded speech B) E-mail C) A formal report D) An in-person conversation E) A live speech Answer: D Explanation: In-person conversation scores highest in channel richness because it transmits the most information per communication episode. LO: 11.3: Discuss the various forms of virtual communication used in modern organizations. AACSB: Written and oral communication Difficulty: Easy Quest. Category: Concept 57) Forms of communication that score lowest in channel richness are ________. A) formal reports and bulletins B) online discussion groups C) e-mail and voice mail D) telephone conversations E) video conferences Answer: A Explanation: Formal reports and bulletins score lowest in channel richness. LO: 11.3: Discuss the various forms of virtual communication used in modern organizations. AACSB: Written and oral communication Difficulty: Easy Quest. Category: Concept 58) Uma Visok needs to present a proposal to clients. She knows that the client is very busy and she probably only has one chance to get the information across, so she wants to be sure that she uses the richest channel possible. Uma should ________. A) consider giving the client a detailed formal report B) schedule a videoconference or face-to-face meeting C) e-mail a link to a formal report D) write a friendly letter outlining her proposal E) record a speech presenting her ideas and send the link Answer: B Explanation: Uma should schedule a videoconference or face-to-face meeting. Both of these communication channels are high in channel richness. LO: 11.3: Discuss the various forms of virtual communication used in modern organizations. AACSB: Analytical thinking Difficulty: Moderate Employability Skills: Critical Thinking Quest. Category: Synthesis
496 richard@qwconsultancy.com
59) Genepa Corporation manufactures home appliances and other electronic products. Genepa is planning to introduce a new refrigerator model. The marketing manager at Genepa has developed a marketing plan for this new product and wants to communicate this plan to all the employees in the marketing department in a way that allows employees to refer back to the plan at a later time. Which of the following is most likely to be the best form of communication for communicating this plan? A) Nonverbal communication B) Oral communication C) Written communication D) Grapevine communication E) Lateral communication Answer: C Explanation: Written communication includes letters, e-mail, instant messaging, organizational periodicals, and any other method that conveys written words or symbols. Some of these create a digital or physical long-term record. LO: 11.3: Discuss the various forms of virtual communication used in modern organizations. AACSB: Written and oral communication Difficulty: Moderate Employability Skills: Knowledge Application and Analysis Quest. Category: Application 60) Communicating through blogging provides ________. A) a host of nonverbal cues to understand the message better B) the best means to convey negative information C) a way for companies to share primarily written content and engage with others D) a secure communication link that nobody else can access E) an option to hold real-time meetings with people in different locations Answer: C Explanation: A blog (short for "web log") is a website by a single person, team, or company that is used to share primarily written content with others and engage them. LO: 11.3: Discuss the various forms of virtual communication used in modern organizations. AACSB: Written and oral communication Difficulty: Moderate Quest. Category: Concept
497 richard@qwconsultancy.com
61) Which of the following is a key to successful blogging? A) Make the subject line short and topic-related B) Offer a greeting and salutation C) Do not use bullet points D) Always check your spelling E) Make your content searchable Answer: E Explanation: The key to successful blogging is engagement and to make your content searchable. LO: 11.3: Discuss the various forms of virtual communication used in modern organizations. AACSB: Written and oral communication Difficulty: Moderate Quest. Category: Concept 62) When you want to emphasize in-depth communication and storytelling, which type of communication is best? A) Twitter B) E-mail C) Instant message D) Texts E) Podcasts Answer: E Explanation: Podcasts resemble mini-radio shows, with people sharing primarily audio content to serve the same function as a written blog, but with more emphasis on in-depth discussion and storytelling. LO: 11.3: Discuss the various forms of virtual communication used in modern organizations. AACSB: Written and oral communication Difficulty: Moderate Quest. Category: Concept 63) Which of the following types of electronic communications is most likely used to train employees to do their work? A) Twitter B) Video conferencing C) E-learning D) Grapevine E) E-mails Answer: C Explanation: E-collaboration and e-learning build upon corporate social media but rather than being solely devoted to social networking, these media are literal platforms through which employees do their work or train to do their work. LO: 11.3: Discuss the various forms of virtual communication used in modern organizations. AACSB: Written and oral communication Difficulty: Moderate Quest. Category: Concept
498 richard@qwconsultancy.com
499 richard@qwconsultancy.com
64) People ranking high on ________ are more likely to post selfies than people ranking high on the other Big Five personality traits. A) agreeableness B) extroversion C) openness D) conscientiousness E) neuroticism Answer: B Explanation: People ranking high on extraversion are more likely to post selfies than people ranking high on the other Big Five personality traits. LO: 11.3: Discuss the various forms of virtual communication used in modern organizations. AACSB: Written and oral communication Difficulty: Moderate Quest. Category: Concept 65) A(n) ________ is a website about a single person or company. A) e-mail account B) social networking site C) blog D) instant messaging portal E) file transfer protocol Answer: C Explanation: A blog (web log) is a website about a single person or company. LO: 11.3: Discuss the various forms of virtual communication used in modern organizations. AACSB: Written and oral communication Difficulty: Easy Quest. Category: Concept 66) Which of the following communication channels provides the highest information richness? A) e-mail B) in-person conversations C) online group discussions D) voice mail E) memos and letters Answer: B Explanation: Rich communication channels can (1) handle multiple cues simultaneously, (2) facilitate rapid feedback, and (3) be very personal. In-person conversation scores highest in channel richness because it transmits the most information per communication episode. LO: 11.3: Discuss the various forms of virtual communication used in modern organizations. AACSB: Written and oral communication Difficulty: Easy Quest. Category: Concept
500 richard@qwconsultancy.com
67) Which of the following communication channels provides the lowest information richness? A) Online group discussions B) Face-to-face conversations C) Telephone conversations D) Voice mail E) Reports and bulletins Answer: E Explanation: Rich communication channels can (1) handle multiple cues simultaneously, (2) facilitate rapid feedback, and (3) be very personal. Impersonal written media such as formal reports and bulletins rate lowest in richness. LO: 11.3: Discuss the various forms of virtual communication used in modern organizations. AACSB: Written and oral communication Difficulty: Easy Quest. Category: Concept 68) When the middle-level manager of the production department at Wilson Works was laid off, many employees who worked under him became apprehensive about their job security. They were aware of the current trend toward laying off employees in the economy. The termination of their manager combined with the fact that their division had performed only moderately well in the last two quarters led the employees to believe that they were at a high risk of losing their jobs as well. Which of the following, if true, would weaken the argument that the production manager should hold face-to-face meetings to assure the employees in the production division? A) The employees have serious concerns over job security. B) Rumors had spread about impending layoffs at Wilson Works. C) The middle-level manager supervised over five hundred employees. D) Wilson Works has a strong formal communication network in place. E) The company has no fixed policy with regard to rightsizing employees. Answer: C Explanation: The fact that the employees have serious concerns over job security strengthens the argument that the production manager should hold face-to-face meetings to assure the employees in the production division. The fact that rumors had spread about impending layoffs at Wilson Works also strengthens the argument that the production manager should hold face-to-face meetings to assure the employees in the production division. The fact that the middle-level manager supervised over five hundred employees weakens the argument that the production manager should hold face-to-face meetings to assure the employees in the production division as it is not feasible. The fact that Wilson Works has a strong formal communication network in place does not strengthen or weaken the argument presented here. The fact that the company has no fixed policy with regard to rightsizing employees does not strengthen or weaken the argument presented here. LO: 11.3: Discuss the various forms of virtual communication used in modern organizations. AACSB: Analytical thinking Difficulty: Hard Employability Skills: Critical Thinking Quest. Category: Critical Thinking
501 richard@qwconsultancy.com
69) Treesa Mayering recently began her first job at a PR firm called Pro as a copywriter. Her job involves creating scripts for press releases, advertisements, and other media events. She has been working under the guidance of Will Preston, the senior copywriter. Treesa, who has no prior experience in this job, had worked on scripts for advertisements and online brochures alone when she was asked to work on a press release for the first time. Will reviewed Treesa's script and sent her an e-mail full of detailed corrections, which she promptly implemented. However, when Will looked at the script later, he felt that it did not represent a significant improvement of her first draft. Which of the following, if true, would best explain this outcome? A) The feedback provided by Will was not elaborate and comprehensive enough. B) Treesa took a lot of time to implement Will's corrections and revise her script. C) Treesa was hired because she demonstrated considerable analytical and creative thinking skills in her pre-employment tests. D) Will routinely e-mailed feedback on scripts for advertisements, online brochures, and press releases to his team members. E) Will believed that Treesa's understanding of his e-mailed feedback would be accurate. Answer: A Explanation: The fact that the feedback provided by Will was not elaborate and comprehensive enough does explain this outcome as his choice of communication channel was not rich enough to convey the message he intended to communicate. The fact that Treesa took a lot of time to implement Will's corrections and revise her script does not explain this outcome. The fact that Treesa was hired because she demonstrated considerable analytical and creative thinking skills in her pre-employment tests does not explain this outcome. The fact that Will routinely e-mailed feedback on scripts does not explain this outcome. The fact that Will believed that Treesa's understanding of his e-mailed feedback would be accurate is an assumption that does not completely explain this outcome. LO: 11.3: Discuss the various forms of virtual communication used in modern organizations. AACSB: Analytical thinking Difficulty: Hard Employability Skills: Critical Thinking Quest. Category: Critical Thinking 70) One way employees can manage Zoom fatigue is by hiding the self-view. Answer: TRUE Explanation: Videoconferencing may be stressful and intense at times. Some solutions may be to minimize your window so that the other people on the call are not dominating the entirety of your screen. Furthermore, constantly monitoring yourself in the "self-view" can be draining. If the application provides the option, consider hiding the self-view to avoid the temptation of looking at yourself. A bonus to doing so is that you are more likely to focus on others and maintain nonverbal eye contact more effectively. LO: 11.3: Discuss the various forms of virtual communication used in modern organizations. AACSB: Written and oral communication Difficulty: Moderate Quest. Category: Concept
502 richard@qwconsultancy.com
71) Vlogging is an example of a high-channel "media rich" channel. Answer: TRUE Explanation: New virtual communication media, are for the most part, high-channel "media rich" and include videoconferencing, vlogging, and podcasting as well as forms of ecollaboration and e-learning. LO: 11.3: Discuss the various forms of virtual communication used in modern organizations. AACSB: Written and oral communication Difficulty: Moderate Quest. Category: Concept 72) How can employees and managers leverage videoconferencing? Answer: Experts offer the following suggestions for leveraging videoconferencing: (1) Be mindful of your nonverbal behavior, just as you would in person; (2) Set the stage; (3) Be aware of the other people on the call; (4) Leverage (but be sure to test) technology; (5) Manage videoconferencing fatigue; (6) Play with innovative videoconferencing applications. LO: 11.3: Discuss the various forms of virtual communication used in modern organizations. AACSB: Written and oral communication Difficulty: Moderate Quest. Category: Application 73) Discuss the role of videoconferencing as a norm for meeting both internally and with clients. Answer: A 2017 study revealed that videoconferencing was the norm for many companies. Videoconferencing permits employees and clients to conduct real-time meetings with people at different locations. Live audio and video images let us see, hear, and talk with each other without being physically in the same location. People can also share files and videos through "screen sharing" allowing everyone to see the files on their own devices. LO: 11.3: Discuss the various forms of virtual communication used in modern organizations. AACSB: Written and oral communication Difficulty: Moderate Quest. Category: Concept 74) Discuss body language and movement as a form of nonverbal communication. Answer: Nonverbal communication includes body movements, the intonations or emphasis we give to words, facial expressions, and the physical distance between the sender and receiver. A body position or movement does not by itself have a precise or universal meaning, but when it is linked with spoken language, it gives fuller meaning to a sender's message. Intonations can change the meaning of a message. Facial expressions also convey meaning. Physical distance also has meaning. What is considered proper spacing between people largely depends on cultural norms. LO: 11.3: Discuss the various forms of virtual communication used in modern organizations. AACSB: Written and oral communication Difficulty: Moderate Quest. Category: Concept
503 richard@qwconsultancy.com
75) The average adult spends about ________ daily checking e-mail and social media. A) 6 hours B) 45 minutes C) 2 hours D) 3 hours E) 90 minutes Answer: A Explanation: The average adult spends three hours on social media, primarily using smart devices like smartphones and three hours checking e-mail, or a total of 6 hours a day checking email and social media. LO: 11.4: Analyze the issues surrounding smartphones, social media, and cybersecurity confronting modern organizations. AACSB: Written and oral communication Difficulty: Moderate Quest. Category: Concept 76) E-Way is an electronics company that is developing a new cell phone that will be released in the market soon. Which of the following, if true, would strengthen the argument that the company should use social media to promote the new cellphone? A) E-Way's last two releases received a lot of criticism from consumers. B) One of E-Way's rival companies had used social media unsuccessfully to advertise its new product. C) The production team overshot the budget and the marketing team has low funds. D) The target consumers for the cellphone are not restricted to any demographic category. E) The company has a tightly integrated production and distribution mechanism. Answer: C Explanation: The fact that E-Way's last two releases received a lot of criticism from consumers does not strengthen the argument that the company should use social media to promote the new cellphone. The fact that one of E-Way's rival companies had used social media successfully to advertise its new product does not strengthen the argument that E-Way must do the same. The fact that the production team overshot the budget and the marketing team has low funds does strengthen the argument that E-Way should use social media to promote the new cellphone as it is cheaper. The fact that the target consumers for the cellphone are not restricted to any demographic category does not strengthen the argument in favor of E-Way using social media for advertising the new product. The fact that the company has a tightly integrated production and distribution mechanism does not strengthen the argument that E-Way must use social media to advertise the new product. LO: 11.4: Analyze the issues surrounding smartphones, social media, and cybersecurity confronting modern organizations. AACSB: Analytical thinking Difficulty: Hard Employability Skills: Critical Thinking Quest. Category: Critical Thinking
504 richard@qwconsultancy.com
77) The ________ element in a social networking site outlines the user's contacts. A) stream B) broadcast C) message D) network E) feedback Answer: D Explanation: Most social networking sites that comprise social media share common features. They contain a profile outlining user information (e.g., pictures, biographies, interests, etc.), a network outlining the user's contacts (e.g., people, public figures, and companies), a stream outlining activity and interaction on the site (e.g., shared updates, updates from contacts, communications between contacts and other users), and, finally, a message element, which allows users to IM one another. When users log in, they can perform a number of activities. They can broadcast or view information, receive or give feedback on this information, and engage in comparison between these processes and information. Through these typical, interconnected elements, social media empowers employees, supervisors, and other organizational figures to organize by sharing information, collaborating, and communicating with one another quickly and efficiently. LO: 11.4: Analyze the issues surrounding smartphones, social media, and cybersecurity confronting modern organizations. AACSB: Written and oral communication Difficulty: Easy Quest. Category: Concept 78) Which of the following source of cybersecurity threats can companies monitor? A) Terrorists B) Spies C) Professional hackers D) Malicious outsiders E) Disgruntled employees Answer: E Explanation: Cybersecurity threats can come from a variety of sources, all with various motivations: organized attackers (e.g., terrorists and spies), employees (e.g., disgruntled or opportunistic employees and even leaders), professional hackers or criminals, and even amateurs (e.g., malicious or curious outsiders who delight in the thrill of hacking). As employees are the only group in this list companies have direct contact with, management may monitor employee online activity and communications, which can seem invasive to employees. LO: 11.4: Analyze the issues surrounding smartphones, social media, and cybersecurity confronting modern organizations. AACSB: Written and oral communication Difficulty: Moderate Quest. Category: Concept
505 richard@qwconsultancy.com
79) Wearable digital devices and sensors can be used to keep track of team performance and individual performance in real-time so employees can seamlessly provide assistance to another when needed. Answer: TRUE Explanation: Wearable digital devices can be used to keep track of team performance and individual performance in real-time so employees can seamlessly provide assistance to another when needed. LO: 11.4: Analyze the issues surrounding smartphones, social media, and cybersecurity confronting modern organizations. AACSB: Written and oral communication Difficulty: Moderate Quest. Category: Concept 80) The shallowing hypothesis suggests that social media use has led to more reflective and more rapid thought that can result in richer evaluations or judgements. Answer: FALSE Explanation: The shallowing hypothesis suggests that social media use has led to less reflective and more rapid thought that can result in shallower evaluations or judgements. LO: 11.4: Analyze the issues surrounding smartphones, social media, and cybersecurity confronting modern organizations. AACSB: Written and oral communication Difficulty: Moderate Quest. Category: Concept 81) Social media can develop shared cognition among members by integrating content they use to better perform their jobs. Answer: TRUE Explanation: In the team context, social media can help develop shared cognition among members by enabling members to manage and expand their teams' network, by integrating content they can use to better perform their jobs, and by making thoughts and decisions more accessible for use in later team taskwork (e.g., recorded discussions on a Facebook group news feed). Social media can also facilitate the sharing of knowledge among team members as it can lead employees to develop a collective sense of trust and curiosity. LO: 11.4: Analyze the issues surrounding smartphones, social media, and cybersecurity confronting modern organizations. AACSB: Written and oral communication Difficulty: Moderate Quest. Category: Concept
506 richard@qwconsultancy.com
82) Social networking is a tool for prospective employees, hiring managers, and employees. Answer: TRUE Explanation: Social networking has become a tool for prospective employees, hiring managers, employees, and human resource divisions. LO: 11.4: Analyze the issues surrounding smartphones, social media, and cybersecurity confronting modern organizations. AACSB: Written and oral communication Difficulty: Moderate Quest. Category: Concept 83) Young women and extraverted social media users tend to use emojis more than others. Answer: TRUE Explanation: One study of over eighty-five thousand Facebook users found that 90 percent of these users regularly communicated with emojis, but young, women, and extroverted users tended to use them more than others. LO: 11.4: Analyze the issues surrounding smartphones, social media, and cybersecurity confronting modern organizations. AACSB: Written and oral communication Difficulty: Easy Quest. Category: Concept 84) Discuss the use of social media websites in business settings. Answer: Nowhere has online communication been more transformed than in the rise of social networks like Facebook and LinkedIn, and business is taking advantage of the opportunities these social media platforms present. Many organizations have developed their own in-house social networking applications, known as enterprise social software, and most have their own Facebook and Twitter feeds. Social networking has become a tool for prospective employees, hiring managers, employees, and human resources divisions. LO: 11.4: Analyze the issues surrounding smartphones, social media, and cybersecurity confronting modern organizations. AACSB: Analytical thinking Difficulty: Moderate Quest. Category: Synthesis
507 richard@qwconsultancy.com
85) Discuss the major implications of social media on explicit persona and relational outcomes. Answer: First, social media can be a double-edged sword for people. It can be a good thing for employees' happiness and well-being when it enables them to make meaningful connections but a bad thing when it leads to comparing themselves with other people or makes them feel alienated. Second, research has demonstrated that social media can lead employees to engage in constructive dialogue with managers and co-workers, but only when they feel they have a sense of autonomy in what they can say online (e.g., a psychologically safe online space) and identify with their organization. Furthermore, social media can lead employees to develop and build relationships with others who can help them in their careers, a benefit particularly enjoyed by those with extroverted personality traits. However, the relational benefits of social media can be completely undermined if employees become caught up in competition and social comparison. Finally, in the team context, social media can help develop shared cognition among members by enabling members to manage and expand their teams' network, by integrating content they can use to better perform their jobs, and by making thoughts and decisions more accessible for use in later team taskwork (e.g., recorded discussions on a Facebook group news feed). Social media can also facilitate the sharing of knowledge among team members as it can lead employees to develop a collective sense of trust and curiosity. LO: 11.4: Analyze the issues surrounding smartphones, social media, and cybersecurity confronting modern organizations. AACSB: Analytical thinking Difficulty: Moderate Quest. Category: Synthesis 86) How can companies help employees who are being cyberbullied? Answer: (1) Organizations must recognize the harm that online harassment can cause. Creating an environment in which employees feel comfortable, supported, and protected when reporting online abuse is essential. Many individuals who experience online harassment may have also been marginalized in other situations. As such, they may feel uneasy about potentially being dismissed, mocked, or even reprimanded. (2) Survey employees to determine the extent to which they face online harassment and how they are handling it. The survey should investigate how often employees are facing abuse and on what platforms. Also important is assessing the emotional, psychological, and professional impact of the harassment. (3) Develop protocols and training. These can help ensure that employees know the concrete steps they can take to protect themselves in case of online abuse. (4) Promote peer support networks. Online abuse can potentially lead those who experience it to feel isolated and alone. Peer support networks can give employees a space to find community, share experiences, and exchange strategies. (5) Managers can play a crucial role by reaching out and listening to employees. Due to their identities or life experiences, some employees may not feel comfortable sharing their situations publicly, so being discreet might be best. LO: 11.4: Analyze the issues surrounding smartphones, social media, and cybersecurity confronting modern organizations. AACSB: Ethical understanding and reasoning Difficulty: Moderate Employability Skills: Business Ethics and Social Responsibility Quest. Category: Concept
508 richard@qwconsultancy.com
509 richard@qwconsultancy.com
87) ________ is an example of a high-context culture. A) Kenya B) Australia C) Denmark D) Germany E) United States Answer: A Explanation: In high-context culture such as Japan, Kenya, Saudi Arabia, and India, people rely heavily on nonverbal and subtle cues in communicating with others, and a person's official status, place in society, and reputation carry considerable weight. What is not said may be more significant than what is said. In contrast, people from low-context cultures such as the United States, Australia, Germany, and Denmark rely essentially on spoken and written words to convey meaning; body language and formal titles are secondary. LO: 11.5: Recognize how to engage in effective cross-cultural communication in organizations. AACSB: Diverse and multicultural work environments Difficulty: Easy Quest. Category: Concept 88) People from ________ share a low context culture. A) Scandinavia B) Saudi Arabia C) Greece D) South Korea E) China Answer: A Explanation: In high-context culture such as Japan, Kenya, Saudi Arabia, and India, people rely heavily on nonverbal and subtle cues in communicating with others, and a person's official status, place in society, and reputation carry considerable weight. What is not said may be more significant than what is said. In contrast, people from low-context cultures such as the United States, Australia, Germany, and Denmark rely essentially on spoken and written words to convey meaning; body language and formal titles are secondary. LO: 11.5: Recognize how to engage in effective cross-cultural communication in organizations. AACSB: Written and oral communication Difficulty: Easy Quest. Category: Concept
510 richard@qwconsultancy.com
89) Eleanor is preparing for a meeting with a new client. In doing her research, Eleanor has discovered that when it comes to communication, she should expect that the client will be focused on spoken and written words rather than body language and formal titles. Eleanor's client is most likely from ________. A) China B) Japan C) Denmark D) Saudi Arabia E) Greece Answer: C Explanation: Eleanor's client is most likely from Denmark, a low context culture. People from low context cultures rely essentially on spoken and written words to convey meaning; body language and titles are secondary. LO: 11.5: Recognize how to engage in effective cross-cultural communication in organizations. AACSB: Diverse and multicultural work environments Difficulty: Moderate Employability Skills: Knowledge Application and Analysis Quest. Category: Application 90) An individual who puts aside their own cultural preferences, values, practices, and customs often in an improvisational or flexible way is ________. A) avoiding B) imposing C) embracing D) synergizing E) compromising Answer: C Explanation: The five approaches to cross-cultural interaction are: avoiding (putting aside cultural preferences, values, practices, or customs often for tactical or strategic reasons); imposing (asserting one's own cultural preferences, values, practices, and customs without acknowledging others' cultural approaches); embracing (putting aside one's own cultural preferences, values, practices, and customs in order to acknowledge or embrace others' cultural approaches); synergizing (celebrating both interaction partners' cultural preferences, values, practices, and customs, often in an improvisational or flexible way); compromising (treating the cultural interaction as a give-and-take, recognizing when cultural preferences, values, practices, and customs conflict and embracing some and putting aside others in order to reduce conflict). LO: 11.5: Recognize how to engage in effective cross-cultural communication in organizations. AACSB: Diverse and multicultural work environments Difficulty: Easy Quest. Category: Concept
511 richard@qwconsultancy.com
91) A manager who is ________ treats cultural interactions as a give-and-take. A) imposing B) compromising C) embracing D) avoiding E) synergizing Answer: B Explanation: The five approaches to cross-cultural interaction are: avoiding (putting aside cultural preferences, values, practices, or customs often for tactical or strategic reasons); imposing (asserting one's own cultural preferences, values, practices, and customs without acknowledging others' cultural approaches); embracing (putting aside one's own cultural preferences, values, practices, and customs in order to acknowledge or embrace others' cultural approaches); synergizing (celebrating both interaction partners' cultural preferences, values, practices, and customs, often in an improvisational or flexible way); compromising (treating the cultural interaction as a give-and-take, recognizing when cultural preferences, values, practices, and customs conflict and embracing some and putting aside others in order to reduce conflict). LO: 11.5: Recognize how to engage in effective cross-cultural communication in organizations. AACSB: Diverse and multicultural work environments Difficulty: Easy Quest. Category: Concept 92) Harry has decided to acknowledge his colleague's culture and disregard his own when he meets her next week. Harry plans to engage in ________. A) imposing B) avoiding C) embracing D) synergizing E) compromising Answer: C Explanation: The five approaches to cross-cultural interaction are: avoiding (putting aside cultural preferences, values, practices, or customs often for tactical or strategic reasons); imposing (asserting one's own cultural preferences, values, practices, and customs without acknowledging others' cultural approaches); embracing (putting aside one's own cultural preferences, values, practices, and customs in order to acknowledge or embrace others' cultural approaches); synergizing (celebrating both interaction partners' cultural preferences, values, practices, and customs, often in an improvisational or flexible way); compromising (treating the cultural interaction as a give-and-take, recognizing when cultural preferences, values, practices, and customs conflict and embracing some and putting aside others in order to reduce conflict). LO: 11.5: Recognize how to engage in effective cross-cultural communication in organizations. AACSB: Diverse and multicultural work environments Difficulty: Moderate Employability Skills: Knowledge Application and Analysis Quest. Category: Application
512 richard@qwconsultancy.com
93) A manager who celebrates both interaction partners' cultural preferences is engaging in ________ interaction. A) synergizing B) embracing C) avoiding D) imposing E) compromising Answer: A Explanation: The five approaches to cross-cultural interaction are: avoiding (putting aside cultural preferences, values, practices, or customs often for tactical or strategic reasons); imposing (asserting one's own cultural preferences, values, practices, and customs without acknowledging others' cultural approaches); embracing (putting aside one's own cultural preferences, values, practices, and customs in order to acknowledge or embrace others' cultural approaches); synergizing (celebrating both interaction partners' cultural preferences, values, practices, and customs, often in an improvisational or flexible way); compromising (treating the cultural interaction as a give-and-take, recognizing when cultural preferences, values, practices, and customs conflict and embracing some and putting aside others in order to reduce conflict). LO: 11.5: Recognize how to engage in effective cross-cultural communication in organizations. AACSB: Diverse and multicultural work environments Difficulty: Moderate Quest. Category: Application 94) In cross cultural communication, ________ can lead to differences in interpretation of language. A) semantics B) deliberative processing C) selective perception D) controlled processing E) filtering Answer: A Explanation: In cross cultural communication, semantics can lead to differences in interpretation of language. Words mean different things to different people, particularly people from different cultures. For example, "pena ajena" in Spanish and "fremdschämen" in German mean "to be embarrassed for someone" but are essentially untranslatable into English (although English speakers on the Internet may have started using "cringe" to describe this situation). LO: 11.5: Recognize how to engage in effective cross-cultural communication in organizations. AACSB: Diverse and multicultural work environments Difficulty: Moderate Quest. Category: Concept
513 richard@qwconsultancy.com
95) Prior to communicating with people from a different culture, you should ________. A) learn from misunderstandings B) proactively maintain the identity of the group C) proactively maintain the culture of the group D) consider the other person's viewpoint E) know yourself Answer: E Explanation: Fred Casmir, a leading expert in intercultural communication research recommends taking the following steps to improve communication with people from a different culture: Prior to the interaction. 1) Know yourself. 2) Foster a climate of mutual respect, fairness, and democracy. During the interaction. 3) Consider the other person's viewpoint. 4) Learn from misunderstandings. After the interaction, 5) Proactively maintain the identity and culture of the group. 6) Learn from all intercultural interactions. LO: 11.5: Recognize how to engage in effective cross-cultural communication in organizations. AACSB: Diverse and multicultural work environments Difficulty: Moderate Quest. Category: Concept 96) After a cross cultural interaction, managers should ________. A) consider the other person's viewpoint B) foster a climate of mutual respect, fairness, and democracy C) learn from misunderstandings D) learn from all intercultural interactions E) know themselves Answer: D Explanation: Fred Casmir, a leading expert in intercultural communication research recommends taking the following steps to improve communication with people from a different culture: Prior to the interaction. 1) Know yourself. 2) Foster a climate of mutual respect, fairness, and democracy. During the interaction. 3) Consider the other person's viewpoint. 4) Learn from misunderstandings. After the interaction, 5) Proactively maintain the identity and culture of the group. 6) Learn from all intercultural interactions. LO: 11.5: Recognize how to engage in effective cross-cultural communication in organizations. AACSB: Diverse and multicultural work environments Difficulty: Moderate Quest. Category: Concept
514 richard@qwconsultancy.com
97) Rhonda May was transferred to London three months back to oversee the opening of a new branch of the retail chain she worked for. Rhonda, who had never been to London, adjusted well with her British colleagues but received quizzical looks from many of her British colleagues every morning in the initial weeks in London. After asking one of her subordinates to explain what caused her colleagues to look at her in that strange way, she discovered the reason. When her British colleagues greeted her by saying, "How are you today?" every morning, she typically replied by saying "I'm good," while Britishers typically say, "I'm well" in response to this greeting. This is because "I'm good" can have multiple meanings like "I'm not a bad person" or "I'm suitable," while "I'm well" only gives a sense of mental and physical well-being. Rhonda then realized how subtly cultural barriers can operate. From the information provided in the scenario, this is an example of barriers caused by ________. A) tone differences B) semantics and connotations C) intolerance toward other cultures D) situational and physical cues E) power distance Answer: B Explanation: Barriers between cultures can be caused by many factors. Here, semantics and connotation differences seem to be causing the miscommunication. Barriers caused by semantics reflect that words mean different things to different people, particularly people from different national cultures. Barriers caused by word connotations are a result of the fact that words imply different things in different languages. LO: 11.5: Recognize how to engage in effective cross-cultural communication in organizations. AACSB: Diverse and multicultural work environments Difficulty: Hard Employability Skills: Knowledge Application and Analysis Quest. Category: Application 98) Low-context cultures primarily rely on ________ in communicating with others. A) formal titles B) spoken and written words C) situational cues D) status differences E) nonverbal cues Answer: B Explanation: In high-context cultures, people rely heavily on nonverbal and subtle situational cues in communicating with others, and a person's official status, place in society, and reputation carry considerable weight. What is not said may be more significant than what is said. In contrast, in low-context cultures, people rely essentially on spoken and written words to convey meaning; body language and formal titles are secondary. LO: 11.5: Recognize how to engage in effective cross-cultural communication in organizations. AACSB: Diverse and multicultural work environments Difficulty: Moderate Quest. Category: Concept
515 richard@qwconsultancy.com
99) Communication in high-context cultures is characterized by an emphasis on ________. A) directness B) explicit and precise documentation C) trust D) legalized contracts over oral ones E) direct modes of conflict resolution Answer: C Explanation: Communication in high-context cultures implies considerably more trust by both parties. Oral agreements imply strong commitments in high-context cultures. In low-context cultures, enforceable contracts tend to be in writing, precisely worded, and highly legalistic. Similarly, low-context cultures value directness. LO: 11.5: Recognize how to engage in effective cross-cultural communication in organizations. AACSB: Diverse and multicultural work environments Difficulty: Moderate Quest. Category: Concept 100) In the different approaches to cross-cultural interactions, avoiding involves less adoption of one's own culture. Answer: TRUE Explanation: Avoiding involves putting aside cultural preferences, values, practices, or customs, often for tactical or strategic reasons. LO: 11.5: Recognize how to engage in effective cross-cultural communication in organizations. AACSB: Diverse and multicultural work environments Difficulty: Moderate Quest. Category: Concept 101) Less "media rich" channels may be more detrimental to people from high-context cultures. Answer: TRUE Explanation: Less "media rich" channels may be more detrimental to people from high-context cultures who rely heavily on subtle context cues such as nonverbal behavior. LO: 11.5: Recognize how to engage in effective cross-cultural communication in organizations. AACSB: Diverse and multicultural work environments Difficulty: Moderate Quest. Category: Concept
516 richard@qwconsultancy.com
102) Word connotations matter in cross-cultural communications because the same word can imply different things in different languages. Answer: TRUE Explanation: Word connotations matter in cross-cultural communications because the same word can imply different things in different languages. Negotiations between U.S. and Japanese executives can be difficult because the Japanese word hai translates as "yes," but its connotation is "Yes, I'm listening" rather than "Yes, I agree." Although research suggests that the negative perceptions of someone making an error in word usage or connotation are diminished when one finds out they are from a different culture, this is not the case for etiquette violations. As such, misinterpretations from wording and language might be much less impactful than those involving norms and etiquette. What might an etiquette mishap look like? It could be the use of a taboo word, which can be distracting and offensive to the person for whom the word is culturally taboo. LO: 11.5: Recognize how to engage in effective cross-cultural communication in organizations. AACSB: Diverse and multicultural work environments Difficulty: Moderate Quest. Category: Concept 103) Etiquette violations are usually overlooked when one finds out the person is from a different culture. Answer: FALSE Explanation: Although research suggests that the negative perceptions of someone making an error in word usage or connotation are diminished when one finds out they are from a different culture, this is not the case for etiquette violations. As such, misinterpretations from wording and language might be much less impactful than those involving norms and etiquette. What might an etiquette mishap look like? It could be the use of a taboo word, which can be distracting and offensive to the person for whom the word is culturally taboo. LO: 11.5: Recognize how to engage in effective cross-cultural communication in organizations. AACSB: Diverse and multicultural work environments Difficulty: Moderate Quest. Category: Concept 104) Tone differences can lead to emotional misconceptions during cross-cultural interactions. Answer: TRUE Explanation: Tone differences can lead to emotional misconceptions during cross-cultural interactions. In some cultures, language is formal; in others, it is informal. The tone might change depending on the context: People may speak differently at home and in social situations than at work. Using a personal, informal style when a more formal style is expected might be seen as inappropriate. LO: 11.5: Recognize how to engage in effective cross-cultural communication in organizations. AACSB: Diverse and multicultural work environments Difficulty: Moderate Quest. Category: Concept
517 richard@qwconsultancy.com
105) Explain how tone differences can lead to emotional misconceptions during cross-cultural interactions. Answer: Tone differences can lead to emotional misconceptions during cross-cultural interactions. In some cultures, language is formal; in others, it is informal. The tone might change depending on the context: People may speak differently at home and in social situations than at work. Using a personal, informal style when a more formal style is expected might be seen as inappropriate. For example, Mexican employees are relatively formal in their dress and communication in the tire industry, compared with other countries—they also tend to use the formal version of you (usted) in the workplace rather than the informal version (tu). LO: 11.5: Recognize how to engage in effective cross-cultural communication in organizations. AACSB: Diverse and multicultural work environments Difficulty: Moderate Quest. Category: Concept 106) Describe the five approaches to cross-cultural interaction. Answer: The five approaches to cross-cultural interaction are: avoiding (putting aside cultural preferences, values, practices, or customs often for tactical or strategic reasons); imposing (asserting one's own cultural preferences, values, practices, and customs without acknowledging others' cultural approaches); embracing (putting aside one's own cultural preferences, values, practices, and customs in order to acknowledge or embrace others' cultural approaches); synergizing (celebrating both interaction partners' cultural preferences, values, practices, and customs, often in an improvisational or flexible way); compromising (treating the cultural interaction as a give-and-take, recognizing when cultural preferences, values, practices, and customs conflict and embracing some and putting aside others in order to reduce conflict). LO: 11.5: Recognize how to engage in effective cross-cultural communication in organizations. AACSB: Diverse and multicultural work environments Difficulty: Moderate Quest. Category: Concept 107) When communicating with people from a different culture, how can you reduce misinterpretations? Answer: Fred Casmir, a leading expert in intercultural communication research recommends taking the following five steps to improve communication with people from a different culture: 1) Know yourself. 2) Foster a climate of mutual respect, fairness, and democracy. 3) Consider the other person's viewpoint. 4) Learn from misunderstandings. 5) Proactively maintain the identity and culture of the group. LO: 11.5: Recognize how to engage in effective cross-cultural communication in organizations. AACSB: Diverse and multicultural work environments Difficulty: Moderate Quest. Category: Concept
518 richard@qwconsultancy.com
108) Distinguish between high-context and low-context cultures. Answer: Cultures tend to differ in the degree to which context influences the meaning individuals take from communication. In high-context cultures such as China, Korea, Japan, and Vietnam, people rely heavily on nonverbal and subtle situational cues in communicating with others, and a person's official status, place in society, and reputation carry considerable weight. What is not said may be more significant than what is said. In contrast, people from Europe and North America reflect their low-context cultures. They rely essentially on spoken and written words to convey meaning; body language and formal titles are secondary. LO: 11.5: Recognize how to engage in effective cross-cultural communication in organizations. AACSB: Diverse and multicultural work environments Difficulty: Moderate Quest. Category: Concept 109) In high-context cultures, directness is valued more than trust. Answer: FALSE Explanation: Communication in high-context cultures implies considerably more trust by both parties. On the other hand, low-context cultures value directness. LO: 11.6: Recognize how to engage in effective cross-cultural communication in organizations. AACSB: Diverse and multicultural work environments Difficulty: Easy Quest. Category: Concept 110) Explain how managers can reduce misinterpretations when communicating with people from a different culture. Answer: Experts suggest that when communicating with people from a different culture, managers should know themselves and their culture identity and biases; foster a climate of mutual respect, fairness, and democracy; learn the cultural context of each person; when in doubt, listen rather than speaking their own opinions too early; state facts, not interpretations; consider the other person's viewpoint; and proactively maintain the identity of the group. LO: 11.6: Recognize how to engage in effective cross-cultural communication in organizations. AACSB: Diverse and multicultural work environments Difficulty: Moderate Quest. Category: Synthesis Organizational Behavior, 19e (Robbins/Judge) Chapter 12 Leadership 1) Which of the following best describes leadership? A) The ability to merely project one's abilities in the lack of actual accomplishments B) The ability to reduce the dependence of team members on each other C) The ability to induce the team members to focus on individual goals rather than collective goals D) The ability to influence a group toward the achievement of a vision or set of goals E) The ability to use factors like training and experience to reduce dependence on formal leadership Answer: D Explanation: Leadership can be defined as the ability to influence a group toward the 519 richard@qwconsultancy.com
achievement of a vision or set of goals. Leaders can emerge from within a group as well as by formal appointment. LO: 12.1: Summarize the conclusions of trait theories of leadership. Difficulty: Easy Quest. Category: Concept 2) Which of the following statements regarding leadership is true? A) All managers are leaders. B) Formal rights ensure good leadership. C) Formal appointment is essential in creating leaders. D) All leaders are hierarchically superior to followers. E) Nonsanctioned leadership is sometimes more important than formal influence. Answer: E Explanation: Not all leaders are managers, nor are all managers leaders. Nonsanctioned leadership—the ability to influence that arises outside the formal structure of the organization— is often as important as or more important than formal influence. LO: 12.1: Summarize the conclusions of trait theories of leadership. Difficulty: Moderate Quest. Category: Concept
520 richard@qwconsultancy.com
3) A desirable feature of leadership is ________. A) one-directional influence from the leader to the follower B) coercive power and authority C) lack of freedom D) passivity of followers E) coexistence of leaders and managers Answer: E Explanation: Organizations need strong leadership and strong management for optimal effectiveness. We need leaders today to challenge the status quo, create visions of the future, and inspire organizational members to want to achieve the visions. We also need managers to formulate detailed plans, create efficient organizational structures, and oversee day-to-day operations. LO: 12.1: Summarize the conclusions of trait theories of leadership. Difficulty: Moderate Quest. Category: Concept 4) Trait theories of leadership focus on ________. A) the special relationship that leaders establish with a small group of their followers B) the personal qualities and characteristics that differentiate leaders from nonleaders C) the way the leader makes decisions D) the extent to which followers are willing and able to accomplish a specific task E) the match between the leader's style and the degree to which the situation gives the leader control Answer: B Explanation: Trait theories of leadership focus on personal qualities and characteristics. The search for personality, social, physical, or intellectual attributes that differentiate leaders from nonleaders goes back to the earliest stages of leadership research. LO: 12.1: Summarize the conclusions of trait theories of leadership. Difficulty: Easy Quest. Category: Concept 5) Researchers have consistently found ________ to be the strongest predictor of motivation to lead and leader emergence. A) conscientiousness B) openness C) extroversion D) agreeableness E) emotional stability Answer: C Explanation: In examining the Big Five personality traits, researchers have consistently found extroversion to be important for several leadership outcomes. Despite being the strongest predictor of motivation to lead and leader emergence, extroversion also predicts several leadership behaviors or styles. LO: 12.1: Summarize the conclusions of trait theories of leadership. Difficulty: Moderate Quest. Category: Concept 521 richard@qwconsultancy.com
6) Emotional intelligence (EI) is critical to effective leadership because one of its core components is ________, which reflects the consideration that leaders must be able to express. A) conscientiousness B) empathy C) optimism D) introversion E) perfectionism Answer: B Explanation: A core component of EI is empathy. Empathetic leaders can sense others' needs, listen to what followers say (and don't say), and read the reactions of others. The caring part of empathy, especially for the people with whom you work, is what inspires people to stay with a leader when the going gets rough. LO: 12.1: Summarize the conclusions of trait theories of leadership. Difficulty: Moderate Quest. Category: Concept 7) Which of the following do trait theories most accurately predict? A) Distinguishing features of an effective leader B) Differences between an effective and an ineffective leader C) Success of a leader D) Roles to be played by the leader E) Emergence of a leader Answer: E Explanation: Traits are more accurate in predicting the emergence of leaders and the appearance of leadership than actually distinguishing between effective and ineffective leaders. LO: 12.1: Summarize the conclusions of trait theories of leadership. Difficulty: Moderate Quest. Category: Concept
522 richard@qwconsultancy.com
8) Tim Wrench was leading the client services division of AmWeb for seven years when he was asked to move to another region where the company was setting up its office. Before moving, Tim was asked to help in finding a successor for him from his team. Tim's most obvious choice was Judy Judge, and the management accepted his choice as Judy was a popular person across the company. Judy was known for her vivacious nature, was often seen speaking to employees from various divisions, and was always excited to take up a new opportunity. Once she became a leader, she continued to give employees freedom and flexibility even if it resulted in deficiencies on the work front like missed deadlines or low quality. Judy's initial weeks as a leader were full of confusion among her team members, but many felt that the situation would come under control. When things did not improve in the next two months and many complaints poured in from clients, the management realized that Judy was not the best candidate to lead the team. Which of the following, if true, would best explain this outcome? A) Judy was a high-performing employee and enjoyed good relationships with everyone on the team. B) Everyone felt Judy was given insufficient time to prove her worth as a leader. C) Research has shown that traits can predict the emergence of a leader, but not his or her efficiency as a leader. D) The client complaints were a common thing encountered by the company. E) Judy demonstrated low levels of consideration and high levels of task-orientation. Answer: C Explanation: The fact that Judy was a high-performing employee and enjoyed good relationships with everyone on the team does not explain this outcome. The fact that everyone felt that Judy was given insufficient time to prove her worth as a leader does not explain this outcome. The fact that research has shown that traits can predict the emergence of a leader, but not his or her efficiency as a leader does explain this outcome. Traits are more accurate in predicting the emergence of leaders and the appearance of leadership than actually distinguishing between effective and ineffective leaders. The fact that client complaints were a common thing encountered by the company does not explain Judy's failure as a leader. The fact that Judy demonstrated low levels of consideration and high levels of task-orientation as a leader does not explain her failure in leading the team. LO: 12.1: Summarize the conclusions of trait theories of leadership. AACSB: Analytical thinking Difficulty: Hard Employability Skills: Critical Thinking Quest. Category: Critical Thinking 9) Because extroverted leaders are more likely to use transformational leadership styles and consideration behaviors, they are likely to be considered more effective leaders Answer: TRUE Explanation: Because extroverted leaders are more likely to use transformational leadership styles and consideration behaviors, they are likely to be considered more effective leaders. LO: 12.1: Summarize the conclusions of trait theories of leadership. Difficulty: Moderate Quest. Category: Concept
523 richard@qwconsultancy.com
10) Trait theories of leadership help us predict who emerges as leaders, but they do not fully explain leadership actions. Answer: TRUE Explanation: Trait theories help us predict who emerges as leaders (and perhaps who will be effective leaders), but they do not fully explain leadership actions. LO: 12.1: Summarize the conclusions of trait theories of leadership. Difficulty: Moderate Quest. Category: Concept 11) Discuss the role of traits in predicting leadership behaviors. Answer: Studies have shown that the extroversion dimension of the Big Five personality model is an important trait of effective leaders, along with conscientiousness and openness to experience. Empathy, one of the core components of emotional intelligence, is also closely linked to effective leadership. The two major conclusions about trait theories and leadership are that (1) traits can predict leadership and (2) traits do a better job predicting the emergence of leaders than distinguishing between effective and ineffective leaders. LO: 12.1: Summarize the conclusions of trait theories of leadership. Difficulty: Moderate Quest. Category: Concept 12) The Ohio State Studies narrowed the independent dimensions of leader behavior to two that substantially accounted for most of the leadership behavior described by employees: consideration and ________. A) employee-orientation B) empathy C) constructing vision D) initiating structure E) charisma Answer: D Explanation: Seeking to identify independent dimensions of leader behavior, the Ohio State Studies determined that two dimensions accounted for most effective leadership behavior: initiating structure and consideration. Initiating structure is the extent to which leaders are likely to define and structure their roles and those of their employees in the search for goal attainment. Consideration is the extent to which a leader's job relationships are characterized by mutual trust, respect for employees' ideas, and regard for their feelings. LO: 12.2: Identify the central tenets and main limitations of behavioral theories of leadership. Difficulty: Moderate Quest. Category: Concept
524 richard@qwconsultancy.com
13) In the context of behavioral dimensions of leadership identified in the Ohio State Studies, initiating structure refers to the extent to which ________. A) a person's job relationships are characterized by mutual trust, respect for employees' ideas, and regard for their feelings B) a leader engages in participative management C) a leader is accepting of and respects individual differences among various team members D) a leader is likely to define and organize his or her role and those of employees in the search for goal attainment E) a leader initiates efforts to communicate personally with employees Answer: D Explanation: As a behavioral dimension of leaders, initiating structure is the extent to which a leader is likely to define and structure his or her role and those of employees in the search for goal attainment. It includes behavior that attempts to organize work, work relationships, and goals. LO: 12.2: Identify the central tenets and main limitations of behavioral theories of leadership. Difficulty: Moderate Quest. Category: Concept 14) Adrian Atwood, a senior manager at MNC, spends a lot of his time assigning group members to particular tasks and scheduling their work such that deadlines are achievable. Adrian also sets high expectations for standards of performance and holds regular meetings to ensure that productivity and quality are up to the mark. In the light of the Ohio State Studies, this indicates that Adrian, as a leader, is ________. A) low in task orientation B) high in consideration C) relationship oriented D) employee oriented E) high in initiating structure Answer: E Explanation: Initiating structure is the extent to which a leader is likely to define and structure his or her role and those of employees in the search for goal attainment. It includes behavior that attempts to organize work, work relationships, and goals. A leader high in initiating structure is someone who assigns group members to particular tasks, expects workers to maintain definite standards of performance, and emphasizes the meeting of deadlines. LO: 12.2: Identify the central tenets and main limitations of behavioral theories of leadership. AACSB: Analytical thinking Difficulty: Moderate Employability Skills: Knowledge Application and Analysis Quest. Category: Application
525 richard@qwconsultancy.com
15) In the context of behavioral dimensions of leadership identified in the Ohio State Studies, ________ is the extent to which a person's job relationships are characterized by mutual trust, respect for employees' ideas, and regard for their feelings. A) consideration B) transaction C) authentication D) task orientation E) identification Answer: A Explanation: In the context of behavioral dimensions of leadership identified in the Ohio State Studies, consideration is the extent to which a person's job relationships are characterized by mutual trust, respect for employees' ideas, and regard for their feelings. LO: 12.2: Identify the central tenets and main limitations of behavioral theories of leadership. Difficulty: Easy Quest. Category: Concept 16) Nellie Fritz, the head of client support services at Olson Inc., is very popular among her subordinates. Many believe that Nellie has a knack for getting the work done without making the employees feel pushed into a corner. She is often seen speaking to her subordinates and support staff about their families, helping them with any personal problems they have, and praising employees for their good work. In light of the Ohio State Studies, this indicates that Nellie, as a leader, is ________. A) task oriented B) high in consideration C) low in trust propensity D) low in relationship orientation E) production oriented Answer: B Explanation: Consideration is the extent to which a person's job relationships are characterized by mutual trust, respect for employees' ideas, and regard for their feelings. A leader high in consideration helps employees with personal problems, is friendly and approachable, treats all employees as equals, and expresses appreciation and support. LO: 12.2: Identify the central tenets and main limitations of behavioral theories of leadership. AACSB: Analytical thinking Difficulty: Moderate Employability Skills: Knowledge Application and Analysis Quest. Category: Application
526 richard@qwconsultancy.com
17) The Ohio State Studies started with more than a thousand dimensions of leader behavior. Answer: TRUE Explanation: The most comprehensive behavioral theories of leadership resulted from the Ohio State Studies, which sought to identify independent dimensions of leader behavior. Beginning with more than a thousand dimensions, the studies narrowed the list to two that substantially accounted for most of the leadership behavior described by employees: initiating structure and consideration. LO: 12.2: Identify the central tenets and main limitations of behavioral theories of leadership. Difficulty: Moderate Quest. Category: Concept 18) The Ohio State Studies narrowed a list of more than a thousand dimensions to just one that accounted for most of the leadership behavior described by employees: participation. Answer: FALSE Explanation: The most comprehensive behavioral theories of leadership resulted from the Ohio State Studies, which sought to identify independent dimensions of leader behavior. Beginning with more than a thousand dimensions, the studies narrowed the list to two that substantially accounted for most of the leadership behavior described by employees: initiating structure and consideration. LO: 12.2: Identify the central tenets and main limitations of behavioral theories of leadership. Difficulty: Moderate Quest. Category: Concept 19) Trait theories imply that we can train people to be leaders. Answer: FALSE Explanation: Trait theories provide a basis for selecting the right people for leadership. Behavior theories of leadership imply we can train people to be leaders. LO: 12.2: Identify the central tenets and main limitations of behavioral theories of leadership. Difficulty: Moderate Quest. Category: Concept
527 richard@qwconsultancy.com
20) Using the dimensions of initiating structure and consideration, discuss mentoring. Answer: Initiating structure is the extent to which a leader is likely to define and structure his or her role and those of employees in the search for goal attainment. It includes behavior that attempts to organize work, work relationships, and goals. Consideration is the extent to which a person's job relationships are characterized by mutual trust, respect for employees' ideas, and regard for their feelings. A leader high in consideration helps employees with personal problems, is friendly and approachable, treats all employees as equals, and expresses appreciation and support. Mentoring is a process by which a senior employee sponsors and supports a lessexperienced employee or a protégé. Mentoring performs many career functions, like helping the new employee get challenging assignments, opportunities to develop skills, exposure to influential individuals in the company, and nominations for potential advantages. These career functions of mentoring are similar to the initiating structure of leadership. Mentoring also fulfills certain psychosocial functions, like counseling the employee to enhance confidence, providing friendship and acceptance, and providing the new employee with a role model. These are similar to the consideration dimension of leadership. LO: 12.2: Identify the central tenets and main limitations of behavioral theories of leadership. AACSB: Analytical thinking Difficulty: Moderate Quest. Category: Synthesis 21) Contingency theories focus on the ________ that impact leadership success. A) personality variables B) leader's abilities to inspire and transform followers C) situational dimensions D) values and ethics E) features of the followers Answer: C Explanation: Numerous studies have shown that predicting leadership success is more complex than isolating a few traits or behaviors, since leadership styles that are effective in very bad times or in very good times do not necessarily translate into long-term success. This idea led researchers to change their focus from trait and behavior theories to situational influences on leadership styles, or contingency theories. LO: 12.3: Contrast contingency theories of leadership. Difficulty: Moderate Quest. Category: Concept
528 richard@qwconsultancy.com
22) Which of the following theories of leadership is based on situational dimensions? A) Attribution theory B) Trait theories C) Charismatic leadership theory D) Fiedler model E) Behavioral theories Answer: D Explanation: Numerous studies have shown that predicting leadership success is more complex than isolating a few traits or behaviors, since leadership styles that are effective in very bad times or in very good times do not necessarily translate into long-term success. This idea led researchers to change their focus from trait and behavior theories to situational influences on leadership styles, or contingency theories. Fred Fiedler developed the first comprehensive contingency model for leadership. LO: 12.3: Contrast contingency theories of leadership. Difficulty: Moderate Quest. Category: Concept 23) The Fiedler contingency model proposes that group performance depends on the proper match between the leader's style and ________. A) whether followers are able and willing B) the degree to which the situation gives the leader control C) if the members are in the ingroup or out-group D) whether employees prefer servant leadership over situational leadership E) the degree to which the leader is high on the dimension of conscientiousness Answer: B Explanation: The Fiedler contingency model proposes that group performance depends on the proper match between the leader's style and the degree to which the situation gives the leader control. LO: 12.3: Contrast contingency theories of leadership. Difficulty: Moderate Quest. Category: Concept
529 richard@qwconsultancy.com
24) Despite being particularly annoyed by a difficult co-worker in the past, Rachel Chen recently described the co-worker to a colleague in relatively positive terms. Based on your understanding of the Fiedler contingency model what conclusion can you draw about Rachel? A) Rachel tends to become very dominating when given ambiguous tasks B) Rachel is usually much more focused on productivity than on developing relationships C) Rachel tends in general to focus on building good relationships with the other employees D) Rachel has a spotty work history and has tended to switch jobs every couple of years E) Rachel is usually chosen for positions of high responsibility within your organization Answer: C Explanation: If you describe a person you have difficulty working with in favorable terms, Fiedler would label you relationship-oriented. In contrast, if you see your co-worker in relatively unfavorable terms , you are primarily interested in productivity and are task-oriented. LO: 12.3: Contrast contingency theories of leadership. AACSB: Analytical thinking Difficulty: Moderate Quest. Category: Application 25) Fiedler's contingency model assumes that ________. A) an individual's leadership style is essentially fixed B) an individual is constantly striving to develop a more productive style C) an individual's leadership is primarily determined by the features of the followers D) an individual's leadership style can be altered much like his or her personality traits E) an individual's leadership style must be task oriented to be effective Answer: A Explanation: Fiedler assumes an individual's leadership style is fixed. This means if a situation requires a task-oriented leader and the person in the leadership position is relationship oriented, either the situation has to be modified or the leader has to be replaced to achieve optimal effectiveness. LO: 12.3: Contrast contingency theories of leadership. Difficulty: Moderate Quest. Category: Concept 26) Fiedler defines the degree of confidence, trust, and respect that subordinates have in their leader as ________. A) leader-member relations B) task structure C) positional power D) follower variables E) path-goal frameworks Answer: A Explanation: Fiedler identified three contingency or situational dimensions to leadership success, including leader-member relations, task structure, and position power. Leader-member relations is the degree of confidence, trust, and respect members have in their leader. LO: 12.3: Contrast contingency theories of leadership. Difficulty: Easy Quest. Category: Concept 530 richard@qwconsultancy.com
27) Trevor Guerney is a manager who believes that those who are to be affected by a change must be involved in the change. Consequently, he always ensures that his subordinates have the knowledge of what is happening around them, and he often holds meetings to obtain employee opinion and suggestions before making any decision that would apply to them. Similarly, Trevor's team proactively approaches him with problems and potential solutions as they know he will not respond by criticizing them. From the information provided in the scenario, we can say that ________. A) Trevor's leadership is primarily task-oriented B) Trevor's team has a low degree of trust propensity C) Trevor's team requires more directive leadership D) Trevor's team has positive leader-member relations E) Trevor's team is unsatisfied with his power position Answer: D Explanation: Fiedler identified three contingency or situational dimensions to leadership success, including leader-member relations, task structure, and position power. Leader-member relations is the degree of confidence, trust, and respect members have in their leader. LO: 12.3: Contrast contingency theories of leadership. AACSB: Analytical thinking Difficulty: Moderate Quest. Category: Application 28) Fiedler's ________ dimension relates to the degree to which job assignments are regimented, that is, structured or unstructured. A) task reflexivity B) task significance C) task structure D) task complexity E) task orientation Answer: C Explanation: Fiedler identified three contingency or situational dimensions in which certain types of leaders might excel or fail, including leader-member relations, task structure, and position power. Task structure is the degree to which job assignments are regimented (that is, structured or unstructured). Leaders who are task-oriented, Fiedler proposes, perform best when the situation is very favorable or very unfavorable. LO: 12.3: Contrast contingency theories of leadership. Difficulty: Easy Quest. Category: Concept
531 richard@qwconsultancy.com
29) According to the Fiedler contingency model, a very favorable situation in which the leader has a great deal of control is characterized by ________. A) high task structure, good leader-member relations, and strong position power B) limited position power, good leader-member relations, and low task structure C) less structured jobs, strong position power, and moderate leader-member relations D) broad employee responsibilities, low position power, and moderate leader-member relations E) good leader-member relations, low position power, unstructured jobs Answer: A Explanation: The Fiedler contingency model proposes that effective group performance depends on the proper match between the leader's style and the degree to which the situation gives the leader control. Fiedler states that the better the leader-member relations, the more highly structured the job, and the stronger the position power, the more control the leader has. LO: 12.3: Contrast contingency theories of leadership. Difficulty: Moderate Quest. Category: Concept 30) The ________ focuses on followers' readiness as a determinant of effective leadership. A) Big Five personality model B) behavioral theory C) Fiedler contingency model D) laissez-faire leadership E) situational leadership theory Answer: E Explanation: Situational leadership theory (SLT) focuses on the followers. It says successful leadership depends on selecting the right leadership style contingent on the followers' readiness, or the extent to which they are willing and able to accomplish a specific task. LO: 12.3: Contrast contingency theories of leadership. Difficulty: Moderate Quest. Category: Concept 31) According to the situational leadership theory, if employees are not quite competent, but are committed to doing the task, the appropriate leadership style in this situation would be ________. A) participative B) democratic C) person-oriented D) directive E) supportive Answer: D Explanation: According to the situational leadership theory, if employees are not quite competent, but are committed to doing the task, the appropriate leadership style in this situation would be directive. LO: 12.3: Contrast contingency theories of leadership. Difficulty: Easy Quest. Category: Concept
532 richard@qwconsultancy.com
533 richard@qwconsultancy.com
32) According to the situational leadership theory, a follower with the desired ability and willingness is likely to be ________. A) lacking in the area of motivation B) deficient in the specific skills required C) comfortable in his ability to do the job well D) highly skilled in relevant areas but low on experience E) in need of leadership that is highly task oriented Answer: C Explanation: If followers are able and unwilling, the leader needs to use a supportive and participative style; if they are both able and willing, the leader does not need to do much. LO: 12.3: Contrast contingency theories of leadership. Difficulty: Moderate Quest. Category: Concept 33) Which of the following statements is true with regard to leading in times of crisis? A) Emotions play only a very small role in leadership during crisis. B) Visionary leadership is ineffective and too far-reaching during time of crisis. C) During times of crisis, charismatic leadership can be visionary or crisis responsive. D) Transformational leadership used during time of crisis is generally ineffective following the crisis. E) Crisis responsive leadership starts with a vision and then a plea for action. Answer: C Explanation: During times of crisis, charismatic leadership can be visionary or crisis responsive. LO: 12.3: Contrast contingency theories of leadership. Difficulty: Moderate Quest. Category: Concept 34) Which dimension in the Fiedler model relates to the degree of influence a leader has over important variables such as hiring, firing, discipline, promotions, and salary increases? A) Task structure B) Leader-member exchange C) Position power D) Initiating structure E) Leader-member relations Answer: C Explanation: In the context of Fiedler's model, the situational dimension termed position power relates to the degree of influence a leader has over power variables such as hiring, firing, discipline, promotions, and salary increases. LO: 12.3: Contrast contingency theories of leadership. Difficulty: Easy Quest. Category: Concept
534 richard@qwconsultancy.com
35) According to the situational leadership theory, if followers are competent, but have variable commitment to do a task, then a leader needs to ________. A) follow a selling/coaching style B) engage in directing leadership C) refrain from providing too many inputs D) display a participative and supporting orientation E) demonstrate high task and low relationship orientation Answer: D Explanation: If followers are unable and willing, the leader needs to display high task orientation to compensate for followers' lack of ability and high relationship orientation to get them to commit to the leader's desires. LO: 12.3: Contrast contingency theories of leadership. Difficulty: Moderate Quest. Category: Concept 36) Maura Ruiz has been working in the e-learning industry for over eight years. She is aware of the fact that in this industry, once an employee has learned his or her job, the work becomes fairly automatic and competence can be achieved rapidly. She has been noticing how in her team, this competence has come along with a lot of complacency; the employees, though able, are unwilling to work hard. According to the situational leadership theory, to rectify this situation, Maura would benefit the most if she uses the ________ style of leadership. A) task-oriented B) production-oriented C) participative D) directive E) autocratic Answer: C Explanation: If followers are able and unwilling, the leader needs to use a supportive and participative style. LO: 12.3: Contrast contingency theories of leadership. AACSB: Analytical thinking Difficulty: Hard Employability Skills: Knowledge Application and Analysis Quest. Category: Application
535 richard@qwconsultancy.com
37) George has been a project leader at NSys for five years. George's job description involves scheduling work for his team, coordinating their work with that of the other departments, and providing feedback. George, who has successfully led this team, believes that it is his taskoriented and directive approach that has helped him in the last five years. Which of the following, if true, would weaken his argument supporting a directive leadership? A) The members of the team are resentful with George's directives relating to work. B) George has seen that employees in his team lack initiative and motivation to work hard. C) The team's workload is expected to rise substantially as the company is growing at an unprecedented rate. D) George's team is organized as a wheel network and depends on his ability to coordinate amongst members of the team and between other departments. E) NSys hires only highly qualified and experienced employees. Answer: E Explanation: The fact that the members of the team are resentful with George's directives relating to work does not strengthen or weaken his approach to leadership. The fact that George has seen that employees in his team lack initiative and motivation to work strengthens his argument for using a directive approach to leadership. The fact that the team's workload is expected to rise substantially as the company is growing at an unprecedented rate also strengthens his argument for using a directive approach to leadership. The fact that George's team is organized as a wheel network and depends on his ability to coordinate amongst members of the team and between other departments does not weaken his current approach to leadership. The fact that NSys hires only highly qualified and experienced employees does weaken his approach as directive leadership is likely to be perceived as redundant among employees with high ability or considerable experience. LO: 12.3: Contrast contingency theories of leadership. AACSB: Analytical thinking Difficulty: Hard Employability Skills: Critical Thinking Quest. Category: Critical Thinking
536 richard@qwconsultancy.com
38) Leslie is a middle-level production manager at the California branch of ALT Corp. ALT Corp. is an automobile manufacturing company that specializes in the manufacture of heavy motor vehicles. Leslie's job is to supervise his assembly line employees. Leslie has worked in this position for over four years, and he strongly believes that a supportive leadership style is most suitable in his context. Which of the following, if true, would strengthen Leslie's approach to leadership in this case? A) The company has recently announced its move to diversify into manufacturing light motor vehicles because of high profitability in that line. B) Most of the assembly line employees are highly experienced in their jobs and committed to Leslie. C) With developments in technology, the company was enhancing the role of assembly line employees with new tasks combined into natural work units. D) The company has initiated many changes in the tasks of assembly line workers in its attempts to refine the process of manufacturing. E) A good proportion of the assembly line employees are new employees. Answer: B Explanation: The fact that the company recently announced its move to diversify into manufacturing light motor vehicles does not strengthen Leslie's approach to leadership. The fact that most of the assembly line employees are highly experienced in their jobs and committed to Leslie does strengthen Leslie's approach to leadership as the employees are performing structured tasks. The fact that the company is enhancing the role of assembly line employees with new tasks does not strengthen Leslie's approach to leadership. The fact that the company has initiated many changes in the tasks of assembly line workers in its attempts to refine the process of manufacturing weakens Leslie's argument. The fact that a good proportion of the assembly line employees are new employees weakens Leslie's argument. LO: 12.3: Contrast contingency theories of leadership. AACSB: Analytical thinking Difficulty: Hard Employability Skills: Critical Thinking Quest. Category: Critical Thinking
537 richard@qwconsultancy.com
39) Leslie is a middle-level production manager at the California branch of ALT Corp. ALT Corp. is an automobile manufacturing company that specializes in the manufacture of heavy motor vehicles. Leslie's job is to supervise his assembly line employees. Leslie has worked in this position for over four years, and he strongly believes that a supportive leadership style is most suitable in his context. Which of the following, if true, would weaken Leslie's approach to leadership in this case? A) The company is planning to lay off some employees in the near future. B) The company has stopped the production of most of its older models as it wants to focus on the most profitable models. C) The assembly line jobs are routine and highly structured by nature. D) The recent round of OSHA inspections revealed that many assembly line employees were not complying with stipulated safety measures. E) The company's workforce is comprised primarily of employees of the baby boomer generation. Answer: D Explanation: The fact that the company is planning to lay off some employees in the near future does not weaken Leslie's supportive approach to leadership. The fact that the company has stopped the production of most of its older models as it wants to focus on the most profitable models does not weaken Leslie's supportive approach to leadership; it may strengthen his approach as the employees, being familiar with the process for assembling these models, would require less directions and supervision. The fact that assembly line jobs are routine and highly structured by nature strengthens Leslie's supportive approach to leadership. The fact that the recent round of OSHA inspections revealed that many assembly line employees were not complying with stipulated safety measures weakens Leslie's supportive approach. This is because the assembly line employees seem to require someone who would ensure their compliance with safety regulations. The fact that the company's workforce is comprised primarily of employees of the baby boomer generation strengthens Leslie's approach as this generation of employees would be highly experienced in their jobs. LO: 12.3: Contrast contingency theories of leadership. AACSB: Analytical thinking Difficulty: Hard Employability Skills: Critical Thinking Quest. Category: Critical Thinking
538 richard@qwconsultancy.com
40) Max Hiller was recently hired by Sync, a consumer goods company. During his first meeting with the sales team, Max impressed upon his team that work performance is the only criterion he would use to evaluate them. To help them perform well and meet their targets, he pushed his team to work extra hours. He also gave very clear instructions to each member regarding their job responsibilities and continually verified if they were meeting their targets. Which of the following, if true, would weaken Max's approach? A) Max is leading many new employees who have joined his team directly after training. B) Max's team functions in a sluggish manner and picks up pace only a week or so before the monthly operations cycle meetings. C) Max's sales team is comprised of independent and experienced employees who are committed to their jobs. D) Max's team does not display high levels of cohesiveness, and members fail to coordinate with each other. E) Sales figures for the region that Max's team is responsible for have improved in the last quarter. Answer: C Explanation: The fact that Max is leading many new employees who have joined his team directly after training strengthens his directive approach. The fact that Max's team functions in a sluggish manner and picks up pace only a week or so before the monthly operations cycle meetings also strengthens his directive approach. The fact that Max's sales team is comprised of independent and experienced employees who are committed to their jobs weakens his directive approach in leading them. This is because directive leadership is likely to be perceived as redundant among employees with high ability or considerable experience. The fact that Max's team does not display high levels of cohesiveness and members fail to coordinate with each other shows that the team requires a directive leader. The fact that sales figures for the region that Max's team is responsible for have improved in the last quarter does not strengthen or weaken Max's approach as he has been hired only recently and the rise in revenue for this region could be attributed to many other factors. LO: 12.3: Contrast contingency theories of leadership. AACSB: Analytical thinking Difficulty: Hard Employability Skills: Critical Thinking Quest. Category: Critical Thinking 41) The leader-participation model focuses on ________. A) the personality traits of leaders B) the presence of charisma in the leader C) the role followers play D) the mere projection of successful leadership by the leader E) the use of ingroups by the leader Answer: C Explanation: The leader-participation model relates leadership behavior to subordinate participation in decision making. LO: 12.3: Contrast contingency theories of leadership. Difficulty: Moderate Quest. Category: Concept 539 richard@qwconsultancy.com
42) In the context of the Fiedler contingency model, task structure measures the degree to which the job assignments are structured or unstructured. Answer: TRUE Explanation: In the context of the Fiedler contingency model, task structure measures the degree to which the job assignments are structured or unstructured. LO: 12.3: Contrast contingency theories of leadership. Difficulty: Easy Quest. Category: Concept 43) The situational leadership theory focuses on follower readiness to determine the appropriate leadership behavior. Answer: TRUE Explanation: Situational leadership theory (SLT) focuses on the followers. It says successful leadership depends on selecting the right leadership style contingent on the followers' readiness, or the extent to which they are willing and able to accomplish a specific task. LO: 12.3: Contrast contingency theories of leadership. Difficulty: Easy Quest. Category: Concept 44) According to the shared leadership theory, leadership can become an emergent state in which leadership roles are distributed across followers, and all are capable of influencing one another. Answer: TRUE Explanation: According to the shared leadership theory, leadership can become an emergent state in which leadership roles are distributed across followers, and all are capable of influencing one another. LO: 12.3: Contrast contingency theories of leadership. Difficulty: Easy Quest. Category: Concept 45) Using followership theory, explain the role of followers as it relates to the success of leadership. Answer: A key element of followership theory is perception. People's perceptions of what makes for ideal follower traits and behavior matter in the same way they do for leaders. Research suggests that the ideal follower is an industrious, enthusiastic, "good citizen" or "team player." These followers can be described as hardworking, productive, excited, outgoing, happy, loyal, and reliable. They are also more likely to engage in organizational citizenship behaviors (OCBs)—going above and beyond. A poor follower is one who is prone to insubordination, incompetence, and conformity. Followers who are easily influenced, follow trends, and do not speak up when the time calls for it can lead to stagnation in the group. Furthermore, arrogant, rude, or ill-tempered followers can cause or exacerbate relationship conflict that gets in the way of the vision or goals of the group. Lastly, if the followers are not equipped to make the dream a reality (e.g., they lack the knowledge, skills, abilities, or experiences required), they can be a detriment to the leader's vision. LO: 12.3: Contrast contingency theories of leadership. Difficulty: Moderate 540 richard@qwconsultancy.com
Quest. Category: Concept 46) Describe the shared leadership theory and when it is most likely to emerge and when it is effective. Answer: Shared leadership is a theory that suggests leadership can become an emergent state in which leadership roles are distributed across followers and all are capable of influencing one another. Shared leadership is possible in teams and groups with a strongly held shared purpose, flexible roles, composed of proactive and functionally diverse members and in an environment characterized by social support, autonomy, and psychological safety. It is less likely to occur directly after a team forms (when members are still navigating the new structure) and toward the end of a team's tenure (when teamwork and processes are "wrapping up"). Overall, shared leadership is effective at improving team performance, attitudes, and behaviors, especially when members engage in transformational or charismatic practices and when the team tasks are complex. Shared leadership can also lead to creativity and innovation benefits, as followers are more likely to support one another during the idea generation process, derive a sense of purpose and meaning from working together creatively, and be well equipped to make these ideas a reality. LO: 12.3: Contrast contingency theories of leadership. Difficulty: Moderate Quest. Category: Synthesis 47) Which of the following leadership theories argues that leaders establish a special relationship with a small group of their subordinates—the ingroup, who are trusted, get a disproportionate amount of the leader's attention, and are more likely to receive special privileges? A) Situational leadership theory B) Leader-member exchange C) Path-goal D) Expectancy E) Fiedler's theory Answer: B Explanation: Leader-member exchange (LMX) theory proposes that leaders establish a special relationship with a small group of their followers. These individuals make up the ingroup. Members of the ingroup are trusted, get a disproportionate amount of the leader's attention, and are more likely to receive special privileges. Other followers fall into the out-group. LO: 12.4: Describe the positive leadership styles and relationships. Difficulty: Moderate Quest. Category: Concept
541 richard@qwconsultancy.com
48) Which is true of the LMX theory? A) Employees who are proactive tend to be placed in the outgroup. B) Employees tend to value LMX equally. C) Leaders and followers build social capital with one another. D) Leaders and followers of different genders tend to have lower LMX relationships. E) Competent employees tend to have lower-quality relationships with their leaders. Answer: C Explanation: When leaders and followers interact with one another, research suggests that leaders and followers build social capital with one another–they contribute more to the relationship when they feel obligated to and will withhold contribution when they believe they have "done enough." LO: 12.4: Describe the positive leadership styles and relationships. Difficulty: Moderate Quest. Category: Concept 49) In her first few weeks at the marketing division of Rolland Retails, Judith Cox realized that Joshua, Doug, and Carl were closer to her manager, Eric Scott, than the other five team members. Eric, Joshua, Doug, and Carl came to work at the same time, were seen together at the cafeteria, and stayed late and worked when the need arose. In contrast to them, the other five team members did the routine jobs assigned to them, and their interactions lacked the understanding and camaraderie that Eric shared with the others. Joshua, Doug, and Carl make up Eric's ________. A) out-group B) reference group C) nominal group D) ingroup E) peripheral group Answer: D Explanation: Leader-member exchange (LMX) theory argues that, leaders establish a special relationship with a small group of their followers. These individuals make up the ingroup; they are trusted, get a disproportionate amount of the leader's attention, and are more likely to receive special privileges. Other followers fall into the out-group. LO: 12.4: Describe the positive leadership styles and relationships. AACSB: Analytical thinking Difficulty: Moderate Employability Skills: Knowledge Application and Analysis Quest. Category: Application
542 richard@qwconsultancy.com
50) In her first few weeks at the marketing division of Rolland Retails, Judith Cox realized that Joshua, Doug, and Carl were closer to her manager, Eric Scott, than the other five team members. Eric, Joshua, Doug, and Carl came to work at the same time, were seen together at the cafeteria, and stayed late and worked when the need arose. While Judith was in training, she received very good feedback from Eric, and as she transitioned to the floor, she felt that Eric was giving her interesting projects, allowing her more freedom, and seeking her opinion frequently. The information provided in the scenario supports the prediction that ________. A) Judith will develop low trust propensity toward Eric B) Joshua, Doug, and Carl will display low trust propensity in Judith C) Eric's ingroup will remain a reference group for Judith permanently D) Judith will become a part of Eric's ingroup in the marketing division E) Judith will have lower levels of identification-based trust with Eric when compared to other trainees Answer: D Explanation: Leader-member exchange (LMX) theory argues that leaders establish a special relationship with a small group of their followers. These individuals make up the ingroup, and they are trusted, get a disproportionate amount of the leader's attention, and are more likely to receive special privileges. Other followers fall into the out-group. LO: 12.4: Describe the positive leadership styles and relationships. AACSB: Analytical thinking Difficulty: Moderate Employability Skills: Knowledge Application and Analysis Quest. Category: Application 51) The ________ proposes that followers attribute heroic or extraordinary leadership abilities when they observe certain behaviors. A) situational leadership theory B) path-goal leadership theory C) transactional leadership theory D) behavioral leadership theory E) charismatic leadership theory Answer: E Explanation: According to charismatic leadership theory, followers attribute heroic or extraordinary leadership abilities when they observe certain behaviors. A number of studies have attempted to identify the characteristics of charismatic leaders: they have a vision, they are willing to take personal risks to achieve that vision, they are sensitive to follower needs, and they exhibit extraordinary behaviors. LO: 12.4: Describe the positive leadership styles and relationships. Difficulty: Easy Quest. Category: Concept
543 richard@qwconsultancy.com
52) Charismatic leaders influence followers by ________. A) by developing an aura of charisma B) engaging in emotion-inducing and often unconventional behavior C) setting an example for followers through actions and words D) articulating an appealing vision E) communicating high performance expectations Answer: D Explanation: Evidence suggests that charismatic leaders use a process that begins with articulating an appealing vision, a long-term strategy for attaining a goal by linking the present with a better future for the organization. Desirable visions fit the times and circumstances and reflect the uniqueness of the organization. LO: 12.4: Describe the positive leadership styles and relationships. Difficulty: Moderate Quest. Category: Concept 53) Charismatic leaders ________. A) engage in traditional behaviors that follow norms B) focus on their own responsibilities largely ignoring the needs and feelings of employees C) avoid personal risk D) propose a vision of the future that is better than the status quo E) ensure their own position as leader even if it means sacrificing others Answer: D Explanation: Key characteristics of charismatic leaders include vision and articulation (proposing a future that is better than the status quo); personal risk (willing to take on high personal risk); sensitivity to follower needs (perceptive of others' abilities and responsive to their needs and feelings); unconventional behavior (engages in behaviors that are perceived as novel and counter to the norm). LO: 12.4: Describe the positive leadership styles and relationships. Difficulty: Moderate Quest. Category: Concept 54) Which type of leader functions primarily by clarifying role and task requirements? A) Transformational leaders B) Transactional leaders C) Charismatic leaders D) Laissez-faire leaders E) Employee-oriented leaders Answer: B Explanation: Transactional leaders guide or motivate their followers in the direction of established goals by clarifying role and task requirements, allocating rewards and punishment where needed, and (passively or actively) intervening when the situation calls for it. LO: 12.4: Describe the positive leadership styles and relationships. Difficulty: Easy Quest. Category: Concept
544 richard@qwconsultancy.com
55) Which of the following is likely behavior for a transactional leader? A) Rewarding employees for the work that they have done, thus recognizing accomplishments B) Helping followers trust the leader and develop a loyalty toward the common vision C) Stimulating others in the organization to become thinkers of what the organization is all about D) Paying attention to the emotional needs of others and consider individual differences E) Avoiding authoritarian, command, and control behaviors when dealing with employees Answer: A Explanation: A transactional leader is likely to reward employees for the work that they have done, thus recognizing accomplishments. LO: 12.4: Describe the positive leadership styles and relationships. Difficulty: Moderate Quest. Category: Concept 56) Which of the following scenarios reflects a transactional approach to leadership? A) Samuel Perez has been running a small automobile service agency with a set of employees that he hired and trained over a decade ago. Though the business is mediocre and profit margins are not substantial, the employees work well as a team because each of them treats the other as equal, just like Samuel himself does. B) Carol Turner began V Care, a non-profit organization, with the purpose of battling various forms of abuse. As the organization grew, she took care to hire employees who felt the same kind of sensitivity she felt for victims of abuse. Today, she openly proclaims that the organization that she started is successful because every single employee shares and understands what the organization is aiming for. C) When new projects come, it is a time of excitement for the content production team of Impact, an advertising agency. Lori Wood, the department head, begins a project meeting by having her whole team envision what the final product of this campaign must aim at. She then sets progressively higher standards for the team and clarifies what each member of the team stands to gain if the project is a success. In allocating roles and tasks, she often incorporates the needs and preferences of her team members. D) Wayne Torres believes that setting targets, monitoring employee performance, and correcting defaulting employees are his tried and tested ways to accomplish tasks. He rewards employees who do well and match his predetermined expectations and recognizes accomplishments that he values. His team, however, has not grown substantially and seems to be at a functional status quo. E) Glenn Cruzz works at Leverage as a senior consultant. She ensures that her team of consultants has a framework of flexible tools and skills to help clients who come their way with diversity and inclusion issues. Through audits, research, and workshops, her team is encouraged to come up with unconventional solutions that help clients establish and leverage its employee diversity. Answer: D Explanation: Transactional leaders set goals for their employees and define roles and expectations. Unlike transformational leaders, however, transactional leaders are unlikely to motivate their employees to exceed expectations or go beyond the call of duty. LO: 12.4: Describe the positive leadership styles and relationships. AACSB: Analytical thinking Difficulty: Hard Employability Skills: Knowledge Application and Analysis 545 richard@qwconsultancy.com
Quest. Category: Application 57) Which type of leader inspires followers to transcend their self-interests for the good of the organization and can have an extraordinary effect on their followers? A) Transformational B) Transactional C) Task-oriented D) Laissez-faire E) Production-oriented Answer: A Explanation: Transformational leaders can have an extraordinary effect on their followers, inspiring them toward selfless goals that benefit the larger organization. These types of leaders inspire their followers by paying attention to their concerns, helping them rethink old problems in new ways, and encouraging them to achieve goals as a group. LO: 12.4: Describe the positive leadership styles and relationships. Difficulty: Easy Quest. Category: Concept 58) How can transformational leaders enhance employee performance? A) By restricting creativity among employees B) By focusing on short-term goals for employees C) By instilling pride in employees D) By establishing goals, roles, and requirements E) By abdicating all responsibility to employees Answer: C Explanation: Characteristics of transactional leaders: contingent reward, management by exception (active), management by exception (passive), and laissez-faire. Characteristics of transformational leaders: idealized influence; inspirational motivation, intellectual stimulation, individualized considerations. LO: 12.4: Describe the positive leadership styles and relationships. Difficulty: Easy Quest. Category: Concept 59) The ________ leadership behavior is the least effective in terms of the full range of leadership models. A) management by exception (active) B) contingent reward C) management by exception (passive) D) individualized consideration E) laissez-faire Answer: E Explanation: According to the full range of leadership models, laissez-faire is the most passive and therefore least effective of leader behaviors. Management by exception—active or passive— is slightly better than laissez-faire. LO: 12.4: Describe the positive leadership styles and relationships. Difficulty: Moderate 546 richard@qwconsultancy.com
Quest. Category: Concept 60) Which of the following leadership behaviors is likely to differentiate between transactional and transformational leaders? A) Management by exception (active and passive) B) Contingent reward C) Individualized consideration D) Micromanagement E) Laissez-faire Answer: C Explanation: According to the full range of leadership models, leadership behaviors including laissez-faire, management by exception (active or passive), and contingent reward will not get employees to go above and beyond the call of duty. Only with the four styles of transformational leadership—individualized consideration, intellectual stimulation, inspirational motivation, and idealized influence—are leaders able to motivate followers to perform above expectations and transcend their self-interest for the sake of the organization. LO: 12.4: Describe the positive leadership styles and relationships. Difficulty: Easy Quest. Category: Concept 61) In terms of the full range of leadership models, which leadership behavior represents the most active and effective approach for leaders? A) Contingent reward B) Idealized influence C) Management by exception (active) D) Individualized consideration E) Laissez-faire Answer: B Explanation: According to the full range of leadership models, idealized influence, wherein a leader provides vision, establishes a sense of mission, and instills pride is the most active and effective leadership behavior. By doing so, the leader gains respect and trust. LO: 12.4: Describe the positive leadership styles and relationships. Difficulty: Moderate Quest. Category: Concept
547 richard@qwconsultancy.com
62) The ________ related to transformational leadership indicates that transformational approaches motivate employees helping them to become more confident and engaged, and more willing to put in time and effort. A) attitudinal mechanism B) affective mechanism C) motivational mechanism D) social exchange mechanism E) justice enhancement mechanism Answer: C Explanation: Researchers have found that in general, transformational leadership is effective for five reasons: affective or attitudinal mechanism; motivational mechanism (transformational approaches motivate employees helping them to become more confident and engaged, and more willing to put in time and effort); identification mechanism; social exchange mechanism; and justice enhancement mechanism. LO: 12.4: Describe the positive leadership styles and relationships. Difficulty: Easy Quest. Category: Concept 63) A(n) ________ refers to a long-term strategy for attaining a goal by linking the present with a better future. A) transaction B) attribution C) contingent plan D) vision E) substitution Answer: D Explanation: A long-term strategy for attaining a goal by linking the present with a better future for the organization is defined as a vision. LO: 12.4: Describe the positive leadership styles and relationships. Difficulty: Easy Quest. Category: Concept
548 richard@qwconsultancy.com
64) Cooper Mills is a company that accomplished one of the greatest success stories of our time. The company, which began in the garage of its current CEO, Kyle Cooper, went on to become one of the largest producers of textiles in the country. The company has now diversified successfully into other product lines. A group of researchers is now undertaking a study on Cooper Mills as an organization led by a transformational leader. Which of the following, if true, would most support the conclusion that Kyle Cooper is a transformational leader? A) The management follows Kyle's principle of avoiding risks and conducting business conservatively. B) Cooper Mills' goals tend to be very ambitious and to hold personal value for employees. C) Cooper Mills has a centralized decision-making structure. D) Cooper Mills' performance has held at average levels for the past three years. E) Cooper Mills' compensation plans are designed to reward short-term results. Answer: B Explanation: In companies with transformational leaders, followers are more likely to pursue ambitious goals, agree on the strategic goals of the organization, and believe the goals they are pursuing are personally important. There is greater decentralization of responsibility, managers have more propensity to take risks, and compensation plans are geared toward long-term results. All these factors result in superior organizational performance. LO: 12.4: Describe the positive leadership styles and relationships. AACSB: Analytical thinking Difficulty: Hard Employability Skills: Knowledge Application and Analysis Quest. Category: Application 65) Transactional and transformational leadership are opposing approaches. Answer: FALSE Explanation: Transactional and transformational leadership complement each other; they are not opposing approaches to getting things done. Transformational leadership builds on transactional leadership and produces levels of follower effort and performance beyond what transactional leadership alone can do. It appears that transformational leadership is more important for group performance, OCBs, and satisfaction with the leader, whereas transactional leadership (primarily contingent reward) is more important for leader effectiveness, follower performance, and follower job satisfaction. LO: 12.4: Describe the positive leadership styles and relationships. Difficulty: Moderate Quest. Category: Concept 66) An individual who lacks self-esteem and questions his or her self-worth is more likely to absorb a leader's direction rather than establish and individual way of leading or thinking. Answer: TRUE Explanation: An individual who lacks self-esteem and questions his or her self-worth is more likely to absorb a leader's direction rather than establish and individual way of leading or thinking. For these people the situation may matter much less than the charismatic qualities of the leader. LO: 12.4: Describe the positive leadership styles and relationships. Difficulty: Moderate 549 richard@qwconsultancy.com
Quest. Category: Concept 67) The best leaders are transformational, not transactional. Answer: FALSE Explanation: The best leaders are transactional and transformational. LO: 12.4: Describe the positive leadership styles and relationships. Difficulty: Moderate Quest. Category: Concept 68) Transactional leaders guide their followers toward established goals by clarifying role and task requirements. Answer: TRUE Explanation: Transactional leaders guide their followers toward established goals by clarifying role and task requirements. LO: 12.4: Describe the positive leadership styles and relationships. Difficulty: Moderate Quest. Category: Concept 69) Intellectual stimulation and individualized consideration are likely to be seen in the management by exception (active) style of leadership. Answer: FALSE Explanation: Intellectual stimulation and individualized consideration are likely to be seen in the transformational style of leadership, while management by exception (active or passive) is a transactional style of leadership. LO: 12.4: Describe the positive leadership styles and relationships. Difficulty: Moderate Quest. Category: Concept 70) Followers are more likely to perceive that they are supported by the leader, team, and/or organization when there is transformational leadership. Answer: TRUE Explanation: Transformational approaches improve the quality of exchange and relationship between leaders and followers. Followers are more likely to perceive that they are supported by the leader, team, and/or organizations when there is transformational leadership. LO: 12.4: Describe the positive leadership styles and relationships. Difficulty: Moderate Quest. Category: Concept 71) Describe the key characteristics of charismatic leaders. Answer: A charismatic leader is characterized by: a) vision and articulation: having an idealized goal and being able to clarify its importance b) personal risk: willingness to take personal risks, incur high costs, and make sacrifices c) sensitivity to follower needs: recognizing and responding to others' abilities, needs, and feelings d) unconventional behavior: engaging in behaviors that are novel LO: 12.4: Describe the positive leadership styles and relationships. 550 richard@qwconsultancy.com
Difficulty: Moderate Quest. Category: Concept 72) What is the full range of leadership model? Answer: According to the full range of leadership model, laissez-faire is the most passive and therefore least effective of leader behaviors. Management by exception—active or passive—is slightly better than laissez-faire, but it's still considered ineffective. Management-by-exception leaders tend to be available only when there is a problem, which is often too late. Contingent reward leadership can be an effective style of leadership but will not get employees to go above and beyond the call of duty. Only with the four remaining styles—all aspects of transformational leadership—are leaders able to motivate followers to perform above expectations and transcend their self-interest for the sake of the organization. Individualized consideration, intellectual stimulation, inspirational motivation, and idealized influence all result in extra effort from workers, higher productivity, higher morale and satisfaction, higher organizational effectiveness, lower turnover, lower absenteeism, and greater organizational adaptability. Based on this model, leaders are generally most effective when they regularly use each of the four transformational behaviors. LO: 12.4: Describe the positive leadership styles and relationships. Difficulty: Moderate Quest. Category: Concept 73) Why are charismatic leaders not always authentic leaders? Answer: Charismatic leadership is a leadership theory that states that followers make attributions of heroic or extraordinary leadership abilities when they observe certain behaviors. Though charismatic leaders exert great influence on their followers, unfortunately, they do not necessarily act in the best interests of their organizations. Many charismatic leaders have allowed their personal goals to override the goals of the organization. Thus, their followers can never consider them ethical people unless they inspire that confidence. Authentic leaders know who they are, know what they believe in and value, and act on those values and beliefs openly and candidly. The primary quality produced by authentic leadership, therefore, is trust. Authentic leaders share information, encourage open communication, and stick to their ideals. The need to incorporate trust and ethics into charismatic leadership has led to the concept of socialized charismatic leadership, which conveys other-centered (not self-centered) values by leaders who model ethical conduct. Socialized charismatic leaders are able to bring employee values in line with their own values through their words and actions. LO: 12.4: Describe the positive leadership styles and relationships. AACSB: Analytical thinking Difficulty: Moderate Quest. Category: Synthesis
551 richard@qwconsultancy.com
74) Explain charismatic leadership from the understanding of leadership under the attribution theory. Answer: Charismatic leadership is a leadership theory that states that followers make attributions of heroic or extraordinary leadership abilities when they observe certain behaviors. A number of studies have attempted to identify the characteristics of charismatic leaders: they have a vision, they are willing to take personal risks to achieve that vision, they are sensitive to follower needs, and they exhibit extraordinary behaviors. Individuals are born with traits that make them charismatic. Personality is also related to charismatic leadership; charismatic leaders are likely to be extraverted, self-confident, and achievement oriented. However, most experts believe individuals can be trained to exhibit charismatic behaviors. The attribution theory, on the other hand, says leadership is merely an attribution people make about other individuals. Attribution theory suggests what is important is projecting the appearance of being a leader rather than focusing on actual accomplishments. According to this theory, those who want to be leaders merely need to shape the perception that they are smart, personable, verbally adept, aggressive, hardworking, and consistent in their style. This portrayal is enough to increase the probability their bosses, colleagues, and employees will view them as effective leaders. LO: 12.4: Describe the positive leadership styles and relationships. AACSB: Analytical thinking Difficulty: Moderate Quest. Category: Synthesis 75) ________ is the primary quality produced by authentic leadership. A) Efficiency B) Reinforcement C) Supervision D) Trust E) Authority Answer: D Explanation: Authentic leaders know who they are, know what they believe in and value, and act on those values and beliefs openly and candidly. Their followers consider them ethical people. The primary quality produced by authentic leadership, therefore, is trust. LO: 12.5: Discuss the roles of leaders in creating ethical organizations. AACSB: Ethical understanding and reasoning Difficulty: Moderate Quest. Category: Concept
552 richard@qwconsultancy.com
76) Which of the following is a feature of servant leadership? A) Pursuit of the leaders' self-interest B) Use of power and authority to achieve organizational ends C) Universal appeal and effectiveness across cultures D) Focus on growth, development, and well-being of followers E) Applicability restricted to the leader's ingroup Answer: D Explanation: Servant leaders go beyond their own self-interest and focus on opportunities to help followers grow and develop. They do not use power to achieve ends; they emphasize persuasion. Characteristic behaviors include listening, empathizing, persuading, accepting stewardship, and actively developing followers' potential. LO: 12.5: Discuss the roles of leaders in creating ethical organizations. AACSB: Ethical understanding and reasoning Difficulty: Easy Quest. Category: Concept 77) Servant leadership reflects the ________. A) traditional view that followers are expected to serve the leaders B) task-oriented approach to leadership C) use of empathy, listening, and persuasion by leaders D) Western prototype of a leader who gives orders to followers E) tendency of leaders to prioritize their interests over the interests of their employees Answer: C Explanation: Servant leaders go beyond their own self-interest and focus on opportunities to help followers grow and develop. They do not use power to achieve ends; they emphasize persuasion. Characteristic behaviors include listening, empathizing, persuading, accepting stewardship, and actively developing followers' potential. LO: 12.5: Discuss the roles of leaders in creating ethical organizations. AACSB: Ethical understanding and reasoning Difficulty: Easy Quest. Category: Concept
553 richard@qwconsultancy.com
78) Leaders who want to foster a climate that reinforces ethical behavior do all of the following except ________. A) set high ethical standards B) demonstrate good ethical behavior C) encourage integrity among employees D) reward employees demonstrating integrity E) use their charisma to enhance power over followers, directed towards self-serving ends Answer: E Explanation: Top executives set the moral tone for organizations. They should set high ethical standards, demonstrate them through their own behavior, and encourage and reward integrity in others while avoiding abuses of power such as giving themselves large raises and bonuses while laying off employees. LO: 12.5: Discuss the roles of leaders in creating ethical organizations. AACSB: Ethical understanding and reasoning Difficulty: Moderate Quest. Category: Concept 79) Supervision that is hostile both verbally and nonverbally is referred to as abusive supervision. Answer: TRUE Explanation: Abusive supervision is supervision that is hostile both verbally and nonverbally. LO: 12.5: Discuss the roles of leaders in creating ethical organizations. AACSB: Ethical understanding and reasoning Difficulty: Easy Quest. Category: Concept 80) Abusive supervision leads to increases in organizational commitment, job satisfaction, and perceived organizational support. Answer: FALSE Explanation: Abusive supervision is supervision that is hostile both verbally and nonverbally. Abusive supervision leads to decreases in organizational commitment, job satisfaction, and perceived organizational support. LO: 12.5: Discuss the roles of leaders in creating ethical organizations. AACSB: Ethical understanding and reasoning Difficulty: Moderate Quest. Category: Concept 81) Ethical leadership sets the example for how employees should treat one another. Answer: TRUE Explanation: Ethical leadership has a direct influence on how employees think about moral issues and establishes an organizational culture with clear values and principles for what is (un)ethical behavior. Ethical leadership sets the example for how employees should treat one another. LO: 12.5: Discuss the roles of leaders in creating ethical organizations. AACSB: Ethical understanding and reasoning Difficulty: Moderate 554 richard@qwconsultancy.com
Quest. Category: Concept 82) Describe abusive supervision. Answer: Abusive supervision refers to supervision that is hostile both verbally and nonverbally. A recent review suggests that several factors are related to abusive supervision. For one, nearly all forms of justice are negatively related to abusive supervision, suggesting that a sense of injustice is at the core of abusive supervision (especially for interpersonal justice). Although some personality traits such as agreeableness and conscientiousness appear to be negatively (but weakly) associated with perceptions of abusive supervision, negative affect is strongly linked with it. The same review also suggests that abusive supervision comes with dire consequences. First and foremost, abusive supervision negatively affects health. Second, it also leads to decreases in organizational commitment, job satisfaction, and perceived organizational support, along with increased work-family conflict. LO: 12.5: Discuss the roles of leaders in creating ethical organizations. AACSB: Ethical understanding and reasoning Difficulty: Moderate Quest. Category: Concept 83) Describe servant leadership and explain characteristic behaviors of servant leaders. Answer: Scholars have recently considered ethical leadership from a new angle by examining servant leadership. Servant leaders go beyond their own self-interest and focus on opportunities to help followers grow and develop. They do not use power to achieve ends; they emphasize persuasion. Characteristic behaviors include listening, empathizing, persuading, accepting stewardship, and actively developing followers' potential. Servant leadership leads to higher levels of commitment to the supervisor, self-efficacy, and perceptions of justice; it also increases team potency. Servant leadership may be more prevalent and more effective in certain cultures; the East Asian prototype is more like a servant leader, which might mean servant leadership is more effective in these cultures. LO: 12.5: Discuss the roles of leaders in creating ethical organizations. AACSB: Ethical understanding and reasoning Difficulty: Moderate Quest. Category: Concept
555 richard@qwconsultancy.com
84) Which of the following is a psychological state that exists when you agree to make yourself vulnerable to another because you have positive expectations about how things are going to turn out? A) Consideration B) Trust C) Support D) Empathy E) Respect Answer: B Explanation: Trust is a psychological state that exists when you agree to make yourself vulnerable to another because you have positive expectations about how things are going to turn out. Even though you aren't completely in control of the situation, you are willing to take a chance that the other person will come through for you. LO: 12.6: Describe how leaders can have a positive impact on their organizations through building trust. Difficulty: Easy Quest. Category: Concept 85) A dimension of trust defined as honesty, truthfulness, and the ability to display consistency between one's words and action is ________. A) consideration B) sensitivity C) support D) reflexivity E) integrity Answer: E Explanation: Integrity refers to honesty and truthfulness. It is the most critical characteristic in assessing another's trustworthiness. Integrity also means having consistency between what you do and say. LO: 12.6: Describe how leaders can have a positive impact on their organizations through building trust. Difficulty: Easy Quest. Category: Concept
556 richard@qwconsultancy.com
86) The ________ dimension of trust is defined as an individual's technical and interpersonal knowledge and skills. A) integrity B) ability C) propensity D) reflexivity E) complexity Answer: B Explanation: Ability encompasses an individual's technical and interpersonal knowledge and skills. LO: 12.6: Describe how leaders can have a positive impact on their organizations through building trust. Difficulty: Easy Quest. Category: Concept 87) The CEO of Xenon Solutions recently cancelled numerous leave requests and asked several employees to put in extra days of work as the company was slated to meet several deadlines in the immediate future. In addition, he also assured that those who work this extra bit will be given their due leaves and additional incentives once the time crisis had passed. Following this announcement, Joan and Shane were overheard speaking in the cafeteria. Shane was resentful that his holiday plans were disrupted, and he was sure that the management would ultimately not provide any of the leaves and incentives it promised. Joan, however, said that she was sure that their CEO had a valid reason behind making such a request and that if they put in the extra effort, they would be rewarded suitably. From the information provided in the scenario, which of the following statements can be inferred? A) Shane has a high degree of empathy. B) The leader-member relations between Shane and the CEO are the same as that between Joan and the CEO. C) Joan has a high degree of trust propensity. D) Shane has a high degree of identification-based trust in the CEO. E) Shane is one of the members of the CEO's ingroups. Answer: C Explanation: Trust propensity refers to how likely a particular employee is to trust a leader. Some people are simply more likely to believe others can be trusted. In this scenario, Joan reflects a high degree of trust propensity and feels that her CEO's words and actions can be trusted. LO: 12.6: Describe how leaders can have a positive impact on their organizations through building trust. AACSB: Analytical thinking Difficulty: Moderate Employability Skills: Knowledge Application and Analysis Quest. Category: Application
557 richard@qwconsultancy.com
88) The main characteristic we use to determine the trustworthiness of a leader is his or her ability. Answer: FALSE Explanation: There are three key characteristics that lead us to believe that a leader is trustworthy. These are integrity, benevolence, and ability. LO: 12.6: Describe how leaders can have a positive impact on their organizations through building trust. Difficulty: Easy Quest. Category: Concept 89) Explain the relationship between trust and leadership. Answer: Trust is a psychological state that exists when you agree to make yourself vulnerable to another because you have positive expectations about how things are going to turn out. Trust is a primary attribute associated with leadership; breaking it can have serious adverse effects on a group's performance. The key characteristics that lead us to believe that a leader is trustworthy are integrity, benevolence, and ability. Integrity refers to honesty and truthfulness. It is the most critical characteristic in assessing another's trustworthiness. Benevolence means the trusted person has your interests at heart, even if yours aren't necessarily in line with theirs. Ability encompasses an individual's technical and interpersonal knowledge and skills. Another factor that influences trust is trust propensity, which refers to how likely a particular employee is to trust a leader. LO: 12.6: Describe how leaders can have a positive impact on their organizations through building trust. Difficulty: Moderate Quest. Category: Concept 90) A senior employee who sponsors and supports a less-experienced employee, a protégé is referred to as a(n) ________. A) intern B) subordinate C) supervisee D) mentor E) apprentice Answer: D Explanation: A mentor is a senior employee who sponsors and supports a less-experienced employee, a protégé. LO: 12.7: Identify the challenges and opportunities to our understanding of leadership. Difficulty: Easy Quest. Category: Concept
558 richard@qwconsultancy.com
91) As an upper-level manager in his organization, Lionel Tucker has been asked to mentor a less experienced, lower-level employee. His boss wants both the lower-level employee and Lionel to have input into the relationship. Lionel is not happy with the request because he feels his workload is already too heavy. Lionel reluctantly agrees to the request because he ________. A) thinks the lower-level employee is falling behind B) cannot see any way out of the request C) owes his boss a favor D) believes in network ties E) thinks the relationship would be beneficial to him Answer: E Explanation: Mentors must see the relationship as beneficial to themselves and the protégé, and the protégé must have input to the relationship. LO: 12.7: Identify the challenges and opportunities to our understanding of leadership. AACSB: Analytical thinking Difficulty: Hard Employability Skills: Knowledge Application and Analysis Quest. Category: Application 92) According to the attribution theory of leadership, leadership is characterized by ________. A) the significant gains made in the mentoring relationship between the leader and follower B) the act of people ascribing qualities like intelligence or charisma to leaders C) the focus on the actual accomplishments and performance of the leader D) the impact of substitutes and neutralizers in enhancing the role of the leader E) the division of the group into ingroup and out-group members based on the leader's preference Answer: B Explanation: The attribution theory of leadership says leadership is merely an attribution people make about other individuals. Thus, we attribute to leaders intelligence, outgoing personality, strong verbal skills, aggressiveness, understanding, and industriousness. At the organizational level, we tend to see leaders, rightly or wrongly, as responsible for extremely negative or extremely positive performance. LO: 12.7: Identify the challenges and opportunities to our understanding of leadership. Difficulty: Moderate Quest. Category: Concept
559 richard@qwconsultancy.com
93) According to the attribution theory of leadership, a person aiming to be a leader has to ________. A) gain sufficient experience to be a leader B) possess qualities like charisma and intelligence C) make changes on ground before leading people D) possess many significant actual accomplishments E) shape the perception that he or she could be a leader Answer: E Explanation: Attribution theory suggests what's important is projecting the appearance of being a leader rather than focusing on actual accomplishments. Leader-wannabes who can shape the perception that they're smart, personable, verbally adept, aggressive, hardworking, and consistent in their style can increase the probability their bosses, colleagues, and employees will view them as effective leaders. LO: 12.7: Identify the challenges and opportunities to our understanding of leadership. Difficulty: Moderate Quest. Category: Concept 94) The president of a small Asian country was hailed as a visionary and a genius when the nation's economy burgeoned during his first term in office. However, when the currency and the stock markets crashed during his government's second term, he was censured as arrogant, elitist, and shortsighted. Which of the following theoretical approaches is reflected here? A) Trait theories B) Behavioral theories C) Path-goal theory D) Servant leadership perspective E) Attribution theory Answer: E Explanation: The attribution theory of leadership says leadership is merely an attribution people make about other individuals. Thus, we attribute to leaders intelligence, outgoing personality, strong verbal skills, aggressiveness, understanding, and industriousness. At the organizational level, we tend to see leaders, rightly or wrongly, as responsible for extremely negative or extremely positive performance. LO: 12.7: Identify the challenges and opportunities to our understanding of leadership. AACSB: Analytical thinking Difficulty: Moderate Employability Skills: Knowledge Application and Analysis Quest. Category: Application
560 richard@qwconsultancy.com
95) A substitute for leadership in the theory that suggests leaders' actions are irrelevant in many situations is ________. A) lack of experience B) training C) ambiguous task D) high need for supervision E) external feedback Answer: B Explanation: A theory of leadership suggests that in many situations, leaders' actions are irrelevant. Experience and training are among the substitutes that can replace the need for a leader's support or ability to create structure. Organizational characteristics such as explicit formalized goals, rigid rules and procedures, and cohesive work groups can also replace formal leadership. LO: 12.7: Identify the challenges and opportunities to our understanding of leadership. Difficulty: Moderate Quest. Category: Concept 96) Which of the following is true about substitutes for leadership? A) Substitutes are clearly distinguishable from neutralizers. B) Substitutes enhance the need for leader's support or ability to create structure. C) Substitutes increase the levels of identification-based trust. D) Substitutes are factors and conditions that replace formal leadership. E) Organizations cannot provide any substitutes for leadership. Answer: D Explanation: A theory of leadership suggests that in many situations, leaders' actions are irrelevant. Experience and training are among the substitutes that can replace the need for a leader's support or ability to create structure. Organizational characteristics such as explicit formalized goals, rigid rules and procedures, and cohesive work groups can also replace formal leadership. LO: 12.7: Identify the challenges and opportunities to our understanding of leadership. Difficulty: Moderate Quest. Category: Concept
561 richard@qwconsultancy.com
97) The top management of Myers Corp are planning a reorganization of their company to cut costs and increase efficiency. The different department heads have been asked to present their departmental strengths, needs, and concerns at a meeting. The various division heads have come to an agreement that clarity of goals and tasks, detailed procedural guidelines like employee manuals, and a cooperative workforce are their main strengths. Additionally, they are of the opinion that cutting managerial positions to reduce costs and reorganizing Myers Corp as a relatively flat organization is a good approach. This scenario reflects the operation of ________ in replacing the support and ability offered by leaders. A) vision B) traits C) attributes D) substitutes E) charisma Answer: D Explanation: One theory of leadership suggests that in many situations leaders' actions are irrelevant. Experience and training are among the substitutes that can replace the need for a leader's support or ability to create structure. Organizational characteristics such as explicit formalized goals, rigid rules and procedures, and cohesive work groups can also replace formal leadership. LO: 12.7: Identify the challenges and opportunities to our understanding of leadership. AACSB: Analytical thinking Difficulty: Moderate Employability Skills: Knowledge Application and Analysis Quest. Category: Application 98) Which of the following make it impossible for leader behavior to make any difference to follower outcomes? A) Catalysts B) Mentors C) Neutralizers D) Hygiene factors E) Vision and charisma Answer: C Explanation: Neutralizers make it impossible for leader behavior to make any difference to follower outcomes. Indifference to organizational rewards can neutralize the effects of leadership. LO: 12.7: Identify the challenges and opportunities to our understanding of leadership. Difficulty: Easy Quest. Category: Concept
562 richard@qwconsultancy.com
99) ________ is a neutralizer of leadership. A) Indifference to rewards B) Experience and training C) Explicit formalized rules D) Cohesive work groups E) A highly structured task Answer: A Explanation: Neutralizers make it impossible for leader behavior to make any difference to follower outcomes. Indifference to organizational rewards can neutralize the effects of leadership. LO: 12.7: Identify the challenges and opportunities to our understanding of leadership. Difficulty: Moderate Quest. Category: Concept 100) Which of the following is true with regard to getting the most from leadership-training budgets? A) Leadership training is likely to be more successful with low self-monitors. B) It is difficult for organizations to teach implementation skills. C) Teaching trust building and monitoring is virtually impossible. D) Leaders can be trained in transformational leadership skills that have bottom-line results. E) For online leaders, substitutes and neutralizers have no effect. Answer: D Explanation: Leaders can be trained in transformational leadership skills that have bottom-line results. LO: 12.7: Identify the challenges and opportunities to our understanding of leadership. Difficulty: Moderate Quest. Category: Concept 101) Formal mentoring programs are most likely to succeed if they appropriately match the work style, needs, and skills of protégé and mentor. Answer: TRUE Explanation: Formal mentoring programs are most likely to succeed if they appropriately match the work style, needs, and skills of protégé and mentor. LO: 12.7: Identify the challenges and opportunities to our understanding of leadership. Difficulty: Moderate Quest. Category: Concept 102) The primary gains from mentoring are seen in greater compensation and higher job performance. Answer: FALSE Explanation: Mentoring is less valuable for objective outcomes like compensation and job performance, and research suggests the gains are primarily psychological. LO: 12.7: Identify the challenges and opportunities to our understanding of leadership. Difficulty: Moderate Quest. Category: Concept
563 richard@qwconsultancy.com
564 richard@qwconsultancy.com
103) The attribution theory of leadership says leadership is merely an attribution people make about other individuals. Answer: TRUE Explanation: The attribution theory of leadership says leadership is merely an attribution people make about other individuals. LO: 12.7: Identify the challenges and opportunities to our understanding of leadership. Difficulty: Easy Quest. Category: Concept 104) Genetics and experiences shape young people, and the relationship between these factors and CEO success is complex. Answer: TRUE Explanation: The stories of CEOs who start early make for good press reports, but CEOs do not by definition start early. What we can say, though, is that genetics and experiences both shape young people, and that the relationship between those factors and CEO success is complex. LO: 12.7: Identify the challenges and opportunities to our understanding of leadership. Difficulty: Moderate Quest. Category: Concept 105) How do substitutes and neutralizers challenge the effectiveness of leadership? Answer: One theoretical approach of leadership suggests that in many situations leaders' actions are irrelevant. Experience and training are among the substitutes that can replace the need for a leader's support or ability to create structure. Organizational characteristics such as explicit formalized goals, rigid rules and procedures, and cohesive work groups can also replace formal leadership, while indifference to organizational rewards can neutralize its effects. Neutralizers make it impossible for leader behavior to make any difference to follower outcomes. It must be added that the difference between substitutes and neutralizers is not clear-cut. LO: 12.7: Identify the challenges and opportunities to our understanding of leadership. Difficulty: Moderate Quest. Category: Concept 106) What is the most important event an organization needs to plan for? Answer: The most important event an organization needs to plan for is a change in leadership. In general, organizations seem to spend no time on leadership succession and are surprised when their pick turns out poorly. LO: 12.7: Identify the challenges and opportunities to our understanding of leadership. AACSB: Analytical thinking Difficulty: Moderate Quest. Category: Synthesis Organizational Behavior, 19e (Robbins/Judge) Chapter 13 Power and Politics 1) According to the text, the most important aspect of power is probably that it is a function of ________. A) goal congruency B) realization 565 richard@qwconsultancy.com
C) inheritance D) dependence E) altruism Answer: D Explanation: Probably the most important aspect of power is that it is a function of dependence. Power does not require goal compatibility, just dependence. Leadership, on the other hand, requires some congruence between the goals of the leader and those being led. For a power situation to exist, one person or group needs to have control over resources the other person or group values. LO: 13.1: Contrast leadership and power. Difficulty: Moderate Quest. Category: Concept 2) Which of the following statements is true regarding power? A) A is said to have power over B if B controls something that A desires. B) The greater A's dependence on B, the greater A's power in the relationship. C) The more B and C rely on A, the more powerful A becomes. D) A can have power over B only if A is B's leader. E) A can have power over B only when A and B have similar goals. Answer: C Explanation: The more people rely or depend upon the powerful person (who controls something the others rely on or want), the more powerful that person becomes. When people begin to have more alternatives and options or begin to rely on themselves or different people, the powerful person loses power. LO: 13.1: Contrast leadership and power. Difficulty: Moderate Quest. Category: Concept 3) ________ is a differentiating factor between power and leadership. A) Control B) Influence C) Ability D) Goal compatibility E) Negative affect Answer: D Explanation: Power does not require goal compatibility, merely dependence. Leadership, on the other hand, requires some congruence between the goals of the leader and those being led. LO: 13.1: Contrast leadership and power. Difficulty: Moderate Quest. Category: Concept 4) ________ often differentiates power from leadership. A) A lack of dependence of followers B) The use of positive styles over negative tactics C) Downward influence of leader on followers D) The lack of goal compatibility between leaders and followers 566 richard@qwconsultancy.com
E) A lack of control on behavior of followers Answer: D Explanation: Power does not require goal compatibility, merely dependence. Leadership, on the other hand, requires some congruence between the goals of the leader and those being led. LO: 13.1: Contrast leadership and power. Difficulty: Moderate Quest. Category: Concept 5) Which of the following is true with regard to leadership? A) It maximizes the importance of lateral and upward influence patterns. B) It eliminates the requirement of goal compatibility. C) It stimulates research in the area of tactics of influence. D) It often emphasizes style. E) It encourages dependence of followers on leaders. Answer: D Explanation: Leadership often emphasizes style. It seeks answers to questions such as "How supportive should a leader be?" and "How much decision making should be shared with followers?" LO: 13.1: Contrast leadership and power. Difficulty: Moderate Quest. Category: Concept 6) A powerful person begins to lose power when ________. A) leadership style changes B) people begin to rely on themselves C) tactics for gaining compliance of employees increase D) employee rewards are encouraged E) modes of assessing employee performance are reassessed Answer: B Explanation: The more people rely or depend upon a powerful person (who controls something the others rely on or want), the more powerful that person becomes. When people begin to have more alternatives and options or begin to rely on themselves or different people, the powerful person loses power. LO: 13.1: Contrast leadership and power. Difficulty: Moderate Quest. Category: Concept
567 richard@qwconsultancy.com
7) Unlike power, leadership requires some degree of goal compatibility. Answer: TRUE Explanation: Power does not require goal compatibility, merely dependence. Leadership, on the other hand, requires some congruence between the goals of the leader and those being led. LO: 13.1: Contrast leadership and power. Difficulty: Moderate Quest. Category: Concept 8) For a power situation to exist, one person or group needs to have control over resources the other person or group values. Answer: TRUE Explanation: For a power situation to exist, one person or group needs to have control over resources the other person or group values. This is usually the case in established leadership situations. However, power relationships are possible in all areas of life, and power can be obtained in many ways. LO: 13.1: Contrast leadership and power. Difficulty: Moderate Quest. Category: Concept 9) If you want a college degree and have to pass a certain course to get it, and your current instructor is the only faculty member in the college who teaches the course, she has power over you. Answer: TRUE Explanation: A person can have power over you only if he or she controls something you desire. If you want a college degree and have to pass a certain course to get it, and your current instructor is the only faculty member in the college who teaches the course, she has power over you because your alternatives are highly limited, and you place a high degree of importance on the outcome. LO: 13.1: Contrast leadership and power. Difficulty: Moderate Quest. Category: Concept 10) Using examples, define power. Answer: Power refers to a capacity that A has to influence the behavior of B so B acts in accordance with A's wishes. Someone can thus have power but not use it; it is a capacity or potential. Probably, the most important aspect of power is that it is a function of dependency. The greater B's dependence on A, the greater A's power in the relationship. Dependence, in turn, is based on alternatives that B perceives, and the importance B places on the alternative(s) A controls. A person can have power over you only if he or she controls something you desire. LO: 13.1: Contrast leadership and power. Difficulty: Moderate Quest. Category: Concept
568 richard@qwconsultancy.com
11) Compare and contrast leadership and power. Answer: Power does not require goal compatibility, just dependence. Leadership, on the other hand, requires some congruence between the goals of the leader and those being led. A second difference relates to the direction of influence. Power focuses more on the downward influence on followers. It minimizes the importance of lateral and upward relationships, which are important in leadership. For a third difference, leadership often emphasizes style. It seeks answers to questions such as "How supportive should a leader be?" and "How much decision making should be shared with followers?" In contrast, power focuses on tactics for securing compliance. LO: 13.1: Contrast leadership and power. Difficulty: Moderate Quest. Category: Concept 12) What are the two general groupings of power? A) Informational and personal B) Formal and informal C) Informal and legitimate D) Formal and personal E) Direct and indirect Answer: D Explanation: The bases or sources of power are divided into two general groupings, formal and personal. These two groupings are then broken down into more specific categories. LO: 13.2: Explain the three bases of formal power and the two bases of personal power. Difficulty: Easy Quest. Category: Concept 13) Which type of power is based on the fear of the negative results from failing to comply? A) Legitimate B) Coercive C) Punitive D) Referent E) Abusive Answer: B Explanation: The coercive power base depends on fear of the negative results from failing to comply. It rests on the application, or the threat of application, of physical sanctions such as the infliction of pain, frustration through restriction of movement, or the controlling by force of basic physiological or safety needs. LO: 13.2: Explain the three bases of formal power and the two bases of personal power. Difficulty: Easy Quest. Category: Concept
569 richard@qwconsultancy.com
14) In the last one week, May and Phyllis have been putting in extra hours at work so that the project assigned to them is completed on time. Though the manager was due to assign two more people to this project, he had not done so and, instead, was emphasizing to employees the importance of adhering to the needs of the department in regard to workload. As a result, May and Phyllis, who had double their routine workload, complained to the division manager. May was promptly suspended from work for complaining about her immediate supervisor. This scenario describes ________ power. A) reward B) legitimate C) coercive D) expert E) referent Answer: C Explanation: The coercive power base depends on fear of the negative results from failing to comply. At the organizational level, A has coercive power over B if A can dismiss, suspend, or demote B, assuming B values his or her job. LO: 13.2: Explain the three bases of formal power and the two bases of personal power. AACSB: Analytical thinking Difficulty: Moderate Employability Skills: Knowledge Application and Analysis Quest. Category: Application 15) Which type of power represents the compliance that is achieved based on the ability to distribute positive benefits that others view as valuable? A) Legitimate B) Coercive C) Reward D) Personal E) Reflective Answer: C Explanation: The opposite of coercive power is reward power, with which people comply because it produces positive benefits; someone who can distribute rewards others view as valuable will have power over them. These rewards can be either financial, such as controlling pay rates, or nonfinancial, including recognition or promotions. LO: 13.2: Explain the three bases of formal power and the two bases of personal power. Difficulty: Easy Quest. Category: Concept
570 richard@qwconsultancy.com
16) As a regional sales officer, one of Brandon's job responsibilities is to process the yearly appraisal forms of his subordinates and provide them with increments, bonuses, or benefits based on their performance that year. This job responsibility directly reflects his ________. A) active power B) expert power C) referent power D) reward power E) coercive power Answer: D Explanation: The opposite of coercive power is reward power, with which people comply because it produces positive benefits; someone who can distribute rewards others view as valuable will have power over them. These rewards can be either financial, such as controlling pay rates, or nonfinancial, including recognition and promotions. LO: 13.2: Explain the three bases of formal power and the two bases of personal power. AACSB: Analytical thinking Difficulty: Moderate Employability Skills: Knowledge Application and Analysis Quest. Category: Application 17) Legitimate power is directly based on the ________. A) charisma of the leader B) interpersonal trust and commitment of the manager C) structural position of the manager in the organization D) personality traits of the manager E) ability of the manager to serve his or her subordinates Answer: C Explanation: The most common way to access one or more of the power bases is through legitimate power. It represents the formal authority to control and use organizational resources based on structural position in the organization. LO: 13.2: Explain the three bases of formal power and the two bases of personal power. Difficulty: Easy Quest. Category: Concept 18) Which of the following statements is true regarding legitimate power? A) Legitimate power develops out of admiration of another and a desire to be like that person. B) Legitimate power comes from an individual's unique characteristics. C) Legitimate power can dampen the effects of other forms of power. D) Celebrities who endorse products in commercials wield legitimate power over people. E) Legitimate power is a type of personal power. Answer: C Explanation: Legitimate power can dampen the effect of other forms of power. For instance, when an employee stands up against something unethical, they are less likely to suffer the consequences of others' coercive power (e.g., sanctioning, ostracism, etc.) when they, themselves, are high in legitimate power. LO: 13.2: Explain the three bases of formal power and the two bases of personal power. Difficulty: Moderate 571 richard@qwconsultancy.com
Quest. Category: Concept 19) Which of the following types of power can be categorized as personal power? A) Coercive B) Legitimate C) Reward D) Referent E) Positional Answer: D Explanation: Personal power comes from an individual's unique characteristics. There are two bases of personal power: expert power and referent power. LO: 13.2: Explain the three bases of formal power and the two bases of personal power. Difficulty: Easy Quest. Category: Concept 20) The two bases of personal power are ________ and the respect and admiration of others. A) legitimate B) reward C) emergent D) expertise E) coercive Answer: D Explanation: There are two bases of personal power: expertise and the respect and admiration of others. LO: 13.2: Explain the three bases of formal power and the two bases of personal power. Difficulty: Moderate Quest. Category: Concept 21) At the PR firm where Gerald works, everyone considers him to be very good with his work and depends heavily on his knowledge to help the organization to solve its problems. Gerald is often seen teaching interns a simpler way to present an article, helping project managers with scheduling, and even the CEO often asks him for his opinion on important matters because of his experience and skill. Gerald's specialized knowledge depicts his ________. A) expert power B) coercive power C) legitimate power D) referent power E) reward power Answer: A Explanation: Expert power is influence wielded as a result of expertise, special skill, or knowledge. As jobs become more specialized, we become increasingly dependent on experts to achieve goals. It is generally acknowledged that physicians have expertise and hence expert power: Most of us follow our doctor's advice. LO: 13.2: Explain the three bases of formal power and the two bases of personal power. AACSB: Analytical thinking Difficulty: Moderate 572 richard@qwconsultancy.com
Employability Skills: Knowledge Application and Analysis Quest. Category: Application 22) Which type of power is based on identification with a person who has desirable resources or personal traits? A) Coercive B) Legitimate C) Referent D) Expert E) Reward Answer: C Explanation: Referent power is based on identification with a person who has desirable resources or personal traits. If I like, respect, and admire you, you can exercise power over me because I want to please you. LO: 13.2: Explain the three bases of formal power and the two bases of personal power. Difficulty: Easy Quest. Category: Concept 23) Carl believes that he is lucky to be working under a manager like Mr. Parker—conscientious, flexible and easy to work with. It was Mr. Parker who helped Carl learn the demands of the job, gave him opportunities to learn from the mistakes he made, and always provided sound advice. Mr. Parker, who has been in the field for over eighteen years, never made Carl feel like a novice and this only increased Carl's personal confidence in his work and his loyalty toward his manager. Carl, who is now doing very well on the job, aims to be a manager like Mr. Parker. From this scenario, which of the following conclusions can be drawn? A) Mr. Parker is an authoritarian manager. B) Carl has a low degree of trust propensity. C) Mr. Parker has a high degree of referent power. D) Mr. Parker engages in a lot of ingratiation. E) Mr. Parker exercises a high degree of coercive power. Answer: C Explanation: Mr. Parker has a high degree of referent power as Carl identifies with him and wishes to be like him. Referent power is based on identification with a person who has desirable resources or personal traits. LO: 13.2: Explain the three bases of formal power and the two bases of personal power. AACSB: Analytical thinking Difficulty: Moderate Employability Skills: Knowledge Application and Analysis Quest. Category: Application
573 richard@qwconsultancy.com
24) Which of the following is true with regard to bases of power? A) Coercive power involves an identification with and imitation of the person in power. B) Legitimate power is used by companies when they hire celebrities to promote their products. C) Expert and referent power are the two major types of formal power. D) Referent power can be an especially powerful motivator. E) Reward power is the most common and broad type of power seen in operation in organizations. Answer: D Explanation: Referent power can be an especially powerful motivator. LO: 13.2: Explain the three bases of formal power and the two bases of personal power. Difficulty: Moderate Quest. Category: Concept 25) Will Paten is the manager of a team of twelve people in the marketing division of Palmer Inc. Will, who has been working in the company for eleven years, is held in high regard and liked by all his colleagues and subordinates. However, Aaron and Abby, two of his subordinates, are noted for frequently flouting his rules and directives. They continue behaving in this manner even after Will had categorically requested them to refrain from upsetting the momentum of the team. Which of the following, if true, would best explain Aaron and Abby's continued noncompliance with Will's requests? A) The promotions and bonuses of the team members depend heavily on Will's performance ratings. B) The company had provided Will with additional duties, and he was left with little time to ensure that his team members performed well. C) Will believes that since the job demands in marketing are ambiguous and ever-changing, a supportive approach to leadership would suffice. D) Will is uncomfortable exercising coercive measures to control employees. E) Will is considered an expert at analyzing market trends and making accurate projections about demand and revenue. Answer: D Explanation: The fact that promotions and bonuses of the team members depend heavily on Will's performance ratings does not explain Aaron and Abby's noncompliance with his requests. The fact that the company had provided Will with additional duties, and he was left with little time to ensure that his team members performed well does not adequately explain Aaron and Abby's behavior especially since other team members were working well. The fact that Will believes that a supportive approach to leadership would suffice when faced with ambiguous job demands is false and does not explain Aaron and Abby's behavior. The fact that Will is uncomfortable exercising coercive measures to control employees does explain Aaron and Abby's behavior. Will has the right to exercise legitimate, coercive, and reward powers, but his personality is such that he is not at ease forcing people to follow his command. The fact that Will is considered an expert at analyzing market trends and making accurate projections about demand and revenue does not explain Aaron and Abby's behavior toward him. LO: 13.2: Explain the three bases of formal power and the two bases of personal power. AACSB: Analytical thinking Difficulty: Hard Employability Skills: Critical Thinking 574 richard@qwconsultancy.com
Quest. Category: Critical Thinking 26) Joanna Mills has recently been promoted to the position of a project manager at Palmer Inc., a software development firm. As a project manager, she now has the authority to delegate work, provide supervision and feedback, and monitor employee performance. Ever since her promotion, she has been preparing herself to guide her subordinates in different project activities. However, she feels that the employees do not proactively approach her for help. Which of the following, if true, would best explain this situation? A) Joanna has several years of experience in working on software programs. B) The sole criterion for Joanna's promotion was her effectiveness in meeting individual targets. C) Employees identify with Joanna because she successfully rose from the position of a programmer to that of a manager. D) The managers who promoted her believed that her approachable nature would be a key factor in improving team performance. E) The company follows a 360-degree performance evaluation system. Answer: B Explanation: The fact that Joanna has several years of experience in working on software programs does not explain why her team members do not approach her for help. The fact that the sole criterion for Joanna's promotion was her effectiveness in meeting individual targets does explain why her team members do not proactively approach her for help. While the promotion provided her with legitimate power, she does not enjoy referent power, which would encourage team members to reach out to her for help. The fact that the employees identify with Joanna because she successfully rose from the position of a programmer to that of a manager does not explain the current situation. The fact that the managers who promoted her believed that her approachable nature would be a key factor in improving team performance does not explain why her team does not approach her for help. The fact that the company follows a 360-degree performance evaluation system does not explain why her team does not approach her for help. LO: 13.2: Explain the three bases of formal power and the two bases of personal power. AACSB: Analytical thinking Difficulty: Hard Employability Skills: Critical Thinking Quest. Category: Critical Thinking 27) Which of the following types of power is most likely to be negatively related to employee satisfaction and commitment? A) Expert power B) Reward power C) Legitimate power D) Coercive power E) Referent power Answer: D Explanation: The coercive power base depends on fear of the negative results from failing to comply. It is negatively related to employee satisfaction and commitment. When subordinates are being abused by supervisors, coercive power is often the main force that keeps them from retaliating. LO: 13.2: Explain the three bases of formal power and the two bases of personal power. 575 richard@qwconsultancy.com
Difficulty: Moderate Quest. Category: Concept 28) Referent power is based on identification with a person who has desirable resources or personal traits, while expert power is based on expertise, special skills, or knowledge. Answer: TRUE Explanation: Referent power is based on identification with a person who has desirable resources or personal traits. Expert power is influence based on specific skills or knowledge. LO: 13.2: Explain the three bases of formal power and the two bases of personal power. Difficulty: Easy Quest. Category: Concept 29) Describe the bases of formal power. Answer: Formal power can come from the ability to coerce or reward, from formal authority, or from control of information. a) Coercive power: The coercive power base is dependent on fear. One reacts to this power out of fear of the negative results that might occur if one failed to comply. A has coercive power over B if A can dismiss, suspend, or demote B, assuming that B values his or her job. b) Reward power: The opposite of coercive power is reward power. People comply with the wishes or directives of another because doing so produces positive benefits. Therefore, one who can distribute rewards that others view as valuable will have power over those others. These rewards can be either financial or nonfinancial. c) Legitimate power: In formal groups and organizations, probably the most common access to one or more of the power bases is through legitimate power. It represents the formal authority to control and use organizational resources based on structural position in the organization. LO: 13.2: Explain the three bases of formal power and the two bases of personal power. Difficulty: Moderate Quest. Category: Concept
576 richard@qwconsultancy.com
30) Explain the connection between reward power and sexual harassment. Answer: An individual with a high degree of reward power is someone who can distribute rewards others view as valuable and thus, has power over them. These rewards can be either financial—such as controlling pay rates, raises, and bonuses—or nonfinancial, including recognition, promotions, interesting work assignments, friendly colleagues, and preferred work shifts or sales territories. This makes sexual harassment especially disturbing for employees. Because employees want favorable performance reviews, salary increases, and the like, supervisors control resources most employees consider important and scarce. Thus, sexual harassment by the boss typically creates the greatest difficulty for those being harassed. Most studies confirm that power is central to understanding sexual harassment. This seems true whether the harassment comes from a supervisor, co-worker, or employee, although it is especially pronounced for employees who have newly acquired power. Sexual harassment is more likely to occur when there are large power differentials. The supervisor—employee dyad best characterizes an unequal power relationship, where formal power gives the supervisor the capacity to reward and coerce. Because employees want favorable performance reviews, salary increases, and the like, supervisors control resources most employees consider important and scarce. LO: 13.2: Explain the three bases of formal power and the two bases of personal power. AACSB: Analytical thinking Difficulty: Moderate Employability Skills: Critical Thinking Quest. Category: Synthesis 31) Levin and Co. is a group of oil refineries that has been in the news recently. The company had to stop production for over two weeks because their key supplier refused to sell them crude oil at the old prices. Even after rounds of negotiations, the supplier refused to give in to the demands of Levin and Co., and finally, the company had to acquire the raw material from the same supplier because alternative suppliers, though abundant, could not provide the quantities the company demanded at such short notice. This shows the role of ________ in increasing the power one enjoys. A) alternatives B) dependence C) abundance D) substitutability E) exchange Answer: B Explanation: The most important aspect of power is that it is a function of dependence. Dependence increases when the resource you control is important, scarce, or non-substitutable. LO: 13.3: Explain the role of dependence in power relationships. AACSB: Analytical thinking Difficulty: Moderate Employability Skills: Knowledge Application and Analysis Quest. Category: Application
577 richard@qwconsultancy.com
32) Dependence would be low when ________. A) the goods have very few suppliers B) the goods have low accessibility C) the goods in question are scarce D) the products are important E) the goods have substitutes Answer: E Explanation: Dependence increases when the resource you control is important, scarce, or nonsubstitutable. The fewer viable substitutes for a resource, the more power control over that resource provides. LO: 13.3: Explain the role of dependence in power relationships. Difficulty: Moderate Quest. Category: Concept 33) One tool to assess the exchange of resources and dependencies within an organization is social network analysis. Answer: TRUE Explanation: One tool to assess the exchange of resources and dependencies within an organization is social network analysis. This method examines patterns of communication among organizational members to identify how information flows between them. LO: 13.3: Explain the role of dependence in power relationships. Difficulty: Easy Quest. Category: Concept 34) Discuss expert power as it relates to the dimensions of importance, scarcity, and nonsubstitutability. Answer: Expert power is influence wielded as a result of expertise, special skill, or knowledge. As jobs become more specialized, we become increasingly dependent on experts to achieve goals. It is generally acknowledged that physicians have expertise and hence expert power. Expert power arises from the fact that: a) experts have specialized knowledge that others seek to use. In organizations, experts are people who can absorb uncertainty and are believed to be controlling an important resource. b) the expertise, special skill, and knowledge that experts have is not abundantly available and this factor makes expertise a very important determinant of personal power. c) until others in the organization learn and amass enough experience, the experts are a group of people who have no substitutes. The fewer viable substitutes for a resource, the more power control over that resource provides. LO: 13.3: Explain the role of dependence in power relationships. AACSB: Analytical thinking Difficulty: Moderate Quest. Category: Synthesis
578 richard@qwconsultancy.com
35) ________ involves increasing the target's support by involving him or her in decision making. A) Legitimacy B) Rational persuasion C) Consultation D) Ingratiation E) Pressure Answer: C Explanation: The power tactic of consultation involves increasing the target's support by involving him or her in decision making. LO: 13.4: Identify influence tactics and their contingencies. Difficulty: Easy Quest. Category: Concept 36) The ________ power tactic uses warnings and threats and is typically the least effective of all the power tactics. A) coalitions B) exchange C) pressure D) rational persuasion E) consultation Answer: C Explanation: Pressure tends to backfire and is typically the least effective of the nine tactics. LO: 13.4: Identify influence tactics and their contingencies. Difficulty: Easy Quest. Category: Concept 37) Which power tactic uses flattery and creates goodwill before making a request? A) Ingratiation B) Consultation C) Inspirational appeal D) Motivational appeal E) Affective appeal Answer: A Explanation: Ingratiation is defined as using flattery, praise, or friendly behavior prior to making a request, hoping that the kindness will help get what is desired. LO: 13.4: Identify influence tactics and their contingencies. Difficulty: Easy Quest. Category: Concept
579 richard@qwconsultancy.com
38) Rational persuasion is a power tactic that involves the use of ________. A) formal authority B) logical arguments and factual evidence C) emotions to target the values, needs, and hopes of the other party D) benefits or favors in exchange for following a request E) warnings, repeated demands, and threats Answer: B Explanation: Rational persuasion is a power tactic that involves presenting logical arguments and factual evidence to demonstrate that a request is reasonable. LO: 13.4: Identify influence tactics and their contingencies. Difficulty: Moderate Quest. Category: Concept 39) Mike Ford is a senior accountant at Coleman and Co., which is a financial management company. Mike has wanted to be on the board of representatives of his company for a long time, but the company stipulates a minimum number of years an employee must work at the organization before he or she makes it to the board. Mike is unwilling to work two more years to reach that stipulated experience and so he found a way to transgress this rule. He met up with one of the existing board members who was having problems accounting for allocated funds that went missing and offered to settle that for him in his department provided he nominate Mike to the board. Which of the following power tactics is being used here? A) Ingratiation B) Legitimacy C) Inspirational appeals D) Pressure E) Exchange Answer: E Explanation: The power tactic used here is exchange, which involves rewarding the target with benefits or favors in exchange for following a request. LO: 13.4: Identify influence tactics and their contingencies. AACSB: Analytical thinking Difficulty: Moderate Employability Skills: Knowledge Application and Analysis Quest. Category: Application
580 richard@qwconsultancy.com
40) When the contract with one of their major suppliers was about to expire, the management of Pierre and Collins began seeking tenders from potential vendors. One of the top management executives, Todd Hughes, wanted the contract to go to a vendor he personally knew. In order to do so, he held separate discussions with two key members of the organization wherein he downplayed the potentials of the competing tenders and convinced them to support this offer. At the meeting to finalize a supplier, Todd and his supporters strongly supported the tender of the supplier they had chosen amongst themselves and convinced their CEO to hire this supplier despite the higher price quoted by him. Which of the following power tactics is being used here? A) Ingratiation B) Legitimacy C) Coalition D) Pressure E) Inspirational appeals Answer: C Explanation: Todd Hughes used the power tactic of creating a coalition by enlisting the aid or support of others to persuade the target to agree. Coalitions represent a hard power tactic which emphasizes formal power and involves greater costs and risks. LO: 13.4: Identify influence tactics and their contingencies. AACSB: Analytical thinking Difficulty: Moderate Employability Skills: Knowledge Application and Analysis Quest. Category: Application 41) Which of the following power tactics is effective in influencing performance at work? A) Ingratiation B) Consultation C) Personal appeals D) Exchange E) Pressure Answer: B Explanation: Rational persuasion, inspirational appeals, and consultation tend to be equally effective in influencing performance at work. LO: 13.4: Identify influence tactics and their contingencies. Difficulty: Moderate Quest. Category: Concept 42) ________ is effective as a lateral influencing tactic. A) Inspirational appeals B) Ingratiation C) Coalitions D) Legitimacy E) Rational persuasion Answer: C Explanation: As Exhibit 13-4 shows, coalitions are an effective lateral influence tactic. LO: 13.4: Identify influence tactics and their contingencies. Difficulty: Moderate 581 richard@qwconsultancy.com
Quest. Category: Concept 43) ________ is the only tactic effective across organizational levels. A) Legitimacy B) Inspirational appeal C) Rational persuasion D) Ingratiation E) Pressure Answer: C Explanation: The effectiveness of some influence tactics depends on the direction of influence. For instance, rational persuasion is the only tactic effective across organizational levels. LO: 13.4: Identify influence tactics and their contingencies. Difficulty: Moderate Quest. Category: Concept 44) Which of the following "softer" power tactics can be used most effectively for exerting lateral influence? A) Legitimacy B) Inspirational appeal C) Coalition D) Ingratiation E) Pressure Answer: B Explanation: In general, you are more likely to be effective if you begin with "softer" tactics that rely on personal power, such as personal and inspirational appeals, rational persuasion, and consultation. If these fail, you can move to "harder" tactics, such as exchange and coalitions, which emphasize formal power and incur greater costs and risks. LO: 13.4: Identify influence tactics and their contingencies. Difficulty: Moderate Quest. Category: Concept 45) ________ is considered a soft power tactic. A) Rational persuasion B) Exchange C) Coalition D) Politicking E) Pressure Answer: A Explanation: Soft tactics are those that rely on personal power such as personal and inspirational appeals, rational persuasion, and consultation. LO: 13.4: Identify influence tactics and their contingencies. Difficulty: Moderate Quest. Category: Concept
582 richard@qwconsultancy.com
46) ________ would be considered a hard power tactic. A) Rational persuasion B) Inspirational appeals C) Personal appeals D) Coalition E) Consultation Answer: D Explanation: Hard tactics are those that emphasize formal power such as exchange, coalitions, and pressure. LO: 13.4: Identify influence tactics and their contingencies. Difficulty: Moderate Quest. Category: Concept 47) Which of the following statements is true with regard to the effectiveness of tactics? A) Soft tactics are less effective than hard tactics when used individually. B) Research does not support gender differences in the effectiveness of the use of influence tactics–all benefit from soft or neutral tactics as opposed to harder tactics. C) All tactics are equally effective with regard to upward influence. D) Individuals from collectivist cultures are typically more likely to use soft tactics that reflect personal power. E) Individuals from individualistic cultures are typically more likely to use coalition and other hard power tactics. Answer: B Explanation: Research does not support gender differences in the effectiveness of the use of influence tactics–all benefit from soft or neutral tactics as opposed to harder tactics. LO: 13.4: Identify influence tactics and their contingencies. Difficulty: Moderate Quest. Category: Concept 48) For a person to be recognized as having a high degree of political skill, he or she must have the ________. A) ability to recognize that power in an organization does not translate into success B) ability to submit to the demands made by others at all times C) ability to influence others to enhance their own objectives D) ability to respect and consider the needs of others first E) ability to make sacrifices for the betterment of others Answer: C Explanation: Political skills of people indicate their ability to influence others to enhance their own objectives. LO: 13.4: Identify influence tactics and their contingencies. Difficulty: Moderate Quest. Category: Concept
583 richard@qwconsultancy.com
49) Employees exhibiting which of the following features are more likely to engage in political behavior? A) Lack of organizational investment B) External locus of control C) Low need for power D) Machiavellian personality E) Low self-monitoring Answer: D Explanation: The Machiavellian personality—characterized by the will to manipulate and the desire for power—is comfortable using politics as a means to further his or her self-interest. LO: 13.4: Identify influence tactics and their contingencies. Difficulty: Moderate Quest. Category: Concept 50) Being politically skilled can improve your reputation and career success as you earn a higher income and a more prestigious position. Answer: TRUE Explanation: Being politically skilled can improve your reputation and career success as you earn a higher income and a more prestigious position and are more satisfied with your career. LO: 13.4: Identify influence tactics and their contingencies. Difficulty: Moderate Quest. Category: Concept 51) Dale Fisher is the manager of one branch of a retail chain. He often speaks with his customer service attendants about the value they bring to the company and encourages them to continue doing so. He knows that this approach of enhancing the worth of these executives and shaping their aspirations to do well is a more effective approach than using warnings and ultimatums. Dale is using the power tactic of an inspirational appeal here. Answer: TRUE Explanation: Inspirational appeals represent a power tactic that involves developing emotional commitment by appealing to a target's values, needs, hopes, and aspirations. LO: 13.4: Identify influence tactics and their contingencies. AACSB: Analytical thinking Difficulty: Moderate Quest. Category: Application 52) Automatic processing refers to a detailed consideration of evidence and information relying on facts, figures, and logic. Answer: FALSE Explanation: Automatic processing refers to a relatively superficial consideration of evidence and information that takes little time or effort and makes use of heuristics. Controlled processing refers to a detailed consideration of evidence and information relying on facts, figures, and logic. LO: 13.4: Identify influence tactics and their contingencies. Difficulty: Easy Quest. Category: Concept
584 richard@qwconsultancy.com
585 richard@qwconsultancy.com
53) Explain the various power tactics employees use to translate their power bases into specific actions. Answer: Research has identified nine distinct tactics of political influence. 1. Legitimacy involves relying on one's authority position or stressing that a request is in accordance with organizational policies or rules. 2. Rational persuasion involves presenting logical arguments and factual evidence to demonstrate that a request is reasonable. 3. Inspirational appeals involve developing emotional commitment by appealing to a target's values, needs, hopes, and aspirations. 4. Consultation involves increasing a target's motivation and support by involving him or her in deciding how a plan or change will be implemented. 5. Exchange involves rewarding targets with benefits or favors in exchange for following a request. 6. Personal appeals involve asking for compliance based on friendship or loyalty. 7. Ingratiation involves using flattery, praise, or friendly behavior prior to making a request. 8. Pressure involves using warnings, repeated demands, and threats. 9. Finally, coalitions involve enlisting the aid of other people to persuade a target or using the support of others as a reason for the target to agree. LO: 13.4: Identify influence tactics and their contingencies. Difficulty: Moderate Quest. Category: Concept 54) Compare and contrast rational persuasion and inspirational appeals in terms of their operation and direction of influence. Answer: Rational persuasion is a power tactic that involves presenting logical arguments and factual evidence to demonstrate a request is reasonable, while inspirational appeals as a power tactic operate by appealing to one's emotions. When using an inspirational appeal, one is attempting to develop an emotional commitment by appealing to a target's values, needs, hopes, and aspirations. Further, rational persuasion is a tactic that can be used effectively across different levels, while inspirational appeal works best as a downward-influencing tactic with subordinates. LO: 13.4: Identify influence tactics and their contingencies. AACSB: Analytical thinking Difficulty: Moderate Quest. Category: Synthesis
586 richard@qwconsultancy.com
55) Discuss the effectiveness of various influence tactics. Answer: The effectiveness of some influence tactics depends on the direction of influence. Rational persuasion is the only tactic effective across organizational levels. Inspirational appeals work best as a downward-influencing tactic with subordinates. When pressure works, it's generally downward only. Personal appeals and coalitions are most effective as lateral influence. Other factors that affect the effectiveness of influence include the sequencing of tactics, a person's skill in using the tactic, and the organizational culture. You're more likely to be effective if you begin with "softer" tactics that rely on personal power, such as personal and inspirational appeals. If these fail, you can move to "harder" tactics, such as exchange and coalitions. People differ in their political skill, or their ability to influence others to enhance their own objectives. The politically skilled are more effective users of all of the influence tactics. The organizational culture at the company where a person works will have a bearing on which tactics are considered appropriate. Some cultures encourage participation and consultation, some encourage reason, and still others rely on pressure. People who fit the culture of the organization tend to obtain more influence. LO: 13.4: Identify influence tactics and their contingencies. Difficulty: Moderate Quest. Category: Concept 56) Which of the following statements is true? A) The impact of sexual harassment is individual, not organizational. B) More than 50 percent of women report being subject to sexual harassment within the last 12 months. C) Today, the rise of overt forms of sexual harassment presents a major concern for organizations. D) Women in positions of power in an organization are free from sexual harassment by male subordinates. E) Curtailing sexual harassment requires higher-order intervention programs as managers cannot be expected to help in this regard. Answer: B Explanation: According to multiple surveys on sexual harassment up to 52 percent of women and up to 43 percent of men have reported being subject to sexual harassment behaviors within the last twelve months, and up to 22 percent of women and up to 7 percent of men indicated they have experienced sexual harassment at work in their lifetimes. LO: 13.5: Identify the causes and consequences of abuse of power. Difficulty: Moderate Quest. Category: Concept
587 richard@qwconsultancy.com
57) Sexual harassment ________. A) is proportionally prevalent for women across all types of jobs B) is virtually non-existent for men C) is increasing according to the EEOC D) is more likely to occur when there are large power differentials E) is less likely to occur where there are large power differentials Answer: D Explanation: Most studies confirm that power is central to understanding sexual harassment. This seems true whether the harassment comes from a supervisor, co-worker, or employee although it is especially pronounced for employees who have newly acquired power. Sexual harassment is more likely to occur when there are large power differentials. LO: 13.5: Identify the causes and consequences of abuse of power. Difficulty: Moderate Quest. Category: Concept 58) In which of the following situations is a woman most likely to experience sexual harassment? A) As a manager in a customer service call center B) As a teacher in an elementary school C) As a waitress at a sports bar D) As a professor at a small liberal arts college E) As a DJ at a hip radio station Answer: C Explanation: In an examination of the nearly forty-one thousand EEOC sexual harassment charges filed in a ten-year span, over 25 percent of these cases were in the customer-facing sector (i.e., accommodation, food services, retail). LO: 13.5: Identify the causes and consequences of abuse of power. Difficulty: Easy Quest. Category: Concept
588 richard@qwconsultancy.com
59) Connie Cole works as a junior market analyst at SPS Services. Her previous manager retired a couple of months back, and she now has a younger male manager who repeatedly insists that she accompany him for a coffee or dinner after work. Initially, Connie was unsuspecting and refused because she did not have the time for such engagements during weekdays. Later, when her manager's requests became frequent, she grew uncomfortable and firmly told him that she was not interested in him, and his attempts were not welcome. She realized that this step could backfire because, as her superior, he has complete control over her appraisals, bonuses, and promotions. Her manager even told her that if she does not agree, he will "find other ways." Connie's experiences are best described as ________. A) whistleblowing B) scapegoating C) domestic abuse D) sexual harassment E) ambient abuse Answer: D Explanation: Sexual harassment is defined as any unwanted activity of a sexual nature that affects an individual's employment and creates a hostile work environment. Organizations have generally made progress toward limiting overt forms of sexual harassment. This includes unwelcome sexual advances, requests for sexual favors, and other verbal or physical conduct of a sexual nature. LO: 13.5: Identify the causes and consequences of abuse of power. AACSB: Analytical thinking Difficulty: Moderate Employability Skills: Knowledge Application and Analysis Quest. Category: Application 60) Which of the following factors can lead to greater probability of sexual harassment at the workplace? A) An active sexual harassment policy B) Large power differentials C) A no-retaliation policy for victims seeking justice D) Awareness regarding sexual harassment E) Larger percentage of women in positions of power Answer: B Explanation: Most studies confirm that the concept of power is central to understanding sexual harassment. This seems true whether the harassment comes from a supervisor, a co-worker, or an employee. Sexual harassment is more likely to occur when there are large power differentials. LO: 13.5: Identify the causes and consequences of abuse of power. Difficulty: Moderate Quest. Category: Concept
589 richard@qwconsultancy.com
61) To limit sexual harassment in the workplace, managers should strive to ________. A) recognize that sexual harassment occurs in most work environments B) instantly terminate anyone accused of sexual harassment C) provide general guidelines about sexual harassment, but avoid detailed explanations D) reassure employees that they will not encounter retaliation if they file a complaint E) investigate only the most serious complaints Answer: D Explanation: Sexual harassment can have a detrimental impact on individuals and the organization, but it can be avoided. The manager's role is critical: make sure an active policy defines what constitutes sexual harassment, informs employees they can be fired for inappropriate behavior, and establishes procedures for making complaints; Reassure employees that they will not encounter retaliation if they file a complaint.; investigate every complaint, and inform the legal and HR departments.; make sure offenders are disciplined or terminated.; set up in-house seminars to raise employee awareness of sexual harassment issues. LO: 13.5: Identify the causes and consequences of abuse of power. Difficulty: Moderate Quest. Category: Concept 62) Tina Simmons worked as a welder at a shipyard company where she was forced to encounter nude and partially nude pictures posted by her male co-workers in common areas. The men also referred to her using obscene nicknames and made offensive remarks concerning her body. When Tina complained, her supervisor took no corrective action despite having an active sexual harassment policy approved by the EEOC. Tina wished to file a discrimination suit but instead she chose to endure the harassment as she could not risk losing her job. Which of the following, if true, would have strengthened her resolve to file a discrimination suit? A) The colleagues who sexually harassed Tina shared a great rapport with her supervisor. B) Tina had heard of only one prior instance wherein an employee in the company had filed a discrimination suit. C) Tina shared a strained working relationship with her supervisor. D) The company policy prescribed that Tina would face no backlash if she approached the EEOC. E) Tina had few female colleagues at the shipyard where she worked. Answer: D Explanation: The fact that the colleagues who sexually harassed Tina shared a great rapport with her supervisor is likely to weaken her resolve to file a discrimination suit. The fact that Tina had heard of only one prior instance wherein an employee in the company had filed a discrimination suit does not necessarily weaken her resolve to file a discrimination suit. The fact that Tina shared a strained working relationship with her supervisor may weaken her resolve. The fact that the company policy prescribed that Tina would face no backlash if she approached the EEOC would strengthen her resolve as she would not lose her job following her action. The fact that Tina had few female colleagues at the shipyard where she worked may weaken her resolve. LO: 13.5: Identify the causes and consequences of abuse of power. AACSB: Analytical thinking Difficulty: Hard Employability Skills: Critical Thinking Quest. Category: Critical Thinking 590 richard@qwconsultancy.com
63) Discuss how power affects people and consider whether power is corruptive. Answer: There is evidence that there are corrupting aspects of power. Evidence suggests that power leads people to place their own interests ahead of others. Why does this happen? Interestingly, research suggests that power not only leads people to focus on their self-interests because they can, but it also liberates people to focus inward and thus come to place greater weight on their goals and interests. Power also appears to lead individuals to "objectify" others, to value relations with people with less power, and to see relationships as more peripheral. Powerful people react especially negatively to any threats to their competence. They're more willing to denigrate others. People given power are more likely to make self-interested decisions when faced with moral hazard. Power also leads to overconfident decision making. LO: 13.5: Identify the causes and consequences of abuse of power. AACSB: Analytical thinking Difficulty: Hard Employability Skills: Critical Thinking Quest. Category: Synthesis 64) Define sexual harassment and identify ways managers can protect themselves and their employees from it. Answer: Sexual harassment is defined as any unwanted activity of a sexual nature that affects an individual's employment and creates a hostile work environment. Sexual harassment can wreak havoc on an organization, not to mention the victims themselves, but it can be avoided. Managers can do several things to protect themselves and their employees. 1. Make sure an active policy defines what constitutes sexual harassment, informs employees they can be fired for sexually harassing another employee, and establishes procedures for making complaints. 2. Reassure employees they will not encounter retaliation if they file a complaint. 3. Investigate every complaint and inform the legal and human resource departments. 4. Make sure offenders are disciplined or terminated. 5. Set up in-house seminars to raise employee awareness of sexual harassment issues. LO: 13.5: Identify the causes and consequences of abuse of power. Difficulty: Moderate Quest. Category: Concept 65) Which of the following is true of political behavior in an organization? A) Political behavior is usually unethical. B) Using political behavior is not necessary in most organizations. C) Most experienced managers believe political behavior is a major part of organizational life. D) Effective managers have been trained to use political behavior. E) Political behavior is necessary for ensuring role clarity in an organization. Answer: C Explanation: Interviews with experienced managers show that most believe that political behavior is a major part of organizational life. LO: 13.6: Describe how politics work in organizations. Difficulty: Moderate Quest. Category: Concept 591 richard@qwconsultancy.com
66) Political behaviors usually ________. A) lie outside of an individual's specified job requirements B) are counterproductive to individual goals C) are seen only in large organizations D) are sanctioned by organizational leaders E) are expected as part of each job requirement Answer: A Explanation: Political behavior is defined as activities that are not required as part of a person's formal role in the organization but that influence, or attempt to influence, the distribution of advantages and disadvantages within the organization. LO: 13.6: Describe how politics work in organizations. Difficulty: Moderate Quest. Category: Concept 67) Which of the following factors contributes to political behavior in the organization? A) Presence of pure and objective facts B) Existence of uniform goals and interests among all employees C) Provision of clear and objective performance outcomes D) Presence of limited resources in the organization E) Perception that gains of one group are not at the expense of the loss incurred by another group Answer: D Explanation: Organizations are made up of individuals and groups with different values, goals, and interests, and this sets up the potential for conflict over the allocation of limited resources, such as departmental budgets, space, project responsibilities, and salary adjustments. As resources are limited, not everyone's interests can be satisfied. Furthermore, gains by one individual or group are often perceived as coming at the expense of others within the organization. LO: 13.6: Describe how politics work in organizations. Difficulty: Moderate Quest. Category: Concept 68) An organization's informal communication network is referred to as ________. A) the intranet B) the grapevine C) the extranet D) gossip E) politicking Answer: B Explanation: An organization's informal communication network is referred to as the grapevine. LO: 13.6: Describe how politics work in organizations. Difficulty: Easy Quest. Category: Concept
592 richard@qwconsultancy.com
69) Timothy Jenkins spent six years at Hartford Mills, a company that takes a strong stand against unionization. Though Timothy's designation was that of a production supervisor, as the company grew, he also took on additional duties like keeping a check on employee safety, ensuring that legal measures are followed, and implementing programs for labor welfare that the company planned. Many believed that Timothy was doing a great job for the company, and so it came as a surprise when Timothy was rumored to face severe disciplinary action, as he was encouraging the workers to join a union. However, he was let off with a mere warning. Which of the following, if true, would explain this paradox? A) The management had recently instituted a formal employee welfare committee. B) Timothy stressed the fact that he was a meticulous worker who always adhered to company policies. C) The management had recently moved from an open shop mechanism to a closed shop mechanism. D) Timothy claimed that he was only looking to ensure greater employee welfare, which was one of the duties the company had assigned to him. E) Timothy complained that allowing unionization would hurt employees' work/home balance. Answer: D Explanation: The fact that the management had recently instituted a formal employee welfare committee does not explain this paradox. The fact that Timothy was a meticulous worker who always adhered to company policies does not explain this paradox. The fact that the management had recently moved from an open shop mechanism to a closed shop mechanism does not explain why the company let Timothy go with just a warning. The fact that Timothy claimed that he was only looking to ensure greater employee welfare, which was one of the duties the company had assigned to him, does explain this paradox. Since the company had not clearly defined the scope and functions of Timothy's role, he had greater freedom to engage in political activities that the company did not formally prohibit. The fact that Timothy complained that allowing unionization would hurt employees' work/home balance does not explain this paradox. LO: 13.6: Describe how politics work in organizations. AACSB: Reflective thinking Difficulty: Hard Employability Skills: Critical Thinking Quest. Category: Critical Thinking
593 richard@qwconsultancy.com
70) Timothy Jenkins had been working for six years at Hartford Mills, a company that takes a strong stand against unionization. Though Timothy's designation was that of a production supervisor, as the company grew, he also took on additional duties like keeping a check on employee safety, ensuring that legal measures are followed, and implementing programs for labor welfare that the company planned. Many believed that Timothy was doing a great job for the company, and so it came as a surprise when Timothy was rumored to face severe disciplinary action, as he was encouraging the workers to join a union. However, he was let off with a mere warning. Timothy argued that he was only doing his job—a job in which he had responsibility for improving the welfare of employees. Which of the following, if true, would help Hartford Mills avoid such situations in the future? A) Ensuring the provision of a retaliatory policy for employees who go against the company B) Holding town hall meetings to explain to employees why Timothy was suspected C) Enlisting the support of popular employees like Timothy to discharge the employee welfare duties D) Engaging in collective bargaining as it is the only way to motivate employees to work efficiently E) Specifying the permissible and prohibited aspects of an employee's formal role in the organization Answer: E Explanation: Ensuring the provision of a retaliatory policy for employees who go against the company is likely to cause problems for the company as it goes against EEOC directives. Holding town hall meetings to explain to employees why Timothy was suspected is not going to avoid such problems in the future; it may help control the present situation. Enlisting the support of popular employees like Timothy to discharge the employee welfare duties is not a solution to this problem. Engaging in collective bargaining to motivate employees goes against the company's policy of non-unionization and is not a solution. Specifying the permissible and prohibited aspects of an employee's formal role in the organization is a solution as it would limit the scope and functions of the employee's political actions. LO: 13.6: Describe how politics work in organizations. AACSB: Reflective thinking Difficulty: Hard Employability Skills: Critical Thinking Quest. Category: Critical Thinking 71) Whistleblowing is not considered to be a political behavior. Answer: FALSE Explanation: Political behavior includes a variety of behaviors such as withholding key information from decision makers, joining a coalition, whistleblowing, spreading rumors, leaking confidential information to the media, exchanging favors with others in the organization for mutual benefit, and lobbying on behalf of or against a particular individual or decision alternative. LO: 13.6: Describe how politics work in organizations. Difficulty: Moderate Quest. Category: Concept
594 richard@qwconsultancy.com
72) Limited resources are one of the major reasons for the development of organizational politics. Answer: TRUE Explanation: As resources are limited, not everyone's interests can be satisfied. Furthermore, gains by one individual or group are often perceived as coming at the expense of others within the organization. These forces create real competition among members for the organization's limited resources thus leading to political behavior. LO: 13.6: Describe how politics work in organizations. Difficulty: Moderate Quest. Category: Concept 73) Define political behavior and explain its features. Answer: Political behavior in organizations consists of activities that are not required as part of an individual's formal role but that influence, or attempt to influence, the distribution of advantages and disadvantages within the organization. Political behavior is outside specified job requirements. It requires some attempt to use power bases. It includes efforts to influence the goals, criteria, or processes used for decision making. Political behavior may include varied behaviors such as withholding key information from decision makers, joining a coalition, whistleblowing, spreading rumors, leaking confidential information to the media, exchanging favors with others in the organization for mutual benefit, and lobbying on behalf of or against a particular individual or decision alternative. LO: 13.6: Describe how politics work in organizations. Difficulty: Moderate Quest. Category: Concept 74) ________ are reactive and protective behaviors designed to avoid action, blame, or change. A) Repressive behaviors B) Defensive behaviors C) Submissive behaviors D) Impression management behaviors E) Proactive behaviors Answer: B Explanation: When employees see politics as a threat, they often respond with defensive behaviors, which are reactive and protective behaviors to avoid action, blame, or change. LO: 13.7: Identify the causes, consequences, and ethics of political behavior. Difficulty: Easy Quest. Category: Concept
595 richard@qwconsultancy.com
75) Will Cears works at Mesner Corp. as a middle-level manager. Will recognizes that his firm is plagued by organizational politics. Since he is not good at playing games, he frequently engages in defensive behaviors in order to protect himself and his interests. Will is frustrated because he believes that he has little control over his environment and his environment is uncomfortable to him. He has had some success at establishing a sense of control for himself by appearing to be cooperative and supportive to his colleagues. However, he continues to hold his reservations against politicking and does not engage in it. This is an example of ________ in order to avoid action, blame, or change. A) buffing B) buck passing C) scapegoating D) misrepresenting E) stalling Answer: E Explanation: Stalling is appearing to be more or less supportive publicly while doing little or nothing privately. LO: 13.7: Identify the causes, consequences, and ethics of political behavior. AACSB: Analytical thinking Difficulty: Hard Employability Skills: Knowledge Application and Analysis Quest. Category: Application 76) Buck passing is seen when an employee ________. A) avoids an unwanted task by falsely pleading ignorance or inability B) prolongs a task to give the appearance of being occupied C) documents his or her activity rigorously to project an image of competence and thoroughness D) develops explanations that lessen his or her responsibility for a negative outcome E) transfers responsibility for the execution of a task or decision to someone else Answer: E Explanation: Buck passing means transferring responsibility for the execution of a task or decision to someone else. LO: 13.7: Identify the causes, consequences, and ethics of political behavior. Difficulty: Moderate Quest. Category: Concept
596 richard@qwconsultancy.com
77) ________ involves rigorously documenting activity to project an image of competence and thoroughness. A) Bluffing B) Justifying C) Scapegoating D) Stretching E) Overconforming Answer: A Explanation: Bluffing describes the practice of rigorously documenting activity to project an image of competence and thoroughness. LO: 13.7: Identify the causes, consequences, and ethics of political behavior. Difficulty: Moderate Quest. Category: Concept 78) Which of the following involves placing the blame for a negative outcome on external factors that are not entirely blameworthy? A) Scapegoating B) Bluffing C) Stalling D) Justifying E) Overconforming Answer: A Explanation: Scapegoating is placing the blame for a negative outcome on external factors that are not entirely blameworthy. LO: 13.7: Identify the causes, consequences, and ethics of political behavior. Difficulty: Moderate Quest. Category: Concept
597 richard@qwconsultancy.com
79) Daphne is one of the three project managers at Virgo Inc., an art and design studio. Raymond and Gabriel, the other two project managers, are unhappy working with Daphne as she always selects projects that suit her area of expertise and avoids projects which require more ambiguous work, have tight deadlines, and represent a low probability of success. Raymond and Gabriel feel that they deserve greater credit than Daphne for being open and accepting different work demands and situations, as opposed to Daphne, who always takes a conservative and risk-averse approach. Daphne is using the defensive behavior of ________ here. A) scapegoating B) ingratiating C) stalling D) retaliating E) playing safe Answer: E Explanation: Daphne is playing safe here. This involves evading situations that may reflect unfavorably. It includes taking on only projects with a high probability of success, having risky decisions approved by superiors, qualifying expressions of judgment, and taking neutral positions in conflicts. LO: 13.7: Identify the causes, consequences, and ethics of political behavior. AACSB: Analytical thinking Difficulty: Moderate Employability Skills: Knowledge Application and Analysis Quest. Category: Application 80) ________ involves developing explanations that lessen one's responsibility for a negative outcome and apologizing to demonstrate remorse. A) Scapegoating B) Misrepresenting C) Justifying D) Overconforming E) Stretching Answer: C Explanation: One of the defensive behaviors used to avoid blame is justifying. Justifying involves developing explanations that lessen one's responsibility for a negative outcome and/or apologizing to demonstrate remorse, or both. LO: 13.7: Identify the causes, consequences, and ethics of political behavior. Difficulty: Moderate Quest. Category: Concept
598 richard@qwconsultancy.com
81) Which of the following is a defensive behavior that is used to avoid change? A) Stalling B) Scapegoating C) Misrepresenting D) Stretching E) Preventing Answer: E Explanation: Prevention is a defensive behavior that involves trying to prevent a threatening change from occurring. LO: 13.7: Identify the causes, consequences, and ethics of political behavior. Difficulty: Moderate Quest. Category: Concept 82) Which of the following differentiates between high self-monitors and low self-monitors? A) High self-monitors are more likely to use defensive impression management techniques, while low self-monitors are more likely to use self-focused impression management techniques. B) High self-monitors are commonly seen in collectivist cultures, while low self-monitors are commonly seen in individualist cultures. C) The high self-monitor is more sensitive to social cues, exhibits higher levels of social conformity, and is more likely to be skilled in political behavior than the low self-monitor. D) High self-monitors prefer to avoid using power and politics at the workplace, while low selfmonitors engage in a lot of politicking. E) High self-monitors have an external locus of control as they attribute successes or failures to luck and fate, while low self-monitors have an internal locus of control and believe that they control the outcomes. Answer: C Explanation: The high self-monitor is more sensitive to social cues, exhibits higher levels of social conformity, and is more likely to be skilled in political behavior than the low self-monitor. LO: 13.7: Identify the causes, consequences, and ethics of political behavior. Difficulty: Moderate Quest. Category: Concept 83) Which social force refers to the idea that people are most influenced by those with whom they identify? A) Reciprocity B) Social proof C) Authority D) Unity E) Liking Answer: D Explanation: Unity is the newest principle of persuasion. People are most influenced by those with whom they identify. LO: 13.7: Identify the causes, consequences, and ethics of political behavior. Difficulty: Moderate Quest. Category: Concept
599 richard@qwconsultancy.com
600 richard@qwconsultancy.com
84) When employees see politics as a threat, they often ________. A) create role ambiguity B) use democratic decision making C) use a single outcome measure D) respond with defensive behaviors E) allow a significant lapse Answer: D Explanation: When employees see politics as a threat, they often respond with defensive behaviors – reactive and protective behaviors to avoid action, blame, or change. LO: 13.7: Identify the causes, consequences, and ethics of political behavior. Difficulty: Moderate Quest. Category: Concept 85) Under the zero-sum approach, rewards are distributed such that ________. A) rewards are equal and do not increase or decrease with changes in profit margins B) rewards increase for all employees at an identical rate C) any drop in profits reduces the pay of employees by an identical amount D) amount allocated for rewards is variable E) rewards increase for a group at the expense of loss for another group Answer: E Explanation: The zero-sum approach treats the amount allocated for rewards as fixed, so any gain one person or group achieves has to come at the expense of another person or group. LO: 13.7: Identify the causes, consequences, and ethics of political behavior. Difficulty: Moderate Quest. Category: Concept 86) ________ is a self-focused impression management technique. A) Flattery B) Enhancement C) Apologies D) Excuses E) Conformity Answer: B Explanation: Enhancement and self-promotion are self-focused impression management techniques. LO: 13.7: Identify the causes, consequences, and ethics of political behavior. Difficulty: Easy Quest. Category: Concept
601 richard@qwconsultancy.com
87) Which of the following impression management techniques involves an individual admitting responsibility for an undesirable event and simultaneously seeking to get a pardon for the action? A) Flattery B) Favors C) Apologies D) Misrepresentation E) Exemplification Answer: C Explanation: Apologies involve admitting responsibility for an undesirable event and simultaneously seeking to get a pardon for the action. It is a defensive impression management technique. LO: 13.7: Identify the causes, consequences, and ethics of political behavior. Difficulty: Easy Quest. Category: Concept 88) The ________ impression management techniques uses ingratiation. A) flattery B) exemplification C) enhancement D) conformity E) apologies Answer: D Explanation: Conformity, which involves agreeing with someone else's opinion to gain his or her approval, is a form of ingratiation. LO: 13.7: Identify the causes, consequences, and ethics of political behavior. Difficulty: Easy Quest. Category: Concept
602 richard@qwconsultancy.com
89) Recently, Curtis Gibson had an interview for the position of a senior manager at a transit service. At the interview, the interviewer made a passing remark about how policies can make or break the best economy in the world. Immediately, Curtis launched into a discussion of how true the interviewer was, substantiating his claim with stories from real economies around the world. Similarly, when the interviewer suggested that Curtis work the earlier shift because, if selected, he would have a long commute to reach work, Curtis vehemently agreed, saying, "You couldn't be more right." Which of the following impression management techniques is Curtis using? A) Favors B) Exemplification C) Enhancement D) Self-promotion E) Conformity Answer: E Explanation: Conformity, which involves agreeing with someone else's opinion to gain his or her approval, is a form of ingratiation. LO: 13.7: Identify the causes, consequences, and ethics of political behavior. AACSB: Analytical thinking Difficulty: Moderate Employability Skills: Knowledge Application and Analysis Quest. Category: Application 90) ________ is an individual factor that influences political behavior. A) Reallocation of resources B) Unclear performance evaluation system C) High performance pressures D) Perceived job alternatives E) Zero-sum reward practice Answer: D Explanation: An individual's investment in the organization, perceived alternatives, and expectations of success influence the degree to which he or she will pursue illegitimate means of political action. LO: 13.7: Identify the causes, consequences, and ethics of political behavior. Difficulty: Easy Quest. Category: Concept
603 richard@qwconsultancy.com
91) If an individual ________, then he or she is less likely to engage in illegitimate political behavior. A) has invested a lot in the organization in terms of time and efforts B) possesses a scarce skill or knowledge C) has influential contacts outside the organization D) does not expect any future benefits from the organization E) has an external locus of control Answer: A Explanation: An individual's investment in the organization, perceived alternatives, and expectations of success influence the degree to which he or she will pursue illegitimate means of political action. The more a person expects increased future benefits from the organization, the more that person has to lose if forced out and the less likely he or she is to use illegitimate means. LO: 13.7: Identify the causes, consequences, and ethics of political behavior. Difficulty: Moderate Quest. Category: Concept 92) An organizational factor that increases illegitimate political behavior is ________. A) internal locus of control B) low expectations of success C) high trust D) low organizational investment E) zero-sum reward practice Answer: E Explanation: Cultures characterized by low trust, role ambiguity, unclear performance evaluation systems, zero-sum reward allocation practices, democratic decision making, high pressures for performance, and self-serving senior managers will also create breeding grounds for politicking. LO: 13.7: Identify the causes, consequences, and ethics of political behavior. Difficulty: Easy Quest. Category: Concept
604 richard@qwconsultancy.com
93) Winston Mayer is a sales manager for a large company producing beverages and ready-to-eat meals. Winston is often seen working late and on weekends although the company stipulates a five-day workweek. Winston justifies his working extra by claiming that "in sales, one cannot afford to waste a minute" and he is often condescending when subordinates take their allotted time off. He frequently speaks to his manager about the deals he finalizes when he is working beyond the normal working hours to prove his point. From the information provided, we can say that Winston is using ________ as an impression management technique here. A) flattery B) exemplification C) favors D) apologies E) conformity Answer: B Explanation: Exemplification is an impression management technique that involves an individual doing more than he or she needs to in an effort to show how dedicated and hardworking he or she is. LO: 13.7: Identify the causes, consequences, and ethics of political behavior. AACSB: Analytical thinking Difficulty: Hard Employability Skills: Knowledge Application and Analysis Quest. Category: Application 94) In the last month, Stacey, an intelligent and astute employee, has finished only half the work she was scheduled to complete. In addition, she has been taking frequent leaves, citing her ill health as the reason, but her manager suspects she has been playing truant. When the manager confronted her about her pending work, Stacey gave excuses: the work was time consuming, she was unwell, and it was not a high-priority project in the first place. In this scenario, Stacey is using a(n) ________ impression management technique. A) self-focused B) assertive C) aggressive D) defensive E) ingratiation Answer: D Explanation: When one provides explanations of a predicament-creating event aimed at minimizing the apparent severity of the predicament, it represents a defensive IM technique. LO: 13.7: Identify the causes, consequences, and ethics of political behavior. AACSB: Analytical thinking Difficulty: Hard Employability Skills: Knowledge Application and Analysis Quest. Category: Application
605 richard@qwconsultancy.com
95) Which of the following can be used to understand how power and politics in an organization can help you achieve your career goals? A) Counseling B) Constructing your power base relationships C) Mapping your own political career D) Alleviating your own power struggles E) Positioning your own power system Answer: C Explanation: A political map can help you sketch out your relationships with the people upon whom your career depends. LO: 13.7: Identify the causes, consequences, and ethics of political behavior. Difficulty: Moderate Quest. Category: Concept 96) Victor Ortiz was a successful and popular manager at the marketing division of a consumer goods company. Many attributed the company's immense success in the last two years to Victor's highly creative marketing plans, which cost the company very little but increased the visibility of its products. He also effectively tied up with new retail chains so the demand for their products grows as the new retail chains grow. Recently, a subordinate alleged that Victor had sexually harassed her, and stories of similar allegations made against him in the past resurfaced. The situation was laid to rest when Victor made an appearance with the victim by his side claiming that the allegations were baseless as he was romantically involved with her. Though this announcement did not convince people, the allegations against Victor were withdrawn. From the information provided in this scenario, this outcome can be explained by the fact that ________. A) Victor has a high degree of referent power B) Victor has enough power to get away with his misdeeds C) Victor has abided by the standards of equity and justice D) Victor has used a defensive impression management technique E) Victor has used scapegoating to free himself from the charges Answer: B Explanation: Immoral people can justify almost any behavior. Those who are powerful, articulate, and persuasive are most vulnerable to ethical lapses because they are likely to be able to get away with unethical practices successfully. LO: 13.7: Identify the causes, consequences, and ethics of political behavior. AACSB: Analytical thinking Difficulty: Hard Employability Skills: Knowledge Application and Analysis Quest. Category: Application 97) The low self-monitor is more sensitive to social cues, exhibits higher levels of social conformity, and is more likely to be skilled in political behavior than the high self-monitor. Answer: FALSE Explanation: The high self-monitor is more sensitive to social cues, exhibits higher levels of social conformity, and is more likely to be skilled in political behavior than the low self-monitor. LO: 13.7: Identify the causes, consequences, and ethics of political behavior. Difficulty: Moderate 606 richard@qwconsultancy.com
Quest. Category: Concept 98) The Machiavellian personality trait is characterized by the will to manipulate and the desire for power. Answer: TRUE Explanation: The Machiavellian personality is characterized by the will to manipulate and the desire for power and is comfortable using politics as a means to further his or her self-interest. LO: 13.7: Identify the causes, consequences, and ethics of political behavior. Difficulty: Moderate Quest. Category: Concept 99) Typically, the grapevine and gossip are viewed positively by most people. Answer: FALSE Explanation: Typically, the grapevine and gossip are viewed negatively by most people. LO: 13.7: Identify the causes, consequences, and ethics of political behavior. Difficulty: Moderate Quest. Category: Concept 100) When the production manager at CalTrack was asked to provide reasons for the delay in the last batch of stock to be produced for the month, he clearly stated that this was the result of one of his front-level managers taking two days off and that the fault was not his. The manager is using buffing as a defensive behavior here. Answer: FALSE Explanation: The manager is using buck passing as a defensive behavior here. Buck passing involves transferring responsibility for the execution of a task or decision to someone else, while buffing involves the practice of rigorously documenting activity to project an image of competence and thoroughness. LO: 13.7: Identify the causes, consequences, and ethics of political behavior. AACSB: Analytical thinking Difficulty: Moderate Employability Skills: Knowledge Application and Analysis Quest. Category: Application 101) Interviewers are easily able to detect when an individual is engaging in impression management. Answer: FALSE Explanation: Interviewers are rarely able to detect when an individual is engaging in impression management, especially when applicants are using deception to engage in impression management. LO: 13.7: Identify the causes, consequences, and ethics of political behavior. Difficulty: Moderate Quest. Category: Concept
607 richard@qwconsultancy.com
102) Self-promotion works well as an impression management technique for interviews as well as performance evaluations. Answer: FALSE Explanation: Ingratiating always works because everyone—both interviewers and supervisors— likes to be treated nicely. However, self-promotion may work only in interviews and backfire on the job because, whereas the interviewer has little idea about one's accomplishments, the supervisor knows because it is his or her job to observe the employee's work. LO: 13.7: Identify the causes, consequences, and ethics of political behavior. Difficulty: Moderate Quest. Category: Concept 103) Ingratiation is negatively related to performance ratings. Answer: FALSE Explanation: Ingratiation is positively related to performance ratings, meaning those who ingratiate themselves with their supervisors get higher performance evaluations. LO: 13.7: Identify the causes, consequences, and ethics of political behavior. Difficulty: Moderate Quest. Category: Concept 104) The more effort applicants put into highlighting their skills, motivation, and admiration for the organization, the more likely they are to be hired. Answer: TRUE Explanation: The more effort applicants put into highlighting their skills, motivation, and admiration for the organization, the more likely they are to be hired. A study examined this relationship, finding that interviewers saw applicants who talked confidently about their qualifications as a better fit for the job, and applicants who said positive things about the organization as a better fit for the organization. LO: 13.7: Identify the causes, consequences, and ethics of political behavior. Difficulty: Moderate Quest. Category: Concept 105) One of the best ways to influence people is indirectly. Answer: TRUE Explanation: One of the best ways to influence people is indirectly. LO: 13.7: Identify the causes, consequences, and ethics of political behavior. Difficulty: Moderate Quest. Category: Concept
608 richard@qwconsultancy.com
106) If you work hard at your job, advancement should come easy. Answer: FALSE Explanation: You may think that if you work hard, advancement should come easy. However, there is a different way to think about this, as Professor Bud Bilanich of the University of Denver notes: "Working hard … isn't going to be enough because your competition is also working hard and making contributions." "To me, I think that hard work … is kind of like the price of admission." LO: 13.7: Identify the causes, consequences, and ethics of political behavior. Difficulty: Moderate Quest. Category: Concept 107) Discuss the various individual factors that make a person more likely to engage in political behavior. Answer: At the individual level, researchers have identified certain personality traits, needs, and other factors likely to be related to political behavior. In terms of traits, employees who are high self-monitors, possess an internal locus of control, and have a high need for power are more likely to engage in political behavior. The high self-monitor is more sensitive to social cues, exhibits higher levels of social conformity, and is more likely to be skilled in political behavior than the low self-monitor. Because they believe they can control their environment, individuals with an internal locus of control are more prone to take a proactive stance and attempt to manipulate situations in their favor. Not surprisingly, the Machiavellian personality, characterized by the will to manipulate and the desire for power, is comfortable using politics as a means to further his or her self-interest. LO: 13.7: Identify the causes, consequences, and ethics of political behavior. Difficulty: Moderate Quest. Category: Concept 108) What are defensive behaviors? Provide three examples of defensive behaviors and explain why an employee would engage in the behavior. Answer: Defensive behaviors occur when people perceive politics as a threat rather than an opportunity. They respond with reactive and protective behaviors to avoid action, blame, or change. Defensive behaviors are often associated with negative feelings toward the job and work environment. In the short run, employees may find that defensiveness protects their self-interest. But in the long run, it is the only way they know how to behave. At that point, they lose the trust and support of their peers, bosses, employees, and clients. Buck passing is transferring responsibility for the execution of a task or decision to someone else. Employees engage in buck passing with the goal of avoiding action. Playing safe is evading situations that may reflect unfavorably. It includes taking on only projects with a high probability of success, having risky decisions approved by superiors, qualifying expressions of judgment, and taking neutral positions in conflicts. Employees engage in playing it safe with the goal of avoiding blame. Self-protection is acting in ways to protect one's self-interest during change by guarding information or other resources. Employees engage in self-protection in order to avoid change. LO: 13.7: Identify the causes, consequences, and ethics of political behavior. Difficulty: Moderate Quest. Category: Concept 609 richard@qwconsultancy.com
109) Explain impression management and describe who is most likely to engage in it. Answer: The process by which individuals attempt to control the impression others form of them is called impression management. High self-monitors are more likely to engage in impression management. Low self-monitors tend to present images of themselves that are consistent with their personalities, regardless of the beneficial or detrimental effects for them. In contrast, high self-monitors are good at reading situations and molding their appearances and behavior to fit each situation. LO: 13.7: Identify the causes, consequences, and ethics of political behavior. Difficulty: Moderate Quest. Category: Concept 110) What is impression management and which techniques are most effective in an interview? Answer: The process by which individuals attempt to control the impression others form of them is called impression management (IM). Some IM techniques work better in interviews than others. Researchers have compared applicants whose IM techniques focused on promoting their accomplishments (self-promotion) to those who focused on complimenting the interviewer and finding areas of agreement (ingratiation). In general, applicants appear to use self-promotion more than ingratiation. What's more, self-promotion tactics may be more important to interview success. Applicants who work to create an appearance of competence by enhancing their accomplishments, taking credit for successes, and explaining away failures do better in interviews. These effects reach beyond the interview: applicants who use more self-promotion tactics also seem to get more follow-up job-site visits, even after adjusting for grade-point average, gender, and job type. Ingratiation also works well in interviews: applicants who compliment the interviewer, agree with his or her opinions, and emphasize areas of fit do better than those who don't. LO: 13.7: Identify the causes, consequences, and ethics of political behavior. AACSB: Analytical thinking Difficulty: Moderate Employability Skills: Knowledge Application and Analysis Quest. Category: Synthesis Organizational Behavior, 19e (Robbins/Judge) Chapter 14 Conflict and Negotiation 1) The process that begins when one party perceives another party has or is about to negatively affect something the first party cares about is known as ________. A) problem solving B) assessment C) conflict D) negotiation E) collective bargaining Answer: C Explanation: Conflict is defined as a process that begins when one party perceives another party has or is about to negatively affect something the first party cares about. LO: 14.1: Describe the three types of conflict and the three loci of conflict. Difficulty: Easy 610 richard@qwconsultancy.com
Quest. Category: Concept 2) In the past, researchers ________. A) considered all conflict to be harmful B) considered all conflict to be necessary to generate discussion C) considered all conflict to be necessary to improve the creativity of a group D) argued about whether conflict was uniformly good or bad E) argued about whether conflict was rational and cannot be avoided Answer: D Explanation: In the past, researchers argued about whether conflict was uniformly good or bad. LO: 14.1: Describe the three types of conflict and the three loci of conflict. Difficulty: Moderate Quest. Category: Concept 3) Conflict that hinders group performance is destructive or ________ conflict. A) process B) traditional C) task D) dysfunctional E) dyadic Answer: D Explanation: Conflict that hinders group performance is destructive or dysfunctional conflict. A highly personal struggle for control in a team that distracts from the task at hand is dysfunctional. LO: 14.1: Describe the three types of conflict and the three loci of conflict. Difficulty: Easy Quest. Category: Concept
611 richard@qwconsultancy.com
4) Gwyn is struggling with personal issues of control within her team. Her outbursts and underhanded nature of functioning within the team is destructive. This could be considered ________. A) relationship conflict B) dysfunctional conflict C) dysfunctional task conflict D) dyadic relationship issues E) intragroup conflict Answer: B Explanation: Conflict that hinders group performance is destructive or dysfunctional conflict. A highly personal struggle for control in a team that distracts from the task at hand is dysfunctional. LO: 14.1: Describe the three types of conflict and the three loci of conflict. AACSB: Interpersonal relations and teamwork Difficulty: Moderate Employability Skills: Knowledge Application and Analysis Quest. Category: Concept 5) Studies demonstrate that ________ conflicts are almost always ________. A) relationship; formal B) relationship; informal C) relationship; dysfunctional D) relationship; functional E) task; reactive Answer: C Explanation: Studies demonstrate that relationship conflicts, at least in work settings, are almost always dysfunctional. LO: 14.1: Describe the three types of conflict and the three loci of conflict. AACSB: Interpersonal relations and teamwork Difficulty: Moderate Quest. Category: Concept 6) Which type of conflict relates to the content and goals of work? A) Job B) Task C) Relationship D) Process E) Communication Answer: B Explanation: Task conflict relates to the content and goals of the work. Low to moderate levels of task conflict stimulate discussion of ideas. Task conflict is related to positive outcomes only when all members share the same goals and have high levels of trust. LO: 14.1: Describe the three types of conflict and the three loci of conflict. Difficulty: Easy Quest. Category: Concept
612 richard@qwconsultancy.com
7) Which type of conflict relates to how the work gets done? A) Communication B) Task C) Job D) Relationship E) Process Answer: E Explanation: Process conflict relates to how the work gets done. LO: 14.1: Describe the three types of conflict and the three loci of conflict. Difficulty: Easy Quest. Category: Concept 8) Mike and Keith are on the same team; they work well with other people in the same team but don't get along with each other. This type of conflict can best be described as ________. A) intragroup B) bilateral C) intergroup D) dyadic E) dual Answer: D Explanation: Dyadic conflict takes place between two people. Intragroup conflict occurs within a group or team. Intergroup conflict is conflict between groups or teams. LO: 14.1: Describe the three types of conflict and the three loci of conflict. AACSB: Interpersonal relations and teamwork Difficulty: Moderate Employability Skills: Knowledge Application and Analysis Quest. Category: Application 9) ________ conflict occurs between two people. A) Intragroup B) Bilateral C) Intergroup D) Dyadic E) Dual Answer: D Explanation: Dyadic conflict takes place between two people. Intragroup conflict occurs within a group or team. Intergroup conflict is conflict between groups or teams. LO: 14.1: Describe the three types of conflict and the three loci of conflict. AACSB: Interpersonal relations and teamwork Difficulty: Easy Quest. Category: Concept It seems that your work group is in conflict much of the time. One colleague has suggested that you, as the supervisor, are responsible for eliminating the conflict so that your work group can function harmoniously. Another colleague has suggested that conflict is good for stimulating creativity and productivity within the work environment. You are unsure about whether you 613 richard@qwconsultancy.com
should try to eliminate the conflict within your group or learn to deal with it positively. 10) Contemporary perspectives tend to differentiate types of conflict based on their ________. A) effects as functional or dysfunctional B) being good or bad C) emotional impact D) task orientation E) relationship orientation Answer: A Explanation: Contemporary perspectives differentiate types of conflict based on their effects as functional or dysfunctional. LO: 14.1: Describe the three types of conflict and the three loci of conflict. AACSB: Analytical thinking Difficulty: Moderate Quest. Category: Concept 11) Which of the following is false about views of conflict? A) There is no consensus over the role of conflict in groups and organizations. B) The notion that conflict is good or bad is somewhat outdated. C) Contemporary perspectives differentiate types of conflict based on their good or bad effects. D) The idea that conflict is good or bad can be viewed as simplistic. E) Contemporary perspectives differentiate types of conflict based on their effects. Answer: C Explanation: Contemporary perspectives differentiate types of conflict based on their effects, not on the notion of conflict being good or bad. LO: 14.1: Describe the three types of conflict and the three loci of conflict. AACSB: Analytical thinking Difficulty: Moderate Quest. Category: Concept 12) When your work group disagrees, the disagreements usually concern how the group's work should be accomplished. The type of conflict experienced by your group is known as a ________ conflict. A) task B) relationship C) process D) traditional E) reactive Answer: C Explanation: Process conflict relates to how the work gets done. LO: 14.1: Describe the three types of conflict and the three loci of conflict. AACSB: Analytical thinking Difficulty: Moderate Employability Skills: Knowledge Application and Analysis Quest. Category: Application
614 richard@qwconsultancy.com
615 richard@qwconsultancy.com
13) Scholars generally agree that task and process conflicts are functional. Answer: FALSE Explanation: While scholars agree that relationship conflict is dysfunctional, there is considerably less agreement about whether task and process conflicts are functional. Early research suggested that task conflict within groups correlated to higher group performance, but a review of 116 studies found that generalized task conflict was essentially unrelated to group performance. However, there were factors of the conflict that could create a relationship between conflict and performance. LO: 14.1: Describe the three types of conflict and the three loci of conflict. Difficulty: Moderate Quest. Category: Concept 14) Relationship conflicts are almost always functional. Answer: FALSE Explanation: Studies demonstrate that relationship conflicts are almost always dysfunctional. It appears that the friction and interpersonal hostilities inherent in relationship conflicts increase personality clashes and decrease mutual understanding, which hinders the completion of organizational tasks. LO: 14.1: Describe the three types of conflict and the three loci of conflict. AACSB: Interpersonal relations and teamwork Difficulty: Moderate Quest. Category: Concept 15) Conflict is uniformly good or bad. Answer: FALSE Explanation: In the past, researchers tended to argue about whether conflict was uniformly good or bad. Such simplistic views eventually gave way to approaches that recognized that not all conflicts are the same and that different types of conflict have different effects. LO: 14.1: Describe the three types of conflict and the three loci of conflict. AACSB: Analytical thinking Difficulty: Moderate Quest. Category: Concept 16) Conflict is more likely to have a positive effect on performance when a few members perceive strong task disagreement. Answer: TRUE Explanation: Research suggests that conflict is more likely to have a positive effect on performance when a few members perceive strong task disagreement and most others on the team perceive weak task disagreement. This is because those of the minority opinion are much more likely to present their disagreements in a careful, cooperative, open manner. LO: 14.1: Describe the three types of conflict and the three loci of conflict. AACSB: Interpersonal relations and teamwork Difficulty: Moderate Quest. Category: Concept
616 richard@qwconsultancy.com
17) Why are relationship conflicts in work settings almost always dysfunctional? Answer: It appears that the friction and interpersonal hostilities inherent in relationship conflicts increase personality clashes and decrease mutual understanding, which hinders the completion of organizational tasks. Of the three types, relationship conflicts also appear to be the most psychologically exhausting to individuals. Because they tend to revolve around personalities, you can see how relationship conflicts can become destructive. LO: 14.1: Describe the three types of conflict and the three loci of conflict. AACSB: Interpersonal relations and teamwork Difficulty: Moderate Quest. Category: Concept 18) Describe a functional conflict and dysfunctional conflict that could exist because of structure. Answer: Some conflicts support the goals of the group and improve its performance; these are functional, constructive forms of conflict. Additionally, there are conflicts that hinder group performance; these are dysfunctional or destructive forms of conflict. Structural conflict includes variables such as size, degree of specialization in the tasks assigned to group members, jurisdictional clarity, member-goal compatibility, leadership styles, reward systems, and the degree of dependence between groups. A functional-structural conflict that could exist in a longterm situation would be that of a salesperson and a credit manager who are in constant conflict because they have differing goals; the salesperson wishing to make the highest number of sales, and the credit manager needing to ensure that all the sales will be paid. Although there is conflict, it is functional because it is supportive of the overall group performance. A dysfunctional-structural conflict could exist, for example, because tasks are not adequately defined in a work group. The group would always be at odds and would not be advancing the goals set and the tasks would be hindered. LO: 14.1: Describe the three types of conflict and the three loci of conflict. AACSB: Analytical thinking Difficulty: Hard Quest. Category: Synthesis 19) Identify the three basic types of conflict from the perspective of where the conflict occurs and discuss why it is important to understand the loci of conflict in order to address conflict in the workplace. Answer: The three basic types of conflict are dyadic (conflict between two people), intragroup (conflict within a group or team), and intergroup (conflict between groups or teams). It is important to recognize the loci of conflict in an organization in order to better understand why the conflict is taking place and its impact on the organization. LO: 14.1: Describe the three types of conflict and the three loci of conflict. AACSB: Analytical thinking Difficulty: Moderate Quest. Category: Analytical
617 richard@qwconsultancy.com
20) Using examples, compare and contrast intergroup conflict and intragroup conflict. Answer: Intragroup conflict is conflict within a group or work team, whereas intergroup conflict is conflict between groups or teams. Intragroup conflict is the focus of much of the research on conflict in the workplace and the groups and teams that exist to perform a particular task. In this type of group, it is important that a supportive climate exists if conflict is to have a positive influence. Within an NFL team, for example, team members may be motivated to improve when team members are supportive. The intergroup conflict that exists between members of one team and those of another is a very different type of conflict, however, and one that may have little bearing on conflict within a group. LO: 14.1: Describe the three types of conflict and the three loci of conflict. AACSB: Analytical thinking Difficulty: Moderate Quest. Category: Analytical 21) Conservationists have had a perpetual conflict with the government of the United States over the fast and rampant depletion of Earth's natural resources. They argue that the United States must reduce its consumption level significantly to rectify this problem. Which of the following, if true, would most strengthen the conservationists' argument? A) The United States has been conserving forests for several years. B) Most countries have not taken any measures to reduce their consumption levels of natural resources. C) There are several countries that have more requirements for resources than the United States. D) The United States accounts for one-third of the total world resource consumption. E) New resource deposits are constantly being discovered. Answer: D Explanation: The fact that the United States has been conserving forests for several years has nothing to do with the conservationists' argument. Neither does the fact that there are several countries that have more requirements for resources than the United States nor the fact that new deposits are being discovered. The fact that most countries have not taken any measures to reduce the consumption level does not provide support for the conclusion. The fact that the United States consumes one-third of all resources used in the world supports the conclusion as the United States cannot increase consumption to more than one-third of the world's resources or they risk depleting Earth's resources even faster. LO: 14.2: Outline the conflict process. AACSB: Analytical thinking Difficulty: Hard Employability Skills: Critical Thinking Quest. Category: Critical Thinking
618 richard@qwconsultancy.com
22) Matt works as a sales representative at Tandem Industries, which manufactures and sells bicycles. He's the best salesperson on the floor, but he also receives the highest number of customer complaints for late deliveries. Which of the following, if true, best explains this situation? A) The company uses enterprise solutions to manage inventory. B) The company's manufacturing facility is functioning at optimum capacity. C) The company's sales and logistics functions are tightly integrated. D) The company keeps inventory levels low in order to reduce costs. E) The company has an efficient credit verification process. Answer: D Explanation: In this situation, if the company keeps inventory levels low in order to reduce costs, then it directly conflicts with Matt's job of selling bicycles and ultimately results in late deliveries. This conflict is structural in nature. If the company uses enterprise solutions to manage inventory and if its sales and logistics functions are tightly integrated, then the problem of late deliveries should not arise. Similarly, if the firm's manufacturing facility is functioning at optimum capacity, then there should be an adequate of supply of bicycles. If the company's credit verification process is efficient, then it would not result in late deliveries. LO: 14.2: Outline the conflict process. AACSB: Reflective thinking Difficulty: Hard Employability Skills: Critical Thinking Quest. Category: Critical Thinking 23) Conditions that can create conflicts can be classified into three categories. Which of the following is one of these three categories that includes variables such as jurisdictional clarity, member-goal compatibility, and leadership styles? A) Communication B) Structure C) Personal variables D) Group interactions E) Process variables Answer: B Explanation: Conflicts related to structure result from variables such as size, degree of specialization in the tasks assigned to group members, jurisdictional clarity, member-goal compatibility, leadership styles, reward systems, and the degree of dependence between groups. LO: 14.2: Outline the conflict process. Difficulty: Easy Quest. Category: Concept
619 richard@qwconsultancy.com
24) Which of the following statements is true regarding conflict? A) The smaller the group, the greater the likelihood of conflict. B) The less specialized the activities of the group, the greater the likelihood of conflict. C) People low in the personality traits of disagreeableness, neuroticism, or self-monitoring are more likely to engage in a conflict. D) Emotions can cause conflict even when they are not directed at others. E) As the ambiguity about where responsibility for actions lies decreases, the potential for conflict increases. Answer: D Explanation: Emotions can cause conflict even when they are not directed at others. People high in the personality traits of disagreeableness, neuroticism, or self-monitoring are more likely to engage in a conflict. LO: 14.2: Outline the conflict process. Difficulty: Moderate Quest. Category: Concept 25) Stage II of the conflict process deals with the conflict being ________. A) perceived and felt B) apparent and experienced C) expressed and resolved D) analyzed E) internalized Answer: A Explanation: The potential for opposition or incompatibility becomes actualized in the second stage. Because a conflict is a perceived conflict does not mean it is personalized. It is at the felt conflict level, when individuals become emotionally involved, that parties experience anxiety, tension, frustration, or hostility. LO: 14.2: Outline the conflict process. Difficulty: Easy Quest. Category: Concept 26) Conflict issues are defined in the ________ stage of the conflict process. A) potential opposition B) cognition and personalization C) intention D) behavior E) reaction and transference Answer: B Explanation: The cognition and personalization stage of the conflict process is important because it's where conflict issues tend to be defined. This is the point when the parties decide what the conflict is about. LO: 14.2: Outline the conflict process. Difficulty: Moderate Quest. Category: Concept
620 richard@qwconsultancy.com
27) In the ________ stage of the conflict process intentions intervenes between people's perceptions and emotions and their overt behavior. A) potential opposition B) intention C) cognition D) attribution E) behavioral manifestation Answer: B Explanation: Intentions intervene between people's perceptions and emotions and their overt behavior. They are decisions to act in a given way. LO: 14.2: Outline the conflict process. Difficulty: Moderate Quest. Category: Concept 28) The conflict-handling intention cooperatives indicates the degree to which ________. A) one party attempts to satisfy his or her own concerns B) one party attempts to suppress the conflict C) the parties attempt to find a win-win solution D) one party attempts to satisfy the other party's concerns E) the parties are willing to involve a third party in conflict resolution Answer: D Explanation: Cooperativeness indicates the degree to which one party attempts to satisfy the other party's concerns. LO: 14.2: Outline the conflict process. Difficulty: Moderate Quest. Category: Concept 29) Assertiveness indicates the degree to which ________. A) one party attempts to satisfy his or her own concerns B) one party attempts to suppress the conflict C) the parties attempt to find a win-win solution D) one party attempts to satisfy the other party's concerns E) the parties are willing to involve a third party in conflict resolution Answer: A Explanation: Assertiveness indicates the degree to which one party attempts to satisfy his or her own concerns. LO: 14.2: Outline the conflict process. Difficulty: Moderate Quest. Category: Concept
621 richard@qwconsultancy.com
30) A person is using the conflict handling technique of ________ when the one person seeks to satisfy his or her own interests regardless of the impact on the other parties to the conflict. A) competing B) avoiding C) accommodating D) compromising E) collaborating Answer: A Explanation: When one person seeks to satisfy his or her own interests regardless of the impact on the other parties to the conflict, that person is competing. LO: 14.2: Outline the conflict process. Difficulty: Easy Quest. Category: Concept 31) A conflict-handling technique in which the parties in a conflict each desire to satisfy fully the concerns of all parties is ________. A) collaborating B) competing C) accommodating D) avoiding E) compromising Answer: A Explanation: When parties in conflict each desire to fully satisfy the concerns of all parties, there is cooperation and a search for a mutually beneficial outcome. In collaborating, the parties intend to solve a problem by clarifying differences rather than by accommodating various points of view. LO: 14.2: Outline the conflict process. Difficulty: Easy Quest. Category: Concept 32) The conflict-handling intention of collaborating is ________. A) assertive and uncooperative B) assertive and cooperative C) unassertive and uncooperative D) unassertive and cooperative E) affective and reflective Answer: B Explanation: Collaborating intentions are assertive and cooperative. LO: 14.2: Outline the conflict process. Difficulty: Moderate Quest. Category: Concept
622 richard@qwconsultancy.com
33) The ________ conflict-handling intention is unassertive and uncooperative. A) competing B) collaborating C) avoiding D) compromising E) accommodating Answer: C Explanation: Avoiding intentions are unassertive and uncooperative. LO: 14.2: Outline the conflict process. Difficulty: Moderate Quest. Category: Concept 34) The conflict-handling intention of accommodating is ________. A) assertive and uncooperative B) assertive and cooperative C) unassertive and uncooperative D) unassertive and cooperative E) reflective and assertive Answer: D Explanation: Accommodating intentions are unassertive and cooperative. LO: 14.2: Outline the conflict process. Difficulty: Moderate Quest. Category: Concept 35) David feels his blood pressure rise everyday around lunch time when his cubicle neighbor Carlos has lunch. Carlos likes to watch Euro soccer while he eats and always turns the volume up so that he can hear it over his chewing. Carlos is a friend of David's and David feels that they work well together, so he's decided not to talk to him about his lunch habits. David's conflict handling intention is called ________. A) competing B) avoiding C) accommodating D) compromising E) collaborating Answer: B Explanation: A person may recognize a conflict exists and want to withdraw from or suppress it. Examples of avoiding include trying to ignore a conflict and avoiding others with whom you disagree. LO: 14.2: Outline the conflict process. AACSB: Analytical thinking Difficulty: Moderate Employability Skills: Knowledge Application and Analysis Quest. Category: Application
623 richard@qwconsultancy.com
36) A party who seeks to appease an opponent may be willing to place the opponent's interests above his or her own, sacrificing to maintain the relationship. We refer to this conflict-handling intention as ________. A) collaborating B) avoiding C) compromising D) competing E) accommodating Answer: E Explanation: A party who seeks to appease an opponent may be willing to place the opponent's interests above his or her own, sacrificing to maintain the relationship. We refer to this conflicthandling intention as accommodating. LO: 14.2: Outline the conflict process. Difficulty: Moderate Quest. Category: Concept 37) Macy does not like a few of the standard operating procedures adapted for the new project. However, she discussed the items with the team and told them that she realized she was in the minority and that she would adapt the new procedures to maintain smooth operations within the team. This type of conflict-handling intention is called ________. A) negotiating B) accommodating C) collaborating D) compromising E) competing Answer: B Explanation: A party who seeks to appease an opponent may be willing to place the opponent's interests above his or her own, sacrificing to maintain the relationship. We refer to this conflicthandling intention as accommodating. Supporting someone else's opinion despite your reservations about it, for example, is accommodating. LO: 14.2: Outline the conflict process. AACSB: Analytical thinking Difficulty: Hard Quest. Category: Application 38) The ________ conflict-handling intention is assertive and uncooperative. A) compromising B) avoiding C) competing D) collaborating E) accommodating Answer: C Explanation: The conflict-handling intention of competing is assertive and uncooperative. LO: 14.2: Outline the conflict process. Difficulty: Easy Quest. Category: Concept 624 richard@qwconsultancy.com
39) The ________conflict-handling intention involves accepting a solution that provides incomplete satisfaction of both parties' concerns. A) collaborating B) compromising C) avoiding D) competing E) accommodating Answer: B Explanation: In compromising, there is no clear winner or loser. Rather, there is a willingness to ration the object of the conflict and accept a solution that provides incomplete satisfaction of both parties' concerns. LO: 14.2: Outline the conflict process. Difficulty: Easy Quest. Category: Concept 40) Conflict becomes visible in the ________ stage of the conflict process. A) outcomes B) intentions C) incompatibility D) behavior E) personalization Answer: D Explanation: When most people think of conflict situations, they tend to focus on Stage IV because this is where conflicts become visible. The behavior stage includes the statements, actions, and reactions made by the conflicting parties, usually as overt attempts to implement their own intentions. LO: 14.2: Outline the conflict process. Difficulty: Moderate Quest. Category: Concept
625 richard@qwconsultancy.com
41) A conflict-intensity continuum has "annihilatory conflict" at the upper end and "no conflict" at the lower end. In the context of this conflict-intensity continuum, which of the following statements is true of conflict intensities? A) Aggressive physical attacks fall in the lower range of the conflict-intensity continuum. B) Conflicts that reach the lower ranges of the conflict-intensity continuum are almost always dysfunctional. C) Strikes, riots, and wars fall in the lower range of the conflict-intensity continuum. D) In cases of annihilatory conflict, there is overt questioning or challenging of others. E) Functional conflicts are typically confined to the lower range of the conflict-intensity continuum. Answer: E Explanation: All conflicts exist somewhere along the conflict-intensity continuum. At the lower part are conflicts characterized by subtle, indirect, and highly controlled forms of tension. Conflict intensities escalate as they move upward along the continuum until they become highly destructive. Strikes, riots, and wars clearly fall in this upper range. Conflicts that reach the upper ranges of the continuum are almost always dysfunctional. Functional conflicts are typically confined to the lower range of the continuum. LO: 14.2: Outline the conflict process. Difficulty: Moderate Quest. Category: Concept
626 richard@qwconsultancy.com
42) The division manager at Xperience Inc. wants to replace the SAP ERP currently used in his division with Epicor ERP as it costs 30 percent less to train new staff in Epicor ERP. But this leads to a conflict with the general manager, who insists that they can simply hire only people who already know how to use SAP ERP. Which of the following, if true, weakens the general manager's objection to the replacement of SAP ERP with Epicor ERP? A) Currently all employees in the company are required to attend workshops on how to use SAP ERP. B) Once employees learn how to use an ERP system, they tend to change employers more readily than before. C) Experienced users of SAP ERP command much higher salaries than do prospective employees who have no experience in the use of any ERP. D) The average productivity of employees in the company is below the average productivity of the employees of its competitors. E) The high costs of updates and upgrades make Epicor ERP more expensive to maintain than SAP ERP. Answer: C Explanation: The general manager's objection is based on avoiding training costs altogether. But if hiring experienced users of SAP ERP is significantly more costly than hiring otherwise qualified people who would have to be trained to use Epicor ERP, the objection is weakened. Hence, the fact that experienced users of SAP ERP command much higher salaries weakens it. The fact that the high costs of updates and upgrades make Epicor ERP more expensive to maintain than SAP ERP argues independently against replacing SAP ERP with Epicor ERP and thus is also incorrect. The rest of the options do not provide any information relevant to an evaluation of SAP ERP as compared with Epicor ERP. LO: 14.2: Outline the conflict process. AACSB: Reflective thinking Difficulty: Hard Employability Skills: Critical Thinking Quest. Category: Critical Thinking
627 richard@qwconsultancy.com
43) According to a newly added office smoking regulation, only employees who have an enclosed office may smoke at their desks. This leads to a major conflict between various employees as virtually all employees with enclosed offices are higher-level managers and all other employees lack enclosed offices. Therefore, the lower-level employees who smoke argue that they should be offered enclosed offices. Which of the following, if true, strengthens the employees' argument? A) The company is a zealous supporter of the "Kick the Butt" campaign—a corporate antismoking campaign. B) Higher-level managers, who have enclosed offices, are willing to share their offices withlower-level employees. C) The smoking regulations allow all employees who smoke an equal opportunity to do so, regardless of an employee's job level. D) Employees at the higher level, who do not smoke, do not have enclosed offices. E) The company has a limited budget for infrastructure modifications. Answer: C Explanation: If the smoking regulations allow all employees who smoke an equal opportunity to do so, regardless of an employee's job level, then the employees are justified in demanding enclosed offices. Hence, this strengthens their argument. If higher-level managers are willing to share, this only weakens the employees' argument as they don't need separate offices. The other options are all against rather than for the employees' argument. The fact that the company is a zealous supporter of the "Kick the butt before you kick the bucket campaign" is out of scope. LO: 14.2: Outline the conflict process. AACSB: Reflective thinking Difficulty: Hard Employability Skills: Critical Thinking Quest. Category: Critical Thinking 44) ________ is an example of engagement in conflict management. A) Expansion of resources B) Compromise C) Involving others D) Exercising authoritative command E) Problem solving Answer: C Explanation: Involving others, strategically observing, and escalating conflict are examples of engagement in conflict management. LO: 14.2: Outline the conflict process. Difficulty: Moderate Quest. Category: Concept
628 richard@qwconsultancy.com
45) Which of the following is not a conflict-handling intention in the conflict process? A) Avoiding B) Collaborating C) Compromising D) Accommodating E) Communication Answer: E Explanation: Conflict-handling techniques in the conflict process include competing, collaborating, compromising, avoiding, and accommodating. LO: 14.2: Outline the conflict process. Difficulty: Easy Quest. Category: Concept 46) When is conflict dysfunctional? A) When it provides a medium to release tension B) When it reduces group cohesiveness C) When it fosters an environment of self-evaluation D) When it provides a means for expressing frustration E) When it leads to change Answer: B Explanation: Among the more undesirable consequences of dysfunctional conflict are hampered communication, reductions in group cohesiveness, and subordination of group goals to the primacy of infighting among members. LO: 14.2: Outline the conflict process. Difficulty: Moderate Quest. Category: Concept 47) Which of the following statements is true regarding negotiation that takes place in a collectivist culture? A) Collectivists are less likely to seek to preserve relationships and promote the good of the group as a whole than individualists. B) Collectivists are more likely to confront differences of opinion directly and openly than individualists. C) Collectivist cultures see people as autonomous, not as people deeply embedded in social situations. D) Compared to individualists, collectivists are less likely to see offers from their counterparts as unfair and to reject them. E) The method of conflict management preferred by collectivist managers is competing. Answer: D Explanation: Compared to collectivist culture negotiators, their more individualist counterparts are more likely to see offers from their counterparts as unfair and to reject them. LO: 14.2: Outline the conflict process. AACSB: Diverse and multicultural work environments Difficulty: Hard Quest. Category: Concept
629 richard@qwconsultancy.com
630 richard@qwconsultancy.com
48) Which of the following conflict management techniques is most likely to be used by a collectivist Chinese manager? A) Direct confrontation B) Authoritative command C) Avoiding D) Pressurizing E) Competing tactics Answer: C Explanation: To preserve peaceful relationships, collectivists will avoid direct expression of conflicts, preferring to use more indirect methods for resolving differences of opinion. Whereas individualist U.S. managers are more likely to use competing tactics in the face of conflicts, compromising and avoiding are the most preferred methods of conflict management in China. LO: 14.2: Outline the conflict process. AACSB: Diverse and multicultural work environments Difficulty: Moderate Quest. Category: Concept 49) All of the following are benefits of conflict in organizations except ________. A) increased self-evaluation and change B) increased groupthink C) increased creativity and innovation D) increased curiosity among group members E) increased opportunity to air problems Answer: B Explanation: Conflict is constructive when it improves the quality of decisions, stimulates creativity and innovation, encourages curiosity and interest among group members, provides the medium for problems to be aired and tension released, and fosters self-evaluation and change. LO: 14.2: Outline the conflict process. AACSB: Analytical thinking Difficulty: Moderate Quest. Category: Application 50) If a conflict is expected to be perpetual and ongoing, a strategic conflict management strategy might not target resolving the conflict completely, but rather how people can adaptively address the conflict over time. Answer: TRUE Explanation: If a conflict is expected to be perpetual and ongoing, a strategic conflict management strategy might not target resolving the conflict completely, but rather how people can adaptively address the conflict over time. LO: 14.2: Outline the conflict process. Difficulty: Easy Quest. Category: Concept
631 richard@qwconsultancy.com
51) A review that examined the effects of the four sets of behaviors across multiple studies found that openness and collaborating were both associated with superior group performance, whereas avoiding and competing strategies were associated with significantly worse group performance. Answer: TRUE Explanation: A review that examined the effects of the four sets of behaviors across multiple studies found that openness and collaborating were both associated with superior group performance, whereas avoiding and competing strategies were associated with significantly worse group performance. LO: 14.2: Outline the conflict process. Difficulty: Moderate Quest. Category: Analytical 52) During the behavior stage of the conflict process, the parties decide what the conflict is about. Answer: FALSE Explanation: During the cognition and personalization stage, the parties decide what the conflict is about. The definition of a conflict is important because it typically delineates the set of possible settlements. LO: 14.2: Outline the conflict process. Difficulty: Moderate Quest. Category: Concept 53) Cross-cultural negotiations have no issues of trust. Answer: FALSE Explanation: Cross-cultural negotiations can also create issues of trust. One study of Indian and U.S. negotiators found that respondents reported having less trust in their cross-culture negotiation counterparts. The lower level of trust was associated with lower discovery of common interests between parties, which occurred because cross-culture negotiators were less willing to disclose and solicit information. LO: 14.2: Outline the conflict process. AACSB: Diverse and multicultural work environments Difficulty: Moderate Quest. Category: Concept 54) Individualists are more interested in demonstrations of concern and working through third parties to resolve disputes, whereas collectivists are more likely to confront differences of opinion directly and openly. Answer: FALSE Explanation: Collectivists are more interested in demonstrations of concern and working through third parties to resolve disputes, whereas individualists are more likely to confront differences of opinion directly and openly. LO: 14.2: Outline the conflict process. AACSB: Diverse and multicultural work environments Difficulty: Moderate Quest. Category: Concept
632 richard@qwconsultancy.com
633 richard@qwconsultancy.com
55) Groups that are extremely polarized do not manage their underlying disagreements effectively and tend to accept suboptimal solutions. Answer: TRUE Explanation: Groups that are extremely polarized do not manage their underlying disagreements effectively and tend to accept suboptimal solutions, or they avoid making decisions altogether rather than work out the conflict. LO: 14.2: Outline the conflict process. Difficulty: Moderate Quest. Category: Concept 56) Describe the various causes of conflict. Answer: The three general categories of conditions that may be the sources of conflict are communication, structure, and personal variables. a) Communication represents those opposing forces that arise from semantic difficulties, misunderstandings, and "noise" in the communication channels. Differing word connotations, jargon, insufficient exchange of information, and noise in the communication channel are all barriers to communication and potential antecedent conditions to conflict. The potential for conflict increases when either too little or too much communication takes place. b) Structure includes variables such as size, degree of specialization in the tasks assigned to group members, jurisdictional clarity, member-goal compatibility, leadership styles, reward systems, and the degree of dependence between groups. c) Personal variables include individual value systems that each person has and the personality characteristics that account for individual idiosyncrasies and differences. LO: 14.2: Outline the conflict process. Difficulty: Moderate Quest. Category: Concept 57) What are the five conflict-handling intentions that are based on the dimensions of cooperativeness and assertiveness? Answer: The five conflict-handling intentions that are based on the dimensions of cooperativeness and assertiveness are: a) Competing: When one person seeks to satisfy his or her own interests, regardless of the impact on the other parties to the conflict, that person is competing. b) Collaborating: In collaborating, the intention of the parties is to solve the problem by clarifying differences rather than by accommodating various points of view. c) Avoiding: In avoiding, a person may recognize that a conflict exists and want to withdraw from it or suppress it. d) Accommodating: A party who seeks to appease an opponent may be willing to place the opponent's interests above his or her own, sacrificing to maintain the relationship. We refer to this conflict-handling intention as accommodating. e) Compromising: In compromising, there is no clear winner or loser. Rather, there is a willingness to ration the object of the conflict and accept a solution that provides incomplete satisfaction of both parties' concerns. LO: 14.2: Outline the conflict process. Difficulty: Moderate Quest. Category: Concept 634 richard@qwconsultancy.com
58) Identify and explain the five stages of the conflict process. Answer: The conflict process can be seen as comprising five stages: potential opposition or incompatibility, cognition and personalization, intentions, behavior, and outcomes. a) The first step in the conflict process is the presence of conditions that create opportunities for conflict to arise. They need not lead directly to conflict, but one of these conditions is necessary if conflict is to surface. These conditions are communication, structure, and personal variables. If the conditions related to these variables negatively affect something that one party cares about, then the potential for opposition or incompatibility becomes actualized in the second stage. The antecedent conditions can only lead to conflict when one or more of the parties are affected by, and aware of, the conflict. b) "Stage II" is cognition and personalization. This is where conflict issues tend to be defined. c) In "Stage III," intentions intervene between people's perceptions and emotions and their overt behavior. These intentions are decisions to act in a certain way. This is "Stage III" of the conflict process. d) When most people think of conflict situations, they tend to focus on "behavior" because this is where conflicts become visible. The behavior stage includes statements, actions, and reactions made by the conflicting parties. This is "Stage IV" of the conflict process. e) The action-reaction interplay between the conflicting parties results in consequences. These outcomes in "Stage V" may be functional in that the conflict results in an improvement in the group's performance, or dysfunctional in that it hinders group performance. LO: 14.2: Outline the conflict process. Difficulty: Hard Quest. Category: Concept 59) Which conflict resolutions strategies are used in collectivist cultures? Answer: Collectivist cultures see people as deeply embedded in social situations. As a result, collectivists are more likely to seek to preserve relationships and promote the good of the group as a whole than individualists. To preserve peaceful relationships, collectivists will avoid direct expression of conflicts, preferring to use more indirect methods for resolving differences of opinion. Compromising and avoiding are the most preferred methods of conflict management in China. Smoothing, or playing down differences while emphasizing common interests between the conflicting parties, is also a good technique to maintain relationships. Collectivists such as the Chinese may also be more interested in demonstrations of concern and working through third parties to resolve disputes. LO: 14.2: Outline the conflict process. AACSB: Diverse and multicultural work environments Difficulty: Moderate Quest. Category: Concept
635 richard@qwconsultancy.com
60) ________ refers to the process that occurs when two or more parties communicate and confer with one another to come to a mutual agreement on the exchange of goods or services. A) Integration B) Association C) Negotiation D) Unionization E) Differentiation Answer: C Explanation: We can define negotiation as a process that occurs when two or more parties communicate and confer with one another to come to a mutual agreement on the exchange of goods or services. LO: 14.3: Contrast distributive and integrative bargaining. Difficulty: Easy Quest. Category: Concept 61) ________ is a characteristic of integrative bargaining. A) Win-lose motivation B) Interests focus C) Opposed interests D) Low information sharing E) Short-term relationship Answer: B Explanation: Integrative bargaining is characterized by a goal of expanding the pie so that both parties are satisfied, a win-win motivation, interests focus, congruent interests, high information sharing, and long-term relationships. LO: 14.3: Contrast distributive and integrative bargaining. Difficulty: Moderate Quest. Category: Concept 62) Which of the following is not a necessary condition for integrative bargaining to succeed in organizations? A) Opposing parties have a zero-sum outlook. B) Opposing parties are open with information. C) Opposing parties are candid about concerns. D) Opposing parties are sensitive to the other's needs and trust. E) Opposing parties maintain flexibility. Answer: A Explanation: For integrative bargaining to succeed in an organization, opposing parties must be open with information, candid about concerns, sensitive to the other's needs and trust, and flexible. LO: 14.3: Contrast distributive and integrative bargaining. Difficulty: Moderate Quest. Category: Analytical
636 richard@qwconsultancy.com
63) Negotiation is a process that begins when one party perceives that another party has negatively affected, or is about to negatively affect, something that the first party cares about. Answer: FALSE Explanation: We can define negotiation as a process that occurs when two or more parties communicate and confer with one another to come to a mutual agreement on the exchange of goods or services. LO: 14.3: Contrast distributive and integrative bargaining. Difficulty: Easy Quest. Category: Concept 64) Explain how distributive and integrative bargaining can be used within the same negotiation episode. Answer: Early on in the episode (see Exhibit 14-7), integrative strategies can be used, while later in the episode, distributive strategies can be used. In a workplace situation, the employee could first clarify her needs and articulate her interests to the supervisor and the supervisor would do the same, all without trying to come to a compromise right away. If a compromise is reached too early, it is more likely that one or more of the parties would settle for less than they could have obtained if they had been forced to consider the other party's interests, trade off issues, and be creative. It is important not to walk away too soon or cave into the negotiation partner's ultimatum—persisting through the process can pay off in better negotiation outcomes. Once all of the needs and interests are established for both parties, the supervisor could then switch to a distributive strategy, setting goals aligned with the company's needs and interests while attempting to maximize the extent to which both parties' goals are met. LO: 14.3: Contrast distributive and integrative bargaining. AACSB: Analytical thinking Difficulty: Moderate Quest. Category: Synthesis 65) The two general approaches to negotiation are ________ bargaining and ________ bargaining. A) emotional; rational B) affective; reflective C) distributive; integrative D) formal; informal E) legal; restrictive Answer: C Explanation: There are two general approaches to negotiation: distributive bargaining and integrative bargaining. In distributive bargaining, negotiation seeks to divide up a fixed amount of resources into a win-lose situation. With integrative bargaining, negotiation seeks one or more settlements that can create a win-win solution. LO: 14.4: Apply the five steps of the negotiation process. Difficulty: Easy Quest. Category: Concept
637 richard@qwconsultancy.com
66) ________ bargaining is negotiation that seeks to divide a fixed amount of goods or services. A) Distributive B) Integrative C) Reflective D) Affective E) Conjunctive Answer: A Explanation: The essence of distributive bargaining is negotiating over who gets what share of a fixed pie. Fixed pie refers to a set amount of goods or services to be divided up. When the pie is fixed, or parties believe it is, they tend to engage in distributive bargaining. LO: 14.4: Apply the five steps of the negotiation process. Difficulty: Easy Quest. Category: Concept 67) Santiago is the labor union for his Local 453. Workers have been on strike for the past three months. They want to get a new three-year contract that includes better benefits. Santiago will be meeting with management later today. The negotiations between Santiago and management are likely to use ________ bargaining. A) integrative B) reflective C) distributive D) evaluative E) associative Answer: C Explanation: The most widely cited example of distributive bargaining is labor-management negotiations over wages. LO: 14.4: Apply the five steps of the negotiation process. AACSB: Analytical thinking Difficulty: Moderate Employability Skills: Knowledge Application and Analysis Quest. Category: Application 68) In the case of distributive bargaining, the ________ point indicates what a person would like to achieve out of the negotiation. A) resistance B) distribution C) target D) bargaining E) focus Answer: C Explanation: Each party in a negotiation has a target point that defines what he or she would like to achieve. Each also has a resistance point, which marks the lowest outcome that is acceptable, or the point below which the party would break off negotiations rather than accept a less favorable settlement. LO: 14.4: Apply the five steps of the negotiation process. Difficulty: Easy 638 richard@qwconsultancy.com
Quest. Category: Concept 69) In the case of distributive bargaining, the ________ point marks the lowest outcome that is acceptable—the point below which the party would break off negotiations rather than accept a less favorable settlement. A) resistance B) tolerance C) target D) focal E) distribution Answer: A Explanation: Each party in a negotiation has a target point that defines what he or she would like to achieve. Each also has a resistance point, which marks the lowest outcome that is acceptable, or the point below which the party would break off negotiations rather than accept a less favorable settlement. LO: 14.4: Apply the five steps of the negotiation process. Difficulty: Easy Quest. Category: Concept 70) Which of the following statements is true regarding distributive bargaining? A) It operates under zero-sum conditions. B) It focuses on long-term relationships. C) It involves high information sharing. D) It attempts to create a win-win solution. E) It is rarely used in labor-management negotiations. Answer: A Explanation: Distributive bargaining operates under zero-sum conditions. The essence of distributive bargaining is negotiating over who gets what share of a fixed pie. LO: 14.4: Apply the five steps of the negotiation process. Difficulty: Moderate Quest. Category: Concept 71) Making an initial offer during negotiations leads to ________ bias. A) self-serving B) framing C) attributional D) anchoring E) belief Answer: D Explanation: During a negotiation, making an initial offer leads to the anchoring bias. People tend to fixate on initial information. Once that anchoring point is set, they fail to adequately adjust it based on subsequent information. LO: 14.4: Apply the five steps of the negotiation process. Difficulty: Moderate Quest. Category: Concept
639 richard@qwconsultancy.com
640 richard@qwconsultancy.com
72) Which of the following statements is true regarding integrative bargaining? A) Integrative bargaining leaves one party a loser. B) Integrative bargaining is used in labor-management negotiations most of the time. C) Integrative bargaining focuses on long-term relationships. D) Integrative bargaining operates under zero-sum conditions. E) Integrative bargaining involves low information sharing. Answer: C Explanation: Integrative bargaining involves high information sharing and attempts to create a win-win solution. In terms of intraorganizational behavior, all things being equal, integrative bargaining is preferable to distributive bargaining because the former builds long-term relationships. LO: 14.4: Apply the five steps of the negotiation process. Difficulty: Moderate Quest. Category: Concept Labor and management at Leo Trucking cannot seem to agree upon a contract for the truck drivers. As each side contends that they are bargaining fairly, no agreement seems to be possible. The drivers are threatening to go on strike, and management knows that such a strike would prove to be costly. 73) As both sides agree that they are competing over a fixed amount of resources, each side feels that what one side wins, the other loses. Based on this information, we can say that the two sides are engaged in ________. A) conciliation B) distributive bargaining C) mediation D) integrative bargaining E) arbitration Answer: B Explanation: The essence of distributive bargaining is negotiating over who gets what share of a fixed pie. When the pie is fixed, or parties believe it is, they tend to engage in distributive bargaining. Probably the most widely cited example of distributive bargaining is labor management negotiations over wages. LO: 14.4: Apply the five steps of the negotiation process. AACSB: Analytical thinking Difficulty: Moderate Employability Skills: Knowledge Application and Analysis Quest. Category: Application
641 richard@qwconsultancy.com
74) None of the parties involved in the dispute wants to reach a win-win agreement. They are each highly focused on their demands, which they would like to achieve, otherwise known as their ________ points. A) focal B) distribution C) resistance D) tolerance E) target Answer: E Explanation: With distributive bargaining, each party has a target point that defines what he or she would like to achieve. Each also has a resistance point, which marks the lowest outcome that is acceptable, or the point below which the party would break off negotiations rather than accept a less favorable settlement. LO: 14.4: Apply the five steps of the negotiation process. AACSB: Analytical thinking Difficulty: Easy Employability Skills: Knowledge Application and Analysis Quest. Category: Application 75) The management has decided to try to find a win-win solution to help both parties resolve their differences. Management is attempting to engage in ________ bargaining. A) integrative B) reflective C) surface D) distributive E) reformatory Answer: A Explanation: In integrative bargaining, the parties try to expand the pie so that both parties are satisfied and reach a win-win agreement. LO: 14.4: Apply the five steps of the negotiation process. AACSB: Analytical thinking Difficulty: Easy Employability Skills: Knowledge Application and Analysis Quest. Category: Application
642 richard@qwconsultancy.com
76) The first step in the negotiation process is ________. A) definition of ground rules B) clarification and justification C) preparation and planning D) bargaining and problem solving E) closure and implementation Answer: C Explanation: The negotiation process is made up of five steps: (1) preparation and planning, (2) definition of ground rules, (3) clarification and justification, (4) bargaining and problem solving, and (5) closure and implementation. LO: 14.4: Apply the five steps of the negotiation process. Difficulty: Easy Quest. Category: Concept 77) The lowest value acceptable to you for a negotiated agreement is your ________. A) BATNA B) margin of error C) bid price D) asking price E) hidden value Answer: A Explanation: As part of your strategy, you should determine your and the other side's best alternative to a negotiated agreement (BATNA). Your BATNA determines the lowest value acceptable to you for a negotiated agreement. LO: 14.4: Apply the five steps of the negotiation process. Difficulty: Easy Quest. Category: Concept 78) In a negotiation process, ________ is most likely to be included in the preparation and planning step. A) justifying your original demands B) determining your best alternative to a negotiated agreement C) determining where the negotiation will take place D) designing procedures for implementing the agreement that has been worked out E) exchange of the initial proposals or demands Answer: B Explanation: In a negotiation process, your best alternative to a negotiated agreement (BATNA) is determined during the preparation and planning step. LO: 14.4: Apply the five steps of the negotiation process. Difficulty: Moderate Quest. Category: Concept
643 richard@qwconsultancy.com
79) During the ________ phase of the negotiation process parties exchange their initial proposals or demands. A) preparation and planning B) definition of ground rules C) clarification and justification D) bargaining and problem solving E) integration of preferences Answer: B Explanation: After planning and developing a strategy, it is important to begin defining with the other party the ground rules and procedures of the negotiation itself. During this phase, the parties will also exchange their initial proposals or demands. LO: 14.4: Apply the five steps of the negotiation process. Difficulty: Moderate Quest. Category: Concept 80) Which of the following activities in the negotiation process is most likely to be included in the definition of ground rules step? A) Determining your best alternative to a negotiated agreement B) Determining time constraints, if any, applicable to negotiation C) Justifying your original demands D) Formalizing the agreement that has been worked out E) Determining the history leading up to the negotiation Answer: B Explanation: Once you've done your planning and developed a strategy, you're ready to begin defining with the other party the ground rules and procedures of the negotiation itself. Points, such as who will do the negotiating, where it will take place, and what time constraints, if any, will apply, are discussed during the definition of ground rules step. LO: 14.4: Apply the five steps of the negotiation process. Difficulty: Moderate Quest. Category: Concept
644 richard@qwconsultancy.com
81) Bill is negotiating for the labor union. Today, he is presenting the evidence and papers that demonstrate the average wages in the area, the cost of healthcare for the average worker, and a pie graph demonstrating how these costs affect the new salary request. Which phase of the negotiation process is Bill currently in? A) Preparation and planning B) Definition of ground rules C) Clarification and justification D) Bargaining and problem solving E) Closure and implementation Answer: C Explanation: The clarification and justification phase are the opportunity for both parties to explain, amplify, clarify, bolster, and justify their original demands. It's an opportunity for educating and informing each other on the issues, why they are important, and how the initial demands were determined. At this point, documentation that helps support each position is presented. LO: 14.4: Apply the five steps of the negotiation process. AACSB: Analytical thinking Difficulty: Moderate Employability Skills: Knowledge Application and Analysis Quest. Category: Application 82) During of the ________ phase of the negotiation process the parties have the actual discussion to hash out an agreement. A) preparation and planning B) definition of ground rules C) closure and implementation D) bargaining and problem-solving E) integration of preferences Answer: D Explanation: The essence of the negotiation process is the actual give-and-take in trying to hash out an agreement. This is where both parties will undoubtedly need to make concessions. LO: 14.4: Apply the five steps of the negotiation process. Difficulty: Moderate Quest. Category: Concept
645 richard@qwconsultancy.com
83) Which of the following activities is included in the closure and implementation step in the negotiation process? A) Determining your best alternative to a negotiated agreement B) Justifying your original demands C) Formalizing the agreement that has been worked out D) Determining time constraints, if any, applicable to negotiation E) Exchange of the initial proposals or demands Answer: C Explanation: The closure and implementation step is the final step in the negotiation process. It involves formalizing the agreement that has been worked out and developing any procedures necessary for implementation and monitoring. LO: 14.4: Apply the five steps of the negotiation process. Difficulty: Moderate Quest. Category: Concept 84) Integrative bargaining attempts to create a win-win solution for both the parties. Answer: TRUE Explanation: Unlike distributive bargaining which operates under zero-sum conditions, integrative bargaining assumes that one or more of the possible settlements can create a win-win solution. LO: 14.4: Apply the five steps of the negotiation process. Difficulty: Moderate Quest. Category: Concept 85) In terms of intraorganizational behavior, all things being equal, distributive bargaining is preferable to integrative bargaining. Answer: FALSE Explanation: In terms of intraorganizational behavior, all things being equal, integrative bargaining is preferable to distributive bargaining because the former builds long-term relationships, while the latter tends to build animosity and deepen divisions when people must work together on an ongoing basis. LO: 14.4: Apply the five steps of the negotiation process. Difficulty: Moderate Quest. Category: Concept
646 richard@qwconsultancy.com
86) Compare and contrast integrative and distributive bargaining and explain why integrative bargaining is preferable. Answer: In terms of intraorganizational behavior, all things being equal, integrative bargaining is preferable to distributive bargaining because the former builds long-term relationships. Integrative bargaining bonds negotiators and allows them to leave the bargaining table feeling they have achieved a victory. Distributive bargaining, however, leaves one party a loser. It tends to build animosities and deepen divisions when people have to work together on an ongoing basis. Research shows that over repeated bargaining episodes, when the "losing" party feels positive about the negotiation outcome, he or she is much more likely to bargain cooperatively in subsequent negotiations. This points to an important advantage of integrative negotiations, even when you "win," you want your opponent to feel good about the negotiation. LO: 14.4: Apply the five steps of the negotiation process. Difficulty: Moderate Quest. Category: Concept 87) Describe the first step of the negotiation process. How does this step affect negotiating parties' relationship? How does the negotiating parties' relationship help determine the kind of bargaining that will be done (integrative or distributive)? Answer: The first step of the negotiating process is planning and preparation. Before negotiating it's important to prepare by considering the nature of the conflict, the history leading up to the negotiation, who's involved and what their perceptions of the conflict are, what is desired from the negotiation, and what you hope to achieve. Relationships can change as a result of a negotiation, so that's another outcome to take into consideration in the planning and preparation phase. If one side completely "wins" a negotiation, it can push the other side into resentment or animosity. It might be wiser to pursue a more compromising style. If preserving the relationship will make you seem weak and easily exploited, you may want to consider a more aggressive style. In terms of intraorganizational behavior, all things being equal, integrative bargaining is preferable to distributive bargaining because the former builds long-term relationships. Integrative bargaining bonds negotiators and allows them to leave the bargaining table feeling they have achieved a victory. Distributive bargaining, however, leaves one party a loser. LO: 14.4: Apply the five steps of the negotiation process. AACSB: Analytical thinking Difficulty: Moderate Quest. Category: Synthesis
647 richard@qwconsultancy.com
88) Define BATNA and explain its significance. Answer: BATNA stands for best alternative to a negotiated agreement. During a negotiation process, each of the parties involved should develop their BATNA during the first stage of preparation and planning. Your BATNA determines the lowest value acceptable to you for a negotiated agreement. Any offer you receive that is higher than your BATNA is better than an impasse. Conversely, you shouldn't expect success in your negotiation effort unless you're able to make the other side an offer it finds more attractive than its BATNA. If you go into your negotiation having a good idea of what the other party's BATNA is, even if you're not able to meet it, you might be able to elicit a change. LO: 14.4: Apply the five steps of the negotiation process. Difficulty: Moderate Quest. Category: Concept 89) One stereotype with regard to gender in negotiations is that women tend to ________ than men. A) act more assertively B) show higher self-interest C) be more cooperative and pleasant D) place a higher value on the economic outcome E) be more proactive in getting negotiations underway Answer: C Explanation: With regard to gender in negotiations, one stereotype is that women are more cooperative and pleasant in negotiations than men and, as a result, obtain worse outcomes. However, decades off research tend to suggest that the influence of gender in negotiations is highly context dependent, despite very slight differences in favor of men. LO: 14.5: Show how individual differences influence negotiations. AACSB: Diverse and multicultural work environments Difficulty: Moderate Quest. Category: Application 90) Which of the following is not true regarding gender differences in negotiations? A) Women may be more effective negotiators in collectivist cultures. B) Women are more hesitant to initiate negotiations. C) Women tend be slightly less assertive than men. D) Men place a lower value on economic outcomes than women. E) Women are less self-interested than men. Answer: D Explanation: Men tend to value economic outcomes more than women. LO: 14.5: Show how individual differences influence negotiations. AACSB: Diverse and multicultural work environments Difficulty: Moderate Quest. Category: Concept
648 richard@qwconsultancy.com
91) Stratford Inc. is a company that hires culturally diverse people to perform Shakespearean plays for high school students. The company employs actors and actresses from all over the world. The actors and actresses have joined together to ask the company for higher salaries. The employees are struggling over deciding who will negotiate for them. Which of the following statements is true pertaining to the effect of individual differences on negotiations? A) Women tend to negotiate better outcomes than men. B) Extraverts tend to be very successful in distributive bargaining. C) People who are anxious are not good negotiators and obtain worse outcomes. D) Individuals who score high on agreeableness dimension tend to be very successful in distributive bargaining. E) Positive moods and emotions negatively affect integrative bargaining. Answer: C Explanation: Anxiety appears to have an impact on negotiation. People who experience more anxiety about a negotiation may use deception more frequently in dealing with others. Moreover, anxious negotiators expect lower outcomes, respond to offers more quickly, and exit the bargaining process sooner, leading them to obtain worse outcomes. LO: 14.5: Show how individual differences influence negotiations. AACSB: Analytical thinking Difficulty: Moderate Employability Skills: Knowledge Application and Analysis Quest. Category: Application 92) In cross-cultural negotiations, it is especially important that the negotiations be high in openness. Answer: TRUE Explanation: In cross-cultural negotiations, it is especially important that the negotiations be high in openness. This suggests that a good strategy is to choose cross-cultural negotiators who are high on openness to experience, and to avoid factors such as time pressure that tend to inhibit learning about the other party. LO: 14.5: Show how individual differences influence negotiations. AACSB: Diverse and multicultural work environments Difficulty: Moderate Quest. Category: Concept 93) When it comes to negotiation effectiveness, people who are ambitious and likeable tend to fare the best. Answer: TRUE Explanation: Studies of marketing managers, lawyers, and construction supervisors suggest that when it comes to negotiation effectiveness, people who are ambitious and likeable tend to fare the best. LO: 14.5: Show how individual differences influence negotiations. Difficulty: Moderate Quest. Category: Concept
649 richard@qwconsultancy.com
94) Jeff has a reputation for shouting angrily while negotiating. In contrast, Anita, who is otherwise highly regarded, is known for lack of confidence during negotiations. Jeff is more likely to get the concessions he wants than Anita. Answer: TRUE Explanation: Those who are less confident and experience more threats to their self-worth during negotiation are less likely to be assertive and, therefore, experience suboptimal negotiation outcomes. LO: 14.5: Show how individual differences influence negotiations. AACSB: Diverse and multicultural work environments Difficulty: Moderate Employability Skills: Knowledge Application and Analysis Quest. Category: Concept 95) How does self-efficacy impact negotiation outcomes? Answer: Self-efficacy is one individual-difference variable that does seem consistently to relate to negotiation outcomes. This is a fairly intuitive finding—it isn't too surprising to hear that those who believe they will be more successful in negotiation situations tend to perform more effectively. It may be that individuals who are more confident stake out stronger claims, are less likely to back down from their positions, and exhibit confidence that intimidates others. LO: 14.5: Show how individual differences influence negotiations. Difficulty: Hard Quest. Category: Concept 96) Explain how gender affects negotiating and describe what this means to the outcomes of the negotiation process. Answer: Compared to men, women tend to behave in a slightly less assertive, less selfinterested, more cooperative and concessional manner. However, these differences are reduced when there are restrictions on communication. The research also suggests that women can actually be more competitive than men when the other negotiator engages in a "tit-for-tat" strategy. Furthermore, men actually behave more unethically in negotiations than women, behaving more opportunistically and morally disengaging from the negotiation. However, when it comes to initiating negotiations, women are less likely to initiate than men, especially when the situation was ambiguous and when gender role reinforcing information is present in the situational context. For instance, women are less likely than men to engage in salary negotiations following a job offer. LO: 14.5: Show how individual differences influence negotiations. AACSB: Diverse and multicultural work environments Difficulty: Moderate Quest. Category: Synthesis
650 richard@qwconsultancy.com
97) What strategies can be used to navigate ethical challenges in negotiations? Answer: Build relationships: Negotiators are less likely to utilize unethical tactics when they know their counterpart well and trust them. They understand that their relationship will be in jeopardy if the other side discovers that they have engaged in unethical behavior. Negotiate in person: Research has shown that direct face-to-face contact between negotiating parties decreases the use of ethically ambiguous tactics compared with negotiation conducted via e-mail or phone. Get to know the whole person: Negotiators tend to rely on stereotypes when attempting to manage cultural differences in international negotiation, Instead, take the time to learn about the negotiating partner's culture and get to know them individually, including their skills and work experience. LO: 14.6: Describe the social factors that influence negotiations. AACSB: Ethical understanding and reasoning Difficulty: Moderate Employability Skills: Business Ethics and Social Responsibility Quest. Category: Concept 98) Describe the preferred negotiating techniques that would be implemented by a manager from China. What conflict management techniques would he use? What role would a third-party negotiator be likely to have if no agreement could be reached? Answer: Collectivist cultures see people as deeply embedded in social situations. As a result, collectivists are more likely to seek to preserve relationships and promote the good of the group as a whole than individualists. To preserve peaceful relationships, collectivists will avoid direct expression of conflicts, preferring to use more indirect methods for resolving differences of opinion. Compromising and avoiding are the most preferred methods of conflict management in China. Smoothing, or playing down differences while emphasizing common interests between the conflicting parties, is also a good technique to maintain relationships. Collectivists such as the Chinese may also be more interested in demonstrations of concern and working through third parties to resolve disputes. A Chinese manager would most likely call in a mediator, a neutral third party who facilitates a negotiated solution by using reasoning and persuasion, earlier in the negotiation process in order to avoid conflict and maintain a relationship with all parties. LO: 14.6: Describe the social factors that influence negotiations. AACSB: Analytical thinking Difficulty: Moderate Quest. Category: Synthesis
651 richard@qwconsultancy.com
99) Xanadu Industries manufactures and sells the same calipers as Utopia Industries. Employee wages account for 35 percent of the cost of manufacturing calipers at both Xanadu Industries and Utopia Industries. Xanadu Industries is seeking a competitive advantage over Utopia Industries. Richard, the manager put in charge of devising a strategy to meet this end, suggests lowering employee wages. This leads to a grave conflict between Richard and the labor union. Which of the following, if true, would suggest that the labor union will accept Richard's suggestion to lower the wages? A) As they make a large number of precision instruments, caliper manufacturers receive huge volume discounts on raw materials. B) Utopia Industries recently set up a new manufacturing facility in the vicinity. C) Xanadu Industries has taken away 20 percent of Utopia Industries' business over the last year. D) Utopia Industries pays its employees, on average, 10 percent more than does Xanadu Industries. E) Many people who work for manufacturing plants live in areas in which the manufacturing plant is the only source of employment. Answer: E Explanation: Xanadu Industries would not lose many workers if it did reduce wages as the people live in areas in which the manufacturing plant they work for is the only industry. Reducing the cost of raw materials is a suitable way to reduce manufacturing costs. The employees of Xanadu Industries will look for employment at the new manufacturing facility of Utopia Industries if wages are lowered, hence weakening the argument. If Xanadu Industries has taken away a part of Utopia Industries' business, they already have the upper hand. And as Xanadu Industries' wages are relatively low, lowering it further would not be advised. LO: 14.7: Assess the roles and functions of third-party negotiations. AACSB: Reflective thinking Difficulty: Hard Employability Skills: Critical Thinking Quest. Category: Critical Thinking 100) A neutral third party who facilitates a negotiated solution by using reasoning, persuasion, and suggestions for alternatives is a(n) ________. A) plaintiff B) mediator C) agent D) consultant E) executor Answer: B Explanation: A mediator is a neutral third party who facilitates a negotiated solution by using reasoning and persuasion, suggesting alternatives, and the like. Mediators are widely used in labor-management negotiations and in civil court disputes. LO: 14.7: Assess the roles and functions of third-party negotiations. Difficulty: Easy Quest. Category: Concept
652 richard@qwconsultancy.com
101) In which of the following situations is mediation most likely to be effective? A) The conflict between the parties involved in mediation is at a moderate level. B) One of the parties involved in the negotiation perceives the mediator as biased. C) There is a large gap between the target points of both the parties. D) One of the parties involved in the negotiation perceives the mediator as coercive. E) None of the parties is willing to disclose its BATNA. Answer: A Explanation: Mediation is most effective under moderate levels of conflict. Also, perceptions of the mediator are important; to be effective, the mediator must be perceived as neutral and noncoercive. LO: 14.7: Assess the roles and functions of third-party negotiations. Difficulty: Moderate Quest. Category: Concept 102) A third-party negotiation that always results in a settlement is ________. A) mediation B) consultancy C) arbitration D) conciliation E) execution Answer: C Explanation: An arbitrator is a third party with the authority to dictate an agreement. Arbitration can be voluntary or compulsory. The big plus of arbitration over mediation is that it always results in a settlement. LO: 14.7: Assess the roles and functions of third-party negotiations. Difficulty: Easy Quest. Category: Concept 103) What factor would most likely increase the negative impact of an arbitration session? A) Limited options of the arbitrator B) Heavy-handedness of the arbitrator C) Compulsory nature of the arbitration D) Voluntary nature of the arbitration E) Establishment of an agreement that is nonbinding Answer: B Explanation: Whether or not there is a negative side depends on how heavy-handed the arbitrator appears. If one party is left feeling overwhelmingly defeated, that party is certain to be dissatisfied and unlikely to graciously accept the arbitrator's decision. LO: 14.7: Assess the roles and functions of third-party negotiations. Difficulty: Moderate Quest. Category: Concept
653 richard@qwconsultancy.com
104) In third-party negotiations, a third party who provides an informal communication link between the negotiator and the opponent is known as a(n) ________. A) plaintiff B) arbitrator C) consultant D) conciliator E) executor Answer: D Explanation: A conciliator is a trusted third party who provides an informal communication link between the negotiator and the opponent. In practice, conciliators typically act as more than mere communication conduits. They also engage in fact finding, interpret messages, and persuade disputants to develop agreements. LO: 14.7: Assess the roles and functions of third-party negotiations. Difficulty: Easy Quest. Category: Concept 105) Megatron wants to bring in a neutral third party who will facilitate a negotiated solution by using reasoning, persuasion, suggesting alternatives, and the like. This person is known as a(n) ________. A) mediator B) arbitrator C) plaintiff D) consultant E) bailor Answer: A Explanation: A mediator is a neutral third party who facilitates a negotiated solution by using reasoning and persuasion, suggesting alternatives, and the like. Mediators are widely used in labor-management negotiations and in civil court disputes. LO: 14.7: Assess the roles and functions of third-party negotiations. AACSB: Analytical thinking Difficulty: Moderate Employability Skills: Knowledge Application and Analysis Quest. Category: Application
654 richard@qwconsultancy.com
106) The home buyer wants to bring in a third party who will have the authority to dictate an agreement. This person will be a(n) ________. A) mediator B) arbitrator C) conciliator D) consultant E) plaintiff Answer: B Explanation: An arbitrator is a third party with the authority to dictate an agreement. Arbitration can be voluntary or compulsory. The big plus of arbitration over mediation is that it always results in a settlement. LO: 14.7: Assess the roles and functions of third-party negotiations. AACSB: Analytical thinking Difficulty: Moderate Employability Skills: Knowledge Application and Analysis Quest. Category: Application 107) The two parties eventually agree to ask Sam Spade, an attorney and trusted friend, to help them informally to solve their problem. Sam's role is that of a(n) ________. A) mediator B) arbitrator C) conciliator D) consultant E) plaintiff Answer: C Explanation: A conciliator is a trusted third party who provides an informal communication link between the negotiator and the opponent. In practice, conciliators typically act as more than mere communication conduits. They also engage in fact finding, interpret messages, and persuade disputants to develop agreements. LO: 14.7: Assess the roles and functions of third-party negotiations. AACSB: Analytical thinking Difficulty: Moderate Employability Skills: Knowledge Application and Analysis Quest. Category: Application 108) Negotiators who consistently act in a way that demonstrates competence, honesty, and integrity will usually have better outcomes in the long run. Answer: TRUE Explanation: Negotiators who consistently act in a way that demonstrates competence, honesty, and integrity will usually have better outcomes in the long run. Individuals who have higher reputations are better liked and have more friends and allies—in other words, they have more social resources, which may give them more understood power in negotiations. LO: 14.7: Assess the roles and functions of third-party negotiations. Difficulty: Moderate Quest. Category: Concept
655 richard@qwconsultancy.com
656 richard@qwconsultancy.com
109) When it comes to negotiation, having a reputation for being trustworthy doesn't matter. Answer: FALSE Explanation: When it comes to negotiation, having a reputation for being trustworthy matters. In short, trust in a negotiation process opens the door to many forms of integrative negotiation strategies that benefit both parties. LO: 14.7: Assess the roles and functions of third-party negotiations. Difficulty: Moderate Quest. Category: Concept 110) Individuals who have more social resources may have more implicit power in negotiations. Answer: TRUE Explanation: Individuals who have more solid reputations are better liked and have more friends and allies—in other words, they have more social resources, which may give them more implicit power in negotiations. LO: 14.7: Assess the roles and functions of third-party negotiations. Difficulty: Moderate Quest. Category: Concept 111) The advantage of mediation over arbitration is that there is always a settlement. Answer: FALSE Explanation: The advantage of arbitration over mediation is that it always results in a settlement. LO: 14.7: Assess the roles and functions of third-party negotiations. Difficulty: Moderate Quest. Category: Concept Organizational Behavior, 19e (Robbins/Judge) Chapter 15 Foundations of Organization Structure 1) The term ________ is used to describe the degree to which tasks in an organization are subdivided into separate jobs. A) social clustering B) bureaucracy C) work specialization D) centralization E) departmentalization Answer: C Explanation: We use the term work specialization, or division of labor, to describe the degree to which activities in the organization are subdivided into separate jobs. The essence of work specialization is to divide a job into a number of steps, each completed by a separate individual. LO: 15.1: Identify seven elements of an organization's structure. Difficulty: Easy Quest. Category: Concept 2) Which of the following statements is true regarding work specialization? A) Work specialization indicates to what degree will there be rules and formalized regulations to direct employees and managers. B) Work specialization decreases the time spent in changing tasks. 657 richard@qwconsultancy.com
C) Work specialization increases the cost of finding and training workers to do specific and repetitive tasks. D) Work specialization decreases efficiency and productivity. E) Work specialization hinders efficient use of employee skills. Answer: B Explanation: Work specialization indicates the degree to which activities in the organization are subdivided into separate jobs. Work specialization makes the most efficient use of employees' skills. Less time is spent in changing tasks, putting away tools and equipment from a prior step, and getting ready for another. LO: 15.1: Identify seven elements of an organization's structure. Difficulty: Moderate Quest. Category: Concept
658 richard@qwconsultancy.com
3) Work specialization ________. A) is always a means of encouraging employee satisfaction B) is an effective solution to too much recentralization C) increases efficiency and productivity by encouraging the creation of customized inventions and machinery D) is difficult to implement without automation technology E) is a frustrating cause of reduced efficiency Answer: C Explanation: Work specialization increases efficiency and productivity by encouraging the creation of customized inventions and machinery. LO: 15.1: Identify seven elements of an organization's structure. Difficulty: Moderate Quest. Category: Concept 4) ________ refers to the basis by which jobs are grouped. A) Social clustering B) Span of control C) Work specialization D) Centralization E) Departmentalization Answer: E Explanation: The basis by which jobs are grouped is called departmentalization. LO: 15.1: Identify seven elements of an organization's structure. Difficulty: Easy Quest. Category: Concept 5) A plant manager organizes a plant by separating engineering, accounting, manufacturing, personnel, and purchasing into departments. The plant is departmentalized on the basis of ________. A) target customer B) product C) function D) geography E) service Answer: C Explanation: One of the most popular ways to group activities is by functions performed. A manufacturing manager might organize a plant into engineering, accounting, manufacturing, personnel, and supply specialists departments. LO: 15.1: Identify seven elements of an organization's structure. AACSB: Analytical thinking Difficulty: Moderate Employability Skills: Knowledge Application and Analysis Quest. Category: Application
659 richard@qwconsultancy.com
6) AgriProducers has customers all over the United States that need their soil and crops tested in their labs. The crops and soil are different in the various large areas of the nation, such as the West Coast and the Midwest. Which type of departmentalization would be best for AgriProducers? A) Functional B) Process C) Product D) Geographic E) Temporal Answer: D Explanation: AgriProducers' needs would best be served by departmentalizing on the basis of geography. This form is valuable when an organization's customers are scattered over a large geographic area and have similar needs based on their location. LO: 15.1: Identify seven elements of an organization's structure. AACSB: Analytical thinking Difficulty: Hard Employability Skills: Knowledge Application and Analysis Quest. Category: Application 7) Procter & Gamble has separate departments for Tide, Pampers, Charmin, and Pringles. This is an example of departmentalization by ________. A) function B) process C) geography D) product E) interest Answer: D Explanation: Firms can departmentalize jobs by the type of product or service the organization produces. For example, Procter & Gamble places each major product, such as Tide, Pampers, Charmin, and Pringles, under an executive who has complete global responsibility for it. LO: 15.1: Identify seven elements of an organization's structure. AACSB: Analytical thinking Difficulty: Moderate Employability Skills: Knowledge Application and Analysis Quest. Category: Application
660 richard@qwconsultancy.com
8) E-GO, a company specializing in electric vehicle production, has separate departments for government contracts, commercial fleet contracts, and personal use contracts. E-Go is departmentalizing on the basis of ________. A) differences in product lines B) differences in the type of function being used C) geographic market differences D) the particular type of customer the organization is trying to reach E) service differences Answer: D Explanation: A firm can departmentalize based on the particular type of customer the organization seeks to reach. Microsoft, for example, is organized around four customer markets: consumers, large corporations, software developers, and small businesses. Customers in each department have a common set of problems and needs best met by having specialists for each. EGO. is departmentalized to meet different customer needs. LO: 15.1: Identify seven elements of an organization's structure. AACSB: Analytical thinking Difficulty: Moderate Employability Skills: Knowledge Application and Analysis Quest. Category: Application 9) ________ refers to the unbroken line of authority that extends from the top of the organization to the lowest echelon and clarifies who reports to whom. A) Chain of command B) Path of power C) Span of control D) Line of responsibility E) Web of authority Answer: A Explanation: The chain of command is an unbroken line of authority that extends from the top of the organization to the lowest echelon and clarifies who reports to whom. It answers questions such as to whom do I go if I have a problem and to whom am I responsible? LO: 15.1: Identify seven elements of an organization's structure. Difficulty: Easy Quest. Category: Concept
661 richard@qwconsultancy.com
10) Chain of command is a key element of an organization's structure. Which of the following key questions is addressed by this element? A) How many individuals can a manager efficiently and effectively direct? B) To whom do individuals and groups report? C) On what basis will jobs be grouped together? D) To what degree are activities subdivided into separate jobs? E) To what degree will there be rules and regulations to direct employees and managers? Answer: B Explanation: The chain of command is an unbroken line of authority that extends from the top of the organization to the lowest echelon and clarifies who reports to whom. It answers questions such as to whom do I go if I have a problem and to whom am I responsible? LO: 15.1: Identify seven elements of an organization's structure. Difficulty: Moderate Quest. Category: Concept 11) Span of control is a key element of an organization's structure. Which of the following key questions is addressed by this element? A) On what basis will jobs be grouped together? B) To whom do individuals and groups report? C) To what degree will there be rules and regulations to direct employees and managers? D) How many individuals can a manager efficiently and effectively direct? E) To what degree are activities subdivided into separate jobs? Answer: D Explanation: Span of control determines the number of levels and managers an organization has. All things being equal, the wider or larger the span, the more efficient the organization. LO: 15.1: Identify seven elements of an organization's structure. Difficulty: Moderate Quest. Category: Concept 12) The term ________ refers to the rights inherent in a managerial position to give orders and expect them to be obeyed. A) chain of command B) authority C) power D) span of control E) leadership Answer: B Explanation: Chain of command can't be discussed without also discussing authority. Authority refers to the rights inherent in a managerial position to give orders and expect them to be obeyed. To facilitate coordination, each managerial position is given a place in the chain of command, and each manager is given a degree of authority in order to meet his or her responsibilities. LO: 15.1: Identify seven elements of an organization's structure. Difficulty: Easy Quest. Category: Concept
662 richard@qwconsultancy.com
13) Which principle helps preserve the concept of an unbroken line of authority in chain of command? A) Span of control B) Departmentalization C) Cross-functionality D) Centralization E) Unity of command Answer: E Explanation: Chain of command can't be discussed without also discussing unity of command. The principle of unity of command helps preserve the concept of an unbroken line of authority. It says a person should have one and only one superior to whom they are directly responsible. If the unity of command is broken, an employee might have to cope with conflicting demands or priorities from several superiors. LO: 15.1: Identify seven elements of an organization's structure. Difficulty: Moderate Quest. Category: Concept 14) According to the principle of unity of command, ________. A) managers should limit their oversight to a maximum of 12 employees B) managers should oversee 1 - 4 employees on average C) an individual should be directly responsible to only one supervisor D) an organization should be departmentalized on the basis of functions E) employees should report directly to two supervisors to maintain task balance Answer: C Explanation: Chain of command can't be discussed without also discussing unity of command. The principle of unity of command helps preserve the concept of an unbroken line of authority. It says a person should have one and only one superior to whom he or she is directly responsible. If the unity of command is broken, an employee might have to cope with conflicting demands or priorities from several superiors. LO: 15.1: Identify seven elements of an organization's structure. Difficulty: Moderate Quest. Category: Concept
663 richard@qwconsultancy.com
Mike Morrell is a production supervisor in Porco Rosso, an aircraft manufacturing firm. After evaluating Mike's managerial capabilities, some members of the senior management feel that average productivity would improve if more workers were assigned to him. However, some managers are skeptical about Mike's ability to handle more workers. 15) Which of the following statements, if true, would support the argument favoring increased span of control? A) Mike's team consists of highly experienced and skilled labor. B) Porco Rosso recently became the world's leading aircraft manufacturer. C) Mike is receiving increasing complaints about interpersonal conflicts within the team. D) Mike's team is highly dependent on his guidance, even on minor issues. E) When additional workers were assigned to Mike on a previous occasion, it led to unfavorable results. Answer: A Explanation: As Mike's team consists of highly experienced and skilled labor, Mike's managerial abilities would complement his team's abilities and certainly improve the average productivity. Porco Rosso becoming the leading aircraft manufacturer is simply stating a mere and irrelevant fact. If Mike is receiving increasing complaints about interpersonal conflicts within his team, then he isn't doing a very good job to begin with and hence must not be assigned more workers. If Mike's team is highly dependent on his guidance, his hands are already full and assigning more workers certainly wouldn't be advised. The fact that assigning additional workers to Mike had previously led to unfavorable results indicates that he is incapable of handling more than the present number. LO: 15.1: Identify seven elements of an organization's structure. AACSB: Reflective thinking Difficulty: Hard Employability Skills: Critical Thinking Quest. Category: Critical Thinking
664 richard@qwconsultancy.com
16) Which of the following statements, if true, would weaken the argument favoring increased span of control? A) Mike has a reputation of handling interpersonal conflicts effectively. B) Performance reports from the last few years show that teams that were monitored closely had higher productivity. C) Porco Rosso recently launched its latest engine model, the PR-4. D) Market trends indicate that the demand for aircraft is expected to increase substantially over the next few years. E) When additional workers were assigned to Mike on a previous occasion, it led to favorable and productive results. Answer: B Explanation: Mike's ability to handle conflicts effectively and his track record, which shows that increasing his span of control led to good results in the past, if anything, strengthen the argument. Market trends and the fact that Porco Rosso recently released a new engine are irrelevant as they neither strengthen nor weaken the argument. As previous performance reports show that teams that were monitored closely have higher productivity, it weakens the argument considerably. Monitoring Mike's team closely would benefit the productivity and assigning more workers to him would retard it. LO: 15.1: Identify seven elements of an organization's structure. AACSB: Reflective thinking Difficulty: Hard Employability Skills: Critical Thinking Quest. Category: Critical Thinking 17) The number of subordinates that a manager directs is referred to as ________. A) span of control B) unity of command C) chain of command D) line of responsibility E) leadership web Answer: A Explanation: The number of employees that a manager can efficiently and effectively direct describes the span of control. It is important because it largely determines the number of levels and managers an organization has. LO: 15.1: Identify seven elements of an organization's structure. Difficulty: Easy Quest. Category: Concept
665 richard@qwconsultancy.com
18) A ________ is consistent with recent efforts by companies to reduce costs, cut overhead, speed up decision making, increase flexibility, get closer to customers, and empower employees. A) wider span of control B) high degree of formalization C) longer chain of command D) lack of work specialization E) high degree of centralization Answer: A Explanation: The trend in recent years has been toward wider spans of control, which are consistent with efforts to reduce costs, cut overhead, speed decision making, increase flexibility, get closer to customers, and empower employees. LO: 15.1: Identify seven elements of an organization's structure. Difficulty: Moderate Quest. Category: Concept 19) A narrow span of control ________. A) encourages employee autonomy B) is expensive because it adds levels of management C) simplifies vertical communication D) increases the speed of decision making E) discourages overly tight supervision Answer: B Explanation: Narrow spans of control are expensive because they add levels of management. The added levels of hierarchy slow down decision making and tend to isolate upper management. LO: 15.1: Identify seven elements of an organization's structure. Difficulty: Moderate Quest. Category: Concept 20) Doug's company is falling behind the competition when it comes to being flexible and being in a position to make decisions quickly. If Doug wants to cut costs significantly, increase flexibility, and speed up decision making, Doug should focus on which structural decision? A) Increasing the span of control B) Decreasing the unity of command C) Increasing the degree of formalization D) Decentralizing management decisions E) Departmentalizing the company on the basis of products Answer: A Explanation: The trend in recent years has been toward wider spans of control. They're consistent with firm's efforts to reduce costs, cut overhead, speed decision making, increase flexibility, get closer to customers, and empower employees. LO: 15.1: Identify seven elements of an organization's structure. AACSB: Analytical thinking Difficulty: Moderate Employability Skills: Knowledge Application and Analysis Quest. Category: Application 666 richard@qwconsultancy.com
21) A drawback of a narrow span of control is that it ________. A) increases effectiveness B) is more expensive C) encourages employee autonomy D) decreases the number levels in the organizational hierarchy E) increases participatory decision making Answer: B Explanation: Narrow spans have three major drawbacks. First, they're expensive because they add levels of management. Second, they make vertical communication in the organization more complex. The added levels of hierarchy slow down decision making and tend to isolate upper management. Third, narrow spans encourage overly tight supervision and discourage employee autonomy. LO: 15.1: Identify seven elements of an organization's structure. Difficulty: Moderate Quest. Category: Concept Gordon is a new divisional manager at AskSocrates.com. In reorganizing his division, he must make some decisions regarding the span of control for management within his division. 22) Span of control determines ________. A) the employee grievance policy of the organization B) the number of levels and managers in an organization C) where decisions are made D) how jobs will be grouped E) how employees will be compensated Answer: B Explanation: The number of employees a manager can efficiently and effectively direct refers to the span of control. It is important because it largely determines the number of levels and managers an organization has. All things being equal, the wider or larger the span, the more efficient the organization. LO: 15.1: Identify seven elements of an organization's structure. AACSB: Analytical thinking Difficulty: Moderate Employability Skills: Knowledge Application and Analysis Quest. Category: Application
667 richard@qwconsultancy.com
23) While observing the departments in his division, Gordon notices that some managers with wide spans of control seem to perform more effectively than other managers with similarly sized spans of control. Which of the following statements is most likely to be true regarding the highperforming managers? A) These managers are paid higher salaries than the low-performing managers. B) These managers have external locus of control. C) The employees within their departments tend to compete to reach productivity goals, which boosts performance. D) The employees within their departments score high on agreeableness. E) The employees within their departments are highly skilled and very knowledgeable about their jobs. Answer: E Explanation: To ensure performance doesn't suffer because of wider spans of control, organizations have to invest heavily in employee training. Managers recognize they can handle a wider span when employees know their jobs inside and out or can turn to co-workers when they have questions. LO: 15.1: Identify seven elements of an organization's structure. AACSB: Analytical thinking Difficulty: Moderate Employability Skills: Knowledge Application and Analysis Quest. Category: Application 24) In an organization, top managers make all the decisions and lower-level managers merely carry out their directives. This organization is most likely to be characterized by ________. A) a narrower span of control B) a low degree of formalization C) a high degree of centralization D) a high degree of departmentalization E) a low degree of work specialization Answer: C Explanation: Centralization refers to the degree to which decision making is concentrated at a single point in the organization. In centralized organizations, top managers make all the decisions and lower-level managers merely carry out their directives. In organizations at the other extreme, decentralized decision making is pushed down to the managers closest to the action. LO: 15.1: Identify seven elements of an organization's structure. Difficulty: Moderate Quest. Category: Concept
668 richard@qwconsultancy.com
25) The degree to which decision making is concentrated at a single point in the organization is ________. A) formalization B) departmentalization C) centralization D) work specialization E) line of command Answer: C Explanation: Centralization refers to the degree to which decision making is concentrated at a single point in the organization. In centralized organizations, top managers make all the decisions and lower-level managers merely carry out their directives. LO: 15.1: Identify seven elements of an organization's structure. Difficulty: Easy Quest. Category: Concept 26) According to a survey, many workers in Qtopia Inc. are dissatisfied with their jobs. The survey revealed that most of the dissatisfied workers believe that they have very little control over their job assignments. Therefore, to increase workers' job satisfaction, Boris, a production manager, feels they need to only concentrate on changing the dissatisfied workers' beliefs regarding the degree of control they have over their job assignments. Which of the following, if true, would weaken the conclusion made by Boris? A) Many dissatisfied workers feel that their wages are too low and working conditions are unsatisfactory. B) The number of workers in Qtopia Inc. who are satisfied with their jobs is greater than the number of workers who are dissatisfied. C) The workers in Qtopia Inc. are more dissatisfied than workers in other companies. D) The 2008 edition of a popular magazine included Qtopia Inc. on its list of 100 best companies to work for. E) The workers in the company who are satisfied with their jobs believe that they have sufficient control over their job assignments. Answer: A Explanation: If many workers are also dissatisfied with their wages and working conditions, the management has to do more than just change the workers' beliefs regarding degree of control, hence, weakening the argument. The fact that the number of workers in the company who are satisfied with their jobs is greater than the number of workers who are dissatisfied is out of scope and so is the fact that the workers in the company are more dissatisfied than workers in other companies. That the 2008 edition of a popular magazine included Qtopia Inc. on its list of 100 best companies to work for is a mere fact that isn't relevant to the current problem. Boris' argument talks about the dissatisfied workers and not the satisfied ones. Hence, the fact that the workers in the company who are satisfied with their jobs believe that they have sufficient control over their job assignments is irrelevant. LO: 15.1: Identify seven elements of an organization's structure. AACSB: Reflective thinking Difficulty: Hard Employability Skills: Critical Thinking Quest. Category: Critical Thinking 669 richard@qwconsultancy.com
27) The more the lower-level personnel provide input or are actually given the discretion to make decisions, the ________ within that organization. A) higher is the degree of formalization B) lower is the level of empowerment C) lower is the degree of work specialization D) higher is the degree of departmentalization E) higher is the degree of decentralization Answer: E Explanation: In decentralized organizations, decision making is pushed down to the managers closest to the action. A decentralized organization can act more quickly to solve problems, more people provide input into decisions, and employees are less likely to feel alienated from those who make decisions that affect their work lives. LO: 15.1: Identify seven elements of an organization's structure. Difficulty: Moderate Quest. Category: Concept 28) The degree to which jobs within the organization are standardized is referred to as ________. A) formalization B) empowerment C) work specialization D) departmentalization E) decentralization Answer: A Explanation: Formalization refers to the degree to which jobs within the organization are standardized. If a job is highly formalized, the employee has a minimal amount of discretion over what to do and when and how to do it. LO: 15.1: Identify seven elements of an organization's structure. Difficulty: Easy Quest. Category: Concept
670 richard@qwconsultancy.com
29) Publishing representatives who call on college professors to inform them of their company's new publications have a great deal of freedom in their jobs. They have only a general sales pitch, which they tailor as needed. Based on this information, we can say that the job of these publishing representatives ________. A) scores low on the degree of work specialization B) scores low on the degree of decentralization C) scores high on the degree of departmentalization D) scores high on the degree of centralization E) scores low on the degree of formalization Answer: E Explanation: Formalization refers to the degree to which jobs within the organization are standardized. If a job is highly formalized, the employee has a minimum amount of discretion over what, when, and how to do it. The publishing representatives have a great deal of freedom in their jobs, so we can say that the job scores low on the degree of formalization. LO: 15.1: Identify seven elements of an organization's structure. AACSB: Analytical thinking Difficulty: Moderate Employability Skills: Knowledge Application and Analysis Quest. Category: Application 30) The back-office staff of an investment company are required to start their day at the office by 7:30 a.m. Once at work, the employees follow a precise set of procedures as they complete their tasks. The jobs of the back-office staff ________. A) have a wide span of control B) score high on the degree of departmentalization C) score high on the degree of formalization D) lack a unity of command E) score low on the degree of work specialization Answer: C Explanation: Formalization refers to the degree to which jobs within the organization are standardized. If a job is highly formalized, the employee has a minimum amount of discretion over what, when, and how to do it. A high degree of formalization is characterized by clearly defined procedures covering work processes in organizations. Since the back-office employees have to follow a set of precise procedures dictated by management, their jobs score high on the degree of formalization. LO: 15.1: Identify seven elements of an organization's structure. AACSB: Analytical thinking Difficulty: Moderate Employability Skills: Knowledge Application and Analysis Quest. Category: Application
671 richard@qwconsultancy.com
31) One way to overcome too much division within an organization and retain the positive elements of structure is to encourage or build in boundary-spanning roles. Answer: TRUE Explanation: One way to overcome is too much division within an organization and retain the positive elements of structure is to encourage or build in boundary-spanning roles. Organizations can use formal mechanisms to facilitate boundary-spanning activities through their structures. One method is to assign formal liaison roles or develop committees of individuals from different areas of the organization. Development activities can also facilitate boundary spanning. Employees with experience in multiple functions, such as accounting and marketing, are more likely to engage in boundary spanning. LO: 15.1: Identify seven elements of an organization's structure. Difficulty: Moderate Quest. Category: Concept 32) The chain of command is an unbroken line of authority that extends from the top of the organization to the lowest echelon and clarifies who reports to whom. Answer: TRUE Explanation: The chain of command is an unbroken line of authority that extends from the top of the organization to the lowest echelon and clarifies who reports to whom. LO: 15.1: Identify seven elements of an organization's structure. Difficulty: Easy Quest. Category: Concept 33) Wide spans of control make vertical communication in the organization more complex. Answer: FALSE Explanation: Narrow spans of control add levels of management. This makes vertical communication in the organization more complex. LO: 15.1: Identify seven elements of an organization's structure. Difficulty: Moderate Quest. Category: Concept 34) Geographic departmentalization is valuable when an organization's customers are scattered over a large geographic area and have similar needs within their locations. Answer: TRUE Explanation: When a firm is departmentalized based on geography, each function (for example, sales) may be allocated to different regions or markets (e.g., a country may have northern, western, southern, and eastern regions). This form is valuable when an organization's customers are scattered over a large geographic area and have similar needs within their locations. LO: 15.1: Identify seven elements of an organization's structure. Difficulty: Moderate Quest. Category: Concept
672 richard@qwconsultancy.com
35) Most managers today recognize the economies that specialization provides in certain jobs and the problems when it is carried too far. Answer: TRUE Explanation: Most managers today recognize the economies that specialization provides in certain jobs and the problems when it is carried too far. LO: 15.1: Identify seven elements of an organization's structure. Difficulty: Moderate Quest. Category: Concept 36) As compared to a centralized organization, a decentralized organization can act more quickly to solve problems. Answer: TRUE Explanation: An organization characterized by centralization is inherently different structurally from one that's decentralized. A decentralized organization can act more quickly to solve problems, more people provide input into decisions, and employees are less likely to feel alienated from those who make decisions that affect their work lives. LO: 15.1: Identify seven elements of an organization's structure. Difficulty: Moderate Quest. Category: Concept 37) Departmentalization refers to the degree to which jobs within the organization are standardized. Answer: FALSE Explanation: Formalization refers to the degree to which jobs within the organization are standardized. The basis by which jobs are grouped is called departmentalization. LO: 15.1: Identify seven elements of an organization's structure. Difficulty: Easy Quest. Category: Concept
673 richard@qwconsultancy.com
38) Using examples, describe the five common ways of departmentalization. Answer: The five common ways of departmentalization are as follows: a) One of the most popular ways to group activities is by functions performed. A manufacturing manager might organize his or her plant by separating engineering, accounting, manufacturing, personnel, and supply specialists into common departments. b) Tasks can also be departmentalized by type of product the organization produces. Procter & Gamble is organized along these lines. Each major product is placed under the authority of an executive who has complete global responsibility for that product. c) Another way to departmentalize is on the basis of geography or territory. The sales function, for instance, may have western, southern, Midwestern, and eastern regions. Each of these regions is, in effect, a department organized around geography. d) Process departmentalization works for processing customers as well as products. At an Alcoa aluminum tubing plant in upstate New York, production is organized into five departments: casting; press; tubing; finishing; and inspecting, packing, and shipping. Each department specializes in one specific phase in the production of aluminum tubing. e) A final category is to use the particular type of customer the organization seeks to reach. Microsoft recently reorganized around four customer markets: consumers, large corporations, software developers, and small businesses. LO: 15.1: Identify seven elements of an organization's structure. Difficulty: Moderate Quest. Category: Concept 39) Discuss boundary spanning in organizations and between organizations. Answer: Within a single organization, boundary spanning occurs when individuals form relationships with people outside their formally assigned groups. Boundary spanning activities occur not only within but also between organizations. Positive results are especially strong in organizations that encourage extensive internal communication; external boundary spanning is most effective when it is followed up with internal boundary spanning. LO: 15.1: Identify seven elements of an organization's structure. Difficulty: Moderate Quest. Category: Concept
674 richard@qwconsultancy.com
40) Explain the concept of span of control and identify which common organizational design would be most likely to have the widest span of control. Answer: The question of span of control is important because, to a large degree, it determines the number of levels and managers an organization has. It answers the question "How many employees can a manager efficiently and effectively direct?" All things being equal, the wider or larger the span of control, the more efficient the organization. A simple structure, because of its natural small size, would have a fairly narrow span of control. The bureaucracy is typically characterized by narrow spans of control and decision making that follows the chain of command. A matrix organization, characterized by dual lines of authority that combine functional and product departmentalization, would lend itself to a very wide span of control with managers from the different departments all being in some part responsible for a large number of employees. One of the disadvantages of the matrix lies in the confusion it creates its propensity to foster power struggles, and the stress it places on individuals, somewhat due to the wide span of control. Without the unity-of-command concept, ambiguity about who reports to whom is significantly increased and often leads to conflict. LO: 15.1: Identify seven elements of an organization's structure. AACSB: Analytical thinking Difficulty: Hard Quest. Category: Synthesis 41) What are the differences between centralized organizations and decentralized organizations? Answer: In centralized organizations, top managers make all the decisions, and lower-level managers merely carry out their directives. In decentralized organizations, decision making is pushed down to the managers closest to the action. An organization characterized by centralization is inherently different structurally from one that is decentralized. A decentralized organization can act more quickly to solve problems, more people provide input into decisions, and employees are less likely to feel alienated from those who make decisions that affect their work lives. LO: 15.1: Identify seven elements of an organization's structure. AACSB: Analytical thinking Difficulty: Moderate Quest. Category: Synthesis 42) Define formalization and describe the characteristics of a highly formalized organization. Answer: Formalization refers to the degree to which jobs within an organization are standardized. If a job is highly formalized, the employee has a minimum amount of discretion over what, when, and how to do it. In a highly formalized organization, employees are always expected to handle the same input in exactly the same way, resulting in a consistent and uniform output. There are explicit job descriptions, lots of organizational rules, and clearly defined procedures covering work processes in organizations in which there is high formalization. LO: 15.1: Identify seven elements of an organization's structure. Difficulty: Moderate Quest. Category: Concept
675 richard@qwconsultancy.com
43) Organizations do not always stay with the basis of departmentalization they first adopt, as is the case with Microsoft, explain. Answer: Interestingly, organizations do not always stay with the basis of departmentalization they first adopt. Microsoft for instance, used customer departmentalization for years, organizing around its customer bases: consumers, large corporations, software developers, and small businesses. However, in 2013, Microsoft announced restructuring to functional departmentalization, citing a need to foster continuing innovation. The new departments grouped jobs by traditional functions including engineering, marketing, business development, strategy and research, finance, HR, and legal. Microsoft continued to struggle with the reorganization, announcing further changes in its leadership personnel and team structure less than a year later. LO: 15.1: Identify seven elements of an organization's structure. Difficulty: Moderate Quest. Category: Concept 44) Elena is the senior manager of a scientific operations team at a well-known clinical research organization. The organization delivers more than 4,000 solutions across more than 18 therapeutic areas. Being a perfectionist, Elena has reached her present position through hard work and dedication. Given that she has also worked effectively with teams in the past, she has recently been assigned to lead a team working on a new project which is critical to the company. Elena refuses to allocate any tasks to her team and decides to put in extra hours and weekends to complete the groundwork herself. Based on the information presented here, which of the following would best explain this contradiction? A) Elena feels she will not be able to guide her team well on the project. B) Elena is not very sociable with her colleagues. C) Elena feels that by delegating work, she will complicate her working relationship with the team. D) Elena feels that this project is vital to the team's success. E) Elena is reluctant to delegate work as she lacks confidence in her team's abilities. Answer: E Explanation: Elena doesn't trust her team's abilities. Hence, she refuses to allocate any tasks to her team and decides to put in extra hours and weekends to complete the groundwork herself. The fact that Elena is not very sociable, her self-distrust on being able to guide her team, or her feeling she will complicate her relationship with the team doesn't explain why she refuses to let her team in on the project. If Elena felt this project was vital to her team's success, she would have coordinated with her team to achieve the expected favorable results. LO: 15.2: Identify the characteristics of the simple structure, the bureaucracy, and the matrix structure. AACSB: Reflective thinking Difficulty: Hard Employability Skills: Critical Thinking Quest. Category: Critical Thinking
676 richard@qwconsultancy.com
45) A low degree of departmentalization, wide spans of control, authority centralized in a single person, and little formalization are all characteristics of a ________. A) bureaucracy B) matrix organization C) simple structure D) networked organizational structure E) hierarchical organization Answer: C Explanation: The simple structure is not elaborate. It has a low degree of departmentalization, wide spans of control, authority centralized in a single person, and little formalization. It is a "flat" organization; it usually has only two or three vertical levels, a loose body of employees, and one individual in whom the decision-making authority is centralized. LO: 15.2: Identify the characteristics of the simple structure, the bureaucracy, and the matrix structure. Difficulty: Easy Quest. Category: Concept 46) The simple structure is most widely practiced in small businesses in which ________. A) the manager and owner are one and the same B) the number of employees is between 100 to 150 C) employees are highly skilled D) managers are hired directly by the company's owner E) training budgets are limited Answer: A Explanation: The simple structure is most widely practiced in small businesses in which the manager and owner are one and the same. LO: 15.2: Identify the characteristics of the simple structure, the bureaucracy, and the matrix structure. Difficulty: Moderate Quest. Category: Concept
677 richard@qwconsultancy.com
47) Rosemary owns a local clothing resale shop. Rosemary has five employees reporting to her; however, she makes all of the decisions regarding store displays and inventory. In addition, Rosemary is responsible for the store's finances. Which type of organizational structure does Rosemary have? A) Bureaucracy B) Virtual structure C) Matrix structure D) Boundaryless structure E) Simple structure Answer: E Explanation: A simple structure has a low degree of departmentalization, wide spans of control, authority centralized in a single person, and little formalization. It is a "flat" organization; it usually has only two or three vertical levels, a loose body of employees, and one individual in whom the decision-making authority is centralized. So Rosemary's store has a simple structure. LO: 15.2: Identify the characteristics of the simple structure, the bureaucracy, and the matrix structure. AACSB: Analytical thinking Difficulty: Moderate Employability Skills: Knowledge Application and Analysis Quest. Category: Application 48) Which of the following statements is true about the simple structure? A) It is very expensive to operate. B) It is characterized by a high degree of formalization. C) It becomes increasingly inadequate as an organization grows. D) It is most widely adopted in large businesses in which the manager and owner are different. E) It is characterized by a narrow span of control. Answer: C Explanation: A simple structure has a low degree of departmentalization, wide spans of control, authority centralized in a single person, and little formalization. It's fast, flexible, and inexpensive to operate, and accountability is clear. One major weakness is that it's difficult to maintain in anything other than small organizations. It becomes increasingly inadequate as an organization grows because its low formalization and high centralization tend to create information overload at the top. LO: 15.2: Identify the characteristics of the simple structure, the bureaucracy, and the matrix structure. Difficulty: Moderate Quest. Category: Concept
678 richard@qwconsultancy.com
49) Simple structure is characterized by ________. A) high centralization B) inflexibility C) narrow spans of control D) ambiguous accountability E) high degrees of formalization Answer: A Explanation: The simple structure is not elaborate. It has a low degree of departmentalization, wide spans of control, authority centralized in a single person, and little formalization. It is a "flat" organization; it usually has only two or three vertical levels, a loose body of employees, and one individual in whom the decision-making authority is centralized. It's fast, flexible, and inexpensive to operate, and accountability is clear. LO: 15.2: Identify the characteristics of the simple structure, the bureaucracy, and the matrix structure. Difficulty: Moderate Quest. Category: Concept 50) Bureaucracies are characterized by ________. A) efficiency B) standardization C) increased flexibility D) decentralization E) specialization Answer: B Explanation: Standardization is the key concept that underlies all bureaucracies. They all rely on standardized work processes for coordination and control. LO: 15.2: Identify the characteristics of the simple structure, the bureaucracy, and the matrix structure. Difficulty: Easy Quest. Category: Concept 51) Which of the following statements is true regarding a bureaucracy? A) Bureaucracies are characterized by a low degree of formalization. B) Bureaucracy is an ideal organizational design for those organizations that operate under highly uncertain conditions. C) Typically, bureaucracies have wider spans of control. D) Bureaucracies can get by with less talented middle- and lower-level managers. E) Bureaucracies encourage employee empowerment. Answer: D Explanation: Bureaucracies can get by with less talented—and, hence, less costly—middle- and lower-level managers. Rules and regulations substitute for managerial discretion. Standardized operations and high formalization allow decision making to be centralized. There is little need for innovative and experienced decision makers below the level of senior executives. LO: 15.2: Identify the characteristics of the simple structure, the bureaucracy, and the matrix structure. Difficulty: Moderate 679 richard@qwconsultancy.com
Quest. Category: Concept 52) The ________ combines two forms of departmentalization, functional and product. A) simple structure B) bureaucracy C) matrix structure D) virtual structure E) boundaryless structure Answer: C Explanation: The matrix structure combines two forms of departmentalization: functional and product. Thus, members in a matrix structure have a dual chain of command: to their functional department and to their product groups. LO: 15.2: Identify the characteristics of the simple structure, the bureaucracy, and the matrix structure. Difficulty: Easy Quest. Category: Concept You are interested in explaining the different types of organizational designs to students of basic management. These students need to know the advantages and disadvantages of each structure as well as the structure's characteristics. 53) You talk excitedly about the virtues and benefits of standardization. You are most likely to be promoting the ________. A) matrix structure B) virtual organization C) bureaucracy D) boundaryless structure E) simple structure Answer: C Explanation: Standardization is the key concept that underlies all bureaucracies. The bureaucracy is characterized by highly routine operating tasks achieved through specialization, very formalized rules and regulations, tasks grouped into functional departments, centralized authority, narrow spans of control, and decision making that follows the chain of command. LO: 15.2: Identify the characteristics of the simple structure, the bureaucracy, and the matrix structure. AACSB: Analytical thinking Difficulty: Moderate Employability Skills: Knowledge Application and Analysis Quest. Category: Application
680 richard@qwconsultancy.com
54) You explain to your students the structure of a local grocery store situated in your neighborhood. The store employs two full-time employees, a salesperson and a cashier. The owner of the store acts as its manager and makes all the decisions. Which of the following organizational structures is being described by you? A) Matrix structure B) Virtual structure C) Bureaucracy D) Boundaryless structure E) Simple structure Answer: E Explanation: A simple structure describes an organizational structure characterized by a low degree of departmentalization, wide spans of control, authority centralized in a single person, and little formalization. LO: 15.2: Identify the characteristics of the simple structure, the bureaucracy, and the matrix structure. AACSB: Analytical thinking Difficulty: Moderate Employability Skills: Knowledge Application and Analysis Quest. Category: Application 55) You describe to your students a new committee within the university that brings together specialists from all different departments to develop a new interdisciplinary program. The structure of the committee best meets the definition of the ________ structure. A) matrix B) simple C) boundaryless D) virtual E) bureaucratic Answer: A Explanation: The matrix structure describes an organizational structure that creates dual lines of authority and combines functional and product departmentalization. LO: 15.2: Identify the characteristics of the simple structure, the bureaucracy, and the matrix structure. AACSB: Analytical thinking Difficulty: Hard Employability Skills: Knowledge Application and Analysis Quest. Category: Application
681 richard@qwconsultancy.com
56) Which of the following statements is true regarding a matrix structure? A) It breaks the unity of command concept. B) It cannot achieve economies of scale. C) It reduces ambiguity about who reports to whom. D) It prevents power struggles. E) It avoids duplication of activities. Answer: A Explanation: The matrix structure breaks the unity of command concept and increases ambiguity about who reports to whom. The strength of the matrix is its ability to facilitate coordination when the organization has a number of complex and interdependent activities. LO: 15.2: Identify the characteristics of the simple structure, the bureaucracy, and the matrix structure. Difficulty: Moderate Quest. Category: Concept 57) Which one of the following problems is most likely to occur in a matrix structure? A) Decreased response to environmental change B) Increased social loafing C) Loss of economies of scale D) Increase in groupthink E) Unclear expectations Answer: E Explanation: The major disadvantages of the matrix lie in the confusion it creates its propensity to foster power struggles, and the stress it places on individuals. Without the unity-of-command concept, ambiguity about who reports to whom is significantly increased and often leads to conflict. LO: 15.2: Identify the characteristics of the simple structure, the bureaucracy, and the matrix structure. Difficulty: Moderate Quest. Category: Concept 58) The simple structure is characterized by a low degree of departmentalization and wide span of control. Answer: TRUE Explanation: The simple structure has a low degree of departmentalization, wide spans of control, authority centralized in a single person, and little formalization. LO: 15.2: Identify the characteristics of the simple structure, the bureaucracy, and the matrix structure. Difficulty: Easy Quest. Category: Concept
682 richard@qwconsultancy.com
59) The functional structure groups employees by their similar specialties, roles, or tasks. Answer: TRUE Explanation: The functional structure groups employees by their similar specialties, roles, or tasks. An organization organized into production, marketing, human resources, and accounting departments is an example. Many large organizations utilize this structure, although this is evolving to allow for quick changes in response to business opportunities. Still, there are advantages, including that the functional structure allows specialists to become experts more easily than if they worked in diversified units. LO: 15.2: Identify the characteristics of the simple structure, the bureaucracy, and the matrix structure. Difficulty: Easy Quest. Category: Concept 60) A disadvantage of the matrix structure is that it fosters power struggles. Answer: TRUE Explanation: Some disadvantages of the matrix structure lie in the confusion it creates, its tendency to foster power struggles, and the stress it places on individuals. LO: 15.2: Identify the characteristics of the simple structure, the bureaucracy, and the matrix structure. Difficulty: Moderate Quest. Category: Concept 61) Employees in a matrix structure have two supervisors: their functional department managers and their product managers. Answer: TRUE Explanation: Employees in the matrix have two supervisors: their functional department managers and their product managers. LO: 15.2: Identify the characteristics of the simple structure, the bureaucracy, and the matrix structure. Difficulty: Moderate Quest. Category: Concept 62) A bureaucracy is characterized by centralized authority and narrow spans of control. Answer: TRUE Explanation: A bureaucracy is characterized by highly routine operating tasks achieved through specialization, very formalized rules and regulations, tasks grouped into functional departments, centralized authority, narrow spans of control, and decision making that follows the chain of command. LO: 15.2: Identify the characteristics of the simple structure, the bureaucracy, and the matrix structure. Difficulty: Easy Quest. Category: Concept
683 richard@qwconsultancy.com
63) The divisional structure has the opposite benefits and disadvantages of the functional structure. Answer: TRUE Explanation: The divisional structure has the opposite benefits and disadvantages of the functional structure. It facilitates coordination in units to achieve on-time completion, budget targets, developing and bringing new products to market, and addressing the specific concerns of each unit. It provides clear responsibility for all activities related to a product, but with duplication of functions and costs. Sometimes this is helpful, say when the organization has a unit in Spain and another in China and a marketing strategy is needed for a new product. Marketing experts in both places can incorporate the appropriate cultural perspectives into their region's marketing campaign. LO: 15.2: Identify the characteristics of the simple structure, the bureaucracy, and the matrix structure. Difficulty: Moderate Quest. Category: Concept 64) The matrix structure can increase stress for employees. Answer: FALSE Explanation: The major disadvantages of the matrix lie in the confusion it creates, its propensity to foster power struggles, and the stress it places on individuals. LO: 15.2: Identify the characteristics of the simple structure, the bureaucracy, and the matrix structure. Difficulty: Moderate Quest. Category: Concept 65) Explain the simple structure and its advantages and disadvantages. Answer: A simple structure is not elaborate. It has a low degree of departmentalization, wide spans of control, authority centralized in a single person, and little formalization. It is a "flat" organization; it usually has only two or three vertical levels, a loose body of employees, and one individual in whom the decision-making authority is centralized. The strength of the simple structure lies in its simplicity. It's fast, flexible, and inexpensive to operate, and accountability is clear. One major weakness is that it's difficult to maintain in anything other than small organizations. It becomes increasingly inadequate as an organization grows because its low formalization and high centralization tend to create information overload at the top. As size increases, decision making typically becomes slower and can eventually come to a standstill as the single executive tries to continue making all the decisions. The simple structure's other weakness is that it's risky—everything depends on one person. One illness can literally destroy the organization's information and decision-making center. LO: 15.2: Identify the characteristics of the simple structure, the bureaucracy, and the matrix structure. Difficulty: Moderate Quest. Category: Concept
684 richard@qwconsultancy.com
66) What is a bureaucracy? What are its advantages and disadvantages? Answer: A bureaucracy is characterized by highly routine operating tasks achieved through specialization, very formalized rules and regulations, tasks grouped into functional departments, centralized authority, narrow spans of control, and decision making that follows the chain of command. Its primary strength is its ability to perform standardized activities in a highly efficient manner. Putting like specialties together in functional departments results in economies of scale and minimum duplication of personnel and equipment. In a bureaucracy, rules and regulations substitute for managerial discretion. Standardized operations and high formalization allow decision making to be centralized. There is little need for innovative and experienced decision makers below the level of senior executives. A drawback of bureaucracy is that if cases don't precisely fit the rules, there is no room for modification. The bureaucracy is efficient only as long as employees confront familiar problems with programmed decision rules. LO: 15.2: Identify the characteristics of the simple structure, the bureaucracy, and the matrix structure. Difficulty: Moderate Quest. Category: Concept 67) Define the matrix structure and explain its advantages and disadvantages. Answer: The matrix structure combines two forms of departmentalization: functional and product. The most obvious structural characteristic of the matrix is that it breaks the unity-ofcommand concept. Employees in the matrix have two bosses: their functional department managers and their product managers. The strength of the matrix is its ability to facilitate coordination when the organization has a number of complex and interdependent activities. Direct and frequent contacts between different specialties in the matrix can let information permeate the organization and more quickly reach the people who need it. The major disadvantages of the matrix lie in the confusion it creates, its propensity to foster power struggles, and the stress it places on individuals. Without the unity-of-command concept, ambiguity about who reports to whom is significantly increased and often leads to conflict. Reporting to more than one boss introduces role conflict, and unclear expectations introduce role ambiguity. LO: 15.2: Identify the characteristics of the simple structure, the bureaucracy, and the matrix structure. Difficulty: Moderate Quest. Category: Concept
685 richard@qwconsultancy.com
68) Compare and contrast the three common organizational designs: simple structure, bureaucracy, and matrix structure. Answer: a) The simple structure is said to be characterized most by what it is not rather than what it is. The simple structure is not elaborate. It has a low degree of departmentalization, wide spans of control, authority centralized in a single person, and little formalization. The simple structure is a "flat" organization; it usually has only two or three vertical levels, a loose body of employees, and one individual in whom the decision-making authority is centralized. b) Standardization is the key concept that underlies the bureaucracy. It is characterized by highly routine operating tasks achieved through specialization, very formalized rules and regulations, tasks that are grouped into functional departments, centralized authority, narrow spans of control, and decision making that follows the chain of command. c) The matrix combines two forms of departmentalization: functional and product. It breaks the unity-of-command concept. Employees in the matrix have two bosses—their functional department managers and their product managers. Therefore, the matrix has a dual chain of command. LO: 15.2: Identify the characteristics of the simple structure, the bureaucracy, and the matrix structure. AACSB: Analytical thinking Difficulty: Moderate Quest. Category: Synthesis 69) A small, core organization that outsources major business functions is a ________ structure. A) centralized B) virtual C) bureaucratic D) matrix E) formalized Answer: B Explanation: The virtual structure is also sometimes called the network organization. It is typically a small, core organization that outsources major business functions. LO: 15.3: Identify the characteristics of the virtual structure, the team structure, and the circular structure. Difficulty: Easy Quest. Category: Concept
686 richard@qwconsultancy.com
70) A ________ characterizes an organization in which executives are at the center, spreading their vision outward in rings grouped by function (managers, then specialists, then workers). A) circular structure B) matrix structure C) virtual organization D) bureaucracy E) functional structure Answer: A Explanation: An organization in which executives are at the center, spreading their vision outward in rings grouped by function (managers, then specialists, then workers) is known as a circular structure. LO: 15.3: Identify the characteristics of the virtual structure, the team structure, and the circular structure. Difficulty: Moderate Quest. Category: Concept 71) Another name for a virtual structure is the ________ structure. A) boundaryless B) electronic C) network D) pyramidal E) triangular Answer: C Explanation: A virtual structure is also known as a network structure. LO: 15.3: Identify the characteristics of the virtual structure, the team structure, and the circular structure. Difficulty: Moderate Quest. Category: Concept
687 richard@qwconsultancy.com
72) You are the CEO of Achilles Incorporation. You have decided to hire other organizations to perform many of the basic functions of your business. You have hired an accounting firm to keep your records, a recruiting firm to handle human resource functions, and a computer firm to handle all records. To keep costs down, you are looking for other areas in which to outsource operations. You have chosen to operate your business as a ________. A) matrix organization B) virtual organization C) highly centralized organization D) highly formalized organization E) bureaucracy Answer: B Explanation: A virtual organization is a small, core organization that outsources major business functions. LO: 15.3: Identify the characteristics of the virtual structure, the team structure, and the circular structure. AACSB: Analytical thinking Difficulty: Hard Employability Skills: Knowledge Application and Analysis Quest. Category: Application 73) The major advantage of a virtual structure is its ________. A) decentralized structure B) ability to enhance role clarity C) flexibility D) highly departmentalized structure E) ability to minimize organizational politics Answer: C Explanation: The major advantage of the virtual organization is its flexibility. In structural terms, the virtual organization is highly centralized, with little or no departmentalization. Virtual organizations are in a state of perpetual flux and reorganization, which means roles, goals, and responsibilities are unclear. This sets the stage for political behavior. LO: 15.3: Identify the characteristics of the virtual structure, the team structure, and the circular structure. Difficulty: Moderate Quest. Category: Concept
688 richard@qwconsultancy.com
74) Which of the following statements is true regarding a virtual structure? A) A virtual structure is sometimes called matrix organization. B) A virtual structure is highly decentralized. C) A virtual structure is characterized by a high degree of departmentalization. D) A virtual structure tends to have unclear roles, goals, and responsibilities. E) A virtual structure lacks flexibility. Answer: D Explanation: A virtual structure is sometimes called a network structure. In structural terms, the virtual structure is highly centralized, with little or no departmentalization. In a virtual structure, roles, goals, and responsibilities tend to be unclear. The major advantage of the virtual structure is its flexibility. LO: 15.3: Identify the characteristics of the virtual structure, the team structure, and the circular structure. Difficulty: Moderate Quest. Category: Concept 75) A drawback of the virtual structure is its ________. A) constant flux and reorganization B) increased technology costs and decentralization C) lack of flexibility and innovativeness D) lack of qualified employees and poor management E) poor communication and high degree of formalization Answer: A Explanation: Virtual structures' drawbacks have become increasingly clear as their popularity has grown. They are in a state of perpetual flux and reorganization, which means roles, goals, and responsibilities are unclear. This sets the stage for political behavior. LO: 15.3: Identify the characteristics of the virtual structure, the team structure, and the circular structure. Difficulty: Moderate Quest. Category: Concept
689 richard@qwconsultancy.com
76) Blastkrieg, a chain of audio equipment stores, uses computerized inventory control and customer self-service to eliminate the category of salesclerks from its force of employees. After diversifying into computer electronics, it now plans to employ the same concept in selling desktop PCs and laptops. Which of the following is an assumption made in Blast Krieg's strategy? A) All computer electronics stores use a similar strategy. B) Sales personnel are not required for selling computer electronics successfully. C) The same kind of computers will be used in inventory control for both audio equipment and electronics at Blastkrieg. D) A self-service strategy cannot be employed without computerized inventory control. E) Salesclerks are the only employees of Blastkrieg who could be assigned tasks related to inventory control. Answer: B Explanation: Blastkrieg assumes that sales personnel are not required to sell computer electronics and hence plans to employ the computerized inventory and self-service. The other options are irrelevant and out of scope. The fact that all computer electronics use a similar strategy has no impact on Blastkrieg's decision. If the self-service strategy cannot be employed without computerized inventory control, it is simply stating a fact not an assumption. The kind of computers used in inventory control for selling audio equipment or electronics is not relevant to the strategy used. LO: 15.3: Identify the characteristics of the virtual structure, the team structure, and the circular structure. AACSB: Analytical thinking Difficulty: Hard Employability Skills: Critical Thinking Quest. Category: Critical Thinking
690 richard@qwconsultancy.com
You have learned about the traditional and new design options for organizational structures. You have decided that one of the designs developed during the last decade or two is probably the most appropriate for your newly formed organization. 77) You have recently started your business with a small capital investment. Your business needs a very high degree of flexibility so that you can quickly respond to environmental changes, and you are also interested in minimizing costs. Which of the following types of organizations is best suitable for your business? A) A highly centralized organization B) A virtual structure C) A formalized organization D) A bureaucracy E) A hierarchical organization Answer: B Explanation: The major advantage of the virtual structure is its flexibility, which allows individuals with an innovative idea and little money to successfully compete against larger, more established organizations. LO: 15.3: Identify the characteristics of the virtual structure, the team structure, and the circular structure. AACSB: Analytical thinking Difficulty: Hard Employability Skills: Knowledge Application and Analysis Quest. Category: Application 78) A ________ structure is an organizational structure in which executives are at the center, spreading their vision outward in rings grouped by function (managers, then specialists, then workers). A) virtual B) networked C) circular D) team E) matrix Answer: C Explanation: A circular structure is an organizational structure in which executives are at the center, spreading their vision outward in rings grouped by function (managers, then specialists, then workers. LO: 15.3: Identify the characteristics of the virtual structure, the team structure, and the circular structure. Difficulty: Easy Quest. Category: Concept
691 richard@qwconsultancy.com
79) A structure that replaces departments with empowered teams, and that eliminates most horizontal boundaries and external boundaries between customers and suppliers is a ________ structure. A) circular B) matrix C) horizontal D) team E) networked Answer: D Explanation: A team structure refers to an organizational structure that replaces departments with empowered teams and that eliminates most horizontal boundaries and external barriers between customers and suppliers. LO: 15.3: Identify the characteristics of the virtual structure, the team structure, and the circular structure. Difficulty: Easy Quest. Category: Concept 80) In structural terms, a virtual structure is highly decentralized, with significant departmentalization. Answer: FALSE Explanation: Virtual structures are typically small, core organizations that outsource major business functions. In structural terms, the virtual organization is highly centralized, with little or no departmentalization. LO: 15.3: Identify the characteristics of the virtual structure, the team structure, and the circular structure. Difficulty: Moderate Quest. Category: Concept 81) A major advantage of the virtual structure is its flexibility. Answer: TRUE Explanation: A virtual organization is a small core organization that outsources major business functions. The major advantage of the virtual organization is its flexibility. LO: 15.3: Identify the characteristics of the virtual structure, the team structure, and the circular structure. Difficulty: Moderate Quest. Category: Concept
692 richard@qwconsultancy.com
82) Explain the essence of the virtual structure and its advantages and disadvantages. Answer: The virtual structure is sometimes called the network structure. Typically, a small, core organization outsources major business functions. In structural terms, the virtual organization is highly centralized, with little or no departmentalization. The core of the organization is a small group of executives whose job is to oversee directly any activities done in house and to coordinate relationships with the other organizations that manufacture, distribute, and perform other crucial functions for the virtual organization. The major advantage of the virtual structure is its flexibility, which allows individuals with an innovative idea and little money to successfully compete against larger, more established organizations. The structure also saves a great deal of money by eliminating permanent offices and hierarchical roles for outsourced functions. The drawbacks have become increasingly clear as popularity has grown. Virtual organizations are in a state of perpetual flux and reorganization, which means roles, goals, and responsibilities are unclear, setting the stage for increased political behavior. LO: 15.3: Identify the characteristics of the virtual structure, the team structure, and the circular structure. AACSB: Analytical thinking Difficulty: Moderate Quest. Category: Synthesis 83) A systematic effort to make an organization leaner by selling off business units, closing locations, or reducing staff is ________. A) removing boundaries B) departmentalizing C) decentralizing D) downsizing E) formalizing Answer: D Explanation: The goal of the new organizational forms is to improve agility by creating a lean, focused, and flexible organization. Companies may need to cut divisions that aren't adding value. Downsizing is a systematic effort to make an organization leaner by selling off business units, closing locations, or reducing staff. LO: 15.4: Describe the effects of downsizing on organizational structures and employees. Difficulty: Easy Quest. Category: Concept
693 richard@qwconsultancy.com
84) How can the negative impact of downsizing be minimized? Answer: Companies can reduce negative impacts by preparing for the post-downsizing environment in advance, thus alleviating some employee stress and strengthening support for the new strategic direction. Some of the ways in which the negative impact of downsizing can be minimized are: a) Investment: Companies that downsize to focus on core competencies are more effective when they invest in high-involvement work practices afterward. b) Communication: When employers make efforts to discuss downsizing with employees early, employees are less worried about the outcomes and feel the company is taking their perspective into account. c) Participation: Employees worry less if they can participate in the process in some way. In some companies, voluntary early retirement programs or severance packages can help achieve leanness without layoffs. d) Assistance: Providing severance, extended healthcare benefits, and job search assistance demonstrates a company does really care about its employees and honors their contributions. LO: 15.4: Describe the effects of downsizing on organizational structures and employees. Difficulty: Moderate Quest. Category: Concept 85) A(n) ________ model is generally synonymous with the bureaucracy in that it has highly standardized processes for work, high formalization, and more managerial hierarchy. A) mechanistic B) organic C) network D) boundaryless E) pyramidal Answer: A Explanation: The mechanistic model is generally synonymous with the bureaucracy in that it has highly standardized processes for work, high formalization, and more managerial hierarchy. It is characterized by extensive departmentalization, high formalization, a limited information network, and centralization. LO: 15.5: Contrast the reasons for using mechanistic versus organic structural models. Difficulty: Moderate Quest. Category: Concept
694 richard@qwconsultancy.com
86) A structure that is flat, uses cross-hierarchical and cross-functional teams, has low formalization, possesses a comprehensive information network, and relies on participative decision making is a ________ structure. A) mechanistic B) organic C) boundaryless D) network E) pyramidal Answer: B Explanation: The organic model is flat, has fewer formal procedures for making decisions, has multiple decision makers, and favors flexible practices. It is a structure that is flat, uses crosshierarchical and cross-functional teams, has low formalization, possesses a comprehensive information network, and relies on participative decision making. LO: 15.5: Contrast the reasons for using mechanistic versus organic structural models. Difficulty: Moderate Quest. Category: Concept 87) Decision making in an organic model of organization tends to be ________. A) highly controlled B) centralized C) participative D) highly formalized E) inflexible Answer: C Explanation: The organic model is flat, has fewer formal procedures for making decisions, has multiple decision makers, and favors flexible practices. LO: 15.5: Contrast the reasons for using mechanistic versus organic structural models. Difficulty: Moderate Quest. Category: Concept 88) The mechanistic model of organization is characterized by ________. A) low formalization B) extensive departmentalization C) decentralized management D) wide spans of control E) low specialization Answer: B Explanation: The mechanistic model of organization has high specialization, extensive departmentalization, narrow spans of control, centralized management, and high formalization. LO: 15.5: Contrast the reasons for using mechanistic versus organic structural models. Difficulty: Moderate Quest. Category: Concept
695 richard@qwconsultancy.com
89) Which type of strategy emphasizes the introduction of major new products and services? A) Innovation B) Cost minimization C) Imitation D) Organic E) Mechanistic Answer: A Explanation: An innovation strategy strives to achieve meaningful and unique innovations. Innovation strategy emphasizes the introduction of major new products and services. Organic and mechanistic are types of organizational designs. LO: 15.5: Contrast the reasons for using mechanistic versus organic structural models. Difficulty: Moderate Quest. Category: Concept 90) Organizations following a(n) ________ strategy try to both minimize risk and maximize opportunity for profit, moving into new products or new markets only after innovators have proven their viability. A) innovation B) cost-cutting C) imitation D) organic E) mechanistic Answer: C Explanation: Organizations following an imitation strategy try to both minimize risk and maximize opportunity for profit, introducing new products or entering new markets only after innovators have proven their viability. LO: 15.5: Contrast the reasons for using mechanistic versus organic structural models. Difficulty: Moderate Quest. Category: Concept 91) Which of the following is most likely to be a characteristic of an organization pursuing an innovation strategy? A) High formalization B) High specialization C) Decentralized control D) Rigid departmentalization E) Narrow span of control Answer: C Explanation: Organizations pursuing an innovation strategy tend to be organic with a loose structure, low specialization, low formalization, and decentralized control. LO: 15.5: Contrast the reasons for using mechanistic versus organic structural models. Difficulty: Moderate Quest. Category: Concept
696 richard@qwconsultancy.com
92) Which of the following is most likely to be a characteristic of an organization pursuing a cost-minimization strategy? A) Lack of tight control B) Wide span of control C) Low work specialization D) High centralization E) Low formalization Answer: D Explanation: An organization pursuing a cost-minimization strategy tightly controls costs, refrains from incurring unnecessary expenses, and cuts prices in selling a basic product. It is characterized by extensive work specialization, high formalization and high centralization. LO: 15.5: Contrast the reasons for using mechanistic versus organic structural models. Difficulty: Moderate Quest. Category: Concept 93) The term ________ is used to describe the way an organization transfers inputs into outputs. A) volatility B) formalization C) complexity D) technology E) environment Answer: D Explanation: Technology describes the way an organization transfers inputs into outputs. Every organization has at least one technology for converting financial, human, and physical resources into products or services. LO: 15.5: Contrast the reasons for using mechanistic versus organic structural models. Difficulty: Easy Quest. Category: Concept 94) The key dimensions to any organization's environment are ________. A) conformity, criticality, and diffusivity B) complexity, volatility, and capacity C) complexity, diffusivity, and criticality D) conformity, criticality, and capacity E) criticality, volatility, diffusivity Answer: B Explanation: Any organization's environment has three dimensions: capacity, volatility, and complexity. LO: 15.5: Contrast the reasons for using mechanistic versus organic structural models. Difficulty: Moderate Quest. Category: Concept
697 richard@qwconsultancy.com
95) The degree to which the environment can support growth refers to ________. A) capacity B) flexibility C) volatility D) sustainability E) complexity Answer: A Explanation: Any organization's environment has three dimensions: capacity, volatility, and complexity. Capacity refers to the degree to which the environment can support growth. Volatility describes the degree of instability in the environment. Complexity is the degree of heterogeneity and concentration among environmental elements. LO: 15.5: Contrast the reasons for using mechanistic versus organic structural models. Difficulty: Easy Quest. Category: Concept 96) Volatility refers to the degree of ________ within an environment. A) heterogeneity B) instability C) flexibility D) diffusivity E) conformity Answer: B Explanation: Any organization's environment has three dimensions: capacity, volatility, and complexity. Capacity refers to the degree to which the environment can support growth. Volatility describes the degree of instability in the environment. Complexity is the degree of heterogeneity and concentration among environmental elements. LO: 15.5: Contrast the reasons for using mechanistic versus organic structural models. Difficulty: Easy Quest. Category: Concept 97) The degree of heterogeneity and concentration among environmental elements is known as ________. A) density B) simplicity C) complexity D) intricacy E) permeability Answer: C Explanation: Any organization's environment has three dimensions: capacity, volatility, and complexity. Capacity refers to the degree to which the environment can support growth. Volatility describes the degree of instability in the environment. Complexity is the degree of heterogeneity and concentration among environmental elements. LO: 15.5: Contrast the reasons for using mechanistic versus organic structural models. Difficulty: Easy Quest. Category: Concept
698 richard@qwconsultancy.com
699 richard@qwconsultancy.com
98) Harald is opening a manufacturing plant with a mechanistic and centralized structure. Employees from which of the following countries will be least comfortable with this type of a structure? A) Greece B) Peru C) Canada D) France E) Chile Answer: C Explanation: Organizations that operate with people from high power-distance cultures, such as Greece, France, and most of Latin America, find that their employees are much more accepting of mechanistic structures than are employees from low power-distance countries. Canada is a low power-distance country. LO: 15.5: Contrast the reasons for using mechanistic versus organic structural models. AACSB: Diverse and multicultural work environments Difficulty: Hard Employability Skills: Knowledge Application and Analysis Quest. Category: Application 99) Institutions are cultural factors that act as guidelines for appropriate behavior for organizational structure. Answer: TRUE Explanation: Institutions are cultural factors that act as guidelines for appropriate behavior for organizational structure. Institutional theory describes some of the forces that lead many organizations to have similar structures and, focus on pressures that aren't necessarily adaptive. In fact, many institutional theorists try to highlight the ways corporate behaviors sometimes seem to be performance oriented but are actually guided by unquestioned social norms and conformity. LO: 15.5: Contrast the reasons for using mechanistic versus organic structural models. Difficulty: Moderate Quest. Category: Concept 100) The more scarce, dynamic, and complex the environment, the more organic a structure should be. Answer: TRUE Explanation: Organizations that operate in environments characterized as scarce, dynamic, and complex face the greatest degree of uncertainty because they have high unpredictability, little room for error, and a diverse set of elements in the environment to monitor constantly. The more scarce, dynamic, and complex the environment, the more organic a structure should be. LO: 15.5: Contrast the reasons for using mechanistic versus organic structural models. Difficulty: Easy Quest. Category: Concept
700 richard@qwconsultancy.com
101) What are the similarities and differences among the three primary organizational strategies: innovation, cost-minimization, and imitation? Answer: An innovation strategy strives to achieve meaningful and unique innovations. Innovative firms will use competitive pay and benefits to attract top candidates and motivate employees to take risks. Some degree of mechanistic structure can actually benefit innovation. Well-developed communication channels, policies for enhancing long-term commitment, and clear channels of authority all may make it easier for rapid changes to occur smoothly. An organization pursuing a cost-minimization strategy tightly controls costs, refrains from incurring unnecessary expenses, and cuts prices in selling a basic product. This describes the strategy pursued by Walmart and the makers of generic or store-label grocery products. Costminimizing organizations pursue fewer policies meant to develop commitment among their workforces. Organizations following an imitation strategy try to both minimize risk and maximize opportunity for profit, moving new products or entering new markets only after innovators have proven their viability. Mass-market fashion manufacturers that copy designer styles follow this strategy, as do firms such as Hewlett-Packard and Caterpillar. They follow smaller and more innovative competitors with superior products, but only after competitors have demonstrated the market is there. LO: 15.5: Contrast the reasons for using mechanistic versus organic structural models. AACSB: Analytical thinking Difficulty: Moderate Quest. Category: Synthesis 102) What are the three dimensions that characterize the environment of an organization? Answer: An organization's environment is composed of those institutions or forces that are outside the organization and potentially affect the organization's performance. These typically include suppliers, customers, competitors, government regulatory agencies, public pressure groups, and the like. Any organization's environment has three dimensions: capacity, volatility, and complexity. a) Capacity refers to the degree to which the environment can support growth. Rich and growing environments generate excess resources, which can buffer the organization in times of relative scarcity. b) Volatility describes the degree of instability in the environment. A dynamic environment with a high degree of unpredictable change makes it difficult for management to make accurate predictions. c) Complexity is the degree of heterogeneity and concentration among environmental elements. Simple environments—like in the tobacco industry—are homogeneous and concentrated. Environments characterized by heterogeneity and dispersion—like the broadband industry—are complex and diverse, with numerous competitors. LO: 15.5: Contrast the reasons for using mechanistic versus organic structural models. Difficulty: Moderate Quest. Category: Concept
701 richard@qwconsultancy.com
103) Compare and contrast the mechanistic model and the organic model of organizational structure. Answer: A mechanistic model is generally synonymous with the bureaucracy in that it has extensive departmentalization, high formalization, a limited information network (mostly downward communication), and little participation by low-level members in decision making. At the other extreme is an organic model. The more scarce, dynamic, and complex the environment, the more organic a structure should be. The more abundant, stable, and simple the environment, the more the mechanistic structure will be preferred. LO: 15.5: Contrast the reasons for using mechanistic versus organic structural models. Difficulty: Moderate Quest. Category: Concept 104) In general, less ________ organizations have a greater amount of ________. A) centralized; authority B) organized; autonomy C) authoritarian; control D) centralized; formality E) centralized; autonomy Answer: E Explanation: In general, less centralized organizations have a greater amount of autonomy. LO: 15.6: Analyze the behavioral implications of different organizational designs. Difficulty: Moderate Quest. Category: Concept 105) There is much research to suggest that people tend to select employers randomly. Answer: FALSE Explanation: People don't select employers randomly. They are attracted to, are selected by, and stay with organizations that suit their personal characteristics. LO: 15.6: Analyze the behavioral implications of different organizational designs. Difficulty: Moderate Quest. Category: Concept 106) Employees from high-power distance cultures are much more accepting of mechanistic structures than are employees from low power-distance countries. Answer: TRUE Explanation: Organizations that operate with people from high-power-distance cultures such as Greece, France, and most of Latin America, often find their employees are much more accepting of mechanistic structures than are employees from low-power-distance countries. So consider cultural differences along with individual differences when predicting how structure will affect employee performance and satisfaction. LO: 15.6: Analyze the behavioral implications of different organizational designs. AACSB: Diverse and multicultural work environments Difficulty: Moderate Quest. Category: Concept Organizational Behavior, 19e (Robbins/Judge) 702 richard@qwconsultancy.com
Chapter 16 Organizational Culture and Change 1) A(n) ________ culture is best aligned with customer-oriented outcomes. A) adhocracy B) team C) clan D) low outcome orientation E) innovation Answer: C Explanation: A clan or market culture is best aligned with customer-oriented outcomes. LO: 16.1: Describe the common characteristics of organizational culture. AACSB: Analytical thinking Difficulty: Moderate Quest. Category: Concept 2) The ________ category of the Organizational Culture Inventory groups cultures that value affiliation, encouragement, and achievement. A) supportiveness B) constructive C) passive-defensive D) aggressive-defensive E) team-orientation Answer: B Explanation: The Organizational Culture Inventory groups cultures into three categories: (1) constructive cultures that value affiliation, encouragement, and achievement; (2) passivedefensive cultures that avoid accountability, seek validation and approval from others, and are conventional; and (3) aggressive-defensive cultures that are competitive, perfectionist, and power-oriented. LO: 16.1: Describe the common characteristics of organizational culture. Difficulty: Moderate Quest. Category: Concept
703 richard@qwconsultancy.com
3) Which of the following categories in the Organizational Culture Inventory groups cultures that are competitive, perfectionist, and power-oriented? A) Accountability B) Outcome orientation C) Team orientation D) Aggressive-defensive E) Stability Answer: D Explanation: The Organizational Culture Inventory groups cultures into three categories: (1) constructive cultures that value affiliation, encouragement, and achievement; (2) passivedefensive cultures that avoid accountability, seek validation and approval from others, and are conventional; and (3) aggressive-defensive cultures that are competitive, perfectionist, and power-oriented. LO: 16.1: Describe the common characteristics of organizational culture. Difficulty: Moderate Quest. Category: Concept 4) Which of the following is not an outcome of a strong organizational culture? A) Cohesiveness B) Loyalty C) Organizational commitment D) Resistance E) Meaning Answer: D Explanation: A strong culture should more directly affect organizational outcomes because it demonstrates high agreement about what the organization represents. Such unanimity of purpose builds cohesiveness, loyalty, meaning, and organizational commitment. LO: 16.1: Describe the common characteristics of organizational culture. Difficulty: Moderate Quest. Category: Concept 5) A(n) ________ culture is best for innovation. A) assertiveness B) market C) adhocracy D) hierarchy E) clan Answer: E Explanation: A clan culture is best for innovation. The clan culture is based on human affiliation whereby employees value attachment, collaboration, trust, and support. LO: 16.1: Describe the common characteristics of organizational culture. Difficulty: Moderate Quest. Category: Concept
704 richard@qwconsultancy.com
6) Which of the following best describes how employees view the difference between organizational culture and job satisfaction? A) Job satisfaction depends upon the level of "power distance" in the country, but organizational culture does not. B) Organizational culture is static, whereas job satisfaction is dynamic. C) Job satisfaction is immeasurable, whereas organizational culture is measurable. D) Organizational culture is descriptive, whereas job satisfaction is evaluative. E) Job satisfaction depends on the structure of the organization, but organizational culture does not. Answer: D Explanation: The concept of organizational culture is concerned with how employees perceive the characteristics of an organization's culture, not whether they like them, and is therefore a descriptive term. On the other hand, the concept of job satisfaction seeks to measure how employees feel about the organization's expectations and reward practices and therefore is an evaluative term. LO: 16.1: Describe the common characteristics of organizational culture. Difficulty: Moderate Quest. Category: Concept 7) An aircraft manufacturer with a strong presence in the United States, is looking to expand its market overseas. The firm currently sells its aircraft to several airlines in the United Kingdom but now wants to establish manufacturing units there as well in order to acquire a bigger share in the European market. Hence, it plans to merge with QueenAir, a British aircraft manufacturer. Which of the following, if true, would weaken the company's decision to merge with QueenAir? A) Merging with QueenAir would increase its profits considerably. B) There is increasing economic uncertainty in its U.S. market. C) The preferences of airline customers in Europe and the U.S. are similar. D) There is a striking difference in the organizational cultures of the two firms. E) A competitor in the U.S. market recently went out of business. Answer: D Explanation: As merging with QueenAir would increase the firm's profits, it strengthens the firm's decision. So does the economic uncertainty in the U.S. market and the customers' preferences. The fact that a competitor in the U.S. market recently went out of business is irrelevant. As the two merging firms have different organizational cultures, it may lead to various problems in the future. Hence, it weakens the firm's decision. LO: 16.1: Describe the common characteristics of organizational culture. AACSB: Reflective thinking Difficulty: Hard Employability Skills: Critical Thinking Quest. Category: Critical Thinking
705 richard@qwconsultancy.com
8) Which of the following statements is true regarding an organization's culture? A) Organizational culture is evaluative rather than descriptive. B) Large organizations rarely have subcultures. C) A dominant culture expresses the core values shared by a majority of the organization's members. D) A strong culture reduces employee satisfaction and increases employee turnover. E) Subcultures and dominant cultures do not share any common values. Answer: C Explanation: Organizational culture is descriptive. Most large organizations have a dominant culture and numerous subcultures. A dominant culture expresses the core values shared by a majority of the organization's members. A strong culture should reduce employee turnover, because it demonstrates high agreement about what the organization represents. LO: 16.1: Describe the common characteristics of organizational culture. Difficulty: Moderate Quest. Category: Concept 9) Minicultures that tend to develop in large organizations to reflect common problems or experiences faced by the members in the same department or location are often called ________. A) micro-cultures B) subcultures C) divisional cultures D) microcosms E) countercultures Answer: B Explanation: Subcultures tend to develop in large organizations to reflect common problems, situations, or experiences faced by groups of members in the same department or location. The purchasing department can have a subculture that includes the core values of the dominant culture plus additional values unique to members of the purchasing department. LO: 16.1: Describe the common characteristics of organizational culture. Difficulty: Easy Quest. Category: Concept
706 richard@qwconsultancy.com
10) Jean works for Fahrenheit Publishing, which is a publisher of scientific journals. The company is dominated by low risk taking and high attention to detail. Jean's department is committed to high team orientation and provides many team-building activities in which Jean and other department members work together and socialize. Which of the following statements best describes Jean's department? A) Jean's department is an example of a primary microcosm. B) Jean's department's culture is stronger than the dominant culture in the organization. C) Jean's department has developed a subculture. D) Jean's department's culture is undefined. E) Jean's department's culture has low stability. Answer: C Explanation: Jean's department has developed a subculture that reflects the common situations or experiences faced by group members in the same department or location. It includes the core values of the dominant culture plus additional team values unique to members of the department. LO: 16.1: Describe the common characteristics of organizational culture. AACSB: Analytical thinking Difficulty: Hard Employability Skills: Knowledge Application and Analysis Quest. Category: Application 11) ________ are indicators of a strong organizational culture. A) High levels of dissension B) High rates of employee turnover C) Completely horizontal organizational charts D) Narrowly defined roles E) Widely shared values Answer: E Explanation: In a strong culture, the organization's core values are both intensely held and widely shared. The more members who accept the core values and the greater their commitment, the stronger the culture and the greater its influence on member behavior. LO: 16.1: Describe the common characteristics of organizational culture. Difficulty: Moderate Quest. Category: Concept 12) In the ________ employees value growth, variety, attention to detail, stimulation, and autonomy. A) market B) adhocracy C) clan D) bureaucracy E) hierarchy Answer: B Explanation: In the adhocracy employees value growth, variety, attention to detail, stimulation, and autonomy. LO: 16.1: Describe the common characteristics of organizational culture. Difficulty: Easy 707 richard@qwconsultancy.com
Quest. Category: Concept 13) Which of the following is most likely to result from a strong organizational culture? A) High organizational commitment B) Low employee satisfaction C) Low loyalty D) High absenteeism E) Low behavioral control resulting from the climate within the organization Answer: A Explanation: A strong culture should reduce employee turnover because it demonstrates high agreement about what the organization represents. Such unanimity of purpose builds cohesiveness, loyalty, and organizational commitment. LO: 16.1: Describe the common characteristics of organizational culture. Difficulty: Moderate Quest. Category: Concept 14) A strong culture should increase employee loyalty because it results in ________. A) a highly centralized organization B) narrow spans of control C) cohesiveness and organizational commitment D) a highly formalized organization E) an outcome-oriented organization Answer: C Explanation: A strong culture should increase employee loyalty because it demonstrates high agreement about what the organization represents. Such unanimity of purpose builds cohesiveness, loyalty, and organizational commitment. LO: 16.1: Describe the common characteristics of organizational culture. Difficulty: Moderate Quest. Category: Concept 15) A culture that expresses the core values that are shared by a majority of the organization's members is known as a(n) ________ culture. A) dominant B) primary C) fundamental D) unique E) essential Answer: A Explanation: A dominant culture is a culture that expresses the core values that are shared by a majority of the organization's members. LO: 16.1: Describe the common characteristics of organizational culture. Difficulty: Easy Quest. Category: Concept
708 richard@qwconsultancy.com
Rogue Vogue Corp. is an apparel company. To keep up with the latest changes in the fashion industry, the company has to come up with innovative designs and follow strict timelines. The culture of the company values integrity, high ethical standards, and risk taking. The members of the organization accept these cultural values. They know exactly what is expected of them, and these expectations go a long way in shaping their behavior. In addition to this, the culture of the marketing department is outcome oriented, and the finance department emphasizes attention to detail. 16) The organizational culture of Rogue Vogue Corp. is an example of a(n) ________. A) autocratic culture B) subculture C) lowly formalized culture D) reflective culture E) dominant culture Answer: E Explanation: A dominant culture expresses the core values shared by a majority of the organization's members. LO: 16.1: Describe the common characteristics of organizational culture. AACSB: Analytical thinking Difficulty: Hard Employability Skills: Knowledge Application and Analysis Quest. Category: Application 17) Integrity and high ethical standards are called the ________ of the culture at the Rogue Vogue Corp. A) foundational values B) institutional traits C) core values D) significant traits E) unique values Answer: C Explanation: The dominant culture includes the core values, the primary or dominant values that are accepted throughout the organization. In this example, integrity and high ethical standards are the core values of the organizational culture. LO: 16.1: Describe the common characteristics of organizational culture. AACSB: Analytical thinking Difficulty: Hard Employability Skills: Knowledge Application and Analysis Quest. Category: Application
709 richard@qwconsultancy.com
18) In addition to the organizational culture at Rogue Vogue Corp., the finance department emphasizes attention to detail. In this example, attention to detail is a part of the ________ of the organization. A) core values B) significant traits C) dominant culture D) foundational values E) subculture Answer: E Explanation: Subcultures tend to develop in large organizations to reflect common problems, situations, or experiences faced by groups of members in the same department or location. Subculture includes the core values of the dominant culture plus additional values unique to members of the respective department. LO: 16.1: Describe the common characteristics of organizational culture. AACSB: Analytical thinking Difficulty: Hard Employability Skills: Knowledge Application and Analysis Quest. Category: Application 19) Based on the information in the scenario, we can say that Rogue Vogue Corp. ________. A) is a highly decentralized organization B) is a virtual organization C) has a strong culture D) is a boundary-laden organization E) is a matrix organization Answer: C Explanation: Based on the information in the scenario, we can say that Rogue Vogue Corp. has a strong culture. In a strong culture, the organization's core values are both intensely held and widely shared. The more members who accept the core values and the greater their commitment, the stronger the culture and the greater its influence on member behavior. If an organization has a strong culture, then the members of the organization know exactly what is expected of them, and these expectations go a long way in shaping their behavior. LO: 16.1: Describe the common characteristics of organizational culture. AACSB: Analytical thinking Difficulty: Hard Employability Skills: Knowledge Application and Analysis Quest. Category: Application 20) A hierarchical culture is best for profitability and revenue growth. Answer: TRUE Explanation: A hierarchical culture is based on stability. Employees value communication, formalization, and routine. A hierarchical culture is best for profitability and revenue growth. LO: 16.1: Describe the common characteristics of organizational culture. Difficulty: Moderate Quest. Category: Concept
710 richard@qwconsultancy.com
711 richard@qwconsultancy.com
21) Subcultures have been detected at the country or nation level, forming because of immigration, assimilation of new immigrants, or class differences among the people. Answer: TRUE Explanation: Subcultures have been detected at the country or nation level, forming because of immigration, assimilation of new immigrants, or class differences among the people. LO: 16.1: Describe the common characteristics of organizational culture. AACSB: Diverse and multicultural work environments Difficulty: Moderate Quest. Category: Concept 22) Organizational culture represents a perception that the organization's members share. Answer: TRUE Explanation: Organizational culture represents a perception that the organization's members share. LO: 16.1: Describe the common characteristics of organizational culture. Difficulty: Moderate Quest. Category: Concept 23) Subcultures act to undermine the dominant culture. Answer: FALSE Explanation: Subcultures tend to develop in large organizations to reflect common problems, situations, or experiences faced by groups of members in the same department or location. Subcultures include the core values of the dominant culture plus additional values unique to the respective departments. LO: 16.1: Describe the common characteristics of organizational culture. Difficulty: Moderate Quest. Category: Concept 24) A dominant culture expresses the core values a majority of members share and that give the organization its distinct personality. Answer: TRUE Explanation: A dominant culture expresses the core values a majority of members share and that give the organization its distinct personality. LO: 16.1: Describe the common characteristics of organizational culture. Difficulty: Easy Quest. Category: Concept 25) The "shared meaning" aspect of culture makes it a potent device for guiding and shaping behavior. Answer: TRUE Explanation: The "shared meaning" aspect of culture makes it a potent device for guiding and shaping behavior. LO: 16.1: Describe the common characteristics of organizational culture. Difficulty: Easy Quest. Category: Concept
712 richard@qwconsultancy.com
713 richard@qwconsultancy.com
26) The market is a culture based on achievement. Answer: TRUE Explanation: The market is a culture based on achievement. LO: 16.1: Describe the common characteristics of organizational culture. Difficulty: Easy Quest. Category: Concept 27) How does the Organizational Culture Profile (OCP) view organizational culture? Answer: The OCP draws upon a novel survey method in which employees sort a set of values based on how closely they represent their organization. The OCP suggests an organizational culture can be described by eight dimensions: (1) innovation, (2) attention to detail, (3) decisiveness, (4) team-orientation, (5) outcome-orientation, (6) aggressiveness, (7) supportiveness, and (8) rewards-emphasis. LO: 16.1: Describe the common characteristics of organizational culture. Difficulty: Moderate Quest. Category: Concept 28) What is the difference between dominant culture and subculture? Answer: Organizational culture represents a common perception the organization's members hold. A dominant culture expresses the core values shared by a majority of the organization's members. Subcultures tend to develop in large organizations to reflect common problems, situations, or experiences faced by groups of members in the same department or location. Subcultures include the core values of the dominant culture plus additional values unique to members of the respective department. If organizations were composed only of numerous subcultures, organizational culture as an independent variable would be significantly less powerful. It is the "shared meaning" aspect of culture that makes it such a potent device for guiding and shaping behavior. LO: 16.1: Describe the common characteristics of organizational culture. AACSB: Analytical thinking Difficulty: Moderate Quest. Category: Synthesis 29) What are the characteristics of a strong culture? Answer: A strong culture should more directly affect organizational outcomes because it demonstrates high agreement about what the organization represents. Such unanimity of purpose builds cohesiveness, loyalty, meaning, and organizational commitment. For example, with high cultural consensus and intensity surrounding the adaptability dimension of culture, organizations in one study experienced gains in net income, revenue, and operating cash flow. A study of nearly 90,000 employees from 137 organizations found that culture strength or consistency was related to numerous financial outcomes when there was a strong sense of mission and high employee involvement. LO: 16.1: Describe the common characteristics of organizational culture. Difficulty: Moderate Quest. Category: Concept
714 richard@qwconsultancy.com
30) Compare and contrast the clan, the adhocracy, the market, and the hierarchy. Answer: A common culture framework groups organizations into one of four types, each with has its own assumptions, beliefs, values, artifacts, and even criteria for effectiveness: 1. "The Clan." A culture based on human affiliation. Employees value attachment, collaboration, trust, and support. 2. "The Adhocracy." A culture based on change. Employees value growth, variety, attention to detail, stimulation, and autonomy. 3. "The Market." A culture based on achievement. Employees value communication, competence, and competition. 4. "The Hierarchy." A culture based on stability. Employees value communication, formalization, and routine. LO: 16.1: Describe the common characteristics of organizational culture. AACSB: Analytical thinking Difficulty: Moderate Quest. Category: Synthesis 31) Narratives about the organization's founders, rule breaking, and reactions to past mistakes are called ________. A) stories B) material symbols C) rituals D) organizational charts E) corporate chants Answer: A Explanation: Stories circulate through organizations. They typically contain narratives about the organization's founders, rule breaking, rags-to-riches successes, reductions in the workforce, relocation of employees, reactions to past mistakes, and organizational coping. LO: 16.2: Show how culture is transmitted to employees. Difficulty: Easy Quest. Category: Concept
715 richard@qwconsultancy.com
32) Bruce is new at Wayne Corp., but after a week he already knows that the founder of the corporation started the business in his garage with only $4,000 and one client. This information was most likely transmitted to Bruce by way of ________. A) stories B) material symbols C) rituals D) organizational charts E) corporate chants Answer: A Explanation: Bruce most likely knows this legend through the passing of stories. Stories circulate through organizations. They typically contain a narrative of events about the organization's founders, rule breaking, rags-to-riches successes, reductions in the workforce, relocation of employees, and organizational coping. LO: 16.2: Show how culture is transmitted to employees. AACSB: Analytical thinking Difficulty: Moderate Employability Skills: Knowledge Application and Analysis Quest. Category: Application 33) Nike is using ________ to perpetuate its organizational culture when employees reminisce about the days when Oregon track star Steve Prefontaine worked to make running a professional sport. A) rituals B) material symbols C) stories D) company policies E) business strategies Answer: C Explanation: Today, a number of senior Nike executives spend much of their time serving as corporate storytellers. When new hires hear tales of Oregon running star Steve Prefontaine's battles to make running a professional sport and attain better performance equipment, they learn of Nike's commitment to helping athletes. Stories such as these circulate through many organizations, anchoring the present in the past and legitimizing current practices. LO: 16.2: Show how culture is transmitted to employees. Difficulty: Moderate Employability Skills: Knowledge Application and Analysis Quest. Category: Concept
716 richard@qwconsultancy.com
34) ________ refer to repetitive sequences of activities that express and reinforce the key values of the organization, which goals are most important, which people are important, and which are expendable. A) Symbols B) Rituals C) Stories D) Communication strategies E) Spans of control Answer: B Explanation: Repetitive sequences of activities that express and reinforce the key values of the organization, which goals are most important, which people are important, and which are expendable are called rituals. LO: 16.2: Show how culture is transmitted to employees. Difficulty: Easy Quest. Category: Concept 35) Which of the following can act as a common denominator to unite members of a given culture or subculture? A) Mechanistic organizational structure B) 360-degree appraisals C) Jargons and acronyms that are specific to the organization D) Bureaucratic organizational structure E) Narrow span of control Answer: C Explanation: Many organizations and subunits within them use language to help members identify with the culture, attest to their acceptance of it, and help preserve it. New employees may at first be overwhelmed by acronyms and jargon, that, once assimilated, act as a common denominator to unite members of a given culture or subculture. LO: 16.2: Show how culture is transmitted to employees. Difficulty: Moderate Quest. Category: Concept 36) Alcoa headquarters has few individual offices, even for senior executives. The space is essentially made up of cubicles, common areas, and meeting rooms. This informality conveys to employees that Alcoa values ________. A) autocracy B) competitiveness C) rigidity D) equality E) formalization Answer: D Explanation: The informality at the Alcoa headquarters conveys to employees that Alcoa values openness, equality, creativity, and flexibility. LO: 16.2: Show how culture is transmitted to employees. Difficulty: Moderate Quest. Category: Concept 717 richard@qwconsultancy.com
Ask Socrates Inc. is a computer software company that employs highly intelligent, but somewhat unusual people. Every Friday, free lollipops, toys, or other treats are given out to encourage employees to remember how creative they were when they were children. All the new members of the organization are told about the founders who were three young people who "got lucky" and sold a video game that they invented. The employees are allowed to dress informally and can set their own working hours. 37) Which of the following is most likely to be used by Ask Socrates Inc. to inform its new employees about its founders? A) Stories B) Material symbols C) Organizational charts D) Rituals E) Corporate chants Answer: A Explanation: Stories circulate through organizations. They typically contain a narrative of events about the organization's founders, rule breaking, rags-to-riches successes, reductions in the workforce, relocation of employees, reactions to past mistakes, and organizational coping. These stories anchor the present in the past and explain and legitimize current practices. LO: 16.2: Show how culture is transmitted to employees. AACSB: Analytical thinking Difficulty: Moderate Employability Skills: Knowledge Application and Analysis Quest. Category: Application 38) The regular distribution of lollipops, toys, or treats every Friday is an example of a ________ that helps reinforce Ask Socrates' culture. A) dominant mechanism B) ritual C) primary process D) protective mechanism E) fundamental procedure Answer: B Explanation: Rituals are repetitive sequences of activities that express and reinforce the key values of the organization, what goals are most important, which people are important, and which people are expendable. LO: 16.2: Show how culture is transmitted to employees. AACSB: Analytical thinking Difficulty: Moderate Employability Skills: Knowledge Application and Analysis Quest. Category: Application
718 richard@qwconsultancy.com
39) Ask Socrates Inc. employees are allowed to dress informally. This is an example of a ________ through which organizational culture is transmitted. A) primary procedure B) ritual C) material symbol D) symbolic act E) fundamental mechanism Answer: C Explanation: Material symbols convey to employees who is important, the degree of egalitarianism top management desires, and the kinds of behavior that are appropriate. A few examples of material symbols include the size of offices, the elegance of furnishings, executive perks, and attire. LO: 16.2: Show how culture is transmitted to employees. AACSB: Analytical thinking Difficulty: Moderate Employability Skills: Knowledge Application and Analysis Quest. Category: Application 40) How can organizational culture be transmitted to employees? Answer: Culture is transmitted to employees in a number of forms, the most potent being stories, rituals, material symbols, and language. a) Stories: Stories contain a narrative of events about the organization's founders, rule breaking, rags-to-riches successes, reductions in the workforce, relocation of employees, reactions to past mistakes, and organizational coping. These stories anchor the present in the past and provide explanations and legitimacy for current practices. b) Rituals: Rituals are repetitive sequences of activities that express and reinforce the key values of the organization, what goals are most important, which people are important, and which are expendable. c) Material symbols: The layout of corporate headquarters, the types of automobiles top executives are given, and the presence or absence of corporate aircraft are a few examples of material symbols. These material symbols convey to employees who is important, the degree of egalitarianism desired by top management, and the kinds of behavior that are appropriate. d) Language: Many organizations and units within organizations use language as a way to identify members of a culture or subculture. By learning this language, members attest to their acceptance of the culture and, in doing so, help to preserve it. Organizations, over time, often develop unique terms to describe equipment, offices, key personnel, suppliers, customers, or products that relate to its business. New employees are frequently overwhelmed with acronyms and jargon, that, once assimilated, act as a common denominator that unites members of a given culture or subculture. LO: 16.2: Show how culture is transmitted to employees. Difficulty: Moderate Quest. Category: Concept
719 richard@qwconsultancy.com
41) Using an example, explain how a company can use material symbols to transmit culture. Answer: One example of the intentional use of material symbols is Texas electric company Dynegy. Dynegy's headquarters doesn't look like a typical operation. There are few individual offices, even for senior executives. The space is essentially made up of cubicles, common areas, and meeting rooms. This informality conveys to employees that Dynegy values openness, equality, creativity, and flexibility. While some organizations provide their top executives with chauffeur-driven limousines and a corporate jet, other CEOs drive the company car themselves and travel in airlines' economy section. At some firms, like Chicago shirtmaker Threadless, an "anything goes" atmosphere helps emphasize a creative culture. At Threadless, meetings are held in an Airstream camper parked inside the company's converted FedEx warehouse, while employees in shorts and flip-flops work in bullpens featuring disco balls and garish decorations chosen by each team. LO: 16.2: Show how culture is transmitted to employees. AACSB: Analytical thinking Difficulty: Moderate Quest. Category: Synthesis 42) The ultimate source of an organization's culture is ________. A) its top management B) its environment C) the country in which the organization operates D) its founders E) the sociocultural backgrounds of its employees Answer: D Explanation: An organization's customs, traditions, and general way of doing things are largely due to what it has done before and how successful it was in doing it. Because of this, the ultimate source of an organization's culture tends to be its founders. LO: 16.3: Identify the factors that create and sustain an organization's culture. Difficulty: Moderate Quest. Category: Concept 43) The ________ process helps candidates learn about the organization, and if employees perceive a conflict between their values and those of the organization, they can remove themselves from the applicant pool. A) orientation B) training C) performance evaluation D) selection E) institutionalization Answer: D Explanation: Selection is a two-way street, allowing employer or applicant to avoid a mismatch and sustaining an organization's culture by selecting out those who might attack or undermine its core values. LO: 16.3: Identify the factors that create and sustain an organization's culture. Difficulty: Moderate Quest. Category: Concept 720 richard@qwconsultancy.com
44) The selection process helps sustain the organization's culture by ________. A) establishing and enforcing norms B) hiring candidates who fit well within the organization C) socializing the new employees D) developing performance evaluation criteria E) rewarding conformity Answer: B Explanation: The selection process helps sustain the organization's culture by hiring candidates who fit well within the organization. Selection also provides information to applicants. Those who perceive a conflict between their values and those of the organization can remove themselves from the applicant pool. LO: 16.3: Identify the factors that create and sustain an organization's culture. Difficulty: Moderate Quest. Category: Concept 45) Top management has a major impact on the organization's culture by ________. A) establishing norms that filter down through the organization B) ensuring a proper match of personal and organizational values C) socializing new applicants in the pre-hiring phase D) providing a framework for metamorphosis of new hires E) properly rewarding employees' initiatives Answer: A Explanation: The actions of top management also have a major impact on the organization's culture. Through words and behavior, senior executives establish norms that filter through the organization about, for instance, whether risk taking is desirable, how much freedom managers should give employees, what is appropriate dress, and what actions pay off in terms of pay raises, promotions, and other rewards. LO: 16.3: Identify the factors that create and sustain an organization's culture. Difficulty: Moderate Quest. Category: Concept 46) The process that helps new employees adapt to the prevailing organizational culture is known as ________. A) satisficing B) reciprocal interdependence C) socialization D) formalization E) social loafing Answer: C Explanation: No matter how good a job the organization does in recruiting and selection, new employees are not fully indoctrinated in the organization's culture and can disrupt beliefs and customs already in place. The process that helps new employees adapt to the prevailing culture is socialization. LO: 16.3: Identify the factors that create and sustain an organization's culture. Difficulty: Moderate 721 richard@qwconsultancy.com
Quest. Category: Concept 47) Which of the following is one of the three stages in the process of socialization? A) Metamorphosis B) Pre-encounter C) Evaluation D) Post-arrival E) Post-encounter Answer: A Explanation: We can think of socialization as a process with three stages: prearrival, encounter, and metamorphosis. This process has an impact on the new employee's work productivity, commitment to the organization's objectives, and eventual decision to stay with the organization. LO: 16.3: Identify the factors that create and sustain an organization's culture. Difficulty: Easy Quest. Category: Concept 48) Identify the correct order of stages in the socialization process. A) Encounter, metamorphosis, post-encounter B) Prearrival, arrival, evaluation C) Prearrival, arrival, post-arrival D) Prearrival, encounter, metamorphosis E) Metamorphosis, pre-encounter, arrival Answer: D Explanation: We can think of socialization as a process with three stages: prearrival, encounter, and metamorphosis. LO: 16.3: Identify the factors that create and sustain an organization's culture. Difficulty: Easy Quest. Category: Concept 49) Which stage of the socialization process explicitly recognizes that each individual the organization comes across during the selection process has a set of values, attitudes, and expectations about both the work to be done and the organization? A) Post-encounter B) Prearrival C) Metamorphosis D) Pre-encounter E) Post-arrival Answer: B Explanation: The prearrival stage explicitly recognizes that each individual arrives with a set of values, attitudes, and expectations about both the work to be done and the organization. LO: 16.3: Identify the factors that create and sustain an organization's culture. Difficulty: Easy Quest. Category: Concept
722 richard@qwconsultancy.com
50) A new employee compares his or her expectations with the realities in the organization during the ________ stage of the socialization process. A) prearrival B) encounter C) metamorphosis D) post-arrival E) post-encounter Answer: B Explanation: On entry into the organization, the new member enters the encounter stage and confronts the possibility that expectations, about the job, co-workers, the boss, and the organization in general, may differ from reality. LO: 16.3: Identify the factors that create and sustain an organization's culture. Difficulty: Easy Quest. Category: Concept 51) If there is a basic conflict between the individual's expectations and the reality of working in an organization, the employee is most likely to be disillusioned and quit during the ________ stage of socialization. A) prearrival B) post-arrival C) encounter D) metamorphosis E) post-encounter Answer: C Explanation: On entry into the organization, the new member enters the encounter stage and confronts the possibility that expectations about the job, co-workers, the boss, and the organization in general may differ from reality. If there occurs a mismatch between the expectations of the new member and the reality, then the new member may become disillusioned enough with the reality to resign. LO: 16.3: Identify the factors that create and sustain an organization's culture. Difficulty: Moderate Quest. Category: Concept
723 richard@qwconsultancy.com
52) Higgins has recently joined a new law firm expecting to participate in exciting environmental law cases and cutting-edge research. After one month at the firm, he still hasn't been assigned a case and spends most of his time filing standardized appeals for title disputes with insurance companies. In which stage of the socialization process is Higgins? A) Prearrival B) Encounter C) Metamorphosis D) Post-encounter E) Post-arrival Answer: B Explanation: On entry into the organization, the new member enters the encounter stage and confronts the possibility that expectations about the job, co-workers, the boss, and the organization in general may differ from reality. Higgins is in the encounter stage. LO: 16.3: Identify the factors that create and sustain an organization's culture. AACSB: Analytical thinking Difficulty: Hard Employability Skills: Knowledge Application and Analysis Quest. Category: Application 53) During the socialization process, to work out any problems discovered during the encounter stage, the new member changes or goes through the ________ stage. A) post-arrival B) metamorphosis C) post-encounter D) analysis E) evaluation Answer: B Explanation: During the socialization process, to work out any problems discovered during the encounter stage, the new member changes or goes through the metamorphosis stage. LO: 16.3: Identify the factors that create and sustain an organization's culture. Difficulty: Moderate Quest. Category: Concept 54) Which of the following statements is true regarding random socialization? A) Random socialization tries to strip away certain characteristics of the recruit. B) Random socialization is carried out per a fixed time schedule. C) In random socialization, new employees are left on their own to figure things out. D) Random socialization involves socializing the newcomers in groups. E) Specific orientation and classroom training programs are examples of random socialization. Answer: C Explanation: In random socialization, role models are deliberately withheld. New employees are left on their own to figure things out. LO: 16.3: Identify the factors that create and sustain an organization's culture. Difficulty: Moderate Quest. Category: Concept
724 richard@qwconsultancy.com
725 richard@qwconsultancy.com
55) Which of the following types of socialization involves putting the new employee directly into the job, with little or no special attention? A) Investiture socialization B) Serial socialization C) Collective socialization D) Fixed socialization E) Informal socialization Answer: E Explanation: Informal socialization involves putting the new employee directly into the job, with little or no special attention. LO: 16.3: Identify the factors that create and sustain an organization's culture. Difficulty: Moderate Quest. Category: Concept 56) ________ socialization assumes that the newcomer's qualities and qualifications are the necessary ingredients for job success, so these qualities and qualifications are confirmed and supported. A) Variable B) Collective C) Serial D) Investiture E) Formal Answer: D Explanation: Investiture socialization assumes that the newcomer's qualities and qualifications are the necessary ingredients for job success, so these qualities and qualifications are confirmed and supported. LO: 16.3: Identify the factors that create and sustain an organization's culture. Difficulty: Moderate Quest. Category: Concept 57) Which of the following statements is true about formal socialization? A) Formal socialization involves socializing the new members individually. B) Specific orientation and training programs are examples of formal socialization. C) Apprenticeship and mentoring programs are examples of formal socialization. D) Formal socialization lacks a fixed time schedule. E) In formal socialization, new employees are left on their own to figure things out. Answer: B Explanation: The more a new employee is segregated from the ongoing work setting and differentiated in some way to make explicit his or her newcomer's role, the more formal socialization is. Specific orientation and training programs are examples of formal socialization. LO: 16.3: Identify the factors that create and sustain an organization's culture. Difficulty: Moderate Quest. Category: Concept
726 richard@qwconsultancy.com
58) ________ socialization tries to strip away certain characteristics of the recruit. A) Random B) Fixed C) Collective D) Divestiture E) Formal Answer: D Explanation: Divestiture socialization tries to strip away certain characteristics of the recruit. Fraternity and sorority pledges go through divestiture socialization to shape them into the proper role. LO: 16.3: Identify the factors that create and sustain an organization's culture. Difficulty: Moderate Quest. Category: Concept 59) Apprenticeship is an example of ________ socialization. A) divestiture B) variable C) serial D) informal E) investiture Answer: C Explanation: Serial socialization is characterized by the use of role models who train and encourage the newcomer. Apprenticeship and mentoring programs are examples. LO: 16.3: Identify the factors that create and sustain an organization's culture. Difficulty: Moderate Quest. Category: Concept 60) By the end of the ________ stage of socialization, the new members internalize and accept the norms of the organization and their work group, are confident in their competence, and feel trusted and valued by their peers. A) encounter B) post-arrival C) establishment D) metamorphosis E) adaptation Answer: D Explanation: To work out any problems discovered during the encounter stage, the new member changes, or goes through the metamorphosis stage. It is the last stage in the three-stage socialization process. At the end of this stage, the new members internalize and accept the norms of the organization and their work group, are confident in their competence, and feel trusted and valued by their peers. LO: 16.3: Identify the factors that create and sustain an organization's culture. Difficulty: Moderate Quest. Category: Concept
727 richard@qwconsultancy.com
You are new to an organization and do not really know what to expect about the socialization process. You recently received your MBA and have an undergraduate degree in computer science. Your new firm is a software development company with an emphasis in the healthcare industry. Your hiring process included campus interviews, a daylong trip for an interview at the company, an offer phone call and letter, and some promotional material sent via the mail. When you arrive for your first day at work, you spend half a day in an orientation session that is conducted by the human resources department, where you complete paperwork and receive a company handbook. Then you spend the rest of the day with your supervisor, who gives you a tour, introduces you to your co-workers, and explains your first project. After that, you begin working and getting to know the others in the company. You find that in most respects, your experience fits your expectations, but in some ways, you are surprised by realities that you hadn't expected. None of these surprises is too difficult to accept, so you eventually begin to feel at home and happy with your new job. 61) The information that you receive during the interviewing and hiring process is a part of the ________ stage of employee socialization. A) pre-encounter B) prearrival C) encounter D) metamorphosis E) post-encounter Answer: B Explanation: The prearrival stage explicitly recognizes that each individual arrives with a set of values, attitudes, and expectations about both the work to be done and the organization. One way to capitalize on the importance of pre-hire characteristics in socialization is to use the selection process to inform prospective employees about the organization as a whole. LO: 16.3: Identify the factors that create and sustain an organization's culture. AACSB: Analytical thinking Difficulty: Moderate Employability Skills: Knowledge Application and Analysis Quest. Category: Application
728 richard@qwconsultancy.com
62) Your first day at work is a part of the ________ stage of socialization. A) orientation B) prearrival C) encounter D) metamorphosis E) post-encounter Answer: C Explanation: On entry into the organization, the new member enters the encounter stage and confronts the possibility that expectations about the job, co-workers, the boss, and the organization in general may differ from reality. If expectations were fairly accurate, the encounter stage merely cements earlier perceptions. LO: 16.3: Identify the factors that create and sustain an organization's culture. AACSB: Analytical thinking Difficulty: Moderate Employability Skills: Knowledge Application and Analysis Quest. Category: Application 63) When you begin to notice things that are not as you expected, you are in the ________ stage of socialization. A) orientation B) prearrival C) encounter D) metamorphosis E) post-arrival Answer: C Explanation: On entry into the organization, the new member enters the encounter stage and confronts the possibility that expectations, about the job, co-workers, the boss, and the organization in general may differ from reality. LO: 16.3: Identify the factors that create and sustain an organization's culture. AACSB: Analytical thinking Difficulty: Hard Employability Skills: Knowledge Application and Analysis Quest. Category: Application
729 richard@qwconsultancy.com
64) Which of the following steps could your supervisor take to best help you develop a commitment to your new company? A) Encourage you to work independently at first to learn the ropes B) Explain the rules and policies of the organization to you C) Discourage you from putting too much emphasis on your perception of the organization D) Encourage you to look carefully at your own assumptions, which may be biased E) Encourage you to develop friendship ties within the organization Answer: E Explanation: It is important to encourage the newcomer to develop friendship ties in the organization because newcomers are more committed when friends and co-workers help them "learn the ropes." LO: 16.3: Identify the factors that create and sustain an organization's culture. AACSB: Analytical thinking Difficulty: Hard Employability Skills: Knowledge Application and Analysis Quest. Category: Application 65) When you start to accept the differences between your expectations and the reality of the organization, you move into the ________ stage of socialization. A) prearrival B) post-arrival C) acceptance D) metamorphosis E) post-encounter Answer: D Explanation: To work out any problems discovered during the encounter stage, the new member changes, or goes through the metamorphosis stage. LO: 16.3: Identify the factors that create and sustain an organization's culture. AACSB: Analytical thinking Difficulty: Moderate Employability Skills: Knowledge Application and Analysis Quest. Category: Application
730 richard@qwconsultancy.com
66) Your supervisor provides you a mentor to help you with your work and guide you. This shows that your supervisor is using a(n) ________ socialization technique. A) divestiture B) random C) informal D) serial E) variable Answer: D Explanation: Serial socialization is characterized by the use of role models who train and encourage the newcomer. Apprenticeship and mentoring programs are examples. LO: 16.3: Identify the factors that create and sustain an organization's culture. AACSB: Analytical thinking Difficulty: Moderate Employability Skills: Knowledge Application and Analysis Quest. Category: Application 67) The three-part entry socialization process is complete when new members have internalized and accepted the norms of the organization and their work groups, are confident in their competence, and feel trusted and valued by their peers. Answer: TRUE Explanation: The three-part entry socialization process is complete when new members have internalized and accepted the norms of the organization and their work groups, are confident in their competence, and feel trusted and valued by their peers. LO: 16.3: Identify the factors that create and sustain an organization's culture. Difficulty: Moderate Quest. Category: Concept 68) The encounter stage of the socialization process consists of the period of learning that occurs before a new employee joins an organization. Answer: FALSE Explanation: On entry into the organization, the new member enters the encounter stage and confronts the possibility that expectations—about the job, co-workers, the boss, and the organization in general—may differ from reality. The socialization process consists of the period of learning that occurs before a new employee joins an organization is the prearrival stage. LO: 16.3: Identify the factors that create and sustain an organization's culture. Difficulty: Easy Quest. Category: Concept
731 richard@qwconsultancy.com
69) During the metamorphosis stage of the socialization process, a new employee compares his or her expectations—about the job, co-workers, the boss, and the organization in general—with the reality in the organization. Answer: FALSE Explanation: During the encounter stage of the socialization process, a new employee compares his or her expectations—about the job, co-workers, the boss, and the organization in general— with the reality in the organization. To work out any problems discovered during the encounter stage, the new member changes, or goes through the metamorphosis stage. During the metamorphosis stage, new employee changes an adjusts to the job, work group, and organization. LO: 16.3: Identify the factors that create and sustain an organization's culture. Difficulty: Easy Quest. Category: Concept 70) Serial socialization is characterized by the use of role models who train and encourage a newcomer. Answer: TRUE Explanation: Serial socialization is characterized by the use of role models who train and encourage a newcomer. Apprenticeship and mentoring programs are examples. LO: 16.3: Identify the factors that create and sustain an organization's culture. Difficulty: Easy Quest. Category: Concept 71) Apprenticeship and mentoring programs are examples of random socialization. Answer: FALSE Explanation: Apprenticeship and mentoring programs are examples of serial socialization. Serial socialization is characterized by the use of role models who train and encourage the newcomer. LO: 16.3: Identify the factors that create and sustain an organization's culture. Difficulty: Easy Quest. Category: Concept 72) Formal socialization involves putting the new employee directly into the job, with little or no special attention. Answer: FALSE Explanation: The more a new employee is segregated from the ongoing work setting and differentiated in some way to make explicit his or her newcomer's role, the more formal socialization is. Specific orientation and training programs are examples. Informal socialization puts the new employee directly into the job, with little or no special attention. LO: 16.3: Identify the factors that create and sustain an organization's culture. Difficulty: Easy Quest. Category: Concept
732 richard@qwconsultancy.com
73) Top managers' actions set the general climate in an organization, including what is acceptable behavior and what is not, and employees sustain and perpetuate the culture. Answer: TRUE Explanation: Top managers' actions set the general climate in an organization, including what is acceptable behavior and what is not, and employees sustain and perpetuate the culture. LO: 16.3: Identify the factors that create and sustain an organization's culture. Difficulty: Moderate Quest. Category: Concept 74) Explain how organizational culture develops. Answer: An organization's current customs, traditions, and general way of doing things are largely due to what it has done before and the degree of success it has had with those endeavors. The founders of an organization traditionally have a major impact on that organization's early culture. They have a vision of what the organization should be. The small size that typically characterizes new organizations further facilitates the founders' imposition of their vision on all organizational members. The process of culture-creation occurs in three ways. a) First, founders only hire and keep employees who think and feel the way they do. b) Second, they indoctrinate and socialize these employees to their way of thinking and feeling. c) And finally, the founders' own behavior acts as a role model that encourages employees to identify with them and thereby internalize their beliefs, values, and assumptions. When the organization succeeds, the founders' vision becomes seen as a primary determinant of that success. At this point, the founders' entire personalities become embedded in the culture of the organization. LO: 16.3: Identify the factors that create and sustain an organization's culture. Difficulty: Moderate Quest. Category: Concept
733 richard@qwconsultancy.com
75) What are the primary methods of maintaining an organization's culture? Answer: Once a culture is in place, there are practices within the organization that act to maintain it by giving employees a set of similar experiences. Three forces play a particularly important part in sustaining a culture: selection practices, the actions of top management, and socialization methods. a) Selection: Selection can ensure the hiring of people who have values essentially consistent with those of the organization, or at least a good portion of those values. In addition, the selection process provides information to applicants about the organization. Candidates learn about the organization and, if they perceive a conflict between their values and those of the organization, they can self-select themselves out of the applicant pool. b) Top management: In addition to selection, the actions of top management also have a major impact on the organization's culture. Through what they say and how they behave, senior executives establish norms that filter down through the organization as to whether risk taking is desirable; how much freedom managers should give their employees; what is appropriate dress; what actions will pay off in terms of pay raises, promotions, and other rewards; and the like. c) Socialization: Finally, no matter how good a job the organization does in recruiting and selection, new employees are not fully indoctrinated in the organization's culture. Because they are unfamiliar with the organization's culture, new employees are potentially likely to disturb the beliefs and customs that are in place. The organization will, therefore, want to help new employees adapt to its culture. This is done through the process of socialization. LO: 16.3: Identify the factors that create and sustain an organization's culture. Difficulty: Moderate Quest. Category: Concept 76) What are the three stages involved in the process of socialization? Answer: The process that helps new employees adapt to the prevailing culture is socialization. The three stages of socialization are prearrival, encounter, and metamorphosis. a) The prearrival stage recognizes that each individual arrives with a set of values, attitudes, and expectations. These cover both the work to be done and the organization. b) Upon entry into the organization, the new member enters the encounter stage. Here, the individual confronts the possible dichotomy between his or her expectations and reality. c) Finally, the new member must work out any problems discovered during the encounter stage. This may mean going through changes—hence, this is called the metamorphosis stage. LO: 16.3: Identify the factors that create and sustain an organization's culture. Difficulty: Moderate Quest. Category: Concept
734 richard@qwconsultancy.com
77) A strong culture supported by ________ ensures that all employees will act in a relatively uniform and predictable way. A) institutionalization B) organizational infrastructure C) socialization D) centralization E) social support Answer: B Explanation: A strong culture supported by formal rules and regulations (i.e., an organizational infrastructure) ensures that employees will act in a relatively uniform and predictable way. LO: 16.4: Compare the functional and dysfunctional effects of organizational culture on people and the organization. Difficulty: Moderate Quest. Category: Concept 78) Which of the following statements is true regarding the functions of culture in an organization? A) It hinders the generation of commitment to something larger than individual self-interest among employees. B) It conveys a sense of identity for organization members. C) It reduces the stability of the organizational system. D) It reduces distinctions between one organization and others. E) It does not affect employees' attitudes and behavior. Answer: B Explanation: Culture facilitates the generation of commitment to something larger than individual self-interest. It conveys a sense of identity for organization members. Culture has a boundary-defining role: it creates distinctions between one organization and others. It is a sensemaking and control mechanism that guides and shapes employees' attitudes and behavior. LO: 16.4: Compare the functional and dysfunctional effects of organizational culture on people and the organization. Difficulty: Moderate Quest. Category: Concept
735 richard@qwconsultancy.com
79) ________ ensures that employees will act in a relatively uniform and predictable way. A) Formal rules and regulations B) Shared meaning C) Rituals D) Employee socialization E) A rigid hierarchy Answer: A Explanation: A strong culture supported by formal rules and regulations (i.e., an organizational infrastructure) ensures that employees will act in a relatively uniform and predictable way. Research has shown that a positive organizational culture improves employee job attitudes and, as such, bolsters talent attraction and retention. LO: 16.4: Compare the functional and dysfunctional effects of organizational culture on people and the organization. Difficulty: Moderate Quest. Category: Concept 80) The shared concept of right and wrong behavior in the workplace that reflects the time value of the organization and shapes the ethical decision making of its members is known as ________. A) a cultural liability B) rules-based culture C) ethical culture D) a caring culture E) a sharing culture Answer: C Explanation: The shared concept of right and wrong behavior in the workplace that reflects the time value of the organization and shapes the ethical decision making of its members is known as ethical culture. LO: 16.4: Compare the functional and dysfunctional effects of organizational culture on people and the organization. Difficulty: Easy Employability Skills: Business Ethics and Social Responsibility Quest. Category: Concept
736 richard@qwconsultancy.com
81) Which of the following statements is true regarding the establishment of the organizational culture and its effects on the organization? A) Today's trend toward decentralized organizations makes it is easier to establish a strong culture. B) In a virtual organization, a strong culture can be established quickly and easily. C) Employees organized in teams always show greater allegiance to the values of the organization as a whole than to their team and its values. D) Culture acts as a control mechanism and guides the behavior of employees. E) Cultures reduce the stability of the social system in an organization. Answer: D Explanation: Today's trend toward decentralized organization makes establishing a strong culture more difficult. In virtual organizations, the lack of frequent face-to-face contact makes establishing a common set of norms very difficult. When formal authority and control systems are reduced, culture's shared meaning points everyone in the same direction. However, employees organized in teams may show greater allegiance to their team and its values than to the values of the organization as a whole. Culture enhances the stability of the social system and acts as a control mechanism that guides and shapes employees' attitudes and behavior. LO: 16.4: Compare the functional and dysfunctional effects of organizational culture on people and the organization. Difficulty: Hard Quest. Category: Concept 82) The shared perceptions organizational members have about their organization and work environment are referred to as ________. A) organizational climate B) institutionalization C) microcosm D) groupthink E) organizational apprehension Answer: A Explanation: Organizational climate refers to the shared perceptions organizational members have about their organization and work environment. This aspect of culture is like team spirit at the organizational level. LO: 16.4: Compare the functional and dysfunctional effects of organizational culture on people and the organization. Difficulty: Easy Quest. Category: Concept
737 richard@qwconsultancy.com
83) Grace works for a pet store where everyone is committed to the happiness of the animals. Often employees, bosses, and hourly workers alike come into the store "off the clock" and spend time training the animals. Everyone loves the store, the animals, and their jobs. This attitude of her co-workers inspires Grace to do her best. Based on this information, we can say that Grace is experiencing the effects of ________. A) decentralization B) organizational climate C) high departmentalization D) low formalization E) high work specialization Answer: B Explanation: Organizational climate refers to the shared perceptions organizational members have about their organization and work environment. This aspect of culture is like team spirit at the organizational level. LO: 16.4: Compare the functional and dysfunctional effects of organizational culture on people and the organization. AACSB: Analytical thinking Difficulty: Hard Employability Skills: Knowledge Application and Analysis Quest. Category: Application 84) Someone who encounters a ________ climate will have higher levels of job satisfaction and organizational commitment, have better health, and be more prone to engage in safety behaviors. A) negative B) risk-taking C) enacted D) safety E) espoused Answer: D Explanation: Someone who encounters a safety climate will have higher levels of job satisfaction and organizational commitment, have better health, and be more prone to engage in safety behaviors. LO: 16.4: Compare the functional and dysfunctional effects of organizational culture on people and the organization. Difficulty: Moderate Quest. Category: Concept
738 richard@qwconsultancy.com
85) Culture is most likely to be a liability when ________. A) the employees of the organization are highly skilled B) the organization's environment is undergoing rapid change C) the organization's management is highly efficient D) the organization is highly centralized E) the organization scores low on the degree of formalization Answer: B Explanation: Culture is a liability when the shared values are not in agreement with those that further the organization's effectiveness. This is most likely when an organization's environment is dynamic and is undergoing rapid change. In this case, its entrenched culture may no longer be appropriate. LO: 16.4: Compare the functional and dysfunctional effects of organizational culture on people and the organization. Difficulty: Moderate Quest. Category: Concept 86) Rainbow Corp. hires a new secretary, Polonova, who differs from the vast majority of the company's employees in terms of her ethnicity. The company has a collectivist culture with a culturally diverse workforce and several policies to support the minorities. Yet, after a few weeks, she quits the company. Which of the following, if true, helps explain why she quit? A) Rainbow Corp. recently had a huge turnover and plans to expand its market. B) All secretaries, regardless of their gender, are paid equal salaries at Rainbow Corp. C) Polonova is gregarious and comfortable in changing contexts and ambiguous situations. D) Polonova has a strong sense of personal ambition and independence. E) Polonova is more likely to conform to others' ideas and opinions rather than come up with her own. Answer: D Explanation: Hiring new employees who differ from the majority in race, age, gender, disability, or other characteristics creates a paradox: management wants to demonstrate support for the differences these employees bring to the workplace, but newcomers who wish to fit in must accept the organization's core cultural values. In this case, Polonova couldn't accept the organization's collectivist culture due to her individualistic nature and hence quit. All other options are either irrelevant or add to the seeming paradox. If all secretaries were paid equal salaries, then it offers no grounds for Polonova to quit. LO: 16.4: Compare the functional and dysfunctional effects of organizational culture on people and the organization. AACSB: Reflective thinking Difficulty: Hard Employability Skills: Knowledge Application and Analysis Quest. Category: Critical Thinking
739 richard@qwconsultancy.com
87) Culture is a liability ________. A) when the entrenched culture is appropriate B) in a stable environment C) when the organization is undergoing rapid change D) when uniformity is desired E) in results-oriented organizations Answer: C Explanation: Culture is a liability when shared values do not agree with those that further the organization's effectiveness. This is most likely when an organization's environment is undergoing rapid change, and its entrenched culture may no longer be appropriate. Consistency of behavior, an asset in a stable environment, may then burden the organization and make it difficult to respond to changes. LO: 16.4: Compare the functional and dysfunctional effects of organizational culture on people and the organization. Difficulty: Moderate Quest. Category: Concept 88) Results-oriented cultures are toxic in-and-of-themselves. Answer: FALSE Explanation: Toxic cultures do not just influence conflict in organizations–other dysfunctions matter as well. Some organizations are solely focused on the bottom-line numbers and results without caring how those numbers and results are achieved. It is not that results-oriented cultures are toxic in and of themselves–it is just that they can be detrimental without an ethical culture to keep employees from disengaging from the ethical aspects of their work. LO: 16.4: Compare the functional and dysfunctional effects of organizational culture on people and the organization. Difficulty: Moderate Quest. Category: Concept 89) Sustainability refers to practices that can be maintained over very long periods of time because the tools or structure that support the practices is not damaged in the process. Answer: TRUE Explanation: Sustainability refers to practices that can be maintained over very long periods of time because the tools or structure that support the practices is not damaged in the process. Social sustainability practices address the ways social systems are affected by an organization's actions over time, and in turn, how changing social systems may affect the organization. LO: 16.4: Compare the functional and dysfunctional effects of organizational culture on people and the organization. Difficulty: Easy Quest. Category: Concept
740 richard@qwconsultancy.com
90) Sustainability practices can affect organizations' reputation, productivity, talent acquisition, retention, engagement, cost efficiency, innovation, and financial performance. Answer: TRUE Explanation: A substantial majority of executives view sustainability part of future success. Research has demonstrated that sustainability practices can affect organizations' reputation, productivity, talent acquisitions, retention, engagement, costs efficiency, innovation, and financial performance. LO: 16.4: Compare the functional and dysfunctional effects of organizational culture on people and the organization. Difficulty: Moderate Quest. Category: Concept 91) Social sustainability practices address the ways social systems are affected by an organization's actions over time and, in turn, how changing social systems may affect the organization. Answer: TRUE Explanation: Social sustainability practices address the ways social systems are affected by an organization's actions over time and, in turn, how changing social systems may affect the organization. Organizations should consider the effect of their labor practices on people over time, such as how job design (e.g., working hours) affects stress and health, how layoff policies and health insurance affect employees' well-being, and so on. LO: 16.4: Compare the functional and dysfunctional effects of organizational culture on people and the organization. Difficulty: Moderate Employability Skills: Business Ethics and Social Responsibility Quest. Category: Concept 92) To create a truly sustainable business, an organization must develop a long-term culture and put its values into practice. Answer: TRUE Explanation: To create a truly sustainable business, an organization must develop a long-term culture and put its values into practice. In other words, there needs to be a sustainable system for creating sustainability! In one workplace study, a company seeking to reduce energy consumption found that soliciting group feedback reduced energy use significantly more than simply issuing materials about the importance of conservation. LO: 16.4: Compare the functional and dysfunctional effects of organizational culture on people and the organization. Difficulty: Moderate Quest. Category: Concept
741 richard@qwconsultancy.com
93) Culture defines "the rules of the game." Explain the statement. Answer: Culture defines "the rules of the game." First, it has a boundary-defining role: It creates distinctions between organizations. Second, it conveys a sense of identity for organization members. Third, culture facilitates commitment to something larger than individual self-interest. Fourth, it enhances the stability of the social system. Culture is the social glue that helps hold the organization together by providing standards for what employees should say and do. Finally, it is a sense-making and control mechanism that guides and shapes employees' attitudes and behavior. LO: 16.4: Compare the functional and dysfunctional effects of organizational culture on people and the organization. AACSB: Analytical thinking Difficulty: Hard Employability Skills: Critical Thinking Quest. Category: Critical Thinking 94) Explain the concept of organizational climate. Answer: Organizational climate refers to the shared perceptions organizational members have about their organization and work environment. This aspect of culture is like team spirit at the organizational level. When everyone has the same general feelings about what's important or how well things are working, the effect of these attitudes will be more than the sum of the individual parts. One meta-analysis found that across dozens of different samples, psychological climate was strongly related to individuals' level of job satisfaction, involvement, commitment, and motivation. A positive overall workplace climate has been linked to higher customer satisfaction and financial performance as well. LO: 16.4: Compare the functional and dysfunctional effects of organizational culture on people and the organization. Difficulty: Moderate Quest. Category: Concept
742 richard@qwconsultancy.com
95) When is culture a liability? Answer: The various situations under which culture acts as a liability are: a) Institutionalization: When an organization undergoes institutionalization and becomes institutionalized—that is, it is valued for itself and not for the goods or services it produces—it takes on a life of its own, apart from its founders or members. It doesn't go out of business even if its original goals are no longer relevant. Acceptable modes of behavior become largely selfevident to members, and although this isn't entirely negative, it does mean behaviors and habits that should be questioned and analyzed become taken for granted, which can stifle innovation and make maintaining the organization's culture an end in itself. b) Barriers to change: Culture is a liability when the shared values are not in agreement with those that further the organization's effectiveness. This is most likely when an organization's environment is undergoing rapid change and its entrenched culture may no longer be appropriate. c) Barriers to diversity: By limiting the range of acceptable values and styles, strong cultures put considerable pressure on employees to conform. d) Barriers to acquisitions and mergers: Historically, when management looked at acquisition or merger decisions, the key factors were financial advantage and product synergy. In recent years, cultural compatibility has become the primary concern. LO: 16.4: Compare the functional and dysfunctional effects of organizational culture on people and the organization. Difficulty: Moderate Quest. Category: Concept 96) Explain how organizational cultures have an impact on employee performance and satisfaction. Answer: Employees form an overall subjective perception of the organization based on leader and member behaviors, the organizational structure, and the organization's values and beliefs as well as artifacts, stories, and symbols (Exhibit 16-6). This overall perception represents, in effect, the organization's culture, which varies in its strength (or degree to which others perceive the culture in the same way). The culture is then revealed through organizational climates, which manifest culture's underlying values and beliefs. The climates then affect employee performance and satisfaction, along with other outcomes relevant to organization. LO: 16.4: Compare the functional and dysfunctional effects of organizational culture on people and the organization. AACSB: Analytical thinking Difficulty: Moderate Quest. Category: Synthesis
743 richard@qwconsultancy.com
97) In innovative organizations, ________. A) there is limited use of committees and cross-functional teams B) creativity is usually expected but is not rewarded C) formalization is high D) flexibility is discouraged E) an abundance of resources allows the organization to absorb failures Answer: E Explanation: Having an abundance of resources allows an organization to afford to purchase or develop innovations, bear the cost of instituting them, and absorb failures. LO: 16.5: Describe the similarities and differences in creating positive, ethical, and innovative cultures. Difficulty: Moderate Quest. Category: Concept 98) Pam works for a corporation that recently fired three top managers who were caught using the company credit cards to lavishly furnish their offices and even purchase "office" furniture that was found in their personal homes. Which method of maintaining an ethical culture is Pam's company pursuing? A) Serve as a visible role model B) Communicate ethical expectations C) Provide ethical training D) Visibly punish unethical acts E) Provide protective mechanisms Answer: D Explanation: Research suggests managers can have an effect on the ethical behavior of employees by adhering to the following principles be a visible role model, communicate ethical expectations, provide ethical training, visibly reward ethical acts and punish unethical ones, and provide protective mechanisms. In this example, the company is visibly punishing unethical acts. LO: 16.5: Describe the similarities and differences in creating positive, ethical, and innovative cultures. AACSB: Analytical thinking Difficulty: Moderate Employability Skills: Business Ethics and Social Responsibility Quest. Category: Application
744 richard@qwconsultancy.com
99) A positive organizational culture emphasizes ________. A) the use of negative reinforcement and punishment B) the use of rituals in transmitting organizational culture C) individual vitality and growth D) the significance of highly centralized management E) organizational standardization and institutionalization Answer: C Explanation: A positive organizational culture emphasizes building on employee strengths, rewards more than it punishes, and emphasizes individual vitality and growth. LO: 16.5: Describe the similarities and differences in creating positive, ethical, and innovative cultures. Difficulty: Easy Quest. Category: Concept 100) Larry has an employee who is amazing at clearly seeing rules and enforcing them. He has recently promoted her to the lead position in quality assurance. Which method of creating a positive organizational culture is Larry utilizing? A) Rewarding more than punishing B) Building on organization strengths C) Using standardized practices D) Building on employee strengths E) Transmitting organizational culture through rituals Answer: D Explanation: Larry is building on employee strengths. Because his employee is good at enforcing rules, he is showing her how she can put that strength to use. Although a positive organizational culture does not ignore problems, it does emphasize showing workers how they can capitalize on their strengths. LO: 16.5: Describe the similarities and differences in creating positive, ethical, and innovative cultures. AACSB: Analytical thinking Difficulty: Moderate Employability Skills: Knowledge Application and Analysis Quest. Category: Application
745 richard@qwconsultancy.com
101) Patricia is a team leader. When looking at an e-mail from one of her team members, Patricia noticed that the e-mail was sent at nine-thirty in the evening. The next day she personally went to the employee's cubicle and told him how much she appreciated him staying late to get the project to the client on time. Which aspect of creating a positive organizational culture is Patricia utilizing? A) Rewarding more than punishing B) Building on organization strengths C) Emphasizing individual growth D) Building on employee strengths E) Providing extrinsic rewards Answer: A Explanation: Patricia is utilizing rewards, rather than punishment. Most organizations are focused on extrinsic rewards such as pay and promotions and often forget about the power of smaller rewards such as the praise that Patricia is giving. LO: 16.5: Describe the similarities and differences in creating positive, ethical, and innovative cultures. AACSB: Analytical thinking Difficulty: Hard Employability Skills: Knowledge Application and Analysis Quest. Category: Application 102) Liz is the lead editor for a small, city newspaper. One of her advertising sales representatives is pulling in a lot of restaurant clients. Liz realizes that the representative is quite knowledgeable about food and speaks quite elegantly when describing different dishes. She has begun discussing with the representative the possibility of collaborating with a writer to include a weekly restaurant review that includes the representative on the byline. After the first couple of reviews, the representative is able to increase ad sizes and sales. He is very happy with his new job diversification. Which aspect of creating a positive organizational culture is Liz utilizing? A) Rewarding more than punishing B) Building on organization strengths C) Emphasizing individual vitality and growth D) Creating loose management oversight E) Using narrow spans of control Answer: C Explanation: A positive organizational culture emphasizes not only organizational effectiveness but also individuals' growth. No organization will get the best from employees who see themselves as mere cogs in the machine. Liz is helping her employee discover new aspects of his abilities and giving him new career opportunities. LO: 16.5: Describe the similarities and differences in creating positive, ethical, and innovative cultures. AACSB: Analytical thinking Difficulty: Moderate Employability Skills: Knowledge Application and Analysis Quest. Category: Application
746 richard@qwconsultancy.com
103) Individuals who take an innovation and actively and enthusiastically promote the idea, build support, and overcome resistance, and ensure that the idea is implemented are known as idea champions. Answer: TRUE Explanation: Individuals who take an innovation and actively and enthusiastically promote the idea, build support, and overcome resistance, and ensure that the idea is implemented are known as idea champions. LO: 16.5: Describe the similarities and differences in creating positive, ethical, and innovative cultures. Difficulty: Easy Quest. Category: Concept 104) A positive organizational culture emphasizes building on employee strengths, rewards more than it punishes, and emphasizes individual vitality and growth. Answer: TRUE Explanation: A positive organizational culture emphasizes building on employee strengths, rewards more than it punishes, and emphasizes individual vitality and growth. LO: 16.5: Describe the similarities and differences in creating positive, ethical, and innovative cultures. Difficulty: Easy Quest. Category: Concept 105) How can management create a more ethical culture within an organization? Answer: To create a more ethical culture, management can take a number of steps. a) Management can be a visible role model. Employees will look to top management behavior as a benchmark for defining appropriate behavior. b) Management can communicate ethical expectations. Ethical ambiguities can be minimized by creating and disseminating an organizational code of ethics. It should state the organization's primary values and the ethical rules that employees are expected to follow. c) Management can provide ethical training. Training sessions can be used to reinforce the organization's standards of conduct, to clarify what practices are and are not permissible, and to address possible ethical dilemmas. d) Management can visibly reward ethical acts and punish unethical ones. Performance appraisals of managers should include a point-by-point evaluation of how his or her decisions measured against the organization's code of ethics. Appraisals must include the means taken to achieve goals as well as the ends themselves. e) Finally, the organization needs to provide formal mechanisms so that employees can discuss ethical dilemmas and report unethical behavior without fear of reprimand. This might include creation of ethical counselors, ombudsmen, or ethical officers. LO: 16.5: Describe the similarities and differences in creating positive, ethical, and innovative cultures. Difficulty: Moderate Employability Skills: Business Ethics and Social Responsibility Quest. Category: Concept
747 richard@qwconsultancy.com
106) What are the characteristics of a positive organizational culture. Answer: A positive organizational culture emphasizes building on employee strengths, rewards more than it punishes, and emphasizes individual vitality and growth. a) Building on employee strengths: Although a positive organizational culture does not ignore problems, it does emphasize showing workers how they can capitalize on their strengths. b) Rewarding more than punishing: Although most organizations are sufficiently focused on extrinsic rewards such as pay and promotions, they often forget about the power of smaller (and cheaper) rewards such as praise. c) Emphasizing vitality and growth: No organization will get the best from employees who see themselves as mere cogs in the machine. A positive culture recognizes the difference between a job and a career. It supports not only what the employee contributes to organizational effectiveness but also how the organization can make the employee more effective—personally and professionally. LO: 16.5: Describe the similarities and differences in creating positive, ethical, and innovative cultures. Difficulty: Moderate Quest. Category: Concept 107) What are the similarities and differences between an ethical organizational culture and a positive organizational culture? Answer: The organizational culture most likely to shape high ethical standards among its members is one that's high in risk tolerance, low to moderate in aggressiveness, and focused on means as well as outcomes. This type of culture also takes a long-term perspective and balances the rights of multiple stakeholders, including the communities in which the business operates, its employees, and its stockholders. Managers are supported for taking risks and innovating, discouraged from engaging in unbridled competition, and guided to pay attention not just to what goals are achieved but also to how. If the culture is strong and supports high ethical standards, it should have a very powerful and positive influence on employee behavior. Research suggests managers can have an effect on the ethical behavior of employees by adhering to the following principles: be a visible role model, communicate ethical expectations, provide ethical training, visibly reward ethical acts and punish unethical ones, and provide protective mechanisms. A positive organizational culture emphasizes building on employee strengths, rewards more than it punishes, and emphasizes individual vitality and growth. The steps to create a positive work culture are more focused on the behavior of the employee, whereas the steps to creating an ethical work culture focus more on the behavior and responsibilities of management. They both take a long-term perspective and are concerned with balancing the rights and happiness of the employees and the investors. LO: 16.5: Describe the similarities and differences in creating positive, ethical, and innovative cultures. AACSB: Analytical thinking Difficulty: Moderate Employability Skills: Business Ethics and Social Responsibility Quest. Category: Synthesis
748 richard@qwconsultancy.com
108) An example of change in the nature of the workforce is an increase in ________. A) college attendance B) mergers and consolidations C) capital investment D) divorce rates E) cultural diversity Answer: E Explanation: Almost every organization must adjust to a multicultural environment, demographic changes, immigration, and outsourcing. This represents the changing nature of the workforce. LO: 16.6: Discuss how change operates both within and outside organizations. Difficulty: Moderate Quest. Category: Concept 109) Tescra Education is an educational publishing company that provides textbooks in the area of higher education. The company has been in business for almost 20 years now and has earned substantial profits during this period. Which of the following trends, if true, would drive the company to change its business strategy? A) A rise in the cost of electronic products such as iPads and e-book readers and subscription charges for educational apps B) An upward trend in students' preference for face-to-face interaction with instructors using traditional classroom learning techniques C) A need for learning environments where students feel safe to apply the knowledge they've acquired in the classroom to real-life situations D) A decrease in the number of people going back to school to upgrade their skills, due to an economic boom E) An increase in the number of laws and regulations related to intellectual property Answer: C Explanation: If students feel the need for learning environments where they feel safe to apply the knowledge they've acquired in the classroom to real-life situations, then online learning methods would be best suited to meet this need. This would require the company to change its business strategy from providing textbooks to focusing on online learning. A rise in the cost of technological tools and a preference for face-to-face interaction with instructors using traditional classroom learning techniques would increase the requirement for textbooks and would not have an impact on the company's business strategy. A decrease in the number of people going back to school to upgrade their skills would imply that there are a fewer number of students and hence the company would not need to change its strategy. Intellectual property laws apply to both online and traditional publishing and would not drive a company to change its business strategy. LO: 16.6: Discuss how change operates both within and outside organizations. AACSB: Analytical thinking Difficulty: Hard Employability Skills: Critical Thinking Quest. Category: Critical Thinking
749 richard@qwconsultancy.com
110) A paper sales company designs and offers a new method of purchasing and shipping commercial quantities of paper through its Web site. The company used to do its business through its sales representative. This is an example of a(n) ________ change. A) accidental B) secondary C) planned D) cognitive E) integrated Answer: C Explanation: The change mentioned in the case is an example of a planned change, or activities that are intentional and goal oriented, compared to accidental change, which refers to activities that just happen. LO: 16.6: Discuss how change operates both within and outside organizations. AACSB: Analytical thinking Difficulty: Moderate Employability Skills: Knowledge Application and Analysis Quest. Category: Application 111) Who are change agents? Answer: Change agents are the people in an organization who are responsible for presenting and managing change activities. They see a future for the organization that others have not identified, and they are able to motivate, invent, and implement this vision. Change agents can be managers or nonmanagers, current or new employees, or outside consultants. LO: 16.6: Discuss how change operates both within and outside organizations. Difficulty: Moderate Quest. Category: Concept 112) How can change agents deal with resistance to change? Answer: Eight tactics can help change agents deal with resistance to change: (1) communication; (2) participation; (3) building support and commitment; (4) develop positive relationships; (5) implement changes fairly; (6) manipulation and cooptation; (7) selecting people who accept change; (8) coercion. LO: 16.6: Discuss how change operates both within and outside organizations. Difficulty: Moderate Quest. Category: Concept
750 richard@qwconsultancy.com
113) ________ is a change process based on systematic collection of data and selection of a change action based on what the analyzed data indicate. A) Organizational development B) Action research C) Planned change D) Process consultation E) Organizational restructuring Answer: B Explanation: Action research is defined as a change process based on systematic collection of data and selection of a change action based on what the analyzed data indicate. Its value is in providing a scientific methodology for managing planned change. LO: 16.7: Compare the four main approaches to managing organizational change. Difficulty: Easy Quest. Category: Concept 114) Which of the following is one of the three steps in the Lewin's three-step change model? A) Analysis B) Movement C) Evaluation D) Dreaming E) Designing Answer: B Explanation: The three steps in the Lewin's three-step change model are unfreezing, movement, and refreezing. LO: 16.7: Compare the four main approaches to managing organizational change. Difficulty: Easy Quest. Category: Concept 115) Which of the following is the first step in Kotter's eight-step plan for implementing change? A) Create a new vision to direct the change and strategies for achieving the vision. B) Establish a sense of urgency by creating a compelling reason for why change is needed. C) Plan for, create, and reward short-term "wins" that move the organization toward the new vision. D) Form a coalition with enough power to lead the change. E) Communicate the vision throughout the organization. Answer: B Explanation: The first step in Kotter's eight-step plan for implementing change is establishing a sense of urgency by creating a compelling reason for why change is needed. LO: 16.7: Compare the four main approaches to managing organizational change. Difficulty: Easy Quest. Category: Concept
751 richard@qwconsultancy.com
116) Organizational development (OD) is a collection of change methods that try to improve organizational effectiveness and employee well-being. Answer: TRUE Explanation: Organizational development (OD) is a collection of change methods that try to improve organizational effectiveness and employee well-being. LO: 16.7: Compare the four main approaches to managing organizational change. Difficulty: Easy Quest. Category: Concept 117) Describe Lewin's three-step model used for managing change. Answer: Kurt Lewin argued that successful change in organizations should follow three steps: unfreezing the status quo, movement to a desired end state, and refreezing the new change to make it permanent. Before implementing a change, organizations tend to be in an equilibrium state. To move from equilibrium—to overcome the pressures of both individual resistance and group conformity— unfreezing must happen in one of three ways. The driving forces, which direct behavior away from the status quo, can be increased. The restraining forces, which hinder movement away from equilibrium, can be decreased. A third alternative is to combine the first two approaches. Research on organizational change has shown that to be effective, change has to happen quickly. Organizations that build up to change do less well than those that get to and through the movement stage quickly. Once change has been implemented, to be successful the new situation must be refrozen so it can be sustained over time. Without this last step, change will likely be short-lived, and employees will attempt to revert to the previous equilibrium state. LO: 16.7: Compare the four main approaches to managing organizational change. Difficulty: Moderate Quest. Category: Concept Organizational Behavior, 19e (Robbins/Judge) Chapter 17 Human Resource Systems and Practices 1) A(n) ________ is given to demonstrate to job-seekers what they would be doing on the job if they were hired. A) background check B) situational judgement test C) structured interview D) unstructured interview E) realistic job preview Answer: E Explanation: A realistic job preview is given to demonstrate to jobseekers what they would be doing on the job if they were hired. LO: 17.1: Describe the value of recruitment methods. Difficulty: Easy Quest. Category: Concept 2) Strategic recruitment has become a cornerstone for many companies, with some of the most effective recruiting techniques including internal referrals and internship programs. Answer: TRUE 752 richard@qwconsultancy.com
Explanation: Strategic recruitment has become a cornerstone for many companies. These recruitment practices are developed in alignment with long-term strategic goals. As for defining "success" in recruiting, most research suggests that the best system attract the most qualified candidates. Such candidates are likely to have a better fit between their skills and the job requirements, and to be more satisfied in the jobs they take. LO: 17.1: Describe the value of recruitment methods. Difficulty: Moderate Quest. Category: Concept 3) Companies are increasingly turning away from outside recruiting agencies and relying on their own executives and HR professionals for talent searches. Answer: TRUE Explanation: Companies are increasingly turning away from outside recruiting agencies and relying on their own executives and HR professionals for talent searches. LO: 17.1: Describe the value of recruitment methods. Difficulty: Moderate Quest. Category: Concept 4) A realistic job preview is a job tryout given to demonstrate to job seekers what they would be doing on the job if they were hired. Answer: TRUE Explanation: A realistic job preview is a job tryout given to demonstrate to job seekers what they would be doing on the job if they were hired. LO: 17.1: Describe the value of recruitment methods. Difficulty: Easy Quest. Category: Concept 5) ________ are most likely to be used for preliminary "rough cuts" to decide whether an applicant meets the basic qualifications for a job. A) Drug tests B) Application forms C) HR interviews D) Written tests E) Technical interviews Answer: B Explanation: Initial selection devices are the first information applicants submit and are used for deciding whether the applicant meets the basic qualifications for a job. Application forms are initial selection devices. Application forms help determine if an applicant has the proper credentials (education, certification, experience) for the position. LO: 17.2: Specify initial selection methods. Difficulty: Moderate Quest. Category: Concept 6) Charlie Woods started working for E-Z Freeze, a major frozen food company, as a college intern. Most recently, Charlie had held the position of VP Marketing. Now however, as a result of a change in corporate leadership, Charlie finds himself unemployed for the first time in 753 richard@qwconsultancy.com
decades. Looking for a new job has been an eye-opening experience for Charlie. He has been especially frustrated with his lack of success in getting even a call back for the applications he has completed online. Which of the following best describes why Charlie may not be getting the initial feedback he thinks he should be receiving? A) His online applications don't have the right keywords. B) His background checks have been negative. C) He is unwilling to relocate. D) He lacks cross-company experience. E) He lacks cultural diversity. Answer: A Explanation: When you are a candidate, be careful about what you put on your online applications. Many HR departments, faced with an overwhelming number of electronic submissions, are using software to preselect candidates based on keyword matches between applications on the qualifications needed for the job. Their software often seeks to screen out unacceptable candidates rather than select potentially good ones. While you will want to incorporate all the keyword that accurately describe your experience, including paid and volunteer work and use whatever prompts you are given to outline the personal characteristics that qualify you, be careful not to overstate. LO: 17.2: Specify initial selection methods. AACSB: Reflective thinking Difficulty: Hard Employability Skills: Critical Thinking Quest. Category: Critical Thinking
754 richard@qwconsultancy.com
7) A(n) ________ is a form of background check in a job application. A) aptitude test B) offer letter C) letter of recommendation D) application form E) work-sample test Answer: C Explanation: Letters of recommendation are a form of background check. These aren't as useful as they may seem because applicants select those who will write good things about them, so almost all letters of recommendation are positive. LO: 17.2: Specify initial selection methods. Difficulty: Moderate Quest. Category: Concept 8) Stefan is looking for a job. Today he went to the website of Qriosity Inc., where he filled out an online application and attached a copy of his resume. In which part of the selection process is Stefan? A) Physical selection B) Job offer C) Initial selection D) Contingent selection E) Substantive selection Answer: C Explanation: Applicants go through three stages in the selection process, initial selection, substantive selection, and contingent selection. Initial selection devices are the first information applicants submit and are used to decide whether the applicant meets the basic qualifications for a job. Application forms are initial selection devices. Today many organizations encourage applicants to apply online. LO: 17.2: Specify initial selection methods. AACSB: Analytical thinking Difficulty: Moderate Employability Skills: Knowledge Application and Analysis Quest. Category: Application
755 richard@qwconsultancy.com
9) A ________ is most likely to be an initial selection device. A) technical interview B) drug test C) written test D) background check E) work sample test Answer: D Explanation: Initial selection devices are the first information applicants submit and are used to decide whether the applicant meets the basic qualifications for a job. Application forms (including letters of recommendation) are initial selection devices. Background checks are either an initial selection device or a contingent selection device, depending on how the organization chooses to structure its selection process. LO: 17.2: Specify initial selection methods. Difficulty: Moderate Quest. Category: Concept Scarlett has received authorization to add an assistant manager to her department. This person will be working closely with Scarlett and must have excellent interpersonal as well as technical skills. Scarlett wants to make certain that she maximizes the chance of choosing the correct individual and is trying to decide on the most appropriate selection device. 10) Which of the following is most likely to be used by Scarlett during the initial selection stage to check if an applicant meets the basic qualifications for a job? A) Assessment center B) Drug test C) Application form D) Work sample test E) Integrity test Answer: C Explanation: Initial selection devices are the first information applicants submit and are used for preliminary rough cuts to decide whether the applicant meets the basic qualifications for a job. One of the initial selection devices is an application form. LO: 17.2: Specify initial selection methods. AACSB: Analytical thinking Difficulty: Moderate Employability Skills: Knowledge Application and Analysis Quest. Category: Application 11) The goal of initial selection is to determine the most qualified applicants from among those who meet basic qualifications. Answer: FALSE Explanation: The goal of initial selection is to decide whether an applicant meets the basic qualifications for a job. LO: 17.2: Specify initial selection methods. Difficulty: Moderate Quest. Category: Concept 756 richard@qwconsultancy.com
12) What are initial selection methods and how are they used? Answer: Initial selection devices are the first information applicants submit and are used for preliminary rough cuts to decide whether the applicant meets the basic qualifications for a job. Application forms (including letters of recommendation) are initial selection devices. We list background checks as either an initial selection device or a contingent selection device, depending on how the organization handles them. Some organizations prefer to look into an applicant's background right away. Others wait until the applicant is about ready to be hired, contingent on everything else checking out. LO: 17.2: Specify initial selection methods. Difficulty: Moderate Quest. Category: Concept 13) The ________ selection stage immediately follows the initial selection stage in the selection process. A) cognitive B) associative C) analytic D) behavioral E) substantive Answer: E Explanation: If an applicant passes the initial screens, next are substantive selection methods. These are the heart of the selection process and include written tests, performance tests, and interviews. LO: 17.3: Identify the most useful substantive selection methods. Difficulty: Moderate Quest. Category: Concept 14) Between the late 1960s and the mid-1980s, the use of written tests declined because they were characterized as ________. A) unprofessional B) discriminatory C) unreliable D) invalid E) mathematically flawed Answer: B Explanation: Long popular as selection devices, written tests declined in use between the late 1960s and mid-1980s, especially in the United States. They were frequently characterized as discriminatory, and many organizations had not validated them as job related. LO: 17.3: Identify the most useful substantive selection methods. Difficulty: Moderate Quest. Category: Concept
757 richard@qwconsultancy.com
15) ________ proven to be a particularly good predictor for jobs that include cognitively complex tasks. A) Intelligence tests have B) Integrity evaluations have C) Work sampling has D) Drug tests have E) Behavioral assessment have Answer: A Explanation: Intelligence tests have proven to be particularly good predictors for jobs that include cognitively complex tasks. Many experts say intelligence tests are the single best selection measure across jobs and that they are at least as valid in the European Union (EU) nations as in the United States. LO: 17.3: Identify the most useful substantive selection methods. Difficulty: Easy Quest. Category: Concept 16) In the ________ stage of the selection process, written tests are used. A) cognitive B) associative C) analytic D) substantive E) initial Answer: D Explanation: Written tests are used during the substantive stage of the selection process. LO: 17.3: Identify the most useful substantive selection methods. Difficulty: Moderate Quest. Category: Concept 17) Performance-simulation tests have higher ________, which measures whether applicants perceive the measures to be accurate. A) nominal validity B) nominal reasonability C) face cognition D) face validity E) face legitimacy Answer: D Explanation: Performance-simulation tests have higher face validity (which measures whether applicants perceive the measures to be accurate). LO: 17.3: Identify the most useful substantive selection methods. Difficulty: Moderate Quest. Category: Concept
758 richard@qwconsultancy.com
18) Elaborate sets of performance-simulation tests specifically designed to evaluate a candidate's managerial potential are ________. A) less effective than work sampling B) administered in assessment centers C) similar to personality tests D) considered drawbacks of some management development programs E) likely to skew the results of behavioral samples Answer: B Explanation: As compared to work sample tests, a more elaborate set of performance-simulation tests specifically designed to evaluate a candidate's managerial potential is administered in assessment centers. Line executives, supervisors, and/or trained psychologists evaluate candidates as they go through one to several days of exercises that simulate real problems they would confront on the job. LO: 17.3: Identify the most useful substantive selection methods. Difficulty: Moderate Quest. Category: Concept 19) Assessment centers are typically used to evaluate a candidate's ________ potential. A) linguistic B) technical C) cognitive D) managerial E) analytical Answer: D Explanation: Performance-simulation tests, designed to evaluate a candidate's managerial potential, are typically administered in assessment centers. LO: 17.3: Identify the most useful substantive selection methods. Difficulty: Moderate Quest. Category: Concept 20) ________ tend to have a disproportionate amount of influence on employee selection decisions. A) Interviews B) Written test C) Performance-simulation tests D) Work sampling methods E) Personality tests Answer: A Explanation: Of all the selection devices organizations around the globe use to differentiate candidates, the interview remains the most common. It also tends to have a disproportionate amount of influence. The candidate who performs poorly in the employment interview is likely to be cut from the applicant pool regardless of experience, test scores, or letters of recommendation. LO: 17.3: Identify the most useful substantive selection methods. Difficulty: Moderate Quest. Category: Concept 759 richard@qwconsultancy.com
21) Which type of interview is short, causal, and made up of improvised questions? A) Unstructured B) Audition-type C) Performance-simulation D) Problem-solving E) Structured Answer: A Explanation: Unstructured interviews are short, casual, and made up of improvised questions. LO: 17.3: Identify the most useful substantive selection methods. Difficulty: Easy Quest. Category: Concept 22) Many organizations have started to use situational judgment tests ________. A) because past behavior is the best predictor of future behavior B) because technical knowledge and skills are the best predictor of job performance C) after finding that personality is the best predictor of job performance D) to offset the correlation between personality and mood E) to reduce the cost of job simulations Answer: E Explanation: To reduce the costs of job simulations, many organizations have started to use situational judgment tests, which ask applicants how they would perform in a variety of job situations and then compare their answers to the answers of high-performing employees. LO: 17.3: Identify the most useful substantive selection methods. Difficulty: Moderate Quest. Category: Concept
760 richard@qwconsultancy.com
Scarlett has received authorization to add an assistant manager to her department. This person will be working closely with Scarlett and must have excellent interpersonal as well as technical skills. Scarlett wants to make certain that she maximizes the chance of choosing the correct individual and is trying to decide on the most appropriate selection device. 23) In order to assess the management skills of the applicants and managerial potential, Scarlet should ________. A) use an assessment center B) verify the applicants' job histories C) conduct a medical test D) conduct a drug test E) conduct a behavioral assessment test Answer: A Explanation: An elaborate set of performance-simulation tests specifically designed to evaluate a candidate's managerial potential is administered in assessment centers. Line executives, supervisors, and/or trained psychologists evaluate candidates as they go through one to several days of exercises that simulate real problems they would confront on the job. LO: 17.3: Identify the most useful substantive selection methods. AACSB: Analytical thinking Difficulty: Hard Employability Skills: Knowledge Application and Analysis Quest. Category: Application 24) The ________ is a common contingent selection method. A) IQ test B) interview C) interest inventory D) drug test E) work sample test Answer: D Explanation: If applicants pass the substantive selection methods, they are ready to be hired, contingent on a final check. One common contingent method is a drug test. LO: 17.3: Identify the most useful substantive selection methods. Difficulty: Moderate Quest. Category: Concept
761 richard@qwconsultancy.com
25) Drug testing is controversial. Which of the following is an argument used against drug testing? A) The results of drug tests can be faked easily. B) Drug use is a private matter. C) Drug tests are costly. D) Drug tests are generally inaccurate. E) Drug tests are injurious to a person's health. Answer: B Explanation: Drug testing is controversial. Many applicants think testing without reasonable suspicion is invasive or unfair and say they should be tested on job performance factors, not lifestyle choices that may not be relevant. LO: 17.3: Identify the most useful substantive selection methods. Difficulty: Moderate Quest. Category: Concept 26) Due to the passage of the Americans with Disabilities Act, job applicants ________. A) can be subjected to a drug test only if they are being considered for senior-level management positions B) are never required to submit to a physical exam for a job C) cannot be subjected to a work sample test after a contingent job offer D) cannot be subjected to behavioral assessment tests before a contingent offer E) may be required to undergo a medical examination after they receive a contingent offer Answer: E Explanation: Under the Americans with Disabilities Act, firms may not require prospective employees to pass a medical exam before a job offer is made. However, they can conduct medical exams after making a contingent offer, to determine whether an applicant is physically or mentally able to do the job. LO: 17.3: Identify the most useful substantive selection methods. Difficulty: Moderate Quest. Category: Concept 27) Performance-simulation tests have lower face validity (which measures whether applicants perceive the measures to be accurate) than written tests. Answer: FALSE Explanation: Although they are more complicated to develop and administer than written tests, performance-simulation tests have higher face validity (which measures whether applicants perceive the measures to be accurate), and their popularity has increased. LO: 17.3: Identify the most useful substantive selection methods. Difficulty: Moderate Quest. Category: Concept
762 richard@qwconsultancy.com
28) In assessment centers, candidates are evaluated as they go through several days of exercises that simulate real problems they would confront on the job. Answer: TRUE Explanation: An elaborate set of performance-simulation tests typically designed to evaluate a candidate's managerial potential is administered in assessment centers. Line executives, supervisors, and/or trained psychologists evaluate candidates as they go through one to several days of exercises that simulate real problems they would confront on the job. LO: 17.3: Identify the most useful substantive selection methods. Difficulty: Moderate Quest. Category: Concept 29) A common contingent check just prior to employment is a drug test. Answer: TRUE Explanation: If applicants pass the substantive selection methods, they are ready to be hired, contingent on final checks. One common contingent check is a drug test. LO: 17.3: Identify the most useful substantive selection methods. Difficulty: Easy Quest. Category: Concept 30) Some companies use interviews as a recruiting tool, trying to "sell" the applicant on the value of the job an organization. Answer: TRUE Explanation: Some companies use interviews as a recruiting tool, trying to "sell" the applicant on the value of the job an organization. This strategy may sometimes be necessary because of a tight labor market, but it may also be problematic. One study showed that interviewers who were trying to promote the organization during interviews were significantly worse at identifying applicant qualifications than those who focused exclusively on assessing candidate qualifications. LO: 17.3: Identify the most useful substantive selection methods. Difficulty: Moderate Quest. Category: Concept 31) Under the Americans with Disabilities Act, firms cannot require employees to pass a medical exam before or after a job offer is made. Answer: FALSE Explanation: Under the Americans with Disabilities Act, firms may not require prospective employees to pass a medical exam before a job offer is made. However, they can require medical exams after making a contingent offer. LO: 17.3: Identify the most useful substantive selection methods. Difficulty: Moderate Quest. Category: Concept
763 richard@qwconsultancy.com
32) Identify the substantive selection methods and explain how they are used to assess job candidate who have passed an initial screening. Answer: Selection devices include written tests, performance-simulation tests, and interviews. a) Typical written tests are tests of intelligence or cognitive ability tests, personality tests, integrity tests, and interest inventories. Managers have come to recognize that there are valid tests available, and they can be helpful in predicting who will be successful on the job. Applicants, however, tend to view written tests as less valid and fair than interviews or performance tests. b) What better way is there to find out if an applicant can do a job successfully than by having him or her do it? That's precisely the logic of performance-simulation tests. The two best-known performance-simulation tests are work sampling and assessment centers. The former is suited to routine jobs, whereas the latter is relevant for the selection of managerial personnel. c) Of all the selection devices that organizations use to differentiate candidates, the interview continues to be the most common. Not only is the interview widely used, but it also seems to carry a great deal of weight. LO: 17.3: Identify the most useful substantive selection methods. Difficulty: Moderate Quest. Category: Concept 33) Choose three of the written tests that are used in the employee selection process and explain how they are used. Answer: Typical written tests include tests of intelligence or cognitive ability tests, personality tests, integrity tests, and interest inventories. a) Tests in intellectual ability, spatial and mechanical ability, perceptual accuracy, and motor ability have proven to be valid predictors for many skilled, semiskilled, and unskilled operative jobs in industrial organizations. Intelligence tests have proven to be particularly good predictors for jobs that require cognitive complexity. b) Personality tests are relatively inexpensive and simple to use and administer. Organizations use numerous measures of the Big Five traits in selection decisions. c) As ethical problems have increased in organizations, integrity tests have gained popularity. These are paper-and-pencil tests that measure factors such as dependability, carefulness, responsibility, and honesty. LO: 17.3: Identify the most useful substantive selection methods. Difficulty: Moderate Quest. Category: Concept
764 richard@qwconsultancy.com
34) What is work-sample tests and assessment centers and how are they used? Answer: Work sample tests are hands-on simulations of part or all of the job that must be performed by applicants. By carefully devising work samples based on specific job tasks, management determines the knowledge, skills, and abilities needed for each job. Then each work sample element is matched with a corresponding job performance element. Work samples are widely used in the hiring of skilled workers, such as welders, machinists, carpenters, and electricians. The results from work sample experiments are impressive. Studies almost consistently demonstrate that work samples yield validities superior to written aptitude and personality tests. A more elaborate set of performance-simulation tests specifically designed to evaluate a candidate's managerial potential is administered in assessment centers. In assessment centers, line executives, supervisors, and/or trained psychologists evaluate candidates as they go through one to several days of exercises that simulate real problems that they would confront on the job. Based on a list of descriptive dimensions that the actual job incumbent has to meet, activities might include interviews, in-basket problem-solving exercises, leaderless group discussions, and business decision games. LO: 17.3: Identify the most useful substantive selection methods. Difficulty: Hard Quest. Category: Concept 35) What are the drawbacks of interviews in the employee selection process? Answer: Not only is the interview widely used, but it also seems to carry a great deal of weight. That is, the results tend to have a disproportionate amount of influence on the selection decision. The candidate who performs poorly in the employment interview is likely to be cut from the applicant pool, regardless of his or her experience, test scores, or letters of recommendation. LO: 17.3: Identify the most useful substantive selection methods. Difficulty: Moderate Quest. Category: Concept 36) Explain the difference between structured and unstructured interviews. Answer: The unstructured interview are short, casual interviews made up of improvised questions. Structured interviews are planned interviews designed to gather job-related information. Data gathered in an unstructured interview are typically biased and often only modestly related to future job performance. Structured interviews limit subjectivity as much as possible, and therefore can provide more reliable responses. LO: 17.3: Identify the most useful substantive selection methods. AACSB: Analytical thinking Difficulty: Moderate Quest. Category: Synthesis
765 richard@qwconsultancy.com
37) What is the contingent selection stage in the selection process? Answer: If applicants pass the substantive selection methods, they are ready to be hired, contingent on a final check. One common contingent method is a drug test. Drug testing is controversial. Many applicants think testing without reasonable suspicion is invasive or unfair and say they should be tested on job performance factors, not lifestyle choices that may not be relevant. The U.S. Supreme Court has concluded that drug tests are minimally invasive selection procedures that as a rule do not violate individuals' rights. Under the Americans with Disabilities Act, firms may not require prospective employees to pass a medical exam before a job offer is made. However, they can require medical exams after making a contingent offer—to determine whether an applicant is physically or mentally able to do the job. Employers also sometimes use medical exams to find out whether and how they can accommodate employees with disabilities. For jobs requiring exposure to heavy physical or psychological demands, such as air traffic controllers or firefighters, medical exams are obviously an important indicator of ability to perform. LO: 17.3: Identify the most useful substantive selection methods. Difficulty: Moderate Quest. Category: Concept 38) Increased ________ is not an outcome of effective ethics training. A) job satisfaction B) cynicism C) trust D) respect E) social Answer: B Explanation: One area of soft skills that is particularly important in the workplace is ethics training, which focuses on recognizing moral issues, regulating moral emotions, considering different perspectives, and acting ethically by treating others with respect and civility. For example, holding directed conversations about ethics and supporting the reduction of incivility on an ongoing basis is one form of training. Following this intervention, civility, respect, job satisfaction, and trust increased, while incivility, cynicism, and absences decreased. LO: 17.4: Compare the main types of training. AACSB: Ethical understanding and reasoning Difficulty: Moderate Employability Skills: Business Ethics and Social Responsibility Quest. Category: Concept
766 richard@qwconsultancy.com
39) Utilizing the knowledge, skills, and abilities learned from training on the job is referred to as ________. A) ethical B) technical C) problem-solving D) transfer of training E) cultural Answer: D Explanation: The number one goal of training is for it to transfer. This transfer of training refers to the knowledge, skills, and abilities learned from training on the job. What good is dedicating millions and billions of dollars to training if no one puts it to use? When training is done well, it can help boost the organization's human capital and even the employees' career competencies, transferrable to future jobs. LO: 17.4: Compare the main types of training. Difficulty: Easy Quest. Category: Concept 40) Andrea wants to improve the training program for her sales representatives. Her initial focus in developing the new training program is to establish what skills, abilities, and other characteristics need to be trained. Andrea is working on the ________ part of the Instructional System Design framework. A) evaluation B) develop C) design D) analyze E) implement Answer: D Explanation: In the analyze stage of the Instructional System Design framework, the goal is to establish what skills, abilities and other characteristics need to be trained. Establish what goals should be accomplished by the training. Define who the trainees would be and what they would need to be successful. LO: 17.4: Compare the main types of training. AACSB: Analytical thinking Difficulty: Moderate Employability Skills: Knowledge Application and Analysis Quest. Category: Application
767 richard@qwconsultancy.com
You are involved in training and development for your division at Dunder Mifflin Inc. You want to target some specific organizational problems through formal training using the Instructional System Design approach. 41) You need to decide where training will take place and get the media you need to what will be needed to ensure that training is successful. You are in the ________ step of the Instructional System Design approach. A) develop B) implement C) analyze D) design E) evaluation Answer: A Explanation: Establishing where training will take place and what will be needed to ensure that training is successful is the develop step of the Instructional System Design approach. LO: 17.4: Compare the main types of training. AACSB: Analytical thinking Difficulty: Moderate Employability Skills: Knowledge Application and Analysis Quest. Category: Application 42) One of the goals of your company is to encourage transfer of training. Which step of the Instructional System Design focuses on transfer of training? A) Analyze B) Implement C) Evaluation D) Design E) Develop Answer: B Explanation: In the implement step of the Instructional System Design framework, you engage trainees in the training session or program, provide opportunities for transfer and encourage transfer following training. LO: 17.4: Compare the main types of training. AACSB: Analytical thinking Difficulty: Moderate Employability Skills: Knowledge Application and Analysis Quest. Category: Application
768 richard@qwconsultancy.com
43) ________ training involves employees helping each other out at the workplace in an unplanned and unstructured environment. A) Formal B) Ethics C) Interpersonal D) Informal E) Classroom Answer: D Explanation: Most workplace learning takes place in informal training, which is unstructured, unplanned, and easily adapted to situations and individuals. In reality, most informal training is nothing other than employees helping each other out. LO: 17.4: Compare the main types of training. Difficulty: Moderate Quest. Category: Concept 44) ________ are an on-the-job training method. A) Live classroom lectures B) Apprenticeships C) Public seminars D) Podcasts E) Self-study programs Answer: B Explanation: On-the-job training methods include job rotation, apprenticeships, understudy assignments, and formal mentoring programs. LO: 17.4: Compare the main types of training. Difficulty: Moderate Quest. Category: Concept 45) Which of the following is an off-the-job training method? A) Job rotation B) Formal mentoring programs C) Internet courses D) Job enlargement E) Apprenticeship Answer: C Explanation: As on-the-job training methods often disrupt the workplace, organizations also invest in off-the-job training, the most popular method being live classroom lectures. But it also encompasses self-directed programs, e-learning, Webinars, podcasts, and massively open online courses (MOOCs). LO: 17.4: Compare the main types of training. Difficulty: Moderate Quest. Category: Concept
769 richard@qwconsultancy.com
46) The fastest growing training method is ________. A) job rotation B) live classroom lectures C) apprenticeships D) formal mentoring programs E) e-training Answer: E Explanation: The fastest-growing training method is computer-based training, or e-training. Elearning systems emphasize learner control over the pace and content of instruction, allow elearners to interact through online communities, and incorporate other techniques such as simulations and group discussions. LO: 17.4: Compare the main types of training. Difficulty: Moderate Quest. Category: Concept 47) You have just been appointed as director of your company's corporate training division. The CEO of your company has been displeased with your company's prior training programs, so you are tasked with rehauling the entire training division. You convene a meeting of all training division managers to decide on the types of training that the division will implement. One of your managers is a firm supporter of e-training programs for employees in your company's international offices. He touts the benefits of e-training programs by stressing that ________. A) e-training approaches have been proven to result in higher levels of knowledge assimilation B) e-training emphasizes learner control over the pace and content of instruction C) many employees find solitary learning to be highly motivating D) online learners are less susceptible to distractions E) e-programs are inexpensive to design and implement Answer: B Explanation: E-learning systems emphasize learner control over the pace and content of instruction, allow e-learners to interact through online communities, and incorporate other techniques such as simulations and group discussions. LO: 17.4: Compare the main types of training. AACSB: Analytical thinking Difficulty: Hard Employability Skills: Knowledge Application and Analysis Quest. Category: Application
770 richard@qwconsultancy.com
48) Training focuses on acquiring knowledge while education emphasizes building skills and developing abilities. Answer: FALSE Explanation: Although some do not make this distinction, training is different from education. Education primarily focuses on acquiring knowledge, such as facts and figures necessary to do a job or the steps in a complicated process. Training, on the other hand, spans beyond merely acquiring knowledge toward application. Training is more concerned with building skills, developing abilities, and even acquiring competencies like adaptability, emotional competence, and leadership. LO: 17.4: Compare the main types of training. Difficulty: Moderate Quest. Category: Concept 49) Most formal training involves employees simply helping each other out. Answer: FALSE Explanation: Most workplace learning takes place in informal training, unstructured, unplanned, and easily adapted to situations and individuals, for teaching skills and keeping employees current. In reality, most informal training is nothing other than employees helping each other out. LO: 17.4: Compare the main types of training. Difficulty: Moderate Quest. Category: Concept 50) Controlling emotions is part of active learning. Answer: TRUE Explanation: Active learning primarily consists of three basic tenets: (1) encouraging exploration and reflection, (2) normalizing errors and mistakes, and (3) controlling emotions. For example, an employee learning a new programming language may build some basic programs to accomplish basic tasks, learn from errors (or willfully commit them to see what would happen), and learn to manage the frustrations with making mistakes or not being an expert right away. LO: 17.4: Compare the main types of training. Difficulty: Easy Quest. Category: Concept 51) Ethics training focuses on recognizing moral issues, regulating moral emotions, considering different perspectives, and acting ethically by treating others with respect and civility. Answer: TRUE Explanation: One area of soft skills that is particularly important in the workplace is ethics training, which focuses on recognizing moral issues, regulating moral emotions, considering different perspectives, and acting ethically by treating others with respect and civility. LO: 17.4: Compare the main types of training. AACSB: Ethical understanding and reasoning Difficulty: Moderate Employability Skills: Business Ethics and Social Responsibility Quest. Category: Concept
771 richard@qwconsultancy.com
52) What are the three basic tenets of active learning? Answer: Active learning primarily consists of three basic tenets: (1) encouraging exploration and reflection, (2) normalizing errors and mistakes, and (3) controlling emotions. For example, an employee learning a new programming language may build some basic programs to accomplish basic tasks, learn from errors (or willfully commit them to see what would happen), and learn to manage the frustrations with making mistakes or not being an expert right away. LO: 17.4: Compare the main types of training. Difficulty: Moderate Quest. Category: Concept 53) Explain what the effectiveness of a training program means. Answer: The effectiveness of a training program can refer to the level of trainee satisfaction, the amount trainees learn, the extent to which they transfer the learned material to their jobs, and/or the company's financial return on investments in training. These results are not always related. Some people who have a positive experience in an upbeat, fun class learn very little; some who learn a great deal have difficulty figuring out how to use their knowledge at work; and changes in employee behavior are sometimes not large enough to justify the expense of training. This means rigorous measurement of multiple training outcomes should be part of every training effort. LO: 17.4: Compare the main types of training. Difficulty: Moderate Quest. Category: Concept 54) Provide a workplace example of informal training for technical skills and another for problem-solving skills. Answer: Informal training is unstructured, unplanned training that takes place at the workplace. Most informal training is nothing other than employees helping each other out. They share information and solve work-related problems together. Informal training is easily adapted to situations and individuals and is good for teaching skills and keeping employees current. An example of informal training for the improvement of technical skills could be as simple as a factory line worker helping another line worker solve a problem with one of the machines and explaining how the machine works and how to solve the problem the next time it occurs. With this informal training, the new line worker is made independent and, by understanding the machine, can troubleshoot with different problems as well. An example of informal training of problem-solving skills could be an impromptu gathering in a cubicle to discuss a problem that a programmer is having to crack a certain code. Together, various programmers could solve a problem and train each other. LO: 17.4: Compare the main types of training. AACSB: Analytical thinking Difficulty: Hard Quest. Category: Synthesis
772 richard@qwconsultancy.com
55) Compare and contrast between on-the-job training and off-the-job training. Answer: On-the-job training methods include job rotation, apprenticeships, understudy assignments, and formal mentoring programs. But because they often disrupt the workplace, organizations also invest in off-the-job training. The most popular off-the-job training method is live classroom lectures. But it also encompasses public seminars, self-study programs, Internet courses, Webinars, podcasts, and group activities that use role-plays and case studies. LO: 17.4: Compare the main types of training. AACSB: Analytical thinking Difficulty: Moderate Quest. Category: Synthesis 56) Discuss e-learning. Answer: The fastest-growing training method is computer-based training, or e-training. Elearning systems emphasize learner control over the pace and content of instruction, allow elearners to interact through online communities, and incorporate other techniques such as simulations and group discussions. Computer-based training that lets learners actively participate in exercises and quizzes is more effective than traditional classroom instruction. Recent research has also highlighted the ways in which computer-based training can be improved by providing learners with regular prompts to set goals for learning, use effective study strategies, and measure progress toward their learning goals. LO: 17.4: Compare the main types of training. Difficulty: Moderate Quest. Category: Concept
773 richard@qwconsultancy.com
57) Imagine that you are a manager of a multifaceted manufacturing operation with a new crew of twenty arriving for training. The crew is comprised of different types of learners. Describe a training program comprised of three different sessions that would cater to four different individual learning styles. State whether each of the training sessions would be on-the-job or offthe-job training and whether it is formal or informal. Answer: In the first session, a good manager could introduce the skills needed for success in the manufacturing line through a short, formal, and direct lecture provided by management. During the lecture, a visual aid and reading material with the information could be distributed. Using this method would encompass the learning styles of the listener, the reader, and to a lesser degree the observer (with the visual aid). Although lecture styles have a poor reputation, evidence shows that they are surprisingly effective and would be a good introduction and icebreaker. Because this training is planned in advance and has a structured format, it would be a formal, on-the-job training. In the second session, the manager could assign small groups of trainees to follow a mentor for a day and observe the job while it is being performed. This formal, on-the-job training would best cater to the observer's learning style. At the end of the day, the mentor could allow each participant to perform part of the work, therefore catering to the participant's style. In the third session, the manager could assign the training class an off-the-job, e-learning course about safety. This would cater to the observer and listener. It would be formal, and yet flexible and not disruptive. LO: 17.4: Compare the main types of training. AACSB: Analytical thinking Difficulty: Moderate Quest. Category: Synthesis 58) Performance management focuses on ________. A) task performance, productivity, tenure B) productivity, efficiency, absenteeism C) job performance, organizational citizenship behavior, counterproductive work behavior D) citizenship, counterproductivity, personality E) leadership, knowledge, efficiency Answer: C Explanation: Performance management focuses on three primary areas (described in the introductory chapter): job performance, organizational citizenship behavior (OCB), and counterproductive work behavior (CWB). LO: 17.5: List the methods of performance management. Difficulty: Moderate Quest. Category: Concept
774 richard@qwconsultancy.com
59) Treating co-workers with respect, making constructive suggestions, and saying positive things about the workplace are a part of the ________ behavior that constitutes performance at work. A) citizenship B) counterproductivity C) social loafing D) deviant workplace E) satisficing Answer: A Explanation: Performance management focuses on three primary areas: job performance, organizational citizenship behavior (OCB), and counterproductive work behavior (CWB). Most managers believe that good performance means doing well on the first two dimensions and avoiding the third. A person who does core job tasks very well but is rude and aggressive toward coworkers is not going to be considered a good employee in most organizations, and the most pleasant and upbeat worker who cannot do the main job tasks well is not going to be a good employee either. LO: 17.5: List the methods of performance management. Difficulty: Moderate Quest. Category: Concept
775 richard@qwconsultancy.com
Sirius Corp, an automobile manufacturer in Texas, has a number of vacancies at lower management levels, which is important for the company's future development, and wants to fill the positions from within the company itself rather than recruit externally. The company plans to e-mail the job specifications to all employees and post the jobs on the company Web site. 60) Which of the following, if true, will strengthen the company's decision? A) The majority of the company employees have only a basic level of education. B) The company recently instituted a technical skills improvement program for shop floor employees. C) A top recruitment agency recently sent the company details of prospective college candidates. D) Several top performing entry level employees have received offers from rival companies. E) A majority of the company's employees are reaching the age of retirement. Answer: D Explanation: If the majority of the company employees have only a basic level of education, then the company needs to hire talent from external sources. This weakens their decision, as does the fact that a top recruitment agency recently sent the company details of prospective college candidates. If a majority of the company's employees are reaching the age of retirement, it is time for the company to hire younger employees. If the company recently instituted a technical skills improvement program for shop floor employees, it has no relevance to internal recruitment of employees from lower-level management. As several top performing employees have received offers from rival companies, the management has to consider internal recruitment due to fear of losing them. Through promotions, the company can prevent these employees from accepting a position elsewhere. LO: 17.5: List the methods of performance management. AACSB: Reflective thinking Difficulty: Hard Employability Skills: Critical Thinking Quest. Category: Critical Thinking
776 richard@qwconsultancy.com
61) Which of the following, if true, will weaken the company's decision? A) The majority of the company's employees have only a basic level of education. B) The company recently instituted a technical skills improvement program for shop floor employees. C) The existing shop floor employees have high potential and personal ambition to rise in their careers. D) Sirius Corp. just opened a new manufacturing facility in New Mexico. E) Several top performing entry level employees have received offers from rival companies. Answer: A Explanation: If the company recently instituted a technical skills improvement program for shop floor employees, it has no relevance to internal recruitment of employees from lower-level management. If the existing shop floor employees have high potential and personal ambition to rise in their careers, it has no relevance to the internal recruitment of entry level employees. Sirius Corp. opening a new facility is out of scope from the present decision. If several top performing employees have received offers from rival companies, the management has to consider internal recruitment due to fear of losing them, hence strengthening their decision. As the majority of the company employees have only a basic level of education, the company needs to hire talent from external sources. This weakens their decision. LO: 17.5: List the methods of performance management. AACSB: Reflective thinking Difficulty: Hard Employability Skills: Critical Thinking Quest. Category: Critical Thinking 62) Most managers believe that ________. A) strong citizenship behavior can overcome most counterproductive work behaviors B) counterproductive work behavior is not an issue in the workplace so long as the employee is friendly C) strong job performance and organizational citizenship behavior lead to good performance evaluations D) satisficing is generally enough for a good performance evaluation E) groupthink is linked to poor performance evaluations and should be avoided Answer: C Explanation: In essence, performance management focuses on three primary areas: job performance, organizational citizenship behavior (OCB), and counterproductive work behavior (CWB). Most managers believe that good performance means doing well on the first two dimensions and avoiding the third. LO: 17.5: List the methods of performance management. Difficulty: Moderate Quest. Category: Concept
777 richard@qwconsultancy.com
63) Performance evaluations are used to ________. A) improve group cohesiveness B) define departmental structure C) help management make HR decisions D) identify how jobs are completed E) decrease conformity within organizations Answer: C Explanation: Performance evaluation serves a number of purposes. One is to help management make general human resource decisions about promotions, transfers, and terminations. LO: 17.5: List the methods of performance management. Difficulty: Moderate Quest. Category: Concept 64) Most organizations use ________ to assess job performance even though they may not be highly correlated with positive task outcomes. A) traits B) task outcomes C) behaviors D) personality E) technical skills Answer: A Explanation: Most organizations use traits to assess job performance even though they may not be highly correlated with positive task outcomes. Having a good attitude, showing confidence, being dependable, looking busy, or possessing a wealth of experience may or may not be highly correlated with positive task outcomes. LO: 17.5: List the methods of performance management. Difficulty: Moderate Quest. Category: Concept 65) The approach to evaluation that uses feedback from those who have daily contact with an employee (everyone from mailroom personnel to customers to bosses to peers) is termed ________. A) critical incidents B) 360-degree evaluation C) assessment center D) multiperson comparisons E) downward evaluation Answer: B Explanation: The latest approach to performance evaluation is 360-degree evaluations. These provide performance feedback from the employee's full circle of daily contacts, from mailroom workers to customers to bosses to peers. LO: 17.5: List the methods of performance management. Difficulty: Easy Quest. Category: Concept
778 richard@qwconsultancy.com
66) Which performance evaluation method requires no complex forms or extensive training to complete? A) Written essays B) Critical incidents C) Graphic rating scales D) Behaviorally anchored rating scales E) Assessment center rating Answer: A Explanation: Probably the simplest method is to write a narrative describing an employee's strengths, weaknesses, past performance, potential, and suggestions for improvement. The written essay requires no complex forms or extensive training to complete. LO: 17.5: List the methods of performance management. Difficulty: Moderate Quest. Category: Concept 67) The ________ evaluation method focuses the evaluator's attention on those behaviors that are key to executing a job effectively. A) forced comparison B) critical incident C) graphic rating scale D) behaviorally anchored rating scale E) written essay Answer: B Explanation: Critical incidents focus the evaluator's attention on the difference between executing a job effectively and executing it ineffectively. The appraiser describes what the employee did in a situation that was especially effective or ineffective, citing only specific behaviors, not vaguely defined personality traits. LO: 17.5: List the methods of performance management. Difficulty: Easy Quest. Category: Concept 68) If a manager uses critical incidents as a method of performance evaluation, then ________. A) the employees generally receive poor ratings B) the evaluator's writing skills become the determining factor of the evaluation C) the employees are more likely to become motivated to achieve their goals D) the focus of the evaluation will center on specific behaviors E) the cost of the evaluation is likely to be incredibly high for the organization Answer: D Explanation: Critical incidents focus the evaluator's attention on the difference between executing a job effectively and executing it ineffectively. A list of such critical incidents provides a rich set of examples to show the employee desirable behaviors and those that call for improvement. LO: 17.5: List the methods of performance management. Difficulty: Moderate Quest. Category: Concept
779 richard@qwconsultancy.com
780 richard@qwconsultancy.com
69) The ________ method of performance evaluation allows for quantitative analysis and comparison. A) critical incident B) graphic rating scale C) assessment center evaluation D) written essay E) group order ranking Answer: B Explanation: Although they don't provide the depth of information that essays or critical incidents do, graphic rating scales are less time consuming to develop and administer and allow for quantitative analysis and comparison. LO: 17.5: List the methods of performance management. Difficulty: Easy Quest. Category: Concept 70) The ________ evaluation involves rating employees based on items on a continuum with the points reflecting actual behaviors on a given job. A) BARS B) critical incident C) graphic rating scale D) group order ranking E) assessment center evaluation Answer: A Explanation: Behaviorally anchored rating scales (BARS) combine major elements from the critical incident and graphic rating scale approaches. The appraiser rates the employees on items along a continuum, but the items are examples of actual behavior on the job rather than general descriptions or traits. LO: 17.5: List the methods of performance management. Difficulty: Easy Quest. Category: Concept 71) Which evaluation method involves evaluating one's performance against the performance of one or more others? A) BARS B) The critical incident method C) An assessment center evaluation D) Forced comparison E) Likert analysis Answer: D Explanation: Forced comparisons evaluate one individual's performance against the performance of another or others. It is a relative, rather than an absolute, measuring device. LO: 17.5: List the methods of performance management. Difficulty: Easy Quest. Category: Concept
781 richard@qwconsultancy.com
72) ________ ranking requires the evaluator to place employees into a particular classification, such as top one-fifth or second one-fifth. A) Individual B) Group order C) Paired D) Fractional E) Percentile Answer: B Explanation: Group order ranking requires the evaluator to place employees into a particular classification, such as top one-fifth or second one-fifth. If a rater has 20 employees, only four can be in the top fifth and, of course, four must also be relegated to the bottom fifth. This method is often used in recommending students to graduate schools. LO: 17.5: List the methods of performance management. Difficulty: Easy Quest. Category: Concept 73) The ________ performance evaluation rank-orders employees from best to worst. A) individual ranking B) group order ranking C) paired comparison D) straight ranking E) associative ranking Answer: A Explanation: The individual ranking approach rank-orders employees from best to worst. If the manager is required to appraise 30 employees, the difference between the 1st and 2nd employee is assumed to be the same as that between the 21st and 22nd. Some employees may be closely grouped, but no ties are permitted. The result is a clear ordering from the highest performer to the lowest. LO: 17.5: List the methods of performance management. Difficulty: Easy Quest. Category: Concept 74) The ________ performance evaluation method is often used in recommending students to graduate schools. A) written essays B) critical incident analysis C) BARS D) group order ranking E) graphic rating scales Answer: D Explanation: Group order ranking requires the evaluator to place employees into a particular classification, such as top one-fifth or second one-fifth. This method is often used in recommending students to graduate schools. LO: 17.5: List the methods of performance management. Difficulty: Easy Quest. Category: Concept 782 richard@qwconsultancy.com
75) Which of the following is a forced comparison method of performance evaluation? A) Written essays B) Trait evaluations C) Group order ranking D) Critical incident method E) Graphic rating scales Answer: C Explanation: The two most popular forced comparisons are group order ranking and individual ranking. Group order ranking requires the evaluator to place employees into a particular classification, such as top one-fifth or second one-fifth. This method is often used in recommending students to graduate schools. LO: 17.5: List the methods of performance management. Difficulty: Moderate Quest. Category: Concept 76) Jeeves Consulting requires a performance evaluation method that is less time consuming to develop and administer and allows for quantitative analysis and comparison. An appropriate evaluation technique for Jeeves Consulting would be ________. A) written essays B) trait evaluations C) forced comparisons D) 360-degree evaluations E) graphic rating scales Answer: E Explanation: One of the oldest and most popular methods of evaluation is graphic rating scales. Although they don't provide the depth of information that essays or critical incidents do, graphic rating scales are less time consuming to develop and administer and allow for quantitative analysis and comparison. LO: 17.5: List the methods of performance management. AACSB: Analytical thinking Difficulty: Moderate Employability Skills: Knowledge Application and Analysis Quest. Category: Application 77) The concept of ________ can be applied to appraisals to increase the perception that employees are treated fairly. A) selective evaluation B) due process C) multiple raters D) selective retention E) counterproductivity Answer: B Explanation: The concept of due process can be applied to appraisals to increase the perception that employees are being treated fairly. LO: 17.5: List the methods of performance management. 783 richard@qwconsultancy.com
Difficulty: Moderate Quest. Category: Concept 78) Marcel hates the annual review process he has to do for the employees in his department. Although he is fully aware of some of his employees' faults, he is reluctant to discuss them in person, and is only comfortable writing them in an unsigned report that his supervisor will review. Marcel's behavior demonstrates ________. A) provision of due process B) inflated assessment of employee performance C) organizational citizenship behavior D) managerial sincerity E) fear of confrontation Answer: E Explanation: Even though almost every employee could stand to improve in some areas, managers fear confrontation when presenting negative feedback. Marcel's reluctance to criticize suggests that he doesn't want to hurt anyone's feelings and that he fears confrontation. LO: 17.5: List the methods of performance management. AACSB: Analytical thinking Difficulty: Hard Employability Skills: Knowledge Application and Analysis Quest. Category: Application 79) Which performance evaluation method can be used to protect against legal liability or for purely authoritarian purposes? A) Graphic rating scales B) Electronic performance monitoring C) Behaviorally anchored rating scales D) Critical incidents E) Forced comparisons Answer: B Explanation: Although many organizations use EPM for performance appraisal, they also may use EPM to protect against legal liability (e.g., ensure safety), to provide developmental feedback on how to perform better, or, sometimes, without clear direction (e.g., for purely authoritarian purposes). In general, employees tend to react negatively to EPM, especially when it is real-time, not transparent (e.g., the employer is secretive), and invasive (e.g., collects too much data without reason). LO: 17.5: List the methods of performance management. Difficulty: Moderate Quest. Category: Concept
784 richard@qwconsultancy.com
80) Companies that want to capture attentional metrics such as how long employees stay logged into collaboration programs use ________ as a performance evaluation method. A) behaviorally anchored rating scales B) forced comparisons C) group order ranking D) electric performance monitoring E) individual ranking Answer: D Explanation: Some organizations use electronic performance monitoring (EPM) to capture objective indicators of employees' performance while on the job. This captures attentional metrics, such as how long employees stay logged into collaboration programs (e.g., Microsoft Teams), how many reports they file, and whether they "cyberloaf" (e.g., visiting shopping websites rather than working). LO: 17.5: List the methods of performance management. Difficulty: Moderate Quest. Category: Concept 81) Which of the following is true of performance feedback? A) Appraisals should be non-specific. B) The mood of the employee has little impact when providing performance feedback. C) The performance review should be a counseling activity rather than a judgment process. D) People tend to under-rate their own performance. E) It's easier to figure out how to improve your performance globally, than focusing on a specific area. Answer: C Explanation: The performance review should be a counseling activity more than a judgment process, best accomplished by allowing it to evolve from the employee's self-evaluation. LO: 17.5: List the methods of performance management. Difficulty: Moderate Quest. Category: Concept
785 richard@qwconsultancy.com
82) Cindy is the manager of a busy dental office. One of her responsibilities is to submit monthly reports showing efficiency data. Cindy creates the report after other staff members file their schedule information. During her annual performance evaluation, one of the owners of the practice commented that Cindy was sometimes late in submitting the reports. Cindy is being evaluated on ________. A) her behavior B) individual task outcomes C) traits D) primary responsibility E) system performance Answer: C Explanation: Cindy is being evaluated on her behavior. Measured behaviors need not be limited to those directly related to individual productivity. LO: 17.5: List the methods of performance management. AACSB: Analytical thinking Difficulty: Moderate Employability Skills: Knowledge Application and Analysis Quest. Category: Application 83) Performance management should take place on a scheduled basis once or twice a year. Answer: FALSE Explanation: Performance management should be a continuous process where HR managers identify, measure, and develop both individual and team performance that is aligned with the strategic goals of the organization. LO: 17.5: List the methods of performance management. Difficulty: Moderate Quest. Category: Concept 84) Performance evaluations identify training and development needs. Answer: TRUE Explanation: Performance evaluations identify training and development needs. LO: 17.5: List the methods of performance management. Difficulty: Moderate Quest. Category: Concept 85) The critical incidents method of performance evaluation focuses the evaluator's attention on the difference between executing a job effectively and executing it ineffectively. Answer: TRUE Explanation: Critical incidents focus the evaluator's attention on the difference between executing a job effectively and executing it ineffectively. The appraiser describes what the employee did in a situation that was especially effective or ineffective, citing only specific behaviors, not vaguely defined personality traits. A list of such critical incidents provides a rich set of examples to show the employee desirable behaviors and those that call for improvement. LO: 17.5: List the methods of performance management. Difficulty: Easy Quest. Category: Concept 786 richard@qwconsultancy.com
86) Graphic rating scales are less time consuming to develop and administer and allow for quantitative analysis and comparison. Answer: TRUE Explanation: Graphic rating scales are less time consuming to develop and administer and allow for quantitative analysis and comparison. LO: 17.5: List the methods of performance management. Difficulty: Moderate Quest. Category: Concept 87) The group order ranking method of performance evaluation combines major elements from the critical incident and graphic rating scale approaches. Answer: FALSE Explanation: Behaviorally anchored rating scales (BARS) combine major elements from the critical incident and graphic rating scale approaches. Group order ranking requires the evaluator to place employees into a particular classification. LO: 17.5: List the methods of performance management. Difficulty: Moderate Quest. Category: Concept 88) Due process systems provide individuals with adequate notice of what is expected of them. Answer: TRUE Explanation: The concept of due process can be applied to appraisals to increase the perception that employees are being treated fairly. Three features characterize due process systems: (1) individuals are provided with adequate notice of what is expected of them; (2) all evidence relevant to a proposed violation is aired in a fair hearing so the individuals affected can respond; and (3) the final decision is based on the evidence and free of bias. LO: 17.5: List the methods of performance management. Difficulty: Moderate Quest. Category: Concept 89) The performance review should be designed more as a mechanism for providing feedback and determining reward allocation. Answer: TRUE Explanation: The performance review should be designed more as a mechanism for providing feedback and determining reward allocation. LO: 17.5: List the methods of performance management. Difficulty: Moderate Quest. Category: Concept
787 richard@qwconsultancy.com
90) How can the use of multiple evaluators improve the performance evaluation process? Answer: As the number of evaluators increases, the probability of attaining more accurate information increases, as does the likelihood that the employee will accept the feedback as valid. We often see multiple evaluators in competitions in sports such as diving and gymnastics. A set of evaluators judges a performance, the highest and lowest scores are dropped, and the final evaluation is made up of those remaining. The logic of multiple evaluators applies to organizations as well. If an employee has ten supervisors, of whom nine rated them as excellent and one poor, we can safely discount the one poor evaluation. By moving employees within the organization to gain a number of evaluations, or by using multiple assessors (as in 360-degree appraisals), we increase the probability of achieving more valid and reliable evaluations. LO: 17.5: List the methods of performance management. Difficulty: Moderate Quest. Category: Concept 91) What is electronic performance monitoring and why might employees react negatively to it? Answer: Some organizations use electronic performance monitoring (EPM) to capture objective indicators of employees' performance while on the job. This captures attentional metrics, such as how long employees stay logged into collaboration programs (e.g., Microsoft Teams), how many reports they file, and whether they "cyberloaf" (e.g., visiting shopping websites rather than working). Although many organizations use EPM for performance appraisal, they also may use EPM to protect against legal liability (e.g., ensure safety), to provide developmental feedback on how to perform better, or, sometimes, without clear direction (e.g., for purely authoritarian purposes). In general, employees tend to react negatively to EPM, especially when it is real-time, not transparent (e.g., the employer is secretive), and invasive (e.g., collects too much data without reason). LO: 17.5: List the methods of performance management. Difficulty: Moderate Quest. Category: Concept 92) What purposes do performance evaluations serve in organizations? Answer: Performance evaluations serve a number of purposes. One is to help management make general human resource decisions about promotions, transfers, and terminations. Evaluations also identify training and development needs. They pinpoint employee skills and competencies for which remedial programs can be developed. Finally, they provide feedback to employees on how the organization views their performance and are often the basis for reward allocations, including merit pay increases. LO: 17.5: List the methods of performance management. Difficulty: Moderate Quest. Category: Concept
788 richard@qwconsultancy.com
93) Identify and explain the three most popular sets of criteria for evaluating employee performance. Answer: The three most popular sets of criteria for evaluating employee performance are individual task outcomes, behaviors, and traits. a) Task outcomes: If ends count, rather than means, then management should evaluate an employee's task outcomes. In many cases, it's difficult to identify specific outcomes that can be directly attributable to an employee's actions. b) Behaviors: It is difficult to attribute specific outcomes to the actions of employees in advisory or support positions or employees whose work assignments are part of a group effort. We may readily evaluate the group's performance, but if it is hard to identify the contribution of each group member, management will often evaluate the employee's behavior. c) Traits: The weakest set of criteria, yet one that is widely used by organizations, is individual traits. They are weaker because they are furthest removed from the actual performance of the job itself. LO: 17.5: List the methods of performance management. Difficulty: Moderate Quest. Category: Concept 94) How are written comments used as a means of performance evaluation? Answer: Probably the simplest method is to write a narrative describing an employee's strengths, weaknesses, past performance, potential, and suggestions for improvement. The written comment requires no complex forms or extensive training to complete. But, in this method, a useful appraisal may be determined as much by the evaluator's writing skill as by the employee's actual level of performance. It's also difficult to compare essays for different employees (or for the same employees written by different managers) because there is no standardized scoring key. LO: 17.5: List the methods of performance management. Difficulty: Moderate Quest. Category: Concept 95) What is the critical incidents method of performance evaluation? Answer: Critical incidents focus the evaluator's attention on the difference between executing a job effectively and executing it ineffectively. The appraiser describes what the employee did that was especially effective or ineffective in a situation, citing only specific behaviors. A list of such critical incidents provides a rich set of examples to show the employee desirable behaviors and those that call for improvement. LO: 17.5: List the methods of performance management. Difficulty: Moderate Quest. Category: Concept
789 richard@qwconsultancy.com
96) Identify and explain the two most popular forced comparison methods of performance evaluation. Answer: The two most popular comparisons are group order ranking and individual ranking. a) The group order ranking requires the evaluator to place employees into a particular classification, such as top one-fifth or second one-fifth. This method is often used in recommending students to graduate schools. b) The individual ranking approach rank-orders employees from best to worst. If the manager is required to appraise 30 employees, this approach assumes that the difference between the first and second employee is the same as that between the twenty-first and twenty-second. Even though some of the employees may be closely grouped, this approach allows for no ties. The result is a clear ordering of employees, from the highest performer down to the lowest. LO: 17.5: List the methods of performance management. Difficulty: Moderate Quest. Category: Concept 97) Provide some suggestions for improving performance evaluations. Answer: Although there are no protections that will guarantee accurate performance evaluations, the following suggestions can significantly help to make the process more objective and fairer. a) The use of multiple evaluators is recommended. As the number of evaluators increases, the probability of attaining more accurate information increases. By moving employees about within the organization so as to gain a number of evaluations or by using multiple assessors, we increase the probability of achieving more valid and reliable evaluations. b) Appraisers should evaluate only those areas in which they have some expertise. The more levels that separate them, the less opportunity the evaluator has to observe the individual's behavior and, not surprisingly, the greater the possibility for inaccuracies. c) In addition, evaluators should be trained. If you can't find good evaluators, the alternative is to make good evaluators. There is substantial evidence that training evaluators can make them more accurate raters. d) Finally, employees should be provided with due process. The concept of due process can be applied to appraisals to increase the perception that employees are treated fairly. Three features characterize due process systems: i. Individuals are provided with adequate notice of what is expected of them. ii. All evidence relevant to a proposed violation is aired in a fair hearing so the individuals affected can respond; and iii. The final decision is based on the evidence and free of bias. LO: 17.5: List the methods of performance management. Difficulty: Hard Quest. Category: Concept
790 richard@qwconsultancy.com
98) Sophia is a valued employee in your department. She is positive and dependable but lacks ambition and has stagnated in her position this year. She also is consistently late on deadlines, although the work she turns in is usually flawless. Choose one of the performance evaluation methods discussed in the chapter and use it to evaluate the three most popular sets of criteria for Sophia. Explain why your evaluation method is strong or weak and how it could be improved. Answer: The simplest evaluation method is to write a narrative describing an employee's strengths, weaknesses, past performance, potential, and suggestions for improvement. The three most popular sets of criteria are individual task outcomes, behaviors, and traits. Task outcomes: Sophia is a valued employee who consistently turns in high quality work. Her work requires little quality assurance and, therefore, I can rely on Sophia when I know that there will not be a lot of time to recheck the work. Behaviors: Sophia is dependable in completing her work, although she is often late with the product. She has recently become stagnant in her position and shows little ambition. Traits: Sophia is a positive and upbeat team player, helping to create a good work environment. And, as I said earlier, she is very dependable at completing her work, although with some delay. This evaluation is a general evaluation of Sophia, but it is only one person's opinion and depends on the person's ability to express Sophia's strengths and weaknesses. It doesn't provide any methods for improvement and is evaluated on a global job performance, rather than job specificity. LO: 17.5: List the methods of performance management. AACSB: Analytical thinking Difficulty: Moderate Quest. Category: Synthesis 99) What is the purpose of performance evaluation? Answer: Performance evaluation serves a number of purposes. One is to help management make general human resources decisions about promotions, transfers, and terminations. Evaluations also identify training and development needs. They pinpoint employee skills and competencies for which remedial programs can be developed. Finally, they provide feedback to employees on how the organization views their performance and are often the basis for reward allocations, including merit pay increases. LO: 17.5: List the methods of performance management. Difficulty: Moderate Quest. Category: Concept
791 richard@qwconsultancy.com
100) Representation of individuals with physical or mental disabilities in the workforce ________. A) is limited by health and safety regulations B) is nearly 100 percent in countries like Canada and New Zealand C) increased in the United States with the passage of the ADA D) demands the use of consistent performance evaluation to be legal E) is facilitated with economic feasibility studies Answer: C Explanation: In the United States, the representation of individuals with disabilities in the workforce rapidly increased with the passage of the ADA in 1990. LO: 17.6: Discuss how reasonable accommodations make accessible workplaces. AACSB: Diverse and multicultural work environments Difficulty: Moderate Quest. Category: Concept 101) Which of the following is true of accommodations for physical disabilities in the workplace? A) Workers with disabilities should always receive higher performance evaluations. B) Accommodations processes should be a transactional determination. C) Government mandates to hire more people with disabilities significantly increased their representation in the workplace in Europe. D) Workloads are notably heavier when an employer hires people with disabilities. E) Germany and France tried to use quota systems to motivate employers to hire individuals with disabilities. Answer: E Explanation: In Europe, policies to motivate employers have often failed to boost the workforce participation rate for workers with disabilities, and outright quota systems in Germany, France, and Poland have backfired LO: 17.6: Discuss how reasonable accommodations make accessible workplaces. AACSB: Diverse and multicultural work environments Difficulty: Moderate Quest. Category: Concept
792 richard@qwconsultancy.com
102) Which of the following is an example of an observable disability? A) Autoimmune disorders B) PTSD C) ADHD D) Insomnia E) Blindness Answer: E Explanation: Disabilities include observable characteristics like missing limbs, illnesses that require a person to use a wheelchair, and blindness. Other disabilities may not be directly observable. Unless an individual decides to disclose a disability that is not easily observable, it can remain hidden at the discretion of the employee. These are called hidden disabilities (or invisible disabilities). Hidden disabilities generally fall under the categories of sensory disabilities (for example, being hard of hearing), autoimmune disorders (like rheumatoid arthritis), chronic illness or pain (like carpal tunnel syndrome), cognitive or learning disabilities (like attention deficit hyperactivity disorder [ADHD]), sleep disorders (like insomnia), and psychological challenges (like PTSD). Mental disabilities may affect performance more than physical disabilities: Individuals with common mental health issues such as depression and anxiety are significantly more likely to be absent from work. LO: 17.6: Discuss how reasonable accommodations make accessible workplaces. AACSB: Diverse and multicultural work environments Difficulty: Easy Quest. Category: Concept 103) Which of the following is an example of a hidden disability that is not truly invisible? A) ASD B) Blindness C) Missing limbs D) Illness that require the use of a wheelchair E) Paralysis Answer: A Explanation: In some ways, a hidden disability is not truly invisible. A person with an undisclosed diagnosis of autism spectrum disorder (ASD) will still exhibit the behaviors characteristic of the condition, such as difficulty with verbal communication and lack of adaptability. You may observe behavior that leads you to suspect an individual has a hidden disability. Unfortunately, you may attribute the behavior to other causes–you may incorrectly ascribe the slow, slurred speech of a coworker to an alcohol dependency rather than to the longterm effects of a stroke. LO: 17.6: Discuss how reasonable accommodations make accessible workplaces. AACSB: Diverse and multicultural work environments Difficulty: Moderate Quest. Category: Application
793 richard@qwconsultancy.com
104) Under the Americans with Disabilities Act Amendments Act (ADAAA), employees must disclose their conditions to their employers in order to be eligible for workplace accommodations and employment protection. Answer: TRUE Explanation: As a result of changes to the Americans with Disabilities Act Amendments Act (ADAAA) of 2008, U.S. organizations must accommodate employees with a broad range of disabilities. However, employees must disclose their conditions to their employers in order to be eligible for workplace accommodations and employment protection. LO: 17.6: Discuss how reasonable accommodations make accessible workplaces. AACSB: Diverse and multicultural work environments Difficulty: Moderate Quest. Category: Concept 105) Countries such as Canada, The Netherlands, New Zealand, Syria, and Mali have specific laws to protect individuals with disabilities. Answer: TRUE Explanation: Countries such as Canada, The Netherlands, New Zealand, Syria, and Mali have specific laws to protect individuals with disabilities. LO: 17.6: Discuss how reasonable accommodations make accessible workplaces. AACSB: Diverse and multicultural work environments Difficulty: Moderate Quest. Category: Concept 106) The EEOC classifies a person as having a disability who has any physical or mental disability that substantially limits one or more major life activities. Answer: TRUE Explanation: The EEOC classifies a person as having a disability who has any physical or mental disability that substantially limits one or more major life activities. LO: 17.6: Discuss how reasonable accommodations make accessible workplaces. AACSB: Diverse and multicultural work environments Difficulty: Easy Quest. Category: Concept 107) Disclosing disabilities may increase job satisfaction and well-being of the individual. Answer: TRUE Explanation: Disclosing disabilities may increase job satisfaction and well-being of the individual, help others understand and assist the individual to succeed in the workplace, and allow the organization to accommodate the situation so that the employee and organization achieve top performance. LO: 17.6: Discuss how reasonable accommodations make accessible workplaces. AACSB: Diverse and multicultural work environments Difficulty: Moderate Quest. Category: Concept
794 richard@qwconsultancy.com
108) Why do some people who qualify for workplace accommodations under the Americans with Disabilities Act Amendment Act fail to get them? Answer: As a result of changes to the Americans with Disabilities Act Amendments Act (ADAAA) of 2008, U.S. organizations must accommodate employees with a broad range of disabilities. However, employees must disclose their conditions to their employers in order to be eligible for workplace accommodations and employment protection. Many employees do not want to disclose their invisible disabilities, so they are prevented from getting the workplace accommodations they need in order to thrive in their jobs. Research indicates that individuals with hidden disabilities are afraid of being stigmatized or ostracized if they disclose their disabilities to others in the workplace, and they believe that their managers will think they are less capable of strong job performance. Add this to the challenge of receiving a diagnosis for a condition that one did not previously have, and these fears are compounded even more so than if the diagnosis was made for employees when they were younger. LO: 17.6: Discuss how reasonable accommodations make accessible workplaces. AACSB: Diverse and multicultural work environments Difficulty: Moderate Quest. Category: Concept 109) A group of human resources practices that work together and reinforce one another to improve organizational outcomes is known as a high-performance work system. Answer: TRUE Explanation: A group of human resources practices that work together and reinforce one another to improve organizational outcomes is known as a high-performance work system. LO: 17.7: Describe the leadership role of human resources (HR) in organizations. Difficulty: Easy Quest. Category: Concept 110) Employment policies differ from benefits in that they provide the guidelines for behavior, not just the working conditions. Answer: TRUE Explanation: Policies differ from benefits in that they provide the guidelines for behavior, not just the working conditions. A company might provide the benefit of a special break room for mothers of young children, but a policy is needed to outline the expectation for conduct. May mothers elect to feed their babies in other places in the facility or only in the break room? What timing is acceptable? Where can collected breast milk be stored? Establishing policies to address potential questions can help minimize confusion for all employees. LO: 17.7: Describe the leadership role of human resources (HR) in organizations. Difficulty: Moderate Quest. Category: Concept Organizational Behavior, 19e (Robbins/Judge) Chapter 18 Stress and Health in Organizations 1) The highest source of stress for young adults is ________. A) family responsibilities B) social life C) work 795 richard@qwconsultancy.com
D) health concerns E) money Answer: E Explanation: The top sources causing stress in young people are money (78%), work (64%), housing costs (52%), job stability (51%), and the economy (44%). LO: 18.1: Describe how the stress process unfolds in the workplace. Difficulty: Moderate Quest. Category: Concept 2) The generally unpleasant perception and appraisal of stressors is ________. A) strain B) stress C) tension D) anxiety E) pressure Answer: B Explanation: The generally unpleasant perception and appraisal of stressors is stress. LO: 18.1: Describe how the stress process unfolds in the workplace. Difficulty: Easy Quest. Category: Concept 3) Which of the following is a condition or event that an individual perceives as challenging or threatening? A) Tension B) Trauma C) Strain D) Anxiety E) Stressors Answer: E Explanation: Stressors refer to conditions or events that an individual perceives as challenging or threatening. LO: 18.1: Describe how the stress process unfolds in the workplace. Difficulty: Easy Quest. Category: Concept
796 richard@qwconsultancy.com
4) An individual experiencing the psychological, physiological, and behavioral consequences of stress is experiencing ________. A) trauma B) strain C) anxiety D) tension E) stress Answer: B Explanation: An individual experiencing the psychological, physiological, and behavioral consequences of stress is experiencing strain. LO: 18.1: Describe how the stress process unfolds in the workplace. Difficulty: Easy Quest. Category: Concept 5) Mike has been feeling overloaded at work. Each time he thinks he is finally catching up, his boss hands him another urgent task. Mike is experiencing ________. A) challenge stressors B) workload anxiety C) employment strain D) hindrance stressors E) psychological tension Answer: A Explanation: Mike is experiencing challenge stressors. Challenge stressors are stressors associated with workload, pressure to complete tasks, and time urgency. LO: 18.1: Describe how the stress process unfolds in the workplace. AACSB: Analytical thinking Difficulty: Moderate Employability Skills: Knowledge Application and Analysis Quest. Category: Application 6) Red tape is an example of ________. A) a hindrance stressor B) a challenge stressor C) stress D) strain E) tension Answer: A Explanation: Red tape is an example of a hindrance stressor. Hindrance stressors keep you from reaching your goals. LO: 18.1: Describe how the stress process unfolds in the workplace. Difficulty: Moderate Quest. Category: Concept
797 richard@qwconsultancy.com
7) Enrique is tired of dealing with office politics. No matter what he does, he feels that office politics stand in the way of his ability to reach his goals. Enrique is facing ________. A) workplace anxiety B) PTSD C) hindrance stressors D) strain E) challenge stressors Answer: C Explanation: Enrique is facing hindrance stressors. Hindrance stressors keep you from reaching your goals. LO: 18.1: Describe how the stress process unfolds in the workplace. AACSB: Analytical thinking Difficulty: Moderate Employability Skills: Knowledge Application and Analysis Quest. Category: Application 8) A manager who is finding it difficult to reach his goals because he is unsure of his job responsibilities is experiencing ________. A) challenge stressors B) spillover effects C) environmental uncertainty D) hindrance stressors E) interpersonal demands Answer: D Explanation: The manager is experiencing hindrance stressors. Hindrance stressors keep you from reaching your goals. Red tape, office politics, and confusion over job responsibilities are all examples of hindrance stressors. LO: 18.1: Describe how the stress process unfolds in the workplace. Difficulty: Moderate Quest. Category: Concept 9) A physiological symptom of stress is ________. A) anxiety B) reduced job satisfaction C) higher job performance D) illness E) increased turnover Answer: D Explanation: Physiological symptom of stress include immediate effects, illness, and chronic health conditions. LO: 18.1: Describe how the stress process unfolds in the workplace. Difficulty: Moderate Quest. Category: Concept
798 richard@qwconsultancy.com
10) As compared to hindrance stressors, challenge stressors lead to ________. A) more engagement B) reduced safety compliance C) decreased motivation D) decreased performance E) reduced organizational commitment Answer: A Explanation: Challenge stressors lead to more motivation, engagement, and performance than hindrance stressors. On the other hand, hindrance stressors appear to negatively affect safety compliance and participation, employee engagement, job satisfaction, organizational commitment, performance, and withdrawal than do challenge stressors. LO: 18.1: Describe how the stress process unfolds in the workplace. Difficulty: Moderate Quest. Category: Concept 11) A behavioral symptom of stress is ________. A) chronic health conditions B) lower emotional well-being C) illness D) lower job satisfaction E) higher turnover Answer: E Explanation: Behavioral symptoms of stress include lower job performance, higher absenteeism, and higher turnover. LO: 18.1: Describe how the stress process unfolds in the workplace. Difficulty: Moderate Quest. Category: Concept 12) A personal factor causing stress is ________. A) economic problems B) economic uncertainty C) technological change D) political uncertainty E) role demands Answer: A Explanation: Personal factors causing stress include family problems and economic problems. LO: 18.1: Describe how the stress process unfolds in the workplace. Difficulty: Moderate Quest. Category: Concept
799 richard@qwconsultancy.com
13) Changes in the business cycle create ________. A) group inertia B) selective information processing C) political uncertainty D) structural inertia E) economic uncertainties Answer: E Explanation: Changes in the business environment create economic uncertainty. LO: 18.1: Describe how the stress process unfolds in the workplace. Difficulty: Moderate Quest. Category: Concept 14) ________ relate to a person's job and include the design of the job and working conditions. A) Task demands B) Challenge stressors C) Hindrance stressors D) Role demands E) Interpersonal demands Answer: A Explanation: Task demands relate to a person's job and include the design of the job, working conditions, and physical work layout. LO: 18.1: Describe how the stress process unfolds in the workplace. Difficulty: Moderate Quest. Category: Concept 15) As a result of changing technology and unstable economic environment, your company is planning to undertake restructuring of the business. The employees in your company are worried about this upcoming change and many of them are anxious due to a fear that they will lose their jobs as a result of this change. These changes are stressors for employees. What is the likely source of those stressors? A) Personal factors B) Environmental stressors C) Organic factors D) Organizational factors E) Global factors Answer: B Explanation: The likely source is environmental factors. Environmental factors include economic and political uncertainty. LO: 18.1: Describe how the stress process unfolds in the workplace. Difficulty: Moderate Quest. Category: Concept
800 richard@qwconsultancy.com
16) High blood pressure, ulcers, irritability, and changes in appetite are all examples of ________. A) hypertension B) strain C) anxiety D) trauma E) stressors Answer: B Explanation: Stress manifests as strain in many ways, such as high blood pressure, ulcers, irritability, difficulty making routine decisions, changes in appetite, accident proneness, and the like. These symptoms fit under three general categories: physiological, psychological, and behavioral symptoms. LO: 18.1: Describe how the stress process unfolds in the workplace. Difficulty: Moderate Quest. Category: Concept 17) A healthy, positive, and constructive appraisal of stressors is referred to as ________. A) eustress B) hindrance stressors C) positive anxiety D) affirmative tension E) inverse hypertension Answer: A Explanation: A healthy, positive, and constructive appraisal of stressors is referred to as eustress. LO: 18.1: Describe how the stress process unfolds in the workplace. Difficulty: Moderate Quest. Category: Concept 18) One theory of the relationship between stress and job performance is that the relationship is ________ whereby low to moderate amounts of stress stimulate the body and lead to higher performance, but too much stress results in lower performance. A) T-shaped B) circular C) in the shape of an inverted-U D) parallel E) L-shaped Answer: C Explanation: A significant amount of research has investigated the positive aspects of stress. One proposed pattern of this relationship is the inverted-U (see Exhibit 18-3). The logic underlying the figure is that low to moderate stress levels stimulate the body and increase its ability to react. Individuals may perform tasks better, more intensely, or more rapidly. But too much stress places impossible demands on a person that results in lower performance. Despite its popularity and intuitive appeal, the inverted-U model has not earned a lot of empirical support. LO: 18.1: Describe how the stress process unfolds in the workplace. Difficulty: Moderate 801 richard@qwconsultancy.com
Quest. Category: Concept 19) The inverted-U model showing the relationship between stress and job performance suggests that when stress is ________, performance will be ________. A) lowest; highest B) highest; highest C) lowest; moderate D) highest; moderate E) moderate; highest Answer: E Explanation: The inverted-U model showing the relationship between stress and job performance suggests that when stress is moderate, performance will be highest. LO: 18.1: Describe how the stress process unfolds in the workplace. Difficulty: Hard Quest. Category: Concept 20) ________ is a generally unpleasant perception and appraisal of stressors. A) Change B) Stress C) Innovation D) Stimulation E) Creativity Answer: B Explanation: Stress is a generally unpleasant perception and appraisal of stressors. LO: 18.1: Describe how the stress process unfolds in the workplace. Difficulty: Easy Quest. Category: Concept 21) Which of the following stressors is likely to produce less strain than the other stressors? A) Confusion over job responsibilities B) Office politics C) Lack of unity of command D) Red tape E) Pressure to complete tasks Answer: E Explanation: Pressure to complete tasks is a challenge stressor whereas red tape, office politics, lack of unity of command, and confusion over job responsibilities are hindrance stressors. Although research is just starting to accumulate, early evidence suggests challenge stressors produce less strain than hindrance stressors. LO: 18.1: Describe how the stress process unfolds in the workplace. Difficulty: Moderate Quest. Category: Concept
802 richard@qwconsultancy.com
22) Political uncertainties and economic uncertainties are examples of the ________ stressors of potential stress. A) structural B) environmental C) organizational D) personal E) social Answer: B Explanation: Economic uncertainty and political uncertainty are environmental stressors. LO: 18.1: Describe how the stress process unfolds in the workplace. Difficulty: Easy Quest. Category: Concept 23) Jerry is feeling very stressed because his boss expects a project to be delivered to the client within a very unreasonable deadline. Jerry has to work nights and weekends to make the deadline. Jerry is feeling stress due to a(n) ________ factor. A) environmental B) organizational C) economic D) structural E) personal Answer: B Explanation: Organizational stress factors include pressures to avoid errors or complete tasks in a limited time, work overload, a demanding and insensitive boss, and unpleasant co-workers. LO: 18.1: Describe how the stress process unfolds in the workplace. AACSB: Analytical thinking Difficulty: Moderate Quest. Category: Application 24) Family issues, economic problems, and family issues are examples of the ________ factor of potential stress. A) social B) environmental C) structural D) psychological E) personal Answer: E Explanation: The personal category of sources of stress at work includes factors of an employee's personal life: family issues and personal economic problems. LO: 18.1: Describe how the stress process unfolds in the workplace. Difficulty: Moderate Quest. Category: Concept
803 richard@qwconsultancy.com
25) Rudolf is dealing with impending layoffs at work. Yesterday, at a pharmacy, he measured his blood pressure, and it was high. He also notices that when he thinks about the future, his heart rate accelerates. Rudolf is demonstrating ________ symptoms of stress. A) psychological B) physiological C) behavioral D) personal E) reactive Answer: B Explanation: Specialists in the health and medical sciences have determined that stress can cause physiological symptoms such as changes in metabolism, increased heart and breathing rates, increased blood pressure, and headaches. Stress can even induce heart attacks. LO: 18.1: Describe how the stress process unfolds in the workplace. AACSB: Analytical thinking Difficulty: Moderate Employability Skills: Knowledge Application and Analysis Quest. Category: Application 26) Excessive smoking, substance abuse, and sleep disorders are all examples of ________ symptoms of stress. A) personal B) psychological C) behavioral D) physiological E) pathological Answer: C Explanation: Behavior-related stress symptoms include changes in productivity, absence, and turnover, as well as changes in eating habits, increased smoking or consumption of alcohol, rapid speech, fidgeting, and sleep disorders. LO: 18.1: Describe how the stress process unfolds in the workplace. Difficulty: Moderate Quest. Category: Concept 27) Which of the following types of jobs is least likely to create stress and reduce satisfaction? A) Jobs that make multiple and conflicting demands B) Jobs that provide a high level of feedback C) Jobs that lack clarity about the employee's duties D) Jobs that provide the employees less control over the pace of their work E) Jobs that provide a low level of variety Answer: B Explanation: Jobs that provide a low level of variety, significance, autonomy, feedback, and identity appear to create stress and reduce satisfaction and involvement in the job. Jobs that make multiple and conflicting demands or that lack clarity about the employee's duties, authority, and responsibilities increase both stress and dissatisfaction. LO: 18.1: Describe how the stress process unfolds in the workplace. Difficulty: Moderate 804 richard@qwconsultancy.com
Quest. Category: Concept 28) Most people respond to stressors in similar ways. Answer: FALSE Explanation: Not everyone becomes "stressed out" by the same things, and not everyone responds in the same way. LO: 18.1: Describe how the stress process unfolds in the workplace. Difficulty: Moderate Quest. Category: Concept 29) Hindrance stressors lead to strain, but challenge stressors do not. Answer: FALSE Explanation: Evidence suggests that both challenge and hindrance stressors lead to strain, although hindrance stressors lead to increased strain levels. LO: 18.1: Describe how the stress process unfolds in the workplace. Difficulty: Moderate Quest. Category: Concept 30) Challenge stressors improve job performance in a supportive work environment, whereas hindrance stress reduces job performance in all work environments. Answer: TRUE Explanation: Challenge stressors improve job performance in a supportive work environment, whereas hindrance stress reduces job performance in all work environments. LO: 18.1: Describe how the stress process unfolds in the workplace. Difficulty: Moderate Quest. Category: Concept 31) The quality and quantity of sleep an individual gets can adversely affect the amount of strain they experience. Answer: TRUE Explanation: The quality and quantity of sleep an individual gets can adversely affect the amount of strain they experience. LO: 18.1: Describe how the stress process unfolds in the workplace. Difficulty: Moderate Quest. Category: Concept 32) If an employee is already experiencing stress from one source, the employee's level of stress will not change if the employee experiences another source of stress. Answer: FALSE Explanation: When we review stressors individually, it is easy to overlook that stress is an additive phenomenon—it builds up. For example, when you experience incivility from customers or coworkers, this can lead you to have a poor night's sleep and experience additional strain beyond the incivility you just experienced. Each new and persistent stressor adds to an individual's stress level. So, a single stressor may be relatively unimportant in and of itself. Still, if added to an already high level of stress, it can be too much. LO: 18.1: Describe how the stress process unfolds in the workplace. 805 richard@qwconsultancy.com
Difficulty: Moderate Quest. Category: Concept 33) The inverted-U relationship between stress and job performance provides a popular and empirically well-supported understanding of the positive aspects of stress. Answer: FALSE Explanation: Despite its popularity and intuitive appeal, the inverted-U model has not earned a lot of empirical support. LO: 18.1: Describe how the stress process unfolds in the workplace. Difficulty: Moderate Quest. Category: Concept 34) Eustress refers to a healthy, positive, and constructive appraisal of stressors. Answer: TRUE Explanation: Eustress refers to a healthy, positive, and constructive appraisal of stressors. LO: 18.1: Describe how the stress process unfolds in the workplace. Difficulty: Easy Quest. Category: Concept 35) What are the similarities and differences between stress, stressors, and strain? Answer: We distinguish between stressors, stress, and strain Stress is defined as generally unpleasant perception and appraisal of stressors. Stressors refer to conditions or events that an individual perceives as challenging or threatening. In essence, stressors are the antecedents to the experience of stress. An upcoming project deadline can be perceived as a challenging or threatening stressor to employees. Then, as a result of stress, an individual experiences, that project deadline and the stress that it causes can lead to strain–for instance, anxiety, lack of sleep, and procrastination. Therefore, strain encompasses the consequences of the experience of stress. LO: 18.1: Describe how the stress process unfolds in the workplace. Difficulty: Moderate Quest. Category: Concept 36) Compare and contrast hindrance stressors and challenge stressors. Answer: Hindrance stressors keep you from reaching your goals. Red tape, office politics, and confusion over job responsibilities are all examples of hindrance stressors. Challenge stressors are associated with workload, pressures to complete tasks, and time urgency. LO: 18.1: Describe how the stress process unfolds in the workplace. Difficulty: Moderate Quest. Category: Concept 37) What are the three sources of stressors? Provide an example of each. Answer: Stressors can come from environmental, organizational, or personal sources. Environmental factors include economic and political uncertainty and technological change. Organizational sources include task, role, and interpersonal demands. Personal factors include family and economic problems. LO: 18.1: Describe how the stress process unfolds in the workplace. Difficulty: Moderate 806 richard@qwconsultancy.com
Quest. Category: Concept
807 richard@qwconsultancy.com
38) Explain what occurs to the body during when an individual experiences eustress. Answer: Although stress is typically discussed in a negative context, it also has a positive purpose. In response to stress, your nervous system, hypothalamus, pituitary, and adrenal glands supply you with stress hormones to cope. Your heartbeat and breathing accelerate to increase oxygen while your muscles tense for action. This is a time when stress offers potential gain. Many professionals see the pressures of heavy workloads and deadlines as positive challenges that enhance the quality of their work and the satisfaction they get from their job. This form of stress has been labeled as eustress, or a healthy, positive, and constructive appraisal of stressors. LO: 18.1: Describe how the stress process unfolds in the workplace. Difficulty: Moderate Quest. Category: Concept 39) Explain the proposed inverted U-shaped relationship between stress and job performance. Answer: A significant amount of research has investigated the positive aspects of stress. One proposed pattern of this relationship is the inverted-U (see Exhibit 18-3). The logic underlying the figure is that low to moderate stress levels stimulate the body and increase its ability to react. Individuals may perform tasks better, more intensely, or more rapidly. But too much stress places impossible demands on a person that results in lower performance. Despite its popularity and intuitive appeal, the inverted-U model has not earned a lot of empirical support. LO: 18.1: Describe how the stress process unfolds in the workplace. Difficulty: Moderate Quest. Category: Concept 40) Compare and contrast challenge stressors and hindrance stressors. Answer: Researchers have argued that challenge stressors—or stressors associated with workload, pressure to complete tasks, and time urgency—operate quite differently from hindrance stressors—or stressors that keep you from reaching your goals (for example, red tape, office politics, confusion over job responsibilities). Although research is just starting to accumulate, early evidence suggests challenge stressors produce less strain than hindrance stressors. A meta-analysis of responses from more than 35,000 individuals showed role ambiguity, role conflict, role overload, job insecurity, environmental uncertainty, and situational constraints were all consistently negatively related to job performance. There is also evidence that challenge stress improves job performance in a supportive work environment, whereas hindrance stress reduces job performance in all work environments. LO: 18.1: Describe how the stress process unfolds in the workplace. AACSB: Analytical thinking Difficulty: Moderate Quest. Category: Synthesis
808 richard@qwconsultancy.com
41) Discuss the psychological symptoms of stress. Answer: Job dissatisfaction is an obvious cause of stress. But stress shows itself in other psychological states—for instance, tension, anxiety, irritability, boredom, and procrastination. One study that tracked physiological responses of employees over time found that stress due to high workloads was related to lower emotional well-being. Jobs that make multiple and conflicting demands or that lack clarity about the employee's duties, authority, and responsibilities increase both stress and dissatisfaction. Similarly, the less control people have over the pace of their work, the greater their stress and dissatisfaction, a finding that has been replicated across 63 countries. Jobs that provide a low level of variety, significance, autonomy, feedback, and identity appear to create stress and reduce satisfaction and involvement in the job. Not everyone reacts to autonomy in the same way, however. For those with an external locus of control, increased job control increases the tendency to experience stress and exhaustion. LO: 18.1: Describe how the stress process unfolds in the workplace. AACSB: Analytical thinking Difficulty: Moderate Quest. Category: Synthesis 42) Which of the following is least likely to negatively affect sleep quality? A) Role conflict B) Poor leadership C) Perceived discrimination D) Unfinished work tasks E) Finished work tasks Answer: E Explanation: Various work-related stressors have been shown to impair sleep quality; such stressors include unfinished work tasks, social stressors, poor leadership, perceived discrimination, and role conflict. LO: 18.2: Discuss how sleep, illness, and injury affect physical health at work. Difficulty: Moderate Quest. Category: Concept 43) The act of working while ill or injured is called ________. A) truancy B) malingering C) skiving D) absenteeism E) presenteeism Answer: E Explanation: The act of working while ill or injured is called presenteeism. LO: 18.2: Discuss how sleep, illness, and injury affect physical health at work. Difficulty: Easy Quest. Category: Concept
809 richard@qwconsultancy.com
44) Most people spend about ________ of their daily waking lives at work. A) one-tenth B) one-half C) one-third D) one-fourth E) three-quarters Answer: C Explanation: Most people spend about one-third of their daily waking lives at work. LO: 18.2: Discuss how sleep, illness, and injury affect physical health at work. Difficulty: Moderate Quest. Category: Concept 45) Roger feels as though he can never catch up at work. His boss has been yelling at him because he keeps missing deadlines. Roger has been staying late at work, smoking and eating to help him stay awake while he tries to meet his boss' expectations. Which might Roger be experiencing as a result of his work situation? A) Increased job satisfaction B) Obesity C) Better sleep D) Coercion E) Decreased absenteeism Answer: B Explanation: Roger might be experiencing obesity as a result of his work situation. Although there may be a direct link between what we experience at work and our health, it is more likely that people behave in unhealthy ways to cope with stressors at work. Employees may skip lunch, eat convenient and unhealthy foods, smoke cigarettes, neglect their sleep needs, or abuse substances to cope with stress at work. Research shows that these behaviors are linked to many metabolic risk factors associated with increased risk for cardiovascular and immune system diseases. More specifically, the regular experience of hindrance stressors at work resulted in moderate increases in high-risk eating and cigarette smoking, leading to increases in worrisome, objective health indicators such as body mass index (BMI), cholesterol levels, and insulin resistance. These metabolic risk factors can lead to health conditions such as obesity, resulting in increased company health costs, the risk for absenteeism and injury, and even prejudice and discrimination. Stress at work can also lead individuals to injury, as they start to neglect safety behavior. LO: 18.2: Discuss how sleep, illness, and injury affect physical health at work. AACSB: Analytical thinking Difficulty: Moderate Employability Skills: Knowledge Application and Analysis Quest. Category: Application
810 richard@qwconsultancy.com
46) Research suggests that what we do in the workplace is just as much of a risk factor for illness as is secondhand smoke. Answer: TRUE Explanation: Research suggests that what we do in the workplace is just as much of a risk factor for illness as is secondhand smoke. LO: 18.2: Discuss how sleep, illness, and injury affect physical health at work. Difficulty: Moderate Quest. Category: Concept 47) Experiencing hindrance stressors in the workplace can lead to obesity. Answer: TRUE Explanation: Research found that the regular experience of hindrance stressors at work resulted in moderate increases in high-risk eating and cigarette smoking, leading to increases in worrisome, objective health indicators such as body mass index (BMI), cholesterol levels, and insulin resistance. These metabolic risk factors can lead to health conditions such as obesity, resulting in increased company health costs, the risk for absenteeism and injury, and even prejudice and discrimination. LO: 18.2: Discuss how sleep, illness, and injury affect physical health at work. Difficulty: Moderate Quest. Category: Concept 48) What is presenteeism? Answer: OB professionals refer to working while ill or injured as presenteeism. Going into work while sick is not optimal for obvious reasons. First, working close to others while sick with a contagious disease increases the risk that other team members will also become ill (and will likely increase their anxieties about working with you, especially during COVID -19, people are not as effective at doing their jobs when ill or injured and can slow work processes. This effect will be even greater if the illness is spread to other members of the team, who then experience the same decrements. LO: 18.2: Discuss how sleep, illness, and injury affect physical health at work. Difficulty: Moderate Quest. Category: Concept 49) The perception that one's job is at risk or that one's employment is not stable is known as ________. A) job insecurity B) absenteeism C) workplace deviance D) presenteeism E) counterproductive work behavior Answer: A Explanation: Job insecurity refers to the perception that one's job is at risk or that one's employment is not stable. LO: 18.3: Recognize how maladaptive mental health conditions can manifest as a consequence of stressors at work. Difficulty: Easy 811 richard@qwconsultancy.com
Quest. Category: Concept 50) People who feel compelled to work due to internal pressures, thinking about work even when not working, and going above and beyond what is reasonably expected are experiencing ________. A) counterproductive workplace behavior B) burnout C) hindrance stressors D) challenge stressors E) workaholism Answer: E Explanation: People who feel compelled to work due to internal pressures, thinking about work even when not working, and going above and beyond what is reasonably expected are experiencing workaholism. LO: 18.3: Recognize how maladaptive mental health conditions can manifest as a consequence of stressors at work. Difficulty: Easy Quest. Category: Concept 51) As a gig worker, Jim loves the flexibility he has in is schedule that allows him to leave work to watch his son play baseball. Lately though, Jim has been experiencing worrisome thoughts that his job could be at risk. Jim is experiencing ________. A) job insecurity B) burnout C) workaholism D) counterproductive workplace behavior E) hindrance stressors Answer: A Explanation: Jim is experiencing job insecurity. The perception that one's job is at risk or that one's employment is not stable is known as job insecurity. Job insecurity is an especially salient stressor for gig workers, and it negatively affects their job attitudes. Although these workers have much greater autonomy than traditional full- or part-time employees, gig workers experience more strain and work—family conflict. As a nature of their work, gig workers are prone to experience fluctuations in their workloads–demands they have to meet to ensure they are hired again and protect their valuable reputation with other clients (pressures that lead to increased strain). LO: 18.3: Recognize how maladaptive mental health conditions can manifest as a consequence of stressors at work. AACSB: Analytical thinking Difficulty: Hard Employability Skills: Knowledge Application and Analysis Quest. Category: Application
812 richard@qwconsultancy.com
52) Which type of worker is especially prone to workaholism? A) Line workers B) Salaried salespeople C) Administrative workers D) R&D employees E) Leaders Answer: E Explanation: Leaders tend to be especially prone to workaholism. Some leaders feel compelled to "sacrifice" their personal lives and recovery time to "keep the ship running," which can lead to workaholic behaviors. Similarly, self-employed entrepreneurs cope with job insecurity by working more than they should. LO: 18.3: Recognize how maladaptive mental health conditions can manifest as a consequence of stressors at work. Difficulty: Moderate Quest. Category: Concept 53) When an individual has a feeling of being overextended and emotionally depleted, the individual is experiencing ________. A) emotional exhaustion B) depersonalization C) reduced personal accomplishment D) depression E) normative anxiety Answer: A Explanation: When an individual has a feeling of being overextended and emotionally depleted, the individual is experiencing emotional exhaustion. LO: 18.3: Recognize how maladaptive mental health conditions can manifest as a consequence of stressors at work. Difficulty: Easy Quest. Category: Concept 54) ________ means losing a sense of productivity or contribution on the job. A) Depression B) Depersonalization C) Normative anxiety D) Emotional exhaustion E) Reduced personal accomplishment Answer: E Explanation: Reduced personal accomplishment means losing a sense of productivity or contribution on the job. LO: 18.3: Recognize how maladaptive mental health conditions can manifest as a consequence of stressors at work. Difficulty: Easy Quest. Category: Concept
813 richard@qwconsultancy.com
55) Burnout has a ________ effect on employee outcomes, including an increase in absenteeism and turnover along with a reduction in job performance. A) very weak B) weak C) moderate D) strong E) very strong Answer: C Explanation: Burnout has a moderate effect on employee outcomes, including an increase in absenteeism and turnover along with a reduction in job performance. LO: 18.3: Recognize how maladaptive mental health conditions can manifest as a consequence of stressors at work. Difficulty: Moderate Quest. Category: Concept 56) Job insecurity is an especially salient stressor for gig workers. Answer: TRUE Explanation: Job insecurity is an especially salient stressor for gig workers, and it negatively affects their job attitudes. Although these workers have much greater autonomy than traditional full- or part-time employees, gig workers experience more strain and work—family conflict. As a nature of their work, gig workers are prone to experience fluctuations in their workloads— demands they have to meet to ensure they are hired again and protect their valuable reputation with other clients (pressures that lead to increased strain). LO: 18.3: Recognize how maladaptive mental health conditions can manifest as a consequence of stressors at work. Difficulty: Moderate Quest. Category: Concept 57) Workaholic behavior generally leads to greater productivity. Answer: FALSE Explanation: Workaholic behavior does not necessarily lead to more productivity, despite the extreme efforts involved. The strain of putting in such a high level of work effort eventually begins to wear on the person, leading to higher levels of work—life conflict, burnout, and higher blood pressure. LO: 18.3: Recognize how maladaptive mental health conditions can manifest as a consequence of stressors at work. Difficulty: Moderate Quest. Category: Concept
814 richard@qwconsultancy.com
58) Burnout refers to a work-related mental health syndrome characterized by emotional exhaustion, depersonalization, and reduced personal accomplishment. Answer: TRUE Explanation: Burnout refers to a work-related mental health syndrome characterized by emotional exhaustion, depersonalization, and reduced personal accomplishment. LO: 18.3: Recognize how maladaptive mental health conditions can manifest as a consequence of stressors at work. Difficulty: Easy Quest. Category: Concept 59) High extroversion in employees is linked with a lower likelihood of emotional exhaustion. Answer: TRUE Explanation: High extroversion in employees is linked with a lower likelihood of emotional exhaustion. LO: 18.3: Recognize how maladaptive mental health conditions can manifest as a consequence of stressors at work. Difficulty: Moderate Quest. Category: Concept 60) Women are more likely to experience emotional exhaustion than men, but men are more likely to experience depersonalization than women. Answer: TRUE Explanation: Women are more likely to experience emotional exhaustion than men, but men are more likely to experience depersonalization than women. LO: 18.3: Recognize how maladaptive mental health conditions can manifest as a consequence of stressors at work. Difficulty: Moderate Quest. Category: Concept 61) Employees are more accepting of changes when they are committed to the organization as a whole. Answer: TRUE Explanation: Cultures characterized by tight, masculine norms tend to be more prone to burnout than looser, egalitarian societies. LO: 18.3: Recognize how maladaptive mental health conditions can manifest as a consequence of stressors at work. AACSB: Diverse and multicultural work environments Difficulty: Moderate Quest. Category: Concept
815 richard@qwconsultancy.com
62) The effects of burnout do not compound. Answer: FALSE Explanation: Research shows that the effects of burnout compound. Although stressors lead to burnout as a form of strain, the experience of burnout also leads to the emergence of newer stressors in a vicious cycle. LO: 18.3: Recognize how maladaptive mental health conditions can manifest as a consequence of stressors at work. Difficulty: Moderate Quest. Category: Concept 63) Explain the different elements of burnout. Answer: Many employees at some point in their careers experience burnout, a work-related mental health syndrome characterized by emotional exhaustion, depersonalization, and reduced personal accomplishment. Emotional exhaustion refers to feeling overextended and emotionally depleted. Depersonalization refers to maladaptively coping with the job through callousness and distancing. Reduced personal accomplishment means losing a sense of productivity or contribution on the job. LO: 18.3: Recognize how maladaptive mental health conditions can manifest as a consequence of stressors at work. Difficulty: Moderate Quest. Category: Concept 64) Explain the similarities between workaholism, burnout, and depression. Answer: Of all mental health conditions, depression is the most common mental health condition (affecting about 9.5 percent of the U.S. population annually), leading to serious difficulties at work. Like workaholism and burnout, depressive states are heavily predicted by constant job insecurity (although depressive symptoms also tend to reinforce perceptions of job insecurity, with suffering employees experiencing a heightened awareness of their job insecurity). Moreover, negative interpersonal interactions (e.g., bullying, social isolation) can cause employees to become depressed over time; those bullied while depressed can experience a psychological sting so painful that some may even contemplate suicide. LO: 18.3: Recognize how maladaptive mental health conditions can manifest as a consequence of stressors at work. AACSB: Analytical thinking Difficulty: Moderate Quest. Category: Synthesis
816 richard@qwconsultancy.com
65) Factors within an individual's control that can be expended toward fulfilling desires, attaining goals, or meeting task demands are ________. A) resources B) rewards C) controls D) demands E) supports Answer: A Explanation: Factors within an individual's control that can be expended toward fulfilling desires, attaining goals, or meeting task demands are resources. LO: 18.4: Compare the four major stress and health theories. Difficulty: Moderate Quest. Category: Concept 66) Which of the following is one way leaders conserve their resources? A) By creating a new vision in line with existing resources B) By selectively building relationships with their followers C) By planning for change D) By forming coalitions for change E) Through better communication designed to protect valuable resources Answer: B Explanation: Leaders tend to conserve their resources by selectively building relationships with their followers (i.e., LMX differentiation). Moreover, they conserve resources when strategically deciding when to try something new (a resource-intensive task) or defaulting to the old ways of doing things. LO: 18.4: Compare the four major stress and health theories. Difficulty: Moderate Quest. Category: Concept 67) Employees are motivated to resolve effort-reward imbalances by ________. A) forming coalitions B) establishing a sense of urgency for change C) focusing on mental health D) putting in less effort E) taking stock of the situation Answer: D Explanation: Employees are motivated to resolve effort-reward imbalances by putting in less effort or working to maximize or increase the reward. LO: 18.4: Compare the four major stress and health theories. Difficulty: Easy Quest. Category: Concept
817 richard@qwconsultancy.com
68) The responsibilities, pressures, obligations, and even uncertainties that individuals face in the workplace are referred to as ________. A) supports B) controls C) demands D) mandates E) ultimatums Answer: C Explanation: Responsibilities, pressures, obligations, and even uncertainties that individuals face in the workplace. LO: 18.4: Compare the four major stress and health theories. Difficulty: Moderate Quest. Category: Concept 69) According to the job demands-resources (JDR) model ________ contribute(s) to performance through their distinctive effect on strain. A) demands B) burnout C) resources D) both burnout and demands E) both demands and resources Answer: E Explanation: The job demands-resources (JDR) model combines elements of COR theory and the JDCS to suggest that demands and resources both contribute to performance through their distinctive effects on strain (e.g., burnout) and engagement. LO: 18.4: Compare the four major stress and health theories. Difficulty: Moderate Quest. Category: Concept
818 richard@qwconsultancy.com
70) Which of the following is true of the job demands-resources (JDR) model? A) Demands are the main contributor to performance through their distinctive effects on strain. B) Resources are the main contributor to performance through their distinctive effects on strain. C) The influence of resources often depends on the organizational climate. D) Research does not support the JDR in the safety context. E) Challenging demands (e.g., challenge stressors) can lead employees to become less engaged. Answer: C Explanation: The job demands-resources (JDR) model combines elements of COR theory and the JDCS to suggest that demands and resources both contribute to performance through their distinctive effects on strain (e.g., burnout) and engagement. The JDR is one of the most popular stress and health theories to date, and research tends to support its utility. Both demands and resources lead to strain (e.g., burnout) as expected. The relationship between demands and engagement does not tend to be as strong as the other relationships in the model, as sometimes challenging demands (e.g., challenge stressors) can lead employees to become engaged. The influence of resources often depends on the organizational climate, supportive leadership, and psychological safety in the team or organization: Will others look down on you if you draw on resources? Researchers have also supported the JDR in the safety context, with strain leading to safety incidents and engagement in compliant safety behaviors, reducing these incidents' likelihood. LO: 18.4: Compare the four major stress and health theories. Difficulty: Moderate Quest. Category: Concept 71) One of Carl's department employees was severely injured on the job. Carl has called in psychological counselors to talk to the other employees and scheduled a series of safety trainings for the department to be completed by the end of the week. He has also asked the HR personnel to talk to the department about the disability insurance that the company offers and make sure that his employees know what is available. Carl is using ________ to help cope with the stress caused by his employee's injury. A) demands B) action research C) resources D) innovation E) appreciative inquiry Answer: C Explanation: Resources are things within an individual's control that he or she can use to resolve the demands. Research suggests adequate resources help reduce the stressful nature of demands when demands and resources match. Having resources to cope with stress is just as important in offsetting it as demands are in increasing it. LO: 18.4: Compare the four major stress and health theories. AACSB: Analytical thinking Difficulty: Hard Employability Skills: Knowledge Application and Analysis Quest. Category: Application
819 richard@qwconsultancy.com
72) Allostasis refers to the idea of working to change behavior and attitudes to find stability. Answer: TRUE Explanation: Allostasis refers to the idea of working to change behavior and attitudes to find stability. LO: 18.4: Compare the four major stress and health theories. Difficulty: Easy Quest. Category: Concept 73) What is basic argument in the conservation of resources theory and how do individuals react when resources are gained or lost? Answer: Conservation of resources (COR) theory suggests that employees strive to obtain, foster, retain, and protect the resources and things they value. Resources are factors within an individual's control that can be expended toward fulfilling desires, attaining goals, or meeting task demands. Employees must invest time and energy to gain resources and protect or replenish resources from loss. Concerning stress, employees are more aware of resource losses than resource gains. In other words, a missed opportunity is not as concerning as a loss of a cherished resource. However, when resources have been lost, employees become motivated to replace what was lost. Moreover, when people are overextended or exhausted, they become desperate, defensive, aggressive, and irrational in their pursuit of regaining or replenishing what was lost. LO: 18.4: Compare the four major stress and health theories. Difficulty: Moderate Quest. Category: Concept 74) What is the job demand-control-support (JDCS) model and what does it say about strain in the workplace? Answer: The job demand-control-support (JDCS) model suggests strain is a function of three factors: demands, control, and support. Demands are the responsibilities, pressures, obligations, and uncertainties that individuals face in the workplace. Control, similar to autonomy refers to the degree to which employees have discretion over how to do their jobs. Support is essentially perceptions of assistance provided by the organization, supervisor, and coworkers. The main tenant behind the JDCS model is that any negative effects of demands on strain can be offset or buffered by control and support. LO: 18.4: Compare the four major stress and health theories. Difficulty: Moderate Quest. Category: Concept
820 richard@qwconsultancy.com
75) An individual who says I always leave work at 5 p.m. to be home in time for dinner is using which form of boundary management tactic? A) Establishing time boundaries B) Marking workspaces C) Making compromises D) Setting expectations E) Negotiating resources Answer: A Explanation: An individual who says I always leave work at 5 p.m. to be home in time for dinner is establishing time boundaries. LO: 18.5: Differentiate between work-life conflict and work-life enrichment. Difficulty: Moderate Quest. Category: Concept 76) An individual is using which form of boundary management tactic when they say Please respect my time by not calling me on vacation unless it is an emergency? A) Making compromises B) Setting expectations C) Establishing time boundaries D) Prioritizing and goal setting E) Facilitating boundary transitions Answer: B Explanation: An individual stating Please respect my time by not calling me on vacation unless it is an emergency is setting expectations. LO: 18.5: Differentiate between work-life conflict and work-life enrichment. Difficulty: Moderate Quest. Category: Concept 77) Which of the following forms of boundary management tactics involves facilitating boundary transitions? A) I will always work in the second bedroom, and if the door is shut, do not come in. B) I will make my working late all week up to the rest of my family by going to the movies with them on Friday. C) I will ask my coworkers if they want to come to a family event with me this weekend, where we can discuss the new project. D) I bought a house a five-minute bike ride away from where I work. E) When the clients are in town this week, I will spend more time building relationships with them. Answer: D Explanation: I bought a house a five-minute bike ride away from where I work is an example of facilitating boundary transitions. LO: 18.5: Differentiate between work-life conflict and work-life enrichment. Difficulty: Moderate Quest. Category: Concept
821 richard@qwconsultancy.com
78) An individual wanting to negotiate resources might state ________. A) I always leave work at 5 p.m. to be home in time for dinner. B) I will make my working late all week up to the rest of my family by going to the movies with them on Friday. C) Please respect my time by not calling me on vacation unless it is an emergency. D) I will ask my coworkers if they want to come to a family event with me this weekend, where we can discuss the new project. E) I will ask my coworkers if they want to come to a family event with me this weekend, where we can discuss the new project. Answer: D Explanation: An individual wanting to negotiate resources might state I will ask my coworkers if they want to come to a family event with me this weekend, where we can discuss the new project. LO: 18.5: Differentiate between work-life conflict and work-life enrichment. Difficulty: Moderate Quest. Category: Concept 79) Which of the following is a psychological boundary management tactic? A) Making compromises B) Setting expectations C) Establishing time boundaries D) Making workspaces E) Negotiating resources Answer: A Explanation: Making compromises and prioritizing and goal setting are psychological tactics. LO: 18.5: Differentiate between work-life conflict and work-life enrichment. Difficulty: Moderate Quest. Category: Concept 80) A physical boundary management tactic is ________. A) negotiating resources B) establishing time boundaries C) setting expectations D) making compromises E) prioritizing and goal setting Answer: B Explanation: Establishing time boundaries is a physical boundary management tactic. LO: 18.5: Differentiate between work-life conflict and work-life enrichment. Difficulty: Moderate Quest. Category: Concept
822 richard@qwconsultancy.com
81) A behavioral boundary management tactic is ________. A) making workspaces B) making compromises C) prioritizing and goal setting D) negotiating resources E) setting expectations Answer: D Explanation: Negotiating resources is a behavioral boundary management tactic. LO: 18.5: Differentiate between work-life conflict and work-life enrichment. Difficulty: Moderate Quest. Category: Concept 82) Laura has been working 80-hour workweeks since the beginning of the year. She used to love coming to work, but now is starting to feel burned out. Laura decides it's time to set some boundaries. She tells her colleagues that she will always leave work by 7:00 pm to be home in time to work out before her club closed. Laura is using which boundary management tactic? A) Setting expectations B) Negotiating resources C) Making compromises D) Facilitating boundary transitions E) Establishing time boundaries Answer: E Explanation: Laura is establishing time boundaries. LO: 18.5: Differentiate between work-life conflict and work-life enrichment. AACSB: Analytical thinking Difficulty: Moderate Employability Skills: Knowledge Application and Analysis Quest. Category: Application 83) Brian Milton has been working from home for the past year. He likes the flexibility of his schedule but feels like his team thinks it's ok to call him 24/7. Brian has scheduled a much need vacation and wants to be sure that his team realize that his vacation means he's out of his virtual office. Brian asks his team to respect his time on vacation and not call unless it's an emergency. Brian is using which boundary management tactic? A) Establishing time boundaries B) Setting expectations C) Facilitating boundary transitions D) Making compromises E) Prioritizing and goal setting Answer: B Explanation: Brian is setting expectations. LO: 18.5: Differentiate between work-life conflict and work-life enrichment. AACSB: Analytical thinking Difficulty: Moderate Employability Skills: Knowledge Application and Analysis Quest. Category: Application 823 richard@qwconsultancy.com
84) Jane Tyson has been on a whirlwind of business travel. She feels like she's only been home about three days in the past three weeks. Jane tells herself that she'll make up for her busy work schedule by spending the entire weekend with them, keeping her phone off. Jane is using the ________ boundary management tactic. A) making compromises B) establishing time boundaries C) marking workspaces D) setting expectations E) prioritizing and goal setting Answer: A Explanation: Jane is making compromises. LO: 18.5: Differentiate between work-life conflict and work-life enrichment. AACSB: Analytical thinking Difficulty: Moderate Employability Skills: Knowledge Application and Analysis Quest. Category: Application 85) Which of the following is not a money-based strategy that has helped employees reduce work-life conflict? A) Insurance subsidies B) Discounts for childcare C) Paid time off D) Flexible benefits E) Domestic partner benefits Answer: C Explanation: Paid time off is a time-based strategy that helps employees reduce work-life conflict. LO: 18.5: Differentiate between work-life conflict and work-life enrichment. Difficulty: Moderate Quest. Category: Concept 86) Flextime is a(n) ________ to help employees reduce work-life conflict. A) information-based strategy B) money-based strategy C) direct service D) culture-change strategy E) time-based strategy Answer: E Explanation: Flextime is a time-based strategy to help employees reduce work-life conflict. LO: 18.5: Differentiate between work-life conflict and work-life enrichment. Difficulty: Moderate Quest. Category: Concept
824 richard@qwconsultancy.com
87) Antonio's company wants to support employee wellness. The company is asking employees to make their weekends email-free. Antonio's company is using a(n) ________. A) money-based strategy B) information-based strategy C) direct service D) culture-change strategy E) time-based strategy Answer: B Explanation: The company is asking employees to make their weekends email-free. Antonio's company is using a(n) information-based strategy. LO: 18.5: Differentiate between work-life conflict and work-life enrichment. AACSB: Analytical thinking Difficulty: Hard Employability Skills: Knowledge Application and Analysis Quest. Category: Application 88) Employees high in the personality trait of conscientiousness are more likely to perceive that they have greater boundary control and feel better equipped to manage cross-role interruptions. Answer: TRUE Explanation: Employees high in the personality trait of conscientiousness are more likely to perceive that they have greater boundary control and feel better equipped to manage cross-role interruptions. LO: 18.5: Differentiate between work-life conflict and work-life enrichment. Difficulty: Moderate Quest. Category: Concept 89) Work-life spillover occurs when psychological responses to one domain (e.g., positive or negative moods) are carried over into another domain and impact it in some way. Answer: TRUE Explanation: Work-life spillover occurs when psychological responses to one domain (e.g., positive or negative moods) are carried over into another domain and impact it in some way. LO: 18.5: Differentiate between work-life conflict and work-life enrichment. Difficulty: Easy Quest. Category: Concept 90) Work-life conflict is a recent phenomenon. Answer: FALSE Explanation: Negative spillover (e.g., work-life conflict) grabbed management's attention in the 1980s, largely due to the increased entry into the workforce of women with dependent children. In response, most major organizations took action to make their workplaces more family friendly. LO: 18.5: Differentiate between work-life conflict and work-life enrichment. Difficulty: Moderate Quest. Category: Concept
825 richard@qwconsultancy.com
91) Most research indicates that gender does not have a significant effect on work-life conflict. Answer: TRUE Explanation: Considering gender, a vast amount of research suggests that gender does not influence the amount or degree of work–life conflict in either direction—everyone experiences it. LO: 18.5: Differentiate between work-life conflict and work-life enrichment. AACSB: Diverse and multicultural work environments Difficulty: Moderate Quest. Category: Concept 92) Life-to-work conflict appears to be stronger in collectivist societies and those with larger economic gender gaps. Answer: TRUE Explanation: Life-to-work conflict appears to be stronger in collectivist societies and those with larger economic gender gaps. LO: 18.5: Differentiate between work-life conflict and work-life enrichment. AACSB: Diverse and multicultural work environments Difficulty: Moderate Quest. Category: Concept 93) Work-family spillover is always a negative concept. Answer: FALSE Explanation: Work-family spillover has a positive side. Just as work and family life can cause conflict, so too may it cause enrichment. An excellent presentation, compliment from a customer, or raise from your supervisor can lead these positive experiences to spillover to other domains of your life. LO: 18.5: Differentiate between work-life conflict and work-life enrichment. Difficulty: Moderate Quest. Category: Concept 94) Typically, work factors tend to more strongly influence work spilling over to life, and life factors tend to more strongly influence life spilling over to work. Answer: TRUE Explanation: What causes work-life conflict? Typically, work factors tend to more strongly influence work spilling over to life, and life factors tend to more strongly influence life spilling over to work. LO: 18.5: Differentiate between work-life conflict and work-life enrichment. Difficulty: Moderate Quest. Category: Concept
826 richard@qwconsultancy.com
95) Work-life boundaries refer to the lines that demarcate our lives. Answer: TRUE Explanation: Work-life boundaries refer to the lines that demarcate our lives. We move and transition between these domains both physically and/or psychologically, and people, things, and communications from some domains can move across boundaries and serve as interruptions in our daily lives. LO: 18.5: Differentiate between work-life conflict and work-life enrichment. Difficulty: Easy Quest. Category: Concept 96) What causes work-life conflict? Answer: Typically, work factors tend to more strongly influence work spilling over to life, and life factors tend to more strongly influence life spilling over to work. Negative job attitudes, long hours, little support, inflexibility, and job stress lead to life conflict. On the other hand, family stress and interpersonal conflict, time spent caring for family members, and having young children are all associated with work conflict. Among these antecedents, stress experienced in either domain appears to be the biggest factor. LO: 18.5: Differentiate between work-life conflict and work-life enrichment. Difficulty: Moderate Quest. Category: Concept 97) Explain why companies should spend less time helping employees with time management issues and more time helping them segment their lives. Answer: Time pressures are not the primary problem underlying work-life conflicts. The psychological incursion of work into the family domain—and vice versa—leaves people worrying about personal problems at work and thinking about work problems at home, creating conflict. This suggests organizations should spend less effort helping employees with time management issues and more effort helping them segment their lives, navigate boundary transitions, and facilitate enrichment between both domains. LO: 18.5: Differentiate between work-life conflict and work-life enrichment. Difficulty: Hard Quest. Category: Concept 98) How can companies help employees segment their lives, navigate boundary transitions, and facilitate enrichment between both domains? Answer: Flexible and supportive policies, such as keeping workloads reasonable, reducing work-related travel, and offering onsite high-quality childcare, are examples of practices that can help in this endeavor. Leaders play a large role here—encouraging followers to use supportive policies (and playing a part in establishing them), establishing high-quality exchange relationships with followers, and ultimately enabling followers in their pursuit of balance and boundary management. LO: 18.5: Differentiate between work-life conflict and work-life enrichment. Difficulty: Moderate Quest. Category: Concept
827 richard@qwconsultancy.com
99) ________ refers to reception, attention, and awareness of the present moment, events, and experiences. A) Resilience B) Mindfulness C) Eustress D) Allostasis E) Laissez-faire Answer: B Explanation: Mindfulness refers to the reception, attention, and awareness of the present moment, events, and experiences. LO: 18.6: Describe individual and organizational approaches to managing stress at work. Difficulty: Easy Quest. Category: Concept 100) ________ refers to resistance to the adverse effects of stress and strain. A) Eustress B) Allostasis C) Resilience D) Time management E) Self-efficacy Answer: C Explanation: Resilience refers to resistance to the adverse effects of stress and strain. LO: 18.6: Describe individual and organizational approaches to managing stress at work. Difficulty: Easy Quest. Category: Concept 101) Meg is worn out and highly stressed at work. She feels that her "to-do" list is getting longer every day even though she usually works through lunch and stays late. To improve her mood, Meg should ________. A) take a walk in the park nearby during her lunch hour B) try to work faster so that she can leave earlier in the evening C) put in the extra hours so that she can cross more off her "to-do" list D) take a nap when she gets home rather than going for a run E) stop going to sunrise aerobics and get to work half an hour earlier Answer: A Explanation: Meg should take a walk in the nearby park during her lunch hour. Physicians have recommended noncompetitive physical exercise, such as aerobics, walking, jogging, swimming, and riding a bicycle, to deal with excessive stress levels and promote physical and mental health. These activities decrease the detrimental physiological responses to stress and allow us to recover from stress more quickly. Research shows that even low-intensity activity, such as walking in a park during lunch, can improve moods along with other emotional benefits (even for people who doubt they will help). LO: 18.6: Describe individual and organizational approaches to managing stress at work. AACSB: Analytical thinking Difficulty: Moderate Employability Skills: Knowledge Application and Analysis 828 richard@qwconsultancy.com
Quest. Category: Application 102) Louise is overwhelmed with the new tasks that she has been assigned after the department merger. Each day, she arrives at work and makes a list of the new tasks, writes a number next to each one according to the urgency of completion, and schedules a time to complete each one. Although she's still stressed, Louise is able to get most of the work done and feels better at the end of the day. Louise is using ________ skills to reduce her stress. A) job enrichment B) goal setting C) relaxation D) time-management E) job enlargement Answer: D Explanation: Louise is using basic time-management skills to better cope with tensions created by job demands. A few of the best-known time-management skills are (1) staying organized through calendars, project management software, and to-do lists (schedule time for rest and recovery!); (2) prioritizing tasks, duties, and responsibilities (while working on [or delegating] them in order of priority); and (3) setting temporally realistic long- and short-term goals. LO: 18.6: Describe individual and organizational approaches to managing stress at work. AACSB: Analytical thinking Difficulty: Moderate Quest. Category: Application 103) Which of the following statements is not true regarding organizational approaches to managing stress? A) In redesigning jobs, managers should pay attention to the specific tasks that may serve as (or preclude) stressors. B) Redesigning jobs to give employees more responsibility, more meaningful work, more autonomy, and increased feedback can reduce stress. C) Organizations should not support programs focusing on the personal decisions of employees regarding smoking, losing weight, or exercising. D) Organizations should be careful when deciding to structure work and organizations so that employees are on call twenty-four hours a day. E) Organizations should encourage employees to rest and recover. Answer: C Explanation: Organizationally supported employee assistance programs (EAPs), work-life programs, or wellness programs can help manage or reduce employee stress. These are organizationally supported programs that focus on the employees' total physical and mental health. They typically provide workshops to help people quit smoking, control alcohol use, lose weight, make healthier food and drink choices, and develop a regular exercise program. Some programs help employees improve their psychological health as well. LO: 18.6: Describe individual and organizational approaches to managing stress at work. Difficulty: Moderate Quest. Category: Concept
829 richard@qwconsultancy.com
104) Organizationally supported programs that focus on the employee's total physical and mental condition, such as helping them quit smoking or control alcohol use, are called ________ programs. A) job redesign B) relaxation C) employee involvement D) organizational development E) wellness Answer: E Explanation: Organizationally supported wellness programs provide workshops to help people quit smoking, control alcohol use, lose weight, eat better, and develop a regular exercise program; they focus on the employee's total physical and mental condition. LO: 18.6: Describe individual and organizational approaches to managing stress at work. Difficulty: Easy Quest. Category: Concept 105) Elaine is very good at what she does, but she is constantly stressed by internal deadlines assigned to co-workers, obligatory meetings, and time lost in unrelated idle cubicle chatter. Today, after a meeting with her supervisor, she has been given more responsibility for her work, is able to set her own deadlines, decline attendance to various department meetings, and telecommute two days a week. Elaine's boss is using which of the following organizational strategies to reduce her stress? A) Job redesign B) Organizational communication C) Employee involvement D) Organizational development E) Process reengineering Answer: A Explanation: Elaine's boss is redesigning her job. Redesigning jobs to give employees more responsibility, more meaningful work, more autonomy, and increased feedback can reduce stress because these factors give employees greater control over work activities and lessen dependence on others. LO: 18.6: Describe individual and organizational approaches to managing stress at work. AACSB: Analytical thinking Difficulty: Hard Employability Skills: Knowledge Application and Analysis Quest. Category: Application
830 richard@qwconsultancy.com
106) Working faster and working harder are two of the best-known time management skills. Answer: FALSE Explanation: Working faster and working harder are two of the worst time-management skills. Some of the best-known time management skills are: (1) staying organized through calendars, project management software, and to-do lists (schedule time for rest and recovery!); (2) prioritizing tasks, duties, and responsibilities (while working on [or delegating] them in order of priority); and (3) setting temporally realistic long- and short-term goals. LO: 18.6: Describe individual and organizational approaches to managing stress at work. Difficulty: Moderate Quest. Category: Concept 107) Psychological techniques such as self-distancing from future stressors can help you adaptively cope with time pressures. Answer: TRUE Explanation: Psychological techniques such as self-distancing from future stressors can help you adaptively cope with time pressures. LO: 18.6: Describe individual and organizational approaches to managing stress at work. Difficulty: Moderate Quest. Category: Concept 108) The physical, emotional, cognitive, and behavioral benefits of physical exercise may translate into better performance at work. Answer: TRUE Explanation: The physical, emotional, cognitive, and behavioral benefits may translate into better performance at work. Exercise can also help buffer the negative spillover effects of worklife conflict and improve work-life balance perceptions, even in the presence of poor sleep. LO: 18.6: Describe individual and organizational approaches to managing stress at work. Difficulty: Moderate Quest. Category: Concept 109) Physicians have recommended noncompetitive physical exercise, such as aerobics, walking, jogging, swimming, and riding a bicycle, as a way to deal with excessive stress levels. Answer: TRUE Explanation: Physicians have recommended noncompetitive physical exercise, such as aerobics, walking, jogging, swimming, and riding a bicycle, as a way to deal with excessive stress levels. LO: 18.6: Describe individual and organizational approaches to managing stress at work. Difficulty: Moderate Quest. Category: Concept
831 richard@qwconsultancy.com
110) Describe three of the best-known time-management skills. Answer: Three of the best-known time management skills are: (1) staying organized through calendars, project management software, and to-do lists (schedule time for rest and recovery!); (2) prioritizing tasks, duties, and responsibilities (while working on [or delegating] them in order of priority); and (3) setting temporally realistic long- and short-term goals. LO: 18.6: Describe individual and organizational approaches to managing stress at work. Difficulty: Moderate Quest. Category: Concept 111) What are three practical steps that managers can take to talk with employees about mental health without overstepping boundaries? Answer: Managers should take the following steps: (1) Avoid trying to fix people. If employees feel that you are attempting to "fix" them, your attempts to offer to help may backfire. Instead, provide employees with suggested resources or ask, "What would be most helpful for you right now?"; (2) Discuss health holistically. Making an effort to ask about someone's mental health can go a long way in normalizing the topic. Sharing your own struggles can also help create a more psychologically safe environment in which your co-workers feel more comfortable being open about their own mental health. (3) Listen more than they talk. Focus on your colleague's experience and remember to separate it from your own. Resist the urge to talk prescriptively. LO: 18.6: Describe individual and organizational approaches to managing stress at work. AACSB: Analytical thinking Difficulty: Moderate Quest. Category: Concept 112) What is the connection between mental wellness and physical fitness? Answer: Physicians recommended noncompetitive physical exercise, such as aerobics, walking, jogging, swimming, and riding a bicycle, to deal with excessive stress levels and promote physical and mental health. These activities decrease the detrimental physiological responses to stress and allow us to recover from stress more quickly. The physical, emotional, cognitive, and behavioral benefits may translate into better performance at work. Exercise can also help buffer the negative spillover effects of work-life conflict and improve work-life balance perceptions, even in the presence of poor sleep. Beyond its strain reduction benefits, physical exercise can reduce the impact of others' unethical or unfair behavior on you— and also make you less likely to be deviant yourself. LO: 18.6: Describe individual and organizational approaches to managing stress at work. Difficulty: Moderate Quest. Category: Concept
832 richard@qwconsultancy.com
113) What are the benefits of wellness programs for companies? Answer: Most firms that have introduced wellness programs have found significant benefits. Johnson & Johnson reported that its wellness program has saved the organization $250 million on health care costs in ten years, and research indicated that effective wellness programs significantly decreased turnover rates for most organizations. Other research sponsored by the U.S. Department of Labor and Department of Health and Human Services indicated that organizational wellness programs create healthier employees with fewer health risk factors. Moreover, the accumulated research suggests that organizations that implement wellness programs (1) observe a $5.93 to $1.00 savings-to-cost ratio, (2) reduce health costs by 26 percent, (3) reduce workers' compensation claims by 30 percent, and (4) observe a 28 percent reduction in sick days used. However, as mentioned earlier, workers' benefits from these programs appear contingent on how much they put into the program, the extent to which they participate, and their attitudes upon entering the program. LO: 18.6: Describe individual and organizational approaches to managing stress at work. Difficulty: Moderate Quest. Category: Concept 114) Do wellness programs work and what role do supervisors play in their success? Answer: Research shows that interventions to help employees reframe stressful situations and use active coping strategies appreciably reduce stress levels. Wellness programs that help employees focus on developing the "good" kind of stress and becoming challenged through their work have also been introduced. Research suggests that these programs impact factors beyond stress relevant to OB, such as job satisfaction, absenteeism, and presenteeism. However, research also suggests that wellness programs are only effective if people use them. Employees have to see the value in the wellness program, know their supervisor supports their participation, believe that their organization supports their continued wellness and is a safe place to seek help without stigma. If they do, they can leave the program with better performance, higher job satisfaction, and lower turnover. LO: 18.6: Describe individual and organizational approaches to managing stress at work. Difficulty: Moderate Quest. Category: Concept
833 richard@qwconsultancy.com